Bruhn + Supplement PDF
Bruhn + Supplement PDF
Bruhn + Supplement PDF
OF
FLIGHT VEHICLE STRUCTURES
By
E. F. Bruhn, B. S., M.S., C.E.
Printed in U.S.A.
PREFACE
The 1965 edition of the book "Analysis and Design of Flight Vehicle Structures", contained
over 950 pages. As time passes it is normal to expect expanded or new material to be added to later
editions of a book. Thus it appeared desirable to add some 380 pages of additional material, which
meant that a new edition would contain over 1300 pages or too large for a single volume. A decision
was made to issue the new edition in two volumes. This two volume edition was practically ready
for binding, when a further final decision was made to remain with one volume edition and place the
380 pages of new material referred to above in 3 separate, rather small sized books, Information on
these 3 separate books is given on the 3 pages following the Table of Contents. The reader will note
that these 3 books deal with special subjects in the structural design field and are available.
This new 1973 one volume edition presents 2 major changes from the 1965 edition. Chapter
A23 has been completely revised and expanded by a new co-author, namely, Dr. Joseph A. Wolf,
formerly on the faculty of UCLA and now with the General Motors Research Laboratories. Dr. Wolf
had the cooperation of Dr. A. F. Schmitt, the author of the A23 Chapter in the 1965 edition. The
other major change is the replacement of Chapter C13 on the subject of fatigue in the 1965 edition
by a new chapter on fatigue. The Author of this new Ch~pter C 13 is C. R. Smith, a widely known
authority in the broad field of fatigue of materials and structures.
This author expresses his thanks to the several co-authors for checking over their material. Only
minor corrections have resulted from this check.
A considerable amount of material in various chapters by the author made use of reports, manu-
als, drawings and photographs supplied to the author by many aerospace companies, in particular,
Boeing of Seattle; Douglas Co.; General Dynamics, Fort Worth and San Diego Divisions; Martin
in Denver; North American Aviation, Columbus and Tulsa Divisions; and the Vought Division of
LTV Corp. The author is deeply grateful for this assistance and cooperation.
As a final note, if any reference in the Chapters of this book refers to Volume 2 it should
be discarded by the reader.
June 1973
1
1
1
1
1
1
1
1
1
1
1
1
1
1
1
1
1
1
1
1
1
1
1
1
1
1
1
1
1
1
1
1
1
1
1
1
1
1
1
1
1
1
1
1
1
1
1
1
1
1
1
1
1
1
I
TABLE OF CONTENTS
Chapter No.
A2 Equilibrium of Force Systems. Truss Structures. Externally Braced Wings. Landing Gear.
A3 Properties of Sections - Centroids, Moments of Inertia, etc.
A4 General Loads on Aircraft.
A5 Beams - Shear and Moments. Beam - Column Moments.
A6 Torsion - Stresses and Deflections.
A7 Deflections of Structures. Castigliano's Theorem. Virtual Work. Matrix Methods.
A8 Statically Indeterminate Structures. Theorem of Least Work. Virtual Work. Matrix Methods.
A9 Bending Moments in Frames and Rings by Elastic Center Method.
Al0 Column Analogy Method.
A11 Continuous Structures - Moment Distribution Method.
A12 Slope Deflection Method.
Chapter No.
CONTE.VTS
At.l Introduction
A~,2 G~ne[al Organization of an Aircraft Company Engineering Division
Al.) T~e ~o[k of the Structur~s Group
A2.1 Introduction
A2.2 Equations of Static Equilibrium
A1.3 Structural Pitting Units for establishing the Force Characteristics
of Direction and Point of Application
A2.4 Symbols for Reacting Fitting Units as used in Problem Solution
A1.5 Statically Determinate and Statically Indeterminate Structures
AZ.6 Examples of Statically Determinate and StaLleally Indeterminate
Struc:u.~s
A2.7 Sxample P.oblem Solutions of Statically Deter~in3te Coplanar
St[uc~ures and Coplanar Loadi~gs
A2.3 Stres~0S in Coplanar Truss Structures under Coplanar Loading
(Stat~cally Determinate/Indetec~~na~e ?)
A19 ~eth()d of Joints
A). i.0 Method of Moments
A2.11 ~et~od of Shears (Sections)
.-\.1. 12 Aircaft Wing Structure. Truss Type with Fa~ric or Plastic Cover
:-1.2,13 Landing Ge1r Structure
A~ . 14 Example Problems of Calc~lacing Reactions and Loads on Members
Of Landi~~ Gear Units
A2.1S Probl>:!ms
.-\.3.1 Intr,)duction
.\] . 1 Centroids, Centre of G~avity
A.' .3 ~oment of Inertia
A3 4 ~oment of Inertia of an Area
A3.S Polar Moment of Inert~a
A3.6 Radius of Gyration
A3.7 Parallel Axis Theorem
A3.7a Mass :-laments of Inertia (Tables of Sections)
AJ .q Product of Inertia
AJ.9 Product of Inertia for Axes of S/~~etr,
A3.1O Parallel Axis Theorem
A3.11 Moments of Inertia ~it~ ~espec: of Inclined fu~es
A3.12 Location of Axes for ~hich Produc~ of Inenia is Ze,o
A3.13 ?rLnci?al AY.es
AJ.14 Section Properties 0: Ty?ical Aircraft Str~ct~ral Sections
A3.;S ?:"oble,:s
A4. [ :ntr0c·-.:ction
A4. J ~imit O~ Applied Loads. ~es~g~ ~oads
A4,3
A4.4 ~~ighc and Inertia Forces
A4.S Air Forces On Ving
A4.6 Forces on Airplane in F~igh:
A4.7 Load rae tors
A4.8 Design Flight Requirements for Airplane
A4.9 Gust Load Factocs
Ai..10 Il1us~r;Hion of Hain ?light C0nditions. VeLocity-Load Factor
Diag:-am
A4.11 Special FliKht Deslgn Conditions
A4.12 Example Problems Involving Accelerated Motion of Rigid Airplane
A4.13 Effect of Airplane not being a Rigid Body
A4.14 General Conclusions on Influence of Dynamic Loading on Structural
Design of Airplane
A4.15 Problems
B&AHS - SHKA.R AND KOKENTS
A5.1 Introduction
AS.2 StaticallY Determinate and Indeter~inate B€amS
AS. ) Shear and Bending Koment
A5.4 Shear and ~ornent Diagra~s
A5.5 Section of Maximum Bending Homent
A5.6 Relation betveen Shear and Bending ~oment
AS.7 Koment Diagr~s as Hade-up of Parts
A5.8 Forces at a Section in terms of Forces at a previous Station
A5.9 Equations for Curved Beams
AS.10 Torsional Moments
A5.11 Shears and ~oments on ~ing
AS.12 Example Problem of Calculating wing Shears and Moments for One
Unit Load Condition
AS .13 Introduction to Shears and ~oments on Airplane Sody
AS.14 Design Condi,ions and Design ~eights
A5.15 Body Veight and Balance Distribution
A5.16 Load Analysis. Unit Analysis
A5.17 Example ?roble~ Illustrating the Calculation of Shears and
Moments on Fuselage due to Unit Load Condi~ions
A5.18 Unit Analysis for ?uselage Shears and Moments
A5.19 Example of Fuselage Shears and Homents for Landing Conditions
AS.20 Inertia Loads due to Angular A~celeration
AS.21 Solution for Inertia Loads due to Unit 100,000 lbs.in
Pitchi:tg ~Olllent
A5.21 ?rob 1e'1Is
BENDING MOMENTS - BEAM - COLUMN ACTION
,1,.5.23 Introduction
,1,.5.24 General Action of a He~be~ Subjected to Combined Axial and
TraIE':er:;~
Loads
AS.25 Equations for a Compressive Axially Loaded Strut ~ith UniEor~ly
Distributed Side Load
A5.26 Formulae for other Single Span Loadings
AS.27 Moments foe Combinations of various Load Syste~s
AS.28 Example Problems
A5.29 Stresses Above Proportional Limit Stress of Material
A5.JO
A5.31 Beam-Columns in Continuous Structures
A7 D8PL&CrIONS OF STRUC11JRES
A7.1 Introduction
A 7.2 ~ork and Strain Energy
A7.3 Strain Energy Expressions for various Loadings
A7.4 Theorems of Virtual ~ork and ~inirnum Potential Energy
A7.5 Theocem of Complimentacy Ene~gy and Castigliano's Theocern
A7.6 Calculations of Stcuctucal Deflections by use of
Casttgliano's TheoLem
A7.7 Calculations of Stcuctural De:lections USing Dummy-Unit
Loads (~ethod of Virtual ~ork)
A7.8 Deflections due to Ther~al Strains
A7.9 Matrix ~ethods in Deflection Calculations
A7.10 Member ?rexibility Coe~:icien:s; Compilation of a Library
A7. 11 Application of Matrix ~~:hods co various StrucCuces
A7.12 Deflections and Angular Chang~ of the Elastic CUrve of
Simp:e Beams by Kohr's Het~od of Elastic Veights
A7.13 Example Problems
A7.14 Deflec:ion of aea~s by ~oment-Area Hethod
A7.15 aeam Fixed End Moments by Ke:hod of Area Moments
A7.16 Truss Deflection by Het~od of Elastic Veights
A7.17 Solution by Example P~oblems
A7.18 Problems
A8 5TAT I CALLY INDE:TRIUUNATR STR UC1.1JRES
A8.00 Introduction
A8.0 The Principle of Super?osition
A8.1 The Statically Indeter~i~ate Problem
A8.2.1 Determination of Reduncant Reactions by Least Vork
A8.2.2 Redundant Stresses by Least _ock
A8.3 Redundant Problems by the Ket~od of Dummy-Unit Loads
A8.4 Example Problems - Trusses ~ith Single Redundancy
A8.5 Trusses ~ith DQuble Redundancy
A8.6 Trusses ~ith Multiple Redundancy
AB.7 Redundant Strllccures with ~emberssubjected to Loadings
in addi~ion to Axial Forces
A8.8 Initial Stresses
A8.9 Thermal Stresses
AS .10 Reduncant Problem Stress Calculations by Matrix Methods
A8.11 Redundant Problem Deflection Calculations by Matrix Methods
'\8.12 Precision and Accllracy in Redu~dant Stress Calculations
AB.ll Thermal St:ess Calcular;ons by Ma:rix ~e~hods
A8.14 Thermal De~:ections by Ma:rix ~erholjs
A9.1 Introduction
A9.2 Der£vation of Equations. Unsymmetrical Frames
A9.J Equations for Structure with Symmetry about One Axis
thrOIJgh Elastic Centre
A9.4 Example Problem Solutions. Structures with at least
One Axis of Sy~metry
A9.S Unsymmetlical St~uctures. Example Ptoble~ Solutions
A9.6 Analysis of Frame ~ith Pinned Supports
A9.7 Analysis of Prame virh One Pinned and One Fixed Support
A9.3 Solution of Unsymmetrical Frames using ?rincipal Axes
A9.9 Problems
AlO.1 General
A10.2 Geno::ral gxplanation of Column Analogy Method
A10.3 Frames One Axis of SymmetrJ
~ith
A10.4 Unsy~metrical Frames or Rings
A10.S Example Problem - Unsymmetrical Section
A10.6 Problems
A11. 1 Introduction
A11.1 Definitions and Derivatives of Terms
All. 3 Calculation of Pixed End Moments
All. 4 Stiffness Pactor - Carryover Factor - Derivation of
All. 5 General Expressions for Stiffness and Carry-over Factor in
terms of Fixation Factor at far end of 3~a~
A11.5a 2xample Problems
All .6 General S~mrnJry of Proced~res
A11 . 7 ContillUQUS Beams with Yielding or Deflec~ed Supports
All. 8 ?ixed End Koments due to Support Deflec~iJns
All.'1 2nd Moments for Conti~uous Frame~0rks ~hose Me~bers are
not in a st.raight line. Joint Rotation Only
A11.10 Continuous Structures \{ith Members of Variable Homent
of Inert ia
.4.11.11 rrames vith UnLLo'~ Joint Deflections due to Sidesyay
:\11.12 Effect of Axial Load on Moment Distribut:on
A~ L 13 Fixed End Kcme~ts, Stiffness and Car:y-o,er Factors for
Beam Columns of Constant Section
Al1.1~ Illustrative Proble~s
A11.15 Secondary 3ending Home~ts in Trusses ~it~ig:d Joints
Al1. 16 Structures ¥ith Cur~ed Kenbers
.\11.17 Structures ~ith Curved He~bers
Al1.18 Stiffness and Carry-over Factors for Cuc~ed ~embers
:\11.19 2xample Pcoble~s. Continuous Structures involving
Curved Members
All. 10 Problems
Al1 SPECIAL KETBQDS - SLOPE DEFLECTION KETBOD
A12. 1 General
:\12.2 Derivation of Slope Deflection Equation
A12. 3 Hinged End. Slope Deflect ion Equa t ior.
A12.4 Exampl~ Problems
Al1.S Loaded Continuous Beam ~ich Yielding Supports
A',2.6 Statically Indeterminate F[a~es. JOLnt Rotation only
A12 . 7 Frames ~ith Joint Displace~e~ts
AL2.S 'example Problems of ?rames ~i th Unkno'.l" Jo~nt DisplaCement
A12.9 Comments on Slope Deflection Method
A~2.10 Problems
Al 3. a Introduction
A! 3 . 1 Location of Neutral Axis
AD.2 Equations for Bending Stress, Homogeneous 3eams, Stresses
below Proportional Limit Stress
AD. :3 Method 1. Stresses for Moments about the Principal Axes
AU.4 Method 2. Stresses ~y use of Neutral Axis for given Plane
of Loading
,
AI]. S Hethod 3. Stresses from Homents, Section Properties and
Distances referred to any pair of Rectangular Axes
through the Centroid of the Section
AI).6 Advantages and Disad~antages of the Three Methods
AI].7 Deflections
AI] .B Illustrative Problems
Al J. 9 Bending Stresses in Beams ~ith ~on-Homogeneous Sections
Stresses .ithin the Elastic Range
All.IO Bending Stresses of Homogeneous Beams Stressed above the
Elastic Li~it Stress Range
Al].l1 Curved Bea~s. Stresses ~ithin the Elastic Range
All.12 Problems
A14 BENDING SHRAR STRESSES - SOLID AND OPEN SECTIONS~ SHEAR CKNTR.&
A14.l Introduction
A14.2 Shear Cen t re
A14.] Derivation of Formula for Flexural Shear Stress
Al4.4 Example Prable~s. Symmetrical Sec~ions. External Shear Loads
act through Shear Centre
A14. S Maximum Shear Stresses for Sim?le Cross-sections
A14.6 Derivation of Flexural Shear F:o,,", Equation. Symmetrical
Beam S02-.:tion
A14.7 Shear Stresses and Shear Centre Eor Beam Sections ~ith One
Ax i S 0 E S:,nllne try
A14,3 Shear Str02SS02S for Unsy~me:rical Bea~ Sections
A14. '1 geams ... ith Constant Shear Flo,,", ~eJs
A14.10 Example Pr,)ble~s for Seams ... ith Constant Shear Flows
bet.een Flange ~ernbers
A14.1l Shear C~ntre Location by uSLng Neutral Axis Method
A14 :2 Proble:1s
Al5 SHEAR FLQtJ IN CLOSW THIN-'ii'ALLEJ SECfIONS
AlS.i IntrodiJc.tion
A1S.2 Singl~Cell Bea~. Symmetrical about One Axis. All Katerial
effec:ive in Resisting Sendi~g Stresses
A15.3 Single Cell ~ T~o flange Beam. Constant Shear Flov Vebs
Al5. ~ Shear Cen:re of a Single Cell - T¥o Fla~ge Beam
Al5,5 Single Cell - Three Flanged Beam. Constant Shear Flo~ Vebs
AlS.6 Shear Centre of Single Cell - Three Flange Beam. Constant
Shear Flo. Vebs
AlS,7 Single Cell - Multiple Flange - One Axis of Symmetry
A1S.8 Single Cell - Unsymmetrical - Multiple Flange
A1S.9 T~o Cell - Kultiple Flange 3eam. Symmetrical about One Axis
A1S.10 Three Cell - Multiple Flange 3eam, Symmetri~al about
One Axi s
AlS.ll Shear Flov in Beam vith Hultiple Cells. Method of Successive
Appr:oxi.na~ions
A1S.12 Example ?c~blem Solution
A15.ll Example Problem Solution
AlS.14 Use of Successive Approximation Method for Multiple
Cell BeaTIS ~hen slJbjected to Combined Bending and
Torsional Loads
A15.1S Shear Flo. in Cellular B~a~s ~ith Vaciable ~oment of Inertia
A15.l6 Determtnation of (~e flexura: Shear Flo~ Distribution by
constdec:ng the changes in ?lang~ ~oads
AIS, :7 Example ?:-JDlell
A 15 .. " Shear flay in Tapered Sheet ?anel
A15.l9 Example ?roblem of Shear FloY in Taper:ed ~ultiple Plange
Single Cell 3eam
A15.20 Problems
A16 KRMBRANP, STRESSES IN PRESSURE YESS~LS
A16.l Introduction
A16.2 Hembrane Equations of Equilibrium. Shells of Revolution under
Rotationally Symmetric Pressure Loadings
A16.3 Applications of Simple Pressure Vessels
A16.4 Displacements, Boundary Conditions and Local Bending in
Thin-Valled Shells
A16. 4 Special Problems in Pressurized Cabin Stress Analysis
A16. 5 Shells of Revolution under Unsymmetrical Loadings
A17.1 Introduction
A17.2 plate Bending Equations
A17.3 An Illustrative Plate 3ending Analysis
A17.4 Compilatioo3 of Results for Plate Bending Problems
A17. 5 ~flection Limitations in Plate Analyses
A17.6 Hembrane Action in very Thin Plates
A17.7 Large Deflections in Plates
A17 . 8 Considerations in the Applications of Large Deflection Plate
and Membrane Analyses
A18 THEORY OF THE INSTABIUTf OF COUJXNS AND THIN SHEETS
A18.1 Introduction
A18.2 Combined Bending and Compression of Columns
AlB.] Elasti~ Stability of a Column
A18.4 Failure of Columns by Compression
Ala.S Buckling Loads of Colu~ns ~ith various End Conditions
A18.6 Inelastic Buckling. Int~oduction
A18.7 Per teet Column. Reduced Modulus Theory
A18.8 Imperfect Colu~n. Tangent-Hodulus Theory
THEORY OF Tfffi E.LASTIC INSTABILITY OF THIN SI!EETS
"8.9 IntLoc.uct-'.on
A18.10 PULe Bending of Thin Pla~es
A18.11 The Differential Equation at the Deflection SurEace
A18.12 Strain Ener 6 J in Pure Be~di~g of Plates
A18.13 Bending of R~ctangular Plates
A13.14 Combined Bending and Tension or Compression of Thin plates
A18.15 Strain Energy of Plates due to Edge Compression and Bending
A18.16 Buckling of Rectangular Plates vith va~ious Edge Loads and
Support Conditions
Al3.17 Buckling of Simply Supported Recta~gular Plates Uniformly
Compressed in One Direction
A1B.18 Buckling of Simply Supported Rectangular Plate compressed
in r~o Perpendicular Directions
A18.19 Buckling of Simply Supported Rectangular Plates under
combined Bending and Compression
A18.20 Inelastic Buckling of Thin Sheets
STRF"..NGTH OF COLUMNS VlrS STABLE CROSS-SECTIONS
A18.22 Hethods of Column Failure. Column Equations
AIB.23 Column End Restraint. Fixity Coefficients. Column Effective
Length
A18.24 Design Column Curves for various Haterials
A:a.25 Selection of Materials for 2levated Temperature Conditil)ns
A:8.26 Exampl~ Problem
A18.27 Comprehensive Treatmer.t of Column Strength
A18.28 SDme Mechanical and Physical Properties of a few Typical
Aircraft Haterials
A19 INTRODUCTION TO VLNG STRESS &~YSIS
.-\21.1 Introduction
A2 ~ • 2 Types of ~ing Const.uction
.J..:': 1 • 3 Dist.ibution of Conce~trated Loads to ~hin Sheet Panels
,... 2 ~ . 4 Rib for Single Cell - Tva Flange Beam
A21.5 Stresses in Rib Eor Three Stringer Single Cell Beam
,\21.6 Stress Analysis of Rib for Single Cell Multiple
Stringer ~ing
,,21.7 Rib ~ads due to Discontinuities in ~ing Skin Covering
,.1,21.8 Viog vith Cut-out subjected to Torsion Example
An.9 ~ing vith Cut-out subjected to Bending and Torsional
Load s - Examp:';·
\.21.10 Fuselage Frames
,-1.21.11 SlJpporting Boundary Forces on Fuselage Frames
.\21.:2 Calculation of Fra~e 30undary Support;ng Fo.ces
,-\21.~) Problems
A24.1 Assumptions
.\24,2 Formulation of the )-Dimer,sional Squa.::ions of Thennoelasticity
A24.3 The Equilibrium Equations
A24.4 Strain - Displacement Eql.Ja:!orls
A24.5 Stress - Stratn Relations
A24.6 General gquations of 3-Dimensional The~moelasticity
A24.7 The Compa'ibility Equations
A24.8 Boundary Equations
A24.9 General ~otes
A26.1 Introduction
A26.2 One-Dimensional Problems
A16.3 Two-Dimensional Problems
Bl FLIGHT VEHICLE KATRRl.ALS AND THEIR PROPERTIES
B1.1 Introduction
B1.1 Failure of Structures
B1.l Geoeral Types of Loading
B1.4 The Static Tension Stress-Strain Diagram
31. 5 The Static Compression Stress-Strain Diagram
B1.6 Tangent Modulus. Secant Hodulu$
B1.7 Elastic - Inelastic Action
B1.8 Ductility
B1.9 Capaci ty to Absorb Enecgy. Res i 1 i ence. Toughness
B1. 10 Poission's Ratio
B1.11 Construction of a Stress-Strain Curve through a given Yield
Stress by using a kno~n Test Stress-Strain Curve
B1. 12 Non-Dimensional Stress-Strain Curves
31. 13 Influence of Temperature on Katerial Properties
81. 14 Creep of Materials
B1. 15 The General Creep Pattern
B1. 16 Stress-Ti~e Design Charts
B1.17 EfEect of Ti:ne of Exposure
B1. 18 Effect of Rapid Rate of Heating
81. 19 Ge~er~l ~ffect of Lo~ Tempera~u~e upon ~aterial Properties
51.20 Fatigue of Materials
gl. 21 Effect of Irnp~ct Loading on Hatecial Properties
B1. 22 Examples of some Results of Impact Testing of Materials
82 KECHANTCAL MID PHYSICAL PROPER7IES OF M.....'""TALLlC MATF.R.IALS FOR
F'LlG!IT VEHICLE STRUC11JRES
C4.1 IntroductiQn
C4.2 Design for 7ens:on
C4.) Design for Com?ression
C4.4 Column Forllulae for Round Steel Tubes
C4.5 Column For~ulae for Aluminium Alloy Tubes
C4.6 Column Formulae for Hagnesium Alloys
C4.7 Short Column Equations for Other Materials
C4.3 Column Failure due to Local Failure
C4.9 Design Column Charts
C4.10 Section Properties of Round Tubing
C4.11 General Facts in Tubing Design
C4.11 Effect of ~elding of Steel Tubes upon the Tension and
Column Strength
C4.1J Illustrative Proble~s in Strength Checking and Design of Round
Steel Tubes as Columns and Te~$ion Hemoers
C4.14 Illustrative Problems using Aluminium Alloy and Magnesium
Round Tubes as Column and Tension Me~bels
C4.15 Strength of Streamline Tubing
C4.16 Strength of Oval and Square Shaped Tubes in Compression
ULTIMATE BE.NDING STRENGTH OF ROUND TUBES
C4. 1 7 Charts for find~ng the Modulus of Rupture Stress
C4. ~8 Problems involving the Be~d:ng Strength of Tubes
ULTIMATE TORSIONAL STR.RNGTH OF ROUND TUBES
C 7.1 Introduction
C7.2 ~ethod 1. The Angle CSeedham) Method
C7 , 3 Des ign CJ:-V<2S
C7.4 ~e(hod 2. F~r St:-ess Crippli~g Ca~cula:10n (Gerard ~e:hod)
C 1.5 Stresses a~d Disp~ac:e~e~,s of riat ?la:es aEter 3uckling
under Condi tions of ~niform End Sho~:~~ing
C 7.6 The Gerard Equations for C~ippling Stress
C 7• 7 Correction for Claddinl
C 7.8 Haximum ~alues for Crippling Stresses
C7.9 Restraint produced by Lip5 and Bulbs
C7.10 Illustrative Problems in Calcula~ing Crippling Stress
C7.11 Introduction to Effective Sheet Vidths
C7.12 Effecti~e Sheet Vidths
C7.12 Efeective ~idth for Shee~ ~irh One Edg~ Free
C7.13 Effective Vidth vhen Sheet and Stiffe~e" have Different
Material Properties
C7.14 Inter-Rivet Buckling Stress
C7. 15 Illustrative ?roblem involv:ng Effective Sheet
C7. 16 Failing Strength of Short Sheet-Stiffener Panels
in Compression
C7.17 Failure by Inter~Rivet Buckling
C7.18 Failure of Short Panels by Sheet Vrinkling
c7.19 ~quation for irinkling Stress
c7.20 Rivet Criterion for Rivet Failure
C7.21 Illustrating Calculation of Short Panel Failing Strength
c7.22 General Design Limitations to prevent Secondary Failure
in Sheet-Stiffener Panel
C7.23 Y Stiffened Sheet Panels
C7.24 Example Problem Y Stiffened Panel
C7.25 Column Curve for H€mbe~s vith Unstable Cross-Sections
C7.26 Methods used for Deter~ining the Column Failing Stress
in the Transition Region
C7.27 Example Problems involving the ?inding of the Column
Strength of Columns ~fth Unstable Cross-Sections
(7.28 Column Strength of Stiffene~ ~ith Effective Sheet
C7.29 Sheet-Stiffener Panels ~ith Relatively 8e3vy Sheet Thickness
C8 BUCKLfNG STR.&..'K...TB OF KONOCOQlJE CYLINDERS
C8.1 Introductlon
C8.2 Buckling of Honoc.oque CirclJ1a~ Cylinde~s unde:- A..:dal
Compression
C8.3 Additiollal Convenient Desig~ Charts for Deter~ining
Compressive Buckling Stc€SS
C8.4 Plasticity Correction
C8.5 Buckling of !'lonocoque Ci~c:Jla~ ~yli:1de:-s under Axial Load and
Internal P:essuce
e3.6 IntrodiJction ~o BIJckli~g of Monocoque Circular Cylinders under
Pure Be.!ding
C8.7 Available Design Cur.es for 3e~ding Based on Ex?erimental
Results
eB.3 B1Jckling Strength of Circular Cylinde~s in Bending vith
Internal ?cessuce
C8.9 External Hydrostatic P~essure
C8.1O 8xternal Radial Pressure
C3.11 Buckling of Honocoque Circular Cylinde:-s under Pure Torsion
C8.12 Buckling under Transverse Shear
C8.13 B1Jckling of Circular Cylinders under Pure Torsion with
Internal Pressure
c8.14 Buckling of Circular Cylincers under Transverse Shear and
Internal Pressure
C3. 1 S Buckling of Circular Cylinde:-s under Combined Load Systems
C8.16 Illustrative P:-oblems for Finding the Buckling Strength
of Circular Honocoque Cylinders
C8.17 BtJckling of Thin-Valled (Honocoque) Conical Shells
C8.18 Allovable Compressive Buc~l:~g Stress for Thin~Valled
Conica.l Shells
C8.19 Additional Desig~ Buckling Curves for Thin-~alled
Conical Shells
c9 BUCKJ.ING STRE.'K:TB OF CURVED SHEET PA.'fELS. ULTIKATE STRENC"1'S
OF ST I PFENED CURVED SIIEET STRUCTlJRES
C9.1 Introduction
C9.2 State of the Theory
c9.3 Compressive Buckling Stress of Curved Sheet Panels
C9.4 Shear Buckling Str~ss of Curved Sheet Panels
c9.5 Buckling Stren&th of Curved Sheet Panels under Combined
Axial Compress ion and Shear
C9.6 Compressive Bl.lckling Stress of Curved Panels vith
Intecnal ?ressure
C9.7 Shear BIJckling Stress of Curved Sheet Panels with
Internal Pressure
C9.3 Example Proble::"s
C9.9 Introduction to the Ulti~ate Strength of SifEened Cylindrical
Structures
C9.10 Types of Instability Failure of Semi-Monocoque Structures
C9. , 1 The Determination of the Stresses in a Stiffened Cylindrical
Structure under External Loads
C9. '2 Panel Instability Strength
C9. 1] Calculation of Gene:al Instability
C9. 14 Buckling of Spherical P~ates under Unifor~ External Pressure
c13 FATIGlJE
e13.1 Introduction
C1 3.2 Stress Concentra~ions
e13.3 Pat~gue Data
C13.3.1 Unnotched Data
C13.3.2 Notched Jata
C13.3.3 Endurance Limi t
C13.3.4 Patigue Testing Kachines
C13.3.5 Test Scatter
C13.3.6 Load-N CUr'.'es
C13.3.7 Elevated Temperature
e13.5 Increasing Fatigue Resistance
e13.5.1 Introduction
C13.5.2 Stress, Strain and Plastic Deformation
e13.5.3 Effect of Residual StresS on Fatigue
e13.5.4 Mechanically Introduced Residual Stresses
e13.5.5 Residual Stresses by Diffusion Rardening
C13.5.6 Increasing Rigidity
e13.5.7 Combined Rigidity and Residual Stress
e13. 5.8 Load Relief at Fasteners
e16.6 Cumulative Fatigue 8amage
C13.6.1 In t roduc t i.on
e13.6.2 Loads
C13.6.3 Predicting LiEe
Ct1.6.4 St:-es5 :::n:eraction
e13.6.5 Fatigue Stcength :ndex
CI3.6.6 SIJmma~j of Life P~oduction
C1 3. 7 Safe :"ife, Fail Slo'-' a,'ld :;'ailsaEe
e13.7.1 Safe Life and Crack Propagation
C13.7.2 Pai lsafe
e13.8 Frequently Overlooked Problem Areas
C13.8.1 Fatigue Failures of Compressive Surfaces
e13.8.2 Heavy Forging Attachments on St~essed Parts
Cl1.8.4 Pastener Sizes
e11.8.5 Acoustic Fatigue
CU.9 r:heck [,ist
C13.10 Concluding Remarks
FITTINGS AND CONNECTIONS. BOLTED ~VD RIVETED
ot. 1 Introduction
D1.1 Economy in Fitting Des~gn
01. ) fitting Design Loads. Kini~um Hargins of SQfety
01. 4 Special or Higher Factors of Safety
D1.5 Aircraft Bolts
D1.6 Aircraft Nuts
D1.7 Bolt Shear, Tension and Bendi~g Strengths
01.8 801ts in Combined Shear and Te~sion
01. 8a Bushings
01. 9 Single Bolt Fitting
01. 10 ~ethods of Failure of Single Bolt Pitting and the Allo~able
Fai ling Loads
D1.11 Lug Strength Analysis under Axial Loading
D1. 12 Lug Strength Analysis under Transverse Loading
01. 1) Lug Strength Analysis under ObI ique Loads
01. 14 Bolt Bending Strength
D1. 15 Illustrative Problems
D1. 16 Bolt Loads for Multiple Bolt Fittillg. Bolt Sizes Different
Concentric Loading
D1. 17 Kultiple Riveted or Bolted Joints subjected to Eccentric Load
01. 13 Types of Rive~s
[)1.19 Rivet Material
Dl.10 Sttength of Ri~ets. ?rotruding R€dd Type
01. 2 1 Strength of Rivets. F~ush Type
D1 .22 Blind Rivets
D1,23 Riveted She~C Splice In~ornation
D1. 24 Illustrative Frob lens
Dl. 25 Rivets in Tension
D1. 26 ~ivet Tension Strengchs
D2 'lEUl8D COmiECTrONS
01. 1 Introduction
J2.2 Gas 'Jelding
n. ] General Notes on the ?ra~tical Design of _elded 10ints
)2.4 General Types o~ 'Je'...ded Steel fitting Units
02.5 Electric Arc ~elding
D1.6 Bffect of Velding on Base ~etal
02.6 qeld-Hetal Allo~able Stress
'J2.7 Allo~able Load for Velded Sea~s
C2.8 Brazing
Li 2 • 9 ~el~~ug of Aluminium Alloys
01.10 Illustrative Proble~s involvin~ Velding
01. 11 Spa t Veld i ng
D2.12 Spot ~elding of Al~~iniurn Alloys
D2.1) Spot Strengths
02 .14 Reduction of Tensile Str~ngth of ?arent Metal due to
Spot 'Jelding
·lJ.1 Introduction
'.1 3. 2 Shear Clips
)3.3 Tension Clips
c, .1. '
LI~.<t .Joggled Ke:-;joes
D3.5 Fi llers
D}.6 CllL-O"rS in 'Jebs or 5kin p;onels
OJ.7 Special Cases of B~a~ Design
D3.3 Structural Skin ?anel Deta~ls
01.9 Additional Important S~~uc:ural Details
\PPfl.NVIX A - ELEMENTARY KATRlI Al).::;ERRAE
SPECIAL BOOKS NOW AVAILABLE
As explained in the preface, instead of changing to a 2 volume edition in order to include 380
pages of new material, it was decided at the last minute so-to-speak, to remain with one volume and
issue this new material on three different subjects as separate books. A description of these 3 books
follows:
"Engineering Column Analysis" has been written to provide the engineer with practical
methods for analyzing nearly all types of columns and beam-columns, including those on elastic
supports and these loaded into the plastic range of the material. The book has been written in such
a manner that no prior study of elastic stability theory is necessary. The only prerequisite is an
understanding of elementary strength of materials as given in undergraduate engineering courses.
The numerous methods and techniques of analysis are presented for "hand solutions" by the
engineer, but these can, of course, be arranged for analysis by computer if desired. The broad scope
of the book is indicated by the following list of Part titles. References are also provided for further
reading on specific subjects. Much of the material has been used by the author in classes for prac-
ticing engineers in the aircraft industry. It is often surprising to one how easily many seemingly
complicated columns or beam-columns can be analyzed for engineering purposes.
Part Title
The Author has presented a basic review of a new structural technology. He has also presented
an insight into the development of the technology as well as the application of the technology in
practical structural elements.
The Author presents the application of the design and analysis methodology from the micro-
mechanics of fiber matrix interaction to the use of composite lamination in various types of struc-
tural elements and methods of joining. The application analysis also includes the description of
failure modes, a statement of failure criteria, and test methods for the experimental evaluation of
composite materials.
The Author demonstrates the use of all analysis methodology and failure criteria with example
problems taken from ten years of experience in the design with composite materials in the aerospace
industry.
TABLE OF CONTENTS
C13.l Introduction
C13.Ll The Development of a Technology
C13.L2 The Material Concept
C13.L3 The Design Approach to Composite Structures
C14.L4 Nomenclature
C13.4
C13.4.l Laminate Failure Criterian
C13.4.2 Failure modes and Test Methods
C13.4.3 Nonlinear characteristics of Laminate and Failure Modes
This section represents an elaboration for Prof. E. F. Bruhn's book, "Analysis and Design of
Flight Vehicle Structures." It has been designed to introduce the student and beginning engineer to
the general field of missile structures and to give a limited presentation of the preliminary load,
stress analysis, and structural design practices of typical boost missiles.
The first portion is both analytical and descriptive in nature and serves to acquaint the reader
with current practices in configuration, material usage, structural techniques, and design factors.
The main body, or Parts E1.7 and E1.8, takes a hypothetical multistage vehicle, derives the critical
flight load conditions, and indicates analysis for these loads. The analysis techniques derive directly
from the body of Professor Bruhn's book, "Analysis and Design of Flight Vehicle Structures"; hence
giving the reader examples of practical usage of that data for missile design.
TABLE OF CONTENTS
(Drawing - Courtesy of N.
APOLLO/SATURN VEHICLE
1959 - 60 BEGINNING OF THE "JET" AGE OF AIR TRANSPORTATION - 550-650 MPH
1928 BOEING MODEL 80-A 1930 FORD TRI-MOTOR 1931 BOEING "MONOMAIL" 1933 LOCKHEED "ELECTRA"
1903 FIRST FLIGHT BY 1912 GLEN. L. MARTIN 1918 STANDARD AIRCRAFT 1926 FOKKER TRANSPORT
WRIGHT BROTHERS IN HIS BIPLANE CORP BIPLANE
L....-_ PHOTOGRAPHS ILLUSTRATING PROGRESS OF AIRCRAFT DESIGN FOR AIR TRANSPORTATION - 40 TO 175 MPH
GENERAL DYNAMICS F-lll AIR FORCE/NAVY
TACTICAL FIGHTER. SPEED 2000 M. P. H.
VARIABLE POSITION WING (1965)
1930 MARTIN P3M-l NAVY 1933 VOUGHT-NAVY SU-4 1936 GRUMMAN-NAVY SF-l 1939 VOUGHT-SIKORSKY S02U
1908 "WRIGHT" AIRPLANE 1918 CURTIS ':JENNY" 1921 BOEING-ARMY 1929 CURTIS P-6 "HAWK"
FIRST ARMY AIRPLANE ARMORED ATTACK AIRPLANE
SOME PRE-WORLD WAR II AIRCRAFT
I----PHOTOGRAPHS ILLUSTRATING TREMENDOUS PROGRESS OF MILITARY AVIATION--.....
LUNAR ORBITER. Will orbit the moon taking
pictures (1966)
The Age for Spacecraft is just dawning. Regardless of the type of Spacecraft,
its creation will involve the work of the structural designer.
Drawing Showing General Layout
of Modern Missile
26
25
24
12
23
22
16
NASA DELTA SPACE VEHICLE
MODEL DSV-3B
DDUG~
1. S-17 (OSO) Spacecraft 7. Retro-System Nitrogen Bottle 12. Fuel Tank 18. Stabilizing Fim 24. Guidance Antenna
2. Fairing 8. BTL 600 Series Guidanee 13. Wiring Tunnels 19. 1st Stage Telemetry 25. 2nd Stage Telemetry 21
3. Spin Table 9. Fuel Tank 14. Telemetry Antenna 20. Anti-vortex BajJle Antenna
20
4. Spin Rocket Holder 10. Helium Sphere 15. Oxidizer Tank 21. Anti-slash Baffles 26. ABL X 248-A5DM Solid
5. 2nd Stage Programmer 11. Aerojet AJIO-H8D 16. Turbopump 22. Interstage Structure Propellant M owr
6. Eledronics Padroge Assembly 2nd Stage Engine 17. Rockeldyne 1st Stage Engine 23. Oxidizer Tank 27. Spaeecraft Attaeh Fitting 19
18
I-
Ql c
en en
c - ."
.;: Z Ql
+-
Vl 0"
--0
a.. I-
0::: ::>
Ou
U ::>
Vll:
= ."
u-=E
0- I -
00 U
I .::
u.. 0
ti:-o
~ .~
0::: Q..
U >...
o:::+-
~ (G
+-
a.. ~
Ot;
0:::2
:1::::
I-
0::: en
0.=
Z ~
~
o
LIFTOFF AT THE CAPE - the huge Saturn V S-IC SEPARATION - first stage drops away, S-IVB IGNITES - $-II stage is jettisoned;
in a 71 /,-million-pound thrust toward the stars. and the Saturn S-II second stage is now ignited. S-IVB hurls Apollo spacecraft into earth orbit.
2 3
TRANSLUNAR INJECTION - S-IVB stage pushes APOLLO - S-IVB SEPARATION - the adapter FREE FLY-AROUND - Apollo craft turns
Apollo spacecraft out of earth's gravity pull. panels are blown free; Apollo craft separates. around in space to dock with the waiting LEM.
4 5 6
~.j/
..."",~
..
..... '\
~
. . . . ..
LEM EXPOSED - two crewmen enter LEM LUNAR ORBIT - Service Module engine THE DESCENT-Apollo croft remains in
for an equipment checkout; S-IVB is left behind. retro-fires to put craft in aO-mile lunar orbit. orbit while two nstronouts guide LEM to the moon.
7 8 9
LUNAR EXPLORATION - during moon stay MOON BLAST-OFF - the LEM employs its LUNAR RENDEZVOUS - the LEM ClstrOllauts
fwo astronauts undertake scientific observations. expended descent stage (JS plotfonn for jift-off. line up ernft for critical dock'm(Jte m(Jneuver.
10 11 12
HOMEWARD BOUND - LEM stays behind in SIM JETTISONED - prior to entry into REENTRY - pilot now orients Command
lunar orbit as three men head Apollo for earth. the earth's atmosphere, S/M is detached and left. Module so base heat shield takes friction heat.
13 14 15
(1) Wind Tunnel and Fluid Mechanics Test The final results of the work of this
Labs. group are formal reports giving complete applied
(2) Structural Test Labs. load design criteria, with many graphs and sum-
(3) Propulsion Test Labs. mary tables. The final results may give com-
(4) Electronics Test Labs. plete shear, moment and normal forces referred
(5) Electro-Mechanical Test Labs. to a convenient set of XYZ axes for major air-
(6) Weapons and Controls Test Labs. craft units such as the wing, fuselage, etc.
(7) Analog and Digital Computer Labs.
THE WORK OF STRESS ANALYSIS AND STRENGTH GROUP
V. Flight Test Section.
Essentially the primary job of the stress
VI. Engineering Field Service Section. group is to help specify or determine the kind
of material to use and the thickness, size and
Since this textbook deals with the subject cross-sectional shape of every structural mem-
of structures, it seems appropriate to discuss ber or unit on the airplane or missile, and
in some detail the work of the Structures Group. also to assist in the design of all joints and
For the detailed discussion of the other groups, connections for such members. Safety with light
the student should refer to the various air- weight are the paramount structural design re-
craft company publications. quirements. The stress group must constantly
work closely with the structural Design Section
AI. 3 The Work of the Structures Group in order to evolve the best structural over-all
arrangement. Such factors as power plants,
The structures group, relative to number of built-in fuel tanks, landing gear retracting
engineers, is one of the largest of the many wells, and other large cut-outs can dictate the
groups of engineers that make up Section II, type of wing structure, as for example, a two
the technical analysis section. The structures spar single cell wing, or a multiple spar
group is primarily responsible for the multiple cell wing.
structural integrity (safety) of the airplane.
Safety may depend on sufficient strength or To expedite the initial structural design
sufficient rigidity. This structural integrity stUdies, the stress group must supply initial
must be accompanied with lightest possible structural sizes based on apprOXimate loads.
weight, because any excess weight has detri- The final results of the work by the stress
mental effect upon the performance of aircraft.. group are recorded in elaborate reports which
For example, in a large, long range missile, show how the stresses were calCUlated and how
one pound of unnecessary structural weight may the required member sizes were obtained to carry
add more than 200 Ibs. to the overall weight of these stresses efficiently. The final size of
the missile. a member may be dictated by one or more factors
such as elastic action, inelastic action, ele-
The structures group is usually divided vated temperatures, fatigue, etc. To insure
into sub-groups as follows:- the accuracy of theoretical calculations, the
stress group must have the assistance of the
(1) Applied Loads Calculation Group structures test laboratory in order to obtain
(2) stress Analysis and Strength Group information on which to base allowable design
(3) Dynamics Analysis Group stresses.
(4) Special Projects and Research Group
THE WORK OF THE DYNAMICS ANALYSIS GROUP
THE WORK OF THE APPLIED LOADS GROUP
The Dynamics Analysis Group has rapidly
Before any part of the structure can be expanded in recent years relative to number of
finally proportioned relative to strength or engineers required because supersonic airplanes,
rigidity, the true external loads on the air- missiles and vertical rising aircraft have pre-
craft must be determined. Since critical loads sented many new and complex problems in the
come from many sources, the Loads Group must general field of dynamics. In some aircraft
analyze loads from aerodynamic forces, as well companies the dynamics group is set up as a
as those forces from power plants, aircraft separate group outside the Structures Group.
inertia; control system actuators; launching,
landing and recovery gear; armament, etc. The The engineers in the dynamics group are
effects of the aerodynamic forces are initially responsible for the investigation of vibration
calculated on the assumption that the airplane and shock, aircraft flutter and the establish-
structure is a rigid body. After the aircraft ment of design reqUirements or changes for its
structure is obtained, its true rigidity can control or correction. Aircraft contain dozens
be used to obtain dynamic effects. Results of of mechanical installations. Vibration of any
wind tunnel model tests are usually necessary part of these installations or systems may be
in the application of aerodynamic principles to of such character as to cause faulty operation
load and pressure analysis. or danger of failure and therefore the dynamic
AI. 3
clnracteristics must be changed or modified in groups have a special sUb-group which is work-
order to insure reliable and safe operation. ing on design problems that will be encountered
in the near or distant future as aviation pro-
Tne major structural units of aircraft such gresses. For example, in the Structures Group,
as the wing and fuselage are not rigid bodies. this sub-group might be studying such problems
Thus when a sllilrp air gust strikes a flexible as: (1) how to calculate the thermal stresses
wing in high speed flight, we have a dynamic in the wing structure at su)ersonic speeds;
load situation and the wing will vibrate. The (2) how to stress-analyze a new type of wing
dynami,~ist must determine whsther this vibration strUi;ture; (:3) what type of body structure is
is serious relative to induced stresses on the best for future space travel and what kind of
wing stru~ture. The dynamics group is also materials will be needed, etc.
responsible for the determination of the
stabi Ii ty ami performan,;e of missile antj flight Chart 1 illustrates in general a typical
vehic;le glLldl:J.,;e and control systems. The make-up of the Structures Section of a large
dynamL;s group must work (~onstantly with the aerospace compa:1Y. Chart 2 lists the many
various test laboratories in order to obtain items which the structures engineer must be
reliable values of certain fa:;tors that are con'cerned with in insuring the structural
necessdry in m'lny theoreti'~'ll cal,;ulations. integrity of the flight vehicle. Both Charts
land 2 are from Chance-Vought Structures
Tnt<; WOH!\: OF THS SPECIAL PHOJ~CTS GROUP Design ~lnual and are reproduced with their
permission.
In g:meral, all the various techIlical
STRUCTURES SECTION
CHIEF OF STRUCTURES
STRUCTURES STRUCTURES
PROJECT MATERIALS
ENGINEERS GROUP
LOADS AND
METHODS
UNIT
STRUCTURES
TECHNICAL
GROUP
- NON-METALLIC
~-
UNIT
METALLIC
UNIT
AEROElASTlC RESEARCi:
UNIT AND
DEVELOPMENT
UNIT
AEROElASTIC
DESIGN
AEROELASTIC
RESEARCH
AND
DEVElOPMENT
STRESS
ANALYSIS
UNIT
- STRUCTURES
TEST
LABORATORY
GROUP
STRUCTURES
TEST UNIT
HYDRAULIC
AND POWER
-
PLANT TEST
DESIGN UNIT
WEIGHT
CONTROL
GROUP
DESIGN
ALLOWA8LES
DESIGN
METHODS
Chart 1.
- MACHINE
COMPUTATION
GROUP
MATERIALS OF
CONSTRUCTION
FASTENERS
WELDING
BONDING
PLATE AND BAR
FORGINGS
STIFFNESS CASTINGS
EXTRUS IONS STRESS
CRITERIA SHEET METAL
FLUTTER SANDWICH
ANALYSIS
PLASTIC LAMINATE SKIN PANELS
CONTROL SYSTEM STABILITY
BEARINGS BEAM ANALYS IS
PANEL FLUTIER-SKIN CONTOURS STRAIN COMPATIBILITY
STRA IN CONCENTRATION
CONTROL SYSTEM DEFLECTIONS
JOINT ANALYSIS
THERMAL EFFECTS BEAR ING ANALYS IS
BULKHEAD ANALYS IS
MECHAN ICAL VIBRAT IONS
FIniNG ANALYSIS
ROLL POWER-D IVERGENCE THERMAL STRESS
MECHANICAL COMPONENTS
AERODYNAMIC CENTER SHIFT
EXPERIMENTAL STRESS ANALYSIS
DYNAMIC RESPONSE
LOADS AND
ENVIROMENT MATERIALS AND
FLI GHT LOAD CR ITER IA QUALITY CONTROL
GROUND LOAD CR ITER IA
FLIGHT LOAD DYNAMICS DUCTILITY
LAUNCHING DYNAMICS STRESS-STRAIN
LANDING DYNAMICS
DYNAMIC RESPONSE HOMOGENEOUS MATER IAL
RECOVERY DYNAMICS RESIDUAL STRESS
FLIGHT LOAD DISTRIBUTIONS
COMPONENT ALLOWABLES HEAT TREAT CONTROL
INERTIAL LOAD DISTRIBUTIONS
YIELDING
FLEX IBIL1TY EFFECTS ANALYSIS FRACTURE
STRESS CORROSION
GROUND LOAD DISTRIBUTIONS
REPEATED LOAD SPECTRUMS UN IT SOLUTIONS STABILITY AT TEMPERATURE
FATIGUE
TEMPERATURE DISTR IBUT IONS INDETERMINATE STRUCTURES SPECIFICATION CONFORMANCE
LOADS FROM THERMAL WEAR, BRINELLING
DEFORMATIONS WINGANALYSIS BLUE PRINT CONFORMANCE
CREEP
PRESSURES-IMPACT TAIL ANALYSIS
DEFLECTIONS
FUSELAGE SHELL ANALYS IS
THERMAL EFFECTS
THERMAL ANALYSIS
STIFFNESS
DEFLECTION ANALYS IS
COMBINED LOADINGS
STIFFNESS
BUCKLING
Chart 2
From Chance- Vought Structures Design Manual
CHAPTER A2
EQUILIBRIUM OF FORCE SYSTEMS. TRUSS STRUCTURES
A2.1 Introduction. The equations of static force system pass through a common point. The
equilibrium must constantly be used by the reSUltant, if any, must therefore be a force
stress analyst and structural designer in ob- and not a moment and thus only 3 equations are
taining unknown forces and reactions or unknown necessary to completely define the condition
internal stresses. They are necessary whether that the resultant must be zero. The equations
the structure or machine be simple or complex. of equilibrium available are therefore:-
The ability to apply these equations is no
doubt best developed by solving many problems. ZF x =0 ZMJ. = oo
This chapter initiates the application of these ZF y =0 or ZM" =o } - - - - -(2.2)
important physical laws to the force and stress
analysis of structures. It is assumed that a
ZFz =0 ZM a =
student has completed the usual college course A combination Oi force and moment equations
in engineering mechanics called statics. to make a total of not more than 3 can be used.
For the moment equations, axes through the point
A2.2 Equations of Static Equilibrium. of concurrency cannot be used since all forces
of the system pass through this point. The
To completely define a force, we must know moment axes need not be the same direction as
its magnitude, direction, and point of applica- the directions used in the force equations but
tion. These facts regarding the force are of course, they could be.
generally referred to as the characteristics of
the force. Sometimes the more general term of EQ.UILIBRIUM OF SPACE PARALLEL-FORCE SYSTEM
line of action or location is used as a force
characteristic in place of point of application In a parallel force system the direction of
designation. all forces is known, but the magnitude and
location of each is unknown. Thus to determine
A force acting in space is completely magnitude, one equation is required and for
defined if we know its components in three location two equations are necessary since the
directions and its moments about 3 axes, for force is not confined to one plane. In general
example Fx , Fy ' Fz and Mx , My and Mz • For the 3 equations commonly used to make the re-
equilibrium or a force system there can be no sultant zero for this type of force system are
resultant force and thus the equations of one force equation and two moment equations.
equilibrium are obtained by equating the force For example, for a space parallel-force system
and moment components to zero. The equations acting in the y direction, the equations of
of static equilibrium for the various types of eqUilibrium would be:
force systems will now be summarized.
ZF y .:: 0, ZMx = 0, ZMz = 0 - -(2.3)
EQ.UILIBRIUM EQ.UATIONS FOR GENERAL
SPACE (NON-COPLANAR) FORCE SYSTEM EQ.UILIBRIUM OF GENERAL CO-PLANAR FORCE SYSTEM
ZFx =0 ZMJ. = 0 In this type of force system all forces lie
ZF y =0 ZM" = 0 } - - - - -(2.1) in one plane and it takes only 3 equations to
ZFz =0 ZM a = 0 determine the magnitude, direction and location
of the reSUltant of such a force system. Either
Thus for a general space force system, force or moment equations can be u~ed, except
there are 6 equations of static equilibrium that a maximum of 2 force equations can be used.
available. Three of these and no more can be For example, for a force system acting in the
force equations. It is often more convenient xy plane, the following combination of equili-
to take the moment axes, 1, 2 and 3, as any set brium equations could be used.
of x, y and z axes. All 6 equations could be
moment equations about 6 different axes. The
force equations are written for 3 mutually ZFx = 0 ZFx = 0 ZF y = 0 ZM z J. = 0
perpendicular axes and need not be the x, y ZF y = 0 or ZMzJ.= 0 or ZMzJ.= 0 or ZMz " = 0 2.4
and z axes.
ZM z= 0 ZMz" = 0 ZMz" = 0 ZM za = 0
EQ.UILIBRIUM OF SPACE CONCURRENT-FORCE SYSTEM (The SUbscripts 1, 2 and 3 refer to different
Concurrent means that all forces of the locations for z axes or moment centers.)
A2.1
A2.2 EQUILIBRIUM OF FORCE SYSTEMS. TRUSS STRUCTURES.
A colinear force system is one where all For any force such as P and Q acting in the
forces act along the same line or, in other xy plane, the line of action of such a force
words, the direction and location of the forces must pass through the pin center since the
are known but their magnitudes are unknown; fitting unit cannot resist a moment about a z
thus only magnitude needs to be found to define axis through the pin center. Therefore, for
the resultant of a colinear force system. Thus forces acting in the xy plane, the direction
only one equation of eqUilibrium is available, and line of action are established by the pin
namely joint as illustrated in the figure. Since a
single pin fitting can resist moments about axes
ZF =0 or ZM~ =0 2.7 perpendicular to the pin axis, the direction and
line of action of out-of-plane forces is there-
where moment center 1 is not on the line of fore not established by single pin fitting units.
action of the force system
_(0
A B
0)_
ex
A2. 3 Structural Fitting Units for Establishing the Force
Characteristics of Direction and Point of Application.
If a bar AB has single pin fittings at
To completely define a force in space re- each end, then any force P lying in the xy
quires 6 equations, or 3 equations if the force plane and applied to end B must have a direction
is limited to one plane. In general a structure and line of action coinciding with a line join-
is loaded by known forces and these forces are ing the pin centers at end fittings A and B,
transferred through the structure in some since the fittings cannot resist a moment about
manner of internal stress distribution and then the z axis.
A2.3
Double Pin - Universal Joint Fittings Another general fitting type that is used
to establish the direction of a force or reaction
is illustrated in the figure at the bottom of the
first column. Any reacting force at joint (A)
must be horizontal since the support at (A) is
so designed to provide no vertical resistance.
_A~B_
fitting support or attachment units are to be
used or are assumed to be used in the final
~
design. The following sketch symbols are com-
monly used for coplanar force systems.
Pin
In order to permit structures to move at _----"(0"'-)_ _-'1--
~Pin
Q,
- ,-I
-&
Rx I
I
I
t
reaction is also known, hence only one force ~Ry +Ry
characteristic, namely magnitude, is unknown
for a roller-pin type of fitting. For the
fitting unit at (A), point of application of the
reaction to the truss is known because of the The above graphical symbols represent a
pin, but direction and magnitude are unknown. reaction in which translation of the attach-
ment point (b) is prevented but rotation of the
Lubricated Slot or Double Roller Type of Fitting attached structure about (b) can take place.
Unit. Thus the reaction is unknown in direction and
magnitude but the point of application is known,
~
"A"
~
namely through point (b). Instead of using
direction as an unknown, it is more convenient
Lubricated Slot t ~ ! Double Ron: to replace the resultant reaction by two com-
4-
t
Rollers ~ Rollers brium. A structure can be statically indeter-
minate with respect to external reactions alone
or to internal stresses alone or to both.
~ +
The additional equations that are needed
to solve a statically indeterminate structure
are obtained by considering the distortion of
The above fitting units usinG rollers fix the structure. This means that the size of all
the direction of the reaction as normal to the members and the material from which members are
roller bed since the fitting unit cannot resist made must be known since distortions must be
a horizontal force through point (b). Hence calculated. In a statically determinate
the direction and point of application of the structure this information on sizes and material
reaction are established and only magnitude is is not required but only the configuration of
unknown. the structure as a whole. Thus design analysis
for statically determinate structures is
straightforward, whereas a general trial and
fixed error procedure is required for design analysis
'-'-1-1-1-\ of statically indeterminate structures.
the structure would be still stable and all mine the axial loads in the members and the re-
member axial stresses could be found by the actions on the spar.
equations of static equilibrium without regard
to their size of cross-section or the kind of Solution: The first thing to decide is whether
material. Adding the second diagonal member the structure is statically determinate. From
in each panel would necessitate knowing the the figure it is observed that the wing spar is
size of all truss members and the kind of supported by five struts. Due to the pins at
material used before member stresSes could be each end of all struts, we have five unknowns,
found, as the additional equations needed must namely, the magnitude of the load in each strut.
come from a consideration involving distortion Direction and location of each strut load is
of the truss. Assume for example, that one known because of the pin at each end of the
diagonal in the upper panel was left out. We struts. We have 3 equations of equilibrium for
would then be able to find the stresses in the the wing spar as a single unit supported by the
members of the upper panel by statics but the 5 struts, thus two more equations are necessary
lower panel would still be statically inde- if the 5 unknown strut loads are to be found.
terminate to 1 degree because of the double It is no~tced that the wing spar includes 2 in-
diagonal system and thus one additional equation ternal single pin connections at points 0 and 0'.
is necessary and would involve a consideration This establishes the fact that the moment of all
of truss distortion. (The solution of static- forces located to one side of the pin must be
ally indeterminate trusses is covered in equal to zero since the single pin fitting can-
Chapter AS.) not resist a moment. Thus we obtain two addi-
tional equations because of the two internal pin
A2. 7 Example Problem Solutions of Statically Determinate fittings and thus we have 5 equations to find 5
Coplanar Structures and Coplanar Loadings. unknowns.
Although a student has taken a course in
statics before taking a beginning course in Fig. 2.15 shows a free body of the wing
spar to the right of hinge fitting at O.
aircraft structures, it is felt that a limited
review of problems involving the application y 2460=82 x 30 1013=(30 + 15)45
of the equations of static equilibrium is quite
justified, particularly if the problems are Lx l;--41"~1"-+20~-2-
possibly somewhat more difficult than most of Xo-6 ----e I
the problems in the usual beginning course in
statics. Since one must use the equations of
l Yo
~~lA
,. 117 Y.
static equilibrium as part of the necessary E A Fig. A2. 15
equations in solving statically indeterminate
structures and since statically indeterminate In order to take moments, the distributed
structures are covered in rather complete detail load on the spar has been replaced by the re-
in other chapters of this book, only limited sultant load on each spar portion, namely, the
space will be given to problems involving total load on the portion acting through the
statics in this chapter. centroid of the distributed load system. The
strut reaction EA at A has been shown in phantom
Example Problem S. as it is more convenient to deal with its com-
ponents YA and XA. The reaction at 0 is un-
Fig. A2.14 shows a much simplified wing known in magnitude and direction and for con-
structure, consisting of a wing spar supported venience we will deal With its components Xo
by lift and cabane struts which tie the wing and YO. The sense assumed is indicated on the
spar to the fuselage structure. The distributed figure.
air load on the wing spar is unsymmetrical about
the center line of the airframe. The wing spar The sense of a force is represented
is made in three units, readily disassembled by graphically by an arrow head on the end of a
using pin fittings at points 0 and 0·. All vector. The correct sense is obtained from the
supporting wing struts have single pin fitting solution of the equations of equilibrium since
units at each end. The problem is to deter- a force or moment must be given a plus or minus
sign in writing the equations. Since the sense
of a force or moment is unknown, it is assumed,
20,*/in. 40,*/in. I 30'*jin
15#/in. and if the algebraic solution of the equilibrium
J.r:rf] f f f f f 1ft I f f f i t t t f T":r"=
equations gives a plus value to the magnitUde
f f.- 45"-+- 82" ---iP'1---60" ---J H~ge then the true sense is as assumed, and opposite
A B 0" A Pin to that assumed if the solution gives a minus
Lift -+- sign. If the unknown forces are axial loads in
Struts 3 " members it is COmmon practice to call tensile
Fuselage stress plus and compressive stress minus, thus
Fig. A2.14 If we assume the sense of an unknown axial load
as tension, the solution of the equilibrium
A2.8 EQUILIBRIUM OF FORCE SYSTEMS. TRUSS STRUCTURES.
equations will give a plus value for the magni- To find strut load B'C' take moments about
tude of the unknown if the true stress is point C.
tension and a minus sign will indicate the
assumed tension stresses should be reversed or ZMc =1325 x 65 + 2000 x 40 + (5880 - 4400)
compression, thus giving a consistency of signs. 30 - 1500 x 10 - 993 x 35 - 30 (B'C')
30/33.6 =
0
To find the unknown YA we take moments
about point 0 and equate to zero for equilibrium whence, B'C' = 6000 lb. with sense as shown.
ZMO = - 2460 x 41 1013 x 102 + 82YA =0 To find load in member B'C use equation
Hence YA =
204000/82 2480 lb. The plus sign = ZF y = =
0 1325 + 2000 + 1500 + 993 - 6000
means that the sense as assumed in the figure (30/33.6) - 2720 (30/33.6) - B'C (30/54)
is correct. By geometry XA = 2480 x 117/66 = =0
4400 lb. and the load in strut EA equals
v4400 2 + 2480 2 =
5050 lb. tension or as whence, B'C =-
3535 lb. The minus sign
assumed in the figure. means it acts opposite to that shown in figure
or is compression instead of tension.
To find Xo we use the equilibrium equation The reactions on the spar can now be
ZFX = 0 = Xo - 4400 = 0, whence Xo 4400 lb. = determined and shears, bending moments and
axial loads on the spar could be found. The
To find YO we use, numerical results should be checked for equili-
ZF y = 0 = 2460 + 1013 - 2480 - YO = 0, whence brium of the spar as a whole by taking moments
YO = 993 lb. of all forces about a different moment center
to see if the result is zero.
To check our results for equilibrium we
will take moments of all forces about A to see Example Problem 9.
if they equal zero.
ZMA = 2460 x 41 - 1013 x 20 - 993 x 82 =0 check RB\ f.--- 1~' + 12"----1 ;rRD
588.Q.
1---
l-- 20,,--15" r
<£==~~======-:i;;==:Jl--4400
50" 5" 20"-1
are colinear with the strut axis, thus direction the aluminum alloy tubular truss. Trussed type
and point of application are known for reaction beams composed of closed and open type sections
RB and RD leaving only the magnitude of each as are also frequently used in wing beam construc-
unknown. The roller type fitting at C fixes tion. The stresses or loads in the members of
the direction and point of application of the a truss are commonly referred to as "primary"
reaction RC' leaving magnitude as the only and "secondary" stresses. The stresses which
unknown. Thus there are 3 unknowns RB' Rc and are found under the following assumptions are
RD and with 3 equations of static equilibrium referred to as primary stresses.
available, the structure is statically determi-
nate with respect to external reactions. The (1) The members of the truss are straight,
sense of each of the 3 unknown reactions has weightless and lie in one plane.
been assumed as indicated by the vector. (2) The members of a truss meeting at a
point are considered as joined together by a
To find RD take moments about point B:- common frictionless pin and all member axes in-
2MB =- 10000 sin 30 0 x 36 - 10000 cos 30 0 x 12 tersect at the pin center.
- RD (12/17) 24 0 = (3) All external loads are applied to the
whence, RD = - 16750 lb. Since the result truss only at the joints and in the plane of
comes out with a minus sign, the reaction RD the truss. Thus all loads or stresses produced
has a sense opposite to that shown by the in members are either axial tension or compres-
vector in Fig. 2.17. Since the reaction RD is sion without bending or torsion.
colinear with the line DE because of the pin
ends, the load in the brace strut DE is 16750 Those stresses produced in the truss mem-
lb. compression. In the above moment equation bers due to the non-fulfillment of the above
about B, the reaction RD was resolved into assumptions are referred to as secondary
vertical and horizontal components at point D, stresses. Most steel tubular trusses are welded
and thus only the vertical component which tovether at their ends and in other truss types,
equals (12/17) RD enters into the equation the members are riveted or bolted to~ether.
since the horizontal component has a line of This restraint at the joints may cause second-
action through point B and therefore no moment. ary stresses in some members greater than the
RC does not enter in equation as it has zero primary stresses. Likewise it is common in
moment about B. actual practical design to apply forces to the
truss members between their ends by supporting
To find RB take ZFy =0 many equipment installations on these truss
members. However, regardless of the magnitude
ZFy = 10000 x cos 30 0 + (- 16750)(12/17) + RB of these so-called secondary loads, it is
(24/26.8) = 0 common practice to first find the primary
whence, RB =
3540 lb. Since sign comes stresses under the assumption outlined above.
out plus, the sense is the same as assumed in
the figure. The strut load BF is therefore GENERAL CRITERIA FOR DETERMINING WHETHER
3540 lb. tension, since reaction RB is colinear TRUSS STRUCTURES ARE STATICALLY DETERMINATE
wi th line BF. WITH RESPECT TO INTERNAL STRESSES.
ZH = (-1250) (30/50)
- 1875 (30/50) - U"U" 0 = The algebraic sign of all unknowns came out
positive, thus the assumed direction as shown
whence, U"U" = - 1875 lb. or opposite in on Fig. A2.22 was correct.
sense to that assumed and therefore compression.
Check results by taking 2MB = 0
Note: The student should continue with succeed-
ing joints. In this example involving a canti- 2MB =
1400 x 150 + 500 x 30 - 500 x 120 - 500 x
lever truss it was not necessary to find the 30 - 1000 x 90 - 1000 x 60 = 0 (Check)
reactions, as it was possible to select ~oint To determine the stress in member F", F" and Fa
La as a j oint involving only two unknowns. In we cut the section 1-1 thru the truss (Fig.
trusses such as illustrated in Fig. A2.22 it is A2.22. Fig. A2.23 shows a free body diagram of
necessary to first find reactions R" or R" which the portion of the truss to the left of this
then provides a joint at the reaction point in-
section.
volving only two unknown forces.
1
:"F~_
500 -_:..---,,---~~-
1400
500
~.
~;,
1400 1
Fig. A2. 23 Fig.A2.24
A2. 10 Method of Moments.
For a coplanar-non-concurrent force system The truss as a whole is in equilibrium.
there are three equations of statics available. Therefore any portion must be in eqUilibrium.
These three equations may be taken as moment In Fig. A2.23 the internal stresses in the mem-
equations about three different points. Fig. bers F", F" and Fa which existed in the truss as
A2.22 shows a typical truss. Let it be re- a whole now are considered external forces in
qUired to find the loads in the members F", F", holding the portion of the truss to the left of
Fa, F 4' F 6 and F 6 ' section 1-1 in equilibrium in combination with
the other loads and reactions. Since the mem-
bers a and b in Fig. A2.23 have not been cut the
Fig. A2. 22 loads in these members remain as internal
10"
stresses and have no influence on the eqUilib-
rium of the portion of the truss shown. Thus
the portion of the truss to left of section 1-1
could be considered as a so11d block as shown
, in Fig. A2.24 without affecting the values of
2
f<-----5 @ 30" = 150" - - - - - - - F", F" and Fa. The method of moments, as the
name implies, involves the operation of taking
moments about a point to find the load in a
The first step in the solution is to find the particular member. Since there are three un-
reactions at points A and B. Due to the roller knowns a moment center must be selected such
type of support at B the only unknown element of that the moment of each of the two unknown
the reaction force at B is magnitude. At point stresses will have zero moment about the selected
A, magnitude and direction of the reaction are moment center, thus leaVing only one unknown
unknown giving a total of three unknowns with force or stress to enter into the equation for
three equations of statics available. For con- moments. For example, to determine load Fa in
venience the unknown reaction at A has been re- Fig. A2.24 we take moments about the inter-
placed by its unknown H and V components. section of forces F" and F" or point O.
Taking moments about point A, Thus ZMO = 1400 x 30 - 18.97 Fa =0
ZMA = 500 x 30 + 1000 x 60 + 1000 x 90 + 500 x 42000
30 + 500 x 120 - 150 VB 0 = Hence Fa = 18.97 = 2215 lb. compression (or
acting as assumed)
Hence VB = 1600 lb.
To find the arm of the force Fa from the
Take ZV 0 = moment center 0 involves a small amount of cal-
ZV = VA - 1000-1000-500-500 + 1600 =0 there- culation, thus in general it is simpler to re-
fore VA = 1400 lb. solve the unknown force into H and V components
at a point on its line of action such that one
Take ZH =0 of these components passes thru the moment
ZH = 500 - HA = 0, therefore HA = 500 lb. center and the arm of the other component can
usually be determined by inspection. Thus in
A2.12 EQUILIBRIUM OF FORCE SYSTEMS. TRUSS STRUCTURES.
Fig. A2.25 the force Fa is resolved into its the other two unknowns Fe and Fe lies at infini-
component FaV and FaH at point 0'. Then taking ty. Thus for conditions where two of the 3 cut
members are parallel we have a method of solving
for the web member of the truss commonly re-
ferred to as the method of shears, or the sum-
mation of all the forces normal to the two
parallel unknown chord members must equal zero.
Fig. A2. 25
Since the parallel chord members have no com-
ponent in a direction normal to their line of
action, they do not enter the above equation of
equilibrium.
1000#
2I
Fig. A2. 26 1
1000
:..:..:;j""-----r-----"O...-.;..-
500#
500#
Fe
1400#
Fig. A2. 27
500
moments about point 0 as before:-
Fig.A2.28
ZMo = 1400 x 30 - 20Fa H =0
Referring to Fig. A2.27
whence, FaH = 2100 lb. and therefore
ZV =1400 - 500 - 1000 - F.. (l/y'2:).= 0
Fa =
2100 (31.6/30) = 2215 lb. as pre-
viously obtained. whence F.. = - 141 lb. (tension or opposite
to that assumed in the figure.
The load F:a. can be found by taking moments
about point m, the intersection of forces Fe To find the stress in member F7 • we cut
and Fa (See Fig. A2.23). section 3-3 in Fig. A2.22 and draw a free body
diagram of the left portion in Fig. A2.26.
ZMm = 1400 x 60 + 500 x 30 - 500 x 30 Since F:a. and Fe are horizontal, the member F7
- 30F:a. = 0 must carry the shear on the truss on this section
3-3, hence the name method of shears.
whence, F:a. = 2800 lb. (Tension as assumed)
ZV = 1400 - 500 - 1000 + F = 0 7
To find force Fe by using a moment equation,
we take moments about point (r) the inter- Whence F = 100 lb. (compression as
7 assumed)
section of forces F:a. and F .. (See Fig. A2.26).
To eliminate solving for the perpendicular Note: The stUdent should solve this example il-
distance from point (r) to line of action of lustrating the methods of moments and shears
Fe, we resolve Fe into its H and V components using as a free body the portion of the truss to
at point 0 on its line of action as shown in the right of the cut sections instead of the
Fig. A2.26. left portion as used in these illustrative ex-
amples. In order to solve for the stresses in
ZMr =- 1400 x 30 + 500 x 60 + 60 FeV =0 the members of a truss most advantageously, one
usually makes use of more than one of the above
whence, FeV = 12000/60 = 200 lb. three methods, as each has its advantages for
certain cases or members. It is important to
Therefore Fe = 200 x V2 = 282 lb. com- realize that each is a method of sections and in
pression a great many cases, such as trusses with paral-
lel chords, the stresses can practically be
A2. 11 Method of Shears found mentally without writing down equations of
equilibrium. The following statements in gen-
In Fig. A2.22 to find the stress in member eral are true for parallel chord trusses:
F .. we cut the section 2-2 giving the free body
for the left portion as shown in Fig. A2.27. (1) The vertical component of the stress in
the panel diagonal members equals the vertical
The method of moments is not sufficient to shear (algebraic sum of external forces to one
solve for member F.. because the intersection of side of the panel) on the panel, since the chord
A2.13
members are horizontal and thus have zero verti- since no external vertical load exists at joint
cal component. E. Similarly, by the same reasoning for ~H = 0,
load in DE = O. The load in the diagonal FD
(2) The truss verticals in general resist equals the value on the diagonal of the panel
the vertica~ component of the diagonals plus index triangle or 167 lb. It is tension by
any external loads applied to the end joints of observation since the shear in the panel to the
the vertical. right is up and the vertical component of the
diagonal FD must pull down for equilibrium.
(3) The load in the chord members is due
to the horizontal components of the diagonal Considering Joint F. ~H =- FG - F~ 0, =
members and in general equals the summation of which means that the horizontal component of the
these horizontal components. load in the diagonal DF equals the load in FG,
or is equal to the value of the horizontal side
To illustrate the simplicity of determining in the index triangle or - 133 lb. It is nega-
stresses in the members of a parallel chord tive because the horizontal component of DF
truss, consider the cantilever truss of Fig. pulls on Joint F and therefore FG must push
A2.29 with supporting reactions at points A and against the joint for equilibrium.
J.
Considering Joint D:-
~v =
= DFV + DG = O. But DFV 100 (vertical side
of index triangle)
••• DG =- 100
~H =
DE + DFH - DC =
0, but DE =
0 and DFH =
133 (from index triangle)
DC =
133
Considering Joint G:-
Fig. A2.29
~H=- GH + GF - GCH = O. But GF =-
133, and GCH
= 300 from index triangle in the second panel.
First, compute the length triangles in Hence GH =-
433 lb. Proceeding in this manner,
each panel of the truss as shown by the dashed we obtain the stress in all the members as shown
triangles in each panel. The other triangles in Fig. A2.29. All the equilibrium equations
in each panel are referred to as load or index can be solved mentally and with the calculations
triangles and their sides are directly pro- being done on the slide rule, all member loads
portional to the length triangles. can be written directly on the truss diagram.
The shear load in each panel is first writ- Observation of the results of Fig. A2.29
ten on the vertical side of each index triangle. show that the loads in the truss verticals equal
Thus, in panel EFGD, considering forces to the the values of the vertical sides of the index
right of a vertical section cut thru the panel, load triangle, and the loads in the truss di-
the shear is 100 lb., which is recorded on the agonals equal the values of the index triangle
vertical side of the index triangle. diagonal side and in general the loads in the
top and bottom horizontal truss members equal
For the second panel from the free end, the the summation of the values of the horizontal
shear is 100 + 150 = 250 and for the third panel sides of the index triangles.
100 + 150 + 150 =
400 lb., and in like manner
550 for fourth panel. The reactions at A and J are found when
the above general procedure reaches joints A
The loads in the diagonals as well as their and J. As a check on the work the reactions
horizontal components are directly proportional should be determined treating the truss as a
to the lengths of the diagonal and horizontal whole.
side of the length triangles. 'Thus the load in
diagonal member DF =
100 (50/30) =
167 and for Fig. A2.30 shows the solution for the
member CG = 250 (46.8/30) =
390. The hori- stresses in the members of a simply supported
zontal component of the load in DF =
100 (40/30) Pratt Truss, symmetrically loaded. Since all
= 133 and for CG = 250 (36/30) =
300. These panels have the same width and height, only one
values are shown on the index triangles for length triangle is drawn as shown. Due to
each truss panel as shown in Fig. A2.29. We symmetry, the index triangles are drawn for
start our analysis for the loads in the members panels to only one side of the truss center
of the truss by considering joint E first. line. First, the vertical shear in each panel
is written on the vertical side of each index
Using ~v =0 gives EF = 0 by observation, triangle. Due to the symmetry of the truss and
A2.14 EQUILIBRIUM OF FORCE SYSTEMS. TRUSS STRUCTURES.
loading, we know that one half of the external horizontal components of diagonal members =
loads at joints U3 and L 3 is supported at re- 312 + 187.5 =
499.5. Therefore, load in U~U2 =
action R~ and 1/2 at reaction R 2, or shear in - 499.5. Similarly at joint L 2, L2L3 = 312 +
187.5 =
499.5. At joint U2, the horizontal
Length
Fig. A2. 30 components of U~U2 and U2U3 =
499.5 + 62.5 562 =
~
25" Triangle
30" which must be balanced by - 562 in member U2U3 •
39" 50 50 50 50 50
The reaction R 2 equals the value on the
U -499.5 U2 -562 U3
vertical side of our index triangle in the end
panel, or 375. This should be checked using
the truss as a whole and taking moments about
R2 •
Wing Tip (End) Bow in all members of the lift and drag trusses will
be determined. A simplified air loading will be
"f4F~~~~--Plywood Tip Fairing assu~ed, as the purpose of this problem is to
Fig. A2. 33 hive the student practice in solving statically
...---Aileron determinate space truss structures .
Aileron Spar
Compression Rib
ASSUMED AIR LOADING:-
Plywood Leading (1) A constant spanwise lift load of 45
Edge Fairing Drag Wire
Ib/in from hinr;e to strut point and t11en taper-
Dragwire ing to 22.5 Ib/in at the wing tip.
Fitting Anti-Drag Wire
Aileron Rib (2) A forward uniform distributed drag
Aileron Hinge load of 6 Ib/in.
Drag Strut or
Leading Compression Tube
Edge The above airloads represent a high angle
Forming or Plain Rib of attack condition. In this condition a for-
Trailing Edge ward load can be placed on the drag truss as
illustrated in Fig. A2.36. Projecting the air
METAL FAIRING
-
Airstream
Drag
Fig. A2. 36
Center of
Pressure Rear
Beam
: Beam
Taken off 1 I D
At (2) I 4 1 '0. L
~ ~ S
Y (2)1 --L-Front
2 -"':1---37. 5 + 37.5+37.5 58.5 ----.j121--- Beam
__U1t :I:
:,--Tl-~,2)
~'S
I Q)
1 __[J(3)
~
~ Po<
....;
~
3 0 Dihedral
T
Wing Chord= 72"
I
1.1% =
22.52"
24.21"
Fig. A2. 37
Front Beam 5
F Front Lift
Truss
A6
R Rear
Lift
Truss
~
w= 30.26#~in.
(2)
114. 5"
-.-.
Taking moments about point (2) Member Sym. V D S L V/L D/L S/L
114.5R~ - 114.5 x 30.26 x 114.5/2 - 15.13 x Front Beam FB 5.99 o 114.34 114.50 .0523 o .9986
70.5 x 149.75 - 15.13 x 35.25 x 138 = O. Rear Beam RB 5.99 o 114.34 114.50 .0523 o .9986
-;J ~ r:
770
R4 = .4875 x 1295 = 631 lb. FB 1)
(1) I D~
S .+V FB
The next step in the solution is the F ~+S
solving for the axial loads in all the members. SF +D
We will use the method of joints and consider
the structure made up of three truss systems ZV = 3770 x .9986 - .0523 FB - .4501 SF =
0 - (1)
as illustrated at the top of the next column, ZS = - 3770x .0523 - .9986 FB - .8878 SF = 0 - (2)
namely, a front lift truss, a rear lift truss ZD =- .0854 SF + D~ = 0 - - - - - - - - - - (3)
and a drag truss. The beams are common to both
lift and drag trusses. Solving equations 1, 2 and 3, we obtain
Joint (3) (Equations of equilibrium) points (2) and (4). In the design of the beam
and fittings at this point, the effect of the
1838 (drag truss actual conditions of eccentricity should of
1
~
838 reaction on
RB course be considered.
7 R
(3)
V-S plane
(3)
RB
SR
" pin (3~)
~V = 1838 X .9986 - .0523 RB - .4486 SR = 0 -(4) Adding the two load systems of Figs. A2.38
~S =-1838 x .0523 - .9986 RB -.8930 SR 0 -(5) = and A2.39, the total drag truss loading is ob-
~D = D" + 0 = 0 - - - - - - - - - - - - - - -(6) tained as shown in Fig. A2.40. The resulting
member axial stresses are then solved for by the
Solving equations 4, 5 and 6, we obtain method of index stresses (Art. A2.9). The
RB =-
4189 lb. (compression) values are indicated on the truss diagram. It
SR =
4579 lb. (tension) is customary to make one of the fittings attach-
Da = 0 ing wing to fuselage incapable of transferring
drag reaction to fuselage, so that the entire
Fig. A2.38 shows the reactions of the lift drag reaction from wing panel on fuselage is
struts on the drag truss at joints (1) and (3) definitely confined to one point. In this ex-
as found above. ample point (2) has been assumed as point where
drag is resisted. Those drag wires which would
(
4)~
be in compression are assumed out of action.
Ga~~36~--------I+---36~
---1
39.5 37.5 58.5
231 225
(2) 8513+ 281.5 254
Fig. A2.38 798 ~~
Drag Truss Panel Point Loads Due to Air Drag
Load.
It was assumed that the air load components -10,313 8513l -413
in the drag direction were 6 Ib./in. of wing 798
acting forward.
The wing beams due to the distributed lift Lengths & Directional Components of Cabane Struts
air loads acting upon them, are also sUbjected
to bending loads in addition to the axial loads. Member Sym. V D S L V!L D/L S/L
The wing be~~s thus act as beam-columns. The Front Cabane CF 30 10 27 41.59 .721 .240 .648
subject of beam-column action is treated in Diagonal Cabane 30 30 27 50.17 .597 .597 .538
CD
another chapter of this book. Strut
Rear Cabane CR 29.5 6 27 40.42 .731 .1485 .668
If the wing is covered with metal skin Strut
instead of fabric, the drag truss can be omitted L =Vv 2 + D2 + S2
since the top and bottom skin act as webs of a
beam which has the front and rear beams as its
flange members. The wing is then considered as The air loads on the outer panel are taken
a box beam subjected to combined bending and identical to those in example problem 1. Like-
axial loading. wise the dihedral and direction of the lift
struts SF and SR have been made the same as in
Example Problem 11. 3-Section Externally Braced example problem 1. Therefore the analysis for
Wing. the loads in the outer panel drag and lift trus
trusses is identical to that in problem 1. The
Fig. A2.41 shows a high wing externally solution will be continued assuming the running
braced wing structure. The wing outer panel has lift load on center panel of 45 Ib./in. and a
been made identical to the wing panel of ex~~ple forward drag load of 6 Ib./in.
problem 1. This outer panel attached to the cen
ter panel by single pin fittings at points (2) Solution of Center Panel
and (4). Placing pins at these points make the
structure statically determinate, whereas if the Center Rear Beam
beams were made continuous through all 3 panels,
692# (630+62)=692#
the reactions of the lift and cabane struts on
the wing beams would be statically indeterminate w= 14. 74#/in.
since we would have a 3-span continuous beam
resting on settling supports due to strut de- (4) r-----t-::.,---------.,--:::-:-t--~
formation. The fitting pin at points (2) and 90
(4) can be made eccentric with the neutral axis R s = 1650# R s = 1650#
of the beams, hence very little is gained by Fig. A2. 42
making beams continuous for the purpose of de-
creasing the lateral beam bending moments. For Fig. A2.42 shows the lateral loads on the center
assembly, stowage and shipping it is convenient rear beam. The loads consist of the distributed
to build such a wing in 3 portions. If a air load and the vertical component of the for-
mUltiple bolt fitting is used as points (2) and ces exerted by outer panel on center panel at
(4) to obtain a continuous beam, not much is pin point (4). From Table A2.2 of example prob-
gained because the design reqUirements of the lem 1, this reSUltant V reaction equals 630 +
various governmental agencies specify that the 62 = 692 lb.
wing beams must also be analyzed on the as-
smnption that a mUltiple bolt fitting prOVides The vertical component of the cabane re-
only 50 percent of the full continUity. action at joint (8) equals one half the total
(l)
OJ) beam load due to symmetry of loading or 65 x
~.S
,0:: 14.74 + 692 =
1650 lb.
(8) :(4) --L
Solution of force system at Joint 8
36"
1650
1650
T
CRB
/~
CR S
f8
D~C
~D
ZV - 1650 - .731 CR = 0
whence
~
2535
(11)
CFB 7 Fig. A2.45
13lli-~~VLj~7
1 t • '228f 2m (2)1
loads.
ponents
Table A2.3 gives the V, D and S com-
of the fuselage reactions.
1908 t iR J R7 t 1908
26341t Fig. A2. 44 2634#
A2.20 EQUILIBRIUM OF FORCE SYSTEMS. TRUSS STRUCTURES.
Table A2.3
Point Kember Load V D S
~'O
~1
/t: z
'!>~ 504
The total side load on a vertical plane thru
centerline of airplane should equal the S com-
ponent of the applied loads. The applied side 0 '1::J
a:>
loads = - 394 lb. (see problem 1). The air load "'bil"
~
on center panel is vertical and thus has zero S
component. T e ri:
r
36"
) '"
From Table A2.3 for fuselage reactions til
have ZS =
16178. From Fig. A2.45 the load in .9
the front beam at ~ of airplane equals - 17308
and 568 for rear beam. The horizontal component
A B C D E \e It>
c>
A B C D E
of the diagonal drag strut at joints 11 equals T
216 x 45/57.6 = 169 lb. 9"
L;My = (6 + 66 + 72 + 72 + 36) 36 - 36 Oz =0
T
24"
whence Oz = 252 lb. acting down as assumed.
To find Oy take moments about z axis through
point (A).
12"
L;Mz =0 + 36 Oy = 0, Oy =0
-"- To find Ay take moments about x axis through
E
--+-12"-1 Fig.A2.48 point N. The moment of the air loads was pre-
viously calculated as - 31752, hence,
simple beams resting at points 2, 3, 4, 5, etc. L;Mx =- 31752 + 9 Ay = 0, whence, Ay =
The distance between 2-3 is 8 inches. The total 3528 lb.
load on this distance is 8 x 9 = 72 lb. One
half or 36 lb. goes to point (2) and the other To find Ny take L;F y =0
half to point (3). The 36 lb. at (2) is then
replaced by an equivalent force system at E and L;Fy =3528 - Ny = 0, hence Ny = 3528 lb.
8 or (36)/3 =
12 lb. to Sand (36)(2/3) =
24 to To find Nz take L;F z 0 =
E. The distance between points (3) and (4) is
8 inches and the load is 8 x 9 =
72 lb. One ZF z = - 252 + 756 - Nz = 0, hence Nz =
half of this or 36 goes to point (3) and this 504 lb.
added to the previous 36 gives 72 lb. at (3).
The load of 72 is then replaced by an equivalent The reactions are all recorded on Fig.
force system at Sand D, or (72)/3 =
24 lb. to A2.46.
8 and (72)(2/3) =
48 to D. The final load at S
=
is therefore 24 + 12 36 lb. as shown in Fig. Solution of Truss Member Loads
A 2.46. Due to symmetry of the triangle CRD,
one half of the total load on the distance CD For simplicity, the load system on the
goes to points (4) and (5) or (24 x 9)/2 108 = structure will be considered separately as two
lb. The distribution to D is therefore (108) load systems. One system will include only
(2/3) = 72 and (108)/3 =
36 to R. Adding 72 those loads acting along the line AE and the
to the previous load of 48 at D gives a total second load system will be remaining loads
=
load at D 120 lb. as shown in Fig. A2.46. which act along line 08. Since no bending
The 108 lb. at point (5) also gives (108)/3 = moment can be resisted at joint 0, the external
36 to R or a total of 72 lb. at R. The student load along spar AE will be reacted at A and N
should check the distribution to other joints entirely or'in other words, the spare alone
as shown in Fig. A2.46. resists the loads on line AE.
To check the equivalence of the derived Fig. A2.49 shows a diagram of this spar
joint load system with the original air load with its joint external loading. The axial
system, the magnitude and moments of each loads produced by this loading are written on
system must be the same. Adding up the total the truss members. (The student should check
joint loads as shown in Fig. A2.46 gives a total these member loads.)
of 756 lb. which checks the original air load.
The moment of the total air load about an x
axis at left end of structure equals 756 x 42 =
31752 in. lb. The moment of the joint load
system in Fig. A2.46 equals (66 x 12) + (72 x
36) + (72 x 60) + (56 x 84) + 144 (24 + 48) +
(120 x 72) + (24 x 84) =
31752 in. lb. or a 224 224 32
check. The moment of the total air load about
line AE equals 756 x 12 =9072 in. lb. The
Fig. A2.49
moment of the distributed joint loads equals TRIANGULAR TRUSS SYSTEM
(6 + 66 + 72 + 72 + 36)36 =
9072 or a check.
The load system along the trailing edge OS
Calculation of Reactions causes stresses in both the spar truss and the
diagonal truss system. The support fitting at
The structure is supported by single pin point 0 provides a reaction in the Z direction
fittings at points A, Nand 0, with pin axes but no reacting moment about the x axis. Since
parallel to x axis. It will be assumed that the loads on the trailing edge lie on a y axis
the fitting at N takes off the spar load in through 0, it is obvious that all these loads
Z direction. Fig. A2.46 shows the reactions flow to point O. Since the bending strength of
0y, 0z, Ay , Ny, Nz • To find Oz take moments the trailing edge member is negligible, the
about y axis along spar AEFN.
A2 22 EQUILIBRIUM OF FORCE SYSTEMS TRUSS STRUCTURES
load of 36 lb. at Joint S in order to be trans- reaction on the spar but does produce a couple
ferred to point 0 through the diagonal truss force on the spar in the Y direction which pro-
system must follow the path SDRCQBPAO. In like duces compression in the top chord of the spar
manner the load of 72 at R to reach 0 must take truss and tension in the bottom chord.
the path RCQBPAO, etc.
Consider Joint R
Calculation of Loads in Diagonal Truss Members:-
The load to be transferred to truss RCJR
Member z y x L z/L y/L x/L is equal to the 72 lb. at R plus the 36 lb. at
S which comes to joint R from truss DRG.
All Diagonal 4.5 12 36 38.2 .118 .314 .943
Truss Members Hence load in RC = (72 + 36)0.5 x (1/.118)
AO, NO 4.5 0 36 37.5 .120 o .960 457 lb.
Whence RJ =457, CQ =457 and JQ =- 457
Consider Joint S
Joint Q
The triangular truss SEF cannot assist in Load to be transferred to truss QBL =72 +
transferring any portion of the 36 lb. load at
S because the reaction of this truss at EF
72 + 36 =
180 lb.
would put torsion on the spar and the spar has Hence load in QB = (180 x 0.5)(1/.118) =
no appreciable torsional resistance. - 762.
TY-#:E159 T 159
T
Y D
'E As pointed out previously, the diagonal
torsion truss produces a couple reaction on the
z spar in the y direction. The magnitude of the
force of this couple equals the y component of
Let Ty and Tz be reactions of diagonal the load in the diagonal truss members meeting
truss system on spar truss at Joint D. at a Spar joint. Let Ty equal this reaction
load on the spar.
L:F x =- 159 x .943 + .943 DR = 0, hence DR =
159 lb. At Joint C:-
L:Fz =- 159 x .118 + 159 x .118 - Tz =0 Ty =- (457 + 457).314 =- 287 lb.
whence Tz =
0, which means the diagonal Likewise at Joint J, Ty =287
truss produces no Z reaction or shear load on
spar truss at D. At Joint B:-
Fig. A2. 51
McDonnell Aircraft
(Military Airplane)
Douglas DC-8
Jet Airliner
Piper-Apache
Piper Tri-Pacer
Navy F4-J
North American Aviation Co.
(f
/
F
6500
3004
t....5690
G
1~109
reaction G is replaced by its components Gy and
GD. For a side load on the landing gear, the
\,n .t 433
H
reaction in the S direction is taken off at
point F by a specially designed unit. Fig.A2.57
A2.27
~
TE the structure as a whole by taking moments about
t D and V axes through point A.
f'V tri 11
ESTE =1;~1lrc ~MA{D) =- 10063 x 14 + 6500 x 8 + 11109 x 8
=- 140882 + 52000 + 88882 = O(check)
24"
f,. ~~~
I r~ ~ = 5690 x 8 - 433 x 8 - 3004 x 14
+
I'~ ~S
I •
,
BIS=-!.LBI
24.6
= 45520 - 3464 - 42056 = 0 (check)
The next step in the solution will be the
calculation of the forces on the oleo strut
unit. Fig. A2.58 shows a free body of the oleo-
strut-axle unit. The brace members BI and CG
~8l=- are two force members due to the pin at each
I 14672# end, and thus magnitude is the only unknown re-
Fig.A2.58 action characteristic at points Band C. The
fitting at point E between the oleo strut and
the top cross member FH is designed in such a
manner as to resist torsional moments about the
oleo strut axis and to provide D, V and S force
reactions but no moment reactions about D and S
axes. The unknowns are therefore BI, CG, ES '
EV' ED and TE or a total of 6 and therefore
statically determinate. The torsional moment
TE is represented in Fig. A2.58 by a vector
with a double arrow. The vector direction
represents the moment axis and the sense of
rotation of the moment is given by the right
hand rule, namely, with the thumb of the right
hand pointing in the same direction as the
arrows, the curled fingers give the sense of
rotation.
Fig. A2. 59
To find the resisting torsional moment TE
take moments about V axis through E.
To find FV' take moments about a D axis
through point H. ~ME{V) =- 3119 x 8 + TE = 0, hence TE
= 24952 in. lb.
~MH(D) = 16 GV + 14672 x 8 - 22 FV = 0
=16 x 6500 + 14672 x 8 - 22 FV = 0 To find CG take moments about Saxis
through E.
Whence FV = 10063 lb. with sense as as-
sumed. ~ME(S) = 3119 x 50 - (24/31.8) CG x 3 -
24 (21/31.8) CG 0 =
To find HD, take moments about V axis Whence, CG = 8610 lb.
through F.
~MF(V) - - 6 GD - 22 HD + 3119 x 14 0 = This checks the value previously obtained
- - 6 x 5690 - 22 HD + 3119 x 14 when the reaction at G was found to be 8610.
=0 The D and V components of CG thus equal,
Whence, HD = 433 lb. CGD = 8610 (21/31.8) = 5690 lb.
~D =- FDHD + GD - 3119
+ 0 = To find load in brace strut BI, take moments
=- FD433 + 5690 - 3119 = 0
+ about D axis through point E.
Whence, FD 3004 lb. = ~ME{D) =- 14672 x 8 + 3 (BI) 22/24.6 +
24 (BI) 11/24.6 0 =
To find HV take ~V =0 Whence, BI = 8775 lb.
~V =- FV + GV - HV + 14672 =
0 and thUS, BIV = (8775)(22/24.6) = 7840 lb.
=- 10063 + 6500 - HV + 14672 =0 BIS = (8775)(11/24.6) = 3920 lb.
Whence, HV =11109 lb.
To find Es take =0~S
Fig. A2.57 summarizes the reactions as found.
The results will be checked for equilibrium of ~s = Es - 3920 = 0, hence ES = 3920
A2.28 EQUILIBRIUM OF FORCE SYSTEMS. TRUSS STRUCTURES.
To find ED talee ZD =0
ZD =5690 - 3119 - ED =0, hence ED =2571
To find EV take ZV 0 =
Zy =-Ey + 14672 - 7840 + 6500 = 0, hence
EV = 13332 lb.
Fig. A2.59 shows a free body of the top
member FH. The unknowns are Fy, FD, FS' Hy and
HD. The loads or reactions as found from the
Lower
analysis of the oleo strut unit are also re- Cylinder
corded on the figure. The equations of
equilibrium for this free body are:- Fig.A2.60
Torque
zs = 0 =- 3920 + 3920 + FS = 0, or FS =0 Link
ZMF(D) =22 Hy -
3920 x 2 - 7840 x 20 -
13332x6=0
=
Whence, Hy 11110 lb. This check value
was obtained previously, and therefore is a
check on our work.
ZMF(y) = 24952- 2571 x 6 - 22 HD =0 The torque to be transferred in our prob-
lem is 24952 in. lb.
whence, HD = 433 lb.
Zy =- Fy + 13332 + 7840 - 11110 =0 The reaction R.. between the two units of
the torque link at point (2), see Fig. A2.60,
whence, Fy = 10063 thus equals 24952/9 = 2773 lb.
ZD - - FD + 2571 + 433 = 0 The reactions R a at the base of the link at
whence, FD =3004 lb. point (3) = 2773 x 8.5/2.75 = 8560 lb. With
these reactions known, the strength design of
Thus working through the free bodies of the link units and the connections could be
the oleo strut and the top member FH, we come made.
out with same reactions at F and H as obtained
when finding these reactions by equilibrium Example Problem 14
equation for the entire landing gear.
The landing gear as illustrated in Fig.
The strength design of the oleo strut unit A2.61 is representative of a main landing gear
and the top member FH could now be carried out which could be attached to the under side of a
because with all loads and reactions on each wing and retract forward and upward about line
member known, axial, bending and torsional AB into a space provided by the lower portion
stresses could now be found. of the power plant nacelle structure. The oleo
strut OE has a sliding attachment at E, which
The loads on the brace struts CG and BI prevents any vertical load to be taken by
are axial, namely, 8610 lb. tension and 8775 member AB at E. However, the fitting at E does
lb. compression respectively, and thus need no transfer shear and torque reactions between the
further calculation to obtain design stresses. oleo strut and member AB. The brace struts
GD, FD and CD are pinned at each end and will
TORQUE LINK be assumed as 2 force members.
The oleo strut consists of two telescoping An airplane level landing condition with
tubes and some means must be provided to trans- unsymmetrical wheel loading has been assumed as
mit torsional moment between the two tubes and shown in Fig. A2.61.
still permit the lower cylinder to move upward
into the upper cylinder. The most common way SOLUTION
of providing this torque transfer is to use a
double-cantilever-nut cracker type of structure. The gear is attached to supporting struc-
Fig. A2.60 illustrates how such a torque length ture at points A, Band C. The reactions at
could be applied to the .oleo strut in our these points will be calcUlated first, treating
problem. the entire gear as a free body. Fig. A2.62
A2.29
__
24"
AlB x 19 + 38 By 0 =
r'.~ rt~~~ whence, By =
78070 lb.
To find Ay, take ~y =0
28 " Brace
("'"
=-
1st'~D •
Fig.A2.61
~y
=0
78070 + 60000 + 40000 + 66666 - YA
;TE
r- 17"+ 17"
;TE
-1
~G___ E--ES~-
E
Fig.A2.62
ED-
<D
<D I DG
,\GV
/
DFV-<y4-
i
571~~ 28" t l/ 28"
shows a space diagram with loads and reactions.
The reactions at A, Band C have been replaced
_0
At
5-
~
DGS- ' I3--r::; DFS
I 'v 1-i.61t
l
by their Y and D components.
41/4-1 1-41/4
To find reaction Cy take moments about an VL
S axis through points AB.
L D
36"
s Fig. A2. 63
~~B
Whence Cy
=-
(15000 + 10000) 64 + 24 Cy = 0
=
66666 lb. With sense as
"0" o~ 25000 ~'O"
assumed in Fig. A2.62. 100,000 lb. 200,00~ --100,000 lb.
t 50000"lb. t 50,000"lb.
~\'%.9~
The reaction at C must have
a line of action along the line
CD since member CD is pinned at 28LS. the axle centerlines have been transferred to
each end, thus the drag compon- 24 D point (0). Thus the total Y load at (0) equals
ent and the load in the strut 60000 + 40000 =
100000 and the total D load
CD follow as a matter of geometry. Hence, equals 15000 + 10000 =
25000. The moment of
CD = 66666 (24/28) =
57142 lb. these forces about Y and D axes through (0) are
CD = 66666 (36.93/28) =
87900 lb. tension =
I"lO(y) (15000 - 10000) 10 50000 in. lb. and =
=
I"lO(D) (60000 - 40000) 10 200000 in.lb. =
To find By take moments about a drag axis These moments are indicated in Fig. A2.63 by
through point (A). the vectors with double arrows. The sense of
the moment is determined by the right hand thumb
A2.30 EQUILIBRIUM OF FORCE SYSTEMS. TRUSS STRUCTURES.
*
ZMO(S) =32143 x 64 - 57142 x 36 = O(check) (1)
Pm
~
10
-10
(2) ,
A ' (3)
10~
REACTIONS ON TOP MEMBER AB
~ + + + ~
(9)~ ~(10)
A
+ + + B ~B
A
+0
.J+V Front View
Side View
Fig. 20
20"
..l
(6) Fig. 21 shows the wing structure of an ex-
1-16'4\ (12) ternally braced monoplane. Determine the
axial loads in all members of the lift and
drag trusses for the following loads.
(14) Front beam lift load =
30 lb./in. (upward)
Rear beam lift load =
24 lb./in. (upward)
100 Wing drag load =
8 lb./in. acting aft
PLAN VIEW
(3) Find the axial loads in the members of the Wing Drag Truss Anti-Drag Wires
trussed structures shown in Figs. 16 to 18. Drag Wires
C' B'
.10 ;1.0 t10
-*- T T (16)
15"~'
T 30"
30"
10 -.t..
1. 24 +24 4024"4 36" -j!!.- 45"--l
P.OOO
~"~-:k
.L
--l..
2000
t
I
- 00
10"
(18)
3t".1-Ct
1 0
CR
DO'
I- I 1 30,0-01 '
I I I
!--30"-+15"+15'.....j
~tt-
<rl A,A'
Fuselage
Fig. 21
(4) Determine the axial loads in the members
of the structure in Fig. 19. The members (7)
are pinned to supports at A, Band C.
I--- 5 Bays @ 3&' ---I
--l rl C 0
j.-15'..j.lo.:r- 20"
I
T
30"
·~-DragTrUssLS
I
I ..i. AI t t f 1ft t t t t t t tt 25 lb/in.
I
I
~2 /.in,
• 5 lb/in.
. ...JI"5"
.. #/in.
I
Pin Reactions I BC B
I 1
at A, B, C I 60" Fuse- I
1. lage I
Fig. 19 t100# E
I
~~
100# Fig. 22
10"
(5) Fig. 20 shows a tri-pod frame for hoisting Fig. 22 shows a braced monoplane wing. For the
a propeller for assembly on engine. Fipd given air loading, find axial loads in 11ft and
the loads in the frame for a load of 1000 drag truss members. The drag reaction on drag
lb. on hoist. truss is taken off at point A.
A2.32 EQUILIBRIUM OF FORCE SYSTEMS. TRUSS STRUCTURES.
(8) (9)
The fitting at In the
points A and B for landing gear
~_ _A B
E
C the landing gear of Fig. 24,
F the brace
structure in Fig. 23
provides resistance members BC
to V, D and S re- and BF are
24" two force
-1.3000
actions and moments N'
members. The
lb. about D and V axes.
Find the reactions at
30" L D
fitting at E
Wheel~_= W~:'j;~'CD
Fig. 23
10000 lb. 10000 A and B and the load provides re-
in member CD for sistance to
given wheel loading.
, 8"
wheel 2
V, D and S
reactions but
15000 10000 Fig. 24 only moment
resistance
about V axis.
Find reactions at E and loads in members BF and
BC under given wheel loading.
y TABLE I
I Area A
Section Properties of Areas
d ---l-- x --<i
I 'j'dA Rectangle Area = bd
t c . g. Fig. A3.2
r-b~
Y=d
:2
bd 3 /12
I
l~-l bd 3
2-~-2
y
3"
The first term,~x2dA, represents the mo-
ment of inertia of the body about its centroidal fJ.-J. .289 d
axis y-y and will be given the symbol I. The t------------------------I
second term is zero because~xdA is zero since Area bh
yy' is the centroidal axis of the body. The 1f
last term, d 2.;rdA = Ad 2 or, area of body times y h/3
the square of the distance between axes yy and
y~y~.
Thus in general, 12 _ 2
I = I + Ad 2 1
This expression states that the amount of
inertia of an area With respect to any axis in bh 3
the plane of the area is equal to the moment of T
inertia of the area with respect to a parallel
centroidal axis plus the product of the area and f~-~ .236 h
the square of the distance between the two axes. ~------------------~--------------------------~
8
I 2 Area bh
Parallel Axis Theorem For Masses. If instead of T~riange 1f
area the mass of the body is considered, the
parallel axis can be written:
1 - -+-2 - ---l-"fI x 2/3 h
b hb 3
I = I + Md 2 , where M refers to the mass of x I~_~ 48
the body. n ~
.204 h
A3.7a Mass Moments of Inertia The product of
the mass of a particle and the square of its
distance from a line or plane is referred to as
the moment of inertia of the mass of the parti-
Trapezoid
r- b1-j Area = d(b + b ~)
Y = d(2b + b~)
2
r"" 1;""
~ y~ ~r:ad~2tld + 2a)
A,A'"
xiW/'+b[1:
x
PI \
L I~
il\ _I
,)v:'
_ I ? ~
dl X
d
'x 0 bd" - aid - 2t)"
12
I y = d(b + a)" - 2a"e - 6ab 2e
tan 20 " Idt-t") Ib"-btl
Ix _ I y
\
I---a--l
-;-= I,~\Y-i
,
)1
Area = .215 a
x = .223 a
2
Area - nr 2
Circle
2 -- I = nr 4
- =2
'-E-:~:
~-~
4 y y~
4 '7
1 2- 2 = 5 n r
4: TABLE 2
e~ -~ = r Properties of solids
2
Vol. - nr~ (r = radius)
Ring Area n (r o 2 ri 2) Solid Circ. Cyl.
1~-
M = W (Total wt.)
I~ _~ = n (r o4 - ri4) g
4:
\~. 1 I~ _~ = Mr 2
ex
A
= .264 r \.
q=outside
For thin hollow eire. eyl
I~ ~ = Mr 2 •
rea _ n (R 2 _ r2) radius
2 Re~ct.
Prism \.
y =i.. (R 2+ Rr + r) ,approx. y-=2r Vol. - abL, M = W
Semi-circular Ring
3n R+r n ' -~ 2 g2
I
x=.1098(R4-r4)-.283R2~ R + r
(R-r)
\.
I:
~
,.....--y 3
12 I
j 0 ~
..ja~ I
~-~= M (a + b )/12
1 2- 2 = ML2/12
~~3 ~
2
-r----------==~~~~------J
I (
x approx)=.3t(r +R)" when t I" " - ML /3
2
Solid Sphere Vol. = (4 n r")/3
is small r
F------;~~-:---~
Ellipse
Sphere I
about axis
2M r 2/5
=
0
Area - nab
•
I = n- a"b
Ix Thin Hollow Sphere Vol. =4 n
JI 4-
~ :3
(r~3 -r2 " )
x- xE'x = a = radius of sphere
~
rl
2 I about dia. -
2Mr~ 2
3
A3 4 CENTROIDS , C ENTER OF GRAVITY MOMENTS OF INERTIA
x~.x
g of body. TABLE 4
I xx =mn 2Ra2 [If + (5a 2/4)) Centroids of Trapizoidal Areas
y
TABLE 3
2
2 2
I yy = ~ mn Ra (4R 2 + 3a )
2
LLET
b
fl- y +xl--
I-l-
,e, = 2 n rt
Circular Arc Area 1.01 0.4992 0.5008
1. 02 0.4984 0.5016
1.03 0.4976 0.5024
I~_~ = n r t
3
1.04 0.4968 0.5032
1.05 0.4960 0.5040
I polar = 2 n r 3 t 1.06 0.4952 0.5048
1.07 0.4944 0.5056
1.08 0.4936 0.5064
ex = .707r 1. 09
1.10
0.4928
0.4920
0.5072
0.5080
E' polar = r 1.11
1.12
0.4912
0.4905
0.5088
0.5095
1.13 0.4898 0.5102
Semi-circle Arc A = n rt 1.14 0.4890 0.5110
- 1.15 0.4883 0.5117
Y = .6366 r 0.5123
~
1.16 0.4877
1.17 0.4870 0.5130
I1.-~ = n r t
3 0.5138
1.18 0.4862
-2- 1.19 0.4855 0.5145
1 ---±-- - 1 1. 20
1. 22
0.4849
0.4835
0.5151
0.5165
1"-2 = .2978 r 3t 1. 24 0.4822 0.5178
1. 26 0.4809 0.5191
1 28 0.4795 0.5205
Quarter-circular Arc Area =n rt 1. 30 0.4782 0.5218
-2- 1. 32 0.4770 0.5230
~'
1. 34 0.4758 0.5242
- 1. 36 0.4746 0.5254
Y = .6366 r 1. 38 0.4733 0.5267
f4--~:
2 1. 40 0.4721 0.5279
I~_~ = n-4-
r t 3
1. 45 0.4693 0.5307
0.5333
1. 50 0.4667
1 1.55 0.4641 0.5359
2 1 1.60 0.4616 0.5384
12 - 2 = .149 r 3 t
,
1. 65 0.4592 0.5408
1. 70 0.4568 0.5432
1. 75 0.4545 0.5455
1. 80 0.4523 0.5477
CIRCULAR ARC 1. 85 0.4502 0.5498
r
1. 90 0.4482 0.5518
NAy 0.5538
Y 1. 95 0.4462
2.00 0.4443 0.5557
2.10 0.4409 0.5591
2.20 0.4375 0.5625
x
2.30 0.4343 0.5657
~
2.40 0.4312 0.5688
2.50 0.4284 0.5716
2 60 0.4259 0.5741
: ·NA 2.70 0.4233 0.5767
Xf - ~
2.80 0.4209 0.5791
I r{MEAN)t y 2.90
3.00
0.4188
0.4168
0.5812
0.5832
-x I I i X 3.20
3 40
0.4128
0.4090
0.5872
0.5910
Y 3.60 0.4060 0.5940
3.80 0.4030 0.5970
Area = art a in Radians 4.00
4.20
0.4000
0.3975
0.6000
0.6025
0.6050
4.40 0.3950
r sin a ,
X= a (Myy =A x = r 2
t sin a) 4.60
4.80
0.3928
0.3908
0.6072
u.t;U1l2
5.00 0.3889 0.6111
5.50 0.3848 0.6152
r3t (a + Si~ 2a) 6.00 0.3810 0.6190
IyY =~ 6 .. 50 o 3778 0.6222
7.00 0.3750 0.6250
7.50 0.3725 0.6275
t (a2 + sin42a _ sina a)
2
INA Y =r 3 8.00
9.00
0.3702
0.3668
0.6298
0.6332
10.00 0.3636 0.6364
y = r (I-COS
a
a) , Mxx = Ay- -- rat (I-cos a)
r3 t
I xx =2 (a _ sin2 2a)
A3.5
Table 4
Item Horizontal Vertical
I~
Weight Arm: x lIoment Arm. y lIoment
No. Name 11'# : Wx : l'y
where Ix, I y , and I z are generally referred to Example Problem 2. Determine the moment of iner-
as the rolling, pitching and yawing moments of tia about the horizontal centroidal axis for the
inertia of the airplane. area shown in Fig. A3.4
w = weight of the items in the airplane Solution. We first find the moment of inertia
x, y and z equal the distances from the about a horizontal reference axis. In this so-
axes thru the center of gravity of the airplane lution, this arbitrary axis has been taken as
and the weights w. The last term in each equa- axis x'x' thru the base as shown. Having this
tion is the summation of the moments of inertia moment of inertia, a transfer to the centroidal
of the various items about their own X, y and Z axis can be made. Table 5 gives the detailed
centroidal axes. calculations for the moment of inertia about
If w is expressed in pounds and the distan- axis x'x'. For simplicity, the cross-section
has been divided into the five parts, namely, A,
ces in inches, the moment of inertia is express- B,e,D, and E.
ed in units of pound-inches squared, which can lex is moment of inertia about centroidal
be converted into slug feet squared by multiply- x axis of the particular part being considered.
ing by 1/32.16 x 144. Distance from axis x'x' to centroidal horizontal
Example Problem 1. Determine the gross weight axis = y = ZAv = 17.97 = 2.91"
center of gravity of the airplane shown in Fig. ZA 6.182
A3.3. The airplane weight has been broken down By parallel axis theorem, we transfer the
into the 10 items or weight groups, With their
individual e.g. locations denoted by the symbol moment of inertia from axis x'x' to centroidal
+. axis xx.
I xx = Ix' x' - Ay 2= 79.47 - 6.18 X 2.91 2 =
Solution. The airplane center of graVity will be 27.2 in·
located with respect to two rectangular axes. In
this example, a vertical axis thru the center-
line of the propeller will be selected as a ref-
Radius of Gyration, Pxx = xx =V27 .2 =2.1" Vl
erence axis for horizontal distances, and the A 6.18
thrust line as a reference axis for vertical dis-
tances. The general expressions to be solved ®
are:-
x- = Zwx = distance to airplane e.g. from
ZW ref. axis S-B
y = ZWy = distance to airplane e.g. from
1iW ref. axis X-X
Table 4 gives the necessary calculations,
whenee
x 417180 = 133.3" aft of t propeller Fig. A3.4 Fig. A3.5
3150 Exampre Problem #3. Determine the moment of in-
y = 5480 -1.74" (below thrust line) ertia of the stringer cross seotion shown in
3150 Fig. A3.5 about the horizontal centroidal axis.
Solution. A horizontal reference axis x'x' is
assumed as shown. The moment of inertia is
Thr~tv--7'''=::;''.. _
first calculated about this axis and then
line transferred to the centroidal axis xx. See
Table 6.
Fig. A3.3
A3.6 CENTROIDS, CENTER OF GRAVITY, MOMl!:NTS OF INERTIA
E 1.50 0.375 0.56 0.21 1/12 x 2 x .75 3.0.07 0.28 Problem #4. Determine the moment of inertia of
the flywheel in Fig. A3.5a about axis of rotation.
Sum6.182 17.97 79.47 in 4
The material is aluminum alloy casting (weight =
.1 lb. per cu. inch.)
Table 6
WfJJ
Sec.AA
Portion Area y Ay Ay 2 I cx lxIx' • I cx + Ay 2
necessary, which is available in the weight and axes, the product of inertia of the weight about
balance estimate of the airplane. the reference axes is necessary. Column (14)
Table 6a shows the complete calculation of gives the values about the reference axes. To
the moments of inertia of an airplaneo This transfer the product of inertia to the cog. axes
of the airplane, we make use of the parallel
table is reproduced from N.AoO.A. Technical note
#575, "Estimation of moments of inertia of air- axis theorem. Thus
planes from Design Data." zwxz c . g . =ZWXZ(Ref. axes) -Zwxz =48,857,589-
Explanation of Table 5325.3x1l5.9x77.8=839,253 lb. in.2
Fig. A3.5b shows the reference of planes To reduce all values to slug ft~ mUltiply 1
and axes which were selected. After the moments 32.17
1
of inertia have been determined relative to x-
these axes the values &bout parallel axes 144
through the center of gravity of the airplane
are found by use of the parallel axis theorem. Hence Ix = 3061, I y = 6680, I z = 9096, I xz = 181
Having the inertia properties about the co-
Column (1) of Table 6a gives breakdown of air- ordinate c.g. axes, the moments of inertia bout
plane units or items.
the principal axes are determined in a manner as
Column (2) gives the weight of each itemo explained for areas. (See A3.13).
Columns (3), (4) and (5) give the distance of
the c.g. of the items from the references planes The angle 0 between the X and Z axes and the
or axes. principal axes is given by,
Columns (6) and (7) give the first moments of tan 2 0 = 2I x z = 2 x 181 =.05998 hence lO =
the item weights about the Y' and X' reference I z - Ix 9096-3061
axes.
1 0 43"
Columns (8), (9) and (10) give the moment of
inertia of the item weights about the reference
axes.
Columns (11), (12) and (13) give the moments of
inertia of each item about its own centroidal
axis parallel to the reference axes. Such items
as the fuselage skeleton, wing panels and engine
, \
\
have relatively large values for their centroid- '- \
al moments of inertia. \, \
The last values in Columns (3) and (5) give "g '<II
\
\
\
o
<II
Center sect10n 108.8 lOa 57 11,098 6,a02 1,131,955 353,491 261,229 261,229 632,563
nose assel!lbly
Center ssction a04.6 lal 57 24,757 11,66a a,995,549 664,745 491,245 491,245 1,411,126
beam, etc.
Center section 84.2 148 55 12,462 4,631 1,844,317 254,705 202,164 33,680 235,844 685,388
ribs, etc.
Flap 2a.0 180 53 3,960 1,166 712,800 61,798 48,598 48,598 209,880
Outer panel nose 104.6 105 166 65 10,983 6,799 1,153,215 2,545,546 441,935 184,514 184,514 713,895
Outer panel beam 155.5 120 156 65 18,672 10,114 2,240,640 3,786,682 657,410 274,478 274,478 1,213,680
Outer panel ribs 89.8 139 156 64 12,482 5,747 1,735,026 2,185,373 367,821 158,407 17,601 176,008 798,861
Ailerons 31.4 172 156 62 5,401 1,947 928,938 764,150 120,702 55,390 55,390 334,850
Horizontal tail 87.1 367 96.7 31,966 8,423 11,731,412 814,463 176,378 176,378 3,091,083
Vertical tail 31.4 352 - 125 11,053 3,925 3,890,586 490,625 110,200 10,174 31 1,381,600
Fueelage skele- 314.0 176 81 55,264 25,434 9,726,464 2,060,154 69,394 1,576,594 1,570,000 4,476,384
ton
Engine mount 40.5 60 80 2,430 3,240 145,800 259,200 5,184 5,184 5,184 194,400
Turtleback 48.5 254 80 12,319 3,880 3,129,026 310,400 15,181 57,861 56,648 985,520
( fairing)
Firewall 11.0 70 80 770 880 53,900 70,400 2,200 1,100 1,100 61,600
Steps 2.0 170 20 70 340 140 57,800 800 9,800 23,800
~.A.C.A. cowling 70.0 50 80 3,500 5,600 175,000 448,000 16,940 15,470 15,470 280,000
Cabin and 66.5 146 - 108 9,709 7,182 1,417,514 775,656 2,394 106,400 108,794 1,048,572
Windshield
Foot troughs 2.0 77 5 68 154 136 11,858 50 ·9,248 10,472
Floor, rear 9.5 al0 66 1,995 6a7 418,950 41,382 608 1,368 1,976 131,670
Wing fillets 18.5 142 20 58 2,627 1,073 373,034 7,400 62,234 18,944 18,944 152,366
Bottom cowling 27.0 140 11 75 3,780 2,025 529,200 3,267 151,875 24,300 24,300 283,500
and side frames
Arresting door 1.3 284 63 369 82 104,853 5,160 5 5 23,260
Tail-woeel pan, 4.0 365 84 1,460 336 532,900 26,224 100 100 122,640
etc.
Side doors 17.0 143 IS 82 2,431 1,394 347,633 5,508 114,308 1,088 28,288 27,200 199,342
Baggage door 1.8 165 63 297 113 49,005 7,144 720 720 18,711
Fabric and dope 13.0 254 80 3,302 1,040 838,708 83,200 4,394 15,509 15,509 264,160
Tail cone 7.5 385 91 2,888 683 1,111,688 62,108 120 120 262,763
Cowling, sta- 12.0 110 95 1,320 1,140 145,200 108,300 1,200 1,200 125,400
tione 1-2
Chassis (re- 232.4 115 54 51 26,726 11,852 3,073,490 677,678 604,472 23,240 23,240 1,363,026
tracted)
Retracting mech- 28.6 110 25 67 3,146 1,916 346,080 17,875 128,385 210,782
anism
Wheels, etc. 91.0 141 54 56 12,831 5,096 1,809,171 265,356 285,376 718,538
Tail wheel 26.0 360 74 9,360 1,924 3,369,600 142,376 26 26 692,640
The principal moments of inertia are given by will be given the symbol I xy , hence
following equation.
2 2 I xy = I xydA - - - - - - - - - - -(1)
I xp = I:z cos 0 + I z sin 0 - I xz sin 2 0. (see The unit, like that of moment of inertia, is ex-
Art. A3.11) pressed as inches or feet to the 4th power.
I yp = I y Since x and y may be either positive or negative,
the term I xy may be zero or either positive or
I z = Ix sin2 0 + I z cos 2 0 + I xz sin 2 0 negative.
substituting Product of Inertia of a Solid. The product of
I xp = 3061 x (0.9996)2 + 9096 x (0.0300)2 -181x inertia of a solid is the sum of the products
.0599 = 3056 obtained by multiplying the weight of each small
portion in which it may be assumed to be divided
I zp =3061x (0.0300)2 + 9096 x (0.9996)2 +181x by the product of its distances from two of the
.0599 = 9102 three coordinate planes through a given point.
I yp = 6680 Thus With respect to planes X and Y
I xy = I xy dW
A3.7b Problems
I xz = I xz dW
I yz = I yz dW
A3.9 Product of Inertia for Axes of Symmetry.
T:-
• J...
If an area is symmetrical about two rec-
tangular axes, the product of inertia about these
'~
1
axes is zero. This follows from the fact that
J..R symmetrical axes are centroidal x and y axes.
1 10 £.4-
8 1"
If an area is symmetrical about only one of
~ k---- It ---+l T ~ two rectangular axes, the product of inertia,
IXYdA, is zero because for each product xydA for
Fig. A3.6 Fig. A3. 7
an element on one side of the axis of symmetry,
(1) Determine the moment of inertia about there is an equal product of opposite sign for
the horizontal centroidal axis for the beam
section shown in Fig. A3.6. the corresponding element dA on the opposite side
of the axis, thus making the expression lydA
(2) For the section as shown in Fig. A3.7 equal to zero.
calculate the moment of inertia about the cen-
troidal Z and X axes. A3.l0 Parallel Axis Theorem
In Fig. A3.11
I X1Y1 =jx y dA=j(xcos0+ysin0)(ycos0-
x siJ 0)dA
= (cos" 0 - s in 2 0 j xydA + cos 0 sin 0 j
Y Fig. A3.10 (y"-x") dA
= I xy cos 20+ ~ (Ix - I y ) sin 2 13
A3.ll Moments of Inertia with Respect of Inclined
Axes 2
Therefore, I X1Y1 is zero when
unsymmetrical beam sections are very com-
mon in aircraft structure, because the airfoil tan 2 13 = 2I xy - - - - - - - - - - - -(6)
shape is generally unsymmetrical. Thus, the Iy_I X
general procedure with such sections is to first
find the moment of inertia about some set of A3.l3 Principal Axes.
rectangular axes and then transfer to other in-
clined axes. Thus, in Fig. A3.11 the moment of In problems involving unsymmetrical bending,
inertia of the area with respect to axis X1X1 is, the moment of an area is frequently used with re-
spect to a certain axis called the principal
Ix 1 =jy 1 "dA= j(ycos0-xsin0)"dA axis. A principal axis of an area is an axis
= cos"0j y"dA+sin"0jx 2 dA-2Sin0 cos 0 about which the moment of inertia of the area is
either greater or less than for any other axis
j xydA passing thru the centroid of the area.
= Ix cos" 0 + I y sin" 0 - 2 I xy sin 0 cos 0 (3) Axes for Which the product of inertia is
zero are principal axes.
and likewise in a similar manner, the following Since the product of inertia is zero about
equation can be derived: symmetrical axes, it follows that symmetrical
I Y1 =I x sin"0 + Iycos"0 + 2 I XY sin 0 cos axes are principal axes.
The angle between a set of rectangular
o- - - - - -(4) centroidal axes and the principal axes is given
by equation (6).
Example Problem 4.
Determine the moment of inertia of the ang-
le as shown in Fig. A3.12 about the principal
axes passing through the centroid. Solution:
x--l----------:=-I--==-------L---..~-x
Reference axes X and Yare assumed as shown in
Fig. A3.12 and the moment of inertia is first
calculated about these axes. Table 8 gives the
calculations. The angle is divided into the
two portions (1) and (2).
Fig. A3.11
Table R
I ICy
Part Area cx
Ix Iy
A x y Ay Ax Ay 2 Ax 2 Axy
.l.xl 5(1)3-= 0019 11 3
1 .375 .75 .125 .0469 .281 .0058 .211 .0351 12 • (4) • 12x4Xl, 5 =.070 .0077 .281
2 .500 .125 1,25 .625 .0625 .7800 .0078 .0781 1 1 3 .9470 .010
12 x4 x2= .167 i2x2X.253 •• 0026
.875 .6719 .3435 .7858 .2188 .1132 .9547 .291
A3.1l
~Icx and Icy = moment of inertia of each portion section has been broken down into 16 stringers
about their own X and Y centroid- as listed in column 1. For the top surface, a
al axes. width of 30 thicknesses of the .032 skin 1s as-
sumed to act with the stringers and a width of
Location of centroidal axes;-
25 thicknesses of the .04 skin (see Col. 3). On
y = ZAy .6719 = .767" the lower surface, the skin half way to adjacent
li .875 stringers is assumed acting with each stringer,
or the entire skin is effective. Column 4 gives
x = ZAx .3435 = .392" the combined area of each stringer wlit and is
ZA .875
considered as concentrated at the centroid of
Transfer moment of inertia and product of iner- the stringer and effective skin. All distances,
tia from reference X and Y axes to parallel x and y, columns 5 and 8, have been scaled from
centroidal axes;- a large drawing.
Ix = Ix - A~?' = .955 - .875 x .767 2 = .440 y
Iy = I y - Ai. 2 = .291 - .875 x .392 2 = .157
I XY = ZAxy - AY:.y = .1132 - .875 x .767 x .392 = .032 Skin
.040 Skin r ef.
- .150
""2 3
Calculate angle between centroidal X and Y axes
and principal axes through centroidal as fol- --,---+_-+-~XRef.
lows;- ~~--.---t~~.- x
o
tan 2 0 = ~ = 2(-.150 = -.30 = 1.06 L9 11
-r-
12 13 16
I y - Ix .157-.440 -.283 .J
ca
2 0 = 46 0 - 40' 0 = 23 0 - 20'
Calculate moments of inertia about centroidal
Fig. A3.13
"/1-
Yp Y y
principal axes as follows;-
I xp = Ix cos 2 0 + I y sin 2 0 - 2I xy sin 0 cos 0 Table 9
11
= .44x .918 2 + .157x .3965 2 -2(-.150)x 1 2 3 4 5 6 7 8 9 10
.3965 x .918= .092 in. 4 '"1 .11 .03 '" .14 E-«
,(
Location of centroidal axes with respect to ref.
axes, -
y = ZAy = - 1.465 = - .396"
li 3.70
Example Problem 5. x=ZAx=- 58.238 15.74"
Fig. A3.13 shows a typical distributed ZA 3.70
flange - 2 cell - wing beam section. The upper
and lower surface is stiffened by Z and bulb I x =187.04-3.70x .396 2 =186.5 in.4
angle sections. Determine the moment of inertia I y = 1348.36 - 3. 70x 15.74 2 = .431. 7 in.4
of the section about the principal axes.
Solution; I XY = -13.35 - (3.70 x .396 x 15.74) = 36.41in. 4
The properties of the cross-section depend
tan 2 0 = ~ = 2 (-36.41) = -.29696
upon the effective material which can develop
resist1ng axial stresses. The question of ef- Iy-IX 431.7-186.5
fective material is taken up in later chapter. 20=16 0 -32.5', 0=8 0 -16.25'
Table 9 shows the calculations for the moment of 2 2
inertia about the assumed rectangular reference Ixp=Yx cos 0+l y sin 0-2 I XY sin0cos0
= 186.46 x .9896 2 + 431. 7 x .1438 2 -2
axes XX and YY (see Fig. A3.13). The cross-
t36.41x .9896x (-.1438)J = 181.2 in. 4
A3.12 CENTROIDS, CENTER OF GRAVITY, MOMENTS OF INERTIA
I yp =Ix sin 2 0 + I y cOS 2 0 + 2 I xy sin 0 A3.14 Section Properties of Typical Aircraft Structural
Sections.
cos 0 =186.46 x .14382 2 + 431. 7 x .9896 2 + Table A3.10 through A3.15 and Chart A3.1
2 -36.41x,9896x(-.1438) =437in. 4 give the section properties of a few structural
shapes common to aircraft. Use of these tables
will be made in later chapters of this book.
y
y
S=1
~ =~~'~1'01613r":12:51_'lll3!1~§'11916§§'!001417~c'10115i8~c'11190~!~'3i4i7~
.625
.063 .125 .123 .0110 .0082 .298 .259 .625 .078 .158.201.0055.0183 .187 .341
.750 .050 .094 .110 .0106 .0122 .310 . 3~3
.750 .875 .050 .094 .117 .245 .0065 .0151 .237 .360
.063 .125 .139 .0128 .0148 .304 .326 . 750 ~ .147.247.0079.0184.233.354
1. 000 .063 .125 .170 .0165 .0367 .312 .464 .078 .177.253.0094.0214.230.348
.625
.050 .094 .104 .0130 .0068 .353 .257 ~ .125 .216 .362 .0213 .0276 .314 .358
.063 .125 .131 .0158 .0083 .347 .251 1. 000 . 125 .328.379.0304.0378.304.339
.050 .094 .117 .0151 .0122 .360 .323 ~ .123.140.0026.0181.144.384
.750 .063 .125 .147 .0184 .0148 .354 .317 .500
.078 .148.145.0029.0211.141.377
.875 .078 .125 .177 .0214 .0175 .348 .314 ~ . 139 . 187 .0049 .0216 . 187 .394
.050 .094 .129 .0172 .0197 .365 .391 .625 .078 . 168 . 193 .0058 .0252 . 186 .388
~
.875 .163 .0210 .0241 .359 .385 ~ . 155 .229 .0083 .0250 .231 .402
.078 .197 .0245 .0287 .353 .382 ~ . 187 . 242 .0098 .0294 .229 .393
.125
1. 000 ~
.078
.178
.216
.0236
.0276
.0367
.0439
.364
.358
.454
.451
1. 000 .750 .094 . 125 .220 .248 .0113 .0335 .226 .390
~ . 170 .289 .0128 .0285 .275 .409
.050 .094 .110 .0176 .0068 .400 .249 ~ .206.294.0152.0335.271.403
.625
.063 .125 .139 .0216 .0083 .394 .244 .875 .094 .244.300.0176.0383.269.397
.0'" .094 .123 .0205 .0121 .408 .315 ~ .265.460.0417.0460.397.417
~
.750 .155 .0251 .0147 .403 .308 1. 250 . 125 . 4 0 6 . 4 7 8 . 0 6 0 7 . 0 6 4 3 . 387 .398
1. 000 f----t--'-;.0",7",,8-1 .187 .0294 .0175 .396 .307
~ .170 .0285 .0242 .409 .377 1.125 f-_._50_0---+1_:'"'~~~",,~-1 .125 : :~~ : : :;~
: : ~~;;
: ~~:: ::~ :~~
.875
~
.094
.125 .206
.244
.0335
.0383
.0287
.0332
.403
.397
.373
.369 .625
~ .147 .178 .0051.0285.186.441
.078 . 177 . 184 .0060 .0334 . 184 .434
.186 .0320 .0367 .415 .444 ~ . 197 .232 .0102 .0388 .22R .444
1. 000
.226 .0377 .0439 .408 .441 .750 .094 .232.237.0117.0444.225.438
.625 .125 .147 .0285 .0083 .441 .237 ~ . 181 .278 .0135 .0379 .274 .458
.094 .142 .0304 .0197 .464 .374 ~ .216.283.0158.0441.271.452
~
~:~~~"'~-1 ;~~ ~~~ ~~~; ~~~~ ~~~ :~;
.178 .0374 .0241 .458 .368
.875
1.125 ~
.094
.216
.256
.0441
.0507
.0287
.0332
.452
.445
.364
.361
1.125 f-._8_7_5f-1 . 125 : : : : : :
.125 ~ .236.336.0232.0495.314.458
1. 125 ~ .210 .0467 .0530 .472 ~~ 1. 000 ~ .279.341.0269.0569.310.452
~
.094
.255
.303
.0548
.0632
.0637
.0744
.464
.457
.500
.496
.125 .359.352.0331.0695.303.440
~ .350.508.0661.0757.435.466
.750 ~
.078
.170
.206
.0421
.0497
.0148
.0175
.497
.491
.295
.291
1. 375 .125 .453.519.0832.0930.428.453
.625 .094 . 2 2 0 . 1 8 1 . 0 0 7 1 . 0 4 9 3 . 179 .473
~ .202 .0532 .0366 .514 .428
~ ~~'j2~67~t~'f27~8S~'~01~9~0~~·t06~5~0¢~·t2~67S~·~4~9i3~
1. 250 1. 000 .125
.094 .291 .0729 .0511 .501 .419 .875 . 125 .344.288.0234.0796.261.481
1. 250 ~
.094
.294
.338
.0765
.0886
.0887
.1039
.519
.512
.558
.555
1,250 .078
I .125 .265 .377 .0334 .0698 .355 .513
1. 125 ~ .314.383.0389.0807.352.507
.750 .063 .178 .0562 .0148 .543 .288
.125 .406.393.0484.0994.345.495
~ .210 .0662 .0367 .562 .418
1. 500 .125 .500.560.1106 .1292 .470 .508
1. 375 1. 000 ~
.094
.125
.255
.303
.0794
.0911
.0439
.0511
.558
.549
.415
.411 .625
~ .197 .169 .0063 .0540 .179 .524
.094 .232 .175 .0073 .0621 .177 .517
~~~
.294 .0951 .0887 .569 .549
1. 250 I : ~ .236.263.0169.0705.268.547
.453 .1365 .1308 .549 .537
~ t~'j2~7!9=~.~26g8~~.~0~19~8t:1=~.+0~81N84:~.~2~66~~.
5~4~3~
.750 .078 .0765 .875
.226 .0175 .582 .278
1. 375 1-_ _-+-'-.'"'12"'5, .125 .359 .278 .0242 .1002 .259 .528
1. 500 1. 000 ~
.094 .125
.265
.314
.0961
.1116
.0439
.0511
.602
.596
.407
.403
~ .326.371.0403.1010.352.557
1.125 ~ . 4 2 2 . 3 8 1 . 0 5 0 4 . 1246 .346 .544
1. 250 .094 .361 .1349 .1040 .611 .536
.156 . 5 1 3 . 3 9 0 . 0 5 9 1 . 1451 .339 .532
.750 .078 .245 .1104 .0175 .671 .267
1. 625 .156 .668 .612 .1710 .2035 .506 .55~
1. 750 1. 000 .078 .125 .284 .1377 .0439 .696 .393
~ .244 .168 .0076 .0768 .177 .561
1. 250 .094 .385 .1925 .1039 .707 .520 .625 .125 .313 .178 .0090.0939.170.548
.750 .063 .218 .1272 .014 .7 5 .261
1.500 ~ .125 .375 .269 .0249 .1236 .258 .574
2.000 1. 000 .094 .361 .2194 .0511 .779 .376 .875
.125 .156 . 4 5 5 . 2 7 8 . 0 2 8 9 . 1 4 8 5 . 267 .572
1. 250 .094 .408 .2621 .1040 .801 .505
1.125 .094 .338.360.0414.1233.350.604
1. 500 .125 .594 .3852 .2346 .806 .629
.750 .063 .249 .2172 .0148 .9:14 .244
2.500
.875 ~
.094 .125
.323
.385
.2890
.3383
.0288
.0333
.945
.938
.298
.294
1. 000 ~
.125
.408
.531
.3723
.4707
.0512
.0636
.955
.941
.354
.346
A3 13
r-x-f
&
Table A3. 12 SoT
Ill-
Properties of Extruded Aluminum T~
J1 ~'-,- - ~-~-r
W '
Alloy Equal Leg Angles. (Ref. 1938 Table A3.14
Alcoa Handbook) B Properties of
Unequal Angles
~l
I.r-- R
Dimensions Area Axis XX or YY Axis ZZ x
W t R sq. in. I jO d I f> II V ---L 1
5/8
3/4
3/4
3/32
1/16
3/32
1/8
1/8
1/8
.111
.089
.132
0.004
0.004
0.183
0.220
0.187
0.199
.0015
.0018
0.117
0.142
45°
45° I y
I A~J-
0.006 0.219 0.214 .0026 0.141 45°
3/4 1/8 1/8 .171 0.008 0.217 0.227 .0034 0.141 45° 0 NOMINAL DIMENSIONS SECTION ELEMENTS
1 1/16 1/16 .122 0.012 0.311 0.271 .0048 0.199 45° z AREA
1 3/32 1/8 .178 0.016 0.301 0.276 .0066 0.193 45° ~
u
A B T R -
Square
Y x Ixx Iyy ,0 xx ,oyy
1 1/8 1/8 .234 0.021 0.298 0.290 .0085 0.191 45° Inches Inch Inches 4 Inch
1 3/16 1/8 . 339 0.029 0.293 0.314 .0124 0.192 45° '"
OJ Inches
1-1/4 3/32 3/32 .230 0.033 0.38 0.34 .014 0.24 45° I .625 .500 .050 .063 .0535 .126 .187 .00Il .0020 .146 .193
1-1/4
1-1/4
1/8
3/16
3/16
3/16
.30
.43
0.042
0.059
0.37
0.37
0.35
0.37
.017
.025
0.24
0.24
45°
45°
c+3 .750
.500
.625
.063
.063 .063
.0739
.0818
.120
.162
.243
.222
.0014
.0027
.0041
.0044
.139
.181
.235
.232
.625 .063 .0922 .150 .269 .0029 .0069 .177 .274
1-1/4 1/4 3/16 .56 0.074 0.36 0.40 .032 0.24 45° ~
1-1/2 3/32 3/16 .28 0.058 0.46 0.40 .024 0.30 45° ~6
.875
.750 ~
.125
.125 .100
.184
.190
.2I1
.250
.272
.0049
.0082
.0073
.0123
.221
.2I1
.270
.258
1-1/2 1/8 3/16 .36 0.074 0.45 0.41 .031 0.29
1-1/2 3/16
45° ~ .625 ~
.125
.100 .140
.160
.320 .0029
.0050
.0100
.0170
.170 .31b
.304
3/16 .53 0.107 0.45 0.44 .044 0.29 45° ~ .184 .344 .165
1-1/2 1/4 3/16 .69 0.135 0.44 0.46 .057 0.29 45° ~ I. 000 .750 ~
.125
.125
.108
.200
.178
.199
.299
.322
.0051
.0086
.0106
.0182
.217
.208
.313
.302
1-3/4 3/32 3/32 .32 0.096 0.55 0.47 .039 0.35 45° f-jf-
1-3/4 1/8 3/16 .42 0.121 0.53 0.47 .050 0.34 45° 't2 .875 ~
.094
. I16
.167
.220.
.231
.281
.292
.0079
.0IlO
.01Il
.0155
.262
.257
.310
.305
1-3/4 3/16 3/16 .62 0.174 0.53 0.50 .072 0.34 45° ~ ~ .124 .159 .403 .0054 .0197 .209 .400
.0276
1-3/4 1/4 3/16 .81 0.223 0.52 0.52 .093 0.34 45° ~
.750
~ .179 .170 .416 .0074 .203 .393
2 1/8 1/4 .49 0.18 0.61 0.53 .08 0.40 45° .4- I. 250
.125 .125 .231 .181 .427 · 0092 · 0346 .200 .387
2 3/16 1/4 .72 0.27 0.61 0.56 .11 0.39 45° 4~ I. 000
~
~
.139
.202
.239
.250
.361
.373
· 0124
.0171
· 0217
.0306
.298
.291
.394
.389
2 1/4 1/4 .94 0.34 0.60 0.58 .14 0.39 45° 18 .125 .262 .260 .384 .0215 · 0383 .286 .382
2 5/16 1/4 1.16 0.41 0.60 0.61 .17 0.39 45° ~ .750 ~ .204 .156 .522 .0077 .0463 .194 .476
.125 .264 .167 .534 .0096 .0584 .191 .465
-¥.J-. .228 .0512
4~ ~ .228 .472 .0182 .282 .474
I. 500 I. 000
~ .156 .295 .239 .485 .0226 .0647 .276 .468
~ .156 .361 .249 .495 · 0265 .0772 .271 .463
~ ~ .251 .310 .433 · 0347 .0550 .372 .468
.462
1} I. 250
~
.156
.327
.400
.321
.331
.444
.455
.0436
.0518
· 0698
.0834
.365
.360 .457
.0998
~ I. 000 .125 .327 .222 .591 .0236 .269 .553
~ I. 750
I. 250 .125 .358 .300 .545 .0465 .1078 .360 .549
~
30 I. 500
~
.156
.156 .389
.478
.383
.393
.506
.517
.0776
.0928
· I144
.1377
.447
.441
.542
.537
-----'--
I
Y
1=:U- A
~--r
-T
T
Table A3.15
l~
X X
Properties of
Table A3.13 ~S I-Sections
L--I--x
,..,...--R
Properties of
Tee-Sections _J--~
r-- 1 T R..L
----j""TT
~B/ll---<l B ---.j----r;
g NOMINAL DIMENSIONS
AREA
SECTION ELEMENTS 0
Z
NOMINAL DIMENSIONS
AREA
SECTION ELEMENTS
c--+-2 I. 250
.625
.750
.050
.063
.094
.125
.094
.127
.121
.160
.0028
· 0055
.0077
.0098
.173
.209
.287
.276 ~
~ .750
I. 250 ~
.078 .125
.207
.250
.0206
.0239
.0193
.0235
.316
.310
.306
.307
.750 .094 .106 .148 .0046 .0103 .209 .312 I. 500 .078 .289 .0284 .04I1 .314 .378
~ I. 000
· 050
.063 .150 .228 .0131 .296 .293
3
m .m · u, .366 .336
r---t- I. 375 I. 000 .078 .183 .235 .0156
· 0129
.0158 .292 .294 ~ I. 375
~ .125 .329 .0425 .0375 .359 .338
~ .125
.216 .242 .0188 .290 .295 f----6
.875
~ I. 000 .094
.094 .240 .327
· 0182
.0345 .0168 .379 .280
I. 625
I. 375
.094
.094
.376
.341
.0497
.0580
.0626
.0375
.364 .408
7 I. 250 ~ .413 .332
r--4--- .750
.750
.063
.094
.141
.204
.141
.159
.0052
.0080
· 0169
.0244
.192
.198
.346
.346 r--+-
9
I. 000 I. 625
I. 750
.094
.094
.125 .388
.4I1
· 0676
.0725
.0626
.0786
.418
.420
.402
.437
---&- I. 000 · 078 .193 .225 · 0160 .0200 .288 .327
~
I. 375 .094 .364 .0970 .0375 .516 .321
--IT- I. 500 I. 000 .094 .125 .228 .232 .0186 .0245 .286 .328
~
I. 625 .4I1 .0626 .524 .390
--rr-
--rr- I. 250
I. 250
.078
· 094
.212
.252
.307
.314
.0301
· 0354
.0206
.0245
.377
.375
.312
.312 ~
13
1.250 1. 875
2.125
· 094
.094
· 094
.125 .458
.505
· I128
.1285
.1442
· 0971
.1423
.530
.534
.460
.531
--IT- I. 250 .125 .326 .327 · 0449 · 0319 .371 .313
~
I. 5uO · u.4 .• I1 .15.1 · 0490 .622 .345
.192 .180 .0IlO .240 .370 .125 .594 .2268 .1023
-¥s- .875 · 078
.125 .295 .198
· 0263
.234 .372
~ I. 500
I. 750
2. 000 .125 .125 .656 .1543
.618
.625
.415
.485
17
.875
1.125 .078 .212 .254
· 0161
· 0228
· 0407
· 0263 .328 .352 ~
17 2.500 .125 .781
.2564
.3156 .3058 .636 .626
--IT- I. 625 1.125 .125 .125 .326· .274 · 0339 · 0408 .322 .364
~ 2. 000 2. 000 ~ .125
.552 .3922 · I183 .843 .463
49 I. 375 · 094 .275 .345 · 0475 · 0314 .415 .338 19 .125 .719 .4946 .1544 .830 .464
r-To- I. 375 .125 .358 .358 · 0605 · 0408 .4I1 .338
~ ~ .607 .6571 .1184 I. 04 .442
r-rr- I. 375 .15b
.094
.437
.240
.370
.179
.0725
.0131
.0499
.0393
.407
.234
.338
.405
21 2.500 2.000 .125 .156 .789
.054
.8315 .1546
.1184
I. 03 .443
~
~
.875
.875 .125 .311
.263
.191
.252
.0164
.0273
· 0512 .230
.322
.406
.386
~
23
3. 000 2. 000
I. 500
-4E--
.078
.156
.851
.542
· •• 23
1. 262
1. 298
.1547
.0413
1.23
1. 22
1. 55
.42
.426
.275
~ 1.125 · 094
.276
· 0393
.0628 .314 .387 ~
~
.156 .418 .0412 .740 .1184 I. 60 .400
~ 1. 125
.125 .287 .333 .0484 .0393 .411 .370 ~ 2.000
.125
.969
I. 888
.1547
~ I. 750 1. 375
I. 375
· 094
.156 .457 .358 .0627 .403 .371 ~
27 4. 000 3.000
.125
.156 1. 52
2.424
4.136 .6586
1.58
1.65
.400
.658
~ · 0741
.0394 .500 ----za
~ 1. 625
1. 625
.094
.125
.310
.405
.420
.434
.0777
.0995 .496
.356
.356 29 3.500 ~
.250
.188 2.00
2.60
5.521
6.968
1. 258
1. 638
1.66
1. 64
.794
.794
~ 1. 625 .156 .497 .446 .1200
· 0513
.0629 .492 .356 -T 4. 000 .313 .250 3.53 9.415 3.037 1. 63 .928
30
A3.14 CENTROIDS, CENTER OF GRAVITY, MOMENTS OF INERTIA
Table A3.16
SECTION PROPERTIES OF TYPICAL AIRCRAFT EXTRUDED SECTIONS
!~ 1 17/32 13/16 1/8 .040 .050 1/16 .068 .00686 .317 .319 .00150 .148 .112
-.-:C i"- 2 1/2 1 7/32 .050 .050 .050 .113 .01515 .3665 .510 .00155 .117 .110
:t~
90- 3 1/2 7/8 3/16 1/16 1/16 3/32 .1025 .0100 .313 .383 .00162 .125 .116
e t
4 9/16 7/8 3/16 .060 .060 3/32 .1031 .0102 .313 .383 .00215 .114 .130
r y.-J
~ e
k- ~
--J,
L 5 9/16 7/8 3/16 .050 .078 13/32 .1061 .0120 .329 .345 .00302 .165 .133
--~~ --
l_e'~_ -It 6 3/4 1-1/8 3/16 .075 .075 i 1/16 .1514 .0239 .397 .435 .00550 .190 .182
- - -.. _ - -
L~A-.I
_.- -
7 3/4 1-5/16 9/32 3/32 3/32 9/64 .230 .0507 .471 .574 .0080 .187 .171
8 7/8 3/8 1/16 1/16 1/16 .095 .0101 .326 .4375 .00111 .108 .108
'j
9 7/8 3/8 3/32 3/32 3/32 .1375 .0136 .314 .4375 .00148 .104 .118
'l(
i
' t
A~.- I --
e
1._ '_
I I'-
I
-x
,--t\
10
11
12
1
1-1/4
1-13/16 1
1/2
11/16
3/32
3/32
3/16
3/32
3/32
3/16
3/32
3/32
3/32
.1736
.2320
.644
.0240
.0536
.2920
.371
.480
.677
.500
.625
.906
.00376
.00992
.0555
.147
.206
.295
.158
.210
.326
ie'~
1
1 13 2-1/2 1 1/8 1/8 1/8 .581 .525 .950 1.25 .0662 .338 .323
~- 84 14 2-1/4 1-1/8 1/8 3/16 1/8 .666 .524 .888 1.125 .0872 .362 .375
15 1-3/8 11/16 1/8 1/16 1/16 1/16 .137 .00425 .176 .133 .0178 .360 .687
~I- -- I~ A .t1 J 16
17
1-5/8
1-5/8
-- I--
3/4
3/4
----
5/32
- -
5/32
1/16
1-----
1/16
1/16
3/32
1/16
5/64
--
.168
.206
.00580
.00589
.186
.170
.133
.121
.0335
.0458
.446
.474
.812
.812
ey--,~C
rrl- - !L_)( 18 2 1-1/16 3/16 3/32 3/32
1---
3/32 .306 .0252 .287 .201 .0791 .509 1.0
~ I,
21 2-5/16 1-1/2 5/16 5/32 5/32 , 5/32 .668 .1075 .401 .338 • 2326 .590 1.156
leF 22 1-1/2 1-1/8 1/16 1/16 1/16 .161 .0186 .342 0255 .0171 .326 .750
I A -----e 23 1-1/2 2 1/8 3/32 1/8 .384 .1562 .641 .658 .0346 • 302 .750
:-T( ~*T" 24 1-3/4 2 9/32 1/8 1/8 .760 .315 .643 .757 .0599 .281 .875
25 1-3/4 2-9/16 1/4 5/32 : 5/32 .918 .600 .809 .959 .0730 .282 .875
I
-L---I- e'
'
~ 26 2 2-1/2 5/32 1/8 i 5/32 .605 .385 .800 .793 .159 .514 1.00
!--
,
11, 27 11/16 1 1/8 .050 .050 .050 3/32 .135 .0218 .404 .500 .0163 .348 .6625
28 13/16 1-1/8 3/16 .060 .060 .050 ! 3/32 .180 .0375 .456 .562 .0295 .405 .7825
,1r.:t;"
I _.~ I
8 ~- -
II---t
-
-- --)\
t...,..-
,
e
! 29
30
5/8
3/4
1-;1/2
2
3/16
1/4
.050
.050
1/16
1/16
1/16
1/16
3/32
3/32
.166
.214
.0603
.1382
.595
.793
.75
1.00
.0151
.0281
.298
.358
.600
.725
~t_ :~;J 31 3/4 2 1/4 1/16 1/16 1/16 3/32 .2378 .1449 .773 1.00 .0282 .341 .719
!Le' .j 32 3/4 2 1/4 1/16 3/32 3/32 3/32 .2855 .1820 .795 1.00 .0365 .356 .719
33 9/16 7/8 .050 .050 1/16 .099 .0120 .349 .437 .0056 .238 .537
I~
35 9/16 1 1/16 .080 3/32 .151 .0239 .397 .500 .00828 .234 .531
-
I ~ I~-t 36 9/16 1 1/4 .075 .075 3/32 .170 .0384 .475 .625 .00876 .227 .525
B -->;
*rl
e
37 11/16 1-1/4 1/16 .100 3/32 .213 .0550 .508 .625 .0194 .302 .655
J -- - L6==J.
~ ~,
38
39
.825
3/4
.-1-------
1-1/2
2
.075
.072
.120
.072
3/32
1/8
.302
.249
.110
.1446
.604
.757
.750
1.00
.0391
.0180
.360
.267
.787
.714,
40 3/4 2 .102 .102 3/32 .330 .1826 .750 1.00 .0228 .265 .700
- - -
CHART A3.1 PROPERTIES OF FORMED CHANNEL SECTIONS
I"
.. 4 1:L+~j n-:;-1 r":,i TTT'
{~~~H
,.~_"
..,.;C . . . ,.:iIIH
_ 0 . 3 5 1lEI::::
4Nt
. . tI:' .•
- , -,
~r::r:"""""'-
T- ,;,: :':T=: _~
'tifF'
i::t:
< .3
; .1
+, jm:1L
+ll'H· tt+
I 0.30 < +co i· :'tt-: t~
<
""e;j
0
II
+~'lli I
...:
.2
'7t:i
.~=+= .
:-:~~:.1:.:...u
t;
,t
.iE,bF
:en
H'
.8
IIf~
~~!
L:';J+;.;: W=2
0.25 .1 ·:t 3
J. ",.j ,"".;1.
'S'" ::tW= lc.1:"4
~-- ~En- ' i--:::-:. :!::t",',U"j
r :j:::::::11
m'
1_,;,c,~
, . c' I.•..
fl If
, _ ~
:.t~~~:-.~ ~
,c"' ...
4-
',1=
~.:..;
0 20
. ~
_ ='---W
~-
'It ::.1="
--= tr 1::=:."
. ii. :s
L = ..- :':~L: !i:
, ,
N.A. t
1
~"
__,,':
_·~'.:,f'
o-~~
n'
.'
. " ::;F'-,
~
~
0::;i
= r-
:::=:: MultIply A&I------J
values
H
{nil
"-1
-:t-_:'
~~---'-I
:s""
~
0
~
:.:4 .5
." -==-
0.15 ::;i
_::.' factors:
p t:-::-, .. ""
::;i
.HH'l"''' t I A::,:... R=:j ~
I . 8 . 064
+t+: .072
.051 1. 00
1. 23
1. 375
1. 00:;::
1. 24 "'
}.~=:;::
~ :-r;
,1
: 3'
.4
- I
EfutE±: I"·l=::
4"
. r:'ll
'~'I ~:..r~t :i ; L'; ..
H
,:::h,
;.l-<j'-<4±k~ ..\v=it:·, -1-1'
1::-1
.3
·~-I-I·. 'i::
-+-~:-
1;: ..•:n: ~
0.05
~ n:-~
.2
-;::L"t it
tj,'+'H
:c1Tltr;y' II· ,~:;-i; n::::t:
+ti-:J~'
--<'-·-It
+!~
:j:: '1:" ;1-t-'
Mom. of Inertia: ~m:EtE!EJ .1
+-i:t:tU-_i_U--J:-'..l..1.-:~'::j
"lL;jit:!:::-~ t :.a-=--L~U1i: Area:
TIl
•• j
o ~:
dh
-13fT
1.0 2.5 3.0 3.5 ::tJ:t:tt:;:tt'clL:
1.5 2.D
;;-;-;!n;':l~;;:n..;r 1::I;,:,j""
H - HEIGHT OF CHANNEL .025 >
w
1.5 2.5 3 3.5 4
....
H : HEIGHT OF CHANNEL
'"
A3.16 CENTROIDS CENTER OF GRAVITY MOMENTS OF INERTIA
~d' + +(0-1
8
/05"''' /.e 0·8 /,2 T L
10''- '2 0 " "/..
1
Fig. A3.19
~
I
,",2. D," 0.8 /.
Fig. A3.14 Fig. A3.15 /5 +1 8
----,:
~
i
a" Trrrll;.,
II " 0• 12,,~L
~
0.12. .020 i-
/,
ng. A3.N.
n
- - - I
0.4-°11
1- r 6 '1"""' @ 6" 36" .1 Fig. A3.20
1. 5"
(8) Fig. A3.21 shows the cross-section of
a small fuselage. The dashed line represents
l I
(
!)"
Fig. A3.18
a cut-out in the structure due to a door. As-
sume each of the 13 stringers have an area of
a" 0.1 sq. in. Consider fuselage skin ineffective.
10 .8
Calculate the moment of inertia of the effective
section about the principal axes.
CHAPTER A4
GENERAL LOADS ON AIRCRAFT
\
~~~d~~~e)~ Land. (wheel or motion is called translation, and from basic
Physics, the accelerating force F = Ma, where M
(2) Landing Loads Landing on Water. is the mass of the body or Wig. In Fig. A4.1
the unbalanced force system F causes the rigid
Arresting. (Landing on Air- body to accelerate to the right. Fig. A4.2 shows
craft Carriers). the effect of this unbalanced force in producing
A4.3
--
r - - - - - - - , Motion
..,.. .!'-~-
Motion are:-
Fn = Mrw s = Mvs;r - - (4)
T
w F = Mra - - - - - - - - - -
(5 )
t
Where w = angular velocity at the point A.
a = angular acceleration at point A.
Fig. A4.1 Fig. A4.2 r = radius of curvature of flight path
at point A.
The inertia forces are equal and opposite
a force on each mass particle of m~a, msa, etc., to these effective forces as indicated in Fig.
thus the total effective force is Zilla = Ma. If A4.3. These inertia forces can then be con-
these effective forces are reversed they are re- sidered as part of the total force system on the
ferred to as inertia forces. The external airplane which is in equilibrium.
forces and the inertia forces therefore form a If the velocity of the airplane along the
force system in equilibrium. path is constant, then at = zero and thus the
From basic Physics, we have the following
relationships for a motion of pure translation inertia force F = 0, leaving only the normal
t
if the acceleration is constant:- inertia force F •
n
=
v - Vo at - - (1) If the angular acceleration is constant,
the following relationships hold.
s = vot + ~ at S (2)
v - v o s = 2as (3) w- We = at - - - - - - - - - - - (6)
Q = wot + ~atS (7)
Where,
WS
_ Wo S = 2aQ - - - - - - (8)
s = distance moved in time t.
where G = angle of rotation in time t.
V o = initial velocity
Wo = initial angular velocity in rad/sec.:
=
v final velocity after time t. w = angular velocity after time t.
In Fig. A4.3 the moment To of the inertia
Inertia Forces on Rotating Rigid Bodies. forces about the center of rotation (0) equals
A common airplane maneuver is a motion Mra(r)= Mrsa. The term Mr 2 is the mass moment
along a curved path in a plane parallel to the of inertia of the airplane about point (0).
XZ plane of the airplane, and generally referred Since an airplane has considerable pitching
to as the pitching plane. A pUll up from steady moment of inertia about its own center of gravity
flight or a pUllout from a dive causes an air- axis, it should be included. Thus by the
plane to follow a curved path. Fig. A4.3 shows parallel axis
an airplane following a curved path. If at
point A the velocity is increasing along its To = loa + I e.ga - - - - - - - - - - - - - (9)
path, the airplane is being sUbjected to two where 1 0 = Mrs and I e.g = moment of inertia of
accelerations, namely, at, tangential to the
curve at point A and equal in magnitude to airplane about Y axis through e.g. of airplane.
Inertia Forces For Pitching Rotation of Airplane
Center of Curvature about Y Axis Through e.g. airplane.
01 Flight
Path
r In flight, an air gust may strike the hori-
\ zontal tail producing a tail force which has a
\ moment about the airplane e.g. In some landing
an=Fws=VsIF conditions the ground or water forces do not
pass through the airplane e.g., thus producing
a moment about the airplane e.g. These moments
cause the airplane to rotate about the Y axis
through the e.g.
Therefore for this effect alone the center
Fig. A4.3 of rotation in Fig. A4.3 is not at (0) but at
A4.4 GENERAL LOADS ON AIRCRAFT
the e.g. of airplane, or r = o. Thus Fn and Ft air forces on the wing. For example, consider
the two air pressure intensity diagrams in Figs.
equal zero and thus the only inertia force for A4.6 and A4.7. These distributed force systems
the pure rotation is I e.g. a, (a couple) and can be replaced by their reSUltant (R), whiCh
thus the moment of this inertia couple about the of course must be known in magnitUde, direction
e.g. = Te.g. = I e.g. a. and location. The location is specified by a
term called the center of pressure which is the
As explained before if the inertia forces point where the resultant R intersects the air-
are included with all other applied forces on foil chord line. As the angle of attack is
the airplane, then the airplane is in static changed the reSUltant air force changes in mag-
equilibrium and the problem is handled by the nitUde, direction and center of pressure
static equations for equilibrium. location.
A4.5 Air Forces on Wing.
The wing of an airplane carries the major
portion of the air forces. In level steady
flight the vertical upward force of the air on
the wing, practically equals the weight of the
airplane. The term airfoil is used when re-
ferring to the shape of the cross-section of a
wing. Figs. A4.4 and A4.5 illustrate the air
pressure intensity diagram due to an air-
Fig. A4.6 Fig. A4.7
•
Lift and Drag Components of ReSUltant Air Force.
Instead of dealing with the reSUltant force
R, it is convenient for both aerodynamic and
stress analysis considerations to replace the
resultant by its two components perpendicular
-
and parallel to the airstream. Fig. A4.8 il-
lustrates this resolution into lift and drag
components.
Angle of Attack Angle of Attack
L
= 120 = _ 60
~~ ,/:----c.ge ;J' D
Flight P: . e>. Fig. A4. 10
orizontal
T=D
Fig. A4.11
W
=
T engine thrust.
forces in steady horizontal flight. L repre-
L = total wing lift plus fuselage lift.
D = total airplane drag. sents the total airplane lift (wing plus tail).
=
Ma moment of Land D with reference to wing Therefore L = W. Now assume the airplane is ac-
celerated upward along the Z axis. Fig. A4.l2
a.c. (aerodynamic center)
shows the additional inertia force wag/g acting
W = weight of airplane.
I L = inertia force normal to flight path. downward, or opposite to the direction of
acceleration. The total airplane lift L for the
I D = inertia force parallel to flight path. Z
1 = rotation inertia moment. I
m nzL
E = tail load normal to flight path.
For a horizontal constant velocity flight
condition, the inertia forces I , I D, and 1 ~ c.g.o_D'
L m T-
would be zero. For an accelerated flight con-
W
dition involving translation but not angular T=D
Fig. A4.12
acceleration about its own c.g. axis, the
inertia moment 1m would be zero, but I and I ~ ~a
g z
L D
would have values. unaccelerated condition in Fig. A4.11 must be
mUltiplied by a load factor ng to produce static
Equations of Equilibrium For Steady Flight. equilibrium in the g direction.
From Fig. A4.l0 we can write:-
Thus, naL
..
- W- ~g a g = 0
weight W of the airplane, hence a definite maximum flight velocity. For com-
mercial airplanes the velocity is limited to a
n W ~ a (See Fig. A14.13) reasonable glide speed which is sufficient to
x g x take care of reasonable flight operations.
If U is taken as 30 ft./sec. and m as the would give an acceleration ~ess than the lim-
change in C with respect to angle of attack ited values given by lines AB and CD.
zA The positive and negative gust accelera-
in absolute units per degree, equation (A) re- tions due to a 30 ft./sec. gust normal to flight
duced to the following path are shown on Fig. A4.l4. In this example
diagram, a positive gust is not critical within
3KmV (B) the restricted velocity of the airplane since
lin = W/S - - - - - - - the gust lines intersect the line BD below the
line AB. For a negative gust, the gust load
Therefore the gust load factor n when air- factor becomes critical at velocities between F
plane is flying in horizontal attitude equals and D with a maximum acceleration as given by
point E.
3KmV For airplanes which have a relatively low
n = 1 + -- - - - - - - - - - - - - - - - (C)
- W/S required maneuver factor the gust accelerations
may be critical for both positive and negative
and when airplane is in a vertical attitude accelerations. Examination of the gust equation
indicates that the most lightly loaded condition
n = -+ ~ - - - - - - - - - - - - - - - - (D)
(smallest gross weight) produces the highest
W/S gust load factor, thus involving only partial
pay load, fuel, etc.
A4.10 lllustration of Main Flight Conditions. On the diagram, the points A and B corre-
Velocity-Load Factor Diagram. spond in general to what is referred to as high
As indicated before the main design flight angle of attack (H,A,A,) and low angle of attack
conditions for an airplane can be given by (L.A.A.) respectively, and points C and D the
stating the limiting values of the acceleration inverted (H.A.A.) and (L.•A.A.+ conditions re-
and speed and in addition the maximum value of spectively.
the applied gust velocity. As an illustration, Generally speaking, if the airplane is de-
the design loading requirements for a certain signed for the air loads produced by the veloc-
airplane could be stated as follows: "The ity and acceleration conditions at points A, B,
proposed airplane shall be designed for applied E, F, and C, it should be safe from a structural
positive and negative accelerations of + 6.0g strength standpoint if flown within the specified
and -3.5g respectively at all speeds from that limits regarding velocity and acceleration.
corresponding to CL up to 1.4 times the Basically, the flight condition require-
max. ments of the Civil Aeronautics Authority, Army,
maximum level flight speed. Furthermore, the and Navy are based on consideration of specified
airplane shall withstand any applied loads due velocities and accelerations and a consideration
to a 30 ft./sec. gust acting in any direction of gusts. Thus a student understanding the basic
up to the restricted speed of 1.4 times the discussion above should have no difficulty un-
maximum level flight speed. A design factor of derstanding the design requirements of these
safety of 1.5 shall be used on these applied three government agencies.
loads If • For stress analysis purposes, all speeds
In graphical form these design require- are expressed as indicated air speeds. The
ments can be represented by plotting load fac- "indicated" air speed is defined as the speed
tor and velocity to obtain a diagram which is Which would be indicated by a perfect air-speed
generally referred to as the Velocity-accelera- indicator, that is, one that would indicate
tion diagram. The results of the above speci- true air speed at sea level under standard at-
fication would be similar to that of Fig. A4.l4. mospheric conditions. The relation between the
Thus, the lines AB and CD represent the re- actual air speed Va and the indicated air speed
stricted positive and negative maneuver load
factors which are limited to speeds inside line Vi is given by the equation
BD which is taken as 1.4 times the maximum
level flight speed in this illustration. These
restricted maneuver lines are terminated at
points A and C by their intersection with the
Vi ='
Vf§.Po Va
Example Problem 1
Fig. A4.15 illustrates an airplane landing
on a Navy aircraft carrier and being arrested by
a cable pUll T on the airplane arresting hook.
If the airplane ·weight is 12,000 lbs. and the
airplane is given a constant acceleration of 3.5g
(112.7 ft/sec 2 ) , find the hook pUll T, the wheel
A4.11 Special Flight Design Conditions. reaction R, and the distance (d) between the line
There are many other flight conditions of action of the hook pUll and the airplane e.g.
which may be critical for certain portions of If the landing velocity is 60 M.P.H. What is the
the wing or fuselage structure. Most airplanes stopping distance.
are equipped with flaps, to decrease the land- W = 12000 lb.
ing speed and such flaps are lowered at speeds
at least twice that of the minimum landing
speed. Since the flapped airfoil has different
values for the magnitude and location of the
airfoil characteristics, the wing structure
must be checked for all possible flap conditions
within the specified requirement relative to
maximum speed at which the flaps may be oper-
R Fig. A4.15
ated. Generally speaking, the flap conditions
will effect only the wing portion inboard of the
flap and it is usually only critical for the Solution: -
rear beam web or shear wall and for the top and
bottom walls of the torsion box. This is due to On contact of the airplane with the arrest-
the fact that the deflection flap moves the ing cable, the airplane is decelerated to the
center of pressure considerably aft thus pro- right relative to Fig. A4.15. The motion is pure
ducing more shear load on the rear shear wall as translation horizontally. The inertia force is
well as torsional moment on the conventional
cantilever box metal beam. Ma = ~g a = (12000)
g'
3 5g = 42000 lb •
The airplane must likewise be investigated
for aileron conditions. Operation of the ailer- The inertia force acts opposite to the direction
ons produce a different air load on each side of acceleration, hence to the left as shown in
of the airplane wing which produces an angular Fig. A4.15.
rolling acceleration of the airplane. further- The unknown forces T and R can now be solved
more, the deflected ailerons change the mag- for by using the static equations of equilibrium.
nitude and location of the airfoil character-
istics, thus calculations must be carried out to ~FX = -42000 + T cos 10° 0
determine whether the loads in the aileron con-
ditions are more critical than those for the hence, T = 42700 lb.
normal flight conditions.
For angular acceleration resulting from -12000 + R - 42700 x sin 10n o
pitching moments due to air gusts on the tail,
A4.9
VS - 0 = 2 x 96.6 x 35
or
SUbt: - v = 82 ft/sec. = 56 M.P.H.
9000 lb.
,:l-~_~ThrustLine __
r-
9'
c'g:--Ma -1- _
A
38'
Fig. A4.17
Solution: -
The airplane is being decelerated horizon-
Fig. A4.16
tally hence the inertia force through the air-
plane c.g. acts toward the front of the airplane.
Solution: - Since the braking force is given we can solVe
for the deceleration factor by the eqUilibrium
The forces will be determined just after equation,
the beginning of the catapult run, where the
car velocity is small, and thus the lift on the ~FX = 35000 - Max = 0
airplane wing and the airplane drag can be
neglected. hence, Ma = 35000
x
Horizontal inertia force acting toward the air- or W
plane tail equals, (-)
g x
a = 35000
whence
Ma = (9000) 3.0g 27000 lb. 35000 / s
g ax = (100000) 32.2 = 11.27 ft sec
From statics: -
To find landing run (s),
4Fx = -900 - P + 27000 = 0, hence P = 26100 lb.
VS - Vo s = 2 axs
To find Rs take moments about point A,
o - 125 s = 2 (-11.27) s
ZM = 9000 x 55 + 27000 x 78 - 900 x 83 - 85R s hence,
A = 0 8 = 695 ft.
A4.10 GENERAL LOADS ON AIRCRAFT
To find Rs take moments about point (A) Airplane Load Factor = AirPl~e Lift
ZMA = 100,000 x 21 - 35000 x 9 + 38 Rs 0
111600 - 4100
Rs = 47000 lb. (2 wheels) = 14000 7.7
93500 lb.
T T
111,600 lb.
210"
14000
Solution: - Solution: -
s
s
V 733
Acceleration a~ - r - 2500 = 215 ft/sec
2 Fig. A4.l9 shows a free body of the air-
plane with the lift and inertia forces as found
or 214.5/32.2 = 6.68g (upward). in Problem 4.
The inertia force normal to the flight path The additional inertia force due to the
and acting down equals angular acceleration a = 4 rad/sec 2 • equals,
L = 111600 lb.
Ma = 99009 lb.
~
A4.11
~40-'i'.+..d'~"r-J~-:-:~_--1_----.-----~ ~
Inertia force due to a g equals,
~oo
Mra = 32.2 x 12 x 50 x 4 = 776 lb. (down ) 319000 lb. Fig. A4.20
Calculations of resultant load on pilot: - are for design loads, which in general are 1.5
times the applied loads. It would not be correct
x- = 372" to say that the wing deflections under the ap-
plied loads for these two High Angle of attack
conditions would be 2/3 the deflections shown in
the photograph since under the design loads a
considerable portion of the wing would be stressed
beyond the elastic limit of the material or into
the plastic range where the stiffness modulus is
0.93
180
= 32.2 x 12 x 40 x 0.93 = 17 lb.
-
F~ = Mxa = 32.2180x 12 x 372 x 0.93 = 161 lb.
Up until World War II practically all air- for applying the load on the wing when striking
planes were assumed as rigid bodies for struc- the air gust.
tural design purposes. During the war failure NACA Technical Note 2424 reports the flight
of aircraft occured under load conditions which test results on a twin-engine Martin transport
the conventional design procedure based on rigid airplane. Strain gages were placed at various
body analysis, indicated satisfactory or safe points on the wing structure, and strains were
stresses. The failures were no doubt due to read for various gust conditions for Which the
dynamic overstress because the airplane is not normal airplane accelerations were also recorded.
a rigid body. Then slow pUll-up maneuvers were run to give
Furthermore, airplane design progress has similar airplane normal accelerations. The wing
resulted in thin wings and relatively large had a natural frequency of 3.8 cps and the air-
wing spans, and in many cases these wings carry plane speed was 250 M.P.H. Two of the con-
concentrated masses, such as, power plants, clusions given in this report are: - (1) The
bombs, wing tip fuel tanks etc,. Thus the bending strains per unit normal acceleration
flexibility of wings have increased which means under air gusts were approximately 20 percent
the natural bending frequencies have decreased. higher than those of slow pull-Ups for all mea-
This fact together With the fact that airplane suring positions and flight conditions of the
speeds have greatly increased and thus cause tests, and (2) The dynamic component of the wing
air gust loads to be applied more rapidly, or bending strains appeared to be due primarily to
the loading is becoming more dynamic in char- excitation of the fundamental wing bending mode.
acter and thus the overall load effect on the These results thus indicate that air gusts
wing structure is appreciable and cannot be apply a air load more rapidly to a wing than a
neglected in the strength design of the wing. maneuver load giving the same airplane normal
acceleration for a commercial transport type of
General Dynamic Effect of Air Forces on airplane, and thus the dynamic strain effect on
Wing Loads. the wing is more pronounced for gust conditions.
Figs. A4.23, 24 and 25 show results of dy-
The critical airloads on an airplane are namic effect of air gusts on a large wing as de-
caused by maneuvering the airplane by the pilot termined by Bisplinghoff*. The results in these
or in striking a transverse air gust. A trans- figures show that dynamic effects tend to con-
port airplane does not have to be designed for siderably increase wing forces on some portions
sharp maneuvers producing high airplane accel- of the wing and decrease it on other portions.
erations in its job of transporting passengers,
thus the time of applying the maneuver loads is Fig. A4.23
considerably more than a fighter type airplane -600
pUlling up sharply from high speeds. .r Comparative Shear
1\ Distribution
Fig. A4.22 shows the result of a pull-Up :; -400
maneuver on the Douglas D.C.3 airplane at 180
M.P.H. relative to load factor versus time of
CIl
~
", ',- ~ -\
" r-, - - Dynamic Analysis
- - - Rigid Airplane
-200 Analysis
application of load. As indicated the peak I'
load of load factor 3.25 was obtained at the -.......::
No..
end of one second of time. o .2 .4 .6 .8 1.0
.5 1 1.5 2
6
0
)1
"- ~
-10
Pull-up of DC-3 Airplane at 180 mph. I%l ~ "-
f""-.. "-
The author estimates the natural frequency of 0
.2 .4 .6 .8 1.0
the D.C.3 wing to be around 10 to 15 cycles per
second, thus a loading time of 1 second against Fig. A4.25
a time of 1/10 or 1/15 for half a wing deflec- ~ c. Comparative Distri-
-3
tion cycle indicates that dynamic overstress ~ bution of Torque
sqould not be appreciable. In general, it can
be said that dynamic over-stress under maneu-
-2
~C' -1 -- - tl/l About Elastic Axis.
S
~ General Data:-
vering loads on transport airplanes is not as ~ 0 '" '" I
,/"",'
-- ~
Wing Span = 189 ft.
great as from other conditions such as air Eo-< 1 '" Gross wt. = 184000 lb.
t....
gusts or landing. 2 Airlane Vel. =260 mph.
o .2 .4 .6 .8 .10
Dynamic Effect of Air Gusts. Fraction of Semi-Span
The higher the air gust velocity and the * Report on an Investigation on Stresses in Aircraft Struc-
higher the airplane velocity, the less the time tures under Dynamic Loading. M. I. T. Publication.
A4.14 GENERAL LOADS ON AIRCRAFT
g
axial and torsional forces. However, the bend- 100=P
ing forces and the resulting beam stresses due 10 -1 :9
to bending of the beam are usually of primary It)
>"'1
A5.2 Statically Determinate and Statically Indeterminate Fig. A5. 2 a Fig.A5.3
Beams. 75 75
A beam can be considered as subjected to
known applied loads and unknown supporting re-
actions. If the distribution of the applied 100 = 1
known loads to the supporting reactions can be
determined from the conditions of static equil- :9~ CTI~100=P
ibrium alone, namely, the summation of forces
and moments equal zero, then the beam is con-
sidered as a statically determinate beam. How-
klH
la
::e 75
T
d >"'
la 75
ever, if the distribution of the known applied Fig. A5.4 Fig. A5.5
loads to the supporting beam reactions is in-
fluenced by the behavior of the beam material right side portion as a free body in equilibrium
during the loading, then the supporting reactions as shown in Fig. A5.2. For static equilibrium,
cannot be found by the statical equilibrium ~V, ~H and ZM must equal zero for all forces and
equations alone, and the beam is classified as a moments acting on this beam portion. Consider-
statically indeterminate beam. To solve such a ing ~V = 0 in Fig. A5.3: -
beam, other conditions of fact based on the
beam deformations must be used in combination ~V = 75 - 100 =- 25 lb. (1)
With the static equilibrium equations.
thUS, under the forces shown, the force system
A5.3 SheaL_and Bending Moment. is unbalanced in the V direction, and therefore
A given beam is subjected to a certain ap- an internal resisting force Vi equal to 25 lb.
plied known loading. The beam reactions to hold must have existed on section a-a to produce
the beam in static equilibrium are then calcu- equilibrium of forces in the V direction. Fig.
lated by the necessary equations of static equi- A5.3 shows the resisting shear force, Vi =
librium, namely: - 25 Ib, which must exist for equilibrium.
~V =0, or the algebraic summation of all verti- Considering ZM =
0 in Fig. A5.3, take
cal forces equals zero. moments about some point 0 on section a-a,
~H = 0, or the algebraic summation of all hori- ZM o =- 75 x 15 + 100 x 5 =- 625 in.lb.(2)
zontal forces equals zero.
ZM = 0, or the algebraic summation of all the or an unbalanced moment of - 625 tends to ro-
moments equals zero. tate the portion of the beam about section a-a.
A counteracting resisting moment M = 625 must
With the entire beam in static equilibrium, exist on section a-a to provide equilibrium.
it follows that every portion of the beam must Fig. A5.4 shows the free body with the Vi and
likewise be in static equilibrium. Now consider
the beam in Fig. A5.1. The known applied load Mi acting.
Now ~H must equal zero. The external
of P = 100 lb. is held in equilibrium by the two forces as well as the internal resisting shear
reactions of 25 and 75 Ibs. as shown and are
calcUlated from simple statics. (Beam weight is Vi have no horizontal components. Therefore,
neglected in this problem). Now consider the the internal forces producing the resisting
beam as cut at section a-a and consider the moment Mi must be such as to have no horizontal
A5.1
A5.2 BEAMS -- SHEAR AND MOMENTS
unbalanced force, which means that the resisting rOOO lb. 3°~1
1,500 lb. lb.
moment Mi in the form of a couple, as shown in ,1 ~,3 4,5 6,7 8,
Fig. A5.5, or Mi =
Cd or Td and T must equal C
~ 5'
6 '7
~
to make ,ZH .. O.
~
2 '3
The tendency of the loads and reactions 4' 5' 3'
acting on a beam to shear or move one portion of RA = 1110 lb. RB = 690 lb.
a beam up or down relative to the adjacent por-
tion of the beam is called the External Vertical Fig. A5.6
Shear, or commonly referred to as the beam Vert-
ical Shear and is represented by the term V. Calculations for Shear Diagram: -
From equation (1), the Vertical Shear at We start at the left end of the beam.
any section of a beam can be defined as the al- Considering a section just to the right of the
gebraic sum bf all the forces and reactions 500 lb. load, or section 1-1, and considering
acting to one side of the section at which the the portion to the left of the section, the
shear is desired. If the portion of the beam to Vertical Shear at 1-1 = ,ZV =..;; 500 (negative,
the left of the section tends to move up rela- down on left.)
tive to the right portion, the sign of the + 610 lb. + 610 lb.
Vertical Shear is taken as positive shear and
negative if the tendency is opposite. Or in ~OOlb.
other words, if the algebraic sum of the forces
~
is up on the left or down on the right side,
then the Vertical Shear is positive, and nega- _ 500 500 lb. - 390 lb. - 390 lb.
tive for down on the left and up on the right.
Fig. A5.7 (Shear Diagram)
From equation (2), the Bending Moment at
any section of a beam can be defined as the al- ~in.lb.
gebraic sum of the moments of all the forces
acting to either side of the section about the
section. If this bending moment tends to pro-
duce compression (Shortening) of the upper fib-
ers and tension (stretching) of the lower fibers
of the beam, the bending moment is classed as a
positive bending moment, and negative for the
reverse condition.
A5.4 Shear and Moment Diagrams.
In aircraft design, a large proportion of
the beams are tapered in depth and section, and
also carry a variable distributed load. Thus,
to design or check the various sections of such
beams, it is necessary to have a complete pic-
ture as to the value of the vertical shear and
bending moment at all sections along the beam.
If these values are plotted as ordinates from a
base line, the resulting curves are referred to
as Shear and Moment diagrams. A few example
Shear and Moment diagrams will be plotted, to
refresh the' students knowledge regarding these
diagrams.
Example Problem 1-
Draw a shear and bending moment diagram for
the beam shown in Fig. A5.6. Neglect the weight
of the beam.
In general, the first step is to determine
the reactions.
To find RB, take moments about point A.
ZMA =-4 x 500 + 1000 x 5 + 300 x 13 - 10RB = 0
hence RB .. 690 lb.
,ZV=- 500 + RA - 1000 - 300 + 690= 0
hence RA = 1110 lb.
A5.3
Section 8-8: Vx =
270 - lOx, and hence, the shear de-
ZV =- 300 (positive shear). creases at aconstant rate of 10 Ib./in. from 270
at A to 180 at C.
Fig. A5.7 shows the plotted values on the shear The vertical shear at section D, just to
diagram. the right of load is,
Calculation of the Moment Diagram. VD =
ZVleft =270 - 10 x 9 - 120 =60 up,
Start at section 1-1, and consider the or positive.
forces to the left only:
ZM =-500 x 0 0 = I--
9" -l120l/=P .
C.D w=10lb./m.
Since sections 2-2 and 3-3 are only a differen- At!!l!}!ll!)llli
tial distance apart, assume a section just above • 36
RA and consider the forces on the left side only: RA=270 RB=210
ZM =-500 x 4 =-
2000 in. lb. (Negative
270 lb. Fig. A5.9
VC=180 lb.
moment, because of tension in the top fibers).
Consider the section under the 1000 in. lb. load~ VD=60 lb.
ZM to left =-
500 x 9 + 1110 x 5 = 1050 in.
lb. (positive moment, compressing the top
t= 15"
f--6"
Shear Diagram -210 lb.
fibers).
Check by considering the forces to the right:
flg~
.n ....
o
in
0
....
.... ....
ZM right = 300 x 8 - 690 x 5 = - 1050 in. lb. ~~
JX 2 Vdx
The equation for the bending moment between
D and B (x greater than 9) is M2
Mx =RAx - P (X-9) -
wx 2
2""" (3 ) J M1
dM '"
x1
Thus, the area of the shear diagram between any
:: 270 x - 120 (x-9) _ 1~2 two points equals the change in bending moment
between these two points.
= 1080 + 150 x - 5x 2 - - - (4) To illustrate this relationship, consider
the shear diagram in example problem 2 (Fig.
At section C, x :: 9n , substitute in equation (4) A5.10). The change in bending moment between
the left reaction RA and the load is equal to
Me = =
1080 + 150 x 9 - 5 X 9 2 2025 in. lb. the area of the shear diagram between these two
(positive, compression in top fibers). points, or
At the point of zero shear, x :: 15 n • 270 + 180
2 x 9 :: 2025 in. lb. Since the bending
=
M 1080 + 150 x 15 - 5 x 15 2 :: 2205
moment at the left support is zero, this change
Thus, by sUbstituting in equation (2) and therefore equals the true moment at a section
(4) the moment diagram as plotted in Fig. A5.11 under the load P.
is obtained. Adding to this the area of the small tri-
angle between point D and the point of zero
A5.5 Section of Maximum Bending Moment. 60
The general expression for the bending mo- shear, or 1f x 6 = 180, we obtain 2205 in. lb.
ment on the beam of example problem 2 is from as the maximum moment. This can be checked by
equation (3); taking the area of the shear diagram between
the point of zero shear and point B =
2~0 x 21 :: 2205 in. lb.
Now, the value of x that will make MX a
maximum or minimum is the value that will make Example Problem 3.
the first derivative of Mx with respect to x Fig. A5.12 illustrates a landing gear oleo
equal to zero, or strut ADEO braced by struts BD and CEo A land-
ing ground load of 15000 lb. is applied through
the wheel axle as shown. Let it be required to
(5 ) find the axial load in all members and the shear
and bendIng moment diagram for the oleo strut.
Therefore, the value of x that will make Mx a
maximum or minimum may be found from the equation
RA - P - wx :: 0 HBT
B
But, observation of this equation indicates 10"
that the term Ra - P - wx is the shear for the
section at a distance x from the left reaction.
Therefore, where the shear is zero, the bending
moment is maximum. Thus, the shear diagram
---t 16"
which shows where the shear is zero is a con- Fig. A5.12
venient medium for locating the points of maxi-
___L
Pins at Points B,
mum bending moment. e, E. No Vertical 15000 lb. 1
Resistance at Point 16"
A5.6 Relation Between Shear and Bending Moment. A.
Equation (5) can also be written ~ :: V, n,-----'L
"0"
since the right hand portion of equation (5) is
equal to the shear.
Which means that the difference dM between To find Vc take moments about point B,
the bending moments at two sections that are a 2MB =- 15000 x 0.5 x 42 + 15000 x 0.866 x 5.77
+ 20.77 Vc :: O.
distance dx apart, is equal to the area Vdx
under the shear curve between the two sections. hence, Vc :: 11550 lb.
Thus, for two sections x 1 and x 2, The axial load in member CEtherefore equals
A5.5
~
Axial Load
-13000 lb. Diagram loads to the beam at points C, D, F and E.
W/00/& '--_-=-=Compression
13340
~"""",,_~';::am
1
T5
r}'"
~ ~OO, ~<I. <: >
2 " ; f J 500
~-840 8" 8" I /'J) 5"
-7500 lb.
Bending
M
= 3000
HC=50
C=500
L HO=500
MO=4000
H -500
F-
L
E
~'10'1.1
\\"E'"
VF=1000 VE=7071
~Oiagram Moment
Fig. a Fig. b Fig. c
Fig. A5.15
Fig. d
The load of 1000 at 45° and applied at point E" have maximum peak moments without the Vertical
will be referred to point E the centerline of Shear passing through zero. To illustrate
beam. Fig. d shows the reaction at E due to the this fact, consider the beam of Fig. A5.20,
load at E'. The reaction at B should also be namely, a simple supported beam with an ex-
referred to the beam centerline. Fig. A5.16 ternally applied couple moment of 10 in. lb.
shows the beam with the applied loads at points magnitude at point C the center point of the
C D E' F and B'. Figs. A5.17, 18 and 19 show beam. The shear and bending moment diagrams
the axial load, vertical shear and bending mo- are as indicated and a maximum bending moment
ment diagrams under the beam loading of Fig. occurs at C but the shear diagram does not pass
A5.16. through zero.
707.1 1000
MC=10"#
4285
I
.999.J-r 5788.7
forces equal to 100 lb. each and an arm between
them of 0.1 inch as illustrated in Fig. A5.21.
-207.1
------'-----t 1
1-1 lb.
Fig.A5.19
1CI ======u Shear Dia.
Fig. A5.21 99
The shear diagram is determined in the same
manner as explained before. The applied exter- The shear diagram is as shown in Fig. A5.21 and
nal couples do not enter into the vertical shear now passes through zero under each of the couple
calculations. The bending moment diagram can be forces. Thus if we assume the couple moment has
calculated by taking the algebraic sum of all a dx arm the shear to the right of C is one lb.
couples and moments of all forces lying to the and then changes to some unknown negative value
one side of a particular section. If it is de- and then back to one lb. positive as the dist-
sired to use the area of the shear diagram to ance dx is covered ih going to the left. Thus
obtain the bending moments, it is necessary to the shear goes to zero twice in the region of
add the couple moments to the shear areas to ob- point C.
tain the true bending moment. For example, the
bending moment just to the left of point E will A5.7 Moment Diagrams as Made up of Parts.
be equal in magnitude to the area of the shear In calculating the deflection of statically
diagram between C and E plus the sum of all ap- determinate beams (See Chapter A7) and solving
plied couple moments between C and E but not in- statically indeterminate structures (See Chapter
cluding that at E. A8), the area under the bending moment curve is
To illustrate the calculations are: - required, thus it is often convenient to treat
(- 500 x 5) + (707.8 x 10) 4578 in. lb. (from = each load and reaction as a separate acting
force and draw the moment diagram for each force.
area of shear diagram).
(3000 - 4000) =-
1000 in. lb. (from sum of The true bending moment at a particular point
will then equal the algebraic summation of the
couple moments).
Thus bending moment at E .. 4578 - 1000 = ordinates of all the various moment curves at
3578 in. lb. left this particular point or adding the various
separate moment diagrams will give the true
The bending moment at Eright will equal that at bending moment diagram. Figs. A5.22 and A5.23
Eleft plus the couple moment at E or 3578 + 707 illustrate the drawing of the bending moment
:: 4285 in. lb. diagram in parts. In these examples, we start
from the left end and proceed to the right end
The student should realize that when couple and draw the moment curve for each force as
moments are applied to a beam it is possible to though the beam was a cantilever with the fixed
A5.7
Due to P e
P e =lO* -=:::::::::J -50
f-5' 5,,-1 Due to P~
-601 1-60
r Fig. A5. 28
i1nli~O~
P,,=lO*
Final Bending
-60 Moment Diagram
-6,~ -+....!£i Free 60 -50 90 (Tension on inside
of frame is posi-
Fig. A5.25 tive moment)
V F~-"
~~ t- a ~v Fig. A5. 32 Fig. A5. 33
M~c~nM"
(1) (2)
Fig. A5.31
Then from Fig. A5.33 we can write for the
resultant forces and moment at point (2) at
of the beam portion between stations 1 and 2. station 2: -
The Vertical Shear V~ at station 1 equals the
summation of the forces to the left of station 1 V" =V~ + F~-"
and M~ the bending moment at station 1 equals
the algebraic sum of the moments of all forces
lying to left of station 1 about station 1.
Now considering station 2: - The Vertical
Shear Va = V~ + F~_", or stated in words, the
Shear V" equals the Shear at the previous sta- Having the resultant forces and moments for a
tion 1 plus the algebraic sum of all forces F given point on a given station, it is usually
lying between stations 1 and 2. Again consider- necessary in finding beam stresses to resolve
ing Fig. A5.31, the bending moment M" at station the forces into components normal and parallel
2 can be written, M" =
M~ + V~d + F~_"a, or
to the beam cross-section and also transfer
their location to a point on the neutral axis
stated in words, the bending moment Ma at sta-
tion 2 is. equal to the bending moment M~ at a of the beam cross-section.
previous station 1, plus the Shear V at the For example Fig. A5.34 shows the resultant
previous station 1 times the arm d, the dist-
ance between stations 1 and 2 plus the moments
of all forces lying between stations 1 and 2
about station 2.
A5.9 Equations for Curved Beams.
Many structural beams carry both longitud-
ional and transverse loads and also the beams
may be made of straight elements to form a frame
~ S
Mo=M~-Ne
or all beam elements may be curved to form a Fig.A5.34 Fig.A5.35 Fig.A5.36
curved frame or ring. For example the airplane
fuselage ring is a curved beam subjected to
forces of varying magnitude and direction along
its boundary due to the action of the fuselage
skin forces on the frame. Since the complete
bending moment diagram is usually desirable, it forces and moment at point 1 of a beam cross-
is desirable to minimize the amount of numerical section. They can be resolved into a normal
work in obtaining the complete shear and bending force N and a shear force S plus a moment M~ as
moment values. Fig. A5.32 shows a curved beam shown in Fig. A5.35 where,
loaded with a number of different vertical loads
F and horizontal loads Q. Fig. A5.33 shows the N = H cos a + V sin a
beam portion 1-2 cut out as a free body. H~ S =V cos a - H sin a
represents the resultant horizontal force at
station 1 and equals the algebraic summation of Later on when the beam section is being de-
all the Q forces to the left of station 1. V~ signed it may be found that the neutral axis
represents the resultant vertical force at
station 1 and equals the sum of all F forces to
lies at point °
instead of point 1. Fig.
A5.36 shows the forces and moments referred to
left of station 1, and M~ equals the bending point 0, With Mo being equal to M~ - Ne.
moment about point (1) on station 1 due to the
moments of all forces lying to the left of
point 1.
A5.9
airplane as a rigid body. Unit load analyses ratio or the spanwise variation of the lift
are also made for angular accelerations of the coeff!cient CL in terms of a uniform distribu-
airplane which can also occur in flight and tion CL • In this example we have taken this
landing maneuvers. ratio as unity since we have no wind tunnel or
The subject of the calculation of loads on aerodynamic calculations for this wing relative
the airplane is far too large to cover in a to the spanwise distribution of the lift force
structures book. This subject is usually cover- coefficient. In an actual problem involving an
ed in a separate course in most aeronautical airplane a curve such as that given in Fig.
curricula after a student has had initial A5.36 would be available and the values to
courses in aerodynamics and structures. To il- place in Column 3 of Table A5.1 would be read
lustrate the type of problem that is encountered from such a curve. Column (2) gives the wing
in the calculation of the applied loads on the chord length at each station. Column (4)
airplane, simplified problems concerning the gives the wing running load per inch of span
wing and fuselage will be given. at each station point. Since a total unit
load of 17760 lb. was assumed acting on the
AS.12 Example Problem of Calculating Wing Shears and half wing and since the wing area is 17760 sq.
Moments for One Unit Load Condition. in., the running load per inch at any station
Fig. A5.37 shows the half wing planform of equals the wing chord length at that station.
a cantilever wing. Fig. A5.38 shows a wing In order to find shears and moments at the
section at station O. The reference Y axis has various station points, the distributed load is
been taken as the 40 percent chord line which now broken down into concentrated loads which
happens to be a straight line in this particular are equal to the distributed load on a strip
wing layout. and this concentrated strip load is taken as
acting through the center of gravity of this
distributed strip load. Columns 5, 6, and 7
show the calculations for determining" the
(~pz) strip loads. Column 8 shows the lo-
cation of the ~Pz load which is at the centroid
of a trapezoidal distributed load whose end
values are given in Column (4). In determin-
ing these centroid locations it is convenient
to use Table A3.4 of Chapter A3.
The values of the shear Vz and the mo-
ment Mx at each station are calculated by the
method explained in Art. A5.8. Columns 9, 10,
11 and 12 of Table A5.1 give the calculations.
For example, the value of Mx = 9884 in Col-
~ IZ Ref. Axis umn (12) for station 220-equals 2436, the Mx
moment at the previous station in Column (12)
~S% s%l--....-r40% line plUS 4908 in Column (10) which is the shear
aerodYll!!,mic I at the previous station (230) times the dist-
center --a.c.- 0 -"Ref. - ance 10 inches plus the moment 2540 in Column
I I?/
I
96"
Ref. Axis Fig. AS. 38
(9) due to the strip load between stations
230 and 220, which gives a total of 9884 the
value in Column (12).
The strip loads ~Pz act through the
The total wing area is 17760 sq. in. For aerodynamic center (a.c.) of each airfoil strip.
convenience a total unit distributed load of Column (13) and (14) give the x arms which is
17760 Ibs. will be assumed acting on the half the distance from the a.c. to the reference Y
wing and acting upward in the Z direction and axis. (See Fig. A5.38). Column 15 gives the
through the airfoil aerodynamic center. The M moment for each strip load and Column 16
spanwise distribution of this load will be ac- the My moment at the various stations which
cording to the (C L ) lift coefficient spanwise
distribution. For simplicity in this example equals the summation of the strip moments as
it will be assumed constant. one progresses from station 240 to zero.
Table A5.1 shows the calculations in table Fig. A5.39 shows the results at station
form for determining the (Vz ) the wing shear in (0) as taken from Table A5.1.
the Z direction, the bending moment Mx or mo- i51
ment about the X axis and My the moment about
the Y axis for a number of stations between the V z=17760/
sJ¢iy;~_o--_x
wing tip station 240 and the centerline station
O. x
Column 1 of the table shows the number of
stations selected. Column 3 shows the CL/
CL CMy=214071 Fig. AS. 39
A5.11
TABLE A5.1
CALCULATION OF WING SHEAR Vz AND WING MOMENTS M" AND My
DUE TO TOTAL UNIT DISTRIDUTED HALF WING LOAD OF 17760 LBS.,
ACTING UPWARD IN Z DIRECTION AND APPLIED AT AERODYNAMIC
CENTERS OF WING SECTIONS. (See Figs. A5.37, 38 for Wing Layout)
1 2 3 4 5 6 7 8 9 10 11 12 13 14 15 16
......
:g -aU .s
8~"
0
c: ;g 8
.
0
CL .s .s 'c:" N "
I:! + 0
... ~'"....
;-
a:a
e~
... r.. ...
..c:
~
I CL
Ratio,
og~
O'C:
..::I ..c:",•
§
~
~
iii
.c
d-
'" 0
I:
p,
<l~
og...:
It>
'"
uo.
0-
"'/li
-a
g
00
-a
g
'"0" ~~
"t" . ",u
u- '"
.9'
" u
1;;3
...."
Q>
. -a H 0
NS
p,-
g '" '" '" Assumed OJ)
::rr C> ~~~ ~g <l~
....'0
• Q)
00 0.;':
c:u~ ..::I bIlu.c '" > - c:
Q", ..::I~ 8 N "
-ct:" :al:.. " '"
°:alli
0I~
N8iO
.5l;
~12i
J B,s
" u
-c..c: Unity. .!'LS..::I
§"-:'
~
"-c ·t II IJ'" 0. ...
:':p'-c
<1
:;<l ~-a
<lu N
'0
• II
N
• ~'S ai-c ·a
H
N
.,..
•
II 8 ....
".
... Gl os:: <J II cIj
~N
P, >< '0 u P,'0 ~fI)"O
~i5~ UU ~~~ <.3
'" 01
<llll ~ II ,,'a~
"
N~
When the time comes to deSign the structural forces are the major forces. For the body load
make-up of a cross-section to withstand these analysis the direct air pressures are secondary,
applied shears and moments, the structural de- the major forces being of a concentrated nature
signer may Wish to refer the forces to another in the form of loads or reactions from units
Y axis as for example one that passes through attached to the body, as the power plant, wing,
the shear center of the given section. This landing gear, tail, etc. In addition, since the
transfer of a force system With reference to an- body usually serves as the load carrying medium,
other set of axes presents no difficulty. important forces are produced on the body in re-
sisting the inertia forces of the weight of the
SHEARS AND MOMENTS ON AIRPLANE BODY interior equipment, installations, pay load etc.
A5.13 Introduction.
As in the case of the wing, a large part
The body of an airplane acts essentially as of the load analysis can be made without much
a beam and in some conditions of flight or land- consideration as to the structural analysis of
the body. The load analysis of an airplane body
ing as a beam column which may be also subjected involves a large amount of calculation, and
to twisting or torsional forces. Thus to design
thus the treatment in this chapter must be of a
an airplane body requires a complete picture of simplified nature, and is presented chiefly for
the shearing, bending, twisting and axial forces the purpose of showing the student in general
which may be encountered in flight or landing.
In the load analysis for wings, the direct air how the problem of load analysis for an air-
plane body is approached.
A5.12 BEAMS -- SHEAR AND MOMENTS
A5.14 Design Conditions and Design Weights. A5.15 Body Weight and Balance Distribution.
The airplane body must be designed to with- The resisting inertia forces due to the
stand all loads from specified flight conditions dead weight of the body and its contents plays
for both maneuver and gust conditions. Since an important part in the load analysis for the
accelerations due to air gusts vary inversely airplane body. When the initIal aerodynamic
as the airplane weight, it customary to analyze and general layout and arrangement of the air-
or check the body for a light load condition for plane is made, it is neceBsary that a complete
flight conditions. In general, the design weight and balance estimate of the airplane be
weights are specified by the government agen- made. This estimate is usually made by an en-
cies. For landing conditions, however, the gineer from the weight control section of the
normal gross weight is used since it would be engineering department who has had experience
more critical than a lightly loaded condition. in estimatinp, the weight and distribution of
The general design conditions which are airplane units. This estimate which is pre-
usually investigated in the design of the body sented in report form gives the weights and
are as follows: (e.g.) locations of all major airplane units
Flight Conditions: or installations as well as for many of the
minor units which make up these major airplane
H.A.A. (High angle of attack) assemblies or installations. This weight and
L.A.A. (Low angle of attack) balance report forms the basis for the dead
LL.A.A. (Inverted low angle of attack) weight inertia load analysis which forms an
I.H.A.A. (Inverted high angle of attack) important part in the load analysis of the air-
plane body. The use of this weight and balance
The above conditions generally assume only
translational acceleration. In addition, it is estimate will be illustrated in the example
sometimes specified that the forces due to a problem to follow later.
certain angular acceleration of the airplane A5.16 Load Analysis. Unit Analysis.
about the airplane e.g. must be considered. Due to the many design conditions such as
The body is usually required to withstand
special tail loads both symmetrical and unsym- those listed in Art. A5.14, the general pro-
metrical which may be produced by air gusts, cedure in the load analysis of an airplane body
engine forces, etc. Also, the body should be is to base it on a series of unit analyses.
checked for forces due to unsymmetrical air The loads for any particular design condition
loads on the wing. then follows as a certain combination of the
unit results with the proper multiplying fac-
Landing Conditions: tors. A simplified example problem follows
In general, the body is investigated for the which illustrates this unit method of approach.
following landing conditions. The detailed re- A5.17 Example Problem lllustrating the Calculation of
qUirements for each condition are given in the Shears and Moments on Fuselage Due to Unit
government specifications for both military and Load Conditions.
commercial airplanes.
Fig. A5.40 and A5.41 shows a layout of the
Landplanes: Level landing. airplane body to b e used in this example prob-
Level landing With side load. lem. It happens to be the body of an actual
Three point landing. airplane and the wing used in the previous ex-
Three point landing with ample problem was the wing that went with the
ground loop. airplane.
-+!~
Nose over or turn over
condition.
.~ 1:,1,y-'~--1~ne"=~
Arresting. (Usually for only
Navy Carrier based air- 1
planes) • N'
QI
I 11'2"
Seaplanes or Boats: C.l
c:
Step landing With and Without ~ II
angular acceleration.
Bow landing. ~J, Fig.A5.40
-.1
!1
Stern landing. I~.-------- 27'2"
Two wave landing.
Beaching conditions.
Catapulting conditions (Navy
airplanes).
SpecIal Conditions or Forces:
Towing of airplane.
I
Front View I
Body supercharging. I I
~47., ' Fig. A5.41 I
20' - 0 " ' - - - - - - _ .
A5.13
2 STATION
o 11 50 80 120 170 200 230 011 120 170 200 230 260 290
I
I I 1 I 'I I I
I
I I I I I
_ ---
I
~ +5
I
I
Fig. A5.42. Location of weight items of Table A5. 3. Fig. A5.46. Weight items from Table A5. 3 acting in X direction.
o 11 50 80 120 170 200 230 290 011 50 80 120 170 200 230 290
I I I
I I I I 'I I I I I i I I I
I I I I I I I I 1.,.--1_-'-...1 I I
-~ -~l----J --fg-k
...-f
...-f
M
...-f
Oan
N en
lit>co "" ...
il9
t -
I
... 0
L4"
h(ff ~,; L
8
65 ~52
-
-1-30 LI0 .£..:10-<:6
----r6,-; rp.- fJ.,,~,,-
I I
•5 U
I
I
I I I I I II! I I I I
Fig. A5.43. Weight items of Table A5. 3 acting in Z direction. Fig. A5. 47. Vertical distribution of fuselage dead weight.
011 120 170 200 230 290 50 80 120 170 200 230 260 290 315
I I ,
I I I I I I I I I I I I
I I I .,.--1_-'-...1 I I
I I
--l·---r--t· -+-!
1
I I
"
9# 520#" 312#" 30#" 10'# 10"# 3"# 0#
19# 80# 78# 65# 30# 9# .&80
3 # W4#
--e-65# -.~5# t-30~10~1~i;)-0#
I I I I I
I I ! I I I I
Fig. A5.44. Results of fuselage weight distribution to stations. Fig. A5.48. Fuselage weight referred to X axis plus couples.
120 170 200 230 290 011 50 80 120 170 200 230 260 290 315
I I
I I I I I I I I I I I I I
.;r---'I_--'-......I I I I I I.;r---'I_--'-......I I I I
19 8 758 682 1650 490 10 27 --·-2j4?
:3 -C..,.388G-307-(~409-(?-311G-76G-I0 (-+21 (1):]~O
I I I I
I I I! I I I
Fig. A5.45. Final weight distribution to station points. Fig. A5.49. Final weight distribution in X direction referred
to X axis plus proper couples.
A5 15
shows how the dead weight of 350* was distribu- manner as to have the same resultant e.g. 10-
ted to the various station points considering cation as the original weight system which is
the weights to be acting in the Z direction. illustrated in Fig. A5.46. Fig. A5.47 illus-
Table A5.4 shows the results of this sta- trates how the fuselage weight distributed
tion point weight distribution for the weight system as shown in Fig. A5.44 is distributed
items of Table A5.3. The values in the hori- in the vertical direction at the various
zontal rows opposite each weight item shows the station points so that the moment of this sys-
distribution to the various fuselage stations. tem about the X axis is equal to that of the
The summation of the weights in each vertical original fuselage weight of 350#. For con-
column at each station point as given in the venience, these distributed fuselage weights
third horizontal row from the bottom of the can be transferred to the X axis plus a moment
table gives the final station point weight. as shown in Fig. A5.48.
These weights are Shown in Fig. A5.45 for Table A5.4 shows the vertical distribution
weights acting in the Z direction. The moment of the various items at the various station
of each total station load about the Z axis is points. The bottom horizontal row gives the
given in the second horizontal row from the moment about the X axes of the loads at each
bottom of Table A5.4. The summation of the station point, Which equals the individual
moments in this row must equal the total WX loads times their Z distances. The summation
moments of Table A5.3 or 219700"#. This check of the values in this horizontal row must equal
is shown in the last vertical column of Table the total wz moment of Table A5.3. This check
A5.4. is shown at the bottom of the last vertical
The distributed system must also be distri- column. Fig. A5.49 shows the results as given
buted in the Z or vertical direction in such a in Table A5.4 for the weight distribution in
the X direction.
TABLE A5.4
,
.
I Powerplant group 1100 874 0 226 0 1100
Fuselae::e group "00 .0 -4 ',. -0 00 0 0 0 ou >u >u 0 '.0 "00
4 a . eroup
5 I Surface contro s
10
85
'" ,4 "9 '4 110
73 - 4 - 4 85
6 Electrical s stern 130 82 4 48 4 130
•
9
Ta' whee l 2:roup
Furnishiruzs
"5
220 76 5 100 44 5
13 -10 22 -10
220
10 Radio 125 79 10 46 10 125
II Pilot 00 76 4 124 4 200
12 Student 200 105 4 95 4 200
Total w 2555 893 .02 388 .02 307 2.47 409 1. 67 311 5.3 76 6.45 10 1.0 21 13lO6 118 19.1 22 -10 2555
Horizontal Moment = Z wx = 9820 19390 24550 49100 52800 15200 2300 5460 34200 6920 219700
Vertiea! Moment = Z wz= 19 8 758 682 1650 490 10 274 2249 - 220 5920
x : distance of station from z reference axis which is 5" forward of propeller.
z : distance above or below thrust line or x axis.
A5.18 Unit Analysis for Fuselage Shears and Moments. (1) Unit acceleration or load factor in Z
Since there are many flight and landing direction and acting up.
conditions, considerable time can be saved if a
unit analysis is made for the fuselage shears, (2) Unit acceleration or load factor in X
axial and bending forces. The design values in direction and acting forward.
general then follow as a summation of the values (3) Unit tail load normal to X axis acting
in the unit analysis times a proper multiplica- down.
tion factor.
The loads on the fuselage in general con- Unit analyses are also usually carried out
sists of tail loads, engine loads, wing re- for engine thrust and engine torque, side load
actions, landing gear reactions if attached to on tail and angular acceleration, but to keep
fuselage and inertia forces due to the airplane the example calculations from becoming too
acceleration which may be due to both transla- lengthy only the above 3 unit conditions will
tional and angular acceleration of the airplane. be carried out in detail. The others will be
For simpl1city, these loads can be resolved into discussed in detail in later paragraphs.
components parallel to the Z and X axes.
To illustrate the unit analysis procedure, Solution for Unit Load Factor in Z Direction.
a unit analysis for our example problem will be
carried out for the following unit conditions: Fig. A5.50 shows the dead weight loads
A5.16 BEAMS -- SHEAR AND MOMENTS
o 161
230 ~ - 10 171 - 4830 - 9580
RF =1780 lb., RR = 775 lb. 1----+---+----+- 30 - - f - - - - + - - - - /
o 171
200 ~ -
- 5130
76
-14710
247
Table A5.5 gives the calculations for the 1 - - - - 4 - - - + - - - - - + - - 30 --+----+-----1
fuselage shears and bending moments at the var- o 247
ious station points. 170 ~ - 311 558
- 7410 -22120
I----+---+----+-- 50 -+----1------1
Sta.O 11 50 80 120 170 200 230 260 290 31,5 120 ~
o 558
-27900 -50020
- 409 967
I:
I
:
I
i
I
I I
I
I
I
I I
I
:
o 967
4--+----+-----1
116 ~
I I 1 1
- 3868 -53888
I I I,A'\ I \ I I) 775 192
I ~V \ 22)
+--7619 118 ~ o
I.
I
893 388 307
-~-+-----t--J:: -~.
17"
409
20"
311
~- +-
21
.+-- '- 80 : - 307
192
499 - 6912 -60800
.1 ' - - ~_ • -----_0::--_-"-
1------1----1-----+--- 7 --+----+-----1
~I -l-- --,~
73 ~
o 499
- 3493 -64290
... RH=O 1-" 43" --I, 1780 -1281
P=:I RF=1780 RR=775 Fig. A5.50 1---+---+----+- 23 - - f - - - - + - - - - /
I o -1281
STA.73 STA.116 50 ~ +29463
- 388
-34827 -893
1-----1----+-----+- 39 --+----+-----1
Solution for Unit Load Factor in X Direction. o - 893
11 ~ - 893 o +34827 o
Fig. A5.51 shows the panel point dead (3) Up on left and down on right side of a section is
weight distribution for loads acting in the X positive shear.
direction and aft, as taken from Table A5.4 or (6) Tension in upper fuselage portion is negative bending
Fig. A5.40. To place the fuselage in equili- moment.
brium the wing reactions at points (A) and (B) (1) + refers to aft side of station.
will be calculated. - refers to forward side of station.
* M = M at previous station in col. 6 plus !:J.M in col. 5.
ZFx = 2555 - RH = 0, hence RH = 2555 lb. (forward)
Take moments about point (A)
11 50 80 120 172 200 230 260 290 315
rnA = 2555 x 17 + 5920 - 43 RR = 0, oI I I I I I I
, I
I
hence RR m 1147.8 (up) III I I
I
I :
!,,~A<--'-I----+--.\
I I
I I
I I
I
I
I
(5920 equals the sum of the couples from Table 19 I Y8758 682 1650 490 10 274 2Z49
A5.4. ," (~93~,,~388(-307 (-409 (-311(-76 ~10 (-21 C-1lB~22
~ ----fB.
H=2555 220
ZF z =
1147.8 - RF 0, = I ,'!::--A
43" -tOj' ---
hence RF =
1147.8 lb. (down) i RF=1147.'8 IRR=1147.8 Fig. A5.51
STA.73 STA.116
Table A5.6 gives the calculations for the shears,
moments and axial loads for the loading of Fig.
A5.5l.
A5.17
0 0
Load or V = shear lIX = dist. LIM Moment
315 + 0 0
22 0
22
0
220 0
220 sta. Reaction =~ w between
= VlIx in. lbs.
25 w lbs. lbs. stations
290 + 0 22 0 220
0 0 0
118 140 - 2249 - 2029
30 + 0 0 0
315 0
260 + 0
21 0
140
161
0
-
0
274 0
- 2029
- 2303
- 0 0 0
30
25
230 + 0 161 0 - 2303 + 0 0 0
0 0
- 0 290 0
10 171 10
30
- 2313 - 0 0 0
0 171 0 - 2313 12.5
200 + 0 0 0
76 247 - 490 - 2803 + 0 0 0
272.5 0
170 + 0 247 0
30
- 2803
- -100 100 0
311
0
558 0
- 1650
0
- 4453 17.5
50 + 0 100 - 1750
260 - 1750
120 + 0
- 409 0
558
967 0
-
0
682 0
- 4453
- 5135
- 0 100 - 1750
4 30
116+ 0 967 - 5135 + 0 100 - 4750
0 230 - 3000
0 1147.8 967 -1147.8 0
36
- 5135 - 0 100 - 4750
0 0 967 -1147.8 41320 36185 30
80 +
307 0 1274 -1147.8 - 758 0 35427 + 0 100
- 3000
- 7750
200
73 + 0 0 1274 -1147.8 0
7
8035 43462
- 0 100 - 7750
2555 -1147.8 -1281 0 -43435 0 27 30
23 + 0 100 -10750
170 - 3000
50 :
388
0
0
-1281
- 893 0
8
0 0
27
19 - 0 100
50
-10750
39
11 + 893
0
- 893
0
19
0
19 + 0 100 -15750
120 - 5000
. -
0 0 0 0
0 100 -15750
: 17 x (2555) : 43435 4
to refers to aft side of station + 0 100 -16150
Col. 1 - refers to forward side of station 116
- 475.6 -375.6 - 400 -16150
Px is plus for tension in -fuselage.
(Col. 9) M = M at previous station in Col. 9. plus II MI of col. 8 plus 36
lIM2 of col. 8 + 0 -375.6 - 2629
80 13521
- 0 -375.6 - 2629
7
Solution for Unit Horizontal Tail Load Acting Down. + 0 -375.6 0
73 2629
- -375.6 0 0
The fuselaga shears and moments will be 23
computed for a unit tail load of 100 lb. on the + 0 0 0
50
tail acting in the Z direction, with balancing - 0 0 0
39
reactions at the wing attachment points. The + 0 0 0
center of pressure on the horizontal tail is at 11
- 0 0 0
station 277.5. Fig. A5.52 shows the fuselage
loading. To find wing reactions at (A) and (B): (Col. 6) =M =M at previous station in Col. 6 plus
1I M in Col. 5.
ZMA = 100 x (277.5 - 73) - 43 RR = 0,
hence RR = 475.6# (up)
CALCULATION OF APPLIED FUSELAGE SHEARS,
- - 100 + 475.6 - RF - 0,
~Fz
MOMENTS AND AXIAL LOADS FOR A SPECIFIC
hence RF = 375.6# (down)
FLIGHT CONDITION.
Table A5.7 gives the detailed calculations
for the shears and moments at the various sta- Using the reSUlts in Tables A5.5, A5.6, and
tion points. A5.7, the applied shears and moments for a given
o 11 50 80 120 170 200 230 flight condition follow as a matter of propor-
II I I , I I I tion and addition. To illustrate, the applied
I I values for one flight condition will be given.
I' I I I : It will be assumed that the aerodynamic
II L.-r--t-------:--.,.I I I
calculations for this airplane for the (H.A.A.)
high angle of attack condition gave the follow-
ing reSUlts, which the student will have to
accept without knowledge of how they were
obtained.
RF=375.6 RR=475.6
Fig. A5.52
STA.73 STA.116
A5.18 BEAMS -- SHEAR AND MOMENTS
TABLE A5.8
L
Col. 3 - 1. 333 x values in column 5 of Table A5.6.
Col. 4 - -1.10 x values in column 3 of Table' AS. 7.
Col. 5 - column (1) .. column (2) .. column (3).
Col. 6 - 6 x values in column 6 of Table A5.5. Landing with Angular Accelerati~g
Col. 7 - 1. 333 x values in column 9 of Table AS.6.
Col. 8
Col. 9
-
-
1.10 x values in column 6 of Table A5.7.
columns (6) • (7) • (8).
In a level landing condition, it is some-
Col.IO - 1. 333 x values in column (4) of Table A5.6. times specified that the horizontal component
of the ground reaction must be a certain pro-
A5.19 Example of Fuselage Shears and Moments for
portion of the vertical component, which causes
Landing Conditions. the line of action of the ground reaction R in
Fig. A5.53 illustrates the airplane in a Fig. A5.53 to not pass through the c.g. of the
level landing condition. The ground reaction is airplane, which creates an external pitching
assumed to pass the center of landing gear wheel moment on the airplane. This moment is us-
and e.g. of airplane. The fuselage shears, mo- ually balanced by the inertia forces due to
ments and axial loads are reqUired when the the angular acceleration produced by the un-
vertical ultimate load factor is 7. (Gross balanced moment about the C.g. The shears and
weight = 4300#). moments on the fuselage due to t his external
moment could be found as explained in Art.
SOLUTION: A5.20.
The vertical or Z component of the ground A5.20 Inertia Loads Due to Angular Acceleration.
reaction R is specified as 7 load factors which In some of the flying conditions, it is
equals 7 x 4300 =
30100.. One half of this is sometimes specified that the airplane must be
acting on each wheel. subjected to an angUlar acceleration as well as
The horizontal or X component of R is 30100 translational acceleration. This angular ac-
tan 23° =
425 x 30100 12800# and acting aft. = cele~ation of the airplane produces inertia
The horizontal load factor on airplane forces which must be calculated if the air-
equals 12800/4300 = 2.98. plane is to be treated as a body in static
A5.19
=-wgra' =-My
260 21 13.06 260 22.46 172.50 30270 635000 22.5 - 2.9
F but a Iy
g 290
315
118 19.10
22 -10.0
290
315
28.50 202.50
- 0.6 227.50
31085
51800
3670000
1140000
148.3
31. 2
-20.9
- 0.1
170 230
III. Fig. A5.62 shows the plan form of a canti-
11 50 80 120
I I I I lever wing. Assume a constant normal
I I I distributed load on the surface equal to
I I
I I I 50 lb./sq.ft. Write expressions for shear
and bending moment on wing and find values
52. at 25, 100, 150 and 200 inches from end.
426
~O"
I
I f..g
100"
IX:_--"p=la:::n=fo=-r-m---- .~ 30" Fig. A5. 62
Fig. A5.54 L L -- _\20"
A5.22 Problems
I
1 - < - .- 200"
- - - .
Planform
(5
o
IX: Leading Edge
150"
x
1. 01 t 1 f f t t f~ Relative Span-
~O"~ wise Distribution
1
1001 hl::o-i~l.~t;oo
20 100
r1~~~------~~~~~---
n
·n
" 44 T
~'/l"I\~" ]~." 1" 60 27 PLAN VIEW
r-lO"-1
I1 k!Iinge Front Beam
60a 60b 60 c
4 '------ --J
t400#
II Fig. A5. 64
30~M8" 100
i 11
(a) 16"
L
1--16"~
Fig. A5. 61
A5.21
Table A5. I
Tan~
x=L
2"
= M2 - Ml cos..!:..
M Side Load M j j
Mmax = Ml
cosx
---r-~
II o
sin L M1 sin..!:.
T j
T
p1;x::::j L
Uniform Side Load. No
(-r-1)]
iPLU ~ l-p
End Moments w#/in. wj2 [cos
ill x: .5 L
P t ttttf sink
j
T
Triangular Loading. No (NOTE A)
End Moments ----.---rT" J. o To obtain Maximum Moment, compute
VI ~TTII w#/in. moment at 3 or 4 points in span. Draw a smooth
l-=x==:--l~~==ffP
p-;:::' curve thru plotted results.
L---
Triangular Loading. No End
1. Moments wj2(l-E (See Note A)
VII
w#/~~ L
L/i in 6L
Radians
Sin L /j S£ L I COS L Ii Cos Ii Tan L /j Ta~ L /j L/i in
J Radians
(l
1.....
.
O. 00l3:r"=-
= - -.0,681:641"" O. 73169 _ ..
..
+t--J_""",=-/I!
. 9220: ==
0. 01959-·
::;-
~0~6
----
iii
(J.00961L
0:00963
03l6639:.
0.96377
0:002li2C=
0: 00271
i 0.2'1670"
0.01080
0.77
889:F-
a:-=28=
0.007~
!! 69614" .. O. 00714~
----0=8 _
0.71791
L/j in
Radians
Sin L/. Si~ L/. Cos L/. co~L/ Tan L/. Ta~ L/ L/j in
Radians
Sin L/. Si~ L/ Cos L/
j
Co~ L/. Tan L/
j
Ta~ L/.
J J J J J J J J J J
in L:.. ~ ~
L/ L/
j Bin L/ Sin Cos L/ Cos L/ Tan L/ Tan L/
Radians j j j j j j
these values in the general expression for M as of the beam, or 4420 x .75 = 3315"# positive be-
given at the top of the Table. cause it produces compression in the top fibers.
In a beam-column member, the bending mom- The moment at (2) due to the cantilever overhang
ents do not vary directly as the load is increa equals (20 + 10) 36 x 16 = 8640"#. Fig. A5.68
ed. Thus, the student should realize that marg- 2
ins of safety based on direct proportion of mom- shows the beam portion between points (1) and (2)
ents to loads are incorrect and lie on the un- as a free body.
safe side. From Art. A5.25, we have the following pre-
It is recommended that four significant cise equations for a beam carrying a transverse
figures be used in computations, making use of uniform distributed load With end compressive
the so-called precise equations, since the re- loads.
sults in many cases involve small differences
x L
between large numbers. tan J = D" - D~ cos J - - - - - - - - -(A)
A5.28 Example Problems* D~ sin L
Example Problem #1 J
Fig. A5.67 illustrates a typical upper, and
outer panel wing beam of a biplane. Let it be M
-iIIaX
= ~
cos x + Wj" - - - - - - - - - - -(B)
required to determine the maximum negative }
bending moment between points (1) and (2), gen-
erally referred to as the maximum span moment. Evaluating terms for substitution in these equa-
To obtain the true bending moments on the beam, tions, we obtain,
the axial beam load as well as the end moments
M~ 3315"#
at (1) and (2) are necessary since they influ-
ence the deflection of the beam. M" 8640"#
Solution:- p = 4420# compression
To obtain the horizontal component Th of I = 10 in 6 given and assumed constant
the lift strut load, we take moments about the throughout the span.
from the left end will be computed. Substituting values of C~ and C.. and f(W) from
Table A5. I in the above equations:
M.a =3315 + 48 x 20 x 24 - 940 x 48 = -18765"#
Thus the secondary bending moment equals M= (M.. - M~ cos L/j) sin x/j + M cos x/j
sin L!j ~
- 28700 + 18765 = - 9935"# which is a large per-
centage of the primary moment. The transverse
deflection of the beam at the point of max. span
moment then equals - 9935 = 2.25 inches upward.
- 4420
20 000 '"
'~I- > - - - - - L = 41. 762 -j
..
-
,,2 1/2 - 083 Steel Tube 1~02000,000'"
1t
(3) Using these values of L' /p and Fc , (4) Determine the bending moment at the
compute centerline of the beam-columns shown in Fig.
AIO.IO Assume EI =
64,000,000 lb. in. sq.
E' =!iL(L'
n p )a a
Fig. A5.75
500# (5) For the beam-column in Fig. A5.75
~35" --I W=30#/in. calculate the bending moment of the centerline
60 _ ~_ _ _ l!oo~ f t f t t +t ~O# I = 10 in:
=
of the member. Assume E 1,300,000 psi. and
I.. 50"
Fig. A5.71
·1 r 't-
30
300#
60 "---1
Fig. A5.72
(1) Fig. A5.71 shows a 1-1/2 - .065 steel
tUbe sUbjected to both end and lateral loads.
Determine the maximum bending moment on the 5000#~
=] ,t
r 10#/in.
f , fTTTT1EO#/in~
~ 5000#
tUbe. Compare the result with the bending
moment due to the side load only. E 29 X 10 8 = 9000 f-- 5 0 " + 50" ---j 18000"#
psi. I of tUbe =
.075 in.· Compute lateral 500*
deflection at point of maximum bending moment. Fig. A5.76
Area tUbe = 0.293 sq. in.
(6) For the beam-column loading in Fig.
(2) The beam column member in Fig. A5.72 is A5.76, calculate bending moment at center
made of 24ST aluminum alloy. Calculate and pOInt of beam. Take E - 1,200,000 psi and
plot a curve of the bending moments on the I = 10 in!
member. Also plot bending moment due to
lateral loads only. E 10.3 x 10 8 psi. I = = A5.31 Beam-Columns in Continuous Structures.
5.0 in.· The secondary moments in a particular
(3) Determine the maximum bending moment member due to beam-column action also effect or
for the wood wing beam and loading of Fig. influence the deflections in adjacent members
of a continuous structure. This rather involved
AIO.9. I of beam section 17 in.· E 1.3 X = = problem can be handled qUite simply and rapidly
10 8 • by the moment distribution method as explained
and illustrated in Arts. All.12 to 15 of
---96"~40"~
W = 25#/in.
Chapter All and in Part F of Chapter C2.
30
arJo"
1-48"~96"-+56'~
1~
-~t"1 ' f f f t , , 1 , , f f f t , 1 t Jt,'l.OOO
,- 10#/in. 0"1\
10000#
• 200" 1
Fig. A5.74
CHAPTER A6
TORSION. - STRESSES AND DEFLECTIONS
where J =
polar moment of inertia of the shaft
cross section and equals twice the moment of in-
ertia about a diameter.
Fig. A6.2
L _- _~
From equa t1. on (1) GG r
A6.1
A6.2 TORSION
r-
Fig. A6.4).
150#
Then 3w + (0.5 w)12 = 4.16
24 ST Al Alloy
ontrol Stick { G = 3,800,000 psi
hence w = 0.463 Ib./in.
The total load P1 forward of the center-
Oo
N I~---- 28" -----l~ line of torque tube = 0.463 x 3 = 1.389 lb. and
P 2 the load on aileron portion aft of hinge line
1 -~~~
Bearing
Elev. Control Wire
Fig. A6.3
aileron operating system attached to aileron
horn and the horn pull equals 3900/11 = 356 lb.
The polar moment of inertia of a It - 0.058 The torsional moment per running inch of
round tube equals 0.1368 in 4 • torque tUbe: = - 1.389 x 1.5 + 2.778 x 4 = 9~0
in. lb. Hence, the maximum torque, which occurs
Maximum Shearing stress = ~ = Tr/J = at the center of the aileron, equals 9.0 x 29 =
(3900 x 0.75)/0.1368 = 21400 psi. 261 in. lb.
The angular twist of the tube between
points A and B equals
261 x 0.625 = 2450 psi.
0.06678
Q =GJ
TL = 3900 x 28
3,800,000 x 0.1368
= 0.21 radians (J 0.06678 in 4 .)
varies directly as the distance from the end of twists of non-circular cross-sections is given
the ailerons, the angle of twist Q can be com- in Art. A6.6.
puted by using the average torque as acting on
entire length of the tube to one side of horn or AG. 5 Elastic Membrane Analogy.
=
a distant L 29", hence The shape of a warped cross-section of a
non-circular cross-section in torsion is
Q = TL __-::::----;=2~6~1_:::x~2:..:9..",...._=_=_=:__5 needed in the analysis by the theory of elas-
GJ 2 x 3800000 x 0.06678 7.3 - 0.86 ticity, and as a result only a few shapes such
degrees as rectangles, elipses, triangles, etc., have
AG.4 Torsion of Members with Non-Circular Cross- been solved by the theoretical approach. How-
Sections. ever, a close approximation can be made ex-
The formulas derived in Art. A6.2 cannot be perimentally for almost any shape of cross-
used for non-circular shapes since the assump- section by the use of the membrane analogy.
tions made do not hold. In a circular shaft It was pointed out by Prandtl that the
sUbjected to pure torsion, the shearing stress equation of torsion of a bar and the equation
distribution is as indicated in Fig. A6.5, for the deflection of a membrane subjected to
namely, the maximum shearing stress is located uniform pressure have the same form. Thus if
at the most remote fiber from the centerline an elastic membrane is stretched over an open-
axis of the bar and is perpendicular to the ing which has the same shape as the cross-
radius to the stressed point. At a given dist- section of the bar being considered and then if
ance from the axis of rotation the shear stress the membrane is deflected by subjecting it to a
slight difference of pressure on the two sides,
the resulting deflected shape of the membrane
provides certain quantities which can be mea-
sured experimentally and then used in the
theoretical equations. However, possibly the
main advantage of the membrane theory is, that
Fig. AG.5 it provides a method of visualizing to a
considerable degree of accuracy how the stress
conditions vary over a complicated cross-section
of a bar in torsion.
The membrane analogy provides the follow-
Fig. AG.7
ing relationships between the deflected mem-
brane and the twisted bar.
(1) Lines of equal deflection on the membrane
'L max.
(contour lines) correspond to shearing
stress lines of the twisted bar.
is constant in both directions as illustrated in
Fig. A6.5, which means that ends of segments of (2) The tangent to a contour line at any point
the bar as it twists remain parallel to each on the membrane surface gives the direction
other or in other words the bar sections do not of the resultant shear stress at the corre-
warp out of their plane when the bar twists. sponding point on the cross-section of the
If the conditions of Fig. A6.5 are applied bar being tWisted.
to the rectangular bar of Fig. A6.6, the most (3) The maximum slope of the deflected membrane
stressed fibers will be at the corners and the at any point, with respect to the edge
stress will be directed as shown. The stress support plane is equal in magnitude to the
would then have a component normal to the sur- shear stress at the corresponding point on
face as well as along the surface and this is the cross-section of the tWisted bar.
not true. The theory of elasticity shows that
the maximum shear stress occurs at the center- (4) The applied torsion on the tWisted bar is
line of the long sides as illustrated in Fig. proportional to twice the volume included
A6.6 and that the stress at the corners is zero. between the deflected membrane and a plane
Thus when a rectangular bar tWists, the shear through the supporting edges.
stresses are not constant at the same distances
from the axis of rotation and thus the ends of To illustrate, consider a bar With a
segments cut through the bar would not remain rectangular cross-section as indicated in Fig.
parallel to each other when the bar tWists or in A6.8. Over an openin~ of the same shape we
other words, warping of the section out of its stretch a thin membrane and deflect it normal
plane takes place. Fig. A6.7 illustrates this to the cross-section by a small uniform pres-
action in a twisted rectangular bar. The ends sure. Equal deflection contour lines for this
of the bar are warped or suffer distortion deflected membrane will take the shape as il-
normal to the original unstressed plane of the lustrated in Fig. A6.9. These contour lines
bar ends. which correspond to direction of shearing
Further discussion and a summary of equa- stress in the tWisted bar are nearly circular
tions for determining the shear stresses and near the center region of the bar, but tend
A6.4 TORSION
to take the shape of the bar boundary as the From Table A6.1 it is noticed that for
boundary is approached. Fig. A6.9a shows a sec- large values of bit, the values of t he con-
tion through the contour lines or the deflected stants is 1/3, and thus for such narrow rec-
membrane along the lines 1-1, 2-2 and 3-3 of Fig. tangles, equations (6) and (7) reduce to,
A6.9. It is obvious that the slopes of the de-
flected surface along line 1-1 will be greater 3 T (8)
than along lines 2-2 or 3-3. From this we can 't"MAX = bta
conclude that the shear stress at any point on
line 1-1 will be greater than the shear stress _ 3 T
(9)
for corresponding points on lines 2-2 and 3-3. Q - bPG
The maximum slope and therefore the maximum
a. Although equations (8) and (9) have been
derived for a narrow rectangular shape, they
i, can be applied to an approximate analysis of
shapes made up of thin rectangular members
I such as illustrated in Fig. A6.10. The more
f--._.-+-.-.-
generous the fillet or corner radius, the
smaller the stress concentration at these jun-
ctions and therefore the more accuracy of these
approximate formulas. Thus for a section made
up of a continuous plate such as illustrated in
Slope Slope
2~ 3 ~
Fig. a
Fig. A6.9a b can be taken as centerline length for above type
of sections
stress will occur at the ends of line 1-1. The I-- b 3 ---->I .1: t
~
slope of the deflected membrane will be zero at a T3
the center of the membrane and at the four ba
corners, and thus the shear stress at these .!..
points will be zero. I---b ~ -->I 'Et~
A6.6 Torsion of Open Sections Composed of Thin Plates. Fig. A6.10
Members having cross-sections made up of
narrow or thin rectangular elements are some- Fig. (a) of Fig. A6.10, the width b can be taken
times used in aircraft structures to carry tor- as the total length of the cross-section. For
sional loads such as the angle, channel, and Tee sections such as the tee and H section in Fig.
Shapes. A6.10, the polar moment of inertia J can be
For a bar of rectangular cross-section of taken as ~ bt 3/3. Thus for the tee section of
width b and thickness t a mathematical elasticity Fig. A6.10:
analysis gives the following equations for maxi-
mum shearing stress and the angle of tWist per T 3 T
unit length. Q = GJ = G ~ bt 3
(6 ) =G 3 T
(b~t~ + bat~)
<Il 0.141 0.196 0.214 0.22910.249 0.283 0.281 0.299 0.307'0.313 0.333
1" = 3 T (13)
t S (b 1 + b s )
A6.5
EXAMPLE PROBLEM SHOWING TORSIONAL STIFFNESS OF A6.7 Torsion of Solid Non-Circular Shapes and Thick-
Walled Tubular Shapes.
CLOSED THIN WALLED TUBE COMPARED
Table A6.3 summarizes the formulas for
TO OPEN OR SLOTTED TUBE. torsional deflection and stress for a few
Fig. A6.11a shows a 1 inch diameter tUbe shapes. These formulas are ~ased on the as-
with .035 wall thickness, and Fig. A6.11b shows sumption that the cross-sections are free to
the same tube but with a cut in the wall making warp (no end restraints). Material is homo-
it an open section. geneous and stresses are within the elastic
range.
For the round tube J 1 = 0.02474 in·.
A6.8 Torsion of Thin-Walled Closed Sections.
The structure of aircraft wings, fuselages
For open tube J s = 31 x 3.14 x .035
.§
= 0.000045 and control surfaces are essentially thin-walled
tubes of one or more cells. Flight and landing
loads often produce torsional forces on these
major structural units, thus the determination
of the torsional stress and deformation of such
structures plays an important part in aircraft
structural analysis and design.
Fig. A6.12 shows a portion of a thin-
walled cylindrical tUbe which is under a pure
Fig. A6.lla Fig. A6.11b torsional moment. There are no end restraints
on the tUbe or in other words the tube ends and
Let Q1 equal tWist of closed tUbe and Qs tube cross-sections are free to warp out of
equal tWist of open tube. The twist, will then their plane.
be inversely proportional to J since Q = J~ •
Therefore the closed tube is J1/J S 0.02474/ =
0.000045 = 550 times as stiff as the open tube.
This result shows why open sections are not ef-
ficient torsional members relative to
torsional deflection. i dS t-; F=qds
e
T
TABLE A6.3
q constant
G = Modulus of Rigidity.
K (1n4) From Table. Fig. A6.12 Fig. A6.13
FORMULA FOR
SECTION K
SHEAR STRESS
'0
(a) on the cross-section and qb that at point
n a3 b3 ";MAX =~(at ends (b) •
K = a2+b2 nab2
of minor axis). Now consider the segment a a ~ b of the tube
wall as shown in Fig. A6.12 as a free body. The
applied shear force intensity along the segment
SOLID SQUARE.
edges parallel to the y axis will be given the
values Qa y and qb y as shown in Fig. A6.12. For
T
D1
SOLID RECTANGLE.
K • 0.141a4 ";MAX = 0.208a3 (at
midpoint of each side).
a plate in pure shear the shearing stress at a
point in one plane equals the stress in a plane
at right angles to the first plane, hence
~ = qa and qb = qb •
Y Y
";MAX = T(3a - 1. 8b) Since the tube sections are free to warp
DI
1- 2a -l
8aZbZ
midpoint of longside. there can be no longitudional stresses on the
tube wall. Considering the equilibrium of the
segment in the Y direction,
SOLID TRIANGLE. ~Fy =0 = Qa~ - qb~ = 0, hence QaY = qb y and
6~ K = 1. 73a4
80
";MAX = -¥-
a
at midpoint of side.
therefore Qa =
qb or in other words the shear
force intensity around the tube wall is con-
stant. The shear stress at any point ~ q/t. =
If the wall thickness t changes the shear stress
For an extensive list of formulas fOr many shapes both solid and hollow,
refer to book, "Formulas For Stress and Strain" by Roark, 1954 Edition.
A6.6 TORSION
changes but the shear force q does not change, The elastic strain energy dU stored in this
or element therefore equals,
dU = qds 0
2
The product ~ is generally referred to as
the shear flow and is given the symbol q. The However the shear strain 0 can be written,
name shear flow possibly came from the fact that
the equation ~t = constant, resembles the equa- o =.!G =...1.- but q =-.!.
tion of continuity of fluid flow qS = constant Gt' 2A
where q is the flow velocity and S the tube
cross-sectiona~ area.
We will now take moments of the shear flow hence
q on the tube cross-section about some point (0).
T"
In Fig. A6.13 the force dF on the wall element
dF =
qds. Its arm from the assumed moment cen- or U =f 8A"Gt ds, the integral j
ter (0) is h. Thus the moment of dF about (0)
is qdsh. However, ds times h is twice the area is the line integral around the periphery of
of the shaded triangle in Fig. A6.13. the tube. From Chapter A7 from Castigliano's
Hence the torsional moment dT of the force theorem,
on the element ds equals,
Q =
au = ~ _T_ ds _ _ _ _ _ _ _ _ _ _ _ (17)
dT • qhds .. 2qdA
aT 4A"Gt r
since all values except t are constant, equa-
and thus for the total torque for the entire tion (17) can be written,
shear flow around the tUbe wall equals,
Q = 4AT,.G f~ --------- (18)
T = fA 2qdA and since q is constant and since T =2 qA, then also,
Q = 2~G f ~s - - - - - - - (19)
T = q2A (14)
where Q is angle of twist in radians per unit
or length of one inch of tube. For a tube length
of L
q
T
= 2A (15) Q = ~A~ f dtS - - - - - - - - - - - - - - - ( 20 )
where A is the enclosed area of the mean periph- A6.9 Expression for Torsional Moment in Terms of
ery of the tube wall. Internal Shear Flow Systems for Multiple Cell
The shear stress ~ at any point on the tube Closed Sections.
wall is equal to q, the shear force per inch of Fig. A6.16 shows the internal shear flow
wall divided by the area of this one inch length pattern for a 2-cell thin-walled tUbe, when
or 1 x t or the tUbe is subjected to an external torque.
q~, q" and q3 represent the shear load per inch
T on the three different portions of the cell
.~ • q/t .. 2At (16 )
walls.
For equilibrium of shear forces at the
TUBE TWIST junction point of the interior web With the out-
Consider a small element cut from the tube side wall, we know that
wall and treated as a free body in Fig. A6.14, - - - - - - (21)
with ds in the plane of the tUbe cross-section m
and a unit length parallel to the tube axis.
Under the shearing strains the plate element
,10-- 1 ---+I
qdS~~2
t"
t ~
magnitude to twice the area of the geometrical inch in the web 1-2 and (q2 - q3) for web 2-3.
shape formed by radii from the moment center to For equilibrium, the torsional moment of
the ends of the wall element ds times the shear the internal shear system must equal the ex-
flow q. ternal torque on the tube at this particular
Let To = moment of shear flow about point section. Thus, from the conclusions of article
(0). Then from Fig. A6.16, A6.9, we can write:
.T = 2q1A~ + 2q2A2 + 2q3A3 - - - - - (24)
2GQ = .s.A !j ds
t
(25)
- (23)
where A~ = area of cell (1) and A2 = area of ThUS, for each cell of a multiple cell struc-
cell (2). Therefore, the moment of the internal
shear system of a multiple cell tUbe carrying
ture an expression tf
d: can be written and
equated to the constant value 2GQ. Let au,
pure torsional shear stresses is equal to the
sum of twice the enclosed area of each cell
times the shear load per inch which exists in
represent a line integral f d: for cell wall
1-0, and a~2' a 20 , a 23 and a 30 the line in-
the outside wall of that cell. (Note: The web
mn is referred to as an inside wall of either
cell) •
tegrals f
~ for the other outside wall and
interior web portions of the 3-cell tube. Let
A6.10 Distribution of Torsional Shear Stresses in a clockwise direction of wall shear stresses in
Multiple-Cell Thin-Walled Closed Section. any cell be positive in sign. Now, substituting
Angle of Twist. in Equation (25), we have:
= 2GQ - - - - - - - - - - - - - - - - - - (27)
Fig. A6.17
Cell 2
- - - - (29)
2GQ =38~.4 [- (q2 - q~) 335 - 1735 q",J =
.865 q~ - 5.34 q2 - - - - - - - - - - (34)
Fig. A6.18
Equating (33) and (34)
I = 25.25"
Q =GTJ - - - - - - - - - - - - - - - - - - (32)
Fig. A6.19 shows a typical 2-cell tubular 83450 - 2 x 105.8 q~ - 2 x 387.4 q2 = 0 - {36)
section as formed by a conventional airfoil
shape, and having one interior web. An external Solving equations (35) and (36), q~ = 55.6i/in.
applied torque T of 83450 in. lb. is assumed and q2 =
92.5#/in. Since results come out
acting as shown. The internal shear resisting positive, the assumed direction of counter-
pattern is required. clockwise was correct for q~ and q2 or true
signs are q~ = - 55.6 and q2 = - 92.5.
Calculation of Cell Constants
Cell areas; - A~ = 105.8 sq. in.
q~2 =- 55.6 + 92.5 =36.9#/in. (as viewed
from Cell 1).
A", = 387.4 sq. in.
A = 493.2 sq. in. Fig. A6.20 shows the resulting shear pat-
Line integrals a = f ~s ; -
tern. The angUlar twist of the complete cell
can be found by SUbstituting values of q~ and
q2 in either equations (33) or (34), since twist
a~o =26.9
•025
= 1075'' a
u
- 13.4 - 335
- .04 -
of each cell must be the same and equal to
twist of tube as a whole •
=~
25.25 + 15.7 25.3
.05 + .032
=1735 Fig.A6.20
= .002456 T
= 89.76
1075 X 387.4 + 335 X 493.2 ] T Q =...!.. = x ~9. 76 = .01116 ~ (rad.) per unit
[ 1735 X 105.8 2 + 335 X 493.2 2 + 1075 X 387.4 2 2 JG G length of cell.
~~~~~:::J~
Calculation of
terms .04"
Area of cells: -
2
A1 = 100 A2 100
SOLUTION:
A - A1 + A2 200
Line integrals a =
f ~s : -
Calculation of cell constants
Cell areas: -
100 A3 = 100
a 10 -- .05
10 + 20 = 866.7
.03
Line integrals a =J!~t .
- 10 - 333 3
- .03 - . nx 10
a 12 = 10 = 200
2 x .025 =
a 10 = 629
.05
a 20 -- .03
20 + .1 0 = 916.7 _ 20
04 a20 - .03 = 667 a23 = ~= 333
.03
Solution of Equations from Article A6.11: -
a30 = .03
20 + ~
.04
= 917
Equating the external torque to the internal re-
sisting torque: -
_1 r. 916.7 X 100 + 333.3 X 200 J
q1 -2 L916.7 X 100 2 + 333.3 X 200 2 + 866.7 X 1002 T
Subst i tut ing:
= .002540 T
78.6 q1 + 200 q2 + 200 q3 - 100,000 =0 - (37)
Fig. A6.24
Substituting:
With one unknown shear flow q for each cell,
the solution by the previous equations becomes
2aG = 1;0 [200 q .. - 200 q1. + 667 q .. + 333 q .. - qUite laborious.
The method of successive approximations
provides a simple, rapid method for finding the
333 qaJ - - - - - - - - - - - - - - - - - (39) shear flow in multiple cells under pure torsion.
Substituting:
such a shear flow q1.
as to make aG1. 1.
From equation
= (2)
cen\T
(19) we can write,
2GG = 1;0 [ 333 qa - 333 q .. + 917 qaJ - - - (40) aG =~ ~ ds
2AJt
Solving equations (37) to (40), we obtain, Now assume aG = 1,
then Fig. a
q1. = 143.4#/in.
_ 2A
q .. = 234.1#!in.
qa = 20S.S#/in. q-fct;
q .. q1. = 90.7!l=/in.
Since practically all cellular aircraft
q.. - qa = 25:3#/in. beams have wall and web panels of constant
thickness for each particular unit, the term
tI; ds L
j t for simplicity will be written ~t' where
L equals the length of a wall or web panel and
t its thickness. Thus we can write,
Fig.A6.23
2 A
q= -------------- (41)
L
~cell t
234.1#/in.
Therefore assuming aG1. =
1 for cell (1) of
Fig. a, we can write from equation (41): -
Fig. A6.23 shows the resulting internal
shear flow pattern. The angle of tWist, if de- * Based on Paper, "Numerical Transformation Procedures
sired, can be found by SUbstituting values of for Shear Flow Calculations" by S. U. Benscoter.
shear flows in any of the equations (3S) to (40). Journal, Institute of Aeronautical Sciences, Aug. 1946.
A6.11
2 x 89.3
25.7 + 10 -_ 178.6 _- 0 • 212 lb. I in.
(1-2) is sUbjected to a shear flow q,. =- .109#
842 lin. (counterclockwise with respect to cell (1)
.04 .05 and therefore negative), in addition to the
shear flow q~ =
.212 of cell (1). The negative
Fig. b. shows the results. shear flow q,. =-
.109 on web 1-2 decreases the
twist of cell (1) as calculated above With the
t-- q = .109
resulting value for GQ 1 =
0.875 instead of 1.0
as started With.
~ Thus in order to make GQ 1 = 1 again, we
f (2) i will have to add a constant shear flow q~ to
cell (1) which will cancel the negative twist
f due to q" acting on web (1-2). Since we are
considering only cell (1) we can compare cell
Fig. b Fig. c wall strains instead of cell tWist since in
equation (41), the term 2A is constant.
In a similar manner assume cell (2) sub- Thus adding a constant shear flow qi to
jected to a shear flow q" to make GQ,. = 1, Then cancel influences of q,. on web 1-2, we can write:
q,.
2A"
=Z L
2 x 39.3
lQ. + 15.7 = 7~i; = 0.109#/in.
qi (z ~) cell (1) - q" (~) web 1-2 =0
t .05 .03 hence
= 0.875
C.O.F. (2 to 1) which equals
Cell (2), GQ" =_1_
2A.,
Z q~ =
t
= 2 x 139.3 [.109
15 .237 as found above in substitution in equation
x ..03
7 - (.212 - .109) .05
10J = (42) •
Now consider cell (2) in Fig. d. In
bringing up and attaching cell (1), the common
= 0.466 web (2-1) is subjected to a shear flow of q~ =
- 0.212*/in. (counterc~ockwise as viewed from
Since Q1 must equal 9" if the cross-section cell 2 and therefore negative). This additional
is not to distort from its original shape, it is shear flow changes GQ" tWist of cell (2) to a
evident that the above shear flows are not the relative value of 0.4375 instead of 1.0 (see
true ones when the two cells act together as a previous GQ" calculations). Therefore to make
unit. GQ,. equal to 1.0 again, a corrective constant
Now consider cell (1) in Fig. d. In bring- shear flow q~ must be added to cell (2) to
ing up and attaching cell (2) the common web
A6. 12 TOR S ION
= .0139
-
C.O.F. (1 to 2)
= (4-) web (2-1) = 200 = .277 qT = .237 x .00717 = .0017~/in.
(z i) cell (2)
723
q~ = .277 x .0139 = .00385i/in.
Fig. e shows the constant shear flow qk and Fig. g shows the results.
qi that were added to make Gg =
1 for each cell.
However these cor- The final or re-
rective shear flows q'" =. 0587 sulting cell shear
were added assuming flows then equal the
the cells were again original shear flows
independent of each plus all corrective
other or did not cell shear flows, or
have the common web
(1-2) • Thus in Fig. e q1 (final) = q1 + qi +
Fig. g
bringing the cells q~ + qT
together again the interior web is subjected to
be resultant shear flow of qi - q~. In other
words if we were to add the shear flows of Fig. e
q "(final) = qo~ + q~ + q~ + q~
~ ~ ~
(~) 10
OPERATIONS TABLE 1
Cell 1 Cell 2
web 1-2 .05
(1-2) = = 10 + 30 = .40
C.O.
1
2
C.O. Factor
Assumed a
.277
.212
.237
109
~ (~) cell 2 .05 0.1
3
4
C.O. a
C.O. a
.0258
.0139
= .0587
.00717
10
- (~) web 2-1
.n017 - - 00~8"
6 Fna .<::>34 lin
7 T = 2A
Total T
45.2"# 14.1"# C.O.
(2-1) = = .: = .25
~ (~)
8 59.3"fI
cell 1 .05
Explanation of Table 1 Line 2 gives the shear flow q in each cell,
Line 1 gives the carryover factors for when it is assumed each cell is acting separ-
each cell, computed as explained before. Line 2 ately and is sUbjected to a unit rate of twist
gives the necessary constant shear flow q in or GQ '" 1.
each cell to give unit rate of twist to each The calculations for the q values are as
cell acting independently. (GQ = 1). Line 3 follows: -
gives the first set of constant corrective shear
flows to add to each cell. The corrective q re- For cell (1)
ferred to as the carryover q or C.O.q in the
table consists of the q in the adjacent cell 2 x 100
40 = .25
times the C.O. factor of that cell.
Thus .237 x .109 = .0258 is carryover from .05
cell (2) to cell (1) and .277 x .212 = .0587 is For cell (2)
carryover from cell (1) to cell (2).
Line 4 gives the second set of corrective
carryover shear flows, namely .277 x .0258 = 2 x 100 "'.4
.00717 to cell (2) and .237 x .0587 - .0139 to 10 + ~
.05 0.1
cell (1). Line 5 repeats the corrective carry
over process once more. Line 6 gives the final A1 and As equal cell area of cells (1) and (2)
q values which equal the original q plus all respectively. L = length of wall element and t
carryover q values. its corresponding thickness.
Example Problem 1 (2 cells) In order not to start out with decimal values
Determine the internal shear flow system of q, the values above will arbitrarily be
for the two cell tUbe in Fig. A6.25 when sub- multiplied by 100 to make q1 =
25#/in. and
jected to a torque of 20,000 in. lbs. qs = 40~/in. Since we want only relative
I-- 10" - - \ - - 10" --l values of terms this is permissible. These
values are shown in line 2 of Table (2). The
T
10" .05
.05
Cell
1 .05
0.10
Cell
2 O. 10
corrective carryover process proper is started
in line 3 of table (2). In cell (1) the amount
carried over of q1 =
25 to cell (2) equals the
C.O. factor times 25 or 0.4 x 25 10 which is =
written along the vertical line under cell (2).
1 .05 O. 10
Fig. A6.25
Likewise in cell (2) the amount of qs - 40 that
is carried over to cell (1) equals the C.O.
factor times qs or .25 x 40 =
10, which is
written along the vertical line lmder cell (1).
OPERATIONS TABLE 2
The second set of corrective carryover
constant shear flows are given in line 4 of
Cell 1 Cell 2
Table (2), thus, .4 of the q1 = 10 = 4.0 is
1 C.O. Factor 0.4 0.25 carried over to cell (2) and .25 x value of
2
3
Assumed a
C.O.
25 0
10 ~
40 0
10 q1 = 10 = 2.5 is carried over to cell (1). Line
4 C.O. 2.5 4 5 repeats the process namely 0.4 x 2.5 = 1 to
5
6
7
C.O.
C.O
CO
1.0
o 25
o 10
1
u.4 cell (2) and .25 x 4 =
1.0 to cell (1).
8 F1na "ll.ll:> :>i
This process of carrying over values of q
9 2Aa = " 7770 is continued until the values are small or
11100
10 T Total 18870
11 a far T=20 000 41 2 58 9
negligible. Line 8 gives the final q values in
each cell as the summation of the assumed q
Explanation of solution as given in Table (2); value plus all carryover values of q. Thus in
cell (1) q 38.85#/in. and for cell (2)
Line 1 gives the values of the carryover q = 55.5. Line 9 gives the torque that these
factors. The values are calculated as follows: values of cell shear flow can produce.
C.O. factor cell (1) to cell (2); T = 2Aq
A6 14 TORSION
For cell (1) T = 2 x 100 x 38.85 = 7770 in. lb. OPERATIONS TABLE 3
For cell (2) T 2 x 100 x 55.5 =
11100 in. lb. = Cell 1 Cell 2 Ce.U 3
Line 10 gives the sum of the above two values 1 COFactor 23 .21 .284 .288
2 Assumed q 25.30-... ..- 18.45 _ .--10.70
which equals 18870 in. lb. 3 C 0 3 88....- ~ 5.82 3.u.. ..,;; 5.25
The original requirement of the problem was 4 C 0 1 87 - ... 0.894 1 51-- 2.53
5 CO. 0.505 0.430 0.73 0.685
the shear flow system for a torque of 20,000"#. 6 C.O. 0.244 0.116 0.197 0.330
Therefore the required q values follow by direct 7 CO. 0.066 0.056 0.095 O.UI'II
T .04 .032
.04
Cell (2) to (3)
Fig. A6.26
A6.15
OPERATIONS TABLE 4
centage of the total torque carried by each Fig. A6. 33 Fig. A6. 34 Fig. A6. 35
action depends on the dimensions and shape of Fig. A6.34 shows the torque dividing into two
the cross-section and the length of the member. parts, namely the couple force F-F formed by
Fig. A6.32 illustrates the distortion of an
open section, namely, a channel section sUbject- bending of the flanges of the I-beam and the
pure shearing stress system on the cross-
ed to a pure torsional force T at its free end
section. Fig. A6.35 shows the tWisting of the
and fixed at the other or supporting end. Near
the fixed end the applied torque is practically section through a distance 0.
all resisted by the lateral bending of the top The solution will consist in computing the
and bottom legs of the channel acting as short angle of tWist Q under the two stress con-
cantilever beams, thus forming the couple with H ditions and equating them.
forces as illustrated in the Figure. Near the Let TB be the proportion of the total
free end of the member, these top and bottom torque T carried by the flanges in bending
legs are now very long cantilever beams and thus forming the couple F-F in Fig. A6.34. From
their bending rigidity is small and thus the Fig. A6.35, the angle of twist can be written
pure torsional rigidity of the section in this
region is greater than the bending rigidity of
the channel legs. QB =_0_
0.5h
=3F EI
L = § TB
3 h£
3 L3
EI
0.5h
A6.17
Note: The deflection of a cantilever beam with The shaft rotates at constant speed. The dif-
a load F at its end equals FL 3 /3EI, and I the ference in belt pullan two sides of a pulley
moment of inertia of a rectangle about its cen- are shown on the figure. Calculate the maximum
ter axis = tb 3 /12. torsional shearing stress in the 1-3/4 inch
shaft between pUlleys (1) and (2) and between
hence (2) and (3).
(2) A 1/2 HP. motor operating at 1000 RPM
Now let Tt be the portion of the total torque rotates a 3/4-.035 aluminum alloy torque tube
carried by the member in pure torsion. The ap- 30 inches long which drives the gear mechanism
proximate solution for open sections composed of for operating a wing flap. Determine the maxi-
rectangular elements as given in Art. A6.6, mum torsional stress in the torque shaft under
equation (12) will be used. full power and RPM. Find the angular deflection
of shaft in the 30 inch length. Polar moment of
=
Tn
fi = 3 Tt L inertia of tube .01 in. Modulus of rigidity
t Gt 3 (b + b w) =
G 3800,000 psi.
in. lb.
Equating fiB to fit , we can write
TB = 3 E h2 b3 II
Tt 8L 2 Gt 2 (2b + bw) I / I
I
\ / I
SUbstituting values when L = 10 inches. " '1-----
I /,
I /
I
TB = 3 x 10.5 X 10 6 x 3.4 x 1.75 = __- - r/ .......
/
I
= 9.65
Fig. A6. 36
Therefore, percent of total torsional T
taken by bending of flanges equals,
(3) In the cellular section of Fig. A6.36
TB TB determine the torsional shear flow in resisting
TB + Tt (100) = TB + TB (100) = 90.5 percent. the external torque of 60000 in. lb. Web and
9.65 wall thickness are given on the figure. Assume
the tube is 100 in. long and find the torsional
If we consider the section 40 inches from deflection. Material is aluminum alloy. (G =
the fixed end, then L = 40 inches. Thus if 40 2 3800,000 ps 1. )
is placed in the above substitution instead of (4) In Fig. A6.36 remove the interior .035
10 2 the results for TB/Tt would be 0.602 and web and compute torsional shear flow and de-
the percent of the torque carried by the flanges flections.
in bending would be 36 as compared to 90.5 per-
cent at L = 10 inches from support. Thus in T = 100,000 in. lb.
general the effect of the end restraint decreases
rapidly with increasing value of L.
The effect of end restraint on thin walled
n
tubes With longitudional stiffeners is a more
involved problem and cannot be handled in such
a simplified manner. This problem is considered
in other Chapters.
A6.19 Problems.
T
12"
Fig. A6.37
I- 12" -+-8"--.j
(5) In the 3-cell structure of Fig. A6.37
Fig. A6.36 determine the internal resisting shear flow due
to external torque of 100,000 in. lb. For a
(1) In Fig. A6.36 pUlley (1) is the driving length of 100 inches calculate twist of cellular
pulley and (2) and (3) are the driven pulleys, structure if G is assumed 3,800,000 psi.
A6.18 TORSION
(6) Remove the .05 interior web of Fig. I----- 14" 'I' 14"----1
A6.37 and calculate shear flow and twist.
.04 .05
.03
.04
.03
.03 10"
T
(a)
(7) Remove both interior webs of Fig.
A6.37 and calculate shear flow and twist. .03 .03 -:L
1-6"-+8" +-10" --!-10"--I
(8) Each of the cellular structures in
Fig. A6.38 is subjected to a torsional moment
T .03 .03 .03 .03
10" .04 04 .035 .035 .03 .0 .03 (b)
of 120,000 in.lbs. Using the method of
successive approximation calculate resisting
shear flow pattern.
1 .03 .03 .03 .03
The big helicopters of the future will be used in many important industrial and military opera-
tions. The helicopter presents many challenging problems for the structures engineer.
(Sketches from United Aircraft Corp. Publication "BEE-HIVE", Jan. 1958. Sikorsky Helicopters)
CHAPTER A7
DEFLECTIONS OF STRUCTURES
ALFRED F. SCHMITT'1}
A7.1 Introduction.
Calculations of structural deflections are P
important for two reasons: -
(1) A knowledge of the load-deformation Work =L" Pi
I~I
6 Oi
characteristics of the airplane is of primary in the limit as
importance in studies of the influence of
structural flexibility upon airplane perform- 66 -0
ance.
Work =1,0 P dO
(2) Calculations of deflections are neces-
sary in solving for the internal load distri-
butions of complex redundant structures.
Fig. A7.1
The elastic deflection of a structure
under load is the cumulative result of the curve is a straight line whose equation is
strain deformation of the individual elements P = k6 and the strain energy is readily dom-
composing the structure. As such, one method puted to be
of solution for the total deflection might in-
k62 p2
volve a vectorial addition of these individual U 2 or, equally, U = 2k'
contributions. The involved geometry of most
practical structures makes such an approach
A7.3 Strain Energy Expressions for Various Loadings.
prohibitively difficult.
For complex structures the more popular STRAIN ENERGY OF TENSION
techniques are analytical rather than vectorial.
They deal directly with quantities which are not A tensile load S acting at the end of a
themselves deflections but from which deflec- uniform bar of length L, cross sectional area
tions may be obtained by sUitable operations. A and elastic modulus E causes a deformation
The methods employed herein for deflection cal- 6 = SL/AE. Hence S = A~ 6 and
cUlations are analytical in nature.
A7.2 Work and Strain Energy. rO _ AE 6 2
U = Jo Sd6 - L 2" (1)
Work as defined in mechanics is the prod-
uct of force times distance. If the force Alternately,
varies over the distance then the work is com-
puted by the integral calculus. Thus the work
done by a varying force P in deforming a bOdy (2 )
an amount 6 is
Work = ~ Pd6 and is represented Equations (1) and (2) are equivalent ex-
by the area under the load deformation (P-6) pressions for the strain energy in a uniform bar,
curve as shown in Fig. A7.1. the former expressing U in terms of the de-
If the deformed body is perfectly elastic formation and the latter expressing U in~erms
the energy stored in the body may be completely of load. The second form of expression is more
recovered, the body unloading along the same convenient for general usage and succeeding
P-6 curve followed for increasing load. This strain energy formulas will be put in this form.
er.ergy is called the elastic strain energy of For a tensile bar having non-uniform prop-
deformation (hereafter the strain energy, for erties (varying AE), or for which the axial load
brevity) and is denoted by the symbol U. Thus S varies, the strain energy is computed by the
U = ~ Pd6. Should the body be linearly calculus. Thus the energy in'a differential
element of length dx is given by eq. (2) as
elastic (as are most bodies dealt with in
structural analysis) then the load-deformation S2dx
dU = 2AE where Sand AE
TUTx
L40" ~
A(x) = A o (l _X)
Ao= 2 m
.8~
tary strength of materials
Solution:
(6 )
Then
or
lT 2 L (7 )
U = 2 GJ
For a shaft of non-uniform properties and
varyinG loading one has
Example Problem 4
Determine the strain energy of flexure of
the beam of Fig. A7.5. Neglect shear strain u = ~
2
STGJdX
2
__ (8)
energy.
In passing it is worth remarking that one
often encounters the group symbol "GJ" in use
rEI = Constant for the torsional stiffness of a non-circular
shaft or beam such as an aircraft wing. In
such a case the torsional stiffness has not been
computed literally as G x J, but rather as-de-
L fined by eq. (6), viz. the ratio of torque to
"2 rate of twist.
Fig. A7.5
Solution: Example Problem 5
For a certain flight condition the torque
The bending moment diagram was drawn first on an airplane wing due to aerodynamic loading
(Fig. A7.5a) and analytic expressions were is given as shown graphically in Fig. A7.6. The
written for M. variation of torsional stiffness GJ is given in
like manner. Find the strain energy stored.
M~ ~~
~(X)' FOX O<X>~
1. 01"0::::--_ _
Fig. A7.5a
Fig. A7.6
Inspection of the diagram revealed that o:-__ -'-_--.:+_
L . -_ _- ' - -_ _- ' - -_ _
0 1.0 1.0 1.0 1.0 .0667 .0667 The shear load diagram was drawn first
.2 .98 . 96 .76 1. 26 .267 .336 from the right end •
.4
.6
.8
.94
.67
.29
.88
.45
.084
.54
.37
.24
1. 63
1. 22
.35
.133
.250
.200
.216
.305
.070
r- L
2 t L
2~-o
P
2U = LTR" x
GJR
.994
Then
1
U = 2AG x 2 x r:
= Po "L
2
Po'OX
2AG
STRAIN ENERGY OF SHEAR
Eq. (9) may be used to compute the shear
A rectangular element "dx" by "dy" of strain energy in a thin sheet. The element
thickness "t" into the page under the action of ~ x dy is visualized as but one of many in the
uniform shear stress,. (psi) is shown in Fig. sheet and the total energy is obtained by
A7.6a. summing. Thus
From elementary strength
of materials the angle of
shear strain r is pro-
U = ~ JJ,."t gx dy - - - - - - - - - - - - (11)
portional to the shear
stress ,. as Here a double integration is required, the
summation being carried out in both directions
'll' = ~
G
where G is the over the sheet. In dealing With thin sheet the
use of the shear flow q == ~ is often convenient
material elastic modulus so that eq. (11) rewrites
Fig. A7.6a
in shear. For the dis-
placements as shown in
the sketch only the down
load on the left hand side does any work. (In
U = ~ S J q" ~t dy - - - - - - - - - - - - (11a )
general all four sides move, but if the motion A very important special case occurs when a
is referred to axes lying along two adjacent homogenous sheet of constant thickness is
sides of the element, as was done here, the de- analyzed assuming q is constant everywhere. In
rivation is simplified). This load is equal to this case one has
q"
,. x dy x t and moves an amount ~ x dx. Then
1
U = 2GtJf dxdy = q"s
2Gt - - - - - - - - - - - (12)
dU = 2 ,.n dx dy where S = JJ dxdy is the surface area of the
1
sheet.
= 2 (9)
t::. U =PM
t::.U
Limit Mj .. 0,
dU = PdO
1L-------l.LL-
0
M~ f.-
Fig. A7.8
Fig. A7.9a
Solution: Thus
T T In words,
q :: 2A :: 2bc , "The rate or change of strain energy with
respecttod8flection is equal ~ the assoCiirted
constant around the perimeter of any section. "load". -
Then Eq. (14) and the above quotation are state-
ments of the Theorem of Virtual Work. The
reader may find this theorem stated quite dif-
ferently in the literature on rigid body me-
chanics but should be able to satisfy himself
that the expressions are nevertheless compatible.
A useful restatement of the above theorem
THE TOTAL ELASTIC STRAIN is obtained by rewriting eq. (14) as
ENERGY OF A STRUCTURE
dU - Pdo :: 0
The strain energy by its definition is
always a positive quantity. It also is a It is next argued that if the change in dis-
scalar quantity (one having magnitude but not placement do is sufficiently small the load P
direction) and hence the total energy of a remains sensibly constant and hence
composite structure, having a variety of ele-
ments under various loadings, is readily found dU - d(Po) :: 0
as a simple sum.
d(U-Po) :: 0 - - - - - - - (15)
the complementary energy,U*), are related to the Force (lbs.) Translation (inches)
load and deformation by (see Fig. A7.9(b). Moment (in. Ibs.) Rotation (radians)
Torque (in. Ibs.) Rotation (radians)
Pressure (lbs/in") Volume (in 3 )
L h/-'-/""'/""7"//..,,-L--,---,--:-c-;~ Shear Flow (lbs/in) Area (in")
T dU* = odP Any generalizations of the meanings of
liP
"force" and "deflection" are possible only so
long as the units are such that their product
.£ .-------'--0 Fig. A7.9b yields the units of strain energy (in. Ibs) •
Once again for emphasis it is repeated that,
while the complementary nature of eqs. (14) and
(17) are clearly eVident, the use of eq. (17)
dU* (Castigliano's Theorem) is restricted to linearly
dP = a - - - - - - - - - ~ - - - - - - (16) elastic structures. A brief example will serve
for illustration of the possible pitfalls.
This is the Theorem of Complementary Energy. The strain energy stored in an initially
Now for the linearly elastic body a very straight uniform column under an axial load P
important theorem follows since (Fig. A7.9c) when deflected into a half sine wave is
dU = dU* so that
P~U* \
~ II U
\ Y = Yo sin~; M = PY
L
Y Consider Flexural Energy Only
~ a =SM"dx = P"Y~L
U 2EI 4EI
.. U = P"oL"
J12EI
dU x Fig. A7.10
dP = a - - - - - - - - - - - - - - - - - - (17)
where a is the end shortening due to bowing.
In words, Because the deflections grow rapidly as P ap-
"The rate of change of strain energy with proaches the critical (buckling) load the
respectto load 1s equal to the assocra:ted-- problem is non-linear. The details of the
deflection"-.- - -- calculation of U are given with Fig. A7.10.
Eq. (17) and the above quotation are state- Now according to the Theorem of Virtual
ments of Castigliano's Theorem. Work (eq. 14)
For a body under the simultaneous action of dU = P
several loads the theorem is written so as to do
apply individually to each load and its associ- but
ated deflection, thus*: dU P"L"
do = n 2 EI
au = ai (17a)
Therefore
aP i
or
The partial derivative sign in eq. (17a) n"EI
indicates that the increment in strain energy is P = r::;r
due to a small change in the particular load P ,
all other loads held constant. i (Euler formula for uniform column). The
Note that by "load" and "deflection" may correct result.
be meant: Application of Castigliano's Theorem,
eq. (17), leads to the erroneous result:
Load Associated
Deflection
dU = a
dP
dU 2PoL"
dP = n2EI = <5
* The proof of the theorem for the case of multiple loads is
generally formulated more rigorously, appeal to a simple
diagram such as Fig. A7.9c being less effective. See, ** See Art. A17. 6, Chapter A-17 for detailed
for example, "Theory of Elasticity" by S. Timoshenko. derivation of this equation.
A7.7
P (incorrect) Then _6
2U = 43.26 x 10 p" LB FT
dU T.r. ds
Q = dT = 4A"G t t
Solution:
USE OF FICTITIOUS LOADS
The strain energy considered here was that
due to axial loading in each of the four mem- In the following example the desired deflec-
bers. The load distribution was obtained from tion is at the free end of the bar where no load
statics and the energy calculation was made in is applied. A fictitious load will be added for
tabular form as follows: purposes of the calculation. The rate of change
of strain energy with respect to this fictitious
load will be found after which the load will be
TABLE A7.2 set equal to zero. This technique gives the de-
sired result in as much as the deflection is
MEMBER AE X 10"S S"L X 10 6 equal to the rate of change of strain energy
S LBS L FT
LBS AE with respect to the-load and such a rate eXists
OA -1. 50 P 40.0 136 0.66 P" even though the load itself be zero.
AB 2.50 P 50.0 11.8 26.48 P
AC 1. 58 P 63.0 11. 8 13.33 P" Example Problem 10
OC -2.12 P 84.5 136 2.79 P" Compute the axial motion of the free end of
Z = 43.26 P" the tapered bar of Fig. A7.12.
A7.8 DEFLECTIONS OF STRUCTURES
-fJ.
q - J
constant J
A = Ao (1- x)
2L
q
Ao
L
= 40#/in.
= 4 in."
= 40 in.
erally are easier to evaluate.
Example Problem 11
Find the deflection at point B of the beam
j E = 10 X 10 6 psi of Fig. A7.13.
+Pl.
Ix Fig.A7.12
lP EI constant
t~i~L~J
3 3 3
Solution:
Fig. A7.13
After addition of the fictitious end load
R the axial load from statics was found to be
Solution:
S(x) = R + q (L - x)
A fictitious load Pl. was added at point B
Hence, since loadings other than tensile are of and the bending moment diagram was drawn in two
a secondary nature parts.
2
= ~ JL SAEdx - See Eq. 3 O<X<L/3
U 2
o Mp
= ~ JL [R + q (L - X)J2 dx O<Y<L/3
Fig. A7.13a
U = X + 2L" Rq (1 - In 2)
AoE
+ X
+ -.l:.- JL/
3[(P + 2Pl.) (L d~ •
2EI 9
Then o
ir I:/3~12L ~'(L
the calculation of deflections by Castigliano's
theorem. + - 3y) + + 3y)](L ; 3y) dy
In the strain energy integrals arising in
this class of problems, the load Pi' with respect
to which the deflection is to be found, acts as
an independent parameter in the integral. Pro-
+ E~J:/3[p
; 2P'IL _ 30)] 21L ; 30) do .
vided certain requirements for continuity of the
functions are met - and they invariably are in
these problems - the differentiation with re- * For beams of usual high length - to - depth ratios the shear
strain energy is small compared to the energy of flexure.
spect to P may be carried out before the inte- Neglecting the shear energy is equivalent to neglecting the
gration is made. The resulting integrals gen- shear deflection contribution. (see p. A7. 14)
A7.9
The fictitious load P~, having served its pur- The total loadings
pose, was set equal to zero before completing were
the work.
Plan View
2P jL/3x2dx
-.l.... T=T~+T2= Tocos g + PR
°B - EI • 9 o
(1 - cos g)
Lf3 Thus
1 P
+ EI 81
f0 (2L - 3y)(L + 3y) dy Fig. A7.14b
1
U = 2EI
-
PL"
7
-- + 2GJ
1
Je:t/2[Tocos 2
g + PR(l - COS g)J Rd g
486 EI o
(Note use of "Rdg" for the length of a dif-
Example Problem 12 ferential beam element instead of "dx"). Dif-
Fig. A7.14 shows a ferentiating under the integral sign
cantilever round rod of
diameter D formed in a
quarter circle and acted
upon by a torque To.
Find the vertical move-
ment of the free end.
Solution:
R2
+ GJ Jo
rn / 2[ToCOS g + PR(l - cos g)J(l - cos g) dg.
Fig. A7.14a shows Putting P, the fictitious load equal to zero and
the vector resolution of Plan View integrating gave
the applied torque To on
beam elements. T~ (g) = M~
~U _ To R (4 - n - EnI)
2
Tocos g and the moment T~
OVERT = ~P - L i
M~ (g) = To sin g. Ap- GJ
plication of a fictitious
vertical load P (down) at Since J = 21 and G ~ E/2 . 6 the deflection was
the point of desired de- negative (UP).
flection gave the loadings Fig. A7.14a
shown in Fig. A7.14b.
A7.7 Calculation of Structural Deflections by the Method of Dummy-Unit Loads (Method of Virtual Loads).
The strict application of the calculus to Castigliano's theorem as in Art. A7.6, leads to a
number of cumbersome tecl111iques ill-suited to the solution of large complex structures. A more
flexible approach, readily adapted to improved "book keeping" techniques is the Method of Dwnmy-
Unit Loads developed independently by J. C. Maxwell (1864) and O. Z. Mohr (1874)-.---- - -----
--- That the equations for the Method of Dwnmy-Unit Loads may be derived in a number of ways is
attested to by the great variety of names applied to this method in the literature ®. Presented
below are two derivations of the equations stemming from different viewpoints. One derivation ob-
tains the equations by a reinterpretation of the symbols of Castigliano's theorem - essentially an
appeal to the concepts of strain energy. The other derivation uses the principles of rigid body
mechanics. Based as they are upon a common set of consistent assumptions, all the methods must, of
course, yield the same result.
®Variously called the Maxwell-Mohr Method, Method of Virtual Velocities, Method of Virtual Work, Method of Auxilliary
Loads, Dummy Loads, Method of Work, etc.
A7.10 DEFLECTIONS OF STRUCTURES
I Cont'd. II Cont'd.
U= ~
2
JSAEdx
2 + ~ JMEIdx
2
2
~
+ 2 JTGJdx +
2
6
i
=
AE
j S~~idXEI
+ J M~dX + - - - - - - - etc.
Consider now the set of equilibrium forces
applied to the truss of Fig. A7.15(b). The set
may be divided into two parts: the "external
system", consisting of the unit load applied at
Consider the symbols the point whose deflection is desired and the
~S ~M ~T two reactions fixing the line of reference, and
c'lP 'JP' aP ' - - - etc. the "internal system" consisting of the axial
iii loads acting on the truss members to produce
Each. of these is the "rate of change of so equilibrium. These latter are denoted by the
-and-so with respect to Pi" or "how much so- symbol "u". This set of forces is considered
and-so changes when Pi changes a unit a:nount" OR small enough so as not to. affect the actual be-
Eq,UALLY, "the so-and-so loading for a unit load havior of the structure during subsequent ap-
Pi"· plication of a real set of major loads. This
Thus, to compute these partial derivative unit load set is present solely for mathematical
terms one need only compute the internal load- reasons and is called a "virtual loading" or
ings due to a unit load (the virtual load) ap- "dummy loading".
plied at the point of desired deflection. For Assume now that the structure undergoes a
example, the term c\Mjc\p' could be computed in deformatIon due to application of a set of real
either of the two ways "shown in Fig. A7.15a. loads, the virtual loads "going along for t~
ride". Each member of the structure suffers a
RATE METHOD UNIT METHOD deformation denoted by ~ ®. The virtual load-
ing system, being in equilibrium (zero resultant)
by hypothesis, does work ("virtual work") equal
to zero. Or, considering the subdivision of the
virtual loading system, tue work done by the
external virtual load must equal that absorbed
Fig. A7.15a by the internal virtual loads. The work done by
the external virtual forces is equal to one
pound times the deflection at joint C, the re-
actions R:>. and R2 not moving. That is
M = P:>.x m = dummy loading
External Virtual Work = l~ x 6
c
eHl The internal virtual work is the sum over the
aP:>. = x
structure of the products of the member virtual
Likewise, dsAoP c , where Pc is a load (real loads u by the member distortions~. That is,
or fictitious) applied at joint c of Fig. A7.15b, Internal Virtual Work = Z u.~ .
is given by the loadings for the unit load ap-
plied as shown. Then equating these works,
In practice the use of the unit load is 1 x 6 = Z u·~ .
most convenient. Using the notation c . ..
If the deformations ~ are the result of
u == ~Sp ,m ==!!':L.- t ==~ elastic strains due to real member loads 3 then
i 3P i ' aP
i ~ = 3L/AE for each member and one has
_ av - = aq 3L
6c = Z u AE
v = -c\- q
Pi - c)P i
for the unit loadings, the deflection equation The argument given above may be extended
becomes qUickly to include the internal virtual work of
6 = Sudx +
i AE
J
Mmdx + Ttdx
EI
J
GJ
virtual bending moments (m), torsion loads (t),
J
shear loads (v), and shear flows (q) doing work
during deformations due to real moments (M),
+ ~+ J JJ
q qG~dY - - - - - - - - - (18) torques (T), shear loads (V~nd shear flows
(q). The general expression becomes
6
i
J3udx
=
AE
JMmdx
EI
+JTtdx +
GJ
+ f Vvdx 11 q q dxdy - - -
AG
+
Gt
- - (18)
Af-~::....a...:..~~~~--I.....:...\!~4B
r. 5=R:>. 't. 5=R 2 @ Note that the deformations are not restricted to those due
to elastic strains only. They may be the result of elastic
"un loads due to a unit (virtual) load. or inelastic strains, temperature strains or misalign-
ment corrections.
i
• m
A7.11
Z = 286.4 Z =-58.72
Answers: 0G = 0.286"
VER
me
A7 12 DEFLECTIONS OF STRUCTURES
part (d) the system of unit loads of Fig. A7.16d Rotations, both absolute and relative are
was used. Table A7.4 completes the solution, determined by applying unit (virtual) couples
the real loads and member flexibilities to the member or portion of structure whose
rotation is desired. The unit couple is resist-
Ui;) being the same as for example problem 13. ed by reactions placed on the line of reference
for the rotation. Thus Figs. A7.16e and A7.16f
0 -.6 C 0 0 -.375 -.375 show the unit (virtual) loadings for par ts (e)
and (f) respectively. Table A7.5 completes the
t ~V I :\Vli F
~
R=l#
/
/
-.6
{o
I
0
0
.5
FOG
~U'
1# • .5
H
calculation, the real loads and member flexibil-
ities
problem 13 •
(L/
AE )
being the same as for example
-/~~J(°II\Vo
Fig. A7.16c Fig. A7.16d
AB 6.270 10,500 0 - 0 0 0
BF 13.012 2,000 0 0 0 0
CG 13.012 -1,000 0 0 0 0
Therefore the movement of joint c towards
joint F was {j = .06587 inches and the motion of DH 13.012 2,000 0 0 0 0
Example Problem 15
For the truss of Fig. A7.16 determine
e) the absolute rotation of member DG Therefore the absolute rotation of member
f) the rotation of member BG relative to DG was gDG = .00473 radians and the rotation of
-
CG = .00053 radians
member CG. BG relative to CG was gBG _
L -
..
A7.13
Origin at B: -
With origin at B
M = 30 x • ~ = 15 2
x
M= - Px (Fig. A7.17)
w • '0 Ib./in
For virtual loading (Fig. A7.17a)
m = 0, for x<b
m = -1 (x - b), for x>b
Hence Mrndx - - Px(-x + b)dx = (Px 2 - Pbx)dx
Whence 0c P
--
EI
fL
b
(x 2 -
P [ ~"_ bX
bX)dx = --
EI 3 2 b
L
2
J
m 0, x < 20
m -.5 (x - 20) = -.5x + 10, when x = 20 to 50
m -.5 (x - 20) + 1 (x -50) = .5x - 40, when
If b = zero, then 0B = PL"/3EI x = 50 to 80
~A
C x-jB °D(CE) -- J~
Mrndx
t== a L
..I..
b 4p Fig. A7.17
Ih 50
h 80
r C
hf
B Fig. A7.17a
= --
EI 20
15 x 2 (-.5x + 10)dx + EI
1
50
15 x 2
(.5x - 40) dx
r-
1A C
-' uni 1:.
B
Fig. A7.17b
150x~l50 + ~ [7.5 x" _ 600x" l80
ccuple =1. [-7.5 x.. +
EI 4 3 J20 EI 4 3 jso
11.72 + 6.25 + .300 - .400 +
For virtual loading see Fig. A7.17b 76.8 - 102.4 - 11.72 + 25.0]
m = 0, x<b, m =- 1, x>b
17.9 X 10 6
Hence Mrndx = (-Px) (-1) dx = Px dx EI
Therefore deflection of point D relative to line
J joining CE is down because result comes out nega-
:
e =
C
Mrndx
EI
=E (L xdx
EI ) b
tive and therefore opposite to direction of vir-
tual load.
[I:j ;x-~ n *~
PL 2 Example Problem
If b = 0, BB = 2EI h rm:1
,. '\ f'
Example Problem 17
For the uniformly loaded cantilever beam M.
of Fig. A7.18, find the deflection of point D
relative to the line joining points C and E on
1 ~D Mdiag'l _ rodiag. _
lf R... diXg.j
the elastic curve of the beam. This is repre- L w~ wLf
R=1/L unit R=1/L
sentative of a practical problem in aeronautics, 2" 2"
(a) (b) (c) couple
in that AB might represent a rear wing beam and
Fig. A7.19 Fig. A7.20
A7.14 DEFLECTIONS OF STRUCTURES
Find the horizontal deflection of point C
for the frame and loading of Fig. A7.19. Also dy = J dx and ~ = A~s' where A = cross sectional
angular deflection of C with respect to line CD. area of beam at section and Es = modulus of
rigidity~ and assuming that the shearing stress
Solution:
V
A is uniform over the cross-section.
Fig. b shows the static moment curve for
the given loading and Fig. c the moment diagram Vvdx
Therefore 1 x 0 = ~.Then the total deflec-
for the virtual loading of a unit horizontal s
load applied at C and resisted at D. tion for the shear slips of all elements of the
beam equals
F< - JEI'
Mindx • m= h
f
uC(H) - L Vvdx _______ (a)
o total = ~
o S
hence
where V is the shear at any section due to given
L(WLx - ~2) hdx = ~(WLx2 _WX") J!. JL
loads. v = shear at any section due to unit
o
J2 EI L 4 6. EI 0 hypothetical load at the point where the deflec-
tion is wanted and acting in the desired direction
of the deflection. The reactions to the hypo-
1 whL" thetical unit load fix the line of reference for
12 EI the deflection.
A is the cross-sectional area and Es the
To find angular deflection at C apply a unit
imaginary couple at C with reactions at C and D. modulus of rigidity. Equation (a ) Is slightly
F.1g. A7.20 shows the virtual m diagram. in error as the shearing stress is not uniform
over the cross-section, e.g. being parabolic for
EI
o
2JL
a rectangular section. However, the average
shearing stress gives close results.
For a uniform load of w per unit length, the
center deflection on a si~ply supported beam is: -
°center =2 J~ Vvdx
AE s
=2 J~ (wL 2
WX) !dx
2
AE s 0'
o wL 2 0
=
8AE s
Linear Deflection of Beams Due to Shear by Virtual Work.
Generally speaking, shear deflections in For bending deflection for a simply supported
beams are small compared to those due to bending beam uniformly loaded the center deflection is
except for comparatively short beams and there-
fore are usually neglected in deflection calcu- 5 wL 4
lations. A close approximation is sometimes 384EI
made by using a modulus of elasticity slightly Hence
less than that for bending and using the bending wL 2
deflection equations.
The expression for shear deflection of a
beam is derived from the same reasoning as in
8AE s
5wL 4 24 using Es = .4E In:r,
r = radius of gyration.
previous derivations. The virtual work equa- 384EI.
tion for the hypothetical unit load system for a 1
shear detrusion dy (Fig. A7.21) considering only For I beams and channels r is approximately 2 d
dx elastic is 1 x o=vdy where v is shear on
section due to unit hypothetical load at point d
and for rectangular sections r = V12 (d = depth)
0, and dy is the shear detrusion of the element
dx due to any given load system or any other d
cause. In aircraft structures a ratio of L is
1
seldom greater than 12 •
f/_. EiL~
.=.J .' . - =ft 0 Thus the shearing deflection in percent of
tlon+1o"-i A
Unit.
Load
~f:========:::ilA
!
acting in the plane of the desired de-
flection. (inch lbs/inch lb)
Example Problem
Fig. A7.22
Example Problem 20
Find the vertical deflection of free end A Fig. A7.23 shows a cantilever landing gear
due to shear deformation for beam of Fig. A7.22 strut-axle unit ABC lying in XY plane. A load
assuming shearing stress uniform over cross- of 1000# is applied to axle at point A normal to
section, and AE constant. XY plane. Find the deflection of point A normal
s to XY plane. Assume strut and axle are tubular
= I Vvdx and of constant section.
°A AEs
Solution:
V 100# for x = o to 10
V = 150 for x 10 to 20 The loading shown causes both bending and
tWisting of the strut axle unit. First find
v = 1 for x = o to 20 bending and torsional moments on axle and strut
hence due to 1000# load.
1 flO 20
AE
s 0
100·1 dx
10
+ -1-
AEs f10
150·1 dx
20
1 2500
= =
AEs [lOOxJ + Ais [15Ox] AEs
o 10
Method_of Virtual Work Applied to Torsion of Cylindrical Bars.
11#
B A
The angle of twist of a circular shaft due Fig. A7.24
to a torsional moment may be found by similar
reasoning as used in previous articles for find- Ttdx
ing deflection due to bending or shear forces. o= fMmdx +
EI EJ
The resulting expressions are: - s
o = J TEtsdJX - - - - - - - - - - - - - (A )
Member AB M = 1000 x, (for x = o to 3)
T = 0
Member BC MBC 3000 sin 20° + 1000 x,
(for x = o to 36)
g = JTis~ - - - - - - - - - - - - - - (B)
TBC 3000 cos 20° constant
between Band C.
In equation (A), for translation deflections,
T = twisting moment at any section due to Now apply a unit 1# force at A normal to xy
applied tWisting forces. plane as shown in Fig. A7.24 and find bending
t = torsional moment at any section due to and torsional moments due to this 1# force.
a virtual unit 1 lb. force applied at
the point where deflection is wanted
and applied in the direction of the Member AB
desired displacement. (in lbs/lb)
Es shearing modulus of elasticity for the m 1. x x, (for x o to 3)
material. (also "G")
J = polar moment of inertia of the circular t = 0
cross-section.
In equation (B), for rotational deflections, Member BC
g = angle in twist at any section due to
the applied twisting moments in planes m 3 sin 20° + 1. x (for x = 0 to 36)
perpendicular to the shaft axis.
Angle in radians. t = 3 cos 20° constant between Band C.
ET
A7.16 DEFLECTIONS OF STRUCTURES
= i, J: 1000 x . X <Ix + i, J:
6
KlOOO X + 1026)
I
(x + 1.026)J dx + E JJ36(2820) (2.82) dx
s 0
I
A r A=. 15 B cA=. 15 C cA=.15 D The integrations in the trapezoidal sheet
~
panels were made using the shear flows on the
L
200321l '=.032: . '=.032 T
t=. A=. 08 I A=. 08
A=.05"'
I
-L
(nearly) horizontal sides as average values,
assumed constant over the panel. With this
ElF
j.:- 20"
G
----0+--20" --I-- 20"
H+
-I
P=180041 simplification JJ qAV q~~ dxdy = qAV qAV ;t
Fig. A7.25
where S is the panel area.
Solution: The calculation was completed in Table
A7.6.
It was assumed that the webs did not buckle TABLE A7. II
(10)
and carried shear only. Fig. A7.26 is an ex- (1) (2) (3) (4) (5) (6) (7) (8) (9)
~ ~ (8)X(9)"2f~~XI0.
ploded view of the beam showing the internal Ul Uj iEx1o+e (2)x{6)+(3)x(7)
real loads carried as determined by statics: AB 3.00 P 2.184 P 2.00 l.09 1.697 1.393 13.34 8.133 P lOB,;) P
CD 1.201 P 0 0 0 13.34
~d~I~'D
-3.00 P _2.164 P -2.00 -1.09 -1.697 -1.393 13.34 8.133 P 108.5 P
EF
GH -1.201 P 0 0 0
DB 0 0
ca 1 0 .67 .33
3.OP BF o 0
\ Z 271.0 pj2
Fig. A7.26 B-C-F~G .0491 P .0546 350 8.14 7.64 The denominatorofterma in
cola. 6 &. 7 should be 6 instead
C~D-G-H .0600 P 01' 3. To correct this error,
* The equations of statics for tapered beam webs are Z ==13.44 P
total sum of col. 10 Is divided
by2.
derived in Art. AI5.18, Ch. A-15.
..,
!
A7.l7
/ T
II1111I11111111111111
_3 I
.147 (1800) 10 in = .265 in. ~ I
:::
A7.8 Deflections Due to Thermal strains.
As noted in the "virtual work derivation"
of the dummy-unit load deflection equations, the L
a
real internal strains of the structure may be
due to any cause including thermal effects.
Fig. A7.29
Hence, provided the temperature distribution and
thermal properties of a structure are known, the
dummy-unit load method provides a ready means Solution:
for computing thermal deflections.
The axial deformation of a differential
Example Problem 22 element of the upper flange (subscript u) was
Find the axial movement at the free end of assumed given by ~ = a T dx where a was the
...., a uniform bar due to heat application to the
material thermal coefficient of expansion.
fixed end, resulting in the steady state tem-
perature distribution shown in Fig. A7.28. As- The lower flange, having received no heating
sume material properties are not functions of underwent no expansion.
Inasmuch as a thermal expansion is uniform in all
temperature. directions no shear strain can occur on a material element.
Hence no shear strain occurs in the web. The apparent
anomaly here - that web elements appear to undergo shear
T temperature above
ambient tempera- deformations ll' = a~ (Fig. A7. 29b) - is explained as
T(X)= To (l-tanh ~X) ture follows: The temperature varies linearly over the beam
K = an empirical con- depth. The various horizontal beam "fibers" thus undergo
axial deformations which vary linearly also in the manner
stant depending of Fig. A7. 29b giving the apparent shear deformation. No
upon thermal prop- virtual work is done during this web deformation since no
erties and rate of axial virtual stresses are carried in the web.
Fig. A7.28 heat addition. With the addition of a unit (Virtual)
load to the free end, the Virtual loadings ob-
tained in the flanges were:
u L --
= - x (= - u )
Solution:
U h L
The thermal coefficient of expansion of the
rod material was a. Hence a rod element of Then the deflection equation was
length dx experienced a thermal deformation
~ = a • T • dx. Application of a unit load at
the bar end gave u = 1. Therefore
Ju . J~( l-tanh ~)
. \;
6 a . T . dx a To dx
1 Example Problem 24
= a ToL [1 -R (In cosh K)J The first step in computing the thermal
stresses in a closed ring (3 times indeterminate)
involves cutting the ring to make it statically
Example Problem 23 determinate and finding the relative movement of
The idealized two-flange cantilever beam the two cut faces.
of Fig. A7.29a undergoes rapid heating of the Fig. A7.30a shows a uniform circular ring
upper flange to a temperature T, uniform span- whose inside surface is heated to a temperature
wise, above that of the lower flange. Deter- T above the outside surface. The temperature is
mine the resulting displacement of the free end. constant around the circumference and is assumed
A7.18 DEFLECTIONS OF STRUCTURES
Remarks:
In the three elementary examples given
above no stresses were developed inasmuch as the
idealizations yielded statically determinate
structures which, with no loads applied, can
have no stresses. Indeterminate structures are
treated in Chapter A.S.
A7.9 Matrix Methods in Deflection Calculations.
a b Introduction. There is much to recommend the use of
Fig. A7.30 matrix methods~ for the handling of the quantity
of data arising in the solutions of stress and
Solution: deflection calculations of complex structures:
The data is presented in a form suitable for use
An element of the beam of length Rd~ is in the routine calculatory procedures of high
shown in Fig. A7.30c. Due to the linear tem- speed digital computers; a fleXibility of opera-
perature variation an angular change dG = tion is present which permits the solution of ad-
RaT ditional related problems by a simple expansion
~ d~ occurred in the element. The change in
of the program; The notation itself suggests new
length of the midline (centroid) of the section and improved methods both of theoretical ap-
RaT d'/'.
was I::" = 2 th
Unit redundant loads were ap- proach and work division.
The methods and notations employed here and
plied at the cut surface as shown in Fig. A7.30b later are essentially those presented by Wehle
giving the following unit loadings around the and Lansing@ in adapting the Method of Dummy-
ring. Unit Loads to matriX notation. Other appropriate
From unit redundant couple (X) references are listed in the bibliography.
mg = -R sin ¢
ug - sin ~
5 = 0·1::" +
2n RaT
1· --- d
~ RaT
= 2n---
X
J J o
h h
2n RaT 2n RaT
5y = :os ~ 2 d¢ + 0(-) R(l-cos ~) h d~
J ~ For the reader not familiar with the elementary arithmetic
rules of matrix operations employed here, a short appen-
dix has been included in the present edition.
f @ L. B. Wehle Jr. and Warner Lansing, A Method for Re-
ducing the Analysis of Complex Redundlillt"""'StrUctures to a
2n R2 aT (negative indicating movement to RoutineProcedure, Journ. of Aero. Sciences, 19, October
h the right) ~ -
A7.19
relate the internal generalized forces qi, qj to the above expression is a property of the struc-
the external applied loads, Pm or Pn • With this tural element (variation of EI) and of the nature
relationship the strain energy expression ob- of the associated generalized force (exponent on
tained in I, above is then transformed to give variable). Introducing the notation
U as a function of the P's.
III. Castigliano's Theorem is used to
compute deflections.
= r"ydy
Consider for example the problem of writing
the strain energy (of flexure) of the stepped a .. 3 EI.. 0
a 33 r"dy
0
EI ..
cantilever beam of Fig. A7.32a, assuming ex-
ternal loads are to be applied as transverse the strain energy becomes
point loads at A and B. The set of internal
generalized forces of Fig. A7.32b will com- 1 q" a .... + 2 x 1 q .. q3 a ..
pletely determine the bending moment distribu- U = 12 q
..
~
a~~ + 2
.. 2 3
~
qj)--,
q3 (b)
I
~q~ q3
II Ir" q~1,
(d)
in the inner beam segment by q .. and q3' are like-
wise readily written, with proper account taken
for the cross influence of one force upon an-
other (the "a .. 3 q .. q3" term).
M = q;>,x O<x<L~
M = q3 + q ..y O<y<L ..
q13~ It.. , q~l~
(e)
Note that
= = a ji - - - - - - - - (20)
Fig. A7.32. Some possible choices
of generalized forces.
Hence
a (Maxwell's Reciprocal Theorem)
'*("Relative displacements in the individual member") ji
F
A7.20 DEFLECTIONS OF STRUCTURES
UtN
I
I
I
-I
I
I
I
{2 tJ.
t. --+-
_I
I I I
I
+q4qJ.
, u4J. + - - - - - - - - - - - - - _ -)
I
I LJ. ---1
I
I
1
I I Fig. A7.32f
I J
I
I I
T qNqJ. uNJ. + - - - - - - - - - - + qN qN aNN
In matrix notation
~2
q,21. U2 2 ------:
I I
I
I
I
I I
(21) Symbolically these relationships are written
I
I I
, I I
I
~;:------- ~
I
I
qN - - - - - - - - - - - - (22)
or, more concisely, The matrix [Gim] is called the ~it load
distribution" inasmuch as anyone column of
(26)
Fig. A7.33
Solution:
The steps in passing to eq. (26) may be
demonstrated readily by writing out eq. (25) The choice and numbering of generalized
for, say, a set of three loads (m, n = 1,2,3), forces are shown on the figure. These forces
differentiating successively with respect to were placed so that previously derived express-
PJ., Ps and P" and then re-collecting in matrix ions for the a's could be used. The following
form. member flexibility coefficients were computed.
The matrix ~mnJ gives the deflection at Note that the only non-zero coefficients of
mixed SUbscripts (i not equal to j) are those
the external points "m" for unit values of the for loads common to an element.
loads P and is therefore, by definition, the
n
l
matrix of influence coefficients. L/~sdx L"
COMPARISON WITH DUMMY-UNIT LOADS EQUATIONS a~~ = lEI = 81EI = a
0 S2 =a 44
It is instructive to write eq. (26) out as This expression was adopted from that de-
veloped for a transverse shear force on a canti-
...
.).. lever beam segment in the preceding illustrative
example.
ing to ~.
AE
The matrix product [! jn] 1Pn \
This expression is for the cross influence
gives the member load distriuutions due to the of a couple and a shear load on a cantilever
real applied loads, hence these are the "S" segment. Collecting in matrix form,
loads. Finally the operation of matrix multi-
-
•--.III-r...iiiiiiiiiiiii ................= =
~-----,-_ .
A7.22 DEFLECTIONS OF STRUCTURES
1
as per eq. (26).
27 0 0 0 0 The matrix [!mnJ is seen to be symmetric
1 1 about the main diagonal as it must be: from
0 0 0
27 6L Maxwell's reciprocal theorem A = A (see
[U ij] L 3
3EI 0
1
6L V
1
0 0 eq. (20)). mn nm
L/3
1 1 0 u = _1
2AE
I[L q
i
+
[GimJ 0 0 o
1 1 1
2L13 L/3 Then referring to eq. (20), Fig. A7.34
0
~
3
1 L/3 1 2L/~ 0 -L
27 -L
6L 0 0 1 1 0
~rr;j
L
= 3EI 0 1 OIL 13 0 lL C" 0 0
L/3
0 0 L
o 0 0 1 0 1 1
6AE
o 0 0
276
1 1 1
1 1 2L L/ 0
An equally likely choice of generalized
o 0 0
6L L" 3 3 forces for the above case is shown in Fig.
A7.34a. The strain energy is (x measured from
free end)
Multiplying (see appendix),
_ L
= 1...JLdx - AE
Should the deflections be desired at the AE
o
three points one forms
A7.23
Fig. A7.34f
The trapezoidal shear
panel (Fig. A7.34g) is treated approximately by
using the average shear flow on the non-parallel
sides as though it were constant throughout the
sheet. Thus
S surface area
S
Fig. A7.34c Q11 = Gt
Area varies linearly
Since by statics qj
Fig. A7.34g
h"
qi~' one could use qj as
. an alternate choice of generalized force and
BEAMS
The energy in the uniform beam of Fig.
, jj = (hh:'J\" GtS .
Q
A7.34d is given by
TORSION BAR
A uniform shaft under torque qi has strain
u = 2~' It q
,
energy
ql"L L
Fig. A7.34d U = 2GJ Then Q11 = GJ •
Then
A7.11 Application of Matrix Methods to Various
a11 structures.
Example Problem 26
The tubular steel truss of Fig. A7.35 is to
be analyzed for vertical deflections at points E
and F under several load conditions in which
vertical loads are to be applied to all joints
An alternate choice of generalized forces excepting A and D. The cross sectional areas of
for the beam of Fig. A7.34d is shown in Fig. tUbe member~ are given on the figure.
A7.34e. Set up the matriX form of expression for the
A7.24 DEFLECTIONS OF STRUCTURES
20" c*I
T
Constant Axial
f'R 1 2 3 4
.217
3'1 0
C>
. ./~ ~ 15"
./. I
-L
Then
-=-
Load} qi-qj
at i - e(ii+ O(jj+
1 0 1.0 0 1.0
. F 2 0 0 0 0
o<.ij+<:>(ji
_ L/AE 3 0 -1.0 0 0
Fig. A7.35 Fig. A7.35a 4 0 1.25 0 1.25
deflections at points E and F. 5 0 -1.03 0 -1.03
Solution: 6 -1.0 - .75 0 - .75
The member flexibility coefficient for a 7 1.13 .848 1.13 .848
uniform bar under constant axial load is
L/AE • Fig. A7.36a gives the numbering scheme 8 -.825 -1.65 -.825 -1.65
applied to the members and the q's (these being
one and the same, since q is constant in a given 9 .20 0.40 .20 0.40
member). Fig. A7.36b shows the numbering scheme
adopted for the external loading points.
Then the matrix triple product
~mn]
1 2
= Emi] [a ij ]
[SI3J 4 3
@jn]
9 6
5 was formed giving, per eq. (26),
8
6~ 440 389 257 389 P~
Fig. A7.36a Fig. A7.36b
62 1
389 927 252 789 P2
=-
Member flexibility coefficients were col- 1I 3 E 257 252 257 252 P3
lected in matrix form as
6.. 389 789 252 789 p ..
inches
~ 1 2 3 4 5 6 7 8 9
1 1.0 0 0
Solution:
2 0 1.0 0
The structure was divided into elements
3 0 0 1.0
and the set of internal generalized forces ap-
plied as shown in Fig. A7.37a. (Torques and 4 .342 0 0
moments are shown vectorially by R.H. rules).
Axial stresses were neglected in C-B. 5 3 0 0
6 0 -.937 1.026
7 0 .342 2.811
8 0 0 1.0
'-- -
At this point the engineer may consider the
problem as solved, for the remaining computation
is a routine operation:
Example Problem 29
The following member flexibility coefficients The beam of example problem 21 is to be re-
were determined solved by the matrix methods presented herein.
A7.26 DEFLECTIONS OF STRUCTURES
Solution:
Fig. A7.38 shows the choice and numbering
of generalized forces.
completes the calculation.
/ ' (coeff's
Q 33 Q~3 + Q ~3/'
Example Problem 30
= coll'td
qe \ ' fr.two
Deflections of statically indeterminate
ljfdTof
structures often may be computed successfully
by the methods of this chapter provided that
some auxiliary means is employed to obtain an
approximation to the true internal force dis-
tribution. The exact internal force distribu-
~ r, rq~ tion is not necessarily required in making de-
i
, , q7 r , q4 flection calculations inasmuch as such a calcu-
lation amounts to an integration over the
Fig. A7.38 structure - an operation which tends to average
out any errors. Thus one may use the engineering
theory of bending (E.T.B.), experimental data,
previous experience, etc. to obtain reasonable
No forces were shown applied to the lower flange estimates of the internal force distribution for
elements as these were known to ue equal to unit loadings.
those of the upper flange due to symmetry. In the following problem the matrix of in-
Entries were made for a ij in matrix form as be- fluence coefficients is determined for a single
cell, three-bay box beam (3 times indeterminate)
low. Entries for a 33 and a ee were quadrupled as by using the E.T.B.
these occur in two identical members each on 1QQ Fig. A7.39a shows an idealized doubly
and bottom. Entries for a 3e , aee and Gee were symmetric single cell cantilever box beam having
doubled. (See Art. A7.10 for coefficient three bays. Determine the matrix of influence
formulae. ) coefficients for the six point net indicated.
2 3 4 5 7 8 9
1 10
2 2085
3 17.8 4.45 10"
L
tl~ = 10-~ 4
5
6.95
2350'
6 4.45 17.8 4.45
7 7.64
8 2600
9 4.45 8.89
@i~
4 0 l.0 0 ternal generalized forces. Note that only the
upper side of the beam was numbered, the lower
5 .0540 .0600 0 side being identical by symmetry.
6 2.184 l.09 0 Member flexibility coefficients were com-
7 0 0 l.0 puted by the formulas of Art. A7.10 and entered
in matriX form as below. Note that all entries
8 .0447 .0496 .0545 for which there were corresponding loads on the
9 3.00 2.00 l.00 lower surface of the beam were dOUbled. By
this means the total strain energy of the beam
A7 27
4
flow (q = ~) due to the torque developed in
transferring the load to one side.
The matrix triple product
deflection at the point (cf. eq. 18). We now In Fig. f consider Mdx as a load on a simply
show that the deflection expression, using the EI
above equation, is the same as the bending supported beam and determine the bending moment
moment expression for a simple beam loaded by at pOints band c due to Mdx acting at point a.
an "elastic weight" ~~ • L _ MLdx EI
2 - 8EI
In Fig. A7-40, the loading of (a) produces
the real moments of (b). Consider the deflec- L _ MLdx
tions of points Band C due to the angular 4; - 16EI
change lMdx
[ [ in a beam element at A" (Fig. A7-40c ) • These values of the beam bending moments at
points band c are identical to the deflections
at band c by the virtual work equations. The
moment diagram m for a unit load at band c
(Figs. d and e) is numerically precisely the same
as the influence line for moment at points b
and c.
{-h-+.--h-.J.---h--+--hj
4 4 4 4 Therefore deflections of a simple beam can
(a) be determined by considering the M curve as an
EI
~
-->l I--dx
imaginary beam loading. The bending moment at
any point due to this M loading equals the de-
(b) EI
flection of the beam under the given loads.
Mdx
EIr?! Likewise it is easily proved that the angu-
lar change at any section of a simply supported
(c) beam is equal to the shear at that section due to
the M diagram acting as a beam load.
+ - EI
~
(d) mb, moment diagram for unit
A7.13 Example Problems
load acting downward at point B Example Problem 31. Find the vertical
3 deflection and slope of points a and b for beam
;~
and loading shown in Fig. A7.41. The lower Fig.
shows the moment diagram for load P acting at
center of a simple beam.
(e) m c , unit load at point C
P
Mdx
~EI
f
3 Mdx
(f)
t
1Mdx a b
EI Constant
l:'
wL
11111L If
rr 111 wL
Fig. A7.43
Bending moment at point B due to above elastic A7.14 Deflections of Beams by Moment Area Method*
loading = 7,100,000 •.. deflection at B relative For certain types of beam problems the meth-
to line AC ~ 7,100,000 = .71 inch od of moment areas has advantages and this meth-
E-10,000,000 od is frequently used in routine analysis.
Example Problem 34 Angular Change Principle. Fig. A7.44 shows a
Fig. A7.43a shows a section of a cantilev- cantilever beam. Let it be required to deter-
er wing sea plane. The wing beams are attached mine the angular change of the elastic line be-
to the hull at points A and B. Due to wing tween the points Aand B due to any given loading.
loads the wing will deflect vertically relative From the equation of virtual work, we have
to attachment points AB. Thus installations
such as piping, controls, etc., must be so lo-
l\.
cated as not to interfere with the wing deflec-
tions between A and B. For illustrative pur-
poses a simplified loading has been assumed
as shown in the figure. EI has been assumed as
constant Whereas the practical case would in-
\
volve variable I. For the given loading deter-
mine the deflection of point C With respect to
the support points A and B. Also determine the
vertical deflection of the tip Aoints D and E.
o 0 \
o 0 o
o o
t- 400" 9- 300'1 ~,
\
C'<I
~
If)
Fig. A7.43a
D EI Constant E
0 1
01
Fig. A7.43b
\ C
01
0"1 Fig. A7.43c
q<1
.....267'1
o r-----'------'----''''H....1IIi>l'---.L.---L.----. o
C'<I C'<I
q< q<
Fig. A7.43d
M
~
M
~
III
~ ~
III
Solution:- Fig. A7.43b shows the bending moment III I
diagram for the given wing loading. To find the III
deflection of C normal to line joining AB we ~~ Distributed Beam Load
treat the moment diagram as a load on a imaginary
beam of length AB and simply supported at A and B
(See Fig. A7.43c.) The deflection of C is equal
numerically to the bending moment on this ficti-
cious beam.
Hence Elo c = 25920 x 40 - 25920 x 20
or 0c - 518000
-~
A7.15 Beam Fixed End Moments by Method of Area MA =- Pab 2 and MB =- Pba 2 /L 2 where b = (L - a)
Moments --rr
From the two principles of area moments as To find the fixed end moments for a beam with
given in Art. A7.14, it is eVident that the de- variable moment of inertia use the M/I diagrams
flection and slope of the elastic curve depend in place of the moment diagrams.
on the amount of bending moment area and its lo- Example Problem 37
cation or its center of gravity.
Fig. A7.48 shows a fix-ended beam carrying
Fig. A7.47 shows a beam fixed at the ends two concentrated loads. Fin~ the fixed-end
and carrying a single load P as shown. The bend
ing moment shown in (c) can be considered as moments MA and MB'
made up of two parts, namely that for a load P
acting on a simply supported beam which gives J100#
the triangular diagram with value pa (L-a)/L MA (a)
for the moment at the load point, and secondly ~ 9"+-12"+9" ./
a trapezoidal moment diagram of negative sign
with values of MA and MB and of such magnitude
as to make the slope of the beam elastic curve
zero or horizontal at the support points A and (b)
B, since the beam is considered fixed at A and B.
The end moments MA and MB are statically
indeterminate, however, with the use of the two Fig. A7.48
moment area principles they are easily determin-
ed. In Fig. b the slope of elastic curve at A
and B is zero or horizontal, thus the change in
slope between A and B is zero. By the 1st (c)
, i
/...-a--r
/: A
P
EI Constant
.--- L
Solution:- Fig. b shows the static moment diagram
A assuming the beam simply supported at A and B.
~-----~B (b)
For simplicity in finding areas and taking moments
~astic Curve
of the moment areas the moment diagram has been
Fig. A7. 47 diVided into the 4 simple shapes as shown. The
~a~-a) ':?~ centroid of each portion is shown together with
the area which is shown as a concentrated load at
~_~i (c) the centroids.
Fig. C shows the moment diagrams due to un-
I known moments MA and MB. The area of these tri-
angles is shown as a concentrated load at the
principle of area moments, this means that the centroids.
algebraic sum of the moment areas between A and Since the change in slope of the elastic
B equal zero. Hence in Fig. c curve between A and B is zero, the area of these
(-MA -Mb) L+ (Pa (L-a) ~ moment diagrams must equal zero, hence
0 - - - - - - -(A) 5265 + 14040 + 2160 + 6885 + 15MA + 15MB = 0
2 L' 2
or
In Fig. b the deflection of B away from a tang- 15MA + 15MB + 28350 = 0 - - - - - - - - - - (1 )
ent to elastic curve at A is zero, and also de-
flection of A away from tangent to elastic curve The deflection of point A away from tangent to
at B is zero. elastic curve at B is zero, therefore the first
Thus by moment area principle, the moment moment of the moment diagrams about point A
of the moment diagrams of Fig. C about points A, equals zero. Hence,
or B is zero. 5265 x 6 + 15 x 14040 + 17 x 2160 + 24 x 6885 + 150MA +
300 MB = 0 or 150 MA + 300 MB + 444600 = 0- - - -(2)
Taking moments about point A:- Solving equations (1) and (2), we obtain
MA = - 816 in. lb.
mA = Pa 2(L_a) .§g+Pa(L-a) 2x (a + L-a)+~ x ~ + MB = - 1074 in. Ibs.
2L 3 2L 3 23
With the end moments known, the deflection or
MBLx2 L = 0 - - - - - - - - - - - - - - - (B) slope of any point on the elastic curve between
23 A and B can be found by use of the 2 principles
of area moments.
SolVing equations A and B for MA and MB
A7.33
L-'- - - - - ~al--""tL---"o----<"-k--!i'---+------\g
time over the method of virtual work used in 1--. d 6 Panels @ p
previous articles of this chapter. The method Bar bc (2)
in general consists of finding the magnitUde and ~ t ~ection Diagram
location of the elastic weight for each member uLlL = 4pLlLbc~ -fo~··LlL Shortening
of a truss due to a strain from a given truss -r- ~ - LI L of Member bc
~
loading or condition and applying these elastic j ~ Elastic Load on Imaginary Beam
~i
a b
tions for this elastic loading are also given.
Diagram (4) shows the beam bending moment dia- '~
gram due to the elastic load at point O. It is
noticed that this moment diagram is identical to
l~
w=-
the deflection diagram for the truss as shown in h LlL
diagram (2).
The elastic weight of a member is therefore
equal to the member deformation divided by the
arm r to its moment center. If this elastic
load is applied to an imaginary beam correspond-
ing to the truss lower chord, the bending moment
on this imaginary beam will equal to the true
m z\ PiA
a
o LlL
w=-
b
r
I
w=~L
truss deflection. b ~ h~
~
Diagram 5, 6 and 7 of Fig. A7.49 gives a h~~
similar study and the results for a ~L lengthen-
ing of member CK. The stress moment center for ~
w=-
this diagonal member lies at point 0', which lies r
outside the truss. The elastic weight ~L at w= Member Axial Deformation -LlL
-
q Perpendicular Arm to Moment Center r
point 0' can be replaced by an equivalent system The moment center 0 of a chord member is the
at points 0 and k on the imaginary beam as shown intersection of the other two members cut by the
in Diagram (6). These elastic loads produce a section used in determining the load in that memb-
bending moment diagram (Diagram 7) identical to er by the method of moments
the deflection diagram of diagram (5). The sign of the elastic weight wfor a chord
Table A7.7 gives a summary of the equations member is plus i f it tends to produce downward de-
for the elastic weights of truss chord and web flection of its point of application. Thus for a
members together with their location and sign. simple truss compression in top chord or tension
in botton chord produces downward or posi tive
elastic weight
A7 34 DEFLECTIONS OF STRUCTURES
Fig. A7.50
this step are given in the figure, the stresses
being wri tten adjacent to each member. The next
step or steps is to compute the member elastic
weights, their location and their sense or dir-
~ ~
ection. Tables A7.8 and A7.9 gives these calcu-
lations. Table A7:8
Chord Member Elastic Weights
tlL tlL lIembe r Length Area Load PL ~ tiL Arm Elastic Point of
P=Q=-r P=Q=r L A P Ai r wt. appl1ca-
Ea29xl0 6 w
r
=tiL tion
joint
./,
(See Member ab)
a .
+'
il., .,=... ....
.,<
Load
P
tiL =
PL
Ai rl
P •
~
rl
...
'"' r2
Q -
tI L
1'2 '"'
... <'" '"'"
~
:01 < '" <
.... ......
aA 28.25 .242 -5880 -.0236 12.75 -.00185 a 12.75 .00185 A
PI I
Ab
bB
.
.146
.146
4710 .0314 ""
-2355 -.0157
-.00246
-.00123.... A
b
.00246
.00123
b
B
d
P=Q=T
tlL Bc
cC
....
.093
.093
1177 .0123
1177 • 0123 ....
-.000965
-.000965
B
C ....
.000965
.000965
c
c
Cd
Dd
..
• 146
• 146.
-2355 -.0157
4710 . 0314
-.00123
-.00246..
..
d
D
....
.00123
.00246
C
d
Al~lh
De • 242 -5580 • 0236 -.00185 e .00185 D
Den. at c = .275+ (.004408- .0027)15 = .301" beam loaded with the elastic weights from Table
The slope of the elastic curve at the truss joint A7.ll. Table A7.l2 gives the calculation for the
points equals the vertical shear at these points joint deflections.
for the beam of Fig. A7.51.
Example Problem 39
Find the vertical deflection of the joints
of the Pratt truss as shown in Fig. A7.52. The
member deformationsAL for each member due to the
given loading are written adjacent to each memb-
er. Table A7.l0 gives the calculation of member f - - - - - - 5 @ 25" = 125"
elastic weights. Fig. A7.53 shows the imaginary
beam loaded With the elastic weights from Table Fig. A7.54
A7.l0. The deflections are equal numerically to
Table A7.11
the bending moments on ..t his beam.
Elastic Weight of Chord Members
0b .01855 x 25 = .465"
0B = .465 + .053 (AL in Bar Bb) = .518" lIember t:. L r w -ll
r
Joint
0c .01855 x 50 - .00387 x 25 - .833"
0c = .833 + .031 (AL of Cc) = .864 AK
DC
.0825
-.055
15.0
17.17
.00550
.00320
B
K
0D .01855 x 75 - .00387 x 50 - .00623 x KJ .096 20.0 .00480 C
25 = 1.03" CD -.069 21.14 .00322 J
J1 .106 22.50 .00471 D
DE -.0736 21.14 .00348 1
1H .113 20.0 .00565 E
.00437
J EE
HG
-.075
.108
17.17
15.00 .00720
H
F
A,r=~~-';';---'-'~~~L...:..='"-----::-~11 G
Elastic Weight of Web lIembers
AB .061 30 .00203 b
0 0
Be .061 30 .00203 b It) -<
CD .066 30 .0022 c '<t' co
0 0
bc -.083 30 .00277 C q q
Cd -.091 30 .00304 D
A G
Elastic Weight of Web lIembers B K C J D E H F
ex> '<t'
0 a>
P Q I:-
-<
q Fig. A7.55
...q
0
0 0 A
ex>
'<t'
co
I:-
ex>
C')
...
C')
co
0
a>
co
...
C')
co
I:-
ex>
C')
ex>
'<t'
co
AB
BK
.02158
.01!106
.02158
.04064
.27
.508
B
K
0 0 0 -< 0 0 0 KC .01628 .05692
q q .710 C
q q 0 CJ .01350
ql JD .0 13!1
.07042 .881 J
"ts...
j I I I t .07181 .900 D
G
~i
D1 .00248
B c D E F 1B -.00971
.07429 .930 1
...
-< -<
EH
HF
-.U1361
-.024
.06458
.05097
.026"5
.81
.638
.336
E
H
q q FG .02704 F
Fig. A7.53 .000u,," 0 G
.. error
Example Problem 40
Find the vertical joint deflections for the Example Problem 41
unsymmetrically loaded truss of Fig. A7.54. The Fig. A7.56 shows a simply supported truss
AL deformations for all members are given on the with cantilever overhang on each end. This sim-
Figure. Table A7.ll gives the calculation of plified truss is representative of a cantilever
the elastic weights, their signs a nd points of wing beam the fuselage attachment points being
application. Fig. A7.55 shows the imaginary
A7.36 DEFLECTIONS OF STRUCTURES
at e and en. The ~L deformation in each truss lever over-hang portion of the truss relative to
member due to the given external loading is giv- support points e and e' .
en on the figure. The complete truss elastic Since the cantilever portion is not fixed at
loading will be determined. With the elastic e since the restraint is determined by the truss
loading known the truss deflections from various between e' and e, this fact must be taken into
reference lines are readily determined. account in loading the cantilever portion. The
reactions on the beam of Fig. A7.58 represent
the slope at e due to the elastic loading between
e and e'. This elastic reaction in acting in the
reverse direction is therefore applied as a load
to the imaginary beam between e and a as shown in
A7.56 Fig. A7.59.
~ ~ ~ ~ to M eo') o:t'N '"
M ~
oM _ 0
r j. r j. I..• '"f I'
U':lNo:t'NMNQ'l1 C'> C'>
0 0000 0_ o 0
o
~ ~I ~
00 00 0000
·1 . r
L.!... I
,~,
a' Al b l B' c' C' d' D' e' E' f
Elastic Weight Loading
Fig. A7.57
Table A7.13
For this problem we need only to consider (7) For the truss in Fig. A7.66 calculate
the elastic loads between points band d as the deflection of joint C along the direction CEo
loads on a simple beam supported at band d E = 30,000,000 psi.
(See Fig. A7.61) The deflection at C with re-
spect to a line bd of the deflected truss
equals the bending moment at point c for the
loaded beam of Fig. A7.61.
hence
0c = .004743 x 20 - .00239 x 10 =.07 inches
'" 00 '<t'
C") '" C")
f 8i ~j ~j i b
A E
d Fig. A7.67 Fig. A7.68
C c B
Rd=.004743 Rb=.004667 (8) For the truss in Fig. A7.67, find the
vertical and horizontal displacement of joints C
Fig. A7.61 and D. Take area of all members carrying tension
A7.l8 Problems as 2 sq. in. each and those carrying compression
as 5 sq. in. each. E = 30,000,000 psi.
(9) For the truss in Fig. A7.68, determine
the horizontal displacement of points C and B.
E = 28,000,000 psi.
5000 5000
A +B tC D
20" ~OOO#
~t
T
30"
-.L G 4 ~ 4 tE
5060 5000 1-
Fig. A7.64
Fig. A7.69 Fig. A7.70
(1) Find vertical and horizontal deflec- (10) For the truss in Fig. A7.69, find the
tion of joint B for the structure in Fig. A7.62. vertical deflection of joint D. Depth of truss
Area of AB = 0.2 sq. in. and BC = 0.3. E = 180". Width of each panel is 180". The area of
10,000,000 psi. each truss member is indicated by the number on
(2) For the truss in Fig. A7.63, calculate each bar in the figure. E = 30,000,000 psi. Al-
the vertical deflection of joint C. Use AE so calculate the angular rotation of bar DE.
for each member equal to 2 x 107 • (11) For the truss in Fig. A7.70, calculate
(3) For the truss of Fig. A7.64 determine the vertical and horizontal displacement of
the horizontal deflection of joint E. Area of joints A and B. Assume the cross-sectional area
each truss member = 1 sq. in., E = 10,000,000 for members in tension as 1 sq. in. each and
psi. those in compression as 2 sq. in. E = 10,300,000
(4) Determine the vertical deflection of psi.
joint E of the truss in Fig. A7.64 (12) For the truss in Fig. A7.70 calculate
(5) Determine the deflection of joint D the angular rotation of member AB under the given
normal to a line joining joint CE of the truss truss loading.
in Fig. A7.64. 1000# 5000
100'~ t i A====;<:~:¢==~=E
A
10" B
t 1 10' tA
10' rIO'
o.... <=>
o
.... Fig. A7.71 Fig. A7.72
500#
29#/in.
~~=~~==C:==~l~O,~,-t\E
Fig.A7.73 1000#
Reaction
Fig.A7.81 Fig.A7.82
(15) Fig. A7.73 illustrates the airloads on (23) In Fig. A7.81 find the vertical move-
a flap beams ABCDE. The flap beams is supported ment and the angular rotation of point A. Take
at Band D and a horn load of 500# is applied at EI = 12,000,000.
C. The beam is made from a 1"-.049 aluminum (24) Determine the vertical deflection of
alloy round tUbe. I = .01659 in4 ; E = 10,300,00 point A for the structure in Fig. A7.82. EI =
psi. Compute the deflection at points C and E 14,000,000.
and the slope of the elastic curve at point B. 100#
100# 100 100 100 100# 100
1"-3" 1""3" 1:-3 -r- 3 --l F 9" ± 9 " ± 9 " i 't1 it
.~:l:_=======O!;:C Fig. A7.83
t C D t B A ~ 20"
t
Fig. A7. 78 Fig.A7.79 Fig.A7.80
A&
(27) Using the matrix equation 2U = LqiJ
[ aij] {qj} compute the strain energy in the
truss of Fig. A7.63 (Problem 2). The member
flexibility coefficient for a member under
uniform axial load is L/AE (see Fig. A7.35a).
Ans. U 22.4 lb. in. =
(20) In Fig. A7.78 EI is constant throughout. (28) Using matriX equation (23) compute
Calculate the vertical deflection and the angu- the strain energy in the beam of Fig. A7.71.
lar rotation of point A. Note: the choice of generalized forces should
(21) For the curved beam in Fig. A7.79 find be made so as to permit computation of the
the vertical deflection and the angular rota- member flexibility coefficients by the equations
tion of point A. Take EI as constant. of p. A7.19. Ans. U = 3533 lb. in.
(22) For the loaded curved beam of Fig. (29) Re-solve the problem of example
A7.80, determine the vertical deflection and the problem 25 for a stepped cantilever beam whose
angular rotation of the point A. Take EI as I doubles at point "2" and doubles again at "3".
constant. (HeaViest section at built-in end.)
A7.39
T
the influence coefficient matrix relating
vertical deflections due to loads P 1 , Pa, P a
and P 4 applied as shown. Member areas are
shown on the figure.
B c t = .051"
A CD 1.0 I]J G = 3.85 x 106 psi 20"
1~ ~
10"
.-
I.
Ai = 7.5 x10' Ibs
30"
1 \AE = 'xlO'
lbs.
500#
Fig. A7.86
Fig. A7.85
Answer.
(34) Find the influence coefficients re-
lating deflections at points 1 and 2 of the
1
["67
44.95
44.95
106.56
44.95
99.9 ~.~
38.28
simply supported beam of Fig. A7 87. Use
matrix methods.
~mnJ = E 44.95 99.9 99.9 38.28
33.28 38.28 38.28 33.28 ~ 20" - -.... r.-
DD 1,
20" ./ 20"-l
TEXT BOOKS WITH MATRIX APPLICATIONS Wehle, L. and Lansing, W., A Method for Reducing
the Analysis of Complex RedUn~tructures to
"Elementary Matrices", R. A. Frazer, W. J. a-Routine Procedure, Journ. of Aero. Sci.
Duncan and A. R. Collar, Cambridge University VoL 19, 1952.
Press. Langefors, B., Analysis of Elastic Structures by
"Introduction to the Study of Aircraft Vibration Matrix Transformation with Special Regard to
and Flutter", R. Scanlan and R. Rosenbaum, Semi Monocoque Structur~ Journ. of Aero. Sci.
Mac Millan, New York. 19, 1952.
Langefors, B., Matrix Methods for Redundant
Structures, Journ. of Aero. Scr:- VoL 20, 1953.
TECHNICAL PAPERS Falkenheiner, H. Systematic Analysis of Redundant
Elastic Structures by Means of Matrix Calculus,
Benscoter, S. U., "The Partitioning of Matrices Journ. of Aero. SCi~ 20, 1953.-----
!P Structural AnalySIS", Journ. of AppL Mechs.
VoL 15, 1948. Argyris, J. and Kelsey, S., Energy Theorems in
Structural Analysis, Aircraft Engineering,
Oct. 1954, et. seq.
Example Problem A
By way of illustration, the problem posed
Fig. A8.1d by Fig. A8.2 was carried to completion. The
Singly redundant beam with root bending
bending moment was given by (x, y, s measured
from the left ends of the three beam divisions)
moment Me undetermined by statics.
M (500 + R ) x
x
-COROLLARY- (500 + Rx ) L
M 2 + (Rx -500) y
If n member loads have been assigned ar-
bitrarily while establishing equilibrium with
the external loads, relative movements of the M = Rx (L - :6)
elements will result, violating continuity at n
points. n zero-resultant stress (force) dis- Then
tributions may then be superposed to reduce the
relative motions to zero. The resulting stress
distribution is the correct one. U :: 1
2 fM dx
Er:: 1
2EI
2
f L/
2
2
0 (500+Rx )2X 1x
A8. 2 The Theorem of Least Work.
_1_fL[/~500+RX)L
A theorem extremely useful in the solution
of redundant problems may be obtained from + + (Rx - 500)y]2 dy
Castigliano's Theorem. Consider first the 2EI 2
problem of redundant reactions such as in a o
EI = constant
r Differentiating under the integral sign, (see
b 1. p. A7.8)
t---
500 + R x
L --4--
500-2R
L ---t Rx
L/2
x
~ :: l (500 + Rx) x 2dx
Fig. A8.2
A singly redundant beam with one reaction given an ar-
aR x EI
Jo
r
aRx
r:(~
Eq. (1) is true for all redundant reactions
occuring at fixed supports. Because it corre-
sponds to the mathematical condition for the (Rx - 5001 + Y) dy + Rx L - Wdo
minimum of a function, eq. (1) is said to state
AB.3
Rx =- 1500
~ Ibs. =- 93.8 Ibs.,
o
LX
3L
-!.
3L
dx
Example Problem B
Determine the redundant fixed end moments
for the beam of Fig. A8.2(a).
0= ML )~ -:L) dx
Fig. AB.2a
~ + 2PL ~)L
A doubly redundant beam with two reactions given + 3L - 0 X (1 - ;L) dx
two arbitrary values.
M = ~ +
~ + 2PL - ML
3L
x O<x<L
o = ML
I LX
o
- dx +
~
+ 2PL - M LX 2
L
~
0
-
~
dx
°{y<2L
Then
u
1
= 2EI
Jf,~ML + r~+ 2PL
3L
- ML
X
J2
dx
Evaluating the integrals and solVing simultane-
ously gave
~ -- ~ PL
+-1. M + P -
L L -"R
T'Lj2 dy 9
2EI 2
3L ~ = - PL
9
Differentiating under the integral sign (see AB. 2. 2 Redundant Stresses by Least Work.
remarks on p. A7-8) The Theorem of Least Work may be applied to
the problem of determining redundant member
forces within a statically indeterminate structure.
au Thus, in an n-times redundant structure if the
aML redundant member forces are assigned symbols X,
Y, g, - - - etc., the values which these forces
must assume for continuity of the structure are
such that the displacements associated with these
forces (the discontinuities) must be zero.
Hence, by an argument parallel to that used for
redundant reactions, one writes,
A8.4 STATICALLY INDETERMINATE STRUCTURES
au
ax == 0 30 \2 2
- 58.3 X) x dx
au o - - - - - - - - (2)
ay
1
+--
2EI
etc. AC
~ X) \5: 2 dx
A
30 (1000 -
58.3 EI BC 0
30
+ (~)2
58.3
_X_
EI AC
J0
y2 dy 0
Fig. A8.3
A singly redundant structure with one member force given an
arbitrary value (X).
58.3 X + 11,032 X + 6620 X
Solution: EA EI EI
AB BC AC
The tensile load in the cable was treated
as the redundant load and was given the symbol
X (Fig. A8.3(b)). The strain energies con-
sidered were those of flexure in portions AC,
CD and BC and that of tension in the cable AB. Putting
Energies due to axial forces in the beam port-
ions were considered negligible.
AAB 0.025 in 2
The bending moment in BC (origin at B) was
50 gave
MAC == 58.3 X . Y
X 613 lbs.
In CD:
Then
MCD == 50,000
l\c == [1000 - 5~~3 ( 613 )J x = 685x
The strain energy was therefore
50
X L)
U = 2"" AE AB
2
( MAC == - -
58.3 613 • Y 526 Y
A8.5
X 1
a sin " Q d Q
bending moment distribution in the ring.
3
600
I
Rf p
[ 1 (1 - cos Q) sin Q d Q
a
(c) (d)
Fig. A8.3
+ PR J60cos Q sin Q d Q
AE
Solution:
a
The axial load in the floor was taken to
be the redundant (since the floor was assumed 90 0
rigid, this could have been thought of as a re- " Qd Q+ ~ sin" Q d Q
dundant floor reaction from fixed supports).
The loading is shown in Fig. A8.3(d). J60 0
s P cos ~ + X sin ~
Therefore
S X sin Q
U " ~, tE60 0
R sin 0 - PR (I - cos OIJ RdO
Example Problem E
The portal frame
of Fig. A8.3(e) is
I [X 0
three times redundant.
90 Set up the simultaneous
+ 2ir
60°
R sin ~- PR/2 T RdQ
equations in the re-
dundant forces. The
relative bending stiff-
nesses of the segments
are given on the figure.
+ X sin Q J RdQ Solution:
Fig. A8.3e
-+-
1,000# + J~-50,000 sin Q + M + 50 r~;o+ sin Q) -5ar(1 - cos Il)](cos II _ l)bOodQ
S
t .......,
~
Fig. AB.3f 50,000 M Fig. AB.3g After evaluation of the integrals the equations
obtained were
.14-92M + 9.682V 7.4-59T = 2.778 :x' 10"
9.682M + 763.1V 4-84-.OT 288.3 x 10"
{
-7.4-59M - 4-84-.0V + 614-.9T = -111.15 x 10"
V
.. Fig. AB.3h
1
~T
S
Fig. AB.3i
~
- lOOT
direct application by the calculus, as in Art.
A8-2.1 and A8-2.2, is often impractical. For the
majority of problems the work is facilitated if
carried out by the techniques of the Method of
Dummy-Unit Loads.
The following derivation is for a doubly
redundant truss structure. The extension to a
more general n-times redundant structure, in
The composite bending moments as functions of which other loadings in addition to axial (flex-
s, g and s' were ure, torsion and shear) are present, is indicated
later.
MAE M + Vs Consider the doubly redundant truss of Fig.
A8.4-(a). It may be made statically determinate
~C 50,000 sin g + M + 50 V + 50V sin g by "cutting" two members such as the diagonals
indicated. Application of the external loads to
- 50T (1 - cos g) this determinate ("cut") structure gives a load
distribution, "S", computed by satisfying static
MCD = 1000 s' + M - Vs' + 50 V - 100 T eqUilibrium. At this time discontinuities appear
at the cuts "x" and "y" due to the strains
Then since developed.
= 12 JM"ds au au au
~f
u ~ and aM = av = aT = 0,
~
one has,
50
au =0 _ . (M + Vs) d
aM - 2.50 s S loads Ux loads Uy loads
1 o Fig. AB.4a Fig. AB.4b Fig. AB.4c
+ I o
[n_50 ,ooo sin Q + M + 50V(1 + sin Q) - 5ar(1 - cos II)] 5Od1l
1.50
To compute these and subsequent displacements
the Method of Dummy-Unit Loads may be used (Art.
50
A7-7). For this purpose virtual loads are placed
+ (lOOOs' + M - Vs' + 50 V - 100 T) d ' alternately at the x and y cuts as in Figs. A8.4-(b)
J
o 2.25 s
and (c). From the dummy-unit load equations
50
!l!!:=0= (M+Vs)sds
av 2.25
J
0
A8.7
~
Y causes a distribution dundant, eqs. (4) and (5) are applied by setting
YUy • The relative dis- Y=:o giving
placement at cut x due to
redundant load X is given X Z ux"L =: _ Z SUxL
by (0 is read "displace- Fig. A8.4d AE AE
xx
ment at x due to X"). or, simply,
2
6 =: Z Xux • uxL X Z ux L
xx AE AE
X - - - - - - - - - - - - - - - (4a)
and at cut.y by (read 6 as "displacement at
y due to X"). yx
6
yy
=: Z YU y • UyL =: Y Z Uy"L Example Problem #1
AE "AE
19:I~~~~
and
YUy • LlxL =:
Y Z uyuxL
6
xy Z
AE
tlOOO# 1000#
cut under the simultaneous action of the three
stress systems S, X·u and Y·u is Fig. A8. 5 Fig. A8.6 Fig. A8.7
x Y
Fig. A8.5 shows a single bay pin connected
6 + 6 + 6 =: Z SUxL + X Z uxuxL truss. The truss is statically determinate With
xo xx xy AE AE respect to external reactions, but staticaily
indeterminate With respect to internal member
loads, since at any joint there are 3 unknowns
With only two equations of statics available for
a concurrent force system. The truss is there-
and fore redundant to the first degree. The general
6 + 6 + 6 Z SuyL + X Z uxuyL procedure for solution is to make the truss
~ yx YY AE AE statically determinate by cutting one of the
membersjon Fig. A8.6, member bc has been selec-
ted as the redundant member, and it is cut as
+ Y Z UyUyL shown. The member stresses S for the truss of
AE
AB B STATICALLY INDETERMINATE STRUCTURES
\1//
Fig. A8.6 are then determined, the results being ABC
~~<r t~
recorded on the members and also entered in
Table A8.1. In Fig. A8.7, a unit 1# tensile /
dummy load has been applied at the cut section
of the redundant member bc, and the loads in all ~l#
the members due to this unit load are calculated o o
l1000
The results are recorded on the figure and also
in Table A8.1 under the head of u stresses. The Fig. AB.9 Fig. AB.I0
solution for the redundant load X in the redund- S loads u loads
ant member bc is given at the bottom of Table
A8.1. The true load in any member equals the S Example Problem # I-A; Deflection Calculation in
stress plUS X times its u stress. a Redundant Truss
T.ab1e A8.1 Calculations of the deflections under load of a redundan
structure are made by application of the methods of Chapter
lIember L A S u SuL u 2L True Stress A-7. Since, however, there are certain pitfalls as regards
-r A = S + Xu symbols and also some important special techniques, the
ab 30 1 0 -.707 0 15 395 following examples are given at this time. The extension of
bd 30 1 -1000 -.707 21210 15 -605 the method to more complex structures is immediate and no
de 30 1 0 -.707 0 15 395
ea 30 1 0 -.707 0 15 395
further work on deflections of redundant structures is given
eb 42.4 2 0 1.0 0 21.2 -559 in this chapter (excepting in the case of the matrix methods of
ad 42.4 1.5 1414 1.0 40000 28.3 855 Arts. AB. 10 et. seq.).
Yo 61210 109.5
X • true load in redundant member be Find the horizontal movement of point "d"
Su1 of Example Problem # 1 under the action of the
_l: A
X =
-2 • -61210
109.5
= -559# load applied there.
u L
Z A
Solution: The equation used to find the
deflection is Eq. (18) of Chapter A-7. Written
Example Problem # 2 for application tv truss deflections it is
(see Example Problem 13, p. A7.11)
Fig. A8.8 shows a singly redundant
3-member frame. Find the member loadings.
- - - -(A)
Member areas are shown on the figure.
3'~/.4
from their meanings in the redundant stress
calculation. For a deflection calculation the
symbols of eq. (A), above, mean: "S-loads" are
o the true loads of the redundant structure due
hooo# to application of the real external loading;
Fig. AB. B
"u-loads" are the loads due to a dummy-unit
(Virtual) load applied at the external point
where the deflection is desired and in the di-
Solution: Member OC was selected as the rection of the desired deflection.
redundant and was cut in figuring the S-loads,
as in Fig. A8.9. Fig. A8.10 shows the u-load ThUS, the "s loads" for use in Eq. (A) are
calculation. The table completes the calcu- the true stresses (the solution) of Example
lation. Problem # 1.
SuLL:
A
X - - u 2
L - 274.1 lb.
L:-
A
A89
Proof of Theorem
==
Again the symbols "S" and "un are to be re-
interpreted for a deflection calculation as To prove the theorem above we return to th e
explained above in Example Problem #l-A. The virtual work principle and the argument from
"S-loads" are now the "true loads" computed in which the dummy-unit loads deflection equation,
Example Problem #2, above. The "u-loads" are Eq. (18) of Chapter A-7, was derived (refer to
loads due to placing a dummy-unit load acting p. A7 .10). It will be remembered that the de-
horizontally on the structure at point O. flection was shown to be equal to the work done
Since this load acts on a redundant structure by the internal virtual loads (u-loads) moving
it would appear that another redundant stress through the distortions (n) due to the real
calculation is required. However, this is not loads, Le., 6 = Z un. The internal virtual
necessary. loads are those loads due to a unit load acting
at the point of desired deflection.
Theorem: For the u-loads in a deflection
calculation any set-of stresses~loads) in Now for the statically indeterminate
static equilibrium-with the dummy-unit load structure these internal virtual loads (u-loads
may be used, even from thElsimplest -
of -
"cut" are, in general, indeterminate since the dummy-
structures. - - - - - - - unit load is applied at an external point of t he
structure. However, we recall that,
This theorem says that to get the "u-loads' i-any stress distribution in static
for this deflection calculation we may "cut" equlli bri um with the "applied load" (for the
anyone of the three members and get a satis- moment now we are thinking of the dummy-unit
factory set of u-loads by simple staticsl load as the "applied load") differs from the
Before proving the theorem we complete the correct (true) distribution only by a stress
calculation in tabular form as shown. The distribution having zero external resultant
"u-loads" were obtained by cutting member OC (p. A8.1).
and applying a unit load horizontally at O.
ii - a zero-resultant stress distribution
moving through a set of displacements does zer o
work.
A8.10 STATICALLY INDETERMINATE STRUCTURES
6 = ~ A
~RUE = Table A8. 2
CE 50 .5 0 0 1 0 0 0 100 0 -932
Q,.E.D.
BE 40.7 -3000 -.8 -.8 137,000 137,000 36.6 36.6 36.6 -1015
A8. 5 Trusses With Double Redundancy ED 30 .5 -750 0 -.6 0 27,000 0 21. 6 0 -190
which Eqs. (4) were derived, will be solved AF 40.25 0 -.8 0 0 0 105.0 0 0 1240
for a loading P:I. = 2000# and P 2 = 1000#. Z 562,200 478,000 341.1 452.6 36.6
Choices of redundants were made identical with
those of Fig. A8.4a. Figs. A8.11, A8.12 and
A8.13 show the S, u and u load calculations SUbstituting from the table into Eqs. (4)
respectively. x y gives (common factor of E divided out)
Solving,
X - - 1550#
Y--932##
Fig. A8.11
Finally (see Table A8.2)
S loads True Load =S + XUx + YUy.
AB.ll
AB.5 Trusses With Double Redundancy, cont'd. SolVing the two equations for X and Y, one ob-
tains X = 521# and Y = 416#. The true load in
Example Problem 3 any member = S + Xu + Yu which gave the values
x y
Fig. A8.14 shows a structure composed of in the last column of the table.
four co-planar members supporting a 2000# load.
With only two equations of statics available for AB. 6 TrusSi!S With Multiple Redundancy.
the concurrent force system the structure, rel- By induction, eqs. (4) may be extended for
ative to loads in the members, is redundant to application to trusses which are three or more
the second degree. times redundant. Thus for a triple redundancy,
Solution:
u 2L UxuyL UXugL
XZ_x_+YZ--+;?,Z
Fig. A8.15 shows the assumed statically AE AE ~
determinate structure; the two members CE and
DE were taken as the redundants and were cut at UxUyL u ~L uyugL SUyL
points x and y as shown. The member stresses XZ + Y Z -y-- + ;?, Z =- Z -(6)
for this structure and loading are recorded on
AE AE AE AE
the members. Figs. A8.16 and A8.17 give the
ux and uy member stresses due to unit (1#) uxUgL uyugL Ug 2L SUgL
XZ + Y Z - - - + ;?, Z - L:
tensile loads applied at the cut faces x and y. Ali: AE AE AE
Table A8.3 gives the complete calculations for
eqs. (4) and (5). The load in the redundant and after solving for X, Y, ;?',
member CE was designated X and that in DE as Y.
True stresses =S + Xu + Yu + ;?,U - - - - -(7)
r;j +t--3,,2',~ T x Y 13
\ ~L~~./Y
\JL:
AB.7 Redundant Structures With Members Subjected to
6' Loadings in Addition to Axial Forces.
O~/Y Eqs. (6) are extended readily to cover
E~ problems in which flexural, torSional, and shear
t 2000# '2000# loadings occur. ThUS, for a three times redundant
Fig. A8.14 Fig. AB.15 structure
~
"" jil' xo
...;..-:: 1#
~I Xa + Ya +;!,a 6
yx yy YI3 yo - - - - - - - - -(8)
Ux loads u y loads
Xa gX + Ya gy + ;?,a 1313 6
130
Fig. AB.16 Fig. AB.17
A -A- - - Z -A-
L, L,
a
xy
= a
yx
= Z uxuyL +
AE
Jmxmydx
EI
Substituting:
0
Ux uy
.806 1.154
SUx L X 10- 6 S'i L X 10- 6 U 2L
--,;:-
0 0
~
280.5
~
576.5
uXi('L True Load =
402
S+Xux+Yuy
900
a
YY
= L:
2L +
~
U
AE
J 2
my dx +
EI
- - - - - etc •
AE .286.5
BE .172 2000 -1. 564 -1. 729 -2.253 -2.488 1763 2150 1948 465
Solution:
J J
~
Su L +
_x_ Mm
_x_ dx + Tt
_x_dx
axo AE EI GJ
The shear flow in the sheet panels was
chosen as redundant. Because of symmetry the
problem was only singly redundant. Fig. A8.19
qqxdxdy shows the Ux and qx loadings due to the redund-
Gt ant shear flow X = 1. The real loading in the
determinate structure consisted of a constant
load P in the central stringer alone. The
Byo ~ 2 sUi + J EI
Mmydx + J equation solved was (ref. eqs. (8)).
~ AE
-
qqy dx dy
x (J ux: II ~ "G:Xd}dx +
Gt
-0 SU~EdX + Jj q \~dY)
etc. where
Axial Forces =
1bAE )
+ GtL2
True Bending Moments - - - - - - - (9)
Therefore the true stresses were
1
PROOT = P-2LX = P L2 Gt in central stringer
1 + AEb
etc. P 1
= LX = - in side stringers
2 1 + AEb
2
Example Problem 4 L Gt
The symmetric sheet stringer panel of Fig.
A8.18 is to be analyzed for distribution of load Example Problem 5
P between stringers. As a first approximation, The problem of Fig. A8.19 is doubly re-
assume constant shear flow in the sheet panels. dundant as shown. Determine the bending moment
All stringers have the same area. distribution. Both members have equal sectional
L properties.
///////1
t A~
1
AE}
A= x= 0
o L~EI/;}::'bnt '~S'2~
const. L
~~ D .;:;,S~=-~2;:;P::;::~~~:;:;::;:~;i
~LL.--...1L---U1 [1 l1 fJ tJ "Clamped" f-- L ..j.. L i P
/""3" -
"Pinnedj' M~
ry Y ~~;?1- Pt
b-tb
I-- ~
Fig. A8.19 Fig. A8.20
S, M loadings
P ux, iix loadings
x Fig.A8.18
Fig. A8.19
A8.13
Solution: (Lv'2 + L ) (
3
L (1 + v'2!L \
3
AE
L
= 100 EI
17 giving
A
=- ~S:t J"'l0 +
Then using the previous notations, the appropri-
ate equations for the redundant forces are
< x 2L u L
~
u
After evaluation of the integrals and mUlti- X U + Y
plying through by L2 these become 2JL =
~ AE AE
- -(10)
3
~Lv'2
~
x (( 1 + 2v'2) .!:... + L \ + Y + L3 \ u u L ~u 2L ~ Su L
AE 3Ei) AE 6Ei} X 2..JL+Y .lL-=_ -y--a
AE AE AE yi
= _ PL ((2 + 4V2) L _ ~)
\" AE 6EI The ItS loads" of eq. (10) are present because
of applied external loads. These mayor may not
be zero depending upon the problem.
A8.14 STATICALLY INDETERMINATE STRUCTURES
Solution:
0
yT+ 0yx + 0yy = 0
~ 12J
Ux
.8
loading
1#.6
Fig. A8.25
Load equations (ref. Chap. A7, Arts. A7.7 and
A7.S) as
Ie
00
30
'0
20
• .8
• .8 ....
0
0
.853 0
~T
0
-gaT -1.21
.. ....
CD 00 '0 • .8 0 .853 _1.61
load distributions due to virtual loads at the DA '0 20 - 1.0 1.50 •. 80 0.0
(12) are written as integrals rather than finite DB '0 O. 1.0 0 1.111 2.01
sums to allow for possible variation in a and T, 5.008 x la-- 1.50 x 10- -.90 J: 10-· - 9 a T
5.008 X - .90 Y = 9 a T x 10 6
~ Ux 2L
X + Y ~ u.:!:....L
u L = u a T dx { _ .90 X + 1.50 Y o
AE AE -J x
- - (13) Solving,
~ u~+y ~ 2L
X
u L
AE
u-y-=
AE -J uy
a T dx X = 2.01 a T x 10 6
Y = 1.21 a T x 10 6
Equations (13) may, of course, be extended
for application to structures other than True stresses are given in Table AS.4.
trusses. The expressions appropriate to other
loadings have been developed previously (eqs. Example Problem 10
(8) et seq. in this chapter and other equations The upper surface of the built-in beam of
in Art. A7 •8 ) • Fig. A8.26 is heated to a uniform temperature T.
AB.16 STATICALLY INDETERMINATE STRUCTURES
Through the depth of the beam the temperature Then the equations corresponding to eqs. (13)
varies linearly to normal (T = 0) at the lower were written (see also eqs. (8)).
surface. Determine the end moments developed,
neglecting axial constraint and influence of
(J ux 2ds + JmxEI2dS) + (J ~ + mxmYdS)
axial forces.
r- L T -I
X AE Y AE J EI
J
~1
X=1"# X=1"# + g uxu g ds + mxmg dS) =
, :---r , 1'-- 1
---J J (J AE EI
(J
T
u ds + J ~m J-y--
EI constant Virtual Loading
m 2dS)
Fig. AB.26a Fig. AB.26b
X
( Ju~
AE
--y-
EI
dS) + Y u 2ds +
-y--
EI EI
Solution:
J myf!Jm dS)
The problem was only singly redundant be-
cause of symmetry and was made deter.minate by
+g
(J u~
u ds
AE
+
EI
=-0
yT
1 R aT
The redundant moment equation was (by analogy to Ef X - Ef Y + o·g = + 11
eq. 13)
X
m 2dx
_x__ = - 6xT
-R 1 (1
EI X + 2 AE + EI
2) Y + o· g
R aRT
h
EI J
"Therefore O-X + O·y + ( - 1 + -R 2) g=;O
AE EI
L
X EI - - TaL/h Note that from the last of these equations
g = 0, as it must because of the symmetry of the
X = - TaEI ring. Solving the first two equations
-h-
X = aT:I
The redundant moment compresses the upper fibers
as was to be expected.
Y 0
Example Problem 11
Complete the problem begun in Example A non-zero value of Y would produce a vary-
Problem 24 Art. A7.8, viz, that of computing ing bending moment which cannot be because of
the thermal stresses in a closed ring whose symmetry. Hence this result too, is rational.
inner surface is uniformly heated to a tempera-
ture T above the outside. AB.10 Redundant Problem Stress Calculations by
Matrix Methods.
Solution: In the following section the indeterminate
structural problem is formulated in matrix no-
The ring was made determinate by cutting at tation. The reader is assumed to be f~~iliar
the top as in Fig. A7.30(b). The unit loadings With the matrix applications of Art. A7.9 and
and thermal deflections were determined in the the elements of matriX notation and arithmetic
referenced example. The results of deflection (see Appendix).
calculations made previously were The stress distribution of the structure is
specified by a set of internal generalized forces,
°xT = 2n RaT qi' qj'* (ref. Art. A7.9). Unlike the cas~
h
the determinate structure, these qi' qj cannot be
2
oyT = -2n R aT
11 * In the case of indeterminate structures, wherein some of the
support reactions may also be redundant, these reactions
also are denoted by q's. (see Example Problem 13a).
OgT 0
A8.17
(21)
The reader may satisfy himself that the Eq. (21) is a set of simultaneous equations
"cr0ss product" term in the middle of the above for the redundant internal forces q , q. It
r s
result is correct by observing that, because of may be compared with eq. (6) of Art. A8.6, to
the symmetry of [Uij ], which it corresponds. Eq. (21)" may be solved
directly from the form there displayed or its
solution may be obtained by computing ~rs:J,
L~J [~iJ ~ii] ~j;]{Pn}= the inverse of the matriX of coefficients, giVing
(22)
LPmJ [~iJ ~ii] ~j~ {qs}
The various matrix triple products occurring The matrix product - ~r;~ ~r~ gives the
above are assigned the following symbols, each values of the redundant forces for unit values
having the interpretation given (compare With of the external loads. This may be given the
eq. (24) of Art. A7.9)
symbol ~s;] so that
~m;] == [~iJ ~iil ~j~ - - - - - - - - -(16)
- the matriX
AB.19
(~i;] - ~iJ ~s[] ~s;] ) {Pm} The moments qz and q4 were taken as the redund-
ants. With these set equal to zero, the internal
force distributions due to application of unit
(~i~ + [!i~ ~rmJ ){Pm
} values of P~ and Pz were determined, giving
(23)
(24)
1
IlL
0
-IlL
1
~~
q2( tl
ql L
} (bl = 6EI
q4( tl
q3
Fig. AB.27
~r;] = [~!J ~ljJ ~j;]
Solution:
.~ L 'IL
J~'/3 L/2 0
:J [~L
n
1 'lL
EI 0 0 -IlL 1 L/2 1 0
The choice of internal generalized forces
is shown in Fig. A8.27 (b). The appropriate 0 0 L"/3 L/2 0
member flexibility coefficients were arranged in 0 0 L/2 1 0
matrix form as (ref. Art. A7.10 for coefficient
expressions) .
= 6E1
L
.. ~O ~
1 2
l
AS. 20 STATICALLY INDETERMINATE STRUCTURES
The inverse of Cars] was found (ref. though the matrix of member flexibility coe-
appendix) fficients was expanded to a 6 x 6, the
coefficients for qe and qe were zero. Thus,
~rs
a _;I.
7L C
J = 6EI -1
2
-J 4 o o o 0
1 o o o 0
Next, the unit redundant load distribution
was found (eq. 22). o
o o 1 o 0
o o o o 0
o o o o 0
L J
1 0 _l/L 0 o286 -o42 o o
1 0 0 1 0 .143 -.286 o 0
"" L +
0 1 l/L - l/L
and, with P;I. and P sa zero, successive applica-
0 0 0 1 tions of lmit redundant forces qe and qs gave
-1 0
~1 1
1.286 .428
2 L
-1 -1
0
~
1
L 2 - .286L -.428L ~1~ 2L L
3 - .429 .858 1 0
4 .143L -.286L 0 1
(The tabular form of presentation of the Then, multiplying out per eqs. (17 ) and (18)
matrix G , above, is used here only to indicate
im
clearly the functioning of the subscript nota-
tional scheme. In general, it should be unnec-
essary to callout the subscripts in this
fashion excepting for the larger matrices, for
~r~
L2
2EI bJ
the handling of which, the tabular form may
prove helpful.)
The redundant beam problem of Fig. A8.27 is The inverse was found:
to be re-solved using the redundant reactions as
the unknowns.
6EI -51
Solution: 7L 3 lsJ
The support reactions under the loads P;I.
and P sa were given the symbols qe and qe, respec- Finally,
tively, positive up. These forces did not enter
into the strain energy expression so that, al-
A8.21
1.286 .428
.286L -.428L
1 - .428 .857
-[
.143L -.286L
-1.286 -.428 Loading for column { gi2} of the matrix [gimJ
1.714 -.428 Fig. A8.29
Example Problem 14
The continuous truss of Fig. A8.28 is
twice redundant. It is desired to analyze it
for stress distributions under a variety of
loading conditions consisting of concentrated
vertical loads applied at the four external
points indicated.
Solution:
Fig. A8.28
20
Ix 1
3
0
5
0
20 2 .0 0
The internal generalized forces (qi,qj) 20 3 l.0 0
20 4 .5 .5
employed were the axial loads in the various 20 5 0 l.0
members. These were numbered from one to 20 6 0 .5
thirty-one as shown on the figure. The member 20 7 0 0
flexibility coefficients in this case were of 40 8 0 0
the form a i = L/AE (Ref. Fig. A7.35a). The 40 9 -.25 0
40 10 -.75 0
coefficients are written as a column matrix be- 40 11 -.75 -.25
low. (They were employed as the diagonal ele- 40 12 -.25 -.70
ments of a square matrix in the matrix mUlti- 40 13 0 -.70
plications, but are written here as a column to 1
~i~
ai 40 14 0 -.25
conserve space.) E 40 15 0 0
Member loads q3 and q6 were selected as 22.4 16 0 0
redundants. With q3 and q6 set equal to zero 22.4 17 0 0
("cut"), unit loads were applied successively 22.4 18 .56 0
at external loading points one through four, 22.4 19 -.56 0
the four stress distributions:thus found being 22.4 20 .56 0
arranged in four columns giving the matrix 22.4 21 -.56 0
[!i~ (below). By way of illustration, the 22.4 22 -.56 .56
22.4 23 .56 -.06
loading figure used to obtain the second column 22.4 24 -.56 .56
22.4 25 .56 -.56
of [!i;] is shown in Fig. A8.29. 22.4 26 0 -.56
22.4 27 0 .56
Next, unit forces were applied successively 22.4 28 0 -.56
at the redundant cuts "three" and "five" as shown 22.4 29 0 .56
in Figs. A8.30a and A8.30b. These loads were 22.4 30 0 0
arranged in two columns to give the matrix [!i~ 22.4 31 0 0
AB.22 STATICALLY INDETERMINATE STRUCTURES
~ 1 2 3 4
059 .111 .0065 . 003 51
[ .0035 .0065 .111 .059J
1 1.0
2 .50 -.50
3 The calculation was completed as per eq.
4
5 (23) to give [G ] , the values of the member
im
6 -.50 .50 forces for unit applied external loads.
7 1.0
8 - .50
I~
9 - .7::> .2b
1 2 3 4
10 - .2.5 .25
11
12 1 1.0 0 0 0
13 2 .529 -.445 .003 .001
.25 - .25 .059 .111 .006 .003
14 3
.25 - .75 4
15 .031 .059 .059 .031
- .50 5 .003 .006 .111 .059
16 1.12 -6
17 -1.12 .001 .003 -.445 .529
18 - .56 -.56 7 0 0 0 1.0
19 ..')6 .56 8 - .50 0 0 0
20 - .56 .56 9 - .764 .222 -.001 0
21 .56 -.56 10 - .294 .167 -.003 - .002
22 11 - .045 -.085 -.032 - .017
23 12 - .017 -.032 -.085 - .045
24 13 - .002 -.003 .167 - .294
25 14 0 -.001 .222 - .764
26 -.56 .56 15 0 0 0 - .50
27 16 1.12 0 0 0
.56 - .56 17 0 0 0
28 .56 .56 -1.12
29 -.56 - .56
18 - .527 -.498 .003 .001
30 -1.12
19 .527 .498 -.003 - .001
31 1.12
20 - .527 -.622 .003 .001
21 .527 .622 -.003 - .001
NOTE: VOIDS INDICATE ZEROES 22 - .031 -.058 .058 .031
23 - .031 .058 -.058 - .031
Multiplying out gave 24 .031 -.058 .058 .031
25 - .031 .058 -.058 - .031
26 - .001 -.003 -.622 .527
[136 -8·01 27 .001 .003 .622 - .527
L:8.0 136J 28 - .001 -.003 -.498 .527
29 .001 .003 .498 - .527
30 0 0 0 -1.12
31 0 0 0 1.12
From ~ F on Joint C.
~II"~.- _----
..... {\I )1'
!JJ
~ IK_ _ _ _ _llll---_::-
_ [,' _- ·-I From ~ F on Joint B.
rt--- H
F ~--- G .081759 q, - -.07617 q,. = .5227 q.
Fig. AS.31
q, - - .91593 qu = 0
{
-.1410 q, + + .4146 qu :::; -.8523 qae
MEMBER NUMBER LENGTH AREA L/A And so forth, for the other joints.
Note that in each case the equations were
AB 1 35.47 .565 62.8 arranged with the applied loads (P ) and the
BC 2 25.43 .499 51.0 n
BD 3 9.20 .165 55.8 redundant q's (qss, qS3, qS4) grouped on the
CD 4 25.43 .499 51.0 right hand side of the equal sign. This ar-
DE 5 35.47 .565 62.8 rangement was observed for all 21 equations,
FG 6 35.47 .565 62.8 af~er which the equations were placed in matrix
GH 7 25.34 .565 44.85 form as
HI 8 5.00 .165 30.3
IJ 9 25.34 .565 44.85
GJ 10 9.20 .165 55.8
JK 11 35.47 .565 62.8
AG 12 40.4 .630 64.1
BG 13 15.0 .165 90.9 i,j = 1,2, ..•••.. 24
BH 14 27.57 .500 55.14
HC 15 11.70 .395 29.62 n = 1,2,3
IC 16 11.70 .395 29.62 s = 22,23,24
DI 17 27.57 .500 55.14
DJ 18 15.0 .165 90.9 (Note that there were 24 equations here,
EJ 19 40.4 .630 64.1 the additional three equations being the ident-
AD 20 37.36 .565 66.1 ities
FJ 21 37.36 .565 66.1
GI 22 26.23 .500 52.46
DG 23 17.60 .165 106.6
BJ 24 17.60 .165 106.6 qS3 = qS3
qS4 = qS4 )
The structure was three times redundant.
In ~ space framework of p joints, 3p-6 inde- On the right hand side of the above matrix
pendent equations of statics may be written equation the matrices are shown "partitioned".
(p. A8.10). Here, however, stress details in The first three columns of [D] are the coeffic-
the plane AEKF are to be sacrificed; six equa- ients of P and the last three are the
tions are lost thereby since only net forces in 11
two directions in this plane can be summed.
3 x 11- 6 - 6 = 21 equations; 24 member unknowns.
Members 22, 23 and 24 were cut. * For structures other than trusses the equilibrium equations
The next step was to compute the unit are written for the various structural elements, equilibrium
stress distributions [gim] and [gir] • Rather of joints alone bemg inappropnate.
AB.24 STATICALLY INDETERMINATE STRUCTURES
where
~-186.4-
.10 513 2289 ~
~s~ =-~rs~J ~rJ
10-" -238 -201.0
- 31.4- 176 121.3
~ 1
1
.8416
2
2,727
3
6.074
22
-2.3059 0
23 24
0
1
2 -
.84-12
.00017
1.4-75
20208
1.296
,538
2 0 2.727 2.727 .9565 0 0 3 .01523 - .31.53 - .1190
3
4
- .00117 - .2771
0 -2.727
- .2821\.
-2.727
.0991
.9560
0
0
- .5227
0
4- .00017 -2,208 - ,539
5 .8440 -4.059 -7.943 2.786 1.236 -1.236 5 .6529 -3.058 -1.957
6 -1.217 .5456 -5.016 2.725 1.209 -1.209
7 - .8445 0 -3.8607 1.354 0 0 6 -1.4-04- 1.524-7 .832
e .1094 0 .5000 - .1754 0 0 7 - .84-4-3 .7352 - .761
~
0 3.862 -2.3205 0 0
c--=--.8446
0 .00013 - .0007 - .1914 - .5224 0 8 .1094- - .0952 ,0985
11
12
-1.4234 - .5800
.4415 -.6473
5.926
1.371
-3.734
- .4809
-2.417
-1.135
2.417
1.135
9 - .84-50 -1.260 -1.4-50
13
14
- .2623
.9188
.3845
0
- .8145
4.200
.2856
-1.473
0
0
- .8523
0
10 .0973 .0205 - .3338
15 - .5118 0 -2.310 .8;08 0 0
11 -1.04-94- -1.607 -1.84-2
16 - .5118 0 2.340 - .8208 0 0 12 .6638 - .314-3 .733
17
18
.9190
- .2619
0
.4090
-4.202
.7882
1.4737
- .2761
0
- .0327 -
0
.019(2 13 - .2355 .3896 - .264-1
19 .3605 .6884 - .9593 .2445 1.401
-1.302
-1.401 14- .9185 - .7998 .8283
20 .0005~ 1.3745 1.399 .4906 1.302
21 - .06621 - .0003 .3044 .2941 1.302 -1.302 15 - ,5116 .4-4-57 - ,4-61
22
23
0
0
0
0
0
0
1.0
0
0
1.0
0
0
16 - .5120 - .4-4-57 .4-61
24 0 0 0 0 0 1.0 17 .9193 .8002 - 0829
18 - .1061 - .3638 .3205
The member flexibility coefficients L/A 19 .14-34- .24-11 - .8512
were arranged as the diagonal elements of the 20 .2023 1.64-71 .6956
21 - .2680 - .3796 .5580
matrix [Q ij ] Then, multiplying out according 22 .0002 .54-3 2.289
23 - .1864- - .238 - .2010
to eqs. (17) and (18), 24- - .0314- .176 .1213
['77.9 -1137.0
-6430.~ Example Problem 15
The dOUbly symmetric four flange idealized
~r~ 195.0 - 151.7 -2263.0
box beam of Fig. A8.32 is to be analyzed for
-154-.4- 161.6 2273.3 stresses due to load application at the six
points indicated. Flange areas* taper linearly
from root to tip while sheet thicknesses are
-1l1~
1150 constant in each pane10 The beam is mounted
[ 2970
riGidly at the root, prOViding full restraint
~r~ 1150 1221 -1035
-114-8 -1035 1224-
* These are the "effective areas", being the flange area plus
* See references in appendix. adjacent effective cover sheet area plus one-sixth of the
web area. (The factor of one-sixth provides the same
moment of inertia as the distributed web area).
AB.25
against warping of the root cross section due The member flexibility coefficients were
to any torsion loadings. collected in matrix form as shown below. Note
that entries for a 44 , a~~, a 99 and a~o,~o were
collected from two stringers each (as well as
being doubled as discussed above). Coefficients
for these tapered stringers were computed from
the formulas of Art. A7.10.
)z 1 10 11 12 13
1 1220
" 15
2 51 200
3
•
5
6
1220
.. 2830
25.2
20" 7
6
60 000
2630
9 25.2 80.8 15.28
10 160.
11
12 6 , 0
13
I. 15.28 27.6
20" 15 15.28 27.6
\
NOTE: VOIDS INDICATE ZEROES. E = 1, G = .385
A = .12 in 2
(Typ.)
Cover sheet shear flows q2, q7 and q~2 were
selected as redundants. With these set equal to
Fig. AB.32 zero, and with unit loads placed successively at
loading points "one" through "Six", the [gim]
matrix was obtained. When the cover sheets are
Solution: "cut" the two webs act independently as plane-
web beams. The details of the stress calculation
The generalized forces employed are shown for such beams are similar to those of Example
on the exploded view, Fig. A8.33. No forces Problem 21, Art. A7.7 and are not shown here.
were shown for the lower surface, its members
and forces being equal to those of the upper
'{\ 1 2 3 4 5 6
surface because of symmetry. In the [U ij ] 1 .400
2
matrix this fact was accounted for by doubling 3 .400
the member fleXibility coefficients for the 4 4.006
members of the upper surface. 5 4.006
6 .100 .200
"1
8 .100 .200
'"q :l.4 \ q~ 6 9 5.341 2.671
10 5.341 2.671
~\ ~~~"
11 .04445 .08887 .1333
12
13 .04445 .08887 .1333
14 6.231 4.006 2.003
15 6.231 4.006 2.003
'q
ql,~~,
ants q 2, q 7 and q a . The calculations are
illustrated by the exploded view of the end bay
in Fig. A8.34 showing the calculation in that
part of the structure for q2 = 1. Note that
q2 = 1 was applied as a self-equilibrating pair
of shear flows acting one on each side of the
q~ "cut". The ribs were considered rigid in their
own planes.
Fig. AB.33
3774 .1789
'0520~
.1789 .2678 .07322
[ .1254
.0520 .07322
q~ = -11G" .-.-~ q3 = 1 E
t308~ -2.562
- 0113J
~rs-J 10 6
-2.562 6.137 -2.521
- .1137 -2.521 9.499
Fig. A8.34
Then
q~ -1.0
q3 '" 1. 0 ~sJ - ~rs-J ~rJ
~
I'?\
1 - 1.00
2 7 12
2
3
.0699
.0699
4 2.26
5 1.75
- .0699
.330
1.75
2.26
.0060 - .0060 .0002 -.0002
.0060 - .0060 .0002 .0002
- .150
.150
.150 -.0040 .0040
.150 .0040 -.0040
2 l.00 6 .0859 .0141 .168 .0322 -.0040 .0040
3 l.00 7 .0359 - .0359 .0341 - .0341 .0040 -.0040
4 -25.04 8 .0141 .0859 .0322 .168 .0040 -.0040
5 25.04 9 2.83 2.51 1.545 1.126 -.122 .122
6 .3125 - 1.00 10 2.51 2.83 1.126 1.545 .122 -.122
7 l.00 11 .0454 - .0010 .0776 .0112 -.0150 .0150
12 .0157 - .0157 .0187 - .0187 .0158 -.0158
8 - .3125 l.00
13 - .0010 .0454 .0112 .0776 .0150 -.0150
9 -20.87 -29.38 14 3.47 2.76 2.40 1.61 .610 -.610
10 20.87 29.38 15 2.76 3.47 1.61 2.40 -.610 .610
11 .1387 .1955 - 1.00
12 l.00 The reader will observe that the result
13 - .1387 - .1955 1.00 displays the "bending stresses due to torSion";
14 -18.79 -26.44 -31.71 that is, the bUildup of axial flange stresses
15 18.79 26.44 31.71 near the root of a beam under torSion when the
root is restrained against warping. The solu-
NOTE: VOIDS INDICATE ZEROES. tion for application of a torque is readily
A8 27
_1~.J
44 -15.0 o
P 2 = -1. Thus, one finds that under this con-
1 -15.0 38.0 o
dition there is a root flange load of [amn] = ~m!J ~lil ~j!J =E 0 0
38.0
[
q~4 = 3.47 - 2.76 = 0.71 Ibs. o 0 -15.0 144
for a torque of 15 inch Ibs.
472 .888 .0525 .028J
A8. 11 Redundant Problem Deflection Calculations by .888 1.665 .0975 .0525
Matrix Methods.
..0525 .0975 1.665 .888
Deflections (and in particular, the matrix [
.028 .0525 .888 .472
of influence coefficients) are readily computed
from the results of Art. A8.10.
Assume that the redundant forces { qs } have Finally, the sum of the above two matrices gave
been deterTIined from eq. (22). The total de-
~'M
flection of external loading points is then -15.9 - .052 - 002
easily computed as the sum of, ONE, the de-
flection due to external loads acting on the _ 1
- 15.9 36.3 - .098 - .052
Solution:
An alternate procedure to that shown by
eq. (25) was followed. The influence coeffic-
SUbstituting from eq. (22) ient matrix was formed as in Chapter A-7, Art.
A7.9, by the product.
{ Om } = ~rnnJ { Pn } - ~m;] ~rsJ ~r~ {Pn }
= ( ~rnnJ - ~m~ ~rsJ ~r;] ){Pn } This product was formed readily, inasmuch as
[G im] was available from example problem 15.
The matrix expression set off in paren-
theses above, giving as it does the deflections The result was
for unit values of the applied loads, is the
matrix of influence coefficients. Let 4363 3500 1522 1243 29.7 -29.7
3500 4363 1243 1522 -29.7 29.7
1522 1243 897.8 594.2 33.7 -33.7
1243 1522 594.2 897.8 -33.7 33.7
~.7 - 29.7 33.7 - 33.7 36.8 -36.8
- 29.7 29.7 - 33.7 33.7 -36.8 36.8
so that
A8. 12 Precision and Accuracy in Redundant stress
Calculations.
Matters of precision are dependent upon the
number of significant figures obtained and re-
Example Problem 16 tained in dealing with the geometry of the
Determine the matrix of influence coeffic- structure and in the care with which arithmetic
ients for the redundant truss of example operations are performed. In the discussion to
problem 14. follow it is assumed that all due caution is
exercised with regard to the precision of the
Solution: work.
Matters of accuracy have to do with the num-
From the previous work the folloWing matrix ber of significant fiGures finally obtained in
products were formed the answer as influenced by the manner of formu
A8 28 STATICALLY INDETERMINATE STRUCTURES
lation of the problem. The accuracy of the re- inverse cannot be found (ref. appendix). Its
sult may be affected by a nQ~ber of factors, two condition is terrible. Example:
of the most important of which are discussed
here.
Two factors influencing accuracy often are
considered together under the heading "choice of
redundants". They are:
r- L --l.- L -+- L -,
which determines the accuracy with which its in- Fig. A8.35(b) shows the choice of general-
verse can be computed is its condition. The ized forces. Only two (q~ and q2) are required
condition of the matrix is an indication of the to describe the strain energy, but the central
magnitude of elements off the main diagonal support reactions were also given symbols as it
(upper left to lower right) relative to those was desired to consider them in the discussion.
on. The smaller are the relative sizes of Then
elements off the main diagonal, the better is
the condition of the matrix. A weJl conditioned - -
matrix is more accurately inverted than a poorly 2/3 1/6 0 0
conditioned one. Two extreme cases are now
given for illustration: 1/6 2/3 0 0
I~ 3 4
2 1
1 - 3 L - 3 L
r, s '" 1, 2
(4) Combinations of redundants may be em- such a way that the product [gir] [B rp ] does
ployed to yield new unit-redundant stress dis-
tributions which do not "overlap" as extensively exactly what is desired, viz., "take one times
as do those of the individual redundants origin- the q3 column minus one half the q4 column to
ally chosen. give the first column of the new distribution
For example, suppose that in the previous
illustrative problem, the choice q3' q4 for re- [giP] ". Then "take L1inus one half of the q3
dundants had been made originally, leading to
the unit-load dia~rams of Fig. A8.37(a). In- column plus one times the q4 column to give the
spection of the diagrams leads one to anticipate second column of the new distribution [giP] "
a strong degree of cross-coupling and hence a
new set of redundants is sought. Rather than Multiplying out in the above example
return to the structure to choose new "cuts",
combinations of the m3 and m4 diagrams are
looked for which will have less "overlap" and 1
- :2 L 0
hence less cross-coupling.
It is observed by inspection that two new 1
stress distributions-which have the desired 0 - :2 L
property may be formed from the m3, m4 diagrams
by proper combination. Thus, if one half the
~ieJ =
1
1 - :2
m4 diagram is subtracted from the~ diagram-to
form one newstress distribution and one half 1
the ~ diagram ~ subtracted from the m4 dia- - :2 1
gram to form the other new stress distribution,
the results are as shown in Fig. A.37b. There
is· obviously less "overlap" of the diagrams for Now form the matrix of redundant coefficients
these new combinations. for the new unknowns (subscripts p, 0; [gip] =
[gjO] )
m3~3-tm41~_L 1
2 1
m4~ m
4
-"2 m3IL_ l_:L..L...L..J.~...L-Jo.._1
_ _2~
L3
24EI
(a) (b)
Fig. A8.37 The condition of this matrix is greatly
New unit-redundant-force stress distributions (b) obtained by improved over that obtained for q3' q4 alone
(previously computed), viz.,
combining previous distributions (a).
In this way two new unknowns are introducen by ~rJ = L3/18EI (r,s=3,4)
linear combination. In matrix notation, the old
stress distributions [gir] are transformed to a
(That the Capo] matrix obtained here happens
new set [giP] by forming
to be similar to that obtained for q1' q2 in a
previous example, is coincidental.)
Once a transformation has been performed,
leading to a new unit redundant matrix [giP] ,
the problem may be completed in the "p, a
§L ~ L system". The appropriate equations are obtain-
3 3 ed from eqs. (14), (21), (23) and (25) simply
= ~ L § L by replacing all "r, s," by "p, 0". Thus
3 3
1 o (29)
o 1
~r~ = { 9A9}
GUESS -101.
~~:~~{gi7}+ 0 X{gi'U}
The non-homogeneous terms are practically zero,
as they should be*. The redundant forces would (1)---- 1 x{ gi 2}-
be zero also.
3774 .1789
'0520~
~ 1 2 3
.1789
[ .0520
.2678 .07322
1 - 1.00 .07322 .1254
2 l.00
3 l.00
4 -25.04· It remained to select a determinate stress
5 25.04 distribution which would reduce the magnitude of
6 1.023 - 1.00 the redundants. For this purpose, the engin-
7 - .7103 1.00 eering theory of bending was employed to compute
8 - 1.023 1.00 stress distributions satisfying equilibrium for
\:J -29.38 each application of a unit external load. The
10 29.38 result was (refer to Example Problem 30, Art.
11 1.0285 - 1.00 A7.11).
12 - .833~ 1.00
~
13 - 1.0285 1.00 1 2 3 4 5 6
14 - .010 - .026 -31.71
15 .010 .026 31.71 1 .30 .10
2 .10 - .10
NOTE: VOIDS DENOTE ZEROES. 3 .10 .30
4 2.0 2.0
5 2.0 2.0
Fig. A8.39a shows plots of the new distri- 6 .0875 .0125 0.15 .05
butions, these haVing greatly reduced "overlap" 7 .0375 - .0375 .05 - .05
(compare with Fig. A8.39). 8 .0125 .0875 .05 .15
9 2.67 2.67 1.33 1.33
10 2.67 2.67 1.33 1.33
11 .0423 .0023 .0713 .0179 .100 .0333
12 .020 - .020 .0267 - .0267 .0333 -.0333
13 .0023 .0423 .0179 .0713 .0333 .100
-31. 71 31.71 14 3.00 3.00 2.00 2.00 1.0 1.0
15 3.00 3.00 2.00 2.00 1.0 1.0
Fig. A8.39a
~
495 -3495 -1841 1841 0 0 ~
New unit redundant stress distributions, [gip]
~pg] = ~ 1167 -1167 1554 -1554 -1450 1450
1082 -1082 1445 -1445 1802 -1802
·t
was obtained by multiplying out per eq. (30a).
2 584
-.01132
•0000175
-.01132
.2566
-.03124
.000017J
-.03124
.1254
the "p,o system" using the matrices [gip} Capo}
[gim] and [a pn] in eqs. (31) and (32).
where gri is the value of the qi due to a unit * It will be convenient later to designate by qST the solution
to eq. (36) when the mechanical loads P n are zero, the
(virtual) load at cut r. Note that the term ~iT stresses in such a case being purely "thermal".
AB.35
qjn~
qi - qj is (see Art. A7.8, Ex. Prob. 24).
u qj + x
L oT dx
TJ Ti h
Ti - Tj Fig. AB.40
T Tj + x
L where
where
~ (20Ti 6+ OTj) qi + ~L (OTi +6 20T j) qj
OT~OTi
q.~qi
J(_}--x
(
h = constant
Fig. AB.42
Note that variation in the cross sectional
area of the bar does not affect the distortions or
Ll iT , Ll jT •
The alternate choice of generalized forces
for the bar under varying axial load is shown where
in Fig. A8.41. By a
derivation similar to
that above one finds
_ ~ (20Ti: OT~
(OTi +6 20T~
where Fig. AB.41
Special forms of the thermal distortion
expressions for beams of varying depth may be
derived readily as required.
Example Problem 19
Ti + 2Tj The upper surface of the beam of Fig. A8.43
Ll jT = a Lla
6 is subjected to a temperature oT above that of
the lower surface, varying linearly as shown (i.e.,
The simpler cases of uniform load (qi = u
constant) and uniform T (T = T j ) follow im-
i
mediately by specialization of the above forms.
For example, for a bar under constant load
qi = qj' and constant temperature, Ti = T j = T,
one has Ll = a LT.
iT Fig. AB.43
A8.36 STATICALLY INDETERMINATE STRUCTURES
::: o+ EI a aTe 1 0
L
'n
6EI 0 1 .267
2 1 .933
The unit redundant load distribution for q, 1
- L L
and q2 = 1 was 1 2
L - L
I~ 1 2
.267 }
1 1 0 EI a aT e .933
2 0 1 h { .399/L
-1.60/L
3 2 1
- L L Thus the reactions were
1 2
4-
L - L q3 = .399
EI a aTe
Lh
Member thermal distortions were computed. (Note EI a aTe (negative indi-
that aT was negative according to the convention Lh cates DOWN)
adopted earlier).
Example Problem 20
1 The symmetric sheet-stringer panel of Fig.
aL 2 x 3 a Te A8.4-4-(a) is to be analyzed for thermal stresses
h (-) 6
developed by heating the two outside stringers
to a uniform temperature T above the center
aL stringer. Assume G = 0.385E.
- 3H aTe
Solution:
was
The matrix of member thermal distortions
~i~ =
0 1.0 0
0 0 1.0
40 - .025 0 0
40
.025 - .025 0
20
o ...... 0 .025 - .025
{lI iT } = a T< o :>
o Multiplying out per eq. 18,
o
o
o
~1l'2
- 5.7
o ]
Here, for example lilT = 2 x 2 x 10 = 40 (doubled ~r~ = E
1 - 5.7 211.2 - 5.7
once because q1 acts on two stringer ends and 0 - 5.7 105.6
doubled again to account for the other half of
and
r ,
the panel).
q.~ EJl
5 0 0 1.0 0 0 -.025 -.025 40
~rJ { o~
, I I I II , , II
c~::::m--1I----lI[~::J 1 61T} = a
T -.5 0 0 1.0 0 0 -.025 40
r
[
o -.5 0 0 1.0 0 0 -.025 20
F =..02 i=
i= q.~ 18l
0
T~~
:= tryp
Ell..-----'L-~.L~
t: 1
60"
q.~ E] r
=a T
{
-ZO}
-20
-10
0
0
1--12'+12''-l~A=.15 in
2
(Typ. )
(a)
(b) ll.
Then the redundant equations, per eq. (36),
Fig. AB.44 were
The structure was three times redundant.
Stringer loads q4' q& and qe were selected as
redundants. Fig. A8.45 shows the unit redundant
load sketches for q4 = 1, q& = 1 and qe = 1.
Solving,
~ 8l ~ ~'8 f
0.100
~'18 f ~ 0 ~ ~ 0 ~
- 50.0
- 50.0
97.4
10-
3
( 100.0 >
~li4} loading I'll. f1i5} loading ill. {gi6pOadin~ ll. 100.0
- 2.44
Fig. AB.45 - 0.065
o
A8.38 STATICALLY INDETERMINATE STRUCTURES
Example Problem 21 L/ 2 L/ 2
The uniform four-flange box beam of Fig.
A8.46a is to be analyzed for the thermal
stresses developed upon heating one flange to a 1
temperature T, uniform spanwise, above the other
three flanges. L/ 2 L/ 2 L/ 2
L/2 L/ 2 L/ 2 LI2
Fig. A8.46
For a specific case let k2L~ 1. The
Solution: ( b) inverse was computed to be
The beam was divided into four equal bays
giving a four times redundant problem. Four 1.070 - .5291 - .2337 - .06587
self-equilibrating (zero-resultant) independent Gt - .5291 1.366 - .3613 - .1018
stress distributions were taken as the unknowns, ~rs::J L (b + c) - .2337 - .3613 1.497 - .1934
these being shown in Fig. A8.46(b). Such zero-
resultant stress distributions are the only ones - .06587 - .1018 - .1934 1.792
possible in a structure having no applied loads*.
The matrix of member flexibility coeffic-
Member thermal distortions (~iT) were
ients was formed by collecting coefficients from
the several members. Unit redundant stress dis- computed for loads q2' q., q6 and qe and were
tributions were prepared, taking q1, q3' q6 and collected from the one heated flange only.
q7 as the redundants and setting these equal to
unity successively.
o (Note that if two ad-
.500 2 jacent flanges are
.667k' .1667k' o heated equally, one must
.500 a L T 2 set the corresponding ~iT
.1667k' .667k' .1667k' 8 o equal to zero; this be-
.500 2
o cause the virtual loads
.1667k' .667k' .1667k'
1
being of opposite sign in
.500
•333k Ii
adjacent flanges, the
.1667k'
virtual work must cancel).
where k 2 ~ Gt / AE (b + c) Multiplying out:
* Sinceexternal loads are not to be applied it follows from
statics that the generalized forces for adjacent webs and
cover sheets are equal, as are the loads in front and rear
spar caps at any given station. Hence an economy of
numbering in the generalized force scheme is possible.
Much labor is saved in the handling of data when the same
symbol can be employed on several members whose loads
are known to be equal. Then the solution to eq. (36) was written as
A8.39
4'079 }
Gt a L T 1.938
{ qST } :: - 16 (b + c) . 8562 at the external points (compare with eqs. (35)
{ and (37) ). Simultaneously the redundant cuts
.2413 •
will experience relative displacements.
Finally the -complete set of thermal stresses SECOND, the redundant cuts are restored to
were (still for the case L2 k 2 :: 1) zero displacement by the application of redund-
ant forces (this problem was solved in Art.
A8.13). The qs are given by eq. (36); they pro-
4.079
2.039 L duce additional deflections at points m
1.938
3.008 L
.8562
3.437 L
.2413
3.558 L The total deflection of point m is then
Solution:
aT ;; - .289
a L a To
To compute the rotation a unit couple was h
applied (positive counterclockwise) at the right
hand end. An additional generalized force, It is apparent that the values of the re-
called q5, also was added at that point. Then, dundant moments q~, qg could have been chosen
arbitrarily in {gim}' above, without affecting
t.
5T =
aL (-)(~ 6To·: 26 To)~ -4 a L a To
the result. Here, clearly, {gim}for any "cut"
h 9h
structure visualized will lead to the same re-
and, using some results from Example Prob. 19, sUlt, q5 being equal to unity in all cases.
Example Problem 23
m
Compute the axial movement of the free end
{~iT} ;; _ a
L a
9h
To of the central stringer of the panel of Fig.
A8.44.
1 =1, 2, 5
Solution:
Note that q3 and q4, the intermediate support
reactions, were omitted from consideration. An additional generalized force, q~o, was
They do not enter into any expression for the added axially to the free end of the central
internal virtual work of the structure; or, stringer (ref. Fig. A8.44b). Then
equally, they are not used to describe the
strain energy of the structure. Hence they are 177.6 44.4
not included in writing the total strain. 44.4 177.6 44.4
(Their ~iT are zero.) ---- 44.4 88.8
88.8 22.2 22.2
The member flexibility coefficient matrix
22.2 88.8 22.2
was
22.2 44.4
62,400
62,400
62,400
i ;; 1, 2, 5 22.2 44.4
40
From Example Problem 19, the true thermal 40
stress distribution was 20
o
o
o
i ;; 1, 2, 5 o
o
(q6 was zero by inspection) o
o
To determine [gim]' a unit couple q5 ;; 1
Using the qjT as obtained in Example Problem 20
was applied. Taking as the "cut" structure one
where q~ ;; q~ ;; 0 one has simply (with q~o • 0), the following product was
formed:
- 10.87
- 13.26
6.660
10.87
Then substituting into eq. (38), 13.26 x aT
6.66
1.2 0U(\: aL~To {3}~ ~ 1~ EI a ~ To {:9~f
6~I ~ i ~
26~l) -152.2
{ Omr } = + 4.06
0.00
2.162
A8.41
Then, sUbstituting into eq. (38), a unit load q10 1 gives (writing the transpose
of {gim})
29.13
26.74
13.34 l- ~i~ = l-.50 .50 .50 000 .025 0 0 l~
10.87
13.26
6.66 Then mUltiplying out for the thermal deflection:
-152.2 OmT = 33.0 a T (same answer).
- 4.06 It is apparent from the above result that
0.00 the Simplest determinate ("cut") structure should
2.162 be used to compute [gimJ; it is completely
adequate.
It remained to find the determinate stress dis-
tribution for q10 = 1•. CLOSURE
As the determinate distribution {gim} the
The general thermal stress problem is
stresses due to a unit load q10 = 1 were com- complicated by the fact that the material prop-
puted in the "cut" structure obtained when erties E, G and a vary With temperature. The
q, = qs = q3 = O. (Note that this is a differ- problem created thereby is primarily one of book-
ent choice of redundant cuts from that employed keeping - computing the member flexibility coeffi-
in computing the thermal stresses in Example cients (a ij ) and the member thermal distortions
Problem 20.) Thus (~iT) for a structure whose properties vary from
point to point with the temperature. The vari-
o ations of E, G and a with T Will, of course,
o have to be known from test data.
o Two additional complications, not considered
1 here, are the lowering of the yield point With
1 heating (and the attendant increased likelihood
1 of developing inelastic strains) and the phenom-
o enon of "creep" (the time-dependent development
o of inelastic strains under steady loading).
o Should it prove necessary to analyze for
1 thermal stresses under more than one temperature
distribution, the member thermal distortion
and finally,
matrix {~iT } may be generalized easily into a
OmT = aT Lo 0 0 1 1 100 0 IJ 29.13 rectangular form such as
26,74
13.34
10.87
13.26
6.66
'-152.2 where ~iR is the member thermal distortion as-
- 4.06 sociated with force qi from thermal loading con-
0.00
2.162 dition R. The matriX [C ij ] of member thermal
33.0a T (ANSWER) coefficients consists of the constant coefficients
in the ~iT expressions previously presented, while
As a matter of interest, a different unit
stress distribution was employed with a differ- T jR would be the temperature associated With qj
ent choice of "cuts", If the forces q401 qll and for condition R. (compare with eq, (26b),
qs are set equal to zero ("cut"), application of Art. A7.ll).
A8.42 STATICALLY lND:f;TERMINATE STRUCTURES
100"
20"
T
90"
I- A
B
A
B
0'
4000#
1
F~.
100"
a F~.b
~ 60"
+ 60" -lC
~r-------->"::'.i....--_--"",-------,=------"D1 D~C+
~
6000# I
Fig. c
1000# 1000# 180"
E~
(3) For the loaded truss in Fig. c, deter-
mine the axial load i~ all members, Values in 50,000#
parenthesis adjacent to members represent rela-
tive areas. E is constant for all members. Fig. i
Fig. h
C
f ID
(couple and transverse force). Solved in this
way the problem is doubly redundant as no ad-
~3
C vantage is made of the symmetry of the structure.
(11) Add two additional members, diagonals
FB and EC (each with areas 1.0 ina) to the truss
~_i
Fig.e
I-- 120" + of Fig. A7.85, chapter A-7. Find the matrix of
120" ---l
A8.43
Douglas DC-8 airplane. Photograph showing simulated aerodynamic load being applied to
main entrance door of fuselage test section.
A8.44 STATICALLY INDETERMINATE STRUCTURES
DOUGLAS DC-8 AIRPLANE. An outboard engine pylon mounted on a section of wing for static and flutter
tests. The steel box represents the weight and moment of inertia of the engine.
CHAPTER A9
BENDING MOMENTS IN FRAMES AND RINGS
BY
ELASTIC CENTER METHOD
Assumptions
A9.l Introduction
In the derivations which follow the distor-
In observing the inside of an airplane tions due to axial and shear forces are neglect-
fuselage or seaplane hull one sees a large num- ed. In general these distortions are small
ber of structural rings or closed frames. Some compared to frame bending distortions and thus
appear quite light and are essentially used to the error is small.
maintain the shape of the body metal shell and
to provide stabilizing supports for the longi- In computing distortions plane sections are
tUdinal shell stringers. At points where large assumed to remain plane after bending. This is
load concentrations are transferred between body not strictly true ])ocause the curvature of the
and tail, wing power plant, landin~ gear, etc., frame charl";es this linear distribution of bend-
relatively heavy frames will be observed. In ing stresses on a frame cross-section. (Correc-
hull construction, the bottom structural fram- tions for curvature influence are given in
ing transfers the water pressure in landin~ to Chapter 1\13.
the bottom portion of the hull frames which in
turn transfers the load to the hull shell. Furthermore i t is assumed that stress is
proportional to strain. Since the airplane
In general the frames are of such shane stress analyst must calculate the ultimate
and the load distribution of such character strenGth of a frame, this assumotion obviously
that these frames or rings undergo bending does not hold with heavy frames where the rup-
forces in transferring the applied loads to the turing stresses for the frame are above the pro-
other resisting portions of the airplane body. portional limit of the frame material.
These bending forces produce frame stresses in
general which are of major importance in the This chapter will deal only with the the-
strength proportioning of the frame, and thus oretical analysis for bending moments in frames
a reasonable close approximation of such bend- and rings by the elastic center method. Prac-
ing forces is necessary. tical questions of body frame design are covered
in a later chapter.
Such frames are statically indeterminate
relative to internal resisting stress and thus The following photographs of a portion of
consideration must be given to section and the structural framing of the hull of a sea-
physical properties to obtain a solution of the plane illustrate both liGht and heavy frames.
distribution of the internal resisting forces.
A9.1
A9.2 BENDING MOMENTS IN FRAMES AND RINGS
Fig. A9.1 shows an unsymmetrical curved From Chapter A7, which dealt with deflection
beam fixed at ends (A) and (B) and carrying theory, we have the folloWing equations for the
some external loading P~, P e , etc. This movement of point (A):-
structure is statically indeterminate to the
third degree because the reactions at (A) and Q = Z MInds _ -(2)
(B) have three unknown elements, namely, magni- EI
tUde, direction and line of action, making a
total of six unknowns with only three equations ~x = Z MInds
EI
- -(3)
of static equilibrium available.
~y = Z MInds
EI
- - -(4)
P e
P~
Pa In equation (2) the term m is the bending
t moment on a element ds due to a unit moment
\ applied at point (A) (See Fig. A9.3). The bend-
ing moment is thus
equal one or unity
for all ds elements A~
~it moment
Fig. A9.1
A
of frame.
Fig. A9.3
ds
P Then SUbsti-
tuting in equation (2) and using value of M from
equation (1) we obtain -
measured upward from aXis x-x through (A). place of XA, YA and MA respectively.
However the bending moment on the ds element
shown is negative (tension in top fibers), thus The axes x and y through the point (0) are
the value of m = - (1) Y = -Yo The minus sign centroidal axes for the values ds/EI of the
is necessary to give the correct bending moment structure. This fact means that the summations-
sign.
and
SUbstituting in Equation (3) and using M
from Equation (1):-
The expressions Z x 2ds/EI, Z y2 ds /EI and
AX
'-'
= _ZMsydS
EI
_ M ~yds + X ~y2dS
"A"" EI A""~ -
Y ~xydS
A""Er=
(6) Z xyds/EI also appear in equations 6 and 7.
0-- These terms will be referred to as elastic
moments of inertia and product of inertia of
In equation (4) the term m represents the the frame about y and x axes through the elastic
bending moment on a element ds due to a unit center of the frame, and for simplicity will be
load at point (A) acting in Y direction as given the following symbols.
illustrated in Fig. A9.5. Hence, m l(x) x = =
Substituting in
equation (4) and Y ds
using M from
equation (1), we x---~ Equations 5, 6 and 7 will now be rewritten
obtain, t- Fig. A9.5
using the redundant forces at point (0).
e = Z MsdS
EI
+ M Z ds -
0 EI - 0
hence,
- - -(7)
REFERRING REDUNDANTS TO ELASTIC CENTER The term MoZ yds/EI is zero since Z ydS/EI
is zero, thus Mo drops out When substituting in
For the purpose of simplifying equations
5, 6, 7, let it be assumed that end A is Equat i on (6).
attached to a inelastic arm terminating at a
point (0) as illustrated in Fig. A9.6. The - - - -(10)
point (0) coincides with the centroid of the
ds/EI values for the structure. Reference
axes x and y will now be taken with point (0) The term Msds/EI represents the angle
as the origin. The redundant reactions will change between the end faces of the ds element
now be placed at point (0) the end of the When acted upon by a constant static moment Ms'
This angle change which actually is equal in
Y
value to the area of the Ms/EI diagram on the
I element ds will be given the symbol Os' that is,
0s = Msds/EI. With this symbol SUbstitution,
equations 8, 9, 10 can now be rewritten as
follows:-
Z 08 -(11)
Fig. A9.6 Z dS(EI - - - - - -
o y= x= Ix Iy
IX (1 - i);) Por- w=ds dist. dist.
Han I fr05. from
wx wy =wy 2+ ix =wx 2+i
Y
x 1_x y-y
J1
0
= -~ 0s
~ds/EI
- - -(16)
11 x-
T
x ds= 30= h
Fig. A9.7 ~" 18" --1 i x = 2 bh 3 = 12 x"3 x 30 3 = 750
12
shows a rectang-
ular frame with
B ,---_L-
I =2
C
1
fixed supports
at points A and
D, and carrying -x
y
I elastic -x
- -7- T i y =~bh3
Referring to Fig. b,
1 1
= 12 x 30 X-w = .09 (negligible)
A
Fig. a
1
c-ent 12
a single load as 30"
shown. The I y
problem is to de- The distance from the
I =3
termine the bend-
ing moment dia-
gra'll under this
'"3 i
two reference axes to the
elastic center can now be
calculated:-
I
I
I
I
T ds = 30 = b
loading. X'-~ I~ __ D _ I
1
' - _ L:wy _ 660 _
y - ~w - 32 -_20.625 in. I
The first Fig. A9.7 I
step in the
- _~wx 0 -j y 1-1I : 13 = h
solution is to x - ~w 32 - 0
Fig. b
find the location of the elastic center of the
frame and the elastic moments of inertia Ix Having the moment of inertia about axis x~x~ we
and I y . can now find its value about the centroidal
Due to symmetry of the structure about the axis xx of the frame, by use of the parallel
Y axis the centroidal Y axis is located midway axis theorem.
between the sides of the frame, and thus the Ix = Ix - ~W(y2) = 16800 - 32 x 20.625 2= 3188
elastic center (0) lies on this axis.
Iy = Iy - ~W(X2) = 3456 - 32(0) = 3456
Table A9.l shows some of the necessary
calculations to determine the location of the The problem now consists in solving equa-
elastic center and the elastic moments of tions (16), (17) and (18) for the redundants at
inertia. The reference axes used are x~-x~ the elastic center, namely
and y-y.
A9.5
_ - ~¢ _ Area of static M/I diagram along the neutral axis of the frame members.
Mo - ~ds/¥ - Total elastic weight of structure
In Fig. A9.10 the area of the Ms/I diagram
Moment of static M/I diagram about equals 0s = 22.5 x 24/2 = 270. The centroid by
= ~¢sy = x axis simple calculations of this triangle would fall
Xo Ix Elastic moment of inertia about x 10 inches from B. Fig. A9.11 now shows the
axis frame with its Ms/I or its 0s load. 0s is
Yo = -~¢sx
Iy
Moment of static M/I diagram about
= =-_:-:- ..,.y~a_:_x-i..,...s-_;_,...____:_-.,__-
Elastic moment of inertia about y
axis
r,z>s= 270 - I "
~ 1--2
B:
x-----l------ x
0:
C
l'
positive since Ms is
positive. The next
375" step is to solve the
equations for the
redundant at the
I 20.625" elastic center. The
Thus to solve these three equations we : I signs of the distances
must assume a static frame condition consistent A :y D ~ x and y from the axes
with the given frame and loading. In general I x and yare conven-
there are a number of static conditions that can Fig. A9.11 tional.
be chosen. For example in this problem we
might select one of the statically determinate ThUS,
conditions illustrated in Fig. A9.8 cases 1 to
5. _ -Z0 s = - (270) = -8.437 in. lb.
Mo - Zds/I 32 (from Table A9.1
_ Z0 Y _ 270(9.375)
Xo - Ixs - 3188 = 0.7939 lb.
Yo
=-Z0sx
I
= -(270)
3456
(-2)
= 0.1562 lb.
y
L. -:A~-c07
given in Fig. A9.9. The equations 9.81
06 D
Bf-.------=' C,di""(1"'/1di,."m
B C Fig. A9.12
6.06
Fig. A9.13
_ - Z0s _ - (-390)
Mo - Zds!I - 32
=12.19 in. lb.
Yo = -z0s x
Iy
=- 3456
(-540) = 0.1562 lb.
MB = -30+12.19-(-0.203)9.375+0.1562(-12)
- -17.76 in.lb.(checks previous solu-
tions)
B L = 30 C
In order to take moments of the ¢s values 1= 4
in column (4) of the table, the centroid of IY L=24
each portion of the diagram must be determined.
For example, the centroid of the two triangular
r
PI =20# L=24
1=3 __ -i-
1= 3
_-x
Y
Z ds 30+(24)2+ 30 - 38.5 - 1 - 1440 3- 480 -15 10.33 7200 - 4958
I 4 3 2
moment of inertia of the frame about x and y 3 - 1440 3- 480 15 10.33 - 7200 - 4958
axes through the elastic center of the frame.
4 14403 480 15 - 5.67 7206 - 2726
Moment of inertia about x axis = Ix :- 5 36002 1800 5 - 9.67 9000 - 17406
Ix =( i x i X 14.33 3
) 2 = 653.9
7
8
-108000 3 -36000
- 67500 2 -33750
15
5
2.33 -540000 - 83880
Example Problem 4. Hull Frame A'B' 60.0 1.0 60.0 0 0 18000 + 0 = 18000
bOOO'li-
D
In the last column of Table A9.4 the term
i x is the moment of inertia of a particular
member about its o\Vll centroidal x axis. Thus
for member BDB;
D
Fig. A9.28
""""~ ~~=~~-~:}Y ix =..1....
12
b L h2
= ;2x~X38.4X242=922
78400
= 1,007,000 Xo = Z0I s Y =
xx
Consider member AC, _1007000 x 48.16 + 172800 x 33.36 + 156800 x -5.24
- 252400
From free body diagr&~ of bottom portion
of frame (Fig. A9.31J the equation for bending = -98.5#
moment 600~ /4690 6000 Yo is zero because of the symmetrical frame
'""tI
A -- A' and loading. The final or true bending moment
at any point equals
'}.I;)I;)~\~'
Fig. A9.31... Thus for point B
38.4 = 32700
1/2
10o
(4690 x - 100 X")dx
Hence Me = 32700 - 12430 - (-98.5 x -60.64)
14300"#
=
_ 14690 x" 100 x31 38.4 _ Fig. A9.30 shows the general shape of the true
- 1/2 L 2 - 3 J - 784000 frame bending moment diagram.
o
Example Problem 5
Distance to centroid of Mil curve along line
AC from A. Fig. A9.31 shows the general details of one
A9.12 BENDING MOMENTS IN FRAMES AND RINGS
Assumed
analysis purposes the frame has been divided ReT.""Axis I - -
into 20 strips. The centroid of these strips X'X'
located on the neutral axis of the frame sec- 1"'.,1
tions are numbered 1 to 20 in Fig. A9.21. The llJ
tangential skin resisting forces are shown as S
C1l
concentrations on frame strips # 6 to # 16. On
the figure these tangential loads have been UJ
Table A9. 5
Calculation of Frame Elastic Properties Calculation of Moment Weights and Solution of Redundants
Col. 1 2 3 4 5 6 7 8 9 10 11 12 13
.....0 Elastic Arm w y' w y,2 Arm Static Moment Os y Mo -Xoy *Total
<:: Weight to to Moment Weight Moment
0 0<:5
.,.. bJl
~C'l
<:: .~
.~
~ <:: (j)~ Ref. Axis Os = M
~
1 7.6 .065 117.00 26. 1 3050 79700 17.2 0 0 0 -9020 16650 7630
2 7.6 .065 117.00 24.4 2850 69500 15.5 0 0 0 " 15020 6000
3 7.6 .065 117.00 21. 4 2497 53300 12.5 0 0 0 " 12100 3080
4 7.6 .065 117.00 17.3 2025 35020 8.4 0 0 0 " 8130 - 890
5 7.6 .065 117.00 12.0 1403 16820 3.1 0 0 0 " 3000 -6020
6 6.4 .25 25.60 5.9 151 892 - 3.0 1220 31. 2 - 93.7 " - 2910 -10710
7 6.4 .25 25.60 0 0 0 - 8.9 2880 71. 6 - 637 " - 8610 -14840
8 6.4 .25 25.60 - 6.3 -161 1017 -15.2 6230 159.3 - 2420 " -14720 -17510
9 6.4 .25 25.60 -12.6 -323 4060 -21. 5 11260 286.5 - 6160 " -20830 -18590
10 6.4 .25 25.60 -19.1 -488 9320 -28.0 18470 473.0 - 13220 " -27100 -17650
11 6.4 .30 21. 30 -25.5 -543 13850 -34.4 29020 618.0 - 21250 " -33300 -13300
12 6.4 .34 18.82 -32.0 -602 19250 -40.9 43180 814.0 - 33100 " -39700 - 5540
13 6.4 .34 18.82 -38.3 -720 27600 -47.2 58980 1110.0 - 52400 " -45600 4360
14 6.4 .34 18.82 -44.6 -840 57500 -53.5 77330 1455.0 - 77800 " - 51700 16610
15 6.4 .34 18.82 -50.8 -959 48750 -59.9 99030 1865.0 -111300 " -57900 32110
Chine -63.0 111700 -61100 41600
16 7.0 12.0 0.58 -55.4 - 32 1780 -64.3 149700 86.7 - 5570 " -62200 78500
17 7.0 12.0 0.58 -58.4 - 34 1980 -67.3 221400 128.3 - 8650 " -65100 147300
18 7.0 18.0 0.39 -61. 4 - 24 1470 -70.3 271200 105.8 - 7440 " -68100 194100
19 7.0 34.0 O. 21 -63.2 - 13 840 -72.1 308600 64.9 - 4680 " -70000 229600
20 7.0 50.0 O. 14 -64.8 - 9 588 -73.7 327900 45.9 - 3385 " -71400 247500
Keel -75.0 332900 -9020 -72600 251280
Totals 811.48 7228 423237 7315.0 -348106
240 lb
Columns 11 and 12 record the values of
Mo and -Xoy for each station point. For 6" ---1
example, the value of -Xoy for station (1)
T
1
C
equals - (-968 x 17.2) 16650 and for station =
(20) = - [-968(-73. 7)J -71400. = 6" Y=5.242 ,
1
column 13.
£ ~y
A9.5 Unsymmetrical Structures. Example Problem Solutions. 9" 5.032
D
Example Problem 1
Fig. A9.34
Fig. A9.34 shows an unsymmetrical frame
carrying the loads as shown. Determine the
bending moments at points A, B, C and D.
Solution:-
The distance x from the line AB to the Fig. A9.36 shows the centroid locations of
elastic center is, the 0 8 values along the center line of the
frame. The moment of these 0 s values about the
x = (15)0 + 6 x 6 + 10 x 12
31 = 5.032 in.
x and y axes will now be calculated.
+ (10)(0.242)2 =606.51 ~
X
o
= Z03 y - Z0s x
----;".,----_v'--
Iy = (1.5) (5.032)2 + (12)" +(12) (0 968)2 +
2x12 2 • Ix (1 - i" fY
x y
(10) (6.968)2 = 942.96
70290 _ 139700 (217.74)
942.96
Ixy = (15) (-5.032) (-2.858) + (1:) (0.968) (5.242) =--------2=1=7:...:.:...:7:...:4'="2--- = 68.46 Ib.
606.51 (1 - (606.51)(942.96))
+ (10)(6.968)(0.242) = 217.74
__ - [Z0s x - Z0s Y (IfxY)J
The next step is to assume some static Yo
frame condition and draw the static bending I
y
(1 _J.xL)
1XIY
moment diagram. Fig. A9.35 shows that the
frame has been assumed cut near point C which
gives two cantilever beams. The bending
moment diagram in three parts for this static =_ r139700 - 70290 (217.74)J
L..= 606.51 = -132.36 lb.
condition is also shown on Fig. A9.35. 217.742
942.96 (1 - (606.51)(942.96)
¢Sl=-2160 The bending moment at any point from eq.
B J C
T (19) equals,
5.242
= 11s
t
11 + 110 - XoY + Yox, for example,
2.258 .C. x
Consider point A.
~:;=- 1600 6. 750
9.758
x = -5.032, Y = -9.758, 11s = -2520
j ¢S3=-4~60 11A = -2520 + 923 - 68.46 (-9.758) +
(-132.36)(-5.032) = -263 in. lb.
A 5.0~
Fig. A9.35 Fig. A9.36 Point B. x = -5.032, Y = 5.842, 11s = -1440
The 03 values which equal the area of
the 11s diagram divided by the I values of the 11B = -1440 + 923 - 68.46 x 5.242 + (-132.36)
particular portion will be calculated. (-5.032) = - 209 in. lb.
I~ C Member CD, I y = 20
1 X 15 2 +.J:....
12 x 20(1)3 = 4501.7
30" ~ I 1 x 1 x 30 3
1'1emb er AD , I y --12 = 1125 •0
1=1.5 I I 2
1- --- -~E.
x'
Jj
Y
A
:3=50#
10"
C.-----X- 1=1 20"
P4=50#
10"
::lli
L x'
Total I y
Calculation of I xy -
= 10919
30
r:----30" D Member AB, I xy = 1':5 (-15) (3.343) = -1002.9
Iy ' Fig. A9.37
Member BC, 1
= 31 67(13.343)(0) - 12 31.67
Solution: -
I xy
3 x--3-
(30)(10) = -263.92
The elastic weight of frame equals
20
Member CD, I xy =1 (15) (-1.657) = -497.1
2: ~ = 30 + 31.67 + 20 + 30 = 65 58
I 1.5 3 1 2 .
Member AD, I xy = °
The location of the elastic axes will be
determined with reference to assumed axes Total I xy = -1763.9
x'x' and y'y' as shown on Fig. A9.37.
The next step in the solution is to assume
a static frame condition and draw the Ms dia-
___ (;.o~ (0)+ (31 ~67) (15)+ (210) (30)+ (32°) (15) _- gram. Fig. A9.38 shows the assumed static
x
65.58 15" condition, namely pinned at point A and supported
as rollers at point D. The bending moment
diagram is drawn in parts as shown.
(30) (15)+ (31.67) (25 )+(20) (10 )+(30) (0) 750"
- 1.6 3 1 2
Y = ---------=6=5-.5=8=--------- = ~uetoP2
11.657" 600~toPl
60
These distances x
and y locate the x and (1)
y elastic axes as shown in Fig. A9.37.
The elastic moments of inertia and the Due to PI c
elastic product of inertia will now be cal-
culated. Fig. A9.38
Calculation of Ix -
~1+~·-1~ Point C. Ms =0
L 11 657, JI0 ...
I
1
$ __
Me =0 - 124.8 - 31.07 x 8.343 + 4.674 x 15
!
r-15'~
s =5000 = -314 inolb.
Fig. A9.39 Point D. Ms =0
Z¢sx = 6000(-15) + 3167(-5) = -105835 MD =0 - 124.8 + 4.674 x 15 + 31.07 x 110657
3958(13.343) -5000(-11.657) = 208659 Fig. A9.41 shows the true bending moment
diagram.
The values of the redundants at the elastic
center can now be calculated. A9.6 Analysis of Frame with Pinned Supports.
M
II O
= - Z¢s = -(8125)
Zds!I 65 ..58
= -124 • 8 in .lb.
Fig. A9.42 shows a rectangular frame and
loading. This frame is identical to example
problem 1 of Art. A9.3, except it is pinned at
points A and D instead of fixed.
_ Z¢s y - Z¢sx (~)
Xo 2
The first step will be to determine the
Ix (1 -~) elastic weight of the frame, the elastic center
x y
location and the elastic moments of inertia
208659 - (-10583:5) ( l;g:i:) _ about axes through the elastic center.
= [(-1763.9 2 )
6451 1 - 10919 x 6451
31.07 lb. J The term ds/EI of a beam element of length
ds represents the angle change between its two
end faces when the element is acted upon by a
= - [Z¢sx - Z¢sy (~U unit moment. In this chapter this term has
been called the elastic weight of the element.
Yo 2
I (1 _...!xL) Physically, the elastic weight is the ability
Y IxI y
of the element to cause rotation when acted
-[-105835 208659C1763.9] upon by a unit moment. When a unit moment is
applied to a rigid support, the support suffers
= [(_1763.9624)51J = 4.674 lb. no rotation since the support is rigid, there-
10919 1 10919 x 6451 fore a rigid support has zero elastic weight
and therefore does not figure in the frame
elastic properties.
A9.17
Fig. A9.44 shows the static frame con- The static frame condition will be assumed
dition assumed to obtain the Ms values. the same as in the previous example problem,
fr"./J
hence 0 s = 270 and acts 10" from B (Fig. A9.48).
The value of 0s for member BC equals the
area of the Ms curve divided by I for BC, /Os=270
hence 0s 45 x 24 x 1= 270. The centroid = 10
2"" 2" -18.11
of this 0s value is 10 inches from point B.
The redundant forces at the elastic center can
now be solved for 31°" Mo
A Xo
M
o =- ~
~0s
ds
=-:;-:-:--",..2,..,,7:-:;0=
infinity
- 0 Iyo 6.19
Fig. A9.50
EI Fig. A9. 48 Fig. A9.49
Yo = -~0sx
Iy
=-(270)10
infinity
=0 Since the frame is unsymmetrical, the x
and y axes through the elastic center at A are
not principal axes, hence
X = ~0sY = 270 x 30 = 0.482 lb.
o Ix 16800
A9.18 BENDING MOMENTS IN FRAMES AND RINGS
I Y = - Z¢sx p -(21)
Z¢sy - Z¢sx ( tSr) op I yp
Xo = I 2
IX (1 -~)
XY Where Ixp and I y are the elastic moments of
270 X 30 - 270 X 10 (7920) inertia about th~ principal axes and yp and x p
8064 the perpendicular distance from the ¢s loads
= 7920 2 = 0.6038 lb.
to the principal axes.
16800 (1 - 16800 X 8064)
The author prefers to use the longer
equations (14) and (15) which avoid finding the
- [Z¢sX - Z¢sy (Itxy)J principal axes location, the properties about
Yo = these axes and the perpendicular distances
I y (1 -~)
IxI y with respect to these axes, thus decreasing the
numerical work required.
7920 ~ _
- [ 270 X 10 - 270 X 30(16800)
= 7920 2 .2582 lb. The student Should use equations (19),
8064 (1 - 16800 X 8064) (20) and (21) to solve example problem 1 of
n N
Art. A9.5 in order to compare the two methods
of solution.
Mo = Zds!EI
- Z¢s = -270 0
--:i-n"""""f7in--;-it=-y-= A9.9 Problems. 200
00 00 20#/in.
Fig. A9.49 shows the redundant values at 110" 0'1 ,15' .
point A. The bending moments at the points I: 3 ~ 5" 150
B, C and Dare, L=20" L=20"! 100
1= 1 I. 1 ... 6"
MB = -.6038 X 30 =-18.11 in.lb. Fig.A9.52 Fig.A9.53
1 Fig.A9.54
Me =-18.11 + .2582 X 24 =-11.92 (1) Figs. A9.52, 53 and 54 show the same frame
rith three different loadings. Determine bend-
MD = .2582 X 24 = 6.19 !ing moment diagram for each loading.
100100100 w=8#/in. 20#/in.
Adding the static moment curve to that of 16"6" "6"1
Fig. A9. gives the final moment curve of L-21
1"~
Fig. A9.50.
~:'f ~:': 20
L=2'f 9" 9"
A9.8 Solution of Unsymmetrical Frames Using Principal r-'f .::L -*-
Axes. -16"
150 1 0
6" I-- --.18"
96
8"
96
~ 0
•
6"L
I
For the frame of A9.51 the axes X and y
through the elastic center are not principal Fig. A9. 55 Fig. A9.56 Fig. A9.57
axes because the elastic product of inertia (2) Figs. A9.55, 56 and 57 show same frame With
y 3 different loadings. Determine bending moment
I diagram for each loading.
100
/ yp
R1' 4,,-----J4, R2
Fig. A9.51 Fig. A9.52 bending moment diagram for
I xy is not zero. However, for the principal in Figs. A9.57 and A9.58.
axes x p and yp as shown, the product of inertia
is zero. Thus if the redundant forces at the 100 100
elastic center are referred to the principal
axes as illustrated in Fig. A9.52, then the
same form of equations for the redundants
result as for symmetrical structure, namely
~50
W
lliill
- Z¢s
r8+14''t8',
Mo = Zds!I (Same)- -(19) Fig.A9.60 Fig.A9.61
A10.l General. The Column Analogy* method is a Now assume we have a frame whose centerline
method that is Widely used by engineers in de- length and shape is identical to that of the
termining the bending moments in a bent or column section in Fig. AIO.l. The width of each
ring type structure. The method considers portion of this frame will be proportional to
only distortions due to bending of the llEI of the member. Fig. AIO.2 shows this
structure. assumed frame. Furthermore, assume that end B
of the frame is fixed and that a rigid bracket
The numerical work in using the column is attached to the end A and terminating at
analogy method is practically identical to point (0) the elastic center of the frame. The
that carried out in applying the elastic center frame is subjected to an external loading,
method of Chapter A9. w~, wa , etc.
Fig. A10.2
Fig. A10.l
This cantilever structure will suffer bend-
ing distortion under the external load system
w~, wa , etc., and point (0) will be displaced.
1I1I )/ltl/"/llf,'I,
Point (0) can be brought back to its original
undeflected position by applying a couple and
two forces at (0), namely, Mo , Xo and Yo as
shown in Fig. AIO.2. Since point (0) is attached
To find the bearing stress between the to frame end A by the rigid bracket these three
supporting base and the lower end of the forces at the elastic center (0) will cause
column, it is convenient to transfer the load point A to remain stationary or in other words
P to the column centroid plus moments about to be fixed. Therefore, for the frame in Fig.
the principal axes. Then if we let cr equal the AIO.2 fixed at A and B, the moment and two forces
bearing stress intensity at some point a acting at the elastic center cause the static-
distance x and y from the yy and xx axes, we ally indeterminate moments Mi when resisting a
given external loading causing static moments
(;iJ,p wr+t.l'l ,II tN·S",· ::-Ttlc ,~JjoInaJ1," c!"ue~oBt1u,nI@:"Iflultren:iJ :1:
= =.......,;;==========_===:::p=+=-{:;=:'p==a:=;)=y=._=+=::;=(P==..b=;J""X=.==========-115n:",,,·-n:e:¥ti'j"~'fdrr=eq'.,;,a.J[j-T' -=====
i cr-A---r-x----Iy-
are numbered (1), (2), (3) and (4). The area A10.4 Unsymmetrical Frames or Rings.
of each of these portions will represent a
In applying the column analogy method to
load P 1 , P s , etc. on the column in Fig. AIO.ll. unsymmetrical frames and rings, the moment of
Since the static moment is negative on each
portion the load on the column section will be the M/EI diagram must be taken about principal
axes and the moments of inertia with respect to
upward. principal axes.
P1 =-10(30) =-300, P" =-45x9 =-405 However, as explained for the elastic cen-
ter method in Chapter A9, the moments and
Ps =- 15 (6) /2 =- 45 , P4 = - 30 x 30 = - 900 section properties with regard to centroidal
axes can be used if Mx , My, Ix and I are modi-
P = L:P =- 300 - 45 - 405 - 900 =- 1650 fied to take care of the dis-Symmetr~ of the
From Fig. AIO.ll structure. In Art. A9.2 it was shown that the
redundant forces at the elastic center to un-
symmetrical frame sections was, (see equations
Mx = - (45 -\- 405) 9.375 + (300 + 900) 5.635 11, 14, 15 of Art. A9.2).
=
2531
Mo --~-p(
L:dsjEI - T same as for symmetrical
My = 300 x 12 + 45 x 10 - 405 x 6 - 900 x 12 frame)
= -9180
L:.0sY - L:.0s x (~ )
Mi = ~ + MxY + Myx X
o
= ----.,_--=I:.ly~- - - - - - - - (a)
A Ix Iy Ix (1 -~)
IxI y
POINT A. x = - 12, Y =- 20.625
Mi = -1650 + 2531(-20.625) (-9180)(-12) Yo = L:.0sx - L:.0s Y ( ~) - - - - - - - (b)
32 3188 + 3456
I y (1 -~)
= -36.06 IxI y
AlO.5
and k = (1 -~)
x y
The total area (A) of column section equals
(15/1) + (10/1) + (12/2) 31. =
Then sUbstituting in equations (a) and (b) The other properties required are: - Ix'
we obtain, I y and I xy •
The calculations to determine the location
of the centroidal x and y axes and the above
properties about these axes would be identical
From equation (2) we have, to that given in example problem 1 of Art. A9.5
of Chapter A9, where this same problem was
solved by elastic center method. The results
were Ix =
606.51, I y =
942.96 and Ixy 217.74.
The location of the axes were as shown in Fig.
=
Substituting values of Xo and Yo into this
equation, we obtain as the equation for Mi the AIO.13.
indeterminate moment in the column analogy
method, the following - The next step in the solution is to choose
a static frame condition and determine the
Mi =~+ (Mx-My)y + (My-Mx)x (6) static (Ms ) diagram. Fig. AIO.14 shows the
A klx kl y assumed static condition, namely, two cantilevers
because of the frame cut as shown. The figure
The true moment is the same as for the also shows the static bending moment diagram
symmetrical section, namely, made up of three portions labeled (1), (2) and
(3). The area of each portion of the moment
diagram divided by the (I) for that frame por-
tion will give the loads P on the column.
Thus the solution of an unsymmetrical
frame by the column analogy method follows the
same procedure as for a symmetrical section 240
(1)
except that equation (6) is used instead of
equation (3).
~
cut C[
~
AlO. 5 Example Problem - Unsymmetrical Section. (2)
240lb
Br--6 " 4c Fig. AlO.14
T
6"
1=2
6"
T
120# 1 =1 10" x
1
D
P 1.
P2
=- 1440 x 3/2 = -
=- 1440 x 15/1 = -21600
2160
These loads act on the centerline of the Frame Point C. x = 6.968, Y = 5.242
frame members and through the centroid of the
geometrical moment diagram shapes. These Mi = - 923 + 68.37 x 5.242 + 132.36 x 6.968
centroid locations are indicated by the heavy
dots in Fig. AIO.14 and their locations are =357.7
given with respect to the centroidal x and y
axes. The loads P~, P2 and P3 are now placed Me =Ms - Mi = 0 - (357.7) =- 357.7 in .lb.
on the column in AIO.13, acting upward because
they are negative. Frame Point D. x = 6.968, Y =-4.758
We now find the moments Mx and M which Mi =-923+68.37 (-4.758) + 132.36 x6.968
equal the moments of the loads P about the = -326
centroidal axes.
MD =Ms -Mi = 0 - (-326) = 326 in.lb.
Mx = - 2160 x 5.242 + 21600 x 2.258 + 4860
x 6.758 = 70290 The above results check the solution of
this same problem by the elastic center method
My = 2160 x 3.032 + 21600 x 5.032 + 4860 in Art. A9.5 of Chapter A9. The student should
x 5.032 139700= solve this problem by choosing other static
frame conditions.
To solve equation (6) we must have the
terms ~, M; and k. AlO.6 Problems
M' = Mx (~) = 70290 x 217.74/606.51 (1) Determine the bending moment diagram for
'j{ Ix the loaded structures of Figs. A10.15 to
= 25234 AIO.20.
400 400
My =My (~) = 139700 x 217.74/942.96 ~15T15t151 w= 10 lb/in.
I y L= 45"
= 32258 I =3
L=30" L= 30"
I =2
k = (1 _ Ixy 2 ) = 217.74 2
(1 - 606.51 x 942.96)
I =2
IxI y
= .9171 Fig. AlO.15 Fig. AID. 16
1-
(139700 - 25234)x 100 100
20 L = 20 L = 20
0.9171 x 942.96 I = 5
I = 5
whence, "--+-----;-_-=----.-_--11
8~ f.----24" ~
Mi =-923+68.37Y+132.36x - - - - - (7) 100 100
Fig. AID. I? Fig. AID. 18
For Frame Point A. x = - 5.032, Y =- 9.758
100 100 400
Mi = - 923 + 68.37(-9.758) + 132.36(-5.032)
~ ~ i
=- 923 - 667.15 - 666 •0 =- 2256
=
MA = Ms -Mi -1440 -1080 - (-2256) =
-264 in.lb.
MB = Ms-M~ =-1440- (-1230.6) = (2) Solve problems (2) and (3) at the end of
-209.4 in.lb. Chapter A9, Art. A9.9.
CHAPTER All
CONTINUOUS STRUCTURES - MOMENT
DISTRIBUTION METHOD *
Al!.l Introduction. The moment distribution set up in the two beams at B. The question is
method was originated by Professor Hardy Cross.** how much of this moment is developed by each
The method is simple, rapid and particularly beam. The physical condition which establishes
adapted to the solution of continuous structures the ratio of this distribution to the two beams
of a high degree of redundancy, where it avoids at B is the fact that the B end of both beams
the usual tedious algebraic manipulations of must rotate through the same angle and thus the
numerous equations. Furthermore, it possesses unbalanced moment of 270 will be distributed be-
the merit of giving one a better conception of tween the two beams in proportion to their abili-
the true physical action of the structure in ty of resisting the rotation of their Bend thru
carrying its loads, a fact which is usually a common angle. This physical characteristic of
quite obscure in some methods of solution. a beam is referred to as its stiffness. Thus let
The method of procedure in the Cross method it be considered that the stiffness factors of
is in general the reverse of that used ~n the the beam BA and BC are such that 162 is distribut-
usual methods where the continuous structure is ed to BC and - 108 to BA as shown in Fig. All.4.
first made statically determinate by removing (The question of stiffness factors is discussed
the continuous feature and the value of the re- in a following article).
dundant then solved for which will provide the Referring to Fig. All.4 again it is evident
original continuity. In the Cross method each that when the elastic curve rotates over joint B
member of the structure is assumed in a definite that it tends to rotate the far ends of the
restrained state. Continuity of the structure beams at A and C, but since these joints are
is thus maintained but the statics of the struc- fixed, this rotation at A and C is prevented or
ture are unbalanced. The structure is then moments at A and C are produced. These moments
gradually released from its arbitrary assumed produced at A and C due to rotation at B are re-
restrained state according to definite laws of ferred to as carry-over moments. As shown by
continuity and statics until every part of the the obvious curvature of the elastic curves
structure rests in its true state of equilibrium. (Fig. All.4), the carry-over moment is of oppo-
The general principles of the Cross method site sign to the distributed moment at the ro-
can best be explained by reference to a specific tating end. The ratio of the carry-over moment
structure. to the distributed moment, referred to as the
Fig. All.l shows a continuous 2 Span beam. carry-over factor, depends on the physical prop-
Let it be required to determine the pending mom- erties of the beam and the degree of restraint
ent diagram. We first arbitrarily ass~ that of its far end. (Carry-over factors are discus-
each span is completely restrained against rota- sed in a following article. For a beam of con-
tion at its ends. In the example selected ends stant section and fixed at the far end, the car-
A and C are already fixed so no restraint must ry-over factor is -1/2). In figure All.4 a fac-
be added to these points. Joint B is not fixed tor of -1/2 has been assumed which gives carry-
so tnis joint is imagained as locked so it can- over moments of 54 and -81 to A and C respective-
not rotate. The bending moments which exist at ly. To bbtain the final end moments we add the
the ends of eacn member under the assumed condi- original fixed end moments, the distributed bal-
tion are then determined. Fig. All.2 shows the ancing moments and the carry-over moments as
moment curves for this condition. (For calcula- shown in Fig. All.4. With the indeterminate
tion and formulas for fixed end moments see fol- moments thus determined, the question of shear,
lowing articles). Fig. All.3 shows the general reactions and span moments follow as a matter
shape of the elastic curve under this assumed of statics.
condition. It is noticed that continuity of the
structure at B is maintained, however from the All.2 Definitions and Derivations of Terms
moment curves of Fig. All.2 it is found that the
iRternal bending moments in the beams over sup- 1. Fixed-end moments:
By "fixed end moment" is meant the moment
port B are not statically balanced, or specifi- which would exist at the ends of a member if
cally there is an unbalance of 270. The next these ends were fixed against rotation.
step is to statically balance this joint, so
it is unlocked from its imaginary locked state 2. Stiffness Factor:
and obviously joint B will rotate (See Fig. The stiffness factor of a member is a value
All.4) until equilibrium is established, that is, proportional ~o the magnitude of a couple that
until resisting moments equal to 270 have been must be applied at one end of a member to cause
* Also see Chapter C2, Parts F, Land M. unit rotation of that end, both ends of the
** Paper - A. S. C. E. Proceedings, May 1930). member being assumed to have no movement of
All.l
All.2 THE MOMENT DISTRIBUTION METHOD
Example 3
M~ ?P~b~
I L - _ :S;::I M 2
Fig. AII.9
External applied moment
at joint in structure Fig. All. 10
Since the fixed end moments are statically Since the change in slope of the elastic
indeterminate1additional facts must be obtained curve between ends (1) and (2) is zero, theorem
from the laws of continuity in order to solve (2) as applying to fixed end beams can be re-
for them. In this book the theorem of area stated as follows.
moments will be used to illustrate the calcula- The sum of the areas of the moment diagram
tion of the fixed end moments as well as the must equal zero. And from theorem (1) the stat-
other terms which are used in the moment dis- ic moment of the areas of the moment diagram
tribution method. (Ref. Chapter A7) about any point must equal zero or in equation
The following well known principles or form:
theorem of area moments will be used:- ZM = 0 - - - - - - - - - - - - - -(1)
(1) The deflection of any point "A" on the ZMx = 0 - - - - - - - - - - - - -(2)
elastic curve of a beam away from a tangent to For a beam with variable moment of inertia the
the elastic curve at another point "B" is equal conditions for fixity are:-
to the mom~nt of the area of the M diagram be-
EI /Mdx/EI = 0
tween the points A and B about point "A". /Mxdx/EI = 0
(2) The change in slope between two points Figs. All.ll and All.12 show the static and con-
"A" and "B" on the elastic curve of a beam is tinUity moment areas, the total area of each por-
equal to the area of the M diagram between the tion and its c.g. location.
EI Substituting in Equation (1)
two points "A" and "B". M = Pab M~L M2 L 0
The "area moment" theorems will be illus- 2+2+2= - - - - - -(3)
trated by the applications to the solution of a
and from equation (2)
simple problem. Fig. All.5 shows a simply sup-
ported beam of constant moment of inertia and Mx about left end =Pab L+ a M~L L
modulus of elasticity carrying a single concen- 2 ' -3-+2' 3+
trated load. Figs. All.6 and All.7 show the
static moment curve and the shape of the beam M2 L 2L = 0 __ - -(4)
2 . 3
elastic curve. Now assume that the ends are fix-
ed as shown in Fig. All.8 and let the value of The values of M~ and M2 for any value of a or b
the fixed end moments be required. Fig. All.9 can now be found by solving equations (3) and (4)
shows the shape of the final moment curves made Table All.l gives a summary of beam fixed
up of the static moment curve and the unknown end moments for most of the loadings encountered
trapizoidal moment curve formed by the unknown in routine design and analysis.
end moments. Fig. All.10 shows the shape of the
elastic curve, the slope at the two supports be- W TABLE All. 1 W
ing made zero by fixity at these points. 2
~r f rt llWL2~ ------y 5wL 2
"C i '--+-1
WL r:::WL122
p l2 ~ - - --r 192 I !==y 192
2 2
t------L-------f
Fig. All. 5 r wa
2
12L2 ( 4aL- 3a 2 )
l~ Fig. All. 6
Fig. All. 7
Fig. All. 8
Al1.4 THE MOMENT DISTRIBUTION METHOD
wa "
30L" (IOL " - l5aL+ 6a "-I
p/ wa"
20L" (5L- 4a) Let flB unity
Then 1 ML or MB = 3EI = stiffness Factor of
3EI L
M Beam BA of Fig. All.13. A moment applied at B
M~(3~-1) ~ ~ J_M~(3~_1) produces no moment at A since end A is freely
L L I-a -+- b --1 L L supported. Thus the carry-over factor for a
beam freely supported at its far end is obvious-
ly zero. Consider the beam of Fig. All.14. Due
eA=o
(EI is constant
L Bf
)
MB
----r
,....-=:==-------!----1 ---.1.
::Sl~ -1~
Curve MBL
AL ML 2EI
3 2EI
Fig. All. 13
~
t
~=------+----+---==::=.-+
L
3
Fig. All. 14
ness factor of a beam freely supported at its MA = - 2KF - - ~ which is the general ex-
far end is 3/4 as great as one fixed at its far MB K (3 + F) 3 +F
end. Furthermore in one case the carry-over pression for carry-over factor for a degree of
factor is zero and in the other case it is 1/2. fixation F.
It is therefore obvious that the values of these
two terms depends in part upon the restraint AII.5a Example Problems
or degree of fixation of the far end of the
To obtain a definite conception of the true
beam. mechanics of the "Cross" method, the reader is
advised to follow thru the detailed solution of
AII.5 General Expressions for Stiffness and Carry- the following simple problems. In these prob-
over Factor in Terms of Fixation Factor (F) lems, the moment of inertia in any span has been
at Far End of a Beam
taken as constant and all joints have been as-
In the beam of Fig. All.14 (F) fixation sumed to undergo no translation. Problems in-
factor at A was unity since beam had been taken volving variable I and joint translation will be
as completely fixed at A. It was found that:- considered later.
MB = 4EI8B and MA = - 2 EI 8B Example
L L
Then 8B = unity and let EI =K for simplici ty. Problem #1
L
Then MB = 4K
MA = -2K Stiffness Factor Ei = K o1 11 1 0
Likewise the results for the beam of Fig. All.13 Distribution Factor K/1: K o1 .5 .5 10
give Carry Over Factor o .5 .5 .5 .50
MB =3K Fixed End Moments --883768=. 0833wL 2=-768 768=. 0833wL 2 =-768 883
MA =0
1-1s_t_Ba_lan_cl_ng -j0_ll_5~~ ~~_-1_15+0_ _-l
Figs. All.15 and All.16 show these results. Carry Over 1/2 0 ~57. 5 -57.5............... 0
Fig. All.17 shows the general case, the fixa- 2nd Balancing 00 00 00
tion factor at A being F. The difference be-
tween Figs. All.15 and All.17 is that the slope Final Moments -88~ 883 -710.5710.5 -883 883
at end A has changed but 8B the slope at end B Conventional Moment Signs + • • • + +
remains the same. -Overhang Moment = .5 x 51. 5 x 25.75 •. 25 x 51. 5 x 17.17 = 883
end of a member clockwise is positive. 4. Calculate the carry-over factor C for each
We now begin the solution proper by first end of all members C = 2F/(3 + F). Thus for
unlocking joint B from its assumed fixed state. beams fixed at far end F = 1 and thus C = 112.
We find a moment of -883 on one side and 768 on For pinned at far end F = 0, hence C = O.
the other side of joint or a static unbalance 5. Calculate the fixed end moments (f) for giv-
of -115. Joint B therefore will rotate until a en transverse member loadings or support deflec-
resisting moment of 115 is set up in the members tions, using equations in summary Table All.l.
BA and BC. End moments which tend to rotate end of beam
The resistance of these members to a rota- clockwise are positive moments. (See Art. All.2)
tion of Joint B is proportional to their stiff- 6. Considering one joint at a time, unlock it
ness. The distribution factor based on the from its assumed fixed state, all other joints
stiffness factors is 0 for BA and 1 for BC. remaining locked. If an unbalanced moment ex-
Thus 1 x 115 = 115 is distributed to BC at Band ists balance it statically by distributing a
o x 115 = 0 to BA. Joint B is now imagined as counter acting moment of opposite sign among the
again locked against rotation and we proceed to connecting members according to their D or dis-
Joint C, which is now released from its assumed tribution factors.
locked state. Since the joint is already stati- 7. These distributed balancing moments produce,
cally balanced, no rotation takes place and the carryover moments at far-end of members equal to
distributing balancing moment to each span is the distributed moment times the carry-over fac-
zero. Next proceed to joint D, and release it. tor C and of the same sign as the distributed
The unbalanced moment is, 115 so the joint is moment. Record these carry-over moments at far
balanced by distributing -115 between DE and DC ends for all distributed moments.
as explained above for joint B. 8. Repeat the proceedure of unlocking each
As pointed out in Art. All.5, when we ro- joint, distributing, and carrying over moments
tate one end of a beam it tends to rotate the until the desired precision is obtained, stopping
far fixed end of the beam by exerting a moment the solution after a distribution. The final
equal to some proportion of the moment causing moment at the end of any member equals the alge-
rotation at the near end. For beams of constant braic sum of the original fixed end moments and
section and fixed at their far ends, the carry all distributed and carryover moments.
over factor is 1/2 as explained before. Thus
the distributing balancing moments in line 4 Example Problem #2
produce the carry-over moments as shown in line .5#/in. ./
l#/in EI is Constant
5 of the table. This completes one cycle of the
moment distribution methOd, which is repeated
-L
-,,- rrff T11 TIl T TTTTTT TTTTTT TTTTTTTr~
to joint B and release it again from its assumed Carry-Over C o .5 .5 5 .5.5 .5.5 .50
locked state. There is no unbalance since the Factor
Carry-Over
0115?<~ 1 9 2 > < 0 -o
72 57.5
-
o
-
96 -96
X.292
- ~X-1150
0 -57.5 -72
it balanced under the carry-over moments of 57.5
and -57.5. Thus the distributing balancing mom- 2nd Balancing
0-72?X~-32.9 0 O~2.9
~><:: 720
ents are zero. Joint D is likewise in balance Carry-Over -12.3 -36 ;-X~r--
16.5
-
0 36 12.3
since the carry-over moment from C is zero. All 3rd Balancing O12.3:3><~20.6~
-
o
-
0 o XO.6
r----- -
-15.~12.3 0
- - f--
joints can now be released Without any rotation Carry-Over 7.7 6.1 10.3 ·10.3 0 -6.1 -7.7
moments plus all distributed balancing and car- 5th Balancing o 1. 3 1.72. a 00 -,2.2 -1.7 -1. 30
ry over moments. Final Moments -883 883 -610.4 610.4 -344 344 -610.4 610.4 -883 883
Conventional Signs +. ++ ++ ++ ++
All~6 General Summary of Proceedure
1. All computations should be written on, or Example Problem 2 is similar to problem 1 but two
adjacent to the diagram of the structure. spans have been added. We first assume all joints
2. Determine the stiffness factor K for each locked against rotation. The stiffness factor of
end of each member. K = (3 + F) EIIL, where F each span is proportional to EIIL or llL since EI
is the degree of fixity at far end of member. is constant. The carry-over factor is 1/2 as in
If all members are assumed fixed at far end then previous example. Fixed end moments are calCUlat-
K is proportional to IlL assuming E as constant. ed as shown. Unlock joint B, the unbalanced mom-
3. Determine the distribution factor D for each ent is -115. Balance the joint by distributing
member at each joint of structure. D = K/~K. 1 x 115 to BC and zero to BA. Proceed to joint C,
All 7
and unlock, all other joints remaining fixed necessary. These modifications usually involve
against rotation. The unbalanced moment is rather long expressions for expressing the stiff
(-768 + 432) = - 336. Balance by distributing ness and carryover factors of a member in terms
.428 x 336 = 144 to CB and .572 x 336 = 192 to of the fixation given by adjacent members. It is
CD. Proceed to joint D, and release. The un- felt that it is best to keep the method in its
balanced moment is zero which means that joint simplest form which means that very little is to
D is in equilibrium, thus no distribution is be remembered and then the method can be used in-
necessary. Proceed to joint E and F and balance frequently Without refreshing ones mind as to
in a similar manner. The distributed moments many required formulas or equations.
will be the same as the values for joints Band There are however several qUite simple modi
C due to symmetry of structure and loading, fications which are easily understood and re-
however, the signs will be opposite under our membered and which reduce the amount of arithmet-
adopted sign conventions. The next step is the ic required considerably.
carry-over moments Which are equal to 1/2 the For example in Problem #2, joints Band F
distributed balancing moments. This operation are in reality freely supported, thus it is need-
is shown clearly in the table. Values of all less arithmetic to continue locking and unlocking
moments are given only to first decimal place. a joint which is definitely free to rotate.
The first cycle has now been completed. Cycle Likewise due to symmetry of structure and load-
two is started by again releasing joint B. We ing it is only necessary to solve one half of the
find the joint has been unbalanced by the carry- structure. Due to symmetry joint D does not ro
over moment of 72. Balance the joint by distri- tate and thus can be considered fixed, which
buting - 72 x 1 = - 72 to BC and zero to BA. eliminates the repeated locking and unlocking of
Proceed to joint C. The unbalanced moment is this joint.
57.5. Balance by distributing - 57.5 x .428 = A second solution of Problem #2 is given in
- 24.6 to CB and the remainder - 32.9 to CD. Example Problem #3. As before we assume each
Proceed to joint D. There is no unbalance at joint locked and calculate the fixed end moments.
this joint since the carry-over moments are in Now release or unlock joint B and balance as ex
balance, thus no distribution is necessary. plained in previous example #2. Before proceed
Proceed to joints E and F in a similar manner. ing to joint C, carryover to C from B the car-
The carryover moments equal to 1/2 of the ry over moment equal to 115 x 1/2 = 57.5. Joint
2nd set of balancing distributed moments are now B is now left free to rotate or in its natural
carried over as shown in the table. The second condition. Proceed to joint C and unlock. The
cycle has now been completed. This operation unbalanced moment = (-768+432+57.5) =- 278.5 or
has been repeated five times in the solution 278.5 is necessary for equilibrium. This moment
shown, or until the values of the balancing and is distributed between two beams, CD which is
carryover moments are qUite small or negligible. fixed at its far end D and CB which is freely
The final moments equal the algebraic summation supported at the far end B. The stiffness fac-
of the original fixed end moments plus all dis- tor is equal to (3 + F) EI/L (See Art. All.5).
tributed and carryover moments. One require- Hence for CD stiffness factor = (3 + 1)
ment of the final end moments at any joint is EI/L = 4 EI/L. For CB stiffness factor = (3 + 0)
that the algebraic sum must equal zero. The EI/L = 3 EI/L or in other words the stiffness of
other requirement consistant with the common a beam freely supported at its far end is 3/4
slope to all members at any joint is given by as great as when fixed at its far end. Thus the
equation (5) of Art. All.8. The results at stiffness factor of CB at C is .75 x .0104 =
joint C will be checked using this equation. .0078. The carry-over factor C to B is zero
since B is left free to rotate. (See Art. All.5)
fillcd - .5 tl !'Idc = lS.c..d
fillcb - .5 tl Mbc Kcb Example Problem #3. Simplified Solution of Prob-
lem #2
SUbt. values
(610.4-432)-.5 (344-(-432)]_178.6-44_
[-610.4- (-768)J -.5 (883-768f157.6-57.3- rfT·f 'r'
T (TI#(1 TT TT T TiT f ~ixe~
-r
1.343 j:-51. 5~<-B---96---""C--72---;
E
K"rl .0139
Ratio of stiffness factors = ~ = .0104 = 1.34. Stiffness Factor (k) o .0104=k . 75x. 0104=. 0078 .0139 = 1/72 = .0139 pc
Thus the distribution is according to the K Distribution Factor
o = K/ZK 0 1 .36 .64 o 1
ratios of the adjacent members. Carry-over factor (e) a .5 0 .5 .5
-432
Simplifying Modifications - Example Problem #3 Fixed end moments ·883 768 -768 432
Balance joint B 0 115 ~ _
The solution as given in Problem #2 repre- -=C=ar=ry=_o-'-ve-=r"-I/=2-'o-c:C,-----IF-=--- 5 -----+----
----.c--=-57=-.e:-t
Balance joint C ---===100.1 178. 4 ~-;::--,:t----
_
sents the "Cross" method in its fundamental and -=c"-ar"-ry--o-'-ve-r-'o-C:B-,=-o----If::-o-~ 89.2
most elementary detailed form. Many modifica- o =::----+---------+:--O~
-=B::.:ala:::n.::..::ce-,-io:::in:.:..' 0 342.8
tions of the general method have been presented, Carry-over to C
Final moments -883 883 -610.4 610.4 -342.8 342.8
in the most part for the purpose of eliminating Conventional signs + + ++ ++
part of the arithmetic or the number of cycles
All.S THE MOMENT DISTRIBUTION METHOD
to CD. Now carryover to joint B, 0 x 100.1 = Stiffness Factor k o .0104 = 1/96 = .0104 .0139=1/72=.0139 .0139
o and 0.5 x 178.4 = 89.2 to D. Proceed to joint Distribution Factor D;ik o1 .428 .572 .5 .5
D and release it from its assumed locked state. Carry-Over Factor 0.5 .5 .5 .5
The unbalanced moment is -432 + 89.2 = - 342.8, Fixed End Moments -883 768 -768 432 -432
we balance by distributing 342.8 between DC 1st Balancing o 115 144 192 216 216
Carry-Over 72 57.5 108 96
which has a stiffness of 0.139 and the support 2nd Balancing 0-72 -70.7 -94.8 -48 -48
E which has an infinite stiffness or zero goes Carry-Over -35.3 -36.0 -24.0 -47.4
to DC and 342.8 goes to rigid support. The car- 3rd Balancing o 35.3 25.7 34.3 23.7 23.7
Carry-Over 12.9 17.6 11. 8 17.2
ry over moment to C from D is zero since 0.5 x 4th Balancin[. 0-12.9 -12.6 -16.8 -8.6 -8.6
o = O. The final moments thus equals the sum- Carry-Over -6.3 -6.4 -4.3 -8.4
mations as shOwn Which of course are equal to th 5th Balancing
Final Moments
06.3
-883 883
4.6 6.1
-644.3 644.3
4.2 4.2
-187.3 187.3
results shown in Problem #2. Conventional Signs ++ ++ ++
Example Problem #4
.5 x 216 = 108 is carried over to C. Joint D is
Problem 4 is similar to Problem #2 and #3, left unlocked or in its true state of restraint.
except that the support at D is assumed as hav- Joint C is now unlocked. The unbalanced moment
ing 50 percent fixity. Thus 50 percent of any
moment at this point produces rotation of the Solution#2
member DC at D. Example .rU/in 50% Fixed
r-
T tttttttt 1 1
In continuous wing beams, which fasten to- T
gether by fittings on a support, it is commonly 5
1.5
96 72--->
Joint Do Since the moment of inertia is con- Since the fixed end moments are due to both
stant and the spans are constant, the relative lateral loads and support deflection, the values
stiffness factor of the beam is 1. In the so- as listed in the solution table will be explain-
lution shown since beam is freely supported at ed in detail.
B this joint is left free to rotate after re-
leasing and thus the stiffness factor of beam Fixed End 110ment For Lateral Beam Loading
CB is 3/4 x 1 = 3/4, when compared to one llaving The distributed airload is trapezoidal in
fUll fixity at B. shape. The fixed-end moments for a trapezoidal
Since the first step in the solution proper loading from Table All.4 are:
is to assume the joints fixed against rotation,
it is evident that deflecting one support rela- 111_2=L 2 (5u+2v) (See Fig. (a))
tive to an adjacent support will produce mom- 60
ents at the ends which are assumed fixed against 112_1=L 2 (5u + 3v) (See Fig. (a) )
rotation. 60
M ~4EI
Err //,
ML port B is deflected a
distance d relative to
the support point C. If
-L
CD
1& L ,®
~
L ~ -E~ the member is of constant
6 ~~
~L
J cross-section the point
of inflection will fall For Span BC:
6 at the beam midpoint and
40 2
Fig. All. 18 11B - tlc. 60 (5x3+1) = 426 in. lb.
By moment area prin-
40 2
cipal deflection 60 (5x3+l.5) 440 in. lb.
d tlL L -tlL 5 L
4EI (3 4EI (3 For Span CD:
or d 40 2
60 (5x3.5+1) = 494 in. lb.
hence 40 2
11 6EId the magnitude for the fixed end 11dc = 60 (5x3.5+l.5) = 507 in. lb.
L"
moment due to a transverse support settlement Fixed End 110ments Due to Support 110vement
of d.
From Art. All. 8 11 = ~EId/L2
Example Problem #5. Continuous beam with de-
For Span BC:
flected supports.
General 1 Deflection of B relative to C = 5/16 inch. Hence,
4 #/in. 3 #/in. lr.049 AI.
11bc =11 c b = 6 x 10,000,000x .03339 x .3125/40 2 = 390
~ I~ I ~~l~:
data:
Oi;::9
in. lb.
----+- -I--
D
40 40 --110~-
Defleciion of SupP!'rt~
For Span CD:
DI I I J 2
11 cd =11dc = 6 x 10,000,000 x .03339 X .1875/40 = 234
- -Fixed due to
~-r---r----,_,--~-,---rilisymmetry
in. lb.
Soluiion: For signs of the moments due to these deflections
A B C D
see Art All.2. Having determined the fixed end
Siiffness Factor K o1 3/4xl=.75 1 1 0<' moments the general distributing and carrying
Distribuiion Factor = K! LK o1 .429 .571 o 1 over process follows as indicated in the solution
Carry-over Factor 1/2 o 1/2 1/2 table. Thus at joint B, the unbalanced moment =
Fixed
End
} Due to
Lateral Load
50 _426 440 -494 507 (50-426 + 390) = 14. Balance by distributing - 14
Moments Due to .support o 390 390 234 234
x 1 = - 14 to BC and zero to BA. Carryover.5 x
deflectlOn -14 = - 7 to C. Considering joint C, the unbal-
Balance joint B o -14 anced moment = (440 + 390 + 234 - 494 - 7) = 563. Bal-
Carry-over to C -7
Balance joint C -241 -322
ance bydistri buting - 563 x .571 = - 322 to CD and
Carry-over to B, D 0 -161 -563x .429 = - 241 to CB. carryover.5 x -322 =
Balance joint D o -580 - 161 to D. At joint D the unbalanced moment =
Final Moments 50 -50 582 -582 580 -580 (507 + 234 - 161) = 580. Thi sis balanced by di s-
Conventional Signs tributing zero to DC and -580 to the fixed support.
All.lO THE MOMENT DISTRIBUTION METHOD
Af~· _
lem has been solved using three different de-
L grees of restraint at ends A and B. Joint 0 is
a welded joint and full continuity is assumed
thru this joint. The solutions as given in Fig.
Fig. a' -r ~Mba
All.20 give only the moments due to joint rota-
tion under primary bending moments. The effect
c::::::::=------+----i ---±-.E X of axial deformation and secondary moments due
Al~ B to member deflections is omitted in these solu-
_.- - - - - . l . L L/3 tions. These factors are treated in later arti-
3
~MbaL cles.
~ In a practical problem the degree of re-
straint at points A and B would be determined by
_ ~~L 2/3 _ ~~~L . ~ the type of fitting used and also on the rigidity
of the adjacent fuselage or Wing structure. As
__ (AMab + .5 &Iba) illustrated in later examp~es, the moment distri-
3EKab bution method permits the consideration of the
where rigidity of the adjacent structure without adding
t any difficulty, while such methods as least work
r- -+
Kab = of AB
Since the angle QA must be the same for A
18"
------T-----
~2'~ ~ B
members meeting at A, the general relation be-
tween the moment increments of any two members
such as AB and AC must be, .-t.
.,,~
<.,.
'fl'
--"
... "" '%>
~ 0.
I
I
I
I
i>
~'\; ~
~
"'"v:
~'~
....'O b;:
'!::;~
T
~b + .5 &Iba = Kab ?;;(/ " "'o.~?, I ~
<;:)'0 '\>',' 'i:;~'
~ '~" .?>.
--- ~
AMac + .5 &Ica Kac I "" ....'\."""....c:s
00 ~ ~dl 'l> \"'>'V\
~ ~o ~~
To make this equation consistent with the assum-
ed sign convention, that is, the carryover mom- "'~ ~4
ent has the same sign as the balancing distri-
buted moment, the above equation must be modi- 6'~OO# Fig. All. 19
fied as follows:- * Also see Chapter C2, Part H.
All. 11
-'
~ 0 <::I)":J"J
factor D for each member at each joint which Member - K- Value C. O. Factor
equal K/ZK is recorded in Don each member 2F
~ (3+ F)
around each joint. Thus any balancing moment 3-F
is distributed between the joint members as per 2 X 1 _1
these distribution factors. The carryover OA=-.!..(3+l) =.4, 3 + 1 -"2
20
factors for all members is 1/2. The fixed end
moments due to external loading are computed OB"~·;(3+0) =.3, 3+0 2xO =0
0~11~
moment of 300 is provided entirely by the sup- ~c,-b,-145
port and zero by the member AO. The carryover -372.5 t f 155
moment C, to 0 is zero. Releasing joint 0, the ...
..... .0 ....
unbalanced moment of 300 is balanced by distri-
buting - 300 between the three members accord- SOLUTION #2 !
ing to their D values, thus - 300 x .416 =
- 125 to OA; - 300 x .168 = - 50 to OC and 000
. . . <ill.... Fig. All. 22
- 125 to OB. To prevent confusion it is recom- by distributing .75 x 25 = 18.75 to CO and the re-
mended that a line be drawn under all distri- mainder of 6.25 to the support, since the fixity of
buted balancing moments, thus any values above the support at C has been assumed as 25 percent. A
these lines need not be given further consid- line is drawn under the 18.75 and the carryover
eration and only values below the lines need moment of 9.37 is taken over to O. One cycle has
be considered in later balancing of the joints.
Immediately after distributing the moments at now been completed. Returning to joint A, we
find -62.5 below the line. This is balanced by
joint 0 the proper carryover moments should be distributing zero to OA and 62.5 to the fixed
taken over to the far end of each member, thus support. A line is drawn under the zero distri-
- 62.5 to A, - 62.5 to Band - 25 to C. Joint
B is next considered. The unbalanced moment is buted moment to AO and the carryover moment of
zero is placed at O. Considering joint 0 for
-62.5 and it is balanced by distributing 62.5
to BO since the pin support has zero stiffness, the second time the unbalanced moment is 9.37 +
31.25 + 0 = 40.62 or the sum of all values below
or no resistance to rotation. A line is drawn the column horizontal lines. The joint is bal-
under the 62.5 and the carryover moment of
31.25 is placed at O. Joint C is considered anced by distributing - 17 to OA and OB and
-6.62 to OC. Lines are drawn under these bal-
next. The unbalanced moment of -25 is balanced ancing moments as shown in Fig. All.2l and the
SOLUTION #1
carryover moments are taken over to the far
ends before proceeding to joint B.
This general process is repeated until joint
A has been balanced 5 times and the other joints
4 times each, as indicated in the figure the dis-
Stiffness Factor K
tributing values have become qUite small and it
I 2 is evident that a high degree of accuracy has
OA=r:= 20 = .10
been obtained. The final end moments at each
1. 5 - 10
OB = 15-' joint equal the algebraic sum of the values in
OC= -h=~ each column. A double line is placed above the
~ K - Joint 0 =. 24 final moments as a distinguishing symbol. In
C. O. Factor = 1/2 the figure the letters band c refer to balancing
For All Memb.
and carryover moments, the subscripts referring
to the member of the balancing or carryover
oneration. Any order of joint consideration can
be used in reaching the same result.
Solution #2 of Problem 7
Fig. All.22 gives a second solution. With
the end conditions known at A, Band C, the modi-
fied stiffness factors of the members can be .
found together With the modified carryover fac-
tors, thus making it necessary to balance joint
Fig. All. 21 o only once and carryover this final far end
All 13
moments of each member. The figure gives the assumed to give 50% fixity to these joints. In
calculation of the modified or actual stiffness Table All.2 a modified stiffness factor is cal-
and carryover factors. With these known the culated for members GI, FI, and FH using a 50
solution is started as before by computing the percent fixity at their far ends. The last
fixed end moments due to transverse loading on column of Table All.2 gives the summation of the
member AO. Joints Band C are released and member stiffness factors for members intersect-
since no fixed end moments eXist, no balancing ing at each joint.
is required and the joints are left in their 120#
true state of restraint instead of locking and
iI2~r-
3200"#
unlocking as in solution #1. Releasing joint 0 r G I r.
I- /'
r --7l\A1l/~ /
from its imaginary fixed state the unbalanced /'
moment is 300 which is balanced by distributing
50% Fixity
- 300 between the 3 connecting members accord- Assumed
ing to the new distribution factors at joint O. '-.. ~"'" 100# I
Thus - 300 x .482 = - 145 to OA; - 300 x .361 =
- 108.2 to OB and - 300 x .157 = - 47 to OC. Engine
Mount
;r"f
2400~
A
-hO"1-
~ .
{4t:0"# F H
The carryover moment to A - - 145 x .5 = - 72.5 \ /
to B = - 108.2 x 0 = 0 and -47 x .154 = - 7.2 to
~ /
C. Fig. All. 23
Example Problem #8
"- \ !Landing
" Gear
Figure All.23 shows the forward portion of
a fuselage side truss. Due to eccentricity of TABLE All. 2
engine mount and landing gear members, external
moments are produced on joints A, Band D as Member Size Tube
Length
L 1
1 x 1000
L
i~tor
L
each joint
shown. Furthermore lateral loads due to equip- Stiffness il-
ment installation are shown acting on members BE It Factor L
Joint Stiffness
and CD. Assuming the fuselage welded joints
IlK 1-1 8 - .049 41.25" .0240 0.582 A 5.80
produce rigid continuity of members thru the KG 1-1 8 - .049 38.75 .0240 0.619 B 4 81
joint, the problem is to find the end moments in Gl
AC
Ul
1-1
8 - .049 34.0 .0240
.049 19.25 .0588
.618*
3.06
C 6.95
6.14
2 - D
all the members due to the eccentric joint mom- D -1 2 - .049 30.0 .0588 1.96 K 2.58
ents and two lateral loads. The effect of joint DF 1-1 2 .049 30.0 .0588 1.96 F 4.42
Fa 1-3 8 - .049 34.0 .0449 1.16* G 3.21
translation and secondary moments due to deflec- AB U3 4 - .049 34.5 .0947 2.74
tions and axial loads is to be neglected in this ED
FG
-
1
4
8
-
-
• 49 35.4
.049 345
.03339 0.944
0240 0.695
example. BC
CD
1 1
1-1
2 - .049 39.5
.035 41.0
.0588
.0178
1.49
o 434
(K is constant
for all aea-
8 -
Solution: DG 1-1 2 - 049 16.0 .0588 1 28 bers)
Table All.2 gives the calculation of the 1'1 1-1 14 _ .049 18.5 .03339 0.602*
( * stiffness factor - 7/8 L because of 50% fiXity)
stiffness factor for each truss member. The L
fuselage truss aft of joints I and H have been
Fig. All. 24
-2981 1725
-282 b -141
=-sa b -176
::-g
-187 b
::-g
- 94
- 18 b b - 17 -16 b - - x .286 - 4
-1 b 2 2b -1 .:-5b - - - - - 1
1b -0 =----2 b -
-0
5 ob -.JL
-684 b_O --.lb -21 - 5
401 -195 b 0
-115
50% Fixed
}---~
50% Fixed
"'~I'"'~I<-~II'"
COMCOCON
U':>~
I I I
co
~
I
.602
.0 .0
'"'1"'''I~~II~
~~~~ll"J~
I I I I
H 50% Fixed
-2400
- 1268 b - _ - 634 - 4451 1 222
318 - - b 635 406 b 203
183 b ~ 3i4 b 629 ~b 314
~ ---HQ ~b -=-45 - 70 --b-140 - 82 b _ x. 286 - 23
60 b 30 44 b 88 88 b ~ =--9b - 3
- 777 - 33 - 21 b -=-iO -7 - - b - 15 - 0b o
--3
:-""50" 209 b __
7 ~b - 90 - 26
1094 647 b 0
206
All. 14 THE MOMENT DISTRIBUTION METHOD
Fig. All.24 gives the solution of the by distributing 183 to AC and 164 to AB with
problem. The procedure in this solution was as carryover moments of half these values to ends
follows: C and B respectively.
The stiffness factor K for each member as The student should now be able to check the
computed in Table All.2 is recorded in the cir- rest of the solution as given on Fig. All.24.
cles adjacent to each truss member. The carry The solution could be made with any order of
over factors for all members is 1/2 except for joint consideration. If any particular joint ap-
modified members GI, FI and FH for which the pears to be nearly balanced, it is best to skip
carryover factor to the 50% fixed ends is .286. it for the time being and consider those joints
The distribution factor for each member at each which are considerably unbalanced.
joint is recorded at the end of each member, and The final moments at the end of each member
equal s K/L:K. are given below the double lines.
The next step in the solution is to compute
the fixed end moments due to the transverse Example Problem #9
loads on members. Fig. All.25 represents a cross section of a
For member BE welded tubular steel fuselage. The top and bot-
tom members which are web members in the top and
MBE = Pab 2 /L 2 = 120 x 29.25 x 122 /41.25 2 = -al8"
2 2 bottom fuselage trusses are subjected to the
MEB = 120x 29.25 x 12/41.25 = 725"# equipment installation transverse loads as shown.
McD = 100 x 20 2 X 10/302 = - 445 Let it be required to determine the end bending
moments in the rectangular frame due to these
MDC = 100 x 10 2 X 20/30 2 = 222 transverse loads assuming full continuity thru
These moments are placed at the ends of the joints.
members on Fig. All.23 together with the eccen-
tric jOint moments. The process of unlocking Solution:
the joints, distributing and carrying over mom- Fig. All.26 shows the solution. The distri-
ents can now be started. In the solution as bution factors based on the member stiffness fac-
given the order of joint consideration is tors are shown inDat ends of each member. The
ABCDEFG and repeat, and each joint has been bal- first step is to compute the fixed end moments
anced three times. due to transverse loads, on members AB and CD us-
Consider joint A:- ing equations from Table All.l. The magnitudes
Unbalanced moment = 2400. Balance by dis- are 1890"# for AB and 2025"# for CD.
tributing - 2400 as follows:- Joint B is now released from its assumed
To AC 2400 x .527 = -1268. Carryover fixed state. The unbalanced moment of 1890 is
to C = -634 balanced by distributing - 1890 x .247 = - 467 to
To AB 2400 x .473 = -1132. Carryover BA and the remainder of -1423 to BD. The carry
to B = -566 over moment to A = - 465 x .5 = - 233. Due to
symmetry of structure and loading only one half
Proceed to Joint B:- of frame need be considered and hence these car-
Unbalanced moment (-566 + 3200 - 298) = ry over moments to A are not recorded. However,
2336. Joint is balanced by distributing - 2336
in balancing joint A it will throw over to B the
to connecting members as follows:- same magnituae of carryover moments as thrown
To BA = - 2336 x .569 = - 1330. Carryover to over to A from B but of opposite sign since the
A - 665 original fixed end moment at B is minus. Thus
To BC = - 2336 x .310 = - 724. Carryover to
233 comes to B from first balance of A as shown
C - 362 in the figure. The distributing moment to B of
To BE = - 2336 x .121 = - 282. Carryover to -1423 produces a carryover moment of -1423 x .5=
E - 141
The convenient device of drawing a line
under all balancing moments is used to prevent
confusion in later balances of the joint.
-712 at D. ~ 27"
10"
300#
7"
300#
H
1'.... 035 Tube
T
Proceed to Joint C:- K=' 0~~37 = .000458
Unbalanced moment = (-634 - 362 - 445) = -1441
The joint is balanced by distributing 1441 as
follows:- 0>
'<l' '<l'
To CA = 1441 x .44 = 635. Carryover to A = 318 36" o .....
o
CB = 1441 x .214 = 309. Carryover to B = 155 I 0
CE = 1441 x .064 = 92. Carryover to E = 46 co . . . "I..:l
CD = 1441 x .282 = 406. Carryover to D= 203 "'-.
M II
This process is continued for the remainder of 600#
..!. :.::
the truss joints. After all joints has been
balanced once, on returning to joint A we find 1---13.5'+13.5"~
below the lines an unbalanced moment of (318-
665) = -347. The joint is balanced a second time 1-1/8 - . 035 Tube
K =!...:·01782=
L 27 .
00066 Fig. All. 25
All. 15
IIA liB
Static M/I curve Tr1al- curve Trial-curve
I I
Beam 110m. Mom. )10m..
Portion Av. arm about Av. arm Av. Arm 110m.
Fig. All. 26a Ord. left left Ord. left yx Ord. x yx
= y end = x end ... (y) ,~nd x • y
yx
All. 10 Continuous Structures with Members of
1 107 2.67 286 110 2.06 227 6.7 2.67 18
Variable Moment of Inertia* 2 397 6.31 2510 133 6.07 807 24.9 6.27 156
3 770 10.16 7800 143 10.0 1430 48 2 10 16 488
In Arts.All.3, 4 and 5 consideration was 4 1120 14.10 15800 130 13.94 1810 70.0 14.10 990
given to the derivation of expressions for 5 1440 18.07 26000 110 17.96 1976 90.0 18.06 1630
6 1760 22.06 38900 90 21.92 1970 110.0 22.05 2430
fixed end moments, stiffness and carry-over 7 1615 25.88
29.77
41600 67.3 25.88
29.76
1740 123.5 26.04 3480
8 975 29000 40.7 1215 117.0 29.94 3490
factors for beams of uniform cross-section. 9 420 33.65 14180 17.5 33.68 590 91.0 33.88 3080
Many cases occur in routine design where mem- 10 100 37.33 3733 4.2 37.33 157 62.5 37.86 2360
*iy 8504 845.7 743.8
bers have a variable cross section. This ~yx 179809 11922 18122
article will illustrate the calculation of the i=~E· ~ x .11922 .14.1" x .18122 _ 24.4"
i 8504 845.7 743.8
fixed end moments, stiffness and carry-over • 21.15"
factor for a beam With variable moment of in-
ertia. The effect of axial load on these fac- * Actual area is 4 times the results shown. For calculation of
fixed end moments S~c.o. factors only relative values are
necessa~y.
*Also see Chapter C2, Part L.
All. 16 THE MOMENT DISTRIBUTION METHOD
1419 = 100 x 40
4c xl
, whence c = .707 Example Problem 1.
10 was taken as the value at the A end or 1. Fig. All.33 shows a single bay rectangular
(See Fig. All.27). bent carrying a distributed side load on one
Therefore KAB =
.707 IA/L leg as shown. The K or I/L values for each
member are given on the figure.
Similarly for end B:- Under the given loading it is obVious that
100x40
EGB =
4(743.8 -' .532 x 845.7) 4c xl = the frame will sway to the right or in other
words, joints Band C will undergo considerable
whence horizontal movement. The moment distribution
c = .85 and KBA = .85 IA/L method assumes that only joint rotation takes
place. To make this assumption true for this
All. 11 Frames with Unknown Joint Deflections Due structure we will add an imaginary support at
to Sidesway * joint C which will prevent sidesway of the frame
as illustrated in Fig.All.34. The end moments
In the example problems so far treated the in the frame will then be found by the moment
joints of the particular structure were assumed distribution process. Fig. All.34 shows the
to rotate without translation or With a definite results of this process. To explain, the solu-
amount of translatory movement. Translation of tion begins With computing the fixed end moment
the jOints may however, be produced by shorten- on member AB = WL 2/12 = (300 x 25 2)/12 = 15.63
ing and lengthening of the members due to axial thousands of foot Ibs. This value with the
loads and by lateral sway due to lack of diag- proper sign is written at the head of a column
onal shear members. The problem relative to of figures on member AB as shown in Fig.All.34.
the effect of joint translation due to axial Now considering Joint B, the unbalanced
stresses is treated in a later article. In joint moment of 15.63 is distributed as follows:-
this article only the effect of sidesway of To Member BA 15.63 (40/190) =- 3.27. Carry =-
rectangular frames on the frame bending moments
will be considered. over to Joint A 3.27/2 =-
1.63 =-
Fig. All.31 illustrates cases of frames To Member BC =- 15.63 (150/190) =- 12.36. Car-
where only rotation of joints takes place (neg- ry over to Joint C =- 12.36/2 =- 6.18
lecting axial deformation) whereas Fig.All.32
illustrates conditions in Which sidesway takes
place and the joints suffer translation as well
as rotation. 4 6'1' 70'
'1 6' r--
~T
B 1= 10, 500 C
I
K=r;=150
0
i
"-
0
'<t' g II 25'
'lIo
0
..,
0
;;-"":!:J
:.::
D
rm
Fig. All. 33
Symmetry of Structure Frame prevented
& Loading from sidesway by
extern I restraint
Fig. All.3l
~B--------"'''''8LRC
~: l~j~I~II:
Symmetry
of Loading
but Structure
;:I~I~I~II;
+ I
~
I
Gl
I
t')
1+
~
(O.OO)f
-12.36 b1.------+
+ 2.44
- 1. 93 b ..--.....
f (0. 00)
-6.18
b 1. 4.88
-0.96
~+ + + + +
_ B ~.. ~.,~...
-~~~~
Symmetry of Structure,
Loading Unsymetrlcal
Fig. All ...32
Unsymmetrical
111 1
+ 0.38.
- 0.30 b 3
+ 0.06.
- 0.04 b..
~
b .. +0.76
-0.15
b 3 +0. 12
-0.02
jjj j
There are several methods of determining
-11.75 b.. +0.02
-1. 53
Sit) 0.. . 0l~
OCO..-4oor:-
00000 .
the bending moments due to sidesway. Only one - .... ++++
method will be presented here and it can best D
be explained by the solution of example prob- Fig. All. 34
lems.
3517
11750
~ B .........
1530 horizontal shear reaction at the upper end of
L
:=...:.-.+ .II =-+ ~i) +--- - -Roc· 3606 lb. the vertical legs of the frame equal to 3606 lb.
89 C
11750~~B=3517 1~0
Bending Moments Due to 8idesway.
T
B HC=8tC Assume the structure sways sideways as
illustrated in Fig. All.36, but with no
rotation of the upper joints. It was proven
25" in Art. All.8 that the end moments produced by
30O#/in
1 r _A
7570
D'J~
....., 770
i
the lateral movement of one end of a beam whose
ends are fixed against rotation are equal to
6EId/L 2 where d is the lateral movement. In
Fig. All.36 the load P causes both vertical
members to suffer the
Fig. All. 35
same horizontal dis-
placement at their
upper ends, hence their
end moments due to this
Now consider Joint C:- displacement are pro- Fig. All. 36
The unbalanced moment is -6.18. To bal- portional to EI/L 2 •
ance we distribute to CB 6.18 x 150/190 = =
4.88. Carryover to B 2.44; to CD = 6.18 x = Since EI and L are
40/190 =
1.30. Carryover to D 0.65 = the same for each vertical
Sidesway Without
The balancing and carryover procedure is now member in our structure, Rotation of Joints
repeated for joints Band C, until the unbal- the fixed end moments will
anced moments become of negligible magnitude. be the same magnitude for
Fig.All.34 shows that 4 cycles have been car- each member. Therefore, for convenience we will
ried through. By keeping the ratio 150/190 set assume fixed end moments of 10,000 ft.lbs. are
produced by the sidesway. We now use the moment
on the slide rule the unbalanced moments at distribution process in permitting the upper
joints Band C are distributed and chased back joints to rotate as illustrated in Fig. All.37.
and forth as rapidly as one can write them The procedure is similar to that in Fig. All.34.
down. Since joints A and D are assumed fixed, For example the solution is started by consider-
they absorb moments but do not give out any.
ing joint B. The unbalanced moment is -10.
This is balanced by distributing 10 x 150/190 =
Fig. All.35 shows a free body diagram of 7.89 to BC and the remainder of 2.11 to BA.
each portion of the frame. The end moments The carryover moments are 7.89/2 3.95 to C =
are taken from the results in Fig. All.34.
Consider member AB as a free body. To find and 2.11/2 =
1.06 to A. Due to symmetry of
the horizontal reaction HB we take moments loading and structure, the distributing and
about point A. carryover moments at joint C will be same as
at joint B, hence it is needless work to show
calculations at these joints. The carryover
m A = 300x 25 x 12.5 + 11750 -17570 - 25HB = 0 moments from C to B will be identical to those
from B to C. Fig. All.37 shows the 5 cycles
hence HB =87930/25 =3517 lb. have been carried out to obtain the final end
moments as shown below the double lines.
Now consider member CD as a free body. + 7.89 b
To find HC take moments about D. + 3.95 -1/2 Due to symmetry
- 3.12 b of loading and
~MD = 1530 + 770 - 25Hc = 0, hence HC =89 lb. ·1.56 -1/2 structure it is not
+ 1. 23 b necessary to con-
We now place these horizontal forces on the .. 0.62 -- sider right hand
- 0.49 b portion as calcu-
top member BC as a free body as shown in the -0.25 -- lations will be
upper portion of Fig. All.35. The unknown ,. 0.20 b identical as shown
imaginary reaction RC at point C that was + 8.47 for left side.
added in Fig. All.34 to prevent sidesway is
= ~ 51~t:I~I~IE B C
also shown. To find RC take ~Fh 0
+~
II~
0
0
II~
0
frame. In other words, the frame will sway .... 0
0
sideways until bending of the frame develops a 0 0 " " ' .... 00«1
o co «I co co ""I"" ....
~...:ooooa)
I
+ +
A
I ... I I I D
All. 19
~
liquidating force P produced by the moments in
A
Fig. All.37 is 1416 lb. it is obvious the \ Pin Fig. All. 41
values in Fig. All.37 must be multiplied by a
factor equal to 3606/1416 2.545. Therefore, = SOLUTION:
the final bending moment values equal those of
Fig. AIl.34 plus 2.545 times those in Fig. Relative Stiffness Factors: -
All.37. Fig. AIl.39 shows the results and
Fig. All.40 the final moment diagram. KBA -- L31 3 x 24 3
24'
= KBC = 41
L
= =4 16x 16 = 4
K ~D
''\.;
=31L = 3 15
x 10 = 2
708 i t
- - -..B.--------------;;-=:-iC- P=1416 lb.
The distribution factors for each member at
708
8470 joints Band C, which equal K/~K are recorded in
~B=708 ---+ 8470 the small D on Fig. Al1.41a.
T
25"
HC=708
Fixed End Moments: -
For Member AB = PL/8 = 100 x 24/8 =300 in .lb.
Fig. All. 38
= PL/8 =100 x 16/8 = 200
1
For Member BC
A D As explained in example problem 1, the
9230 9230
moment distribution is carried out in two steps,
one for joint rotation only and the uther for
effect of sidesway or horizontal translation of
- 11. 75 - 1. 53 joints Band C. Fig. All.41a shows the moment
+ 21. 56 + 21. 56 distribution for no sidesway by placing an
+ 9.81 + 20.03 imaginary reaction He at joint C. The process
is started at joint A and the order of joint
B
",CO", c balancing is ABCBC. As soon as a joint is
Fig. All. 39 t-1t)CX) ll)IOO balanced the carryover moments are immediately
. COil'"
. . . . . Cl)
It)
"'''''1
"';"";0
""IN carried over before proceeding to the next
-+- I I + I , joint. When a joint is balanced a horizontal
line is drawn.
~~II~
F.E.M. - 200 200 F.E.M.
BJ, - 143 - - - 71.5
+ ,
- 42. 8 - - 85. 7 B J,
II II
I
II
D Bs 24.4 - ----r2.2
- 4.1 - - - 8. 1 B s
B.. ~ - ----r:t
-363.2 ---=...Q...1 B"
..
::!l 47.3
I:il
~ ~ ~ ~ ~ ~
~ ~~~ ~~~
~I~I~II~
41. 07 22.73
Fig. A1l.40
Example Problem 2. Bent with Unequal Length j 1j
Legs and Pinned Ends. 00011 0
Fig. All.41 shows a loaded unsymmetrical
frame. The final bending moments at Band C A Fig. All. 41a
will be determined.
All. 20 THE MOMENT DISTRIBUTION METHOD
<- :1:111:....
::;
.....4OC')lN
""IN
~ B
""IN
I i 111 '
10~
-47.3 + 15Hc = 0, HC = 3. 15
D
Placing these shear reactions on member BC
and writing equilibrium of horizontal forces A Fig. All. 41d
we find that RC =
65 - 3.15 = 61.85 lb. We
now must liquidate this reaction RC and permit To liquidate the reaction RC 61.85 it =
frame to sidesway. will therefore require 61.85/16.87 3.67 times =
the moments in Fig. All.41c. Therefore the
We will assume that the frame is fixed final bending moments equal those in Fig.
ended and that an unknown horizontal force P All.41a plus 3.67 times those in Fig. All.41c.
at C will deflect the frame sideways. The Fig. All.41e shows the results. The bending
fixed end moments for equal horizontal de- moment and shear diagram and the axial loads
flection of Band C will be proportional to now follow as a simple matter of statics.
EI/L& of the vertical member. -363.2 47.3
530.0 ~
I -- 24&
Fo r membe r AB , 1:2 24 -- • 0417 166:8 64IT
B
'"~~II":
F or memb er DC , 1::2 10&-- •
I = 15 OA A t::
'±""hJ
"'0'"
.., ..,I!'> '"....
I I
Fig. All.41d shows the shear reaction on Example Problem 3. Bending Moments in Truss
the vertical members at Band C. These forces Involving One Panel Without Diagonal Shear
reversed on the top member show an unbalanced Member.
force of 6.02 + 10.85 =
16.87 lb. Therefore
=
a force P 16.87 was necessary to produce the Frequently, in aircraft structures a truss
bending moments that resulted on the frame is used in which a diagonal member must be left
due to sidesway. out of one or more truss bays. The external
shear load on such bays must be carried by the
truss chord members in bending. Fig. All.42
shows a 3 bay truss with no diagonal member in
the center bay. The bending moments on the
truss members will be determined for the truss
All. 21
The relative moment of inertia values We will assume the truss center panel is
for each member are given in the circles on fixed at joints b, c, f, and g. The right end
the truss in Fig. All.42. of this assumed fixed ended truss will be given
an upward deflection. This deflection will
The distribution factor to each member cause fixed end moments in members bc and gf
at each joint is then computed and equals which are proportional to IlLS for each member.
K/~K. The values are recorded in thec=J Since L is the same for each member, the fixed
on each member in Fig. All.43. The stiffness end moments will be proportional to I of the
factor K is proportional to IlL for the member. member.
For example for joint c: -
1 7 Ibc =4,
+-=-
60 60
be eb cd de
100 F. E. M. F.E.M. 100
-13.6 -27.2
-28.5
-27.2 . -57
-13.6
-29
5.9 . -14.5
11.8
TO
9.8
---=1.4
4.9
-2.8
4.0
-2.8
~
1.4
4.1
-0.9 . -L 9
0.2
1.0 0.5 0.5 0.9 0.5
-4.4 -0.3 ~ '36.9 -19.4 -0.2
-24.9 45.7 -2.6
ah .10 bg cf de
6
1-:4 '-?~ -7.1 -14 bQ.
2.4
U
~O .@
-3.4
-0.8 fe
~4
2.0 '£?(
0.5
-0.5
~'O
0.4
~'<7
0''/ -3.6 -0.4 -7.0 0.7 <r '~
~'S
~
----0:0
ha
4.4 \9 ,;>
'0
-6.7
-0.4
-0.1
-11. 8
-12.9
1.0
0.2
~
-17.5
&'0'
0''/ ~'S
0'6'
~
3"":lf
h!! -0.9 ~ ,~ ed
1.2 ~
's 0 0.1 0'<7 '~./ 1.5
0.3
'~
~·o -0.1 0.3 '~ M
4.3 O'S -11. 7 -17.2 -0.3
'qy lhQ
0.0
.::...1.
2.0
gh .40 fg .58 .5
hg
.
75 F.E.M.
-19.7 . F.E.M. 75
-39.4 -6.8
ed
-3.4
-1~2 -24.4 -20.6 -10.3 0.5 --.hQ
10.3
-2.6
--.L...l
. 5.1
----.b..l
0.6
2.0
-2.7
0.4
~
-1. 4
0.9
0.6
0.0
0.2
0.3
0.0
--.Q.....l..
28.7 -5.5 -0-
-3.4 -----.lL....Q ~
-22.3 34.0 Fig. All. 43 -2.0
All. 22 THE MOMENT DISTRIBUTION METHOD
Therefore the fixed end moments on member to axial loads produces translation of the
gf will be 75 percent of those on member bc. joints of the structure. Since the angles be-
tween the members at a joint remain the same
Assume 100 as the fixed end moments on due to continuity, this translation of joints
bc. Then a 75 x 10 =
75 is the accompanying bends the members between joints. (2) The
fixed end moment on gf. bendirg moments on the members due to external
joint or lateral loads or those due to joint
We now remove the imaginary fixed supports rotation produce lateral deflection of the
on the center truss and let them rotate to members between joints. The member axial loads
equilibrium by the moment distribution process times these lateral deflections produce
as shown in detail in Fig. All.43. secondary moments. These secondary moments can
be handled by the general method of moment
The first step is to record the assumed distribution however the stiffness and carry
fixed end moments of 100 at each end of member over factors and the fixed end moments are not
bc and 75 at each end of member gf with due constant but become functions of the axial
regard for sign. The moment distribution pro- loads.
cess will be started at joint C. The unbal-
anced moment is 100 or -100 is necessary for Fig. All.45 shows a prismatic beam simply
static balance. Using the distribution fac- supported at A and fixed at B, with a moment
tors on joint C, we find -57 goes to cb, -29 MA applied at A and carrying an axial compres-
to cd and -14 to cf. Short lines are drawn sive load P. Sub. Figure a, band c show the
below each of these numbers. Fifty percent of 3 parts which make up the moment diagram on the
these balancing moments are carried over to beam. Without the axial load P the portion (c)
the far end of each of these members. This would be omitted.
process is repeated at each joint until the
remaining balancing moments are negli~ible.
p--+(M~__ ~ J__ B
In Fig. All.43 the order of joint balance was
Q Y
.?--~, -) p
cfdebghacfdebghacfdebg. If the student will I
I : MB
follow this order he should be able to check I
I I
the figures in Fig. All.43. I
'''' r
f
.'/>///////7/>,~
....• \a) MA~=:: :=! :: ... - - ...
I
I
I _
1. 8
All.13 Fixed End Jloaents, Stiffness and Carry
over Factors for Beaa Columns of Constant
Section * 1.
•
In deriving expressions for fixed end mom-
ents, stiffness and carryover factors the beam ··COIlpr. . .loa
I 1
K = L = 96 = .0104 1. 8
/ /'
From Fig. All.47 when L/j = 2.5, correction fac- //
tor for stiffness factor = .775 when far end is T_lolL, far n4 tlUll ~'~
/
fixed and .36 when far end is pinned. Hence,
1.
•
/
KBC = .0104 x .775 .00806 1
1.
• // /'
KeB = .0104 x .36 .00374
From Fig. All.46, when L/j = 2.5 the carryover 1. 8
/ ./.. '1'. . 1011.. far eD:l planed.
.._/V
/
V
1 I I / I I II
-
"C:~~~
_.lb./lu.
"Ad • • ·b.. .
17
-
,. I r----
-----" T
'"
/
v
~-
as
~- c-- 1\
.11.- -lIB- ~a
/ \ 1II.l.~2
15
/
/
-
\
C'''1
•
.
_+c
-~
L
-
--- "--
v'"
~~r-
•
V
~--'I'e»loll
\
•
V
/ 1\
\
a
----I'-----
r--- ao
--- r----
"---
1
0
"- ___ Compression
\
1\
•
6
-
'" I~ -"s
1"\
~-r-- 1\
• •
\ .. \
• ,-r---- 2
\ L/l
•
L/l •
Fixed~end moment coefficient. Fixed-end moment coefficient.
Uniform Load. Uniformly varying load.
Axial -compression.
Fig. All. 48 Fig. All. 49
Q I 1 1 I I I
50
_
'. c:~
~ II •.,lb./u·i
I
~)'. i·.·-~~oi
--l-J[~-2r
~~
I
Jr.... "'
ID ~ ~.--J
I
3r-~~
I
L/2
"t L/2 -+-+
~~+-
I
~
I
,
I
i
V
-t---
f--r---i 'ii af--- j-
i
f-
:'::A '" -liB· ~ !
I I /
TT
~-
io~
--
50 -,"- -~~
T. .
/
I -.-, \,V
/ 1
V V
2
V 0
1/
V 1/
:IS
0. V
V /
V
// 6
----r---- V
0
_v v
7
KCoepr...l0D
./ ~
V ti '\
/' 1\
~, ~ .0. \
---~ "
~ I-- --- \
18
0
til• • •
L/l
Fixed-end moment coefficient. Fixed-end moment coefficient.
Uniformly vary.ing load. Concentrated load at mid-span.
Axial tension.
Fig·. All. 50 Fig. All. 51
3. <
3. 3
f----- I---
.~rn
Vi.
.JI Ib./ln.
I
Fig. All. 55
! I
0
.
~J'
,
TTITT'
.~.
h
· · I +-171--
-fp
-;;; -'V<,·
I----- _,lb./in.
•
Fig. All. 54
0
+-----_
r. t-
I
~a(~la[6-B(~)+3(~)a]0i
L V ~ ~
'i' 0
>0
3. a ,< 1/ ---
I..---- f--" - <••
/ V
<.0
, "'J
a
a.
o
co
....
a
....
N
....
"
....
(])
....
co
r"
o
I I -
1.. / r!G'
.3
Il::I
II ------- . . _ ..tIo"(A)3[<\..) _ .(..)3] 0B / 1/ /~.
tf'
......>
13 L L L
LL/.
1/ V v ~ 3•• J=
L ./ <.0 ,a
V I..---- / /V V C11
~+-+----1
~
-- "" '"
-
./ 3.0
';) c.
Os
V / 'V J.----- t'!l
... g j V
--- <'.5
~. 77/
~
"'1~ ~l>:ol
~
11
~ 1/V
--
1. < V j.---
--V e----- f.- I--- I I 1\ \ 1\ \ \ "", _ '----'N 0'
II \ '\.
1:"10'
~
oN
\1:/
_
1. 3 - '.0
~ ~ ..::- r-----
1.0
- -
0-
J,
0'
(])
/
/ Ii \ \ '\. ti'
e------
-- i-----
3 ••
3.0
~ .I I 1
• \...-
------- - r-----
g
!
~t--- --r---.. r---..
'------ I'---- ~ ~- I I
II
/
I
I \ ~- I\
\
1\ \
•
.1 .3 .. - •< ..
7
.1 .2
./L
.3 .<
t--
"~I
• r.~--.01
.,
(0/L)(0/L)3 .... (./L)3(0/L) .p. CN t-' II
..... VJ If:o ("
~ ('1 l\)
-f--- .0 ,
;, ;.. . -. • . .' . .
- ~ ~ 9 j.. ;.. ;.. ;.. rt 00000 OOC5lp _
Pl I~tg' i -H
rt
CD
• I
I'":;< 1'1
'1:J
0-
f-'
l\-\+- g aW- I I I I I Y I II I· I I
o ..... '"
1
~tmi1lT1I111
~
"'J
4Q' . ooh
......> "
~~
~
."1 17
C11
c.> ~~
~~
-t.
....
~ . ~ § ~
o
~ ~
o
~'r'i1 l> ~ l:
~.1l f'al!)a[ot- 3(!)'
~
~,
~ 2 ~
.l.(B)a
~
r. _.. + 3(B)31
~ g .
o g ...>...
~ 12L II -L LJ
""
C11
All. 26 THE MOMENT DISTRIBUTION METHOD
Jr---rI --t,--- I !,
1. of---- I-~
~I
+-----c--,
! :
/
I
1.
• /
1/
1. 4
/
1/
1. s
J. .. Coapre••ioa
t. a 1/
/
:" v
I
1. 1
o/,v
Fig. All. 59
1. 0
-----r----
V
~
Fig. All.59 shows the total beam axial loads in
the various portions of the beam by adding the
•
"""'0 .ralWloll
values shown in Fig. All.58. The outer span AB
due to the axial loads induced by drag wires of
0
"" "-...
~
drag truss is subjected to a varying load. As is
customary in design of such wing beams the axial
load in span AB will be taken as equal to the
average load or (-9571-8565-7559)/3=-8565#.
a L/l
Culumn di'Stribution factor.
S
2~
""'"
- a.
The beam bending moments at the support
points will be determined by the moment distribu-
tion method.
Fig. All. 56 Calculation of Factors:-
:: 2.12
, 00~7~~~ x .33~/2 :: 18.82, L/j =1:. 82
0 e
ted, the bending moment, at support A and A'
would be 19480, thus the axial influence in- From Fig. All.47, stiffness factor = .82 x .0083
creases the moment at A approximately 7.5 per- = .00681
cent. From Fig. All.46 C.O •. Factor = .66
w·16 w = 32 w·32 w·16
w =31
Fig.A1l.60
.:..,-r T T T T T T Il--
C' B' A' A B
~25"~ ![I~
,,~~1 ~
Ll~ A
'"
. " .L
~""'..g:>
T
I •
~/ Fig.
o '
o ~
~ ~
C:>
C'l ,
Al1.63
: ~~ co ~ "--C
J
~ I'~ ....~,,:::~<f, -72000
~" .... ~
000
arl::0Fig.
L"-
All. 61
Effect on End Moments Due to Translation of
Joint C Due to Axial Loads.
4"r The movement of joint C normal to each
18000 member will be calCUlated by Virtual work.
Solution:- (Reference Chapter A7). Fig. All.64 shows the
Virtual loading of 1# normal to each member at C.
The magnitude of the axial loads in the The Table shows the calculation of the normal de-
members is influenced by the unknown restraining flections at C.
moments at A and B. To obtain a close approxi-
mation of the axial loads, the end moments in Table
the two members will be determined Without con-
SUaL SUb L
sideration of axial loads. Thus the external lIem. L/AE S Ua AE Ub AE
joint moment of 4 x 18000 = 72000 in. lb. at C
is distributed between the two members as shown AC .0000049 10400 1.89 .0962 1.6 .0816
BC .00000244 -27150 -1.60 .1060 -1.89 .1250
in Fig. All.62. With the member end moments .202 .207
known the axial loads and shear reactions at A
and B can be found by statics. The resulting Thus the deflection of joint C normal to
axial loads are BC equals .202" in the direction assumed for
PAC = 7360# and PBC = - 24240# and the the unit load, and the deflection of C normal to
shear reactions, SA = 198# and BB = 2332#. AC = .207".
With the axial loads known the modifted
beam factors can be determined.
All. 28 THE MOMENT DISTRIBUTION METHOD
in Chapter A9 provides a rapid and simple diagram. Thus to find the distance from Bx
method for determining these values for curved to this centroid we take moments of the M/EI
members. The use of the elastic center method diagram about B and divide by 0 the total area
in determining the value of stiffness and of the M/EI diagram. Thus
carryover factors will now be explained.
Before considering a curved member a Thus the centroid of the total moment weight 0
which equals one lies at point A or a distance
straight member of constant EI will be con-
sidered. Fig. All.66 shows a beam freely L to left of B.
In using the elastic center method to de-
termine the stiffness and carry-over factor for
Fig. All. 66 ~ a straight beam, we assume that the bending
_ /-_d,= I Rad.
moment curve due to a moment applied at A is of
MAJs " * ' ...rdeflectedbeam such magnitude as to turn the end A through an
~A EI constant' {~ MB angle of 1 radian. As shown above, the moment
L ~ weight 0 for this loading is unity or 1 and its
centroid location is at A. Then by the elastic
Fig. All. 67
=
center method we find the moment required to
__ 4EI~2LEI=MB turn end A back to zero rotation. The value of
MA - --r:- - this moment at A will then equal the stiffness
factor of the beam AB. In order to simplify
the equations for the redundant forces the
Fig. All. 68 M/EI Diagram elastic center method refers them to the elastic
center. From Chapter A9 the equations for the
4
-L redundant forces at the elastic center for a
structure symmetrical about one axis are: -
_ -2: 0s
Mo - 2: dS/EI - - - - - - - - - - - (1)
Yo _-2: 0s X (2)
supported at end A and fixed at end B. A - Iy
moment MA is applied at end A of such magni-
tUde as to turn end A through a unit angle of
one radian as illustrated in Fig. All.66. By 2: 0s Y - - - - - - - - - - - - - (3)
definition, the necessary moment MA to cause Ix
this unit rotation at A is referred to as the
stiffness of the beam AB. In Art. All.4 it Fig. All.69 shows beam of Fig. All.66 re-
was shown that this required moment was equal placed by a beam with the reaction at end A
in magnitude to -4EI/L. It was also proved replaced by a rigid bracket terminating at
that this moment at A produced a moment at point (0) the elastic center of the beam, which
the fixed end B of 2EI/L or a moment of one due to symmetry of the beam lies at the mid-
half the magnitude and of opposite sign to point of the beam. The elastic moment loading
that at A. Fig. All.67 shows the bending is 0s = 1 and its location is at A as shown in
moment diagram which causes one radian rota- Fig. All. 69.
tion of end A. Fig. All.68 shows the M/EI
diagram, which equals the moment diagram
divided by EI which has been assumed constant.
EI/L
s " I B~
A Y
Fig. All. 70 The total area of the M/EI curve for the
curve in Fig. All.72 if calculated would equal
one or unity as explained in detail for a
The bending moment at end A equals straight member. The centroid of this M/EI
diagram would if calculated fall at point A.
_ EI _ 6E! (.1::.-) = _ 4EI Thus in Fig. All.73 we apply a unit 0s load at
L L 2 L A and find the redundant force at O. Due to
symmetry of structure about a vertical or y
By definition the stiffness factor is the
moment at A which is required to turn end A axis the elastic center lies on this symmetrical
axis. The vertical distance from base line AB
through an angle of 1 radian. Thus 4EI/L
is the stiffness factor and this result
to elastic center equals y .6366r. (See =
page A3.4 of Chapter A3).
checks the value as previously derived in
Art. Al1.4. The elastic moments of inertia Ix and I y
can be calculated or taken from reference
The bending moment at B in Fig. All.70 sources such as the table on page A3.4.
equals,
Whence,
Ix = .2978r 3
t, but t -- ~
EI
Hence the carry-over factor from A to B
is .5 and the carry-over moment is of opposite .2978r 3
Hence Ix· EI
sign to that of the moment at A.
All. 18 Stiffness and Carry-Over Factors For Curved
Members.
Fig. All.71 shows a curved member, namely, Solving the equations for the redundants
a half circular arc of constant EI cross- at (0), remembering 0 s =
1 and located at point
section. The end B is fixed and the end A is A.
freely supported. A moment MA is applied at
A of such magnitude as to cause a rotation at
Mo - -
.z 0s =.=..i.U.- = - EI
A of 1 radian as illustrated in the Fig. .z ds!EI nr nr
Fig. All.72 shows the general shape of the EI
bending moment curve which is statically in-
determinate. In Fig. All.73 the support at A Y = .z0s x _ - (1)( -r) _ 2EI
o - Iy - nr 3 - nr 2
2EI
end A. The ratio of the bending moment at The method of how to handle this remaining
point B to that redundant force can best be explained by pre-
at point A will senting some example problem solutions.
give the carry-
over factor. -r~.
0
.636,6 r __ .. ' i2EI
2. 4 EI/r
2
All. 19 Example Problems. Continuous Structures
Involving Curved Members.
-'- "'- ~
Bending TJ'r B
Example Problem 1
moment at A, B
Fig. All. 74
Fig. All.76 shows I =1
M - -EI EI 2EI a frame consisting of
A - nr 2.14--
r2
(0.6366r) - (--2)r
nr both straight and
= -0.31SEI 1.36EI _ 0 636
r .
R
r
= 2 . 314- Rr curved members. Al-
r though simplified L=60"
C
relative to shape, I =2
Bending moment at B, this frame is somewhat
representative of a
M _ -0.31SEI 1.36EI + 0.636EI fuselage frame with two L=80" L=80"
"B - 1 . 04-2 Rr 1=1 1=1
r r r cross members, one be-
tween A and C to support
Therefore the stiffness factor for a half- installations above
circular arch of constant EI is 2.314- EI/r. cabin ceiling and the
other between F and D w = 50#/in.
The carry-over factor equals the ratio of to support the cabin
MB to MA or (-1.04-2 EI/r)/(-2.314- EI/r) = floor loads. The frame F f-'-........"""'---'''--''--''-'........."'''-"'--'''---j D
0.452. It should be noticed that the carry- supporting forces are
over moment has the same sign as the applied assumed provided by
moment at A as compared to the opposite sign the fuselage skin as 1= 2
for straight members. In other words, there shown by the arrows
are two points of inflection in the elastic on the side members. E
curve for the curved arch as compared to one Eccentricity of these
for the straight member. skin supporting forces Fig. All. 76
relative to neutral
FIXED END MOMENTS axis of frame member is neglected in this simpli-
fied example problem, since the main purpose of
The fixed end moments on a curved member this example problem is to illustrate the appli-
for any external loading can be determined cation of the moment distribution method to
quite rapidly by the elastic center method as solving continuous structures involving curved
illustrated in Chapter A9 and thus the ex- members.
planation will not be repeated here.
SOLUTION
The student should realize or understand
that when the end moments on a straight member Due to symmetry of structure and loading,
in a continuous structure are found from the no translation of the frame joints takes place
moment distribution process, the remaining due to frame sidesway.
end forces are statically determinate, whereas
for a curved member in a continuous structure, The frame cross members AC and FD prevent
knowing the end moments does not make the horizontal movement of joints A, C, F, and D
curved member statically determinate, since due to bending of the two arches. Any horizontal
we have six unknowns at the two supports as movement of these joints due to axial deforma-
illustrated in Fig. All.75 and only 3 equations tion is usually of minor importance relative to
of static equili- causing bending of frame members. Therefore it
brium. Even when can be assumed that the frame joints suffer
the end moments Fig. All. 75 rotation only and therefore the moment distri-
are determined bution method is directly applicable.
from the moment
distribution Calculation of stiffness (K) values for
process there each member of frame: -
still remains
one unknown, Upper curved member ABC: -
namely the hori-
zontal reaction _ _ EI
at one of the KABC - K CBA - 2.314 r' This value was
beCLll ends. derived in the previous Art. All.lS.
All. 32 THE MOMENT DISTRIBUTION METHOD
JOINT A.
the c=J at each joint. The process is started inertia about x and y axes through the elastic
by placing the fixed end moments with due re- center.
gard to sign at the ends of members AC and FD,
namely, -3000 at AC, 3000 at CA, -15000 at FD
and 15000 at DF. We now unlock joint A and
find an unbalanced moment of -3000 which means ~
d=4.6~
a plus 3000 is needed for static balance. • - ------.; I
Joint A is therefore balanced by distributing _ I E.C. \ I 19.1"
.512 x 3000 = 1536 to AC, .296 x 3000 = 880 to Y
-----.:L- ':- ---, I = 2 ----! C ~
L_-.,
ABC, and .192 x 3000 = 576 to AF. Short hori- A r30"------l Fig. All. 78
zontal lines are then drawn under each of these
distributed values to indicate that these are
balancing moments. Carry-over moments are
immediately taken care of by carrying over to
joint F, .5 x 576 = 288. From A to C the
- = 19.1
Y
x n (30/1)
n x 30 60
= 14.49"
carry-over moment would be .5 x 1536 = 768 and 1 + 2
therefore the carry-over from C to A would be
.5(-1536)= -768 which is recorded at A as (NOTE:- 19.1 equals distance from line AC to
shown. For the arch member ABC, the carry-over centroid of arch member ABC.)
moment from A to C would be -0.452 x 880 =
-401 (not shown) and therefore from C to A = Calculation of moment of inertia Ix: -
-0.452 x (-880) = 401 as shown at joint A in
the figure for arch member CBA. Joint C in Member ABC
the figure has been balanced once for the _ .3r" nrd 2
purpose of helping the student understand the Ix - -1- + - 1 -
sign of the carry-over moments which flow to
the left side from the right side of the frame. = .3 x130" x-
+ n
-l 30x 4.612 = 10100
After balancing joint A and taking care 60
of the carry-over moments, we imagine A as Member AC =2"x 14.49 2 = 6310
fixed again and proceed to joint F where we
find an unbalanced moment of -15000 + 288 = x
total I = 16410
-14712, thus plus 14712 is necessary for Total elastic weight of structure equals
balancing. The balancing distribution is .255
x 14712 = 3750 to FED, .663 x 14712 = 9750 to Z dS/I = (n x 30/1) + 60/2 = 124.24.
FD and .082 x 14712 = 1212 to FA. The carry-
The next step in the solution is to draw
over moments are .5 x 1212 = 606 to A, .452 x the bending moment curves on this frame portion
3750 = 1695 from D to F by way of the arch due to the given load on member AC and the end
member and -.5 x 9750 = -4875 from D on member
FD. We now go back to joint A which has been moments as found by the moment distribution
unbalanced by the carry-over moments and repeat process in Fig. All.77. It is composed of
the balancing and carrying-over cycle. In the three parts labeled (1) to f3) in Fig. All.79.
complete solution as given in Figure All.77 Portions (1) and (2) are due to the end moments,
each joint A and F was balanced five times. and portion (3) due to the distributed lateral
The final bending moments at the ends of the load on member AC.
members at each joint are shown below the
double short lines.
The arch member ABC has 3 unknown forces
at each end A and C or a total of 6 unknowns.
With 3 equations of static equilibrium avail-
able plus the known values of the end moments
at A and C as found from the moment distribu-.
tion process, the arch member is still static-
ally indeterminate to one degree. Thus the Fig. All. 79
horizontal reaction at A or C as provided by
the axial load in member AC must be found be-
fore the bending moments on arch ABC can be The next step is to find the 08 (area of
calculated. M/I curve) for each portion and its centroid
location.
The first step in this problem is to find
the elastic center of the frame portion com-
posed of members ABC and AC, as shown in Fig.
All.78, and then find the elastic moments of
All. 34 THE MOMENT DISTRIBUTION METHOD
0s~ =1094 x n x 30/1 = 103000 46 lb. The member AC also suffers an axial load
due to the shear reactions at the top of members
0 S2 = 60 x 2353/2 = 70590 FA and DC. Fig. All.81 shows free bodies of the
side member FA and DC with the end moments as
0s " = (-.667 x 60 x 4500)/2 .. -90000 found in Fig.All.77.
The shear reaction 3...:.8..:.....1"---- 38:2-
Z0s = 83590 at A and C can be 1259 A C 1259
found by taking (,iA-R
Fig. All.80 shows the 0 s values concen- R- "C
moments about lower A C
trated at their centroid locations and referred ends. Thus for FD, 80"
to the x and yaxes through the elastic center. RA = (1259 + 1791)/80
.. 38.1 lb. Likewise F D
y
RC =
38.1 lb. These ~
1791
~.A
1791
reactions react on
B Fig. All. 81
cross member AC in the
• OSl. = 103000 opposite directions
-----.~-~~oX-O-\---Tt- X thus giving a compression load of 38.1 lb. in
<a = 70590 14.49 member AC, which must be added to the tension
/SL c~ load of 46 lb. from the arch reaction to obtain
A
"n Fig. All. 80 the final load in the cross-member.
~,,= -90000
16410 = 46 lb. E
Iy Fig.A1l.81 Fig. All.82
Yo = -Z I 0s x = 0 because x = zero.
y y = 19.1 x n(30/2)
n x -30 + -60
= 900
57.1
= 15.76" Z dIs = 57.1
The bending moment at any point on ABC 2 6
or AC equals that due to Mo and Xo plus the
moments in Fig. All.79. Ix = .3x30" + 15nx3.34 2 = 4578
For example,
2 (60/6)(15.76)2 = 2478
Ix = 7056
At point A on member ABC,
From Fig. All.82,
MA = 1094 - 670 + 46 x 14.49 = 1091 (should =-
0s~ = -8753 x 60/6 87530
be 1094 since moment as found in Fig. All.77
is correct one. Small error due to slide rule 0S2 = .667 x 22500 x 60/6 = 150000
accuracy.
08 " =- 6962n x (30/2) = -328000
At point B on member ABC: -
Z 0s = -265500
MB = 1094-670-46x15.51 -289 in. lb.
Solving for Redundant forces Mo and Xo (See
The horizontal reaction at A will not Fig. Al1.83)
produce any bending on member AC, thus the
values in Fig. All.79 are the true moments. M - - Z 0 8 = - (-265500)
o - Z ds!I 57.1
= 4650 in. lb.
Fig. All. 83
1000 B 1000
Bending moment at F on member FED, 1 ~ 1. 'j
MF = -6962+4650-294x15.76 = -6962 in.lb.
ME = -6962+4650+ 294x 15.76 = 2340 in.lb.
A C
The axial load in member FD 294 lb. = w = 10 lb/in.
compression plus 38.1 Ibs. tension due to shear
reaction from side members at points F and D 1=1 1=1
or a resultant load of 255.9 lb. compression. Fig. All. 85 L-60 L=60
Fig. All.84 shows the final bending moment
diagram on each member of the frame. 1000 lb.
30'
B
E
1= 3, L = 60
Member ABC
SOLUTION: -
E
3000
Curved member ABC. total elastic weight of member and its elastic
moments of inertia about axes through the
The fixed end moments on this curved elastic center.
member due to the external loads will be de-
termined by the elastic center method. The TOTAL Eh~STIC WEIGHT = 2: ds/I -- 1.5
nr _ n x 30
- 1.5
assumed static frame condition will be an arch
pinned at A and supported on rollers at B. = 62.83
(See Fig. All.S6)
Distance y from line AC to elastic center of
Fig. All.S7 arch ABC eQuals 0.6366r = .6366 x 30 = 19.1 in.
shows the general 1000 =P
1= 1.
B 1000 =P Ix =0.2978r 3 /1 =
2978 x 30 3 /1.5 5350 =
shape of the static
moment.curve. For -JlD=120~
the frame portion "
between the re- <><= 300~ /'
/'
lD sa " 167300
A "f" - ~-
actions and the load
points, the bending Fig. All. 86
moment eQuation is
1'1=P(r-r cos a).
(2)
Y=24.8"
\IX
19.1
~
(I' )
For the beam
portion between the (1)
C
two 1000 lb. loads, Fig. All. 87
the bending moment Static Moment Fig. All. 88
is constant and Curve.
eQuals, Calculation of redundant forces at elastic cen-
1'1=P(r -r cos 30 0 ) ter.
y = r(l-cos a-~) = 30(1- .867-0.5/2) = 1'1A =I'1c =1'10 + XoY = - 3122 + 129.3 (- 19.1)
= - 652 in. lb.
a-sina .524-.50
10 in.
1'1oment Distribution Process
2
¢8 2 = Pr e (1 - cos a). e = 1200 , a - 30 0 •
I Having determined the fixed end moments,
distribution and carry-over factors, the moment
¢ = 1000 x 30 2 X 2.1 (1 _ .867) = 167300 distribution can now be carried out. Fig.
8 2 1.5 All.89 shows the solution. The first cycle will
be explained. Starting at joint (A) the un-
Vertical distance y from line AC to centroid of balanced moment is -3000 - 652 = - 3652. The
¢S2 eQuals, joint is balanced by distributing .432 x 3652
= 1543 to AC; .364 x 1543 =
1333 to ABC and
- 2r sin e/2 .:. : .2.:. : .x: :. .3: :. .0=--=x--.:0,•.-" :. : .8-,-6-,--7 = 24.8 in. .212 Y 3652 = 776 to AD. The carry-over moment
y = e = 2.1 =
from C to A .5(-1543) =
-772; from C on
member CBA to A =-
.452(-1332) = 601; and from
Fig. All.88 shows the ¢s values and their lo- =
A to D .5 x 776 =
388. Now proceeding to
cation with respect to x and y axes through joint D the unbalanced moment is -7500 + 388 =
the arch elastic center. -7112. The joint is balanced by distributing
.75 x 7112 =
5340 to DE and the remainder 25
The elastic center method reQuires the per cent =
1772 to DA.
All. 37
Static Mom.
Curve
(1) (1)
ON
""'
'"
Co
AC 3000
-3000 F. E. M. 2513
2513
~~~i/
-302/
151
-1~~/
-64 / ' Fig. All. 90 -1/8 wL~ =4500
32/
-~~/ 0s = 2513 X 60/2 = 75390
.25 -=..9.- e
-2513
D .75 I - - - - - - - - - J E
L:0 s = 218290
DE 7500
Fig. All. 89 -7500 Calculation of elastic center location and
5340/
-2670 Ix·
2060/
-1030
802/
-401
Distance y from line AC to elastic center =
~/
-156
19.1xnx30/1.5 = 1202 = 12 97"
nx30 + 60 92.83 •
-lli/
-60 1.5 2
46
-3135
TOTAL Eh~STIC WEIGHT = 92.83.
Ix = .2978x30
1.5
3
+ nx30 (6.13)" +..§Qx 12.97"
1.5 2
The carry-over moments are: -
= 12713
From E to D = .5 x (-5340) = -2670
Fig. All.91 shows the elastic center loca-
To A from D = .5 (1772) =886 tion and the 0 s values together with their
centroid locations.
The first cycle is now completed. Five more
cycles are carried out in Fig. All.89 in order
to obtain reasonable accuracy of results. The
final end moments are listed below the double
short lines. [B ~S2 = 167300
On arch member ABC the end moments of 584 0;0 S 3 =30
are correct. However before the end moments 3---,- i
at any other point on the arch can be found 6.13 M
the horizontal reaction on the arch at A must _~_-'-:-::~---'L.-~ Xo x
be determined. This reaction will be de- =-T---Af~ - - do. = 28800
termined by the elastic center method. I ,oJSi .-L
I
A ~ C
Fig. All.90 shows the bending moment ~s = -90000 + 75390 = - 14610
curves for members ABC and AC as made up of Fig. All. 91
5 parts labeled 1 to 5.
Solving for redundant forces at elastic center.
Calculation of 0 s values which equal area
of each moment curve divided by I of member. No = L: L: ds/I
0s = -218290
92.83
= -2356
20s 1. = 28800
Xo=~= 167300 x 11.83 + 36800 x 6.13 +
03 " = 167300 Ix 12713
28800 (-2.97) - 14610 (-12.97)= 181 lb.
0 S3 = 584 x n x 30/1.5 = 36800 12713
0s 4 =- (.667 x 4500 x 60/2) = -90000
All. 38 THE MOMENT DISTRIBUTION METHOD
The bending moment at any point on member (2) Find the bending moments at all joints
AC or ABC equals the values in Fig. All.90 and support points of the loaded structures
plus those due to Mo and Xo in Fig. All.91. in Fig~. 7 to 10.
Thus at point A on member ABC, W= 30lb/in.
~
MABC = 584-2356+181x12.97 = 584 in. lb. L- rf ---'
T II
which is the same as found by the moment distri- 20" 400 I =6 ~400
bution process. ! =4 1=4 10"
--l-
f--8"+-16,,=t
t;t=
1100
.. Ik'
~24"---l
w=10lb/in.
k
wing beam, carrying a uniform air lo~d of 20
lb./in. Determine the beam bending moments at
strut points A and B. Take
Fig. 1 Fig. 2
IAB=17in., 1M' = 20 in., and E = 1.3 x lOS in. 2
Ifo . 200
b
W=r'f /
m
. r"~ Neglect effect of support deflection due to
~=10 =i=!i6 --f strut axial deformation.
Fig. 4
ti°O 1-12j
100 100
t12-1
;eo ~
,ft=l/A1
r-- ------+--
14"
1=2
36"
Fig. 5
--+-- 1=2
16" ---l
100 200 w=12lb/in. (5) Figs. 13 to 15 are loaded structures that
•
I ~ ~IILLi suffer joint translation. Determine bending
~ I- 1 ~ 1= 1.5 P, 1= 2 I - 2 moment diagram.
1-6" +-12''---+-12''-----+1-16''-+ 12"---1 100
500 ~5 10'I.j
Fig. 6 l R = l T.. 1= - 0
.... 12 ~ ... 200
24" 1=2 II -.l.... 1=4
1 .... 6" ..!.I ~II
* Also see Chapter C2, Parts F, L, M and Q.
-i.
f.-20'Y
T
Fig. 14
'--20"
I·
---k--
·1·
24"
I I f-
Fig. 13 Fig. 15
CHAPTER Al2
The problem is to determine the relation- distance ~ which causes end moments M'~ and M":, •
ships between the displacements of the end In deriving the slope deflection equations each
supports of a beam and the resulting end of the three beam deflections will be considered
moments on the beam. separately and the results are then added to
give the final equations. Fig. A12.2 shows Fig.
Fig. A12.1a shows a beam restrained at c repeated.
ends 1 and 2. It is assumed unloaded and of
constant cross-section or moment of inertia.
This beam is now displaced as shown in Fig. b, Fig.A12.2 L
namely, that the ends are rotated through the Os= 0
angles 8 4 and 8 s plus a vertical displacement =..---;.----i2
d 4 and d s of its ends from its original posi- t
tion, which produces a relative deflection ~ Fig. A12.3 Mi
of its two ends. The angle 0 representing the
swing of the member equals ~/L.
A12.1
A12.2 SPECIAL METHODS - SLOPE DEFLECTION METHOD
assumed also positive, as the algebraic solu- Fig. A12.6 shows the moment diagram for an
tion will determine the true sign of M~. applied bending moment -M~ which rotates end (2)
through an angle 8 2 when other end (1) is fixed
Two moment area theorems will be used in (See Fig. A12.5). In a similar manner as
deriving the slope-deflection equations, described before,
namely (I) that the change in slope of the
beam elastic curve between two points on the e = (-M~L/2)
EI +
(-M~L/2) =- (Wi + M~)L
EI 2EI - (5)
beam is equal in magnitude to the area of the 2
The deflection of end (1) away from a Then from equation (6)
tangent at (2) equals zero, thus we take
moments of the moment diagram about (1) and Mil _ - 4Ele g - - (8)
equate to zero. 112 - L
Mk =- 2Ele 1
L
_ - - - - - - - - - - __ (4)
Fig. A12.10
The change in slope of the elastic curve
between the ends equal zero, thus by the moment
L
Fig.A12.5 area theorem the area of M/EI diagram equals
zero.
(M'!' + Mm2 )L
Hence, -"-=c=--=::c-"----L:.=-
2EI
=0
whence MT =-M':- - - - - - - - (9)
Fig.A12.6
From the deflection theorem,
M~'
=_+--=.6E=.I::..:6.::..:
L 2 ------------ (12) (18a)
111-" = 28" - 300 - - - - - - - - - - (a) Fig. A12.12 shows a loaded 3 span beam
fixed at points (1) and (4). This fixity at
11"-1 = 2K (28"+8 1 )+11F,, (1) and (4) causes 8 1 and 8~ to be zero.
11"-1 = 48" + 300 - - - - - - - - - - - (b) I"lF 1_" =- PL/S = 100 x 24/S = - 300,
Solving equations (g) and (h) for 8 2 and 8 3 or free to rotate we use equation (19) in writ-
gives, ing equations for 11,,-1. and 11 3- 4 •
82 = 26.15, 83 =- 19.9
SUbstituting in equations (a) to (f) inclusive MS - 1 = 3xlx8 s +250-0.5 (-250) =
to find final end moments.
38" + 375 - - - - - - - - - - - (a)
MJ.-s = 28 2 - 300 = 2 x 26.15 - 300 =
-247.7 in.lb.
11 3- 4 = 3xlx83 -500-0.5 (500) = 38 3
11 2-1. =4 x 26.15 + 300 = 404.6
= 482 + 300 - 750 - - - - - - - (b)
11"_3 = S8" +463 -534 = Sx26.15+4 (-19.9) Using equations lSa and lSb and substituting,
- 534 =- 404.6
11"-3 = 2K(28" + 83 ) + 11F
11 3- 2 = S63 + 46" + 534 = S (-19.9) + 4 x 26.15 " 3
+ 534 = 479.4 11"-3 = 2xO.5(28 2 +8 3 )+0 = 28,,+ 83 - - (c)
100 lb.
Solving (e) and (f) for 8" and 8 3 gives,
r- 10tl
(1) ~----=---'::=---7ic----;-L-=-----=-:20:;---'7i---;----'----;:;::-----7i\
6" =- 109.4, 83 = 172
I = 10 The end moments at (2) and (3) can now be found
K = 0.5 from equation (a) to (d) inclusive.
Fig. A12.13
11,,-1. =36 2 +375 = 3 (-109.4) +375 = 46.S
in. lb.
_ L"
I1.. _~ - 60 (5u + 3v) ~-4 = 2K(26 + 8 -30) +I1F
3 4 (8 3-4 4 = 0)
whence, 667766" + 166946 2 + 572 = 0 - - - - - (5) MF~_2 = (100x8x12 2 )/20 2 + (100x12x8 2)/
Solving equations (1) and (5) for 8 2 and 20 2 =- 480
8" we obtain 8 2 = 0.00439 and 8" =- 0.00967.
MF 2_1. = 480
SUbstituting in equations (2), (3) and (4)
gives the moments at these points. MF ~_" = 100 x 24/8 = 300. 300
L = 24 L = 24 Which gives,
100 - - . I = 40 I = 401--100
K = 1. 67 K = 1. 67 28~-480+6.678~+3.338,,+300 = 0
Subt. in equation (a) Due to the fixity at joints (1), (3) and
(5), 8~-s = 8 3 - 4 8~-6 = =
O. Therefore, the
1'1,.-s = 2x26.3-480 =-428 in.lb. unknowns are 8s-~, 8 4 - 3 , 8 6 - 6 and ~.
T Tr
L 1- 8
194
Example Problem 1
1 Fig. A12.18
moment curve will be determined.
SOLUTION: -
The relative moment of inertia of each
A12.9
Relative values
of 0 due to sidesway
48#
1-9·"
I = 10
L
K
=15"
= .667
I = 10
K =1
3
ITn
M.._ ..
M3 - ..
= 6.6678.. + 3.3336.. + 144 -
- 2x1(0+8.. -3xO.10)+0
- - - - (d)
of frame: -
The angles 0
are proportional to
1~~ Fig. A12.20
M3 _.. = 28.. - 0.60 - - - - - - - - - - (e)
Hence, SUbstituting: -
MF
l.-"
= - (48 X 6 2 X 9)/15" =- 69.12 in. lb. Writing the bent equation (See Eq. 20).
1J
~~H"
TrH..
Member 3-4 has no lateral loads and thus fixed 48 10
end moments are zero.
9
~~H3
The slope-deflection equations are: -
1 LJ'-H:I. M 3 _ ..
M.._l. = 2K(28.. + 8:1. -30) +MF.. (18) For free body of member 1-2 take moments
about (2) and equate to zero.
Writing above equations for each member
and noting that 81. and 63 are zero because M..-l. + Ml._.. - 6 x 48 + l5H", =0
frame is fixed at support points, gives,
hence
,
H
:I.
= 288 - (M2 -1. +Ml._.. )
15
Ml.-.. = 2 x 0.667(0 + 8.. - 3 x .06670) - 69.12
SUbt. values of M.. _:I. and Ml._.. in the equation,
Ml.-.. = 1.3336.. - 0.26670 - 69.12- - - - (a)
whence, R3 =- (M
3 to+M . 3
)
w = 12 Ib/in.
1illOllll
Substituting values of M3 _ .. and M.. _3 in above 3
equations, gives, 10"
t
R3 =- 0.68 .. + 0.120 - - - - - - - - - (k) 2 4
SUbstituting values of R ~ and R 3 in equation
(i), gives,
1
L
Solving equations (g), (h), (1) for the
unknowns 8 2, 8 .. and 0 gives,
5
o= 196.9, 8 2 = 12.17, e.. = - 6.22 40"
The final end moments can now be found by
sUbstituting these values in equations (a) The bending moment diagram under the given load-
to (f) inclusive. ing will be determined.
M2-" = 6.667 x 12.17 + 3.333 (-6.22) Members 2-3 and 4-3, K = 20/22.4 =0.892
- 144 = 83.6
Fixed end moments:
M..- 2 = 6.667 (-6.22) + 3.333 x 12.17 + 144
- 142.9 MF 2_ 3 = wL'h/12 = (12x20 2 )/12 =-400 in.lb.
M4 - 3 = 4 (-6.22) - 0.6 x 196.9 = -142.9 MF 3 _ 2 = 400, MF.._ 3 = 400, MF 3 _ .. = - 400
M3 - .. = 2 (-6.22) - 0.6 x 196.9 = -130.45 Relative values of 0: -
Fig. A12.22 shows the bending moment Due to the sloping members, the relative
diagram. The student should also draw the transverse deflections of each member are not
shear diagram, find axial load in members as obvious as when the vertical members are con-
since all these loads are needed when the nected to horizontal members as in the previous
example. In this example, jOints (1) and (5)
are fixed. Because of symmetry of frame and
174.7 loading, joints (2) and (4) will move outward
the same distance ~ as indicated in Fig. A12.24.
Furthermore, due to symmetry, joint (3) will
undergo vertical movement only.
11 4- 6 = 2 x 0.5(-28 2 + 0 - 3 x .05¢) + 0
3..t3~~ 11 4- 6 =-28 2 -0.15¢ - - - - - - - (g)
3'---_ j~r
4'
/ - - - - (h)
/
f JOINT (2). Equilibrium equation.
1
112-~+112-3 = 0, sUbstituting-
5
282 + 0.15¢ + 3 ..56882 - 0.5352¢ - 400 =0
Fig. A12.24 whence, 5.5688 2 - 0.3852¢ - 400 =0 - - - - (1)
1 1
"-.:./-H~
Fig. A12.26
M ~-2
114-~ =2x .892(-282 + 0 + 3 x O.l¢) + 400 and in Fig. A12.26, taking moments about (3),
114_~ =- 3.5688 2 + 0.5352¢ + 400 - - - (f) 11 2-~ +11 ~-2 + 240 x 20 - 20 x 12 x 10 - 10H 2-~ =0
11 4_ 6 = 2K(284 + 86 -30) +11F = 11 2 ~ +11 ~ 2 + 2400
4-6 whence, H 2-~ 10
A12.12 SPECIAL METHODS - SLOPE DEFLECTION METHOD
Shear equat i on, H.. _~ = H "-3 Fig. A12.27 shows the resulting bending
moment diagram, first drawn in parts for each
whence, member of the frame and then added to form the
composite diagram plotted directly on the
M~_ .. +M .. _~ _ M.._ 3 +M 3 _ .. +2400 frame. The shears and axial loads follow as a
20 - 10 matter of statics. With the moment diagram
known the frame deflected shape is readily
SUbstituting values for the end moments: - calCUlated.
8.. + 0.150 + 28.. + 0.150 _ 3.5688.. - 0.53520 A12.9 Comments on Slope-Deflection Method.
20 - 10
The example problems solutions indicate
- 400 + 1. 7848.. - 0.53520 + 400 + 2400 that the method of slope-deflection has two ad-
10 vantages, namely, (1) it reduces the number of
equations to be solved simultaneously and (2)
whence, it presents equations that are easily and
rapidly formulated.
0.38528.. - 0.1220 + 240 =0 - - - (j)
Thus for structures with a high degree of
Solving equations i and j for 8 .. and 0, gives redundancy, the slope-deflection method should
be considered as possibly the best method of
o = 2805, 8.. = 265.8 solution. The solving of the equations in this
method are readily programmed for solution by
SUbstituting these values in equations (a) to high speed computing machinery.
(h), the end moments are obtained as follows: -
A12. 10 Problems.
M~_ .. = 8.. + O. 150 = 265.8 + 0.15 x 2805
= 686.5 in. lb. (1) Determine the bending moments at support
points A, B, C, D, for the continuous beam
M.. -~ = 28.. + 0.150 = 531.6 + 420.7 = 952 shown in Fig. A12.28.
M3 - 4 =-1.7848.. +0.53520-400 =-628 (2) Same as problem (1) but consider support A
as freely supported instead of fixed.
M4 - 3 = 952 (3) Determine the bending moment diagram for
M4 _ 6 =- 952 the various loaded structures in Fig.A12.29.
w = 12 lb/in.
M6-4 =- 686.5
,?>f;}
~~...L.,~--"Br- \>~ f;}
\ ~'!)
A
100 100 lb.
300 lb.
A B
1000 1000 lb.
w = 20 lb/in.
4 -L
J~ r
C 6"
V,c. D
= 1 1= 3
20" 1=3
L = 20
Fig. A12.27
B
1 f-- A
20"-------oj
E
A Fig. A12.29
CHAPTER A13
BENDING STRESSES
ShearD~
--hxl<- e - ec or e: L e c (A)
y c c
./ Fig. b Fig. c
. / Moment Dia. .............
We have from Youngs Modulus E, that
In Fig. b the same bar is used as a simply
supported beam with two applied loads P as E : unit stress : -cry
shown. The shear and bending moment diagrams unit strain elL
for the given beam loading are also shown. The
portion of the beam between sections 1-1 and 2-2 where ay is the bending stress under a deforma-
under the given loading are subjected to pure tion e and since it is compression it will be
bending since the shear is zero in this region. given a minus sign.
Experimental evidence for a beam segment Solving for ay,
Ax taken in this beam region under pure bending
shows that plane sections remain plane after a : - E ec y
bending but that the plane sections rotate with y c L
respect to each other as illustrated in Fig. c,
where the dashed line represents the unstressed The most remote fiber is at a distance y : c.
beam segment and the heavy section the shape Hence
after pure bending takes place. Thus the top
fibers are shortened and subjected to compress- E ec
ive stresses and the lower fibers are elongated L
and subjected to tensile stresses. Therefore at whence
some plane n-n on the cross-section, the fibers
suffer no deformation and thus have zero stress. a
y
=- ac y _ - - - _ _ _ _ _ _ _ _ _ _ _ (B)
This location of zero stress under pure bending c
is referred to as the neutral axis.
For equilibrium of the bending stress perpend-
A13.l Location of Neutral Axis. icular to the beam cross-section or in the X
Fig. d shows a cantilever beam subjected to direction, we can write ~ Fx = 0, or
a pure moment at its free end, and under this
The neutral axis does not pass through the Thus, Yn = (y - x tan 0) cos 0 - - - -
beam section centroid when the beam is nonhomo-
geneous that is, the modulus of elasticity is not
- y cos 0 - x sin ---- - o- (2 )
constant over the beam section and also when Then Eq. (1) becomes
Hooke's Law does not apply or where the stress-
strain relationship is non-linear. These beam o = k (y cos o-
x sin 0) - - - - - - (3 )
conditions are described later in this Chapter.
This equation contains three unknowns, 0,
A13.2 Equations for Bending stress, Homogeneous Beams, k, and 0. For solution, two additional equa-
stresses Below Proportional Limit stress. tions are furnished by conditions of equili-
In the following derivations, it will be brium namely, that the sum of the moments of the
assumed that the plane of the external loads external forces that lie on one side of the
contain the flexural axis of the beam and hence, section ABCD about each of the rectangular axes
the beam is not subjected to torsional forces X-X and Y-Y must be equal and opposite, respect-
which, if present, would produce bending stress- ively, to the sum of the moments of the internal
es if free warping of the beam sections was re- stresses on the section about the same axes.
strained, as occurs at points of support. The Let M represent the bending moment in the
questions of flexural axes and torsional effects plane of the loads; then the moment about axis
are taken up in later chapters. X-X and Y-Y is Mx ~ M cos Q and My = M sin Q.
m The moment of the stresses on the beam section
about axis X-X is /0 da y. Hence, taking
moments about axis X-X, we obtain for equil-
Plane ibrium,
of loads
M cos Q =/ 0 da y
N----- m
In similar manner, taking moments about
the Y-Y axis
Fig. A13.1 m Fig. A13. 2 M sin Q =/ 0 da x
whence
Fig. A13.1 represents a cross-section of a
straight cantilever beam with a constant cross- M sin Q = - k sin ¢ / x 2 da + k cos %/ xyda(4a)
section, subjected to external loads which lie
in a plane making an angle Q with axis Y-Y A13.3 Method 1. stresses for Moments About the
through the centroid O. To simplify the figure, Principal Axes.
the flexural axis has been assumed to coincide In equation (4), the term / y'"da is the
with the centroidal axis, which in general is moment of inertia of the cross-sectional area
not true. about axis X-X, which we will denote by Ix, and
Let NN represent the neutral axis under the the term / xyda represents the product of in-
given loading, and let 0 be the angle between ertia about axes X-X and Y-Y. We know, however,
the neutral axis and the axis X-X. The problem that the product of inertia with respect to the
is to find the direction of the neutral axis and principal axes is zero. Therefore, if we se-
the bending stress 0 at any point on the section. lect XX and IT in such a way as to make them
In the fundamental beam theory, it is as- coincide with the principal axes, we can write
sumed that the unit stress varies directly as equation (4):
the distance from the neutral axis, within the
proportional limit of the material. Thus, Fig. M cos Q = k cos 0 Ix (5)
A13.2 illustrates how the stress varies along a p
line such as mm perpendicular to the neutral
axis N-N. In like manner, from equation (4a)
Let 0 represent unit bending stress at any
point a distance Yn from the neutral axis. Then M sin Q k sin 0 Iy (6 )
p
the stress 0 on da is
To find the unit stress 0 at any point on the
(1) cross-section, we solve equation (5) for cos 0
and equation (6) for sin 0, and then SUbstitut-
where k is a constant. Since the position of ing these values in (3), we obtain the follow-
the neutral axis is unknown, Yn will be express- ing expressing, giving 0 the subscript b to
ed for convenience in terms of rectangular co- represent bending stress: -
ordinates with respect to the axes X-X and Y-Y.
A13.3
(7)
A13.5 Method 3. Stresses from Moments, Section Prop-
erties and Distances Referred to any Pair of
The minus signs have been placed before each Rectangular Axes through the Centroid of the
term in order to give a negative value for ab Section.
when we have a positive bending moment, or Mxp The fiber stresses can be found without
is the moment of a couple acting about xp-x p , resort to principal axes or to the neutral
positive when it produces compression in the axis.
upper right hand quadrant. Myp is the moment
Equation (4) can be written:
about the Yp-Y p axis, and is also positive when
it produces compression in the upper right hand Mx =k cos 0 Ix - k sin 0 I xy (ll)
quadrant.
where Ix = I y 2 da and I xy =I xyda, and Mx =
BENDING STRESS EQUATION FOR SY11METRICAL M cos Q.
BEAM SECTIONS In like manner,
Since symmetrical axes are principal axes
(term J xyda = 0), the bending stress equation
My =- k sin 0 Iy 0 I XY - - - - - (12)
+ k cos
for bending about the symmetrical XX and YY axes Solving equations (11) and (12) for sin 0 and
is obviously, cos 0 and substituting their values in equation
(3), we obtain the following expression for the
ab =- MxY _ Myx _ - - (7a) fiber stress ab: -
Ix Iy
ab = _ (MyIX - MxIxy ) x _ (MxI y - MyIXY)y _ (13)
A13.4 Method 2. Stresses by use of Neutral Axis for IXI y - I~y IxI y - I~y
Given Plane of Loading.
The direction of the neutral axis NN, mea- For simplification, let
sured from the XX p principal axis is given by
dividing equation (6) by (5). K~ = Ixy/(IxI y - I XY ) 2
Tan a =- Ix
p
tan Q - - - - - (8) K2 = Iy/(IXI y - I\y)
K3 = Ix/(IXI y - I xy ) 2
It frequently happens that the plane of the A13.8 illustrative Problems. Example Problem 1.
bending moment coincides with either the X-X or Fig. A13.3 shows
the Y-Y axis, thus making either Mx or My equal a unsymmetrical one
to zero. In this case, equation (15) can be cell box beam with
simplified. For example, if My = 0 four corner flange
members a, b, c and d.
Ixy Let it be required to
tan0=- - - - - (16) determine the bending
I y
axial stress in the
and i f Mx =0 four corner members
due to the loads Px
tan o =~-
I xy
- - - - - - - - - - - - - - - (17) and Py acting 50"
from the section
abed.
A13.6 Advantages and Disadvantages of the Three In this example
Methods. solution the sheet 121t
Method 2 (bending about the neutral axis connecting the corner ~ d ~X'
for a given plane of loading) no doubt gives a members will be con - 8[J1t JI'·I.---16-It-~---e.; ,4[J1t
better picture of the true action of the beam sidered ineffective in . y'
relative to its bending as a whole. The point bending. The stresses Fig. A13. 3
of maximum fiber stress is easily determined by will be determined by
placing a scale perpendicular to the neutral each of the three me-
axis and moving it along the neutral axis to thods as presented in this chapter.
find the point on the beam section farthese away
from the neutral axis. In airplane design, SOLUTION
there are many design conditions, which change
the direction of the plane of loading, thUS, The first step common to all three methods
several neutral axes must be computed for each is the calculation of the moments of inertia
beam section, which is a disadvantage as com- about the centroidal X and Y axes. Table A13.1
pared to the other two methods. gives the detailed calCUlations. The proper-
In determining the shears and moments on ties are first calculated about the reference
airplane structures, it is common practice to axes x'x' and y'y' and then transferred to the
resolve air and landing forces parallel to the parallel centroidal axes.
airplane XYZ axes and these results can be used
directly in method 3, whereas method 1 requires TABLE A13.1
a further resolution With respect to the prin- Area
cipal axes. Methods 1 and 3 are more widely Mem. y' x' Ay' Ax' Ay,2 Ax,2 Ax'y'
"Aft
used than method 2. -192
a 1.0 12 -16 12 -16 144.0 256.0
Since bending moments about one principal b 0.5 8 0 4 00 32.0 0 0
axis produces no bending about the other prin- c 0.8 0 -16 0 -12.8 0 204.8 0
cipal aXiS, the principal axes are convenient d 0.4 0 0 0 0 0 0 0
axes to use when calculating internal shear flow Totals 2.7 16 -28.8 176.0 460.8 -192
distribution.
Calculation of Centroid of Section: -
A13.7 Deflections.
The deflection can be found by using the
beam section properties about the neutral axis x = ZAx'
ZA
:: - 28.8
£.7
~ _ 10.667"
for the given plane of loading and the bending
moment resolved in a plane normal to the neutral - ZAy' 16
Y = LU"I.
"'. = - - = 5.926"
2.7
axis. The deflection can also be found by re-
solving the bending moment into the two prin-
cipal planes and then using the properties about Ix = 176 - 2.7 x ~ = 81.18
the principal axes. The resultant deflection is
the vector sum of the deflections in the direc- Iy =460.8 - 2.7 x 10.667 = 153.58
2
Solution by Method 1 (bending about principal axes) (Positive moments produce compression in upper
The angle 0 between the x axis and the right hand quadrant).
principal axes is given by the equation,
Calculation of Stresses
2 I XY
2 (- 21. 33 ) - 42.66
tan 2 0 = -=---'~ = 153.58 - 81.18· 72.40 Myp xp
I y - Ix
I yp
=- .589
Stringer a
2 0 = 30° - 30' , o - 15° = 15' xp =- 6.74, yp = 4.45
I xp = Ix cos 2
0 + I y sin
2
0- 2 I xy sin 0 cos 0 hence
268700 x 4.45 - 156200 (- 6.74)
= 81.18 X .9648 2 + 153.58 X .2636 2 - (- 21.33 x 75.38 159.34
2
- .2636 x .9648)2 15900 - 6600 22500#/in.
Stringer a stringer a
ab
=- 300 200 (- 5.95)
81.18
=22000 42 + 21900 = 21862
Stringer d
y
Yp x = 10.667, Y =- 5.926
9' =14°56'
~ = 15°15'
ab =8 x 10.667 - (3697 x - 5.926) =
9=30°11'
plane of ,loading
= 85 + 21900 = 21985
9' 524' N
X~;;;;;;;::~==-=~:£;F::;=:~ NOTE: The stresses Db by the three methods
-15°15'
were calculated by 10" slide rule, hence the
small discrepancy between the results for the
three methods.
Yp Y Fig. A13.3b
Error in Stresses Due to Assumption that Section
Bends About X and Y Centroidal Axes Due to Mx
Solution by Method 3 (Method Using Properties and My.
About X and Y Axes)
ab =- Mx Y _ My x (Equation 7a)
Mx =300000, My ~ - 80000 Ix Iy
yp = 9.04", x p = 14.24"
yp = - 6.80", x p = - 8.22"
Y Yp - 36.41
Plane of ~' = 186.46 x 431.7 - 36.41"
Resultant 9
Moment-
-.:,...\-++---------. .00046
\
\ I'
K" = I y/ (I x I y - I "xy ) = 186.46 431. 7 _ 36.41"
x 431.7
= 186.46 .002355
793000
stress on Stringer 1:
Y Fig. A13.5
y~ =4.39", x~ =- 17.41"
= .083. Hence, a = 4° - 45'
~ig. A13.5 shows the relative positions of
the neutral axis, principal axes, and plane of
loading. ab = [.002355 x (- 38000) - (- .00046 x 713000)J
~
The component of the external resultant
moment about the neutral axis N equals: - (-17.41 )
MN = 714060 x sin 83° - 26' = 709350 in. lb. - [.00545 x 713000 - (- .00046 x - 38000)J 4.39
r-- 48 "
yg = 6.89, xg = 15.39
- 713000 x 6.89 (- 38000 x 15.39)
°b = 186.46 431.7 The distance from the neutral axis to the
centroid of the trapizoidal stress loading on
_ 25000#/in 2. portion A is 2.64 inches. In like manner,
Example Problem 3.
The previous example problems were solved
TB = CB' = (0 +2 crt) 3 x 0.25 = 0.375crb
by sUbstituting in the bending stress equations.
The student should solve bending stress prob- The arm to TB is 0.667 x 3 = 2 inches.
lems by equating the internal resisting moment
at a beam section to the external bending mom- For equilibrium of the beam free body, take
ent at the same section. To illustrate Fig. moments about point (0) and equate to zero.
A13.6 shows a simply supported loaded beam. The
shear and bending moment diagram for the given ZM o =- 500 x 48 + 2.64 x TA + 2.64 CA' + 2 TB +
beam loading is also shown. Fig. A13.7 shows
the beam section which is constant along the 2 CBl =0
span.
500# 1000 300 Substituting values as found for TA, TB etc,
~ .60 1 •
A ;AI ! - 500 x 48 + 2.64 x 1.15 crb + 2.64 x 1.15 crb
T
!
~ 1
390
p hence crb = 24000
7.58 = 3170 ps i .
100
~OO"# Solution using Bending Stress Formula.
_ Mc
Fig. A13.7
crb - -1- where M =- 24000
c = 3"
I =Moment of inertia about
Neutral axis.
Calculation of I.
For Portions A and A'
I = l~ x 0.25 X 6 3 = 4.50 in 4 •
ITotal =22.76 in
4
•
A13.10 BEAM BENDING STRESSES
_
12#/i~.
Portion Area Average Total y = arm Resisting
~5#/in.
A stress Load to N.A. moment
See in = (2) (4) x (5)
Fig. terms x (3)
~ 100"-------+ 30"~ 100" ~ A13.13 of 0b
1 1 _~tf-- 2" - ..d-f-
Fig. A13. 10 RA RB r 1 1.045 -.3370b -. 353 Ob 1. 39 ... 490 Ob
1
1 1/2 2.35 -1. 780
2 1.00 -.757 -.757
il
I
6 1. 50 • 821O b 1. 2370b -2.57 -3.180 0b
This method of solution involves more cal- For equilibrium, the total compressive
culations than that required in sUbstituting in stresses on the cross-section of the beam must
the bending stress formula, however, the stUdent be equal to the total tensile stresses, or m
should obtain a better understanding of the in- must equal zero. Column 4 of Table A gives the
A13.11
a
b
hence
SOLUTION USING BENDING STRESS FORffiJLA.
=-l1L
IN.A. I N•A. = 27.2 in. 4
J 2.73"
1~
2
I, 22.3x1.5=33.4"
1
16
-\
the stress in the spruce section. Since the re- alloy. The original area of 0.1 sq. in. each
inforcing strips are maple, the stress at the have been multiplied by these stiffness ratio
top edge of these maple strips would be 1.23 values.
times (- 3900) =-
4880 psi. 0.446 0.1615
The bending stress at the lower edge of the ·a-------:cb...
transformed beam section of Fig. A13.16 would be: T
°bt
=_ 60000 x (- 2.73) = 3200
51. 30
Fig. A13. 18
Transformed Beam
Section into Magnesium
Since all these stresses are below the y = 5.35"
Alloy.
elastic limit stress for the 3 materials the
beam bending stress formula as used is
applicable. ~ c
0.10
d
e-=-----=e 0.431
r
10"
Steel ~10 Sta~nl~::
Steel
x-~
Y
tI My = 5000"#
Mx = 10000"#
Ix = 0.6075 x 4.65 + 0.531 x 5.35
Iy = 0.546
..
x 2.365 + 0.165 x 3.635 + 0.431 x
1.635
..
= 6.34 in.·.
..
= 28.27 in.·
11 Magnesium
c .1 Alloy _.10
4" -----it!
Alum. Alloy
Area of Each
Stringer =.1 sq. in.
I xy =0.446 (- 2.365)(4.65)
0.1615 x 3.635 x 4.65
0.10 (- 2.365)(- 5.35)
0.431 x 1.635 (- 5.35)
- - 4.90
= 2.73
= 1.27
- - 3.77
TOTAL - - 4.67 in.· = I xy
Fig. A13.17
SOLUTION; The bending stresses will be calculated
by using method 3 of Art. A13.5.
Since the 4 stringers are made of different
materials we will transform all the materials From Equation (14) Art. A13.5 --
into an equivalent beam section with all 4
stringers being magnesium alloy.
Esteel 29 K = I xy = - 4.67 =
Emag. = 6,500,000
Emag.
= 6.5 = 4.46 1 2
I x I Y - 1xy 28.27 x 6.34 - 4.67 2
Esteel = 29,000,000 28
ES . S . - 4.67 = _ 0 0296
Estain.steel= 28,000,000 Emag. = 6.5 = 4.31 157.4 •
that the maximum failing compressive stress A13.19 shows that for a given strain in the in-
occurs at a strain of 0.01 in. per inch. The elastic range the resulting tensile stress is
problem is to determine the ultimate resisting higher than the resulting compressive stress,
moment developed by this round bar and then and furthermore from internal equilibrium the
compare the result with that obtained by using total tensile stress on the cross-section must
the beam bending stress formula based on linear equal the total compressive stress.
variation of stress to strain. The problem as stated assumed that a
compressive unit strain of 0.01 caused failure.
Fig. b thus shows the strain picture on the
beam just before failure since plane sections
remain plane after bending in the inelastic
range. Table A13.2 gives the detailed calcu-
lations for determining the internal resisting
moment developed under the given strain con-
dition.
TABLE A13.2
1 2 3 4 5 6 7
Thus using the same failing stress at the given in most books on advanced engineering
far extreme fiber the beam formula based on mechanics and will not be repeated here.
linear stress-strain relationship gives an ul- It is convenient however to express the
timate bending strength of 38000 as compared to influence of the beam curvature in the form of
a true strength of 56735 or only 67 percent as a correction factor K by which the stresses ob-
much. tained by the beam formula for straight beams
Fig. c of Fig. A13.10 shows the true stress can be multiplied to obtain the true stresses
distribution on the cross-section, which ex- for the curved beams. Thus for a curved beam
plains why the resisting moment is higher than the maximum stress can be calculated from the
when a triangular distribution is used. equation
The problem of the ultimate bending
strength of structural shapes is discussed in cr =K ~
I
- - - - - (19)
detail in Volume II.
A13.11 Curved Beams. Stresses Within the Elastic Table A13.3 gives the value of K for
Range. various beam section shapes and beam radius of
The equations derived in the previous art- curvatures. The table shows that for only
icles of this chapter were for beams that were rather sharp curvatures is the correction ap-
straight. Thus in Fig. A13.21, the element of preciable. In general for airplane fuselage
length (L) used in the derivation was constant rings on frames the curvature influence can be
over the depth of the beam. The strain (6L/L) neglected. However there are often fittings
was therefore directly proportional to 6L which and mechanical structural units in airplane
had a linear variation. construction whose parts involve enough curva-
In a curved beam, the assumption that plane ture to make the influence on the stress of
sections remain plane after bending still primary importance. The concentration of
applies, however the beam segment of a curved stress on the insirle edge of a curved unit in
beam cannot have equal width over the depth of bending may influence the fatigue strength of
the beam because of the curvature as illustrated unit considerably, thus a consideration of the
in Fig. A13.22, or in other words the length of possible influence of curvature should be a
the segment is greater on the outside edge (L ) regular part of design procedure.
than on the inside edge (Li). Thus in calcu- In the inelastic or plastic stress range,
the influence of beam curvature should be
<1L-1 ~ considerably less since the stiffness of a
material in the inelastic range is much less
MIl
rl
Straight Beam
1M
1 Fig.A13.2l
than in the elastic stress range and changes
rather slowly as the stress increases.
A13.l2 Problems.
R FACTOR K e* FACTOR K
SECTION INSIDE OUTSIDE SECTION R
INSIDE OUTSIDE e*
c R C R
FillER FillER FillER FIBER
C)'
I~j I
1.6
1.8
2.0
3.0
1. 96
1. 75
1. 62
1. 33
0.65
0.68
0.71
0.79
0.108
0.084
0.069
0.030
1.6
1.8
2.0
3.0
2.07
1. 83
1. 69
1. 38
0.76
0.78
0.80
0.86
0.224
0.178
0.144
0.867
~I
4.0 1. 23 0.84 0.016 4.0 1. 26 0.89 0.038
6.0 1.14 0.89 0.0070 6.0 1.15 0.92 0.018
8.0 L10 0.91 0.0039 8.0 1.10 0.94 0.010
10.0 1. 08 0.93 0.0025 10.0 1. 08 0.95 o 0065
1.2 2.89 0.57 0.305 1.2 2.52 0.67 0.408
dJl
1.4 2.13 0.63 0.204 1.4 1. 90 0.71 0.285
1.6
1.8
1. 79
1.63
0.67
0.70
0.149
0.112
r-
-It 1-4t -it I 1.6
, 1.8
1. 63
1. 50
0.75
0.77
0.208
0.160
: I 2.0
3.0
1. 52
1. 30
0.73
0.81
0.090
0.041 i rY±:l I
2.0
3.0
1. 41
1. 23
0.79
0.86
0.127
0.058
f.-- R-j 4.0
6.0
1. 20
1.12
0.85
0.90
0.021
0.0093
UT ~1-1 4.0
6.0
1.16
1.10
0.89
0.92
0.030
0.013
8.0 1.09 0.92 0.0052 8.0 1.07 0.94 0.0076
10.0 1.07 0.94 0.0033 10.0 1. 05 0.95 0.0048
1.2 3.01 0.54 0.336 1.2 2.37 0.73 0.453
rb~ 1.4 2.18 0.60 0.229 1.4 1. 79 0.77 0.319
1.6 1. 87 0.65 0.168 1.6 1. 56 0.79 0.236
T 1 I 1.8 1. 69 0.68 0.128 1.8 1. 44 0.81 0.183
b I 2b I 2.0 1. 58
1. 33
0.71
0.80
0.102
0.046
2.0
3.0
1. 36
1.19
0.83
0.88
0.147
0.067
..1.. 3.0
I
c~-J
1I 4.0
6.0
8.0
1. 23
1.13
1.10
0.84
0.88
0.91
0.024
0.011
0.0060
4.0
6.0
8.0
1.13
1. 08
1. 06
0.91
0.94
0.95
0.036
0.016
0.0089
10.0 1.08 0.93 0.0039 10.0 1. 05 0.96 o 005'1
1.2 3.09 0.56 0.336 1.2 3.28 0.58 0.269
-lCJT
3b 1.6 1. 91 0.66 0.168 1.6 1. 89 0.68 0.134
1.8 1. 73 0.70 0.128 1.8 1. 70 0.71 0.104
I
2.0 1. 61 0.73 0.102 2.0 1. 57 0.73 0.083
r- R --.j
..j c to- 6.0 1.17 0.91 0.001 6.0 1.13 0.90 0.0087
8.0 1.13 0.94 0.0060 8.0 1.10 0.92 0.0049
10.0 1.11 0.95 0.0039 10.0 1. 07 0.93 o 0031
1.2 3.14 0.52 0.352
r-- I 1.2 3.55 0.67 0.409
i8:Jr.
1.4 2.29 0.54 0.243 4t -j 1.4 2.48 0.72 0.292
1.6
1.8
2.0
1. 93
1. 74
1. 61
0.62
0.65
0.68
0.179
0.138
0.110
1tQ}t2
4t t t l 2.0
I
1.6
1.8
2.07
1. 83
1. 69
0.76
0.78
0.80
0.224
0.178
0.144
~11
c r-
! 3.0
4.0
1. 34
1. 24
0.76
0.82
0.050
0.028 1 t/2 I
3.0
4.0
1. 38
1. 26
0.86
0.89
0.067
0.038
6.0 1.15 0.87 0.012 I-c-...j I 8.0
6.0 1.15 0.92 0.018
R~ 8.0
10.0
1.12
1.12
0.91
0.93
0.0060
0.0039 J-- ----J
R 10.0
1.10
1. 08
0.94
0.95
0.010
0.0065
1.2 3.63 0.58 0.418
1 3t
i 42Y2
r- I
1.4
1.6
2.54
2.14
0.63
0.67
0.299
0.229
* e equals distance from centroidal axis to neutral axis.
~~TI
4t I
-l I
1.8
2.0
1. 89
1. 73
0.70
0.72
0.183
0.149
References: Wilson and Quereau. "A Simple Method of
Determining Stresses in Curved Flexural
3.0 1. 41 0.79 0.069 Members, "
-Jcf- I 4.0 1. 29 0.83 0.040 "Advanced Strength of Materials", by Seely.
f.--
R--I 6.0
8.0
1.18
1.13
0.88
0.91
0.018
0.010
10.0 1.10 0.92 0.0065
A13.17
shown in () on figure. External applied (7) Fig. A13.27 shows 3 different beam sections.
bending moments are; They are made of aluminum alloy Whose
stress-strain diagram is the same as that
Mx= -500,000 in. lb. and My 200,000 in. lb.= plotted in Fig. A13.19. Determine the
Find stress on all four stringers by all ultimate internal resisting moment if the
three methods which were explained in this maximum compressive strain is limited to
chapter. 0.01 in./in. Consider that upper portion
is in compression. Compare the results
a
1.2)
obtained with formula M = 0bI/c' where 0b =
compressive stress when unit strain is
(0.5)
1:5 ---r- 0.01.
6"
(1.0) (0.4)-.L
j--G"~12" --!j Fig.A13.24
r- 1" -l ~1/21
10 01
f4
(4) The Zee section shown in Fig. A13.25 is
subjected to bending moments of Mx = 500 1 11/4"
in. lb. and My =
2000 in. lb. Find bending I-- 4
stresses at points a,b,c,d.
1 Fig.A13.27
1"-1
k4
~~4~
_.1.
3/4
~
Mx BO,OOO in.lb. T
EMag.= 6.5xl0 6
ESteel= 29 x 10
6
@Sec. 2
0.1"
Sec. 3
L
0.6"
(6 ) Fig. A13.26 shows (B) Fig. A13.2B shows a curved beam, carrying
a cross-section of a two 500 lb. loads. Find bending stresses
wood beam composed at points C and Cn , when beam cross-section
of 3 kinds of wood is made 3 different ways is indicated by
labeled A, Band C, sections 1, 2 and 3. Use Table A13.3.
glued together to
form a composite x- - - x 6"
beam. If the beam A
~1
is sUbjected to a
bending moment
Mx = 75000 in.lb.,
find intensity of 3/4
l--J'-----"-_'-----' -*'---
bending stress at Fig. A13. 26
top edge of beam.
Also find total end load on portions B
and C.
=
A Spruce. E =1,300,000 psi
=
B Maple. E = 1,500,000 psi
C = Fir. E = 1,600,000 psi
A13.18 BEAM BENDING STRESSES
DOUGLAS DC-8 AIRPLANE. Over-all view of the test wing section representing center wing section of DC-8.
A close-up view of wing test section showing details of wing ribs, stringers, etc.
CHAPTER A 14
BENDING SHEAR STRESSES· SOLID AND OPEN SECTIONS
SHEAR CENTER
A14.1 Introduction.
In Chapter A6, the shear stresses in a Fig. A14.2
Load
I
--Idxl-
member subjected to pure torsional forces were • D' F'
considered in detail. In Chapter A13, the sub-
ject of bending stresses in a beam subjected to _ I yN'A~xr=_=_=='=c-~l ±:_~~=l_x =-:
n'
I
are due to a transfer of external shear forces. ~
~o.~A:~r~·:J
Thus bending of a beam usually involves both
bending (longitudinal tension and compression
stresses) and shear stresses.
The same assumptions that were made in
Chapter A13 in deriving the bending stress
equations are likewise used in deriving the
N.A.
" D F " ,
atf
c Yo L[ RJ J
'y
! D
o. F\.
C Yo
AB.3 Derivation of Formula for Flexural Shear Stress. From Chapter A13, the equation for flexural
Fig. A14.1 shows a loaded simply supported stress a was derived, namely at = Mc/I. Let M
beam. When the beam bends downward due to the equal the bending moment at beam section DD' and
given loading, the beam portion above the neu- M' that at beam section FF' and let I and l'the
tral axis is placed in compression and that be- moment of the inertia of the cross-sectional area
low the neutral axis in tension. Consider a about the neutral axis at these same beam sections
short portion dx of the beam at points DF on the respectively. Then SUbstituting value of at in
beam and treat it as a free body as shown in equations (1) and (2) we can write,
Fig. A14.3. The variation of tensile and com-
pressive stress on each face of the beam portion Total load on face CD =~JCy dA - - - - - (3)
Yo
A14.1
A14 2 BENDING SHEAR STRESSES SOUND AND OPEN SECTIONS. SHEAR CENTER.
2: Fx =- -M-M'
1-- JC ydA + ~ b dx =0 V J3.09
yg ~x-x = -.:.L y dA
Ix b
hence ~ = MI b- M'
dx
Jyc dA - - - - - - - - - - (5)
o
M-M' dM 3.09
However ~ = dx = V = the external shear on y dA.
the beam section.
1o
hence ~ V JC ydA
=r-b (6) TABLE A. REFER TO FIG. a.
Yo
PORTION Area d A y ydA
It is important to note that equation (6) 1 2.09 x 0.5 = 1.045 1. 045 1. 096
applies only to beams of uniform section (con- 2 3 x 0.5 = 1. 50 2.34 3.510
stant moment of inertia). In airplane wing 3 0.5 x 0.5 = 0.25 2.84 0.710
structures the common case is for beams to vary 3' 0.5 x 0.5 = 0.25 2.84 0.710
in cross-section or moment of inertia, and if SUM 6.026
this variation is considerable, equation (6)
should not be used and resort should be made to
~
~~___j:~~~( 2)
b -~
equations (3) and (4). This fact is illustrated (2)
2
in example problem 2. This matter of variable -I -j i, I
I
cross-sections is discussed later in this I I
chapter. Fig. a : Fig. b I I ,III Fig. c
'
Example Problem 2. VARIABLE MOMENT OF INERTIA. Bend. Stress 0b = My/1 Total Bending Stress
Load on: -
Fig. A14.6 shows a cantilever beam loaded On Top Point Point
with a single load of 600 lb. at the end and 801 § Bending or 1" From 2" From
acting through the centroid of the beam cross- .~
"'~
Moment Bottom Top or Top or Portion Portion Portion
"A" "B" "C"
section. The beam section is constant between o:l~ M F'iber Bottom Bottom
Y = 4" Y = 3" y = 2"
stations 0 and 132, then it tapers uniformly to
132 79,200"# +8180 +6135 +4090 7157# 5112# 1023#
the sections shown for stations 175 and 218. .6135 ,4090 7157 10224 1023
175 105,000"# .8180
The shear stress distribution on the beam cross- 218 130,800"# .8180 .6135 ,4090 7157 15336 1023
section at stations 175 and 218 will be deter-
mined.
600#
~43"-----.l...-43"'-'+1~-- 132"------+1
From the results in Table A14.1, it should be
Fixed 1- -----r-- noticed that the change in moment of inertia be-
tween the three stations is directly porportional
to the change in bending moment, hence the same
Sta. '218 Sta: 175 sci. 132 Sta.O value for the bending stresses for all three sta-
tions. Columns 6, 7 and 8 give the total bending
1r2~~
stress load on portions A, Band C of the three
cross-sections (see Fig. A14.6). These values
___ L equal the average stress on the portions times
1" the area of the portion; for example, for station
132, the load on portion A:
c
------ ----
load = 8180 + 6135 x 1 x 1 7157 #, and for
2
portion C:
Sta. 218 Sta. 175 Sta. 132 load = 4090 + 0 x 2 x .25 - 1023#
1= 64.0 in. 4 I = 51. 33 in. 4 I= 38.67 in. 4 2
Fig. A14.7 shows the tension and compressive
Fig. A14-6 stresses acting on a portion of the beam between
A14.4 BENDING SHEAR STRESSES. SOUND AND OPEN SECTIONS. SHEAR CENTER.
stations 175 and 218. Fig. A14.8 shows the re- shear stresses between stations 132 and 175
sulting horizontal shear stress pattern result- would be the same, since the change in bending
ing from the loads in Fig. A14.7. For example, moment and moment of inertia have been made the
i f we take a section along the beam 1" from the same as between stations 175 and 218.
top or bottom edge of the beam and treat this Figs. A14.9 and A14.l0 show the shear
V ~~
portion as a free body as shown in Fig. A14.l2
applying ZH = 0, stress patterns if the formula 'L = dA be
used for each station. The discrepancy is con-
ZH =- 7157 + 7157 + 'L X 43 x 1.0 = 0, hence 'L = 0 siderable as the equation does not apply to
beams of varying section.
To illustrate the calculation by the shear
stress formula, the shear stress will be calcu-
lated at the neutral axis for the beam section
~1023# at station 175.
'L"'I\J>dA
600 x 9.0
hence, 'L = 51.33 x 0.25 = 420 psi. as compared
Fig. A14. 8 Fig. A14.10 Fig. A14.9
to the true shear stress of 477 in Fig. A14.8.
TABLE A14.2
3V e :: 0
True Sta 218 Sta 175 N-B-N 'L :: 2A
Shear Shear Stresses
Pattern By Equation (6)
--43"
1+--1' "
6 ---1r-1/ 4"
7157
15336
7157
10224
F s x 43 x. 25
N - 8 -N 'L
4V
= 3A e =0
Fig. A14.13
Fig. A14.12 d
3V
Similarly, treating the portion between the 'L = 2A e = 6"
tN-8-t-N
edge of the beam and a point 2" from the edge as
a free body diagram as shown in Fig. A14.l3,
1
is to calculate the .1" I
section properties
Method 2. The Neutral Axis Method. about centroidal X and
Z axes. Table A14.3
2
x- - -c -x 3"
I
gives the calculations.
- - - - - - - - - - (10) The section has been
divided into 4 portions
labeled 1, 2, 3 and 4.
Method 3. The Method using section properties ,,'-.r....._ L -
about centroidal Z and X axes. For .1"
brevity this method will be called
the k method. Fig.AI4.26
TAbLE A14.3
Vz = 10000 x cos 30° 8667 lb. The shear stresses at these two points (b)
and (c) would equal q/t = 1134/0.1 and - 5260/0.1
Vx = 10000 x sin 30° 5000 lb. or 11340 psi and - 52600 psi respectively,
Resolving these z and x components further SOLUTION BY NEUTRAL AXIS METHOD. (Method 2)
into components along the principal axes we
obtain, In this solution
it is necessary to find
nrf
Vz p = 8667 x .97527 - 5000 x .2210 = 7348 lb. the neutral axis for
the given external load-
Vx = 8667 x .2210 + 5000 x .97527 = 6792 lb. ing. In Fig. A14.28,
p the angle Q is the angle
between the plane of
Calculation of shear flow at point (b). (See loading and the zp prin-
Fig. A14.26). cipal axis, and this
Fig. A14.27 shows the position of the angle Q equals 30° + 12° Fig. A14.28
principal axes as calculated. The shear flow - 46" = 42° - 46". 150-29' z n
at the free edge of the upper portion (1) is Let a equal angle between x p p
zero. For the shear flow at point (b), the principal axis and neutral axis n-n.
area to be used in the summations Z zpA and From chapter A13, we find,
.z xpA is the area of element (1). The arms zp
I xp tan Q
and x p can be calculated by simple trigonometry. tan a =-
Fig. A14.27 shows the value of these distances,
namely x p =-
0.1184 and zp 1.5136. =
0.63316 x 0.9245
0.02782
= - 21.052
Point (c)
qb = - .~~~~9 (1 x 0.1) (- 0.0466) = 1135 Ib/in.
For portion (2), x = 0, z = 0.7, A = 1.4 x .1
= .14
Shear flow at point (c): - Zn = 0.1868 in.
qc = 1135 - 235438 (0) .14 - 65258 x 0.7 x 0.1
7130
qc = 1135 .02919 (1.4 x 0.1) .1868 = 1135 - 0 - 6395 = - 5260 Ib./in.
= 0.6035 - 0.1305
2
x .0574 - (- 0.1305) =
Fig. A14.29 shows a
beam composed of heavy
flange members and a curved ~r""e
Z
kfj111'r
thin web. For bending
= - 0.1305 = _ 7.406 about the X-X aXiS, the
.01762 web on the compressive, x-- t--~h
side of the beam absorbs \ I~
k = Ix Iz = 0.0574 = 3.257 very little compressive ~ I
2 I z - Ix~ .01762 stress, since buckling of ~
the web will take place 'Flange
Ix 0.6035 under low stresses, par- Z Fig. A14.29
k3 = -I-x-I"':'z=---I=-x-=-z = 0.01762 = 34.25 ticularly when the curvature of the web is
small. On the tension side, the web will be
Resolving the external shear load of more effective, but if the flange areas are
10,000 into x and z components, we obtain, relatively large, the proportion of the total
bending tensile stress carried by the web is
small as compared to that carried by the tension
Vx = 10000 sin 30° = 5000 lb.
flange. Thus for beams composed of individual
flange members connected by thin webs it is
Vz = 10000 cos 30° = 8667 lb.
often assumed that the flanges develop the en-
tire longitudinal bending resistance which
SUbstituting values of Vx ' Vz and k values in therefore means that the shear flow is constant
equation (16) we obtain -
AI4.11
over a particular web. In other words in the EXAMPLE PROBLEM - RESULTANr OF A CONSTANr FLOW
FORCE SYSTEM.
shear flow equation q = Vz I z d A, if the area
Ix Fig. A14.30 shows a constant flow force
of the web is neglected then q is constant be- system thru points ABC D E With q = 10 lb.
tween flange members. per inch. The resultant of this force system
is required.
RESULTANr OF CONSTANr SHEAR FLOW FORCE SYSTEMS
Fig. A14.29 shows a beam assumed to be
carrying a downward shear load (not shown) and R= 20(}#
to cause bending about axis x-x Without tWist.
Assuming the two flanges develop the entire
bending reSistance, the shear flow q is con- e
" 1
= 18.93TB
~10" •
il
stant on the web and acts upward along the web q =10#/in.
to balance the assumed external downward load. fA I - - -
The resultant of this resisting shear flow 5" r-- h =20':-
force system will give the lateral position of
the shear center for this beam section. The
t------
o Fig.AI4.30
problem then is to find the resultant of the SOLUTION: -
shear flow system.
Let q = load per inch along web (constant). Draw closing line between the beginning
and end points of force system. (line AE). The
Let R = resultant of the q force system. length h of this closing line is 20 inches.
From elementary mechancis, From eq. (17) R = q h = 10 x 20 200 lb. =
The direction of the resultant is parallel
R = J z q~ + Z q~ ,where qx and qy are the x to line AE or horizontal in this problem. To
find the location of R take moments about any
and y components of the q forces along the web. point such as (0). Draw lines from point (0) to
Since q is constant, Z qx is zero, hence, points A and E. The enclosed area (A) equals
(17) 5xlO+5xlO+ .511X5 2 +lOx 5 = 189.3 sq. in.
2 A
e =~ -------------- (19)
sheet to form the webs and walls. The flange in order to indicate at what point the shear
members are numbered a to h and the areas of flow q is being calculated.
each are given on the figure. It will be as-
sumed that the webs and walls develop no bending %b = 2.5 ~a z A =- 2.5 x 5 x 0.1 =
resistance and thus the shear flow between ad-
jacent flange members will be constant. The = 1.25 Ib./1n.
problem is to determine the shear center for the
beam section. The first letter of the subscript refers
to the flange member where the shear flow q is
SOLUTION: - being calculated and the second letter indicates
on which adjacent side of the particular flange
Since the beam section is symmetrical member. Hence qab means the shear flow at
about the X axis, the centroidal X and Z axes flange (a) but on the side toward (b).
are also principal axes, since the product of
inertia I xz is zero. qab = qba = - 1.25 (Since no additional flange
The vertical position of the beam section area is added, and thus
centroid due to symmetry is midway between the shear flow is constant on
upper and lower flanges. sheet abo
To find the horizontal position of the
centroid, take moments of the flange areas qbc = qba - 2.5 Zb z A
about the left end or line bc: - - 1.25 - 2.5 x 5 x 0.4 = - 6.25 Ib./in.
= 0.4 x 15 + 0.2 x 10 + 0.2 x 5 6.25
1.6 qcb - 2.5 Zc z A
= 5.625 in. =- 6.25 - 2.5 x (- 5 x 0.4) =- 1.25
qdc = qcd =- 1.25
The moments of inertia for the section about
the centroidal x and y axes are: - qde = qdc 2.5 Zd z A =- 1.25 - 2.5 (- 5 x 0.1)
=0
qed = qde = 0
2 2 qef = 0 - 2.5 Ze z A =0 - 2.5 (- 5 x 0.1) = 1.25
Iz = 0.8 x 5.625 + 0.2 x 0.625 + 0.2 x
2 qfe = qef = 1.25
4.375 + 0.4 x 9.375 8 = 64.4 in 4 •
qfg = 1.25 - 2.5 Zf z A = 1.25 - 2.5
HORIZONTAL POOr-PION OF SHEAR CENTER: - (- 5 x 0.2) = 3.75
The horizontal position of the shear center qgf = qfg = 3.75
will coincide with the centroid of the shear
flow system due to bending about axis xx with- qgh = 3.75 - 2.5 Zg z A = 3.75 - 2.5 x 5 x 0.2 =
out twist. For simplicity, to eliminate large 1.25
decimal values for shear flow values an ex- qhg = qgh = 1.25
ternal upward shear load Vz = 100 lb. will be
assumed and the internal resisting shear flow qha = 1.25 - 2.5 Zh z A = 1.25 - 2.5 x 5 x 0.1 =
system will be calculated for this external 0 (checks free edge at h).
loading.
From equation (8) The sign or sense of each shear flow is for
Vz the shear flow in the y direction as explained
q - - -- Z z A , sUbstituting values of Vz and in the derivations of the shear flow equations.
y Ix Ix The procedure now is to determine the sense of
the shear flow in the plane of the cross-section
100 or in the xz plane. It is only necessary to
qy = - 40 Z z A =- 2.5 Z z A determine this sense at the beginning point, that
We could start the solution at either of is in sheet panel abo The surest way to deter-
two points (a) or h since these points are free mine this sense is to draw a simple free body
edges and thus qy is zero. In this solution, sketch of flange member (a) as illustrated in
Fig. A14.31. The shear flow on the cut face is
we will start at the free edge at point (a) and qy(ab) =-
1.25 and
go counterclockwise around the beam section.
The area of each flange member has been concen- this value is shown
q
trated at a point coinciding with the centroid on the free body. By ""''''--~ --f- (ab)
simnle rule given at /j':::- \.~
of each flange area. In solving for the q val- b O'-:"'- - - "
ues the subscript y will be omitted, and sub- the end of Art. A14.6, qX(;;P
scripts using the flange letters will be used the shear flow in the a
plane of the cross- Fig. A14.31
A14.13
section is also directed toward the common We will again start at the free edge ad-
boundary line and thus qx(ab) has a sense as jacent to flange (a) where qy = O.
shown in Fig. A14.31. The sense of the shear
flow on the cross-section will now continue in
qab =- 1.55 Za x A =- 1.55 (- 0.625 x 0.1) =
this direction until the sign changes in the 0.0971 Ib./in.
origional calculation, which means therefore the qba = qab = 0.0971
shear flow sense will reverse. Fig. A14.32
shows a plot of the shear flow pattern with the qbc = 0.0971 - 1.55 Zb x A = 0.0971 - 1.55
sense indicated by the arrow heads.
(- 5.625 x 0.4) = 3.592
qcb = qbc = 3.592
qcd = 3.592 - 1.55 Zc x A = 3.592 - 1.55 x
5.625 x 0.4) = 7.087
-6.25 qdc = 7.087
qde = 7.087 1.55 (- 0.625 x 0.1) = 7.184
qed = 7.184
qef = 7.184 - 1.55 x 4.375 x 0.1 = 6.504
qfe 6.504
Fig. A14.32
qfg = 6.504 - 1.55 x 9.375 x 0.2 = 3.589
The results will be checked to see if
static equilibrium eXists relative to Z Fx and qgf = 3.589
Z Fz O. qgh 3.589 - 1.55 x 9.375 x 0.2 = 0.674
Z Fz = 100 (ext. load) - 10 x 6.25 - 10 x 3.75 +
Fig. A14.33 shows the plotted shear flow
1.25 x 0.5 x 4 - 1.25 x 0.5 x 4 = 0 (check). results. The signs of the calculated shear
flows are for shear flows in the y direction.
Z Fx =- 5 x 1.25 + 5 x 1.25 - 5 x 1.25 + 5 x Simple consideration of a free body of flange
1.25 =0 (check). member (a) will give the sign or sense of the
shear flow in the plane of the beam section.
The shear flow force system in Fig. A14.32 Thus in Fig. A14.34 qx must act as shown when
causes the section to bend about axis xx with- qy is positive.
out tWist. The resultant of this system is 100 .674
.0971
lb. acting down in the z direction. The posi-
tion of this resultant will thus locate the
lateral position of the shear center.
Equating the moments of the shear flow
system about some point such as (c) to the mo- 3.592
ment of the resultant about the same point we
obtain: -
~
CENTER.
-----For convenience as before, we will assume I
- 1" I
a shear load Vx = 100 lb. and compute the re- qx I- 5.625 -j
sisting shear flow system to resist this load
in bending about axiS zz without twist. The I
a t
resultant of this shear flow system will give
the vertical location of the shear center. The
shear flow equation is, Fig. A14.34 1
S.C.
6.43
+
.0971 x 5) Ix 90.667
k" = I = = 0.006037
100 e = 643 I
x z - Ix~ 15019
e = 643/100 = 6.43 inches ResolVing the given shear load of 141.14 into z
and x components, we obtain,
Fig. A14.35 shows the resulting shear center
location for the given beam section. Vz 141.14 x sin 45° = 100 lb.
ElCAMPLE PROBLEM 2. Unsymmetrical Beam Section. Vx 141.14 x cos 45° 100 lb.
Fig. A14.36 shows a four flange beam sec-
tion. The areas of each flange are shown ad- From equation (14) -
jacent to flange. The external shear load V
equals 141.14 lb. and acts in a direction as
shown. The problem is to find the line of
qy =- (k" Vx - k~ Vz ) Z x A - (k s Vz - k~ Vx )
Z z A
action of V so that section will bend without SUbstituting
twisting.
qy =- [.006037 x 100) - (- .00142 x 100J Z x A -
! Load = 141.14
a 1.0 I Z ft45° [.01136 x 100 - (- .00142 x 100)J Z z A
"i
x- ~_- +r:- ~5
I d
--r whence
Hlz
';:b,.......,
•
0.5
..--~----=-=ec
i
qy = - 0.7457 Z x A - 1.278 Z z A
Fig. A14.36
H16"---l
We will start at flange member (a) where qy is
SOLUTION: - zero on the free edge side of the member.
To solve this problem, method (3) will be Qab =- 0.7457 x 1 (- 5.333) - 1.278 x 1 x 6.667
used.
To locate centroidal x and z axes: -
qba = Qab = - 4.544
z -
_ ZAx
~
=1 x 12 +
3
0.5 x 8 =5 •333 i
n. qbc =- 4.544 - 0.7457 x 1 (- 5.333) - 1.278 x
1 (- 5.333) = 6.249 Ib./in.
x-~
-: Z A z = (0.5 + 0.5) 16 =5.333 in. qcb = qbc = 6.249
3
A14.15
qcd = 6.249 - 0.7457 x 0.5 x 10.667 - 1.278 x A14.11 Shear Center Location By Using Neutral Axis
Method.
0.5 (- 5.333) = 5.680
In a beam subjected to bending there is a
~c = qcd = 5.680 definite neutral axis position for each differ-
qda = 5.680 - 0.7457 X .5 X 10.667 - 1.278 X 0.5 ent external plane of loading on the beam. The
shear flow equation with respect to the neutral
X 2.667 = 5.680
- 3.977 - 1.704 = axis is,
o (checks free edge at d where qy
must be zero.) - in Z ZnA - - - (20)
n
Fig. A14.37 shows the resulting shear flow re- where, Vn = Shear resolved normal to neutral axis
sisting pattern. The sense of the shear flow in In =Moment of inertia about neutral axis
~~~ Zn =Distance to neutral axis
U
,,~.
a / v= 141.14
In finding the shear center location of an
Fig.A14.37a unsymmetrical section, it is convenient to as-
jfR= 141.14
d ....- a
qx
q= 5.680
sume that the Z and X axes are neutral axis and
find the shear flow system for bending about
each axis by equation (20). The resultant of
b q= 6.249 c each of these shear flow force systems will pass
Fig. A14.37 through the shear center, thus the intersection
of these two resultant forces will locate the
shear center.
the plane of the cross-section is determined in
web at flange member (a) by the simple free Example Problem
body diagram of stringer (a) in Fig. A14.37a
Check Z Fx and Z Fz to see if each equals The same beam section as used in the
100. previous article (see Fig. A14.36) will be used
to illustrate the neutral axis method.
6.249 X 16 = 99.99 (checks Vx = 100)
Fig. A14.38 shows the section with the
12 X 4.544 - 8 X 5.68 =-99.94 (checks centroidal axis drawn in. The X axis will now
Vz = 100).
~Fx = 1.47 x 16 = 23.52 lb. Take moments about (b) and let (e) equal
distance to reSUltant R.
~Fz = 7.35x12+8x1.47 = 99.96 lb. (check)
100.8e = 3.125xnx6 2
+3.13x8x16
R = V 100 2
+ 23.52 2 = 103 lb. e = 753/100.8 = 7.47 in.
tan Q = 23.52/100 =.2352 Fig. A14.40 shows the position of this re-
hence Q= 13 0 - 16' lOTIR'to> sultant force. Where it intersects the previous
resultant force gives the shear center location.
Let e =
distance from 23.52
A14. 12 Problems
resultant R to point b.
(1) Fig. A14.42
shows the cross-
Equating moments of section of a wood -,
resultant about (b) to that of shear flow
I I
t
beam glued to-
system about (b), gether on lines a a
a-a and a-b. The
103e = -7.35x6n 2
+1.47x8x16 beam is subjected 4"
644 to a vertical
=- =-6.25
e 103
~4l
as shown. Find
must obtain another resultant force which the shear stress
passes through the shear center before we can at sections a-a l00Olb. <So
definitely locate the shear center. Therefore and b-b by three 1-1,,-1
we will now assume that the Z centroidal axis different methods.
is a neutral axis and that a resolution of the ...L .2°"
external load system gives a shear Vx 100 lb. = f T...--.--1--·..L....,·~10-"~.10"
r
=I-=-=-=O-1 3/8"
I T 1" 1\' .11"
qY Vx ~ XA
=- Iz 170.667 = II .05" T} .040" 10" Neglect Web in
_~,14.44~"
Iz
2' :11_F_ig_. . -.L I Resisting Bending
17~~~67 x (-5.333)1 = 3.125 lb./in. ~=:::::JJ5/16L .2°" .1 0 " .1°"
%b - -
1" 1"+-1" T r- 6" + 6 " - 1
qbc = 3.125 -1~g~66 (-5.333)1 = 6.25 lb./i Fig. A14.45 Fig. A14.46
(3) Determine the shear flow diagram and the
shear center location for bending about hori-
qcd = 6.25 -1~g~66 (0.5) (10.667) = 3.13 zontal centroidal axis for the beam sections
as given in Figs. A14.44 to A14.46.
Fig. A14.41 shows the shear flow results.
LO .4 d
~Fx =-6.25 x 16 = -100 lb. Fig. A14.47
(
l~o4/1
EJ 10"
I
1
r-20"-l~'
t-t
+
I
10"
10"
l£50"~
Fig.A14.51
Fig.A14.52
1 sq. in.
a r
,
e i P
c
r- -+ t
1.5°"
12 8 10
1. 0 0.8 1.2
'4j
T
.5 sq. in.
15" 10" 20"
=5 b ...,::::...------:::.a.-"
1.2 0.7°" 0.8 1.0 1
r +-
I
A1S. 1 Introduction. The wing, fuselage and em- open sections we could start the summation at a
pennage structure of modern aircraft is essen- free surface where q would be zero, thus the
tially a single or multiple cellular beam with summation to any other point would give the true
thin webs and walls. The design of such shear flow qy. In a closed cell there is no
structures involves the consideration of the free end, therefore the value of qy is unknown
distribution of the internal resisting shear for any point.
stresses. This chapter introduces the student
to the general problems of shear flow distri- Equation (1) gives the shear distribution
bution. Chapter A14 should be covered before for bending about the X axis without tWist.
taking up this chapter. The general procedure is to assume a value of
the shear flow qy at some point and then find
A1S.2 Single Cell Beam. Symmetrical About One Axis. the shear flow pattern for bending without
All Material Effective in Resisting Bending tWist under the given external load. The cen-
Stresses.
troid of this internal shear flow system will
Fig. A15.1 shows a single cell rectangular be the location where the external shear load
beam carrying the· load of 100 lb. as shown. should act for bending without twist. Since
The problem is to find the internal resisting the given external shear would have a moment
shear flow pattern at section abed. about this centroid, this unbalanced moment
must be made zero by adding a constant shear
100# flow system to the cell.
qo = O. The term
a
*
at point 0 on the web ad.
= 100/62.5 = 1.6
%0 = -1.6 2: ZA = -1.6x2.5x5xO.l = -2 Ib/in.
o
Fig. A1S-l
x+ z
qo'b = -10-1.6 2:~'zA=-10-1.6X2.5X5X.05=
d
.05 = -10
= -2
Solution 1
qod = -z-1.6 2:~ zA =-2-1.6(-2.5)5xO.l=0
Due to symmetry of material the X cen-
troidal axis lies at the mid-height of the
beam. The shear flow equation requires the Fig. A15.2 shows a plot of the shear flow
value of Ix, the moment of inertia of the reSUlts. On the vertical web the increase in
section about the X axis. shear is parabolic since the area varies
directly with distance z.
Ix = 1~X.15XIO"+2~OX.05X5lilJ = 62.5 in. 4
The intensity of qx and qz in the plane of
From Chapter A14, the equation for shear the cross-section is equal to the values of q
flow is, found above which are in the y direction. The
sense of qx and qz is determined as explained
qy = - i~ 2: zA - - - - - (1) in detail in Art. A14.6 of Chapter A14.
This equation gives the change in shear
flow between the limits of the summation. In
A1S.l
A15.2 SHEAR FLOW IN CLOSED THIN-WALLED SECTIONS. SHEAR CENTER.
1. 66 it/in.
1I1-;-':-':-~ q = 11 it/in.
-8.34 it/in.
2.66 it/in.
2 it/in.
4.+y:.,.Lfl---
or 2Ag =~ i
Z L • The right hand 3ide of
Fig. A15-4
6=ZiL----
axis.
- - - - - - (2)
Fig. A15.4 also shows the resultant shear
Using the values of q from Fig. A15.2 load Q on each portion of the cell wall which
and substituting in (2), equals the area of the shear flow diagram for
various portions as shown.
6 =Z qLt =1..:9
f] x 5 +
x .1
(10 + 2) 20 + 10 x 5
2 . 05 .05
ZF z =-
56.9 - 43.1 =-
100, which balances
the external load of 100 lb.
+ (11 - 10)(.667 x 5)1 2 = 7000 ZFx =
0 by observation. Take moments about
.05 J point 0, the intersection of axis XX and side ad.
Since the section and shear flow pattern _ ZM o _ 43.1 x 20 + (-23.8 + 16.58)10 _
is symmetrical about the X axis, the substi- x - ZF z - 100 -
tution above is written for one-half of the
cell and the results multiplied by 2. When 7.90 in.
the shear flow q varies over a portion the
average shear flow is used in the above Hence the shear center lies on the X axis, 7.9"
substitution. from side ad.
If the cell is not to twist the relative The moment of the external load of 100 lb.
tWist of 7000 must be cancelled by adding a about shear center equals 100 x 7.9 =
790 in.
constant shear flow q around the cell to give lb. clockwise. The moment of the internal shear
a total shear strain of -7000. flow of Fig. A15.4 is zero thus we have an un-
balanced moment of 790. Therefore for equil-
The shearing strain for a constant q ibrium of moments we must add a constant shear
equals, flow q around cell to develop -790 in. lb. or
walls and webs which are stiffened by members hence q1 =16.21 Ib/in. (sign comes out pOSitive,
usually referred to as flange members. A hence assumed sense of q1 is correct).
simplifying common assumption is to assume
that the flange members alone develop the Fig. A15.7 shows a plot of the internal
resistance to the bending moment. This shear flow resisting system.
assumption therefore means that the shear flow
is constant between flange members. A single cell beam having only two flanges
can resist only external loads which are parallel
Fig. A15.5 shows a single cell beam with to the line AB, and thus a two flange box beam
two flanges at A and B. Find the internal is not used very often in aircraft structure.
shear flow force system when the beam carries When the bending moment in a plane at right
the external load of 100 lb. as shown. angles to line AB is small, the resistance of
the curved panel to compressive bending stresses
may be SUfficient to resist such external bend-
1-10,J ing moments and thus be satisfactory.
-.---'1-' V=100#
1 100 6.21
I A15.4 Shear Center of Single Cell - Two Flange Beam.
I
1
,~ 1/2" Let it be required to find the shear center
......--.--o;:r=---L of the beam as given in Fig. A15.5. In other
z
i
~10"~
words where would the external load have to be
placed so that the beam would bend without
twist.
A~
6.21
Fig. A15-6 Fig. A15-7
~.=w.I--C1/2"
2.05 7.95
Fig. A15-5
which if added to the shear flow system of • 128. 54 sq. in. Fig. A15.13
A15.10 will give the true shear flow system Fig. A15.11
of A15.8.
Fig. A15.11 shows a single cell beam with
Thus having the shear center location, three flange members, A, Band C, carrying the
the external load system can be broken down external load as shown. A three flange box if
into a load through the shear center plus a the flanges are not located in a straight line
moment about the shear center. The shear can take bending in any direction and therefore
flow due to each is then added to give the is often used in design because of its simpli-
true resisting shear flow. city.
It should be noticed that the web or skin For such a structure, there are six un-
thickness does not influence the magnitude of knowns, namely, the axial load in each stringer
the shear flow system in a single cell beam. and the shear flow q in each of the three sheet
A change in thickness, however, effects the panels that make up the cell. For a space
unit shearing stress and therefore the shear- structure, we have six static equations of
ing strain and thus in computing angular tWist equilibrium, thus a three flange single cell
A15.6 SHEAR FLOW IN CLOSED THIN. WALLED SECTIONS. SHEAR CENT ER.
To find qca take moments about point B The three unknown resisting shear flows
and equate to zero. will be assumed with the sense as indicated by
the arrow heads.
2MB = 100 x 5 - 25 x 10 + qca(128.54 x 2) = 0
To find Qac take moments about B
250 -- - 0 • 972 Ib/in.
h ence qca - - 257.08
2MB =100 x 5 - qac (128.54 x 2) =0
To find qab take ZF z =0
Qac =1.945 Ib/in.
ZF z =100 - 10 x 0.972 - l0Qab =0
ZFx =- 15 x 1.945 + 15qbc = 0
Qab = 9.13 Ib/in.
qbc = 1.945
To find qbc take ZF x =0
ZF z =100 - 10 x 1.945 - l0Qab =0
ZF x - - 15 x 0.972 - 25 - 15qbc =0
Qab =8.055 Ib/in.
hence qbc =- 2.639 Ib/in.
The algebraic signs of the unknown q value
The signs of qca and qbc came out negative, all come out positive, thus the assumed direction
hence the sense of the shear flow on these cell of shear flows in Fig. A15.14 is correct.
wall portions is opposite to that assumed in
Fig. A15.12. The resulting shear flow pattern To make the cell twist zero, we must add a
is plotted in Fig. A15.13. constant shear flow q to the cell (see Fig.
A15.15). The relative tWist under the shear
The student should realize the thickness flow of Figs. 14 and 15 will be equated to zero.
of the wall elements does not influence the
shear flow distribution if we assume the three Z~ =_ 1.945 x 20.71 1.945 x 15 + 8.055 x 10
flanges develop the entire resistance to the t .03 .025 .04
bending moment. + 20. 7lq + 10q + 15q =0
.03 .04 .025
A15.6 Shear Center of Single Cell-Three Flange Beam.
Constant Shear Flow Webs. Whence, q = 0.322 Ib/in. with sense as
assumed in Fig. A15.15. Adding this constant
Let it be required to determine the shear shear flow to that of Fig. A15.14 we obtain the
center location for the beam in Fig. A15.11. shear flow system of Fig. A15.16. The resultant
The shear center is a point on the beam cross- R of this shear flow system is obviously - 100
section through which the resultant external lb., since the external load was 100 lb. The
shear must act if the cell is to bend without location of this resultant R will therefore
twist. locate the horizontal position of the shear
center. Equate moment of resultant R about
The shear center location will be deter- point B to the moment of the shear flow system
mined in two steps, first its horizontal about B, whence,
location and then its vertical location.
100e = 1.623 (128.54 x 2)
Calculation of horizontal location:-
or e = 417/100 = 4.17 in. from line AB.
We will assume any vertical shear load, (Fig. A15.16)
as the example, the same vertical shear as used
in the problem of Art. A15.4, namely, a 100 Calculation of Vertical Position of Shear Center
lb. load acting five inches from A, as
illustrated in the following Fig. A15.14. A convenient horizontal shear load will be
A15.7
assumed acting on the cell. Since we used a A15.7 Single Cell-Multiple Flange-One Axis of
25 lb. load in the example problem of Art. 15.5, Symmetry.
we will assume the same load in this solution.
Fig. A15.17 shows the loading and the assumed Fig. A15.20 shows a single cell beam with
directions of the three unknown shear flows. 8 flange members, carrying a 100 lb. shear load.
The resisting shear flow system will be calcu-
lated •
• ()l\:...~~~--,
R=2.Q. T
e z
~ .40" . 10" . 10" . 20"
J[
B
g
Fig. A15-17 Fig. A15-18 Fig. A15-19
x
Solving for the three unknown shear flows
in Fig. A15.17. d e
----.- """!'---~-----!"--~-
2MB =- 25 x 10 + qca (128.54 x 2) = 0 • 5"
Re = 2MB
25e =0.648 (128.54 x 2)
To check ZMy =
zero take moments about
some point such as C.
Fig. A15-23a
1600=
- Vx
q=75.8
Fig. A15-26
9
Taking moments of all forces both external
7 8 and internal about point (0),
Vx=400# 5
r~ . x
ZM o = 1000x2+400x3+17123 = 20323 in.lb.
10
(17123 equals summation of column 13)
14 15 16
Thus for eqUilibrium a negative moment of -20323
z Fig. A15-27 is needed. This moment is provided by adding a
AI5.11
Table A15.1
1 2 3 4 5 6 7 8 9 10 11 12 13 14
0 0 0 0 0 0 -20.6
1 0.14 4.396 0.615 -17.41 -2.437 0.615 -2.437 1.175 - 3.240 2.065
2 0.14 6.446 0.902 -13.54 -1. 896 1. 517 55.2 114 -18.53
2.089 - 7.991 5.902 44.2 261
3 0.38 7.396 2.810 - 9.11 -3.462 4.327 -4.333 3.758 -22.794 19.036 609
-14.70
4 0.17 7.766 1. 320 - 5.44 -0.925 5.647 -7.795 4.205 -29.748 25.543
32.0
38.2 976
- 1.56
4.94
5 0.17 7.946 1. 351 - 0.86 -0.146 6.988 -8.720 4.2'15 32.590 1075
6 0.17 7.896 1. 342 3.14 0.534 -8.866 -36.865 33.0 11. 99
7 0.17 7.696 1. 308 7.14 1. 214 8.340 -8.332 4.017 -43.935 39.908
47.393
33.2 1325 19.31
8 0.17 7.296 1. 240 11.74 9.648 -7.118
1.996 10.888 3.432 -50.825 40.2 1905 26.79
9 0.29 6.896 2.000 15.39 4.463 12.888 -5.722 2.470 -57.358 54.888 32.6 1789 34.29
0.318 67.576
16 0.35 -8.411 -2.964 15.39 5.386 9.924 -0.659
4.727
-67.894
-52.280 54.559
251. 8
46
1702
2509
46.98
33. 16
15 0.31 -8.224 -2.587 9.64 2.988 -2.279
14 0.31 -7.734 -2.395 3.32 1. 629 7.327 7.715
4.982 8.744
-3.720 -38.862 42.582
30.44
48.2
46.4
2052
1413
21.98
9.84
13 0.31 -7.004 -2. I'll -0.917 2.813 7.827 -4.216 -26.245
- 2.96 -3.774 -14.819 18.59 47.6 885
12 0.28 -5.554 -1. 554 - 9.11 -2.551 1. 259 5.276 - 2.01
11 0.17 -4.504 -2.544 - 6.632 9.176 36.6 336 -11. 43
-0.766 -13.54 -2.302 0.493 2.974 4.031 42.6 172
10 0.17 -2.904 -0.493 -17.51 -2.976 -1. 434 - 2.597 -16.57
0 0 0 0 0 -20.6
1
L: 17123
constant negative shear flow around cell where A15.9 Two Cell-Multiple Flange Beam. Symmetrical
magnitude equals About One Axis.
Fig. A15-28
46.98
Fig. A15-30
shear flow at any point in each cell is unknown. zero. Thus the shear flow in the vertical web
Therefore, to make the flexural shear flow cc ' is determined by the stringer c alone,
statically determinate, a value for the shear namely
flow q in each cell will be assumed at some
point, and the flexural shear flow for each qcc' = -4Z~' zA = - 4x 1 x 5 = -20 Ib./in.
cell will then be calculated, consistent with
the assumed conditions. These resulting static We can now continue around cell (2) starting
shear flow systems will, in general, produce a with stringer c' where we were previously
different total shearing strain around the stopped.
perimeter of each cell, or in other words,
produce a different cell tWist. Since full
continuity exists between cells, this condition
qc'd' = qb'c' + qcc' 4Z~:zA
cannot eXist, and therefore an unknown constant =0 20 - 4 x (-5) 1 =0
shear flow of q in cell (1) and q 2 in cell (2)
must be added t6 make the tWist or both cells qd'e' =0 4 x (-5) 0.5 =10 Ib./in.
identical. This fact gives us the basis for
one equation and the other equation necessary qe'e =10 - 4 x (-5) 1 =30 Ib./in.
for the solution of the two unknowns q1 and q2
is given by the requirement of equilibrium, qed = 30 - 4 x 5 x 1 = 10 Ib./in.
namely, that the moment of the external and
the internal shear forces about any point in qdc =
10 - 4 x 5 x 0.5 = 0, which
checks
the plane of the cross section must equal zero. the assumed value of q = 0 in panel cd.
In Fig. A15.31 the flexural shear flow has
been assumed as zero just to the left of The shear flows in cell (2) could of
stringer c in cell (1) and just right of course been found by starting in panel cd
stringer c in cell (2). The balance of the where the shear has been assumed zero and
flexural shear system consistent with this proceeding clockwise around cell as for example
assumption is calculated as follows:
%e =0 - 4 x 5 x .5 =-10 Ib./in.
--
o.t>/L}LJ' ~
q=O -
d <Ix=IO
this case has no moment about our assumed
moment center.
mb = -50 x 10 x 5 + 20 x 10 x 5 + 10 x 30 x 15 + 200q~
Fig. A15.32 illustrates the unknown con-
stant shear flow systems q~ and q" which must + 200q" = 0
act on cells (1) and (2) respectively to pro-
duce the same cell tWist when added to the hence, 200q~ + 200q " + 3000 = 0 - - - - - - - (4)
snear flow system of Fig. A15.31. The sense
Solving equations (3) and (4) for q~ and q", we
of q~ and q" has been assumed clockwise or obtain
positive in each cell.
q~ =-4.07 Ib./in. q" = -10.80 Ib./in.
The final or true internal shear flow sys-
t---"7i
ql ~ t~--t·
tem then equals that of Fig. A15.31 plus that of
(I) ~ ~ r(2)
L-___ ~
Fig. A15.32 when q~ = -4.07 and q" = -10.80 Ib./
ft___ ~
in., which gives the shear flow diagram of Fig.
~ A15.33.
Fig. A15-32
in Ib./in.)
using the shear flow values in Figs. A15.31
and 32, the angular tWist of each cell will Fig. A15-33
be calculated by substituting in the above
equation. Solution 2 (By use of shear center)
For cell (1) In this solution, we find the flexural
shear flow for bending about axis X-X without
2A GQ = - 10 x 5 _ 50 x 10 _ 10 x 5 + 20 x 10 + ~ + tWist. The centroid of this internal shear
~ ~ .03 .05 .03 .03 :DO system locates the shear center. The moment of
the external shear load about the shear center
3 :O~Og], - ~~'" Hence, 2A~GQ~ = 1200q~ - 333q" produces pure torsion on the 2 cell beam. Thus,
adding the shear due to this pure torsion to
- 6670 - - - - - - - - - - - - - - - (1) that of pure bending, we obtain the final re-
sisting internal shear flow.
For cell (2)
In bending about axis X-X Without tWist,
2A"GQ,,= 10 .~~ x 2 + 3~MIO _ 2~0310 + 3 :&008 +~ the shearing strain for each cell as given by
equations (1) and (2) must equal zero. Hence:
- ~g~~. Hence, 2A"GQl1l = -333q ] + 1250q ,,+ 1200 q~ -333q" - 6670 =0 (5)
Solving equations (5) and (6) for q:l. and q'a> the resisting internal torque must therefore
we obtain q2 =-
2.0 lb./in. q:l. =
5.00 lb./in. equal -3600. Therefore,
Therefore, taking these values of q:l. and qe
in Fig. A15.32 and adding the results to that qt(l) = .00254(-3600) = -9.17 lb./in.
of Fig. A15.3l, we obtain the shear flow
pattern of Fig. A15.34 which is the shear flow Solving for qt(2)
system for bending without twist about X axis.
The centroid of this shear system locates the lfa.o:l.Ae + a:l. 2AJ
shear center. qt (2) =2"[ 31190000 T
In Fig. A15.34,
1:.Q.. + 10 + J:.Q) 100 + J:Q. x 20~
ZV = 0 = -10 x 45 - 10 x 28 - 10 x 27 = - 1000
lb., which checks the external shear of 1000
=1
2 Q (.03 .03 .05)
31190000
.03 T= 00245T
.
lb. ZH =
0 by observation of Fig. A15.34.
hence: qt(2) = .00245x-3600 = -8.85 lb/in.
V= 1000#
Therefore, if we add to the shear flow sys-
tem of Fig. A15.34, a constant shear flow of
-9.17 lb./in. to cell (1) and -8.85 lb./in. to
cell (2), we will obtain the true internal re-
-45
sisting shear flow of Fig. A15.35, which checks
solution 1, any discrepancy being due to slide
rule accuracy.
Fig. A15-34
Z:t1a = 10 x 27 x 10 +10 x 28 x 20
+ 5 x 8 x 10 - 5 x 2 x 10 = 8600 in .lb.
hence, x = ~ggg = 8.6" to the right of web aa' •
Fig. A15-35
Torsional Deflections
The external shear load of 1000 lb. acts 5" to
the right of aa' , and therefore causes a moment The angular twist of each cell is the same.
The value of the angular twist Q per unit length
about the shear center equal to (8.6 - 5.0)
1000 = 3600 in.lb. To resist this torsional of the beam can be found using the shear flow
pattern of Fig. A15.35 which is the true result-
moment, a constant torsional shear flow qt(l) ant shear flow, or the pure torsional shear flows
and qt(2) must act on cells (1) and (2)
respectively. of qt(l) = -9.17 lb./in. and qt(2) -8.85 lb./in. =
may be used if desired.
The values of qt(l) and qt(2) can be found The results will, of course, be the same.
by using equations (29) and (30) of Art. A6.ll For example:
of Chapter A5. Thus
For cell (1) due to qt(l) = -9.17 lb./in.
and qt(2) =
-8.85 lb./in.
Cell (1) Final stresses - Fig. A15.35 cell the same, since if any twisting takes
place, all cells must suffer the same amount.
2GGA _2:qL=(14.17 x 5)2 + (4.17 x 5)2 + 54.17 x 10 Furthermore, for equilibrium, the moment of
J. t ( •03 ) ( .03) .05
the internal shear flow system plus the moment
_26.68 x 10 = 8000 of the external shear load must equal zero •
•03
For bending about axis XX, the flexural
Cell (2) Final stresses shear flow will be assumed as zero at a point
just to the left of stringer a in cell (3) and
200A _ZqL_(l0.85X5)2+ (0.85x5)2+26.68 x 10 just to the left and right of-stringer c in
2 t ( .03 ) ( .03) .03
cells (1) and (2) respectively. One might con-
-19.15 x 10
.04
= 8000 sider the cells as cut at these three points.
Fig. A15.37 shows the flexural shear flow under
these assumptions. Since the leading edge cell
AI5.IO Three Cell - Multiple Flange Beam. Symmetrical (3) has no stringers and the covering is con-
About One Axis.
sidered ineffective in bending, the shear flow
Fig. A15.36 shows a 3-cell box beam sub- will be zero on the leading edge portion since
jected to an external shear load of 1000 Ibs. the shear flow was assumed zero just to the left
as shown. The section is symmetrical about of stringer a. The resulting flexural shear
axis XX. The area of each stringer is shown flow for the 3 cell section will therefore be
in parenthesis at each stringer point. The identical to Fig. A15.31 and the calculations
internal shear flow system which resists the for the flexural shear flow will be identical to
external load of 1000 Ibs. will be calculated those in Art. A15.7.
assuming that the webs and walls take no
bending loads, or, the stringers are the only
-
IO#/in.
effective material in bending. The moment of
inertia about the XX axis of effective material
equals 250 in ~ (Note: this beam section is
identical to the two cell beam of Fig. A15.30
plus the leading edge cell (3).
50#/in. 20#/in.
IO#/in.
-
.025 Fig. AI5-37
.05 .03 .04
-x Fig. A15.38 shows· the unknown constant
shear flows qJ,' qs' and q3 which must be added
(3) (1) 10" (2) to the flexural shear flow of Fig. A15.37 to
L 5"__
I
~2.0)
a'
'
1(·5)
5"--1- 5 " _
b
.03 c'
1~1.0)
d'
1(·5)
5"(-J-5"~
.03 e'
1(1.0)
sense of each has been assumed po~itive in each
cell.
- t- --r
qi -.t tf ~---"""J----
....
Fig. AI5-36
1 1t
q3
q2
•
Solution No.1 (Without use of shear center)
tt •t •
(3) (1)
~t
(2)
•
The system is statically indeterminate, to H t •,
the third degree, since the value of the shear
flow q at any point in each cell is unknown.
Fig. A15-38
sumed at a point in each cell and the flexural
shear flow for bending about the XX axis will The angular tWist Q for each cell equals Q =
be determined consistent with this assumption. 1
A constant unknown shear flow q J,' q 2' and q" 2AG 2: q Lit
for cells (1) and (2) and (3) respectively
will be added to the static flexural shear Using the values of q in Figs. A15.37 and
flow so as to make the angular twist Q of each A15.38, the value GG will be computed for each
cell.
A15.16 SHEAR FLOW IN CLOSED THIN-WALLED SECTIONS. SHEAR CENT ER.
Cell (3)
2QA G
3
= 2":qL
t
2Qx39.4G = 10x50
•05
+ 15.71 q + 10~ _lOqJ.
--:c525 3 •05 •05 22.91 #/in.!
or 00 = 10.5q21-2.55qJ. + 127 - - - - - - - - (1) (2)
-24.97 #/in.
Cell (1)
2QA G
J.
= 2":qL
t
-12.12#/in. -2.12#/in. -7.09#/in.
2QxlOOG = -50xlO_2(10x5) +10x20+3(10qJ.)
.05 .03 .03 .03 Fig. A15-39
10qJ.
+---:05-~-~
10q3 10q" h ence 00 -- 6qJ.-q3- 1 •67 q"
A15.11 Shear Flow in Beam with Multiple Cells. Method of
Successive Approximations.
- 33.34 - - - - - - - - - - - - - - - - - - (2)
The general trend in airplane structural
Cell (2) design appears to be to the use of a relatively
large number of cells. There are various
2QA G
"
=2":qL
t
reasons for this trend some of which are:
(1) using multiple interior webs, the detri-
2Q x 100 x G =_ 20 x 10 + 2 (10 x 5) + 10 x 30 + mental effect of shear deformation on bending
stress distribution is decreased; (2) the fail
.03 .03 .040
safe characteristic of the wing is increased
3xlO xq"+lOq,, 10q. hence 00 = 6.25q,,- because the wing is made statically indeter-
.03 .04 -.03
minate to a high degree and thus failure of
1. 67qJ. + 20.83 - - - - - - - - - - - - - - - (3) individual units due to fatigue or shell fire
can take place Without greatly decreasing the
Taking moments of the internal shear flow over-all ultimate strength of the wing;
systems of Fig. A15.37 and A15.38 and the ex- (3) the ultimate compressive strength of wing
ternal load of 1000 lbs. about stringer a and flange units is usually increased because
equating to zero:- - column action is prevented by the mUltiple webs
which attach to flange members.
ZMa = 10 x 20 x 10 + 10 x 30 x 20 - 5 x 1000
In Chapter A6, Art. A6.l3, the method of
+ 78.6q3 + 200qJ. + 200q" = 0 successive approximation was presented by de-
= 3000 + 78.6q3 + 200qJ. + 200q,,=0 - - (4) termining the resisting shear flow system when
a mUltiple cell beam was subjected to a pure
Solving equations (1) (2) (3) and (4) for the torsional moment. This method of approach has
unknown qJ.' q", q21 and 00 we obtain: now been extended to determine the resisting
shear flow when the beam is subject to flexural
qJ. - - 2.12 lb./in. bending without tWist*. Using these two methods
the shear flow in a beam with a relatively large
q" - - 7.09 lb./in. number of cells can be determined rather rapidly
q3 = -14.5 lb./in. as compared to the usual method of solving a
number of equations.
00 = - 19.9
Adding these constant shear flows to the flex- PHYSICAL EXPLANATION OF THE METHOD
ural shear flow of Fig. A15.37, we obtain the Fig. A15.40 shows a 3-cell beam carrying
true internal resisting shear flow as shown in and external shear load V acting through the
Fig. A15.39. shear center of the beam section but as yet un-
known in location. In other words, the beam
bends about the symmetrical axis X-X without
twist. The problem is to determine the internal
resisting shear flow system for bending without
tWist. In this example, it is assumed that the Now consider each cell as a separate cell.
bending moment is resisted entirely by the The static shear flow qs acting on each cell
flange members as represented by the small will cause each cell to twist. Since zero
circles on the figure, which means that the twist is necessary a constant shear flow q~ to
shear flow will be constant between the flange cell (1), q~ to cell (2), and q~ to cell (3)
members. must be added as shown in Fig. A15.42, and the
magnitudes of such value as to make the twist
The first step in the solution is to make of each cell zero. However, the cells are
the structure statically determinate relative actually not separate but have a common web be-
to shear flow stresses for bending without tween adjacent cells, thus the shear flow q~
tWist. In Fig. A15.41 imagine each cell cut acts on web 2-1 which is part of cell (1), and
at points a, band c as shown. For the given thus causes cell (1) to twist. Likewise cell
shear load V, the static shear flow qs can be (3) is tWisted by q~ and cell (2) by both q~
calculated, assuming the modified section bends and q~. Therefore to cancel this additional
about axis X with no tWist. Fig. A15.41 shows cell tWist, we must add additional constant
the general shape of this static shear flow shear flows q~, q~ and q: as shown in Fig.
pattern. A15.43, and considering each cell separate
again. However, since the cells are not separ-
ate these additional shear flows effect the
twist of adjacent cells through the common web.
ell Cell Cell As before this disturbance in cell twist is
-x (1)- - (2) - -(3) - -x Fig. A15-40 again cancelled or made zero by adding further
closing shear flows q~, q:, q: as shown in
Fig. A15.44. This procedure is repeated until
the closing shear flows become negligible. In
general the converging of this system is qUite
rapid and only a few cycles are necessary to
give the desired accuracy of results.
Fig.AI5-41
The total closing shear flows qJ.' q2 and
q" are then equal to -
qJ. = q~ + q~ + q~ + ----
qs _
O-t
~I
----------t
~--
qs
----t
~--qr-
qs
Fig. A15-42
q2 =
q" =
q~ +
q~
q: + q: +
+ q: + q: +
'\ ~1lt tqi
t__ -J
t (2) }
~ ________tt qs
(3)
The final shear flow on any panel then
"<:::...........
'<--'
1
--- qs --- qs
equals, (See Fig. A15.45)
q = qs + qJ. + q2 + q" - - - - - - - - - (1 )
The centroid of this final shear flow
Fig.AI5-43 system locates the shear center of the section,
relativA to bending about the X axis.
(~L)
operations form a very simple routine.
a T
L;qsL 1-a tL)
(L; a-at
~~ ~ L q1 +
+ L; L qa - - - (3) A15.12 Example Problem Solution. Problem No. 1.
a tat a t
Fig. A15.47 shows a cellular beam with
The first term in equation (3) represents five cells. The flange areas and the web and
wall thicknesses are labeled on the figure.
the proportion of the static shear flow qs
which must act as a constant shear flow around The problem will be to determine the internal
shear flow pattern when resisting an external
cell (2) to cancel the twist due to qs. The
resulting value of this first term will be
shear load of Vz =
1000 lbs. without tWist of
given the term q~. the beam. Having determined this shear flow
system the Shear center location follows as a
The second and third terms in (3) repre- simple matter.
sent the constant closing shear flows required
Fig. A15.48 shows the assumed static
in cell (2) to cancel the twist of cell (2)
condition for determining the shear flow system
due to the influence of q1 and qa in the ad-
jacent cells acting on the common webs between in carrying a Vz load of 1000 lb. without twist.
The static condition is that all webs except
the cells. The ratio in equation (3) before
the right end web have been imagined cut as
q1 will be referred to as the carryover
indicated thus making the shear flow q at
influence factor from cell (1) on cell (2) and these points zero. s
will be given the symbol C1- a , and the ratio
before qa in equation (3), the carryover
In this example problem it will be assumed
influence factor from cell (3) to cell (2) and
that the flange members develop all the bending
it will be given the symbol Ca_ a • Thus
stress reSistance, which assumption makes the
equation (3) can now be written as,
shear flow constant between adjacent flange
- - - - (4) members.
As explained above, q~ is the value of the The total top flange area equals 5.5 in. a ,
necessary closing shear flow for zero twist and also the total bottom flange area. Due to
when the adjacent cell shear flows are zero. symmetry the centroidal X axis lies at the
Hence first approximations to the final shear mid-depth point.
flows in each cell can be taken as neglecting
the effect of adjacent cells, or in other words Hence, Ix = (5.5x5 a )2 = 275 in. 4
The sign is positive because qs is posi- because the tWist under the static shear flow
tive. (Clockwise shear flow on a cell is must be canceled. The values for q' are re-
positive.) Row 1 in the Table shows the values corded in row 5 of the Table.
as calculated for the 5 cells.
For example for cell (1),
The second step as indicated in row 2 of q' = _ 18180 = _ 21.238
the Table is to calculate the value of the 856
expression ~ Lit for each cell.
For cell (2),
For example, for CellI,
27275 -- - 28. 71
q I = - 95()
(2) .04
.04
(3) .04
.03
(4) .03
.03
(5) .03 X
~ 2a
.04
1b
.04
.5 c
.04
.5 d
.03
.5 e
.03
If
I 5 Cells at 10" = 50" ---------------11
Fig. A15-48 a' 36.36 b' 54.55 c' 63.64 d' 72.73 e' 81. 82 f
Assumed Static
Condition !or cut Cell Lcut • cut Lcut Lcut 100
Shear Flow qs 0 (1) 0 (2) 0 (3) 0 (4) 0 (5)
Row ,OPERATION
Fig. A15-49
Closing Shear
t----~t r - - -- t~- -- --t~-- -- ---tt- - - - t
Flows to Make
Twist of Each
t~ q =33. 51
U
t~ q =52. 45
U
U q =63. 43
U
H q =73. 91
H
U q =84. 39
t
~
Cell Equal Zero
~----.. q tt ~q---.... u ~_~ __tt
In row 13 of the Table, the carryover condition where the static shear flows in the
values are so small that the process is termin- webs should be much closer to the final values
ated. The final constant shear flow that must and thus hasten the convergency in the succes-
be added to each cell to cancel the tWist due sive approximation procedure.
to the static shear flow equals the algebraic
sum of the values from the beginning row 5 to Thus in Fig. A15.51, we have assumed the
row 14. The results are shown in row 14 of the top panel in each cell as cut to give the static
Table. condition. The static shear flow is now con-
fined to the vertical webs and zero values for
The results in row 14 are obtained after a top and bottom sheet. Table A15.3 shows the
considerable number of multiplications and calculations for carrying out the successive
additions of numbers, thus it is easy to make a approximation~ and needs no further explanation.
numerical mistake. To check whether any It should be noticed that after the first ap-
appreciable mistakes have been made, we take proximation was made in row 5, only three carry
the values in row 14 and consider these values over cycles were needed in rows 6, 7 and 8 to
of constant shear flow in each cell as that obtain the same degree of accuracy as required
causing zero twist if cells are separate. Then in 8 cycles in Table A15.2 for solution 1. Fig.
bringing the cells together, through the common A15.52 shows the final shear flows which equal
webs causes a disturbance in twist and this is the constant shear flows in each cell from row
made zero by the carryover values. This step 9 of Table added to the static shear flows in
in the Table is referred to as a reiteration Fig. A15.51. These values check the reSUlts of
and is indicated in row 15. Then adding the solution 1 as given in Fig. A15.50, within slide
values in row 15 to the initial approximation rule accuracy. In Table A15.3 no reiteration
q' in row 5, which value is repeated in row 16, steps were given. The student should make it a
we obtain the final value of q in row 17. The practice to use such checks.
values in row 17 are practically the same
magnitude as in row 14, thus no appreciable A15.13 Example Problem 2.
mistakes have been made. If the difference All Material Effective in Bending Resistance.
was appreciable, then a second, and if needed,
even a greater number of reiterations should The general trend in supersonic Wing struc-
be carried out. In the Table a second reiter- tural design is toward a large number of cells
ation is shown in rows 18, 19, 20 and the and relatively thick skins, thus in general, all
results in row 20 are practically the same as cross-sectional material of the Wing is effective
in row 17. in resisting bending stresses and thus the shear
flow varies in intensity along the walls and
It will be assumed that the solution was webs of the beam cells. Fig. A15.53 shows a ten
stopped after first iteration, and thus the cell beam with web and wall thicknesses as
values in row 17 are the constant shear flows shown. It will be assumed that all beam mater-
that must be added to the static shear flows ial is effective in bending. The shear flow
to produce bending Without tWist. Fig. A15.49 resisting system for bending about the horizontal
shows these final closing constant shear flows. axis Without tWist will be determined. The Cen-
Adding these values to those in Fig. A15.48 we troid of this system will then locate the shear
obtain the final shear flows in Fig. A15.49. center.
The lateral location of the shear center Fig. A15.54 shows the static condition that
for this given 5 cell beam coincides with the has been assumed, namely, that the upper sheet
centroid of the shear flow force system in panel in each cell has been imagined cut at its
Fig. A15.50. The calculations for locating midpoint, thus making the static shear flow zero
the shear center are given below Fig. A15.50. at these points. The static shear flow values
qs are shown on Fig. A15.54. To explain how
Solution 2 of Problem 1 they were calculated, a sample calculation will
be given.
In solution 1, the assumed static condi- The moment of inertia of the entire cross-
tion involved cutting all vertical webs except section about the horizontal centroidal axis is,
the right end web. Thus the static beam
section became an open channel section and the Ix for top and bottom skins,
resulting static shear flows must obviously be = (50 x 0.125 x 2.5 3 ,)2 = 78.0
far different than. the final true shear flow Ix of all webs = (0.912 x 5 3 )/12 = 9.5
values, since the webs always carry the greater Total Ix = 87.5 in 4
shear flows in bending without tWist. This
fact is indicated by the relatively large num- For convenience an external shear load
ber of steps required in Table A15.2 to reach Vz = 8750 lb. will be assumed acting on this
a state where successive corrections were small beam section.
enough to give a desired accuracy of final re- Hence, q = -:!z..."L,zA = - 8750 "L,zA =- 100 "L,zA
sult. Thus it is logical to assume a static Ix 87.5
A15.22 SHEAR FLOW IN CLOSED THIN-WALLED SECTIONS. SHEAR CENTER.
Now consider Fig. A15.55 which shows a Starting at point (a) in cell (1) where the
sketch of cell (1) plus half of cell (2). As shear flow is zero and going counter-clockwise
previously explained the upper cell panels were around the cell, the static shear flows are
assumed cut at their midpoints (a) and (m). as follows: - (Cont. on PI. A 15.23 ----_J--I)
Solution II cut cut cut cut cut
0 0 0 0 0 0 0 0 0 0
Fig. A15-51 ,
Assumed Static
Condition for 36.36 (1) 18.18 (2) 9.09 (3) 9.09 (4) 9.09 (5) 18.18
Shear Flow qs and
Resulting qs Values.
0 0 0 0 0
Table A15. 3
Row OPERATION
Fig. A15-53
00#'
ec (1)
00#'
(2)
00#'
(3)
00#'
(4)
00#'
(5)
00#'
(6) ~
... (7)
...
<Xl (8)
...
t- (9)
... T(10) ~ 5"
o '"o '"
0 '"
0 ~ '"
0 0 0
t-
0
Fig. A15-54 -98 185.4 185.4 185.4 185.4 185.4 181.3 181.3 178.5 178.5 98
-78 ~o-- - o + - - 0 - ~ o - - 0 - - 0 - -
-78 -78 78 -78 78 -78 78 -78 78 -78 78 -78 78
-0- -78 78
~o-
-78 78
-0- -0- -78 78 -78
Table A15.4
109.31 177.66 183.16 185.24 187.11 191.59 176.50 182.07 167.91 161.44 120.61
Fig. AI5-57.
-89.31
Final Shear Flow Values.
66.69 -81.57 - 74.4' -79.33 76.67 -79.17 76.83·80.88 -
75.12 -87.07 68.93 -82.27 73.73 -83.04 72.96·72.45 83.55 -55.39 100.61
(Note: Shear Flow at Ends of Webs Equal Sum of Shear Flows in Adjacent Skin Panels.)
A15.23
c---5"-----+·+<I<:--2.5'L-..+i
10+1
Proceeding to (g)
.cut ,cut
5"
.094) = 156 + 29.4 = 185.4 lb./in.
li·
c - - (1) - - - - g l - - ( 2 ) - -
qfg = 185.4 + 100ZfZA = 185.4 + 100 (-1.25)
g
5
(2.5x .094) = 156 lb./in.
d
'-=-_ _---..:e::-. ...:f..L...- k
qfk = qfg - qfe = - 156 + 78 = - 78
Fig.AI5-55
=0 k
Qa qkf =-78-100Z ZA =-78-100 (-2.5)(2.5x
f
qb =_lOm;bZA =-100x2.5x2.5xO.125
a 0.125) =-78+78 =0
= -78 lb./in.
Fig. A15.56 shows a plot of these calcu-
qc = qb - 100ZcZA = - 78 - 100 (1.25 x 2.5 lated values. The arrows give the sense of the
b shear flows.
x 0.064) =-78-20 =-98 lb./in. iM78~mm[II:Iii~~~~~mrn~
qd = - 98 _ZdZA = - 98 -100 (-1.25)(2.5 x
c
0.064) = -98 + 20 = -78 Fig.
98 (2) A15-56
qe =_78_Z e ZA =-78-100 (-2.5)(2.5 x
d
0.125) = -78 + 78 = 0
qha = 0 + 100ZhZA
a
=0 + 100 (2.5 x 2.5 x 0.125) Having determined the static shear flows
which will be referred to as qs' we can now
= 78 lb./in. start the operations Table A15.4. The first
horizontal row gives the calCUlations of the
With two other webs intersecting at joint twist of each cell under the static shear flOWS,
(f) the shear flow summation cannot continue which is relatively measured by the term
L
past (f), hence we go to point (m) in cell (2) ZqsTfor each cell.
where shear flow is zero due to the assumed
cut at point (m).
With all material effective in bending the
shear flow varies along each sheet. Fig. A15.56
Clm =0 shows this variation on the sheet panels of cell
qhm = O-lOOZhZA =-100(2.5x2.5xO.125) (1). The term Zqs L/t is nothing more than the
m area of the shear flow diagram on each sheet
=-78 divided by the sheet thickness. To illustrate,
consider cell (1) in Fig. A15.56.
Now at joint (h) we have the shear flow of 78
magnitude on each top panel, thus the shear Upper sheet panel: -
flow in the vertical web at (h) equals the sum
of these two shear flows or 156. Zq ..h = -(0 + 78) ~ +(0 + 78) 2.5 =0
s t 2 0.125 2 0.125
A15.24 SHEAR FLOW IN CLOSED THIN-WALLED SECTIONS. SHEAR CENT ER.
Due to constant closing shear flows as Fig. A15.58 shows a single cell distributed
A15.25
flange beam. Consider the beam acts as a canti- flange members, see Fig. A15.60. Again ZF y = 0,
lever beam with the bending moment existing at
section (A) being greater than that eXisting at whence ~Pa + ~P4 + qyd =0
section (B) and that the bending moment produces
(~Pa + ~P 4)
compression on the upper surface. By the use of or qy =- d
the flexural stress equations, the bending
stress on each stringer can be found, which if
Therefore starting at any place where the
multiplied by the stringer area gives the value of qy is known, the change in the average
stringer axial load. Thus at beam section (B), shear flow to some other section equals
let P~, P 2 , Pa' etc. represent the axial loads
due to a bending moment M. The external bending
qy =- Z ~ - - - - - - - - - -(1)
Lx ~
~!4'
-- d(~P3)b
whence, qx
d- - qxbd -- 0
= ~Pa /d - - -(2)
Thus for rectangular sheet panels between
flange members the shear flow qx or qz equals
the average shear qy.
(l :: lO~rD
therefore is the final shear flow system for
this method of sOlution.
f5"
-.l
2 1 2
-r
x 10"
Da
1 1
b
1
T10" Fig. A1S-63 Fig. A1S-64
c' a' b' J. c' 'b' -L Solution No.2, Considering ~P Loads in Flange
h"+s"~ 1rs,,-+s,,~1 Stringers. (Equation 1)
SECTION A-A
I x = SxS 2 x2 =2S0in4
SECTION B-B TO C-C
I x =3 xS 2 x2= lS0in'l:
Bending moment at section AA = 1000 x 50 =50000
in. lb.
Fig. A1S-62 Bending moment at section BB = 1000 x 30 = 30000
obtained by increasing only corner stringers b in.lb.
and c. The shear flow on section A-A will be
computed using equation (3) which applies only Considering Section B-B:
to beams with constant section and also by Bending stress intensity at midpoint of
equation (1) which applies to beams with vary- stringers by the flexural formula:
ing moment of inertia.
a -~ - 30000 x 5 - 1000 psi.
b - Ix - 150 -
Solution 1. Using Shear Flow Equation for
Beams of Constant Cross-Section. Axial load in each of the stringers a, b, and
(Equation 3) c = 1000 x 1 = 1000 lb.
Since q at any point on the cell is un- Considering Section A-A:
known, it will be assumed that the upper sur-
face on Section A-A is cut through the midpoint - Mz _ 50000 x 5 - 1000 i
ab - IX - 250 - ps •
of flange stringer (a), thus making the shear
flow qy equal to zero on this free surface. Axial load in stringer (a) = 1000 x 1 = 1000 lb.
One-half of stringer (a) thus acts with each
side of the top surface. In this solution the Axial load in stringer (b) or (c) = 1000 x 2 =
webs and walls will be assumed ineffective in 2000 lb.
resisting bending stresses, thus the shear flow
is constant between adjacent stringers. These resulting axial loads are shown act-
ing on the portion between points A and B in Fig.
Starting at midpoint of stringer (a) and A15.65, which equals the results as shown in Fig.
going counter-clockwise around cell, A15.66.
qa = 0 (assumed cut)
V 1000
Qac = --?-'L.ZA
x
= - 250 x5xO.5=-10 lb/in
QcA = -10
1000
qcc' = -10- 250 x5x2=- 50 lb/in.
Proceeding around the cell the balance of the
shear flow c·ould be calculated, but due to sym-
metry enough values have been found for the
shear flow to draw the complete shear flow pic-
ture for bending about the X axis when it is Having found the ~P flange loads over a
assumed that one-half of the area of stringer length (d) of 20", the shear flow can be com-
(a) acts with each adjacent web. Fig. A15.63 puted by equation (1).
shows the resulting shear flow diagram. The
resultant of this shear flow pattern is a It will be assumed that one-half of the ~P
1000 lb. force in the Z direction and its load in stringer (a) will flow to each adjacent
location through the midpoint of the box since wet. However, there is no ~P load in stringer
the flow is symmetrical. The external load of (a) hence Qab =
Qac =
O. Then from equation (1),
1000 lb. also acts through the midpoint of the ~?,... 1000
cell hence the external load is in equilibrium qbb' =
O-'L. T = O--mr-= - 50 lb./in.
A15.27
6Pb' -1000
qb I a I = -50 + Z-d- = -50 - 2 0 = 0
"0' +-i.=.~ ~
1111
0'0' "0'
Fig. A15.72 shows a tapered single cell
beam with 6 spanwise stringers or flange members.
The beam is loaded by a 1000 lb. load located as
Fig.AI5-68 Fig.AI5-69 shown. Assuming the webs ineffective in bending
A15. 18 Shear Flow in Tapered Sheet Panel.
the internal resisting shear flow pattern will
be determined.
Major aircraft structural units such as
the wing, fuselage, etc., are tapered in both In this solution the shear flow at Station
plan form and depth and therefore the sheet 120 will be determined by considering the 6P
panels between flange members usually are flange loads over a length of 30" or between
tapered in width. Fig. A15.70 shows a canti- Stations 90 and 120.
lever beam tapered in depth and carrying a
load V at its end. The flange reactions at Consider section at Station 120:-
the left end have been found by statics. A
free body diagram of the web is shown in Fig. Bending stress ab --1-
- Mc _ 1000 x 120 x 5 = 1333.33
- 450
A15.71. Take moments about point (0) and psi.
equate to zero.
The horizontal component of the axial load
mo =(~~) b" - q~b~d = 0 =
in a stringer equals aoA (where A area of the
stringer) •
A15.28 SHEAR FLOW IN CLOSED THIN-WALLED SECTIONS. SHEAR CENTER.
T 3 . 2i a I'l" c 1.75
equation (5).
8.5"
L-
Ix=271
a' b' c'
Sec. Sta. 90 qX(ab) = 24.87x5.5/6 =- 23.2 lb./in.
3.25 2.5 .75
qy(bC) -- 24.87 - 471/30 = - 40.57
Sta.90
1000# f----8" ----l -- 40.57x5.5/6 = - 37.2
t qX(bC)
1
qy(cc' ) =
=
40.57 -196.6/30
47.12x8.5/10 =
=- 47.12
40.0 lb./in.
---.L 90" Sec. Sta. 0 qZ(cc' )
4 k"~6"~ 2 12 O
Sta'1--_ _ _ 120'
Since the ~P loads are the same on the
1
Sec. Sta. 120 lower stringers but tension the shear flow cal-
hence, the horizontal components, of stringer cUlations if continued would give the same
loads are, values as found on the top surface. Fig. A15.74
shows the shear flow pattern on station 120.
Pa = 4x 1333.33 =5333.33 lb.
IN PLANE FORCES PRODUCED BY INCLINATION
Pb =3 x 1333.33 = 4000 lb. OF FLANGE MEMBERS
Pc =2 x 1333.33 = 2666.7 lb.
Since the box tapers in depth and Width, the
Consider beam section at Station 90:- flange stringers are not normal to section 120,
thus X and Z force components are produced on
1000 x 90 x 4.25
crb = 271 = 1411.5 psi. section by the stringer loads.
- 24.87 lb./in.
133.3 t 100 66.7
88.9_
!.-3"-+1! ~
-444
a -23.2 - -G
- -b ~37.2 .
1
10"
t
~
40.0~ Fig. A15-74
j 88.9
I~'
~
23.2 b' 37.2 : ,
- - - - - 44.4
6" 1 6" l
133.3 1(~0 66.7
Fig. A15-73
/
471
A15.29
The forces in Fig. A15.74 will be checked Taking moments about point a of the forces
for equilibrium. in Fig. A15.74 but replacing the shear flows in
the top and bottom panels by the values found
ZF z = 1000 - 266.6 - 200 -133.4 -10 x 40 =0 above, we would obtain an unbalanced moment of
7970 in. lb. instead of 8670 previously found.
ZFx =0 by observation. The correcting shear flow would then be q =
-7970/240 =
-33.2 instead of -36.1 as previously
Take moments about stringer (a), found. The final shear flow pattern would be
as shown in Fig. A15.76, which values should be
ZMa =- 3 x 1000 + 200 x 6 + 133.4 x 12 + 10 x 40 compared to those in Fig. A15.75.
x 12+(+88.9 - 44.4)10 + 23.2 x 6 x 10
+ 37.2 x 6 x 10 = 8670 in. lb. 15.4 2.1
This corresponds to value of 23.2 in (4) Figs. A15.79 and A15.80 show two loaded
previous solution (see Fig. A15.74). single cell - 2 flange beams. Assume the flanges
develop all the bending resistance. Determine
qy(bc) = 17.8+~gx5x3 = 31.11b./in. the shear flow resisting pattern by two solu-
tions, namely, without and with use of shear
As compared to 37.2 in Fig. A15.74. center.
qy(cc') = 31.1+~gx5x2 = 40.0 Ib.in. (5) Fig. A15.81 shows a single cell - 3 flange
beam subjected to loads as shown. Assume the 3
Which is the same as in Fig. A15.74.
A15.30 SHEAR FLOW IN CLOSED THIN-WALLED SECTIONS. SHEAR CENTER.
flanges develop the entire bending resistance. (11) Fig. A15.84 shows a multiple flange-circu-
Determine the internal shear flow resisting lar beam section. Find the resisting shear
system. flow pattern when carrying the external shear
OO#
¥
300 lb. load of 5000 lb. as located in figure. Assume
Q~'A
0
100 lb. L1 cell skin ineffective in resisting beam bending
45 stresses •
•04 .02
fL\'02 10" t1000 lb.
l-_~
•8--,"" 1~2
L1---'1B
.03.4r}
- l
c---2"
r- ~500 lb. 0.5
f V= 3000 lb.
r--10'~ .1
r 6 " ---j
.2
Fig. A15-81 Fig. A15-82 -,..
03 .03 6"
"'--it-----i.-._---i..----4t--__ ----L
.3 .1 .1 .1 .2
(6) Find shear center location for beam in
Fig. A15.81 if the 3 flanges provide the entire
bending resistance.
r-t 0',10''l
I- 4 panels a 5" = 20" 'I
Fig. A15-85 Fig. A15-86
(7) Find the internal resisting shear flow (12) Determine the shear flow resisting system
pattern for the 3 flange-single cell beam of for the beam section of Fig. A15.85. The 6
Fig. A15.82. Assume webs or walls ineffective flanges have areas of 0.2 sq. in. each. Skin
in bending. is .032 thickness. Assume skin ineffective in
bending.
(8) Determine shear center location for beam
of Fig. A15.82. Webs and walls are ineffective (13) Find the shear flow resisting system for
in bending. the unsymmetrical beam section in Fig. A15.86.
t
2000#
Flange areas and skin thicknesses are given on
figure. Assume skin ineffective in bending •
.3 .4
~~041
Top Skin .03 (14) Determine shear center location for beam
section in Fig. A15.86.
Fig. A15-83 .035 10"
Bottom Skin .03 I r "-i 1000 f 500 lb.
$02
5
#
.3 .4
~ 6 panels at 5" = 30"
---1 T8" .06"
.05
.03
[:303
.04
.-l .05 .03
(9) Fig. A15.83 shows a multiple flange-single
cell beam section. Find resisting shear flow
1--12"---+-8" --1
system if webs and skin are ineffective in Fig. A15-87 Fig. A15-88
resisting bending stresses. All skin flange
members have area of 0.1 sq. in. each. (15) Fig. A15.87 shows a 2 cell beam section.
Consider all material effective in resisting
(10) Find shear center location for beam bending stresses. For the given beam loading
section in Fig. A15.83, if webs and skin are determine the internal resisting shear flow
ineffective in bending. system.
~ i .t V = 2000 lb.
I' Top Skin .04
Fig. A15-90
Al6.l
A16.2 MEMBRANE STRESSES IN PRESSURE VESSELS
absent due to the symmetry of the problem. The its support, is neglected.) In the lower hem-
included angles between the pairs of meridional
and tangential forces are dsm/Rm and dStIRt,
respectively.
Summing forces normal to the differential
element, one has
o o
or,
Here p is the internal pressure, positive out- ispherical portion meridional stresses are re-
ward. Note that the shell wall thickness does quired as shown in Fig. A16-4b. Hence, in this
not enter eq. (1). The pressure p may vary in class of problems it is best to derive the nec-
essary second equilibrium equation (correspond-
the meridional direction but is constant in the ing to eq. 2) by considering the individual
tangential direction by hypothesis (rotational characteristics of the structure.
symmetry assumed).
Eq. (1) is one equation containing two un- A16.3 Applications to Simple Pressure Vessels.
knowns. Another equation may be obtained by the
condition of equilibrium of a portion of the Example Problem 1
shell above or below a parallel circle. Thus in Determine the membrane stresses in a cylin-
Fig. A16-3, the pressures acting downward on the drical pressure vessel of circular cross section
lower portion of the shell are equilibrated by (radius Ro ), having hemispherical ends, if the
the upward vertical components of the meridional internal gas pressure is p. Also find the
stresses, Nm. greatest combined normal stresE.
Fig. A16.5
__ Nt __ P Ro 2.7 R o p
stress Nb = 2
t t
Therefore
The meridional stress in the cylindrical portion
is found from eq. (2):
P Ro
= 1.25 Ro P
2
Finally, the compressive ring load, F, is (ref.
stress =P R o
Fig. A16-6c)
2t
F
(0)
From the calculus, the radius of curvature of
this meridian curve is
Fig. Al6.6
The meridional stress is found from eq. (2). Summing forces horizontally at the floor joint:
Thus,
Nf =p (RU cos a], + RL cos all) •
Assume all stresses are equal and are given by
s.
SUbstituting the expressions for Rm, Rt and Nm
into eq. (1) one finds Su =s = p RU/tU
sL = s = P RL/tL
Sf = s = t:-
(RU cos a], + RL cos all)
f
Of particular interest are the stresses at Letting the weight denSity of the material
the seam. Here y = 0 and x = a. One finds be w, the weight per unit length (axially) along
the cabin is (w times the developed length of
walls and floor).
Nm = P2a
w = W\ 2 tuRU (n - a],) + 2 tLR L (n - all)
Nt = pa (1 - 2~:) + 2 tf RU sin a], ~ •
This last result is important since it in- Solving for the various thicknesses from above
dicates that compressive tangential stresses are and substituting, one finds (to obtain a result
possible i f a> b...[2. As will be seen below, symmetric in appearance use was made of the
such a situation is undersirable because of the
large resultant difference in radial expansions
fact that RU sin a], =
RL sin all)
between the cylinder and bulkhead (the bulkhead
actually contracts radially if Nt is negative) W =~ lRU (n - a], + ~ sin 2 a],) +
producing high secondary bending stresses in the
vessel walls. RL (n - all + ~ sin 2 all) ~
Example Problem 4. Since one may show that the cross sectional area of this
Determine the weight per unit length of the fuselage is
double-cylinder fuselage cross section as a
function of the internal pressure, allowable A = Ru(n - a], + -} sin 2 a],) t Ri. (n - Gil + -} sin 2 as}
stress and the geometric parameters of Fig.
an important consequence of this calculation is that the ratio
A16-S. For structural efficiency it is desired of shell weight to shell volume is
to maintain equal membrane stresses in the skin
and floor. w _ 2wp
A --s-
Example Problem 5.
Determine the membrane stresses in a
conical vessel of height h and half apex angle
Fig. A16.8 a. The cone forms the bottom of a large vessel
filled With a liquid of specific weight wand
having a head of liqUid H above the cone. The
complete-unit is supported from above.
A16.5
SOLUTION: (fl. =
Poisson's
ratio, =
.3 for
The meridian curve of the cone has a aluminum)
=
radius of curvature Rm 00 and, at any point a
distance y down from the top of the cone, the
radius Rt is
N - R - w (H + y) (h - y) tan a
t - P t - cos a
n (h - y)2 tan ~ a • w (H + y) + P Ro
= .35 p Ro
E:
tBLKHD -- 2Et (1 - rII) Et
+ W 3 (h - y) 2 tan 2 a • (h - y)
Hence, by integration
Solving
P R~
= .35 E"t
1\1
""ID.
= w-2 -
tan-a
cos a (h - y) [ H + Y + 1
- (h - y) ]
3 Thus the cylinder tends to expand more than
the bulkhead - a situation prevented by the seam
A16.4 Displacements, Boundary Conditions and Local
between these elements. It follows then that
Bending in Thin Walled Shells. the seam experiences a transverse shearing action
It is appropriate at this point to examine as indicated in Fig. A16-11. These shear forces
some of the foregoing illustrative cases to de- in turn produce bending moments in the shell
termine whether or not the membrane stresses wall as shown on the figure.
computed gave satisfactorily accurate measures
of the shell stresses. Anticipating the answer,
we state that, while the membrane analysis will °blkhd
~ive the primary stress system in a shell-like
pressure vessel, a careful (and often lengthy)
analysis of induced bending caused by boundary
effects will reveal localized secondary stress
peaks. In static strength analyses of properly
designed* vessels it is the practice to neglect
While it is not our purpose here to take up shell bending 13% above the membrane stress (as against only
in detail, some indication of the character and magnitude of 3% above for the hemispherical bulkhead) (ref-
these bending stresses should be available to place them in
proper perspective. The most striking thing about these wall
erence 2, p. 410). For this type of bulkhead,
moments is that they are quickly damped out, becoming neg- boiler codes sometimes permit a ratio of alb as
ligibly small (down to 1'10 of their maximum value) at a distance high as 2.6, however.
of about 4VRc;t from the seam. Thus, for an instance, in a
circular cylindrical shell of 40" radius and. 065" wall thick- A16.4 Special Problems in Pressurized Cabin Stress
ness, these moments are so damped at 6.5" from the seam. Analysis.
The next important consideration is an appreciation of Because of functional requirements over
the magnitude of these secondary bending stresses. For the and above those of a simple pressure vessel,
case of the pressure vessel of Fig. A16-5, the meridional the pressurized cabin shell of an airplane has
stresses are increased about 3<Y,0 at the point of maximum a number of stress analysis problems peculiar
moment, while the tangential stresses are increased only to its configuration. Several of the more
about 30/0, Fortunately, in this class of vessel, the tangential general of these will be considered here.
stresses are the ones designed by (they are twice as great as
the meridional stresses) and hence the secondary stresses
have little importance for this case (see Chap. 11, pp. 389-<;22 DISTRIBurION OF STRESSES BETWEEN SHELL
of reference 2). In other configurations one is not always so
fortunate, and detailed analysis may be required. (see refec-
AND STRINGERS.
ences 3, 4, 5 and 6). To stabilize the shell wall in transmitt-
The situation at the seam of the above ing heavy tail loads through the fuselage,
vessel is typical of many seams or boundaries longitudinal stringers are added. These
where elements are joined which would experience same stringers will also help to carry the
different expansions if loaded separately. meridional pressure loads. The skin and
Among such seams and boundaries are those: stringers must, of course, have equal strains
in the longituditudinal_ directions but, because
i-where the meridional curvature changes the skin is in a two-dimensional state of stress,
abruptly. It changes from Rm Ro = they cannot have equal longitudinal stresses:
to Rm = QO at the seam in Fig. A16-5. hence the following analysis.
ii - where a SUdden change in direction of Let the meridional (longitudinal) stresses
the meridian curve occurs. In Example in the skin and stringers be 3m and sL, respect-
Problem 5, above, considerable shell ively. St will be the tangential (hoop) stress
wall bending would be induced near the in the skin. From eq. (1) we again have
seam. In fact, a reinforcing ring
would probably have to be added at the
seam as was done in Example Problem 2, St = P-t-
Ro
above.*
iii - at which structural members of differ- If N is the total number of stringers, each of
ent stiffnesses and different loadings cross sectional area AL' then eqUilibrium longi-
join. In Example Problem 2, the cyl- tudinally reqUires
inder tends to expand the most, the
bulkhead quite a bit less and the re-
inforcing ring, being loaded in com-
pression, tends to contract. Other
seams and/or boundaries of this type The condition of equal longitudinal strai~ in
are those where an abrupt change in the skin and stringers yields
shell wall thickness occurs (addition
of a doubler) or where a shell is fast-
ened into a foundation.
where ~ is Poisson's ratio (= .3 for aluminum).
Good design tends to minimize the magnitude
of the secondary bending stresses by avoiding SolVing these three equations one finds
combinations of elements which would have highly
incompatible distortions. ThUS, the analysis of
Example Problem 3 shows that i f one closes a
circular pressure cylinder with an elliptical
bulkhead in Which a = 2 b, compressive tangen- = P Ro (1 + 2 ~ a) (1 + .6 a)
tial stresses would develop in the bulkhead. In 2t (1 + a) (1 + a)
such a case the bulkhead would tend to contract
radially while the main cylinder would tend to P R o (1 - 2 j.l) _ p R o .4
expand as always. Thus, the shear and induced sL = 2 t (1 + a) - 2t (1 + al
moment at the seam would be aggravated, prodUC-
ing (as it happens) a tangential maXimum stress where a =
NAL/2n Rot is the ratio of total
stringer area to skin area. A little study will
show that t(l + a) is a sort of "effective shell
* certain details of the design of such reinforcing rings are wall thickness": it is the result of taking all
given in the codes and standards. the cross sectional area (skin plus stringers)
A16.7
and distributing it uniformly around the peri- ations of these effects in a specific case are
meter. On this basis, the results are a little to be found in references 2 (PP. 395-406), 6
disappointing: the stringers are carrying only and 8.
40% of the stress one might expect if the net One proposed solution to the ring-shell
longitudinal load (p n R~) were distributed interaction problem is the floating skin.
evenly over the entire cross sectional area Basically, the idea is to reduce the radial
(2 n Rot (1 + a)). stiffness of the connection between the shell
The meridional skin stresses are reduced by and the rings so as to allow the shell to ex-
the factor (1 + .6 a)/(l + a) from what they pand freely under the pressure loading. The
would be without the stringers. For structures connection between ring and skin must still re-
of usual proportions this decrease may amount to tain its shear stiffness so that ring loads may
20 to 30% but clearly can never exceed 40%. In- be transferred to the shell wall by tangential
asmuch as the bending stresses due to tail loads shear flows. Fig. A16-13 shows the basic idea
will be superposed on these pressure membrane of the radially flexible connection. Many
stresses, the reduction is certainly beneficial*.
INTERACTION BETWEEN RINGS AND SHELL.
Because of the necessity for transmitting
various concentrated loads from within the cabin Fig. A16.13 Ring skin cross
and from the wings and tail to the main shell section showing the action of
and because it is also necessary to provide some a radially flexible connection.
lateral restraint which will stabilize the
stringers and skin against an overall instabilit
failure, the pressurized fuselage of an airplane
contains a considerable number of rings and variations of this type of "mount" suggests
frames distributed along the length of the shell. themselves, some of which may have merit for
These rings are seldom, if ever, spaced closely other reasons. For instance, the transmittal
enough such that they can be considered effect- of wind and other vibration noise into the
ive in carrying a part of the hoop stresses (in cabin of a high speed transport is a problem
the way the stringers were effective in carrying which might be treated simultaneously by the
part of the meridional stress). Rather, they proper choice of connection between the ring
act more like widely spaced restraining bands and the shell.
having the effect shown exaggerated in Fig. DOORS AND WINDOWS.
AI6-12.
The various cutouts in the shell of a
pressurized cabin reqUire special considera-
tion if an excessive weight penalty is to be
avoided.
Consider the panel removed from the pres-
surized cylinder of Fig. A16-14a. Following a
In examining the figure to determine what Fig. A16-16 shows the typical cutout
sort of canceling stress system must be supplied, structural arrangement. While analytical
we see that the tangential hoop stresses border- approaches have been tried, it is probably
ing the cutout cannot be canceled by a self- safe to say that the true elastic stress dis-
equilibrating set since they have a radial com- tribution in such a configuration cannot be
ponent. However, the radial component of these computed. The necessity for avoiding high
stresses will actually be supplied by the door intensity stress concentrations (With their
or window pressing outward against its frame. attendant fatigue likelihood) makes empirical
Hence, it is only the component of the hoop information most useful in such cases. On
stresses along a chord which need to be canceled the other hand, a simple rational analysis,
(Fig. A16-14b)*. based on principles outlined above, will very
The immediate problem becomes one of de- likely suffice for a static strength check and
signing a structure to effectively support a set for most design purposes.
of uniformly distributed self-equilibrating The above discussion has concentrated
stresses acting in the plane of the chord con- attention on the problems of carrying the hoop
necting the upper and lower edges of the opening stresses around a cutout. The longitudina-l---
(Fig. A16-15a). pressure stresses, while being smaller them-
selves, are intensified by bending stresses
from the tail loads. Hence, the longitudinal
stresses across the cutout may make this con-
dition (or the combination) most severe.
(a)
Fig. A16.15
A17. I Introduction.
It was seenin the last chapter that thin with another such, we write the equations down
curved shells can resist lateral loadings by by a direct appeal to past experience and
means of tensile-compressive membrane stresses. intuition.
As will be seen later, thin flat sheets, by de- Fig. A17-1 shows the differential element
flecting enough to provide both the necessary of a thin, initially flat plate, acted upon by
curvature and stretch, may also develop mem- bending moments (per unit length) Mx and My about
brane stresses to support lateral loads. In the axes parallel to the y and x directions respect-
analysis of these situations no bending strength ively. Sets of twisting couples Mxy (= - Myx)
is presumed in the sheet (membrane theory). also act on the element.
In contrast to the membrane, the plate is
y
a two-dimensional counterpart of the beam, in z
which transverse loads are resisted by flexural
and shear stresses, with no direct stresses in
?' yx
x2w+ ~ R
a w) -
2
(1)
for instance). Rather than labor the subject
(2)
* As will be seen later, the presence or absence of mem-
brane stresses is not wholly dependent upon the magnitude
of deflections, but is also determined by the form of de-
flection surface assumed by the sheet (in turn dependent upon where
the shape of boundary and loading).
The twist of the element, a 2 w/axay
** the assumptions implicit in the following analysis are (= a~/ayax) is the change in x-direction-
spelled out in detail in Art. Al 7. 5, below. slope per unit distance in the y-direction (and
A17.1
A17.2 BENDING OF PLATES
(and visa versa)***. It is proportional to the In a similar manner, a moment summation about
tWisting couple Mxy • A careful analysis (oee the x axis yields
references 1 and 2) gives the relation as
a2 w (5 )
My
.JC
= D (1 - II)
r-
-axay (3 )
Fig. A17. 2. The differentials are increments which "Hooke's Moment- (a"w a"w)
Mx = D ax" + I-' ay"
should be written more precisely as, for instance, Law" Distortion
d~y ~ (a~y/aY)dy. Relation ( a"w a"w)
My = D ay" + I-' ax" M = EI ~
a"w
The next relations are obtained by summing Mxy = D (1 - 1-') aXdy
moments in turn about the x and y axes. For ex-
ample, we visualize the two loading sets of Fig. aMx aMxy
'i.x=-+-
A17-2 acting simultaneously on the single ele- ax ay
Equilibrium Moments v= ~
ment, and sum moments about the yaxis. ~ aMxy d x
Qy = +
ay ax
Mx dy + (Myx + d Myx) dx + (Qx + d Qx ) dx dy = _ aQX + aQy
Forces q - ax ay q= ~
d x
(Mx + d Mx ) dy + Myx dx
Dividing by dx dy and discarding the term of Finally, one very important equation is
higher order gives obtained by eliminating all internal forces
(Mx , My, Mxy , Qx, Qy) between the above six
Q =aMx_~
equations. The result (which the student
x ax a y should obtain by himself as an exercise) is a
relation between the lateral loading q and the
or,
deflections w*:
Qx = a Mx + a Mxy _ (4 )
ax ay (7 )
*** If w,
the deflection function, is a continuous function of
x and y (as it must be, of course, in any technically im-
portant plate problem) then at each point a 2 w/axdy = * the corresponding equation for a simple beam is
a~;aydx, as is proven in the calculus. q/EI = d.y/dx •.
A17.3
m, n = 1, 2, 3,
M _ _
..x n 2(illa ~mn
2 + nb 22) 2 (illa 22 + I-l n
b 22) S In
. ill- aT1 -X S In. n- bT1 -y
• PP. 128-132.
where the coefficients a and ~ are given in
Table A17.1 for the four most Common edge
• PP. 194-201, 209-211.
conditions. A17.5 Deflection Limitations in Plate Analyses.
In the introductory remarks of this chap-
ter it was stated that a plate may be distin-
gUished from a membrane by the small order of
its deflections (on the order of a few tenths
of its thickness). We will re-examine this
statement here to show that this is not so
much a definition as it is an accuracy limita-
tion imposed by one of the assumptions made in
the plate analysis.
Fig. A17.4 There are several familiar assumptions
from beam theory which, of course, carryover
here, inasmuch as the plate analysis resembles
Similar presentations may be made for many the beam analysis rather closely. These "beam
dozens of other cases. With the readyavaila- theory assumptions" are:
bility of comprehensive catalogings of these
problems in references devoted to the purpose, i-elastic stresses only are presumed,
there appears to be little Virtue in duplication ii - small slopes (so that a 2w/ax 2 and
here. Hence the following list of selected a~/ay2 are good approximations to
references is presented. Additional references the curvatures),
are to be found in turn within these works. We iii - at least one transverse dimension
note that, because of the linearity of the plate (length or width) be large compared to
bending problem, superposition of solutions is
the thickness so that shear deflections
possible to extend even further the usefulness may be neglected.
of these extensive listings.
AI7.5
o• J} ~ (~~ 'J
+ dx - a
We note that the limiting case, alb = 0, cor-
responds to the one-dimensional case analyzed
earlier. Unfortunately, an extrapolation of
these two-dimensional results to that limit
does not show agreement with the one-dimen-
=~ J1:/ dx •
siDnal result. Presumably the discrepancy may
be traced to the excessive influence of inac-
curacies in the assumed deflection shape of the
membrane as used in the approximate two-dimen-
Substituting through the use of eq. (15) and in- sional solutions.
tegrating we find Experimental results reported in reference
4 show good agreement with the theory for
= q" a" square panels in the elastic range.
6 24s"'t"
TABLE A17.2
Now by elementary considerations
Membrane Stress and Deflection Coefficients
sa
Et
2
2
---
10 I------i-=-'£----:;oo"""l'-----+----+-----i
The sum of these two pressures gives the total
lateral pressure, called simply, q.
O~ _ _-'- .L-__:_--'---~.J.---':":'__
q =-1a Et" 1 Et
- - b - WMAx + ~ - 6 WMAJC
"
- - - - - - (20)
7
Fig. A17. 7 Large-deflection theories' mid-
a6 (_) 4 n1 a panel stresses; simply supported square panel.
a
Because of the obVious desirability of
Eq. (20), we see, is based upon summing the in- using the results of the more exact theory, some
dividual stiffnesses of the two extreme be- of these are presented in Table A17.3. The
havior mechanisms by which a flat sheet can treatment of additional cases (other types of
support a lateral load. No interaction between edge support) may be found in reference 6, pp.
stress systems is assumed and, since the system 221, 222.
is nonlinear, the result can be an approximation TABLE A17. 3
only. Large Deflection Rectangular Plate Coefficients
Eq. (20) is best rewritten as (Uniform Pressure Load, q. Simply Supported Edges)
Deflection
Pressure Coefficient, qa 4/Et 4
wit
12.5 25 50 75 100 150 200
1 sb :YEt Ii! 3.191 5.158 7.770 9.720 11.339 14.055 16.364 18.424
Fig. A17-6 shows eq. (21) plotted for a
5MbJil/Et~ 0.609 1.377 2.683 3.792
square plate using values of a and n 1 as taken 4.785 6.542 8.103 9.533
from Tables A17.1 and A17.2. Also plotted are wit 0.6958 0.9463 1.241 1.440 1. 596 1. 840 2.033 2.196
the results of an exact analysis (reference 5).
2 tooo sb 2 lEt 2 4.871 7.164 10.26 12.62 14.61 18.01 20.93 23.55
As may be seen, eq. (21) is somewhat conserva- 2 2
tive inasmuch as it gives a deflection which is sMb /Et 1.293 2.403 4.155 5.612 6.911 9.220 11. 28 13.16
o l..-~:::::::::L':""'-_--l..._ _. J_ _~ L
o 0.5 wit 1.0 1.~ 2.0 * but using n2= .260 in eq. (19), This value gives the
stresses at the center of a square panel whereas n2 =
Fig. A17. 6 Deflections at the midpoint of a simply .356 in Table A17. 2 is for stresses at the panel edge.
supported square panel by two large-deflection
theories.
A17.8 BENDING OF PLATES
PART I
ELASTIC AND INELASTIC INSTABILITY OF COLUMNS
A18. I Introduction. Thus the total bending moment at section z
will be:-
Part 1 of this chapter will be confined
to the theoretical treatment of the instability Qaz on upper portion - - - - ( d)
11 = Pu + --1-
of a perfect elastic column and an imperfect
elastic column. The column is the simplest of
the various types of structural elements that 11 = PU + Q(l-a) (l-z) on lower portion (e)
1
are sUbject to the phenomenon of instability.
The theory as deYeloped for columns forms the From mechanics of simple bending, we have
basis for the study of the instability of thin the deflection equation,
plates, which subject is treated in Part 2.
d 2u 11
A18.2 Combined Bending and Compression of Columns.
dz 2 = -
Ely --- - - - - - - - (1)
Consider a column with one end simply
Thus the deflection u(z) of the column
supported and the other end hinged (Fig. A18.1)
is,
under the simultaneous action of a compressive
load P and a transverse load Q. Without the d 2u Qaz
load P the bending moment due to Q would be:- EI dz2 = - PU 1
, (0 :: z :: 1 - a)
p
d2 u (l-a) (l-z)
EI dz 2 = - Pu - Q 1
/
(1 - a ::= z ::=. 1) - - - (2)
a /
1
1- j---++--Q If we introduce the notation,
I - (3)
I
\ The general solution of eq. (2) is:
\
\ Qaz
u = C], cos Kz + C2 sin Kz - Pl '
-4,U (O:::zl-a) - - - - (4a)
Q sin Ka _ Q sin K(l-a) the body would cause only infinitesimal changes
C,,= PK sin Kl ' C.. - - PK tan Kl - - - - (g) in the displacements and the body recovers if
the added loads are removed. When the dis-
SUbstituting (f) and (g) into eq. (2), placements are continuously increased with
we obtain: little or no further increment of loads, the
system is unstable. If the body will remain
u = Q sin Ka i Kz Qa in the displaced position after the removal
PK sin Kl s n - Pl Z of the disturbance, the body is said to be in
(0 5 Z 5 1 - a) - - - - - - - - - - -(5a) neutral equilibrium. Having these definitions,
we will not investigate the behavior of the
column before and after the critical load is
u = QPKsinsinK(l-a)
Kl
i K(l-)
s n Z
Q(l-a) (l-z)
reached.
Pl
(1 - a 5 Z 5 1) - - - - -(5b) p A B P
~ ~
~~ ~/> z
If load Q is applied at the middle of the s~~ _
column the maximum deflection is:-
Q tan Kl Fig. A18.2
2 Ql
Umax = 2 PK - 4P - - - - - - (6 ) Assu~e, as shown in Fig. A18.2, that a
simply sunported column loaded by an axial load
Kl
It is obvious that for:2 =2n ' P is bent by a small disturbance. If the
deflection at a distance s along the deflected
KI - axis is u, the bending moment due to P is Pu.
tan """"2 00. Thus the maximum deflection of From engineering theory of bending, we
know that,
column becomes infinite for KI = n and from
EI
eq. (3). If = -M , whence
= n"EI EI
Pcr - - - - - - - - - - - - (7) If =-Pu - - - - - - - - - - - - -(8a)
The exact expression for the curvatuve of
Equation (7) is an important equation the neutral axis is:-
derived first by Euler. It gives the critical 1 _ dG
compressive load which causes infinite deflect- R- ds ' where s is the arc length of the
ion in a column and it specifies the ultimate
strength of a column in compression. deformed aXis, and G the angle between the
tangent to the curve and the z axis. Thus,
It is obvious-from eq. (7) that Euler's
critical load is independent of the magnitude EI dG + Pu =0
of the transverse load Q. It seems, therefore, ds - -(8b)
that even in the absence of the transverse
load Q, the maximum deflection becomes infinite Differentiating (8b) with respect to s
under the action of only a compressive load as
given by eq. (7). and since ~~ = sin G, we obtain:
d"G
A18. 3 Elastic Stability of a Column. EI -dB" + P sin G =0 - - - - - - - -(8c)
The above conclusion as to the critical Multiplying (8c) by dG and noting that:-
load was based on purely mathematical reasoning.
We have found a critical value of a compressive
load which causes infinite deflection. ~:~ dG = ~~ d (~~) , and integrating
4K 2
Now let k = ~
EI - - - - - - - - - (8e)
P PlIO
Pcr = n 2 EI =7 (8n)
n
where K = ! 2 dQ is called the Fig. AIB.3
o J I_p2 sin 2 0
IL... m
complete elliptic integral of the first kind,
~
one when 0/1 = 0 and the other (point A) for (b) At P/Pcr = 1, or at point (A), a bifurcation
which 0/1 ~ O. The column thus has two of equilibrium occurs and the column starts
possible equilibrium forms, one straight and to acquire two possible neighbor positions
one bent. of equilibrium, the straight and the bent.
Let us now assume that at 0 = 0, the (c) Above P/Pcr = 1, the column has two possible
column is displaced by a small disturbance equilibrium positions 0/1 = 0 and 0:1./1 ~ O.
and acquires a deflection 0:1.' For p:I. -< Pcr ,
we see from Fig. 3 that m> m:l.. Thus p:I. is Thus as far as initiation of instability is
not sufficient to maintain the column in concerned, the Euler load as given by Eq. 7 can
equilibrium in the bent form and it will spring be considered as the critical load. The question
back to its straight form. Thus for p:I. c: Pcr , arises whether this load has a practical use
the straight form is stable. for design purposes. A logical design criterion
is obviously the maximum load which a column can
If p:I. > Pcr , then m:l. > m. Thus m:l. will sustain. We observe from Fig. A18.4 that the
bend the column still further. This means load P increases for increasing displacement o.
that if p:I. > Pcr , the straight form of This behavior is due to the development of large
equilibrium is unstable. The column will deflections due to bending. However, over a
continue to bend until m:l. becomes equal to m considerable range of deflections 0, the P -+ 0
(point A in Fig. 3). If the column is dis- curve is practically horizontal (for instance,
placed further from A, the deflection becomes between points A and B the ratio 0/1 varies
larger than O2 and m ~m:l. at the new position. from zero to ~ 0.4). For such large deflections
The column will spring back to point A. Point for which the column load does not change
A is therefore stable. practically, it is obvious that the column
ceases to function properly. Therefore, from
At P = Pcr , the m:l. versus 0/1 line is this point of view, the Euler load can be con-
tangent to the m curve at the origin. There- sidered that which characterizes the maximum
fore, for an infinitesimal disturbance, the strength of the column.
column will remain in equilibrium at the
displaced position since for such small The rising part of the curve BD holds as
disturbances m:l. remains equal to m. The column long as the material behaves elastically. At
is therefore in neutral equilibrium. some point B, however, inside the almost flat
portion of the curve, the inner fibers of the
A18.4 The Failure of Columns by Compression. column acquire maximum stress equal to the
yield stress. If we carry out an elastic-
In discussing the stability of a column plastic analysis of the subsequent behavior,
in the previous section, it was shown that we observe that the curve drops almost
below the critical Euler load (Eq. 7), the immediately. Again this maximum load PB is
straight form is stable, above the Pcr the very near the Euler load. For design purposes,
bent form is stable and at Pcr the equilibrium therefore, the Euler load, which is a buckling
is neutral. By plotting the curve p/pcr versus load, is a very good approximation to the
0/1 as shown in Fig. A18.4, we observe the Ultimate load which the column can sustain.
following behavior.
Another argument will confirm the above
(a) Below P/P cr = 1, there is only one conclusion. In discussing the buckling of
equilibrium position, 0/1 = O. columns in the previous paragraphs, we have
assumed that the column is initially straight,
centrally loaded and made of
piper homogeneous material. Actual
columns, however, are imperfect
due to initial crookedness (for
instance, due to unavoidable
tolerances in their manufacture),
due to slight load eccentricities
t B and due to lack of complete
10
• A- - '-'I .... ..... homogenity. Therefore, a certain
I ......... amount of bending is always
I u present even for small loads.
I
I Let us now examine the
I
J behavior of such initially im-
1~/l perfect columns by assuming a
o'1r----::-::--=-=--=-=--...l..::---:-~0 11
O. I 0.2 O. 3 0.4 certain initial deflection 11"
Fig. of the column axis (see Fig.
Fig. A18.4 A18.5 A18.5). For small deflections,
A18.5
the change of curvature due to sUbsequent approaches the horizontal line PIPer 1 =
bending (after loading) is:- asymptotically. This curve, however, is valid
for small deflections for which the approxi-
° (l)R =~
dz
_ d l};o
2
dZ
mation:-
is valid.
In the differential equation of deflection
one can prove that llR is the change of
curvature which for an initial straight column By a treatment similar to that in the
coincides with the curvature itself. Thus in previous paragraph, we will find that for
the present case, where the bending moment is large deflections the load deflection curve
Pu, the equation of deflection becomes:- raises after the point I (curve FIH). Due to
the onset of plasticity, the actual curve
drops at the point 11. (curve FII1.H1.). The
failing load at 11. can be either greater or
smaller than Per' but it is usually very near
to it.
Let us express Uo in Fournier series:-
In the above discussions we have shown
- - - - -(10) that for all practical purposes the Euler
buckling load can be considered as the ulti-
mate load which a real or practical column
Substituting (10) in (9), we find the can sustain. Besides its closeness to the
solution which satisfies the boundary actual ultimate load, the critical load can
conditions (u = 0 for z 0, Z = 1) is:-
be easily calculated from equation (7) with-
00
out the necessity of carrying out a lengthy
u =Z on sin n~ - - (lla) calculation which will include the initial
n=l imperfections and plastic effects.
6n
"un '" _---':..::....,-_ It should be noted, however, that the
were
h - - - - - - - - - (llb) buckling load given by equation (7) is valid
1 - PIPn
when the uniform stress due to a compressive
load (0 = PIA, where A is cross-sectional
area) is below yield stress. If 0 is above
the yield stress, the theory of plasticity
predicts another value for the buckling load.
The deflection of the column at the center Referring now to equations (11) we find:-
is:-
Pn =n 2
Pcr (p cr from equation 7)
- -(12)
6n
If we plot the deflection versus the load on = ---=-~~P~=
1
we obtain the curve (Fig. A18.6), which n 2 Pcr
~ _ 9/8 etc.
c
i\
H
1.0 A
bending-buckling is:-
The buckling load for other end conditions
4 can be derived in similar manner.
d u + k" d"u = 0 - - - - - - (14)
dz" dZ"
INELASTIC COLUMN STRENGTH
and the general solution is:-
AlB. 6 Inelastic Buckling. Introduction.
//~IOading
coefficients of C1 , C", C3 and C4 is equal to
zero. From this equation, we calculate the
buckling load.
/
/
For example, in a simply sUDported beam, /
d"u
(u = 0, dz" = 0, at both ends), the end /
/
Fig. AIB.7
C" =0
AIB.7 Perfect Column. Reduced Modulus Theory.
C1 sin kl + C" cos kl + C3 1 + C4 =0
Let us assume that the perfect column is
For buckling we must have:- first compressed uniformly up to the stress o.
To study the critical value, ocr, of for which °
Al8.7
- - - - (23)
00,.
where,
Q~ = L0 a~ h~ dA , Q,. = La,.
0 h,.dA - - - (24)
are the moments of the cross-sectional areas
~~~~~~E to the right and left of line a a~.
I Fig. Al8.8b From eq. (21) we obtain,
la~ r
I I
----·~IE-- a~ ----+1 -I d"u
E dz" = Pu - - - - - - - - - - (25)
Fig. Al8.8a
where,
plastic stress range and that the compressive
stress prior to instability be a. To consider
the condition of buckling, let the column be E= - (26)
slightly deflected transversely. The stress
on one side of the column will then increase E is the so-called reduced modulus, and
due to the bending following the stress-strain I~ and I,. being the moment of inertia of the
curve, while on the other side the stress will two sides.
decrease and will therefore follow the un-
loading elastic line shown in Fig. A18.8b. We observe that the position of the
neutral axis in terms of the axial stress is
For small changes of the stress, on the given by eq. (24), while the buckling eq. (25)
first side, the variation of stress is related is similar to the elastic buckling eq. (14).
to the variation of strain by:- However, the value of K here is not given by
eq. (3), but by,
- - - - - (18)
where Et (a) is the slope of the stress-strain
curve at stress a. On the second side the
K=Af - - (27)
changes will follow the elastic relation, Therefore all the results of the previous
that is, analysis will be valid for the case of in-
elastic buckling. For instance, for a simply
Al8.8 THEORY OF THE INSTABILITY OF COLUMNS AND THIN SHEETS
(28)
d
000
I ...-CoN"o{RE-'ENGESSER (~." RQ
70
\ I
.".....IJ
~ 0000
\
,~
\\.
'..;\ ~EULER
k
\ 000
ENGESSER- i"..A
(TA-r MDIJ)
t--~_
,
~~
\
\ 000
~
i'- "'""'"-
\ J
0
20 ~ ~ ~ m lro I~ I~ ~
r-..
~
--
2~ 240
"''''~
Coillmn lest TUults on 17S-T solid round rod (0.282 in. diameter).
,.. ,,10m' nl Mea Iils LtJI1rI
(> Fig. Al8.11
0 '-or ~l~
70000
o FlAT END TESTS
.ROUt«) END TESTS
000
Fig. Al8.l0
ooo~ ~= ~
7f~ It:
modulus, even though it does not actually ~ ~
define an unstable configuration, it repre- .......
sents the lower limit of a spectrum of possible ~ ~ ~ r.::::......
buckled configurations, the upper limit of
which is the reduced modulus load which
..~
~
.~
~
corresponds to infinite deflections.
Thus to summarize, sufficient experi-
mental results are available to show that the
'0
0
~ ~ ~
.......
~
~
m
-- f0&-
lro ~
...........
~
r--
~
- ~
r---
~ m
failing stress of a column in the inelastic SLENDERNESS RATIO, (LIf)
('"h",,,, ,,.111 rt'Imllttmt 17:':'-Ta"glt' (O,{jll.I'!ll~ in. iy ~ ;',.).
range can be found by replacing E by the
tangent modulus Et in Euler's equation, or, Fig. Al8.l2
virr-
References:
n 2 Et
= r = A - - - - - - - (34) Ref. 1. Engesser F., Schweezeriche Bauer
(L/r)2 '
Zeitung. Vol. 26, p. 24, 1895.
Figs. A18.11 and 12 show how experimental Ref. 2. Shanley, F.R., Inelastic Column
results check the strength as given by the Theory, Jour. Aeronautical Sciences,
Euler equation using the tangent-modulus Et. 1947, p. 261.
A18.10 THEORY OF THE INSTABILITY OF COLUMNS AND THIN SHEETS
PART 2
y
Fig. 2b
For small de- Let Fig. (2a) represent a thin rectangular
Fig. I flections, the plate loaded by uniformly distributed bending
A18.1l
~-----:;:----,>x
Mx = -D 2
(aaxew + V a
2
w)
aye (5a)
z
Fig. 3
The corresponding strains are: Equations (5) give the moments per unit length
for pure bending and tWisting of a plate.
a
2
w z _ z a2
w -_-L
EX = - z ax 2 = Rx ' Ey - - ay 2 Ry '
A18.12 THEORY OF THE INSTABILITY OF COLUMNS AND THIN SHEETS
aMxy aMx
--+---Qx=O
ay ax --------(6c)
(7 )
1 a"w a"w"
V0 .. "2 D (ax" + ay") -
ow a"w
...-ay+a xOy dx
a"w a"w
2(1-V) [ ax" • ay" - (axay)
a"w "J - - - - -(11)
(12c) Z Z Cum
m=l n=l
In the following solutions for various 1 00 00
edge conditions will be developed. -D Z Z amn sin mroc sin nny - - - - - (14b)
m=l n=l a b
1. Simply supported rectangular plates
This relation is identity if:
Let a plate with sides a and b and axes
x, y, as shown in Fig. 6, be simply supported
around the whole periphery and loaded by a
distributed load q = f (x,y).
- - - (14c)
We can always express f (x,y) in the form
of a double trigonometric (Fourier) series: In the case of a load qo uniformly distributed
over the whole surface we have:
00 00
- - -(13b) n"mn - - - - - (1 5a )
169 0
m+n_
DO l
Wmkx - nS D ~ h (-1) 2
The boundary conditions are:
m=1,3,5 n=1,3,5
W = 0 Mx = 0 at x = 0, x = a
W = 0 My = 0 at y = 0, y = b or:
- - - - - (15c)
(1) W = 0 (2) a"w = 0 at x = 0, x = a
ax" This is a rapidly converging series and a
a"w _ satisfactory approximation is obtained by
(3) ~ = 0 (4) ay" - 0 at y = 0, y = b taking only the first term. For a square
A18.15
solution. 1
The expressions for bending and tWisting
moments are not so qUickly convergent. To y
improve the solution another series solution
can be developed as follows: where for symmetry m = 1,3,5 •
b) Levy alternate single series solution: This equation can be satisfied for all values
of x if:
The method will be developed for uniform
load qo = const. Levy suggested a solution in
the form:
00
2(am tan h am + 2)
Am =-------- la 00
m-l
rr 6 me cos ham (M)
x max
= qa8 + (l-V)qa"n la Z (-1) 2
2
m=1,3,5
Bm=----- - - - - - - - - (21b )
Thus:
00
m-l
W
4qa 4
=-- Z
2-
[1 _ (am tan h am + 2 ) (-1) 2
neD m=1,3,5 me 2 cos hOm
~a fa b Vodxdy = n :b D
4
- - - - - -(21e)
2 cos ham Vi = Z Z emn la
m=l n=l
This series converges very rapidly. Taking a la
square plate, alb = 1, we find from (21a): (m + .!f-)la_ (23)
a" b"
We observe that only the first term of the is found from (23)
series in (21e) need to be taken into con-
sideration.
P sin m;:t . sin n~YJJ Let us consider the case of a uniform load q.
we write the deflection in the form:
6Cmn = ° -(26a)
Since 6C mn is arbitrary its coefficient must where Wi is the deflection of a simply
be zero. Thus: supported strip of length, a, which for the
system of axes of Fig. 8 can be written (see
sin ~
Levy's method in previous section):
4-P a . s i n nnn
b
Cmn = ----"'--,;2---;;-2-"---
n 4abD rl!!- + ..!1.-) 2
'a 2 b2
H-a-7
r-.,..--------.-_x
b
By the conditions (25c) we find:
t Bm = _4_.
nllm ll
(3+J)) (l-V) cos h" Bm + 2V cos hBm -J)(l-VIBm sin h Bm-(l-J)")
(3+V) (I-V) COS h ll Em + (l_V)"Bm ll + (l+V)2
- - - -(268)
em = _4_. ~(3::.::+JJ:..c)-C.:(1:.-:-V,!-):,,:S h Bm~C:.:::OS:...:h:.:Bm~+J):.:-(c:.-
n ::-
ic:: 1 +..:...J))c::.S:.::inc:::h~Bm,,--=-:V(-=-l-..:...V:.::IBm=CO_S..:...h-=.!Bm,,---,--(l--=-V--,-)"~Bm
11 II mil (3+11) (l-V)cos h iii Em + (l_V)liI Elms + (1+V)liI
Fig. 8
y
Assume that the edges x and x a are =° = SUbstituting Am, Em, Cm and Dm in eq. (26d) we
simply supported, the edge y = b free and the find the deflection. The maximum deflection
°
edge y = bUilt-in (Fig. 8). In such a case
the boundary conditions are:
occurs at the middle of the free edge.
A18. 14 Combined Bending and Tension or Compression of
Thin Plates.
w = 0, ~:'::" = 0, for x = 0, x = a (a)
In developing the differential equations
w = °, aw = 0, for y = (b) of eqUilibrium in previous pages, it was
2
ay
a 2 w = 0,
° - - (25)
assumed that the plate is bent by transverse
loads normal to the plate and the deflections
(a w+
ax2 a y 2) were so small that the stretching of the middle
plane can be neglected. If we consider now the
t~~~ + 2(1-V) = ° for
3
a w] y =b (c) case where only edge loads are active coplanar
ax2 ay
A18.l8 THEORY OF THE INSTABILITY OF COLUMNS AND THIN SHEETS
Nx = hO x , Ny = hOy
- - - - - - - - (27) y
Nxy = Nyx = hoxy = hoyx Fig. lOa
Consider an element of the middle surface
dX, dy (Fig. lOa). The conditions of force
eqUilibrium in the x, y-directions are given
byeq. (28). Consider now the projection of
the stresses Nx , Ny, Nxy in the z-direction:
y
and neglecting terms of higher order:
Fig. 9
(N a2w+~ aw)
The equations of equilibrium in the absence x ax 2 ax· ax - - - - - - - - (a)
of body forces can be written now in terms of
these generalized stresses Nx , Ny, Nxy by (b) Projection of Ny: By similar argument
SUbstituting from eq. (2 7 ) to the equations we find that this projection is equal to:
of eqUilibrium
(N a2~ + aNy~) - - - - - - - - - - - (b)
aNxy y ay ay ay
ax - 0 - - - - - (28)
In this chapter the problem of instability and neglecting terms of higher order:
of plates will be examined.
(N ~+ aNxy aw) dxd - - (c)
When the edge loads are compressive and xy axay ax· ay y
give rise to additional bending moments eq. (8)
must be modified. Simil~rly we find the projection of Nyx = Nxy :
A18.19
(Nxy ~+
axay
aNxy . aw) dxd
ay ax y - - - - - - - (d) ViN =~ / / (NXE~ + NyE; + Nxyfxy) dxdy =
Thus in eq. (6a) the terms given by (a), (b), 2~JJ (N~+N;-2IJNxNy+2(1+V)Nxy") (a)
(c) and (d) should be added (divided of course
by dxdy): During bending due to transverse loads
or/and due to buckling we assume that the edge
aQ.x + aQ.y + + 1\1 a"w N a"w + 2N a"w loads and consequently Nx , Ny, Nxy , remain
ax ay q "x ax" + Yay" xy axay +
constant. Its variation is thus zero and we
(aNX + aNxy) aw + (~+ aNxy)aw = 0 (e) do not consider it in the following. Let us
ax ay ax ay ax ay apply now the transverse load that produces
bending. (We can also consider bending due to
But due to the equation of equilibrium (28) the other transverse disturbance, which is the
two terms inside the parentheses in (e) are case of buckling). If u, v, are the displace-
zero. Thus: ments of the middle surface due to the coplanar
loads (which are assumed constant across the
aQ.x + ~ + q + N a"w + thickness) and w the bending deflection of the
ax ay x ax" plate it can be shown that the strains are:
a"w a"w
Ny ayr + 2Nxy axay =0 - - - - - -(29)
EX
= ~+l(aw)"
ax 2 ax
Eq. (29) replaces eq. (6a) when edge loads are
present. Eqs. (6b) and (6c) are, however, still Ey
=~ay
+.l (aw)"
2 ay
valid since they express moment equilibrium
of the element dxdy in which the contribution '0
xy
= au
ay
+ av + aw
ax ax·
aw
ay - - - - - - (b)
of Nx , Ny, Nxy is zero. Thus eliminating Qx ,
Q.y between (6b), (6c) and (29) we find: Let us apply now bending with constant coplanar
stresses. Due to stretching of the middle
surface the energy is:
Obviously the first two integrals represent buckled configuration (deflections different
the work done by the edge loads while the from zero) is possible. It was found in the
second integral is zero due to the equilibrium case of column that this latter solution
equations (28). Thus the work of the edge (Euler's solution) approaches asymptotically
loads is: the first at the limit where the deflections
become extremely large but for even small
deflections the edge load acquires a value
very near the Euler's critical value. The
latter technique is mathematically more
We assume now that for small deflections convenient and it gives for plates also a very
the stretching of the middle surface of the good estimate of their compressive strength.
plate is negligible. (This is the so-called In the following we shall use this latter
inextensional theory of plates). In this case approach by assuming a plate with edge loads
by zeroing the strains in eq. (b) and substi- and no transverse load. Eq. (30) becomes in
tuting in (f) we find: this case:
(32a) (33)
In the strain energy expression, eq. (a) By solving eq. (33) we will find that the
the first two terms cancel each other and the assumed buckling made is possible (w 1 0) for
strain energy is due only to bending: certain definite values of the edge loads, the
smallest of which determines the critical load.
Vi
2
= 21 D!! l,(aax2w2 + aay2
w)2 _ The energy method can also be used in investi-
gating buckling problems. In this method we
2 assume that the plate is initially under the
a w ay"
2(1-21) [ ax2. a~ "]} dxdy - - (32b)
a2w - (axay) plane stress conditions due to the edge loads
and the stress distribution is assumed as known.
We then consider the buckled state as a possible
In the absence of transverse loads the work configuration of equilibrium. The change of the
of external forces is simply due to the edge
loads: work is given by eq. (32a). We interpret here
w as a virtual displacement though we do not use
the variation symbol o. Thus the increment of
(32c) work oW is given by (32a) and the increment of
Expressions (32b) and (c) will be used in strain energy oVi is given by eq. (32b). If
solving the buckling problem by means of the OW<oVi for every possible shape of buckling the
principle of virtual work. flat equilibrium is stable. If OW>oVi for a
certain shape of buckling then the flat con-
A18. 16 Buckling of Rectangular Plates with Various Edge
figuration is unstable and the plate will buckle
Loads and Support Conditions. under any load above the critical load. If
oW = oV i , the equilibrium is neutral and from
General discussion this equation we find the critical load. The
critical load therefore is found from the
equation:
In calculating critical values of edge
loads for which the flat form of equilibrium
becomes unstable and the plate begins to
1 ! ! [ Nx (ax)
- 2: aw" + Ny (ay)
aw " + 2Nxy aw aw] dxdy
ax . ay =
buckle, the same methods and corresponding 2
reasonings as for compressed bars will be ~2 !!{(a"w + a"w)" _ 2(1-))) [a w . a"w _ ( a"w )2l}dXd
ax" ay2 ax" ay2 axay J y
employed.
- - (34)
The critical values can be obtained by
assuming that the plate has a slight initial Here w is a certain assumed deflection which
curvature or a small transverse load. These satisfies the boundary conditions (Virtual
values of the edge loads for which the lateral deflection) .
deflection w becomes infinite are the critical
values (see Part 1 for similar treatment in A18.l7 Buckling of Simply Supported Rectangular Plates
columns) . Uniformly Compressed in One Direction.
Another way of investigating such Let a plate of sides a and b (Fig. 11)
instability is to assume that the plate buckles simply supported around its periphery be com-
due to a certain external disturbance and then pressed by load Nx uniformly distributed
to calculate the edge loads for which such a along the sides x =
0 and x =
a. From the
A1B.2l
__ ....,...-------.-.-~ X
a + c + e +
- - - - (b)
b + d + f +
b2
(35b) (35e) becomes:
n=l
z Z m2 Crnn 2
If
a
we plot k against b for various values of the
m=l n=l
integer ill = 1,2,3, ..... we obtain the curves
Here Cmn is arbitrary. We are interested, of Fig. (12). From these curves the critical
however, to find that values of Cmn which make load factor k and the corresponding number of
Nx minimum. To that effect we use the follow- half-waves can readily be determined. It is
ing mathematical reasoning: only necessary to take the corresponding point
a
Imagine a series of fractions: (j)) as the axis of abcissas and to choose that
curve which gives the smallest k. In Fig. 12
ace the portion of the various curves which give
b ' d ' f ' · .... · .. - - - - - - - (a)
the critical values of k are shown by fUll
lines. The transition from m to (m + 1) half-
If we add the numerators and the denominators waves occurs at the intersection of the two
we obtain the fraction: corresponding lines. From eq. (36c) we find:
A18.22 THEORY OF THE INSTABIl.ITY OF COl.UMNS AND THIN SHEETS
k \
7
6
\ '",3\
\ m=2 \
\
1\ m=4,/ "
/ \ / /
,/
\
" ,,"
,/,/
5
~ '\. /"/
4
~ ¥;:,<.../
I
3
I
I I ,I
I
2
r
I ,
I
I
I I
,
,,
I
1 I ,
o ! 0
, Y Fig. 13
o 1j2 2y16 3/12 4 5 alb
22 22 22 22
m n + N B....B.-
Nx~ = D(.!!!..lL. +B....B.-) 2 _ (37a )
Fig. 12 y bB a2 b2
..§:.
b
=J 1 (1+1) = ,f2 Taking any integer m and n the corresponding
deflection surface of the buckled plate is given
by:
from two to three for
mnx . nny
Wmn = Cmn sin -a-· SIn -b- (38)
a =
b
J2 (2+1) = ~
and the corresponding ox, 0y are given by (37c)
and so on. which is a straight line in the diagram 0y' Ox
(Fig. 14). By plotting such lines for various
The number of half-waves increases with the pairs of m and n we find the region of stability
ratio alb and for very long plates m is veFy and the critical combination of ox, 0y which is
large. on the periphery of the polygon formed by the
full lines of Fig. 14.
A18.18 Buckling of Simply Supported Rectangular Plate A18. 19 Buckling of Simply Supported Rectangular Plate
Compressed in Two Perpendicular Directions. Under Combined Bending and Compression.
Let (Fig. 13) Nx , Ny the uniformly distri- Let us consider a simply supported
buted edge compressions. Using the same as rectangular plate (Fig. 15). Along the sides
before expression for the deflections (eq. 35a) x = 0, x = a there are linearly distributed
and applying the energy equation (33) with edge loads given by the equation:
Nx , Ny = constants (which is the solution of
the corresponding plane stress problem) we find:
Nx = No (1 - A ~) - - - - - - - - - (39a)
A18.23
which is a combination of pure bending and We examine for each value of m the solutions of
pure compression. Let us take the deflection the system (38f). Starting from m = 1 and
again in the form: denoting:
00 00
mnx nny = (No)cr
w Z Cmn sin a-' sin -b- - - - (39b) - - - - - - - - - - (39g)
n=l h
A18.22 Methods of Column Failure. Column Equations. The range AB in Fig, 16 is for a range
of Lip values of below 20 to 25, and repre-
In Part 1, the theory of the elastic and sents a range where failure is due to plastic
inelastic instability of the column was pre- crushing of the column, In other words, the
sented, The equations from Part 1 for a pin column is too short to buckle or bow under
end support condition are:- end load but crushes under the high stresses.
This column range of stresses is usually
For elastic primary failure, referred to as the block compression strength.
{T - { t' f { ( 20,000 29 x 10 6
119.56
6
97.62
84.54
I L L L'
12"=L' r I l
IL=~ I
.70L=L'
1-
50,000
60,000
70, 000
80,000
29
29
29
29
x 10
x 10
x 10
x 10
6
6
6
6
75.62
69.03
63.91
59.78
I 2 I
J
89, 600 29 x 10 6
56.49
96,000 27.55x10 6
53. 19
L/4 102,000 26.10 x 10 6
50.22
90
170
7079-T6 Alum. Alloy Hand Forging
160 t" 6.0 In.
17 -4 PH Stainless ~~f!1 Temp. Exp. • 1/2 HR.
(Bars & Forgings) Ftu = 67000
Temp. Exp. up to 1/2 HR. Fey = 59000
Ftu = 180000 50
140 Fey = 165000
40
120
110
30
100
90
<.>
20
'"'
70
10
60
50
20 30 40 50 60 70 80 90
40
L'/P
Fig. 20
10 20 30 40 50 60 70 80 90 100
L'/f
Fig. 19
75 R. T. 150 R.T.
7075 -T6 Alum. Alloy Extrusions
t ~ 0.25 In. steel A1SI 4130, 4140, 4340
140 Heat Treated To F tu = 150000
Temp. Exp. = 2 HR.
F tu =75000 Exp. = 1/2 HR.
130 .145000
Feu = 70000
60
120
110
50
100
<.>
90
'"'
40
80
<.>
70
'"'
30 60
50
20
40
30
10 600 0 F 20
10
10 20 30 40 50 60 70 80 90 100 10 20 30 40 50 60 70 80 90 100
L'/P L'/f'
Fig. 21 Fig. 22
Values of Fe in KSI. The horizontal dashed line is the compressive yield stress. Values
above these cut-off lines should be substantiated by tests.
A18.28 STRENGTH OF COLUMNS WITH STABLE CROSS-SECTIONS
Fig. 23 Solution:
(1) AISI Steel, F tu = 180,000
Since the column may fail by bending about
(2) 17-7PH. Stainless Steel, F tu =210,000 either the X or Y axes, the column strength for
(3) 7075-76 Alum. Alloy bending about each of these axes will be calcu-
(4) A~31B Magnesium Alloy lated. Since the column strength is a function
of the radius of gyration of the cross-section,
(5) 6AL-4V Titanium Alloy
the first step in the solution will be the cal-
culation of Ix and I y , from which p x and Py can
be found.
800
~ (2) Calculating Ix: In Fig. 25 the section will
700~~ be first considered a solid rectangle 2.5 x
2.75 and then the properties of portions (1)
600 ~ and (2) will be sUbtracted.
g + +~
S 500 (3) (1) Ix (rectangle) = l~ x 2.5 X 2.75 3 = 4.32
:><
::: Portions (1) and (2)
';;.,
<.J
400 - -___
~ Ix = -112 x 1.5 x 1.25 3 - 4( .625 x .25 x 1.292 2 ) =
300 -1.29
200 (1 0 of (2) about its x centroidal axis is
negligible)
100
Ix = 4.32 - 1.29 = 3.03 in. 4
Case 2. Same as Case 1, but sUbjected 1/2 Portion (1) = -(1.25x .75x .875 )2 -(1.25x
2
Case 3. Same as Case 2, but for 600 oF. Portion (2) = -(.25x .625x .833 2
)4-4(.25X
3
1.25 /36) =- .488
Case 4. Material 17-4 PH stainless steel,
hand forging at room temperature. Iy = 3.58 - 1.52 - .488 =1.58 in. 4
Fig. 24 }- ----->=n
------
Py = J 1.58/4.375 = .60"
Column strength is considerably influenced
by the end restraint conditions. For failure
by bending about the x-x axis, the end restraint
against rotation is zero as the single fitting
30 bolt has an axis parallel to the x-x axis and
thus c the fiXity coefficient is 1. For
failure by bending about the y-y axis we have
end restraint which will depend on the rigidity
of the bolt and the adjacent fitting and
structure. For this example problem, this
restraint will be such as to make the end
fixity coefficient c = 1.5.
SEc.A-A
Fig. 25
AIB.29
For failure about x-x axis, these end conditions depend on the type of end
fitting or connection to the adjacent structure.
L' = L/ve= 30/VT= 30, L'/px = 30/.83 Furthermore, to save weight columns are often
tapered in cross-section, that is, a variable
=36 moment of inertia along the column length,
For failure about y-y axis, these equations (1) and (2) which assure con-
stant moment of inertia do not apply. Since
L' = 30/~= 24.6, L'/P y = 24.6/.60 elevated temperature conditions effect the
=41 strength of a material, the column strength of
a structural member is likewise influenced.
Therefore failure is critical for bending (See Fig&. 26, 27, 28).
about y-y axis, with L' /p = 41.
Since this Volume 1 is concerned primarily
Case 1. The material is 7079-T6 Alum. Alloy with stress and strain analysis of structure,
hand forging. Fig. 20 gives the failing the subject of column strength is only slightly
stress Fc for this material plotted against considered or presented. However, Volume 2,
the L'/p ratio. Thus using L'/p = 41 and the which deals primarily with materials and
room temperature curve, we read Fc = 50500 psi. strength analys i sand d es ign of structure,
Thus the failing load if P = FcA = 50500 x presents a very complete and broad coverage of
4.375 = 220,000 Ibs. column design, involving over 180 pages of
theory, design charts and many sample design
Case 2. Using the 3000F curve in Fig. 20 problems.
for the same L'/p value, we read Fc = 40,400,
and thus P = 40,400 x 4.375 = 177,000. AIB.2B Some Mechanical and Physical Properties of a
Few Typical Aircraft Materials.
Case 3. Using the 600 0 F curve, Fc reads 6100
and thus P = 6100 x 4.375 = 26700 Ibs. Thus In order to determine the true deflections
subjecting this member to a temperature of or distortions df structures under a given load
600 0 F for 1/2 hour reduces its strength from system, the values of the stiffness properties
220,000 to 26,700 Ibs., which means that of the material used in the structure must be
Alum. Alloy is a poor material for carrying known. Chapters A7 and A8 involve the deflec-
loads under such temperatures since the tions of structures, thus the value of (E),
reduction in strength is qUite large. modulus of elasticity and (G), modulus of ri-
gidity must be known if true deflections are
Case 4. Material 17-4 PH stainless steel to be wanted. Table A18.2 gives these stiff-
forging. Fig. 19 gives the column curves ness values for a few common aircraft structur-
for this material. For L' /p = 41 and using al materials and also several other strength
the room temperature curve we read Fc = properties.
=
135,200 and thus P 135,200 x 4.375 =
591,000 Ibs. When a material is SUbjected to elevated
temperatures for various periods of time, the
AIB.27 Comprehensive Treatment of Column Strength. strength and ~tiffness properties are con-
siderably influenced as illustrated in Figs.
Columns with stable cross-sections fail as 26 to 28, which have been taken from Volume 2.
described in Art. A18.22. However, compression
members having open sections such as angle, For a comprehensive treatment of the be-
channel and zee shapes, often fail by tWisting havior of aircraft materials and their mechani-
or by local buckling or crippling of one or cal and physical properties, the reader is re-
more of the elements that make up the shape of ferred to Chapter Bl and B2 of Volume 2, where
the cross-section. Furthermore, the end fiXity over 50 pages are devoted to this important
condition influences the column strength and subject.
A18.30 STRENGTH OF COLUMNS WITH STABLE CROSS-SECTIONS
- 100
c
<.)
W
"0
c
0 E
w
80
::J
LL
>.
u:-
- 60
0
c=
Q)
....Q)
ll.
40
20
0'
-400 o 400 800 1200
Temperature, F
TABLE A18. 2
W
Material Symbol Form Ftu Fty E Ec G
psi/lO psi/lO 6 psi/lO lbs./
KSI KSI 6 6
Cu. in.
100
S
E
!l BO
E
!!!
E
&
0 40
...
~
20
IOU 100
~ !
~ ~
I
!!!
80 100 200 300 400 500
Temperature, F
600 700 BOO !l
E
!!!
80
E 60 E 60
& Effect of exposure at elevated temper- ~
0 0
40
atures on the room-temperature tensile 40
~
yield strength (Fty) of 7075-T6 alumi- ...;.
~ 20
num alloy (all products). Iu 20
~ ~
100 200 300 400 500 600 roo 800 100 200 300 400 500 60D 700 800
Temperature, F 100
Temperature, F
100 ~ 20
~ &
~ 80 100 200 300 400 500 600 700 800
~ Temperature, F
!!!
E 60
& Effect of temperature on the compres-
0 w" 60
40 sive yield strength (Fcy) of 7075-T6
~ aluminum alloy (all products). o
W 40
~
~
100 200 300 4CO 500 600 700 800 -400 -200 200 400 600 BOO 1000
~
100 20
!
~ 80
~
!l 100 200 300 400 500 600 700 800
E
!!! Temperature. F
E 60
& Effect of temperature on the ultimate
0 40
shear strength (F su ) of 7075-T6
... aluminum alloy (all products).
~ 20
~
100 200 300 400 500 600 700 800 100 200 300 400 500 600 700 800
Temperature, F 100 Temperature. F
A19.1 Typical Wing Structural Arrangement relatively thin. In general the wing structural
flange arrangement can be classified into two
For aerodynamic reasons, the wing cross-
types; (1) the concentrated flange type where
section must have a streamlined shape commonly
flange material is connected directly to in-
referred to as an airfoil section. The aero-
ternal webs and (2) the distributed flange type
dynamic forces in flight change in magnitude, where stringers are attached to skin between
direction and location. Likewise in the various
landing operations the loads change in magni- internal webs.
tUde, direction and location, thus the required
Fig. A9.3 shows several structural arran(~e
structure must be one that can efficiently
ments for wing cross-sections for supersonic
resist loads causing combined tension, com-
aircraft. Supersonic airfoil shapes are
pression, bending and torsion. To provide
relatively thin compared to subsonic aircraft.
torsional resistance, a portion of the airfoil
surface can be covered with a metal skin and
then adding one or more internal metal webs to Distributed Flange Type of Wing Beam.
produce a single closed cell or a multiple cell
Fig. d ~-----
~--------]
wing cross-section. The external skin surface
which is relatively thin for subsonic aircraft
is efficient for resisting torsional shear
stresses and tension, but qUite inefficient in
resisting compressive stresses due to bending
of wing. To provide strength efficiency, span-
wise stiffening units commonly referred to as Fig. e
flange stringers are attached to the inside of
the surface skin. To hold the skin surface to
airfoil shape and to provide a medium for
transferring surface air pressures to the
cellular beam structure, chordwise formers and
ribs are added. To transfer large concentrated
loads into the cellular beam structure, heavy
ribs, commonly referred to as bulkheads, are
used.
Fig. a
~--------\
~l _
Fig. b Fig.hc=J L
: 1,:n
Fig. c
To withstand the high surface pressures and to ment (Fig. A19.2), a portion of the covering on
obtain sufficient strength much thicker wing the compression side is made effective since it
skins are usually necessary. Modern milling is attached to closely spaced stringers or corru-
machines permit tapering of skin thicknesses. gations. However, the flange stringers between
To obtain more flan~e material integral flange cell webs are supported only at rib or bulkhead
units are machined on the thick skin as illus- points and thus suffer column action normal to
trated in Fig. k. the cell covering. This factor reduces the
flange allowable compressive stresses since it
Fig. j is not practical to space wing ribs less than
12 to 18 times the flange stringer depth. Thus,
if there were no other controlling factors, one
could easily make calculations to determine
which of the above would prove lightest. In
general, if the torsional forces on the wing
Fig. k are small, thus requiring only a thin covering,
the concentrated flange type of wing structure
should prove the lightest.
"~
Fig. the Z direction, which means in general that the
flange material should be placed between the 15
and 50 per cent of wing chord from the leading
edge.
Fig.A19.3 Wing Sections - Supersonic Aircraft The secondary or distributing structure aft
of the structural box beam should be made as
In a cantilever wing, the wing bending light as possible and thus in general the farther
mOments decrease rapidly spanwise from the forward the rear closing web of the box beam, the
maximum values at the fusela~e support points.
Thus thick skin construction must be rapidly lighter the wing structure as a whole.
tapered to thin skin for weight efficiency, but
In the layout of the main spanwise flange
thinner skin decreases allowable compressive members bends or changes in direction should be
stresses. To promote better efficiency sand-
wich construction can be used in outer portion avoided as added weight is required in splicing
of wing (Fig. 1). A light weight sandwich core or in transverse stiffeners which are necessary
is glued to thin skin and thus the thin skin is to change the direction of the load in the flange
capable of resisting hi~h compressive stresses members. If flange members must be spliced, care
since the core prevents sheet from buckling. should be taken not to splice them in the region
of a maximum cross-section. Furthermore, in
A19.2 Some Factors Which Influence Wing Structural general, the smaller the number of fittings, the
Arrangements lighter the structure.
(1) Light Weight: (2) Wing-Fuselage Attachment:
The structural designer always strives for If the airplane is of the low wing or the
the minimum weight which is practical from.a high wing type, the entire wing structure can
production and cost standpoint. The hi~her the continue in the way of the airplane body. How-
ultimate allowable stresses, the lighter the ever, in the mid-wing type or semi-low wing
structures. The concentrated flange type of type, limitations may prevent extending the
wing structures as illustrated Fig. (a, b and c) entire wing through the fuselage, and some of
of Fig. A19.1 permits high allowable compressive the shear webs as well as the wing covering
flange stresses since the flange members are must be terminated at the side of the fuselage.
stabilized by both web and covering sheet, thus If a distributed flange type of cell structure
eliminating column action, which permits design were used, the axial load in the flange string-
stresses approaching the crippling stress of the ers would have to be transferred to the members
flange members. Since the flange members are extending through the fuselage. To provide for
few in number, the size or thickness required is this transfer of large loads requires structural
relatively large, thus giving a high crippling weight and thus a concentrated flange type of
stress. On the other hand, this type of design box structure might prove the best type of
does not develop the effectiveness of the metal structure.
covering on the compressive side, which must be
balanced against the saving in the weight of the (3) Cut-outs in Wing Surface:
flange members.
The ideal arrangement where continuity of
In the distributed type of flange arrange- structure is maintained over the entire surface
A19.3
of the structural box is seldom obtained in ac- the lower side of the Wing. They are usually
tual airplane design due to cut-outs in the fastened to two spanwise stringers with screws
wing surface for such items as retractable and the removable panels are effective in re-
landing gears, mail compartments and bomb and sisting bending and shear load. (See Fig.A19.5)
gun bays. If the distributed flange type of
box be&~ is used, they are interrupted at each
cut-out, which requires that means must be
provided for drifting the flange loads around Removable panel for
the opening, an arrangement which adds weight Fig. A19. 5 assembly and inspection
because conservative overlapping assumptions purposes.
are usually made in the stress analysis. The
additional structure and riveting to provide
for the transfer of flange load around large
openings adds considerably to the production
cost. Cutouts in the wing structural box destroys
the continuity of the torsional resistance of
For landing gears as well as many other the cell and thus special consideration must be
installations, the wing cut-outs are confined to given to carrying torsional forces around the
the lower surface, thus a structural arrangement cut-out. This special problem is discussed
as illustrated in Fig. A19.4 is qUite common. later.
The upper surface is of the distributed flange
type whereas the lower flange material is con- (4) Folding-Wings:
centrated at the two lower corners of the box. For certain airplanes, particularly Carrier
In the normal flying conditions, the lower sur-
face is in tension and thus cell sheet covering based Naval airplanes, it is necessary that pro-
between the cut-outs is equally effective in vision be made to fold the outer wing panels up-
bending if shear lag influence is discounted. ward. This dictates definite hinge points be-
tween the outer and center wing panels. If a
For negative accelerated flying conditions, the distributed flange type of structure is used,
lower surface is in compression thus sheet cov- the flange forces must be gathered and trans-
ering between corner flanges would be ineffec- ferred to the fitting points, thus a compromise
tive in bending. However, since the load fac- solution consisting of a small number of span-
tors in these flight conditions are approximate-
ly one half the normal flight load factors, this wise members is common practice.
ineffectiveness of the lower sheet in bending is (5) Wing Flutter Prevention:
usually not critical.
With the high speeds now obtained by modern
Cut-outs likewise destroy the continuity of airplanes, careful attention to wing flutter
intermediate interior shear webs of such prevention must be given in the structural lay-
sections as illustrated in Figs. (c and i), and out and design of the wing. In general, the
the shear load in these interrupted webs must critical flutter speed depends to a great ex-
be transferred around the opening by special tent on the torsional rigidity of the Wing.
bUlkheads on each side of the cut-out, which When the mass center of gravity moves aft of the
means extra weight. 25 per cent of chord paint, the critical flutter
speed decreases, thus it is important to keep
weight of the wing forward. At high speeds
where "compressibility" effects become important,
the torsional forces on the wing are increased,
Fig. A19.4 which necessitates extra skin thickness or a
larger cell. Designing for flutter prevention
is a highly specialized problem.
:t;
Photograph A19 .6
2"li+r:=:==
1 l.
2679-1- A
22"
<lac
:~3:1.8".-.~1:·- .,1 ~I-. . . .
~o C
1" In Chapter A2, the stress analysis of an
externally braced fabric covered monoplane wing
was considered. To provide sufficient torsion-
316 6 t
I C1ab 45 / Vz=11500 al strength and rigidity, two external brace
2584-+LB_~_+___q_bc
struts were necessary. However, if a wing is
,._:-Vx=look metal covered, a single external brace strut
iJ t My =460,500 can be used, since the closed cell or cells
2 1250 ~10''-1 formed by the metal sheet covering and the in-
Fig. A19.9
ternal webs prOVide the torsional resistance
and the Wing can be designed as a s imply sup-
ported beam with cantilever overhang. An
To find qbc take moments about point (at) excellent example of this type of wing struc-
ture is the Cessna aircraft Model 180 as shown
ZMa , =+ 2679 x .5 - 2584 x 11.5 - 316 in the photograph. An excellent airplane
x 0.375 - 45 x 22 + 700 x 12 relative to performance, ease of manufacture
- 11500 x 39 + 460,500 - qbc and maintenance.
(665) 0 =
To introduce the student to the general
approach of stress analYZing such a wing struc-
A19.8 ANALYSIS OF WING STRUCTURES
ture, a limited discussion with a few calcula- Fig. A19.10 and A19.11 shows the wing
tions will be presented. dimensions and general structural layout of a
monoplane wing with one external brace strut.
The wing panel is attached to fuselage by
single pin fittings at points A and B with pin
axes parallel to x axis. The mating lugs of
the fittings at point A are made snug fit but
those at B with some gap, thus drag reaction
of wing loads on fuselage is resisted entirely
at fitting A. Since the fittings at A and B
cannot resist moments about x axis, it is
necessary to add an external brace strut DC to
make structure stable. The panel structure
consists of a main spar ACE and a rear spar BF.
The entire panel is covered with metal skin
forward of the rear spar.
Cessna Aircraft Model 180
Metal Covered Wing with A simplified air load has been assumed as
One External Strut shown in Fig. A19.12, namely, a lmiform load
=
w 30.27 lb./in. of span acting at the 30 per
cent of chord point. When this resultant load
Rear Web
B-- ----- is resolved into z and x components the results
IA. _ ---.!'ront Spar _ _
are Wz =
30 lb./in. and Wx =
4 lb./in. as shown
l in Fig. A19.12.
C
The general physical action of the wing
structure in carrying these air loads can be
considered as 3 rather distinct actions,
namely, (1) The front spar ACE resists the
bending moments and shears due to load wz .,
(2) The skin and webs of the two-cell tube
resists all moments about y axis or broadly
speaking torsional moments, (3) the entire panel
cross-section resists the bending moment and
Fig. A19.l0 flexural shear due to drag load wx , with the
top and bottom skin acting as webs and the two
spars as the flanges of this box beam.
General Calculations: -
z The unknown external reactions (see Fig.
A19.11) are Ay , Az, Ax, By, Bz and DC., or a
total of 6. Since 6 static equations of
equilibrium are available, the reactions are
statically determinate. Reaction DC is also
the load in brace strut DC.
To find reaction DC take moments about x
axis through points A, B
increasing from zero at tip to 2141 lb. at would have to be replaced by two fittings, one
fuselage attachment points and varying as y2. on the upper flange and the other on the lower
At point C on from spar the axial tension load flange in order to be able to resist a Mx
would be .074 (90 2 ) 600 lb. = moment. Fig. A19.16 shows this modification.
The fitting at B could remain as before, a single
The design of the front spar between pin fitting.
points C and E would be nothing more than a
cantilever beam subjected to a bending forces
plus an axial tensile load plus a torsional
shear flow. The design of the spar between
points C and A is far more complicated since
we have appreciable secondary bending moments
to determine, which must be added to the pri-
mary bending moments. Fig. A19.15a shows a
free body of spar portion AC.
w = 30
t t t t t t t t t J,'i Mc=30X90X45 Fig. A19.16
5085 ---<:- -> -> - -> -> ..... ~
1 Fig. A19. 15a ~ The stress analysis of this wing would consist
of the spar AE resisting all the Mx moments and
The lateral load of 30 Ib./in. bends the the Vz shears and acting as a cantilever beam.
beam upward, thus the axial loads at A and C The torsional moment about a y axis coinciding
will have a moment arm due to beam deflection with spar AE would be resisted by shear stresses
Which moments are referred to as secondary in the cellular tubes formed by the skin and the
moments. To find deflections the beam moment spar webs. The drag bending and shear forces
of inertia must be known, thus the design of would be resisted by the beam whose flanges are
this beam portion would fall in the trial and the front and rear spars and the web being the
error procedure. Articles A5.23 to 28 of top and bottom skin.
Chapter A5 explain and illustrate solution of
problems involving beam-column action and such A19.8 Stress Analysis of Thin Skin - Multiple Stringer
a procedure would have to be used in actually Cantilever Wing. Introduction and Assumptions.
designing this b6am portion.
The most common type of wing construction
The rear spar BF receives two load systems, is the multiple stringer type as illustrated by
namely a varying axial load of zero at F to 2141 the six illustrative cross-sections in Fig.
lb. at B and the web of this spar receives a A19.2. A structure with many stringers and
shear load from the torsional moment. The sheet panels is statically indeterminate to
rear spar is not subjected to bending moments. many degrees With respect to internal stresses.
Fortunately, structural tests of complete wing
In Fig. A9.10 the secondary structure structures show that the simple beam theory
consisting of chordwise ribs and spanwise light gives stresses which check fairly well with
stringers riveted to skin are not shown. This measured stresses if the wing span is several
secondary structure is neceosary to hold wing times the wing chord, that sweep back is minor
contour shape and transfer air pressures to and wing is free of major cutouts and discon-
the box structure. This secondary structure tinuities. Thus it is common procedure to
is discussed in Chapter A21. The broad subject analyze and design a wing overall by the beam
of designing a member or structure to Withstand theory and then investigate those portions of
stresses safely and efficiently is considered the wing where the beam theory may be in error
in detail in later chapters. by using more rigorous analysis methods such as
those explained and illustrated in Art. A8.10
A19.7 Single Spar - Cantilever Wing - of Chapter A8.
Metal Covered
ASSUT1PTIONS - BEAM THEORY
A single spar cantilever wing with metal
covering is often used particularly in light In this chapter the wing bending and shear
commercial or private pilot aircraft. stresses will be calculated using the un-
symmetrical beam theory. The two main assump-
Suppose in the single spar externally tions in this theory are: -
braced wing of Fig. A19.11, that the external
brace strut DC was removed. Obviously the wing (1) Transverse sections of the beam originally
would be unstable as it would rotate about hinge plane before bending remain plane after bending
fittings at points A and B. To make the of beam. This assumption means that longitudi-
structure stable the single pin fitting at (A) nal strain varies directly as the distance from
AI9.11
the neutral axis or strain variation is linear. corner flange members (a) and (b) are stabilized
in two directions by the skin and webs and
(2) The longitudinal stress distribution is usually fail by local crippling.
directly proportional to strain and therefore
from assumption (1) is also linear. This Now continuing the loading of the wing
assumption actually means that each longitudinal after the skin has buckled, the stringers and
element acts as if it were separate from every corner members will continue to take additional
other element and that Hook's law holds, compressive stress. Since the ultimate strength
namely, that the stress-strain curve is linear. of the stringers is less than that of the cor-
ner members, the stringers (c) will start to
Assumption (1) neglects strain due to shear bend elastically or inelastically and will take
stresses in skin, which influence is commonly practically no further stress as additional
referred to as shear lag effect. Shear lag strain takes place. The corner members still
effects are usually not important except near have considerable additional strength and thus
major cut-outs or other major discontinuities additional external loading can be applied until
and also locations where large concentrated finally the ultimate strength of the corner
external forces are applied. members is reached and then complete failure of
the top portion of the beam section takes place.
Assumption (2) is usually not correct if Therefore, the true stress - strain relationship
elastic and inelastic buckling of skin and does not follow Hook's Law when such a structure
stringers occur before failure of wing. In is loaded to failure.
applying the beam theory to practical wings,
the error of this assumption is corrected by In the above discussion the stringers (c)
use of a so-called effective section which is were considered to hold their ultimate buckling
discussed later. load during considerable additional axial
strain. This can be verified experimentally
A19.9 Physical Action of Wing Section in Resisting External by testing practical columns.. Practical col-
Bending Forces from Zero to Failing Load. umns are not perfect relative to straightness,
Fig. A19.17 shows a common type of wing uniformity of material, etc. Fig. A19.18
cross-section structural arrangement generally shows the load versus lateral deflection of
referred to as the distributed flange type. column midpoint as a column is loaded to failure
and fails by elastic bending. Fig. A19.19 shows
similar results when the failure is inelastic
bending.
a b
' - - - -.......v ,.-----'
web c(stringers) web Fig. A19.18 Fig. A19.19
Fig. A19.17
The corner members (a) and (b) are considerably
larger in area than the stringers (c). The
skin is relatively thin. Now assume the wing {j = Central Deflection
{j = Central Deflection
is subjected to gradually increasing bending
forces which place the upper portion of this
wing section in compression and the bottom The test results show that a compression
portion in tension. Under small loading the member which fails in bending, normally con-
compressive stresses in the top surface will tinues to carry approximately the maximum load
be small and the stress will be directly pro- under considerable additional axial deformation.
portional to strain and the beam formula Thus in the beam section of Fig. A19.17 when
0c = MxZ/I x will apply and the moment of the stringers (c) reach their ultimate load,
failure of the beam does not follow since cor-
inertia Ix will include all of the cross-
section material. As the external load is in- ner members (a) and (b) still have remaining
creased the compressive stresses on the thin strength.
sheets starts to buckle the sheet panels and
A19.10 Ultimate Strength Design Requirement
further resistance decreases rapidly as further
strain continues, or in other words, stress is The strength design requirements are: -
not directly proportional to strain when sheet
buckles. Buckling of the skin panels however (1) Under the applied or limit loads, no
does not cause the beam to fail as the stringers structural member shall be stressed above
and corner members are lowly stressed compared the material yield point, or in other
to their failing stress. The stringers (c) words, there must be no permanent de-
are only supported transversely at wing rib formation or deflection of any part of the
points and thus the stringers act as columus and structure.
fail by elastic or inelastic bending. The
A19.12 ANALYSIS OF WING STRUCTURES
\~I-I
I
I I
\ / II
I I
II
r '-foc -.l that for the same unit strain member (a) can
take considerable higher stress. If we take
b
2 a unit strain of .006, the point at which the
maximum stress of 47000 is obtained in member
(a) (see point (3) on Curve A) then the stress
at the same strain for member (C) will be 38000
(see point (2) on Curve C).
Fig. A19.20
T
10"
come out as recorded in the Table.
The next step is to correct for stringer
effectiveness in compression. The failing
.04
Now suppose we would have omitted con- For any given flange member With area (a)
sideration of the stringer effectiveness factor the load Pn on the member would be,
and omitted column (3) of Table I. Carrying
through the calculations of Table I, the value Pn =ona - - - - - - (3)
of Z would be -0.76" and Ix would be 63.08.
The stress intensity on the zee stringers The stresses due to bending moments are
would then be 500000 x 5.36/63.08 42500 psi. = from Chapter A13, Art. A13.5: -
Since the failing stress for stringer is 38000
the margin of safety would be (38000/42500) - 1
= a negative 10.5 percent. The previous result
was a plUS 1 percent, thus a difference of 11.5 where 0b = bending stress with plus being ten-
percent in the results. Sion
The purpose of this simple example problem
was to emphasize to the stUdent that failure of
real aircraft stiffened skin structures occurs K2 = Iz/(Ix Iz
under non-linear stress-strain conditions and
the elastic theory must be modified if fairly
accurate estimates of the failing strength of
K" = I x/ (Ix Iz
a composite structure is to be obtained. The normal component of the axial load in
A19.13 Bending and Shear stress Analysis of Tapered - a flange member equals 0b a) where (a) is the
Multiple stringer Cantilever Wing. Unsymmetrical area of the flange member. Since the angle be-
Beam Method. tween the normal to the beam section and the
In general cantilever wings are tapered in centroidal axis of a stringer is generally qUite
both depth and planform. Fig. A19.24 illus- small, the difference between the cosine of a
trates a typical structural layout of the outer small angle and unity is negligible and thus
wing panel for a small airplane. The structure the normal component can be considered as the
consists of a front and rear beam (spar) with axial load in the stringer.
spanwise stringers between the two beams.
Tapering in cross-sectional material is obtained Before equations 2, 3 and 4 can be solved,
by decreasing size of members by cutting off the effective cross-section area must be known
portions of the spanwise stringers and corner as well as the moments of inertia and product
flanges and decreasing the skin and web thick- of inertia about x and z centroidal axes.
ness.
A19.15
o4OSpolHT
+------ - - - - o
/4 /3 12 /I 10 q B
_
_ _ _ _ .032. - - - - l - - - - .02.S ----+-t Skin thickness
between beams
r--=2 1/4~
~Ot=1.2'~ -+- 1.2-.j
~2
1/4 :::=--I
4
TUpper Front Flange I
.050
c"'\.O~
0. 0'1. 9:>e (X' reference axis
----+--C. - ----I ~
X Centroidal Axis
I
Lower Surface Stringer
area of bulb angle = . 11
area skin = .16
:27
I- ~I
I..
1
area of angles = . 508
area of skin =.120
area of web =.064
.692 Lower Front Flange
area of angles = . 508
r
2
i
area of skin =. 160
area of web =. 100
.768
f t
--rir=.----~_W_=uT-----,__w_=O~f.F---,----.,,,-tr-----'. 032 upper skin
~ W ~-_~ __
en ~ 7
~~ 9:>ec~
: ~~
x...
u
~
!",_,O_'__ fX' reference axis_
_+-- -=====_~ ========-=-======....::4r~. ==--=--~====::.........:=--=t==X'X
~
OW X Centroidal Axis
~ ~
~ ·~w
£ .~
14 C;;<
~
U
N 8
Z'
Fig. A19.26 WING SECTION STATION 47.5
A19.17
TABLE A19. 2
SECTION PROPERTIES ABOUT CENTROIDAL X AND Z AXES
1 " 3 4 5 6 7 8 9 10 11 12 13
Flange Area
No. A Z' AZ' AZ,2 X' AX' AX';': AX'~I Z = Z' - Z X
= x' - x CYb P
= ""b A
1 . 648 5.50 3.56 19.61 -0.10 -0.06 0 -0.36 6.68 -17.4 -37660 -24390
2 ,,353 5.90 2.08 12.30 5.65 I. 99 11.30 11.78 7.08 -1 I. 65 -39940 -14070
3 .300 5.85 I. 76 10.28 11.20 3.36 37.60 19.67 7.03 - 6.10 -39620 -11890
4 .300 5.55 I. 66 9.25 16.85 5.05 85.00 28.10 6.73 - 0.45 -37950 -11370
5 .300 5.05 I. 52 7.65 22.40 6.72 150.50 34.00 6.23 S.IO -35115 -10510
6 .353 4.40 I. 55 6.83 28.00 9.89 277.00 43.50 5.58 10.70 -31450 -11090
7 .630 3.55 2.24 7.95 35.72 23.15 826.00 80.00 4.73 18.42 -26560 -16700
8 .692 -8.40 -5.81 48.90 35.72 24.70 881. 00 -208.00 -7.22 18.42 40750 28230
9 .270 -8.50 -2.30 19.55 27.80 7.50 208.00 - 64.00 -7.32 10.50 41280 11180
10 .270 -8.50 -2.30 19.55 22.60 6.10 138.00 - 52.00 -7.32 5.30 41250 11170
l1 .160 -8.30 -I. 33 II. 05 16.70 2.67 44.60 - 22.20 -7.12 - 0.60 40100 6430
12 .27 -8.00 -2.16 17.30 10.90 2.94 32.00 - 23.50 -6.82 - 6.40 38400 10390
13 .27 -7.50 -2.02 15.20 5.60 I. 51 8.45 - II. 35 -6.32 -11.7 35570 9600
14 .768 -6.50 -5.00 32.50 - 0.10 .08 0 0.50 -5.32 -17.40 29950 23020
TABLE A19. 3
SECTION PROPERTIES ABOUT CENTROIDAL X AND Z AXES
1 ~ 3 4 5 6 7 6 9 10 11 12 13
Flan~;'c Area
No. A Z' AZ' AZ! :~ X' AX' AX· 2 AX'Z' L.
= Z' -Z X = X' - X CYb P = cb A
1 .476 5.CO 2.66 14.95 - 0.10 - 0.05 0 - 0.27 6.21 -15.60 -39275 -18700
" .318 5.83 I. 86 10.80 4.40 1.40 6.16 8.16 6.44 -1 I. 10 -40790 -12990
3 • ~~66 5. "/5 I. 53 8.80 10.00 2.6G 26.60 15.30 6.36 - 5.50 -4025 -10710
4 .266 5.4::::' I. 44 7.81 15.60 4.15 64.75 22.40 6.03 0.10 -38400 -10200
5 .266 4.90 I. 30 6.40 21. 20 5.64 120.00 27.60 5.51 5.70 -35075 - 9320
6 .318 4.30 I. 37 5.88 26.70 8.50 2:'::6. S,Q 36.60 4.91 II. 20 -31360 -10000
7 .476 3.60 1.71 6.17 31.70 15.10 478.00 54.40 4.21 16.20 -27030 -12880
8 .353 -7.10 -3.93 27.90 31.70 17.51 555.00 -124.50 -6.49 16.20 41050 22770
9 • ;J,3~ -7.40 -1.74 12.90 26.50 6.23 165.00 - 46.10 -6.79 11.00 43090 10140
10 .~3~ -7.40 1.74 12.90 21. 30 5.00 107.00 - 37.18 -6.79 5.80 43180 10160
11 • 1;.~5 -7.20 -0.90 6.48 15.10 I. 89 28.50 - 13.60 -6.59 - 0.40 41900 5240
1" · ;,3" -6. 90 -1, 62 11, 20 9.60 ~.26 21. 70 - 15.60 -6.29 -55.90 40085 9410
13 .235 -6. "'5 -I. 47 9.20 4.50 I. 06 4.75 - 6.61 -5.84 -II. 00 36060 8470
14 .60r -5.40 -3.27 17.70 - 0.10 - 0.06 0 0.33 -4.79 -15.60 30725 18600
Z = -:;.80/4.61 ~ -0.61"
General Notes:
X = 71.3/4.61 a 15.50" See Fig, A19.26 for section at Station 47.5.
Reference axes X 1 X 1 and Z·Z' are assuTI1ed as shown.
1)< = 159.1 - 4.61 x.61~
= 157.4
2 Cfb = -14.5 X - 6360 Z
I z = 1304 - 4.61 x 15.5 = 700
~p equals the change in stringer axial load areas are referred as m values (See Fig. A19.27).
over a distance d in the y direction. Column 6 of Table A19.4 records these double
areas which were obtained by use of a planimeter.
Since the cell in our problem is closed the Column (7) gives the moment of each shear flow
value qo at any point is unknown. We assume it about the c.g. and the total of this column
zero on web 1-14 by imagining that the web is gives the moment about the c.g. of the complete
cut as shown in Fig. A19.27. Equation (6) thus shear flow system of Fig. A19.27 or a value of
reduces to, 256060 in. lb.
For equilibrium of all the forces in the Z'Mc.g. =12000 x 33.3 - 2700 x 11.8 - 390000
plane of the cross-section ZMy must equal zero. = 41800 in. lb.
For convenience we will select a moment y axis
through the c.g. of the cross-section. The Moments Produced by Inclination of Flange Loads With
moment of the shear flow q on any sheet element Beam Section.
equal q times double the area of the triangle
formed by joining the c.g. with lines going to Since the flange members in general are not
each end of the sheet element. These double normal to the beam sections, the flange loads
A19 20 ANALYSIS OF WING STRUCTURES
have components in the Z and X directions. Due to external loads = 41800 in.lb.
Columns (4) and (7) of the Table A19.5 give the
values of these in plane components. The slopes Then the total unbalanced moment = 4936 +
dx/dy between stations 20 and 47.5 are found by 256060 + 41800 =
302796 in.lb.
scaling from Fig. A19.24. Fig. A19.29 shows
these induced in plane forces as found in Table For equilibrium, this must be balanced by
A19.5. a constant shear flow q~.
1 2 3
Station 20
4 5 6 7 8
(Note: 461.5 = total area of cell)
Z 7160 - 2224
NOTES:
Column (2); P from Table A19. 2
Column (5) and (8): Values of Z and X are found in
Columns 10, 11 of Table A19. 2
610 417 Fig. A19.30 Final shear flow diagram. For val-
r
~ ues see Column 9 of Table A19.4.
OI ~3 J4 ~5
310
'
262 250
+c.g.
231 2~:
'['50 Havins determined the shear flOWS, the
shear stress on any panel would be q/t. In
checking the sheet for strength in shear and
combined shear and tension, interaction rela-
1060 14 ,13 1.2 l,1 19 ~ 8 594 tionships are necessary. The strength design
~ 1~0 508t
of sheet panels under combined stresses is
416 173
t
1~7 96
t IJ7 covered in considerable detail in Chapter C5.
A19. 14 Bending and Shear Stress Analysis of 2-Cell
Fig. A19.29 In plane forces produced by flange Multiple Stringer Tapered Cantilever
axial loads. Wing.
The moments of these in plane components about A two-cell beam is also quite common in
the section e.g. are given in Columns (5) and wing structural design. A two-cell structure
(8) of Table A19.5. In general, these moments in bending and torsion is statically indeter-
are not large. minate to the second degree since the shear
flow at anyone point in each cell is unknown.
Total Moments of All Forces About Section e.g. However, due to continuity between cells the
at Station 20: angular twist of each cell must be the same,
which gives the additional equation necessary
Due to flanges =7160 - 2224 =
4936 in. lb. for solving a two-cell beam as compared to the
(Ref. Table A19.5) single cell analysis.
lations as was used in the previous single cell The first 7 columns of this table are the same
problem, the bending and shear stresses will be as in Table A19.4, since no stringers have been
determined for the same structure as in the added to cell (1), and the shear q is assumed
previous example except that the leading edge zero in cell (1).
cell is considered effective, thus making a
2-cell structure. Since there are no spanwise To make the twist of each cell the same and
stringers in the leading edge, very little skin also to make the summation of all torsional
on the compressive side will be effective. On forces zero will reqUire two unknown constant
the tension side, the leading edge skin would shear flows, q~ in cell (1) and q2 in cell (2).
be effective in resisting bending axial loads Thus two equations will be written, namely:
and thus the moment of inertia would be slightly
different from that found in example problem 1. (8)
Since this problem is only for the purpose of
illustrating the use of the equations, the
leading edge skin will be neglected in computing
m c •g . 0 (9) =
the bending flexural stresses. With this The twist 9 per unit length of a cell
assumption, the bending stresses and flange equals
loads at stations 20 and 47.5 are the same as
for the previous problem. (See values in column 1
12 and 13 of Tables A19.2 and A19.3.) 9 = 2AG t;qL/t (10)
t
Fig. A19.31
2
.
3 4
.
5
. g~ = _1
2A~
G q Lit + Z q~ Lit + t; q2 L/~
I
6
~ q~
a C,II (l) b Cell (2) 1
Area = 83.5 Area = 461. 5 sq. in. g~ =2 X 83.5 + 1278 - 230 qJ, whenc e
14 1.3 12 1.1 10 .
9
g~ = 7.65 q~ - 1.378 q2- - - - - - - - (a)
TABLE A19. 6
SHEAR FLOW - 2 CELL - MULTIPLE FLANGE TAPERED BEAM
(A veragc Shear Flow Between Stations 20 and 47.5)
1 ~ 3 4 5 6 7 8 9 10 11 12 13 14
Flange
No.
P
at Sta.
P
at Sta. _6P
q .:
i6.P m mq L t
L/t
Cell Cell
a (L/t)
Cell Cell ql q2
qr =
~O 47.5 27.5 27.5 (in. ) (1) (2) (1) (2) q+qrtq 2
1 - 24390 -18700 206.8 206.8 48.4 144 29800 0 -317 -110.2
10000 5.75 .04
2 -14070 -12990 39.1
3 -11890 -10710 42.9
245.9 43.8 10780 5.50 .04 137 33700 0 -317 - 71.1
4 -11370 -10200 42.5
288.8 42.8
331.3 42.6
12120 5.50 .04 137
137
39600
45400
0
0
-317
-317
- 28.2
14.3
14100 5.50 .04
5 -10510 - 9320 43.3 374.6 43.6 16340 5.50 .04 137 51400 0 -317 57.6
6 -11090 -10000 39.6
414.2 42.0 137 56700 0 -317 97.2
17500 5.50 .04
7 -16700 -12880 139.0 553.2 197.0 382 211500 0 -317 236.2
22770 -198.7 108900 12.25 .032
8 28~30
354.5 47.4 .032 172 61000 0 -317 37.5
9 11180 10140 16800 5.5
- 37.8 316.7 37.2 11800 5.0 .032 156 49400 0 -317 0
10 11170 10160
11 6430 5240
- 36.7
_ 43. 2 280.0 40.8 11420 5.8 .032 181 50650 0 -317 - 37
12 10390 9410 - 35. 6
236.8 42.2
201.2 38.0
10000 5.8 .032
.032
181
166
42900
33400
0
0
-317
-317
- 80.2
-115.8
13 9600 8470 .41.1 7650 5.3
160.1 54.0 8650 5.5 .032 172 27600 0 -317 -156.9
14 ~30~0 18600 160.3
0 204 0 11.75 .051 230 230 0 0 -62.8 -317 -254.2
1
Lead
edge 0 371 33.5 .032 1048 0 -62.8 -62.8
web
NOTES: L length of web sheet between flange members. Col. 4 (Col. 2 + Col. 3)/27.5
t :z web thickness.
A19.22 ANALYSIS OF WING STRUCTURES
q~
A19.15 Bending Strength of Thick Skin - Wing Section
Q" =2 x 1
461.5 [733050 - 230 + 2469 qJ
Figs. i and k of Fig. A19.3 illustrate
approximate shapes for airfoils of supersonic
Q" = 794 - .2495 q~ + 2.678 q" - (b) aircraft. Such airfoils have relatively low
thickness ratios and since supersonic military
For continuity Q~ must equal Q", hence
aircraft have comparatively high wing loadings,
equating (a) and (b):
it is necessary to go to thick skin in order to
resist the wing bending moments efficiently.
7.899 q~ - 4.056 q" - 794 = 0 - (c)
The ultimate compressive stress of such struc-
tures can be made rather uniform and occurring
For equilibrium the summation of all mom- at stresses considerably above the yield point
ents in the plane of the cross-section about the
of the material. Since structures must carry
section (e.g.) must be equal to zero, or
the design ,loads without failure, it is neces-
sary to be able to calculate the ultimate bend-
Z Mc • g • =0 ing resistance of such a wing section if the
margin of safety is to be given for various
The moment of the external loads about the
load conditions.
section c.g. is the same as in previous problem.
The question of the ultimate bending
M external forces = 41800 in. lb.
resistance of beam sections that fail at stresses
beyond the elastic stress range is treated in
The induced moment due to the in plane
Article A13.10 and example problem 7 of Chapter
components of the flange axial loads is likewise
A13 and should be studied again before pro-
the same as in previous problem (see Table ceeding with the following example problem.
A19.5) •
A19.16 Example Problem
Mdue to flange 1oa ds = 4936 in.lb.
To illustrate the procedure of Art. A13.10,
The torsional moment due to the static a portion of a thick skin wing section as
shear flow from Column (7) of Table A19.6 equals illustrated in Fig. A19.34 will be considered.
256060 in. lb. The torque due to the unknown
constant shear flows of q~ and q" is equal to
twice the enclosed area of each cell times the
shear flow in that cell, whence
ation of crippling and column strength of the The values in column (3) of Table A19.7
stiffened skin, a subject treated later. represents the true compressive stress at the
midpoint of a strip area when the beam is
Curve (A) of Fig. A19.35a is a portion of resisting its maximum or failing bending moment.
the compressive stress-strain diagram of the The values in column (4) represent the com-
aluminum alloy material from zero to 50,000 psi. pressive stress at the midpoint of the strip
areas if the bending stress is linear and vary-
Due to symmetry about the x-x axis, we ing from zero at neutral axis to 50000 psi at
need only to consider one half of the beam edge of beam section (Curve B of Fig. A19.35a).
section. We divide the upper half of the beam
section into horizontal strips, each 3/8 inch To illustrate, consider strip area number
thick. Each beam portion along these horizontal (2) in Fig. b of A19.35. Project a horizontal
strips can be placed together to form the areas' dashed line from midpoint of this strip until it
labeled (1) to (8) in Fig. b of Fig. A19.35. intersects curves A and B at points (a) and (b)
Since plane sections remain plane after bending respectively. From these intersection points
in both elastic and inelastic stress zones, project downward to read values of 48000 and
Fig. c shows the beam section strain picture. 40600 psi respectively.
Fig. A19.35 In using the linear beam formula, the
Fig. c stress intensity on strip (2) would be 40600
Fig. b
Since the stress analyst must determine (1) Cutouts which cause one or more stringers
critical margins of safety for many conditions, to be discontinued.
it would be convenient to have an interaction
curve involving Mx and Mz bending moments which (2) Large abrupt changes in external load
would cause failure of the wing section. This applications.
interaction curve could be obtained as follows:-
(3) Abrupt changes in stringer areas.
(1) Choose a neutral axis direction and its
location. In Chapters A7 and AS, strains due to
shearing stresses were considered in solving
(2) Assuming that plane sections remain plane, for distortions and stresses in structures in-
and taking the maximum strain as that volving sheet-stringer construction. Even in
causing failure of the compressive flange, these so-called rigorous methods, simplifying
use the stress-strain curve to determine assl®ptions must be made as for example, shear
the longitudinal stress and then the stress is constant over a particular sheet
internal load on each element of the cross- panel and estimates of the modulus of rigidity
section. A check on the location of the for sheet panels under a varying state of
assumed neutral axis is that the total buckling must be made. The number of stringers
compression on cross-section must equal and sheet panels in a normal wing is large,
total tension. Since the location was thus the structure is statically indeterminate
assumed or guessed, the neutral axis must to many degrees and solutions necessitate the
be moved parallel to itself to another use of high speed computors. Before such
location and repeated until the above analyses can be made, the size and thickness of
check is obtained. each structural part must be known, thus rapid
approximate methods of stress analysis are
(3) Find the internal resisting moment about desirable in obtaining accurate preliminary
the neutral axis and an axis normal to the sizes to use in the more rigorous elastic
neutral axes. Resolve these moments into analysis.
moments about x and z axes or Mx and Mz .
These resulting values of Mx and Mz are To illustrate the shear lag problem in its
bending moments which acting together will simplest state, consider the three stringer-
cause failure of the wing in bending. sheet panel unit of Fig. A19.36. The three
stringers are supported rigidly at B and equal
(4) Repeat steps 1, 2 and 3 for several other loads P are applied to the two edge stringers
directions for a neutral axis which results labeled (1) at point (A). The center stringer
will give additional combinations of Mx (2) has zero axial load at (A), but as end B is
and Mz moments to cause wing failure. Thus approached, the sheet panels transfer some of
an interaction curve involving values of the load P to the center stringer by shear
Mx and Mz which cause failure of wing in stresses in the sheet. At the support points B
bending is obtained and thus the margin of the transfer of load from side stringers to
safety for any design condition is readily center stringer is such as to make the load in
obtainable. all three stringers approximately equal or
equal to 2P/3.
A19.25
I
2P/3 - -
Sheet J sect triangle at points (a) on section 2-2. At
section 3-3 point c, stringer (6) becomes fUlly
2P/3
B '(2) J!- effective.
Sheet b
2P/3 ~ ~P
B "'(1) A To handle shear lag effect in a practical
wing problem another column would be inserted
Fig. A19.36 in Table A19.1 between columns (3) and (4) to
take care of the shear lag effect. The shear
lag effectiveness factor which we will call R
The theoretical load in center stringer would equal the effectiveness obtained from a
can be calculated by methods of Chapter A7 and triangle such as illustrated in Fig. A19.38.
AS, and the results would give the solid curve
of Fig. A19.37. To simplify the solution, it For example, the shear lag factor R at
,is common practice to aSSlli~e the load distri- beam section 1-1 in Fig. A19.38 would be zero
bution in the center stringer to vary according for stringers (5), (6) and (7) and one for all
to the dashed curve in Fig. A19.37 which other stringers. At beam section 2-2 stringers
indicates that in a distance 3b, the load 2P (5) and (7) have a factor R = 1.0 since they
is equalized between the three stringers. are fUlly effective at points (a). Stringer
(6) is only 50 percent effective since section
2-2 is halfway from section 1-1 to point (c),
2P/31-- - - - - - - " - - Theory =
thus R 0.5 for stringer (6). At beam section
~
I" ---. Approximate 3-3, stringer (6) becomes fully effective and
Load in "~ =
thus R 1.0 for all stringers. The final
Stringer
(2)
I" ~, modified stringer area (A) in column (4) of
Table A19.1 would then equal the true stringer
o area plus its effective skin times the factors
B IE 3b --jA KR. The procedure from this point would be the
same as discussed before. Thus shear lag ap-
Fig. A19.37 proximations can be handled qUite easily by
modifying the stringer areas. Using these
A19.19 Application of Shear Lag Approximation to modified stringer areas, the true total loads
Wing with Cut-Out. in the stringers are obtained. The true stresses
equal these loads divided by the true stringer
Fig. A19.38 shows the top of a multiple area, not the modified area.
stringer wing which includes a cut-out in the
surface. The stringers (5), (6) and (7) must A19.20 Approximate Shear Lag Effect in Beam Regions
be discontinued through the cut-out region. where Large Concentrated Loads are Applied.
Top Surface A
,~ Effective-
3b ----'1 ..., ~ ness Tri- v
~I ~ ane:le
t -5, /1
Ib ;;1 ./ I
~ ./
I At-- 3b --I /
1
~ t t t f f .j-
Fig. A19.38
*~
~
Dead wi.
inertia forces
It is assumed that the effectiveness of
these 3 interrupted stringers is given by the
triangles in the figure. At beam section 1-1 0t Ben'!lng Mom.
these stringers have zero end load. The
stringer load is then assumed to increase
linearly to full effectiveness when it inter- ~Fi.a
sects the sides of this triangle whose height Fig. A19.39
A19.26 ANALYSIS OF WING STRUCTURES
TTr2~'~
moment cannot be uniform on a beam section
adjacent to section A-A because of the shear
strain in the sheet panels or what is called
shear lag effect. To approximate this stringer
effectiveness, a shear lag triangle of length
3b is assumed, and the same procedure as
discussed in the previous article on cut-outs 150" II I
9= I ~
is used in finding the longitudinal stresses.
It should be understood that the bending moments
1-rgien I
due to the distributed forces on the wing such
as air loads and dead weight inertia loads are j I Y axis at
not included in the shear lag considerations, ,I 33. 3% of chord
only the forces that are applied at concentrated
points on the structure and must be distributed
into the beam. A side load on the gear or -49"1-21" ~ 60" .,
power plant would produce a localized couple
plus an axial force besides a shear force as in tZ r~ A~
Fig. A19.39. The resistance to this couple and 4 x ~lrl) B~
axial force would likewise be based on the 3" l..C
effectiveness triangle in Fig. A19.39. Fig. A19.41
A19.21 Approximation of Shear Lag Effect for Sudden intensity acting upward in the z direction
Change in Stringer Area and 0.25 lb./in.s average intensity acting
stringers of one size are often spliced to rearward in the x direction. The center of
stringers of smaller size thus creating a dis- pressure for z forces is on the 25 percent
continuity because of the sudden change in of chord line measured from the leading edge
stringer area. edge and at mid-height of spar AB for the
x air forces. Assume the 3 stringers A,
Fig. A19.40 shows the stringer arrangement B, C develop the entire resistance to ex-
in a typical sheet-stringer Wing. Stringer B ternal bending moments. Find axial loads
is spliced at point indicated. The stringer in stringers A, B, C and the shear flaw in
area As is decreased suddenly by splicing into the 3 sheet panels of cell (1) at wing
a stringer with less area A1 • stations located 50", 100" and 150" from
wing tip. Consider structure to rear of
Top Surface of Wing
cell (1) as only carrying airloads forward
to cell (1) and not resisting wing torsion
;; Snlk~ Point or bending.
1-- --I
/
Fig. A19.42
/~ ~ J ~
/ s h 1
I I I T b Rear spar
/ I
Single , -- --
/
I
-3b +3b-.j- Pin !
Fittings .-,..,
a
_ _ -
-=-Front §par
C
Fig. A19.40
To approximate the shear lag effect,
assume the area of stringer B at splice point
to be the average area of the two sides or
(A 1 + As )/2. This average area is then assumed
to taper to As and A1 at a distance 3b from the
splice point. The shear lag effectiveness
factor R will therefore be greater than 1.0 on
the side toward the smaller stringer A, and
less than one on the side toward the stringer
with the greater area As, since the average (2) Fig. A19.42 shows a monoplane wing with
area was used for the splice point. one external brace strut. The Wing is
fastened to fuselage by single pins at
A19.22 Problems points (a) and (b). The fitting at (b) is
designed to take off drag reaction. The
(1) Fig. A19.4l shows a cantilever, 3 stringer,
single cell wing. It is subjected to a airloads are Wz =
40 Ib./in. of wing span,
distributed airload of 2 Ib./in. s average with center of pressure at 30 percent of
chord from leading edge and acting upward,
A19.27
and Wx =
5 lb./in., acting to rear and Table A gives the stringer areas at sta-
located at mid-depth of wing. Find re- tions 0 and 150. Assume stringers have
actions at points (a), (b) and (d). Find linear variation in area between these two
axial loads on front and rear spars. Find stations. Use 30t as effective skin with
primary bending moments on front spar. compression stringers.
Find shear flow on webs and walls. Neglect
structure forward of front spar and rear- Find axial loads in stringers at stations
ward of rear spar. 150 and 130 and determine shear flow
system at station 150.
(3) Fig. A19.43 shows a portion of a single
cell - mUltiple stringer cantilever wing. (4) Same as problem (3) but add an internal
The external air loads are: web of .04 thickness connecting stringers
(3) and (8).
Wz =
100 lb./in. acting upward and whose
center of pressure is along a y axis coin- (5) Same as problem (4) but add a leading edge
ciding with stringer (8). cell with radius equal to one-half the
front spar depth. Take skin thickness as
Wx =
6 lb./in. aC4ing to rear and located .04 inches.
at mid-depth of wing.
fDt
-L
6 5
..i
No.
1
2
TABLE A
Flange n,-;=-,.A~r:,=eTa..,-,:::-n-j
Sta.150 Sta 0
1.0
.25
.20
.10
~3..jo.3+-3+ 3j 3 .25 .10
t--- 12 4 .25 .10
5 .80 .10
6 .70 .10
7 .25 .10
8 .25 .10
9 .25 .10
10 .85 .20
II \
/ I
- STA.O
I
I
I~
I
!
I
\\
.032
T
12"
.050
.032
1"1~
1 .025
t:5~ 5 -+ 5
20
+5 j ,.. 150" _I
STA.150 STA.O
Fig. A19.43
A19.28 ANALYSIS OF WING STRUCTURES
North American Aviation FJ3 "Fury Jet". View Shows Bottom Wing Skin.
Note Integral Construction of Skin and Stringers.
North American Aviation F-IOO "Super Sabre" Wing. View Shows End
of Outer Wing Panel. Note Thick Skin.
Douglas DC-8 Wing. View Showing Lower Surface of Outer Wing Panel
Between Center and Rear Spar.
CHAPTER Azo
INTRODUCTION TO FUSELAGE STRESS ANALYSIS
A20.1 General. In general the purpose of an air- to Chapter A5, where exanple calculations of
plane is to transport a commercial payload or a fuselage shears and moments are presented.
military useful load. The commercial payload
of a modern airliner may be 100 or more pass- The basic fuselage structure is essentiall=
engers and their baggage. These passengers a single cell thin walled tube with many trans-
must be transported safely and comfortably. verse franes or rings and longitudinal stringerr
For example, an airliner flies at high altitudes to provide a combined structure which can absorl
where temperatures may be far below zero and and transmit the many concentrated and distri-
where the air density is such as not to sustain buted applied forces safely and efficiently.
human life. These facts mean that the body The fuselage is essentially a bean structure
which carries the passengers must be heated, subjected to bending, torsional and axial
ventilated and pressurized to provide the forces. The ideal fuselage structure would be
necessary safety. Air travel must be acceptable one free of cut-outs or discontinuities, how-
to the passengers, thus the airplane body must ever a practical fuselage must have many cut-
shield the passengers from excessive noise and outs. Fig. (a) shows the basic interior fuse-
vibration, and furthermore efficient, restful lage structure of a small airplane with skin
and attractive furnishings must be provided to removed. It consists of transverse frames and
make travel enroute comfortable and enjoyable. longitudinal stringers. Photographs 1, 2 and 3
The portion of the airplane which houses the illustrate fuselage construction of late model
passengers on payload is referred to as the large aircraft.
fuselage. Fuselages vary greatly in size and
configuration. For exam)le, the fuselage of a
supersonic military airplane may house only one
passenger, the pilot, the remainder of the
fuselage interior space being used to house the
power plant, to provide retracting space for
landing gear, and to house the many mechanical
and electronic installations which are necessary
to fly the airplane and carry out the various
operations for which the airplane was designed
to accomplish. Many groups of engineers with
various backgrounds of training and experience
are therefore concerned with the design of the
fuselage. The structures engineer plays a very
important part because he is responsible for
the strength, rigidity and light weight of the
fuselage structure. Fig. (a)
A20.1
FUSELAGE
PHOTO. NO.2
View Looking Inside of Rear Portion of Fuselage
of Beechcraft Twin-Bonanza Airplane.
PHOTO. NO.3
Fuselage Construction of Boeing
707 Jet Airliner.
(FOR GENERAL DETAILS OF DOUGLAS DC-8 FUSELAGE CONSTRUCTION SEE PAGE A15. 32)
A20.3
A20.3 Stress Analysis Methods. Effective Cross- (1) A small width of sheet w. on each side of
Section. the attachment line of skin to stringer is
It is common practice to use the simplified considered as carrying the same c6mpressive
stress as the stringer, as was discussed in
beam theory in calculating the stresses in the
skin and stringers of a fuselage structure. If Chapter A19. These effective sheet Widths
the fuselage is pressurized, the stresses in the w. are shown as the blackened elements ad-
jacent to the stringers on the compressive
skin due to this internal pressure must be added
to the stresses which resist the flight loads. side in Fig. A20.l.
In wings the skin in the middle region of the
airfoil is relatively flat and thus the skin is (2) The remainder of the curved sheet between
usually considered as made up of flat sheet stringers, namely, b- (w. + we) carries a
panels. In fuselages, however, the skin is maximum compressive stress ocr = .3 E t/r.
curved and curved sheet panels have a higher This value for ocr is conservative. E is
critical compressive buckling stress than flat the modulus of elasticity of the skin
panels of the same size and thickness. In material, t the skin thickness, and r the
small airplanes, the radius of curvature of the radius of curvature of the skin. These
fuselage skin is relatively small and thus the curved sheet elements are shown by the
additional buckling strength due to this curva- hatched skin lengths in Fig. A20.1.
ture may be appreciable. A simple procedure of
approximately including the effect of sheet Since the thin curved skin between the
curvature will now be explained. stringers normally buckles under a compressive
stress far below the buckling strength of the
Fig. A20.l illustrates a distributed stringers, the curved sheet is treated as an
stringer type of fuselage section. Assume that element With varying effective thickness which
external loads are applied which produce bending depends on the ratio of the curved sheet buck-
of the beam about the Y axis With compression on ling stress ocr to the bending stress 0b exist-
the upper portion of the cell. ing at that point for bending of the fuselage
section. Hence the effective sheet thickness
for the curved sheet panels can be written,
- - - - (1)
or an effective area can be written
- - - - (2)
TABLE A20.1
1 2 3 4 5 6 7 8 9 10 11 12
Explanatory Notes for Table A20.l
"
0
II:
1"1
1 ,... 0
"I
~~
.,......
".
.... iO .... TRIAL NO. 1
...."., .....
bll .. Area - ::l a>
., " "'''
.l<l Col. 1 For numbering of stiff-
.." " sq.a in.
.... El
Arm
z'
az' az t2 "I
1
0
.... .......... .....
..." "> ......
az az 2 eners-ind sheet elements, see
Fig. A20.2.
""".... 1>1
" ~
"I
I
"
1>11
iO ..
1>1 ...
0::1>1 - - - 30 x .032 x .032 .12.15+
Col. 2 Stiffener area
2 .15 24.2 3.63 87.8 27.58 -30020 1.0 .032 .152 4.19 115.4 sq. in. For stiffeners
.... 46 .15 22.0 3.30 72.6 25.38 -27700 1.03 .033 .153 3.88 98.5 2, 4, 6, 8, 10. Below
"
.l<l
m 8
.15 18.2 2.73 49.7 21.58 -23550 1.12 .036 .156 3.36 72.4 the centerline each
.15 13.3 1.99 26.4 16.68 -18200 1. 28 .041 .161 2.69 44.7 stiffener is considered
" 10 .15 6.9 1.04 7.2 10.28 -11200 1.66 .053 .173 1. 78 18.3 acting separately. The
.~ 12 .12 0 0 0 3.38 - 3680 2.88 .092 .212 0.72 2.4 entire skin between
t 13 .224 -3.2 -0.72 2.3 -0.18 .224 - .04 0.7 stiffeners is consider-
" 14
~ 15
.120
.224
-6.9
-10.1
-0.83
-2.28
5.7
23.0
-3.52
-6.72 "
....0 ...."
0
.120
.224
-0.42
-1. 52
1.5
10.2
ed as a unit.
Col. 3 All arms z' are measur-
1>1 16 .12 -13.3 -1.60 21.3 -9.92
III ., .12 -1.19 11.8 ~ ed to horizontal cen-
417
., 18
.160
.12
-15.8
-18.2
-2.53
-2.19
40.0
39.8
-12.42
-14.82
""
foo ""
foo
.160
.12
-1. 99
-1. 78
24.7
26.4
terline axis.
"'19
g',,20
;; 21
.16
.12
.16
-20.3
-22.0
-23.7
-3.24
-2.64
-3.80
65.8
58.0
90.0
-16.92
-18.62
-20.32
.."
....
"
"
....
...."
.16
.12
.16
-2.70
-2.24
-3.25
45.6
41. 6
66.0
TRIAL NO. 2
Col. 6 z = distance
axis as found from re-
to neutral
t 22 .12 -24.2 -2.90 70.2 -20.82 ..::" .l<l .12 -2.50 52.1 suIts of Trial No. 1.
m 23 .088 24.9 -2.19 54.5 -21.52 m m .088 -1. 89 40.7 Col. 7 Gb =
1600000 z/1470
SUM _ 2.49 Col. 8 _ effective width based
-12.23 714.3 2.627 -2.86 673 on stress in Col. 7
TABLE A20. 2
TRIAL NO 1 TRIAL NO 2
1 2 3 4 5 6 7 8 9 10 11 12 13 14 15 16 17 18
0 - ...1>1 .0.
'+-l
." ..
II:
,...
. .,
• .0
.. b
,,' ".0
.....
1"1
1
"I
.....;1>1 ,... '"
0"
..
.." "
..::"
mEl "
b' t r
11
... 1>1 ....
'""§
.,
.0
~ bb "<:.: z' az' az t2 -N ill}
• 6'
11
0
a>
0
....I ...
>- (,)
" .. 0::",
.....
• b" ."
.,..::
>",. >< •
..
'".......
".,,,:.: az az 2
...."
1>1 b"'". Po
"b
<
...
... 11
1>1:':
...
... 11
1>1" N 11 0::"":':
II
QJ-I"'4,.c
"
0:: ..
.,"
..::
1 2.27 .032 11 -9000 -31900 .282 .020 24.9 0.50 12.4 28.28 -30800 .292 2.25 .021 .59 16.7
3 4.04 .032 11 -9000 -30300 .296 .038 23.7 0.90 21. 3 27.08 -29500 .305 3.99 .039 1. 06 28.7
'""
.... 5
7
4.04
4.04
.032
.032
24
38
-4510
-2600
-26000
-20200
.173
.128
.022
.016
20.3
15.8
0.45 9.1
0.25 3.9
23.68
19.18
-25800
-20900
.174
.124
3.93
3.80
• 022
.015
0.52 12.3
0.29 5.5
.l<l
";:l
III
9
11
6.04
6.52
.032
.032
38
38
-2600
-2600
-12900
- 4100
.201
.63
.039
.131
10.1
3.2
0.40 4.0
0.42 1.3
13.48
6.58
-14700
- 7200
.177
.362
5.53
4.73
.031
.055
0.42 5.7
0.36 2.4
TOTALS .266 2.92 52.0 TOTALS .183 3.24 71.3
Explanatory Notes for Table A20.2 Explanatory Notes for Table A20.2
Trial No. 1 Trial No. 2
Col. 1, 2, 3, 4 (see Fig. A20.1 for meaning of terms) Col. 12 z - distance to neutral axis
Col. 5 E = 10,300,000 for aluminum alloys as found in results of Trial
Col. 6 0b varies as a straight line from 32000 at No. 1
top of cell to zero at centerline Col. 13 Gb = 1600000 z/1470
Col. 9 z' = distance from centroid of element to Col. 14 based on stress J b of Col. 13
centerline axis of cell
The results of trial No. 1 give a neutral The shear stress ~ = q/t = .0132 Vz /.032 = .413
axis 3.38" below the center line and a moment of Vz
inertia of 1470 in. 4 • In Trial No.2, the ef-
fective sheet widths are based on the moment of The average shear stress on the section would
inertia of 1470. The results of trial No.2 be ~av' = Vz /2ht = Vz /2x50x.032 = .312 Vz ·
give a moment of inertia of 1489 in. 4 with a
neutral axis .135" above the first location. If Thus for this shape of cross-section and
a third trial were used, making use of the 1489 stringer arrangement the maximum shear stress
moment of inertia, the change would be qUite is .413/.312 times the average shear stress or
small since the effect of a small change in approximately 4/3 times as large.
stress on the effective sheet width is negligi-
ble. The procedure as given above is qUite con-
servative relative to the true or actual margin
The compressive stress on stringer No. 2 of safety, because a linear variation of stress
using the resulting moment of inertia and with strain has been assumed and failure of the
neutral axis location, therefore becomes section is assumed to occur when the most remote
stringer reaches its Ultimate compressive stress.
ab = MyZ/I y = 1,600,000x27.45/1489 =29,500 Actually in a static test of a fuselage to
psi destruction, the fuselage section as a whole
will not collapse when one stringer buckles,
The allowable stress was 32000, hence the but will continue to take increasing load until
margin of safety is (32000/29500) - 1 = .08 or other stringers have reached their ultimate
eight percent. If a smaller margin of safety strength. Furthermore, in a typical fuselage
was desired some material would be eliminated structure, stringers of various sizes, shapes
and the calculations of Tables A20.1 and A20.2 and therefore different compressive strengths
would be repeated. are used, and thus to obtain a better measure
of the ultimate strength of a fuselage section,
Calculation of Shear Stress in Skin at Neutral modifications in stress procedures are made to
Axis measure stringer effectiveness. This SUbject
was discussed in some detail in Arts. 11 and 12
The equation for the shear flow q at some of Chapter A19. To illustrate stringer effect-
point on the skin is, iveness in fuselage bending stress analysis, a
simple example problem will be presented.
q qo - Vz Zaz - -' - - - - - - - - - - (3) A20. 5 Ultimate Bending Strength of Fuselage Section.
Iy
Example Calculation.
Due to symmetry of cross-section about the Z Fig. A20.3 shows the cross-section of a
axis the shear flow qo is zero at a point on
the center line Z axis. The summation of the circular fuselage. The Z stringers are arranged
symmetrically with respect to the center line
term az between a point on the Z axis and the Z and X axes.
neutral axis is given in Table A20.3. The
values of areas (a) and arms (z) are taken Three sizes of Z stringers are used as
from Tables A20.1 and A20.2. illustrated in Fig. A20.4 and are labeled S~,
TABLE A20.3
S2 and Sa' These symbols are used on Fig.
A20.3 to indicate where each type of stringer
Element No. az is used. The stringers on each side of the
section are numbered 1 to 13 as shown on Fig •
(1)
(2)
(3)
.021(28.28
.152(27.58
.039(27.08
-
-
-
• 13)
.13)
.13)
··= 0.59
4.17
1,05
A20.3. Fig. A20.5 shows a plot of the stress-
strain curve for the three stringer types loaded
(4) .153(25.38 - .13) = 3.86
in compression and with a column length equal to
·==
(5) .022(23.68 - .13) 0.52
(6) .156(21.58 - .13) 3.34 the fuselage frame spacing. Fig. A20.5 also
(7) .015 (19.18 - .13) 0.28
(8) .161(16.68 ~ .13)
= 2.66 shows a tension stress-strain diagram for the
(9) .031(13.48 - .13) 0.41
(10) .173(10.28 - .13) 1.75 material which is aluminum alloy (2024). The
(11) .055(6.58 - .13) 0.35 ultimate bending strength will be calculated
(12) .212(3.38 -
Total az
.13) 0.69
19.67 for bending which places the upper portion in
compression.
Substituting in equation (3)
A20.7
TABLE A20.4
1 2 3 4 5 6 7 8 9 10 11 12 13
increasing strain, but stringer (3 ) which has to Z. The effective moment of inertia is there
not reached its maximum strength of 39000 fore twice the sum of Column (13) or 3252.
continues to take increasing load.
Calculation of Ultimate Resisting Moment.
Since we wish to use the beam formula ab =
MyZ/I x in computing stresses, we must modify the The maximum stress at the most remote
stringer areas to give a linear stress variation stringer which is number (1) is 36500. From
since the formula is based on a linear stress the beam formula,
variation. The stringer modification factor K
equals the ratio of the true stress in column Mx = ObIx/Z
(9) of Table to linear stress value in column
(4) or K = a/a J.. The results are recorded in = (36500 x 3252)/35.7 + 0.7
column (10). The modified stringer areas are = 3,260,000 in. lb.
then equal to KA and are recorded in column
(ll) . Column (12) gives the first moment of This bending strength when compared to any desi gn
the modified areas about the assumed neutral bending moment about the X axis would give the
axis, giving a total value of -3.16. margin of safety relative to bending strength.
The distance Z from the assumed neutral If the moment of inertia had been computed
axis to the true neutral axis is thus, without regard to non-linear stress variation,
or in other words, using K equal 1 for all
Z = Z KAZ'/ZKA stringers the neutral axis would have come out
_ -3.16 _ 4.9 inches below the centerline axis and the
- 4.159 - - 0.76" moment of inertia would have calculated to be
2382 in. 4 The resisting moment developed
The true N.A. would fall about .70 inches would then be (36500 x 2382)/33.6 = 2,600,000
below assumed position. The effect on total in.lbs. Thus the true strength is 25 percent
greater than the strength for linear stress
sum of column (13) would be negligible, thus
variation. This result explains why such
Table A20.4 will not be revised.
structures test overstrength if designed on
linear stress variation basis.
Column (13) gives the calculation of the
effective moment of inertia with Z 1 being equal After stringer stresses are obtained
A20.9
using the modified areas of Table A20.4, the I y = (15 2 x .1 x 2) + (13.86 2 + 10.61 2 + 5.74 2 ).1
true stringer areas must be used to find the
true stringer loads, which must be used in the x 4 = 180 in. 4
shear flow analysis.
Table A20.5 gives the necessary calculations
A20.6 Shear Flow Analysis for Fuselage Structures for determining the flange bending stresses and
the net total shear load to be taken by the cell
The shear flow analysis can be made once skin. Since the cell is tapered, the stringers
the effective cross-sections of the fuselage have a z component, thus the stringer axial
are obtained. The procedure is the same as was loads help resist the external shear load. The
illustrated for wing structures in Chapter A19. summation of column (8) of Table A20.5 gives
To illustrate, two example problems will be -333.4 lb. for a summation for half the fuselage
presented. section.
Example Problem 1. Symmetrical Tapered Section. Hence, net web shear at station 0 equals:
Fig. A20.6 shows a portion of a tapered VWeb = Vext • + Vnange = 2000 + (2 x -333.4)
circular shaped fuselage structure that might
= 1333.2 lb.
be representative of the rear portion of a
fuselage for a small airplane. Since this
example is only for the purpose of illustrating The results in this particular problem show
shear flow analysis, it will be assumed that that at station 0 the flange stringer system re-
the 16 stringers are the only effective mater- sists one third of the external shear load. At
ial. In an actual stress analysis, the effect- station 150 the web system will resist the en-
ive cross-section would have to be used as tire external shear load of 2000 lb. since the
illustrated in previous articles A20.3 to A20.5. load in the stringers is zero.
The problem will be to determine the In actual design the net web shear should
stringer stresses and the skin shear flow stress be used since in many cases it will decrease
system at Station (0) under a given load system the sheet thickness required one or more gauges.
at Station (150) as shown in Fig. A20.6. Calculation of Flexural Shear Flow.
Solution No. 1 - Solution by Considering Beam Properties
at Only One Section. vZ(web) :6 a z = qo - li~~·2 :6 az
Iy
If the change in longitudinal stringer or
flange material is fairly uniform this method = qo - 7.40 :6 az (A)
can be used with little error in the resulting
shear flow stresses. Due to symmetry of the section about the Z
axis, the flexural shear flow in the web at the
Moment of inertia of section at station (0) center line is zero. Therefore, qo will be taken
about centroidal Y axis: as zero and the summation in equation (A) will
15 t
II
q
5 II
13
II
I
11 7
~
10 8
9 Section at Station
Section at Station 0 14 150" 150
Fig. A20.6
A20.10 FUSE LAGE STRESS ANALYSIS
NOTES:
Col. 4 Db = -Mz/I y = -2000 x 150 x z/180 = -1667z
Col. 5 Total x component of load in stringer member.
For practical purposes, it equals axial load in
stringers since cosine of a small angle is
practically one.
TABLE A20.6
1 2 3 4 5 6 7 8 9 10 11 12 13
Sta. 0 Sta. 30 Sta. 0 Sta. 30 Stringer Load liP Panel Flexural Shear
Stringer Area Area Sta. 0 Sta. 30 -°b=
- °b-= Sta. 0 Sta. 30 30- Taper llPK Flow
No. a a Arm Arm -1667Z -1522Z P x = P x = - (Col. 8 - Col. 9) Corr. 30 q = L: llPK
sq. in. sq. in. z z (psi. ) (psi. ) 0b a °b a 30 Factor 30
K lb./in.
1 .05 .05 15.00 14.00 -25000 -21300 -1250 -1065 6.17 .935 5.76
5.76
2 .10 .10 13.86 12.93 -23100 -19700 -2310 -1970 11. 33 .935 10.60
16.36
3 .10 .10 10.61 9.90 -17700 -15050 -1770 -1505 8.83 .935 8.25
24.61
4 .10 .10 5.74 5.36 - 9550 - 8155 - 955 - 815 4.65 .935 4.34
28.95
5 .10 .10 0 0 0 0 0 0 0 0 0
28.95
6 .10 .10 - 5.74 - 5.36 9550 8155 955 815 - 4.65 .935 - 4.34
24.61
7 .10 .10 -10.61 - 9.90 17700 15050 1770 1505 - 8.83 .935 - 8.25
16.36
8 .10 .10 -13.86 -12.93 23100 19700 2310 1970 -11. 33 .935 -10.60
5.76
9 .05 .05 -15.00 -14.0 25000 21300 1250 1065 - 6.17 .935 - 5.76
NOTES:
Col. 10 Change in axial load in each stringer between stations 0 and 30 divided by distance between Stations. This
result represents the average shear flow induced by the loading up of each stringer between stations 0 and 30.
Col. 11 The width of a skin panel at Station 0 is 5.88 inches and 5.5 inches at Station 30. The shear flow on the edge
of the panels at Station 0 equals (5.5/5.88) llP/30. (See Art. A15. 18 of Chapter A15 for explanation). This
refinement is usually neglected and the average values as given in Col. 10 are used which are conservative.
Col. 13 Due to symmetry of structure, the shear flow is zero on z axis. Thus shear flow at any station equals the
progressive summation of the shear flow values in Col. 12.
second method is recommended for practical will assume the stringers are the only effective
analysis procedure. material. In actual design practice the effect-
iveness of the skin and each stringer would have
Since the section is symmetrical, there are to be considered as explained in Articles A20.4
no moments induced by the in-plane components of and 5.
the stringer forces at station O.
The problem will be to determine the
The torsional shear flow forces are the stringer stresses and the skin shear flow val-
same as in solution method No. 1 and these are ues at station (0) due to the given external
added to the values of column 13 of Table A20.6 loads of Pz = 4000 lb., Py = 1000 lb. and Px =
and give a pattern similar to Fig. A20.7. 1500 acting at station (150) as shown in Fig.
A20.9.
A20.7 Example Problem. Tapered Circular Fuselage
with Unsymmetrical Stringer Areas. SOLUTION:
Fuselage cross-sections are seldom all sym- Since we choose to use the liP method in
metrical relative to stringer and skin areas finding the shear flow system at station (0),
because the practical fuselage has cut-outs such we will find the stringer loads at two stations,
as doors, etc. To illustrate the unsymmetrical namely, station (0) and station (30). The first
case a simplified case will be presented. step is to find the moment of inertia of each
fuselage section about centroidal z and y axes
Fig. A20.9 shows a portion of a tapered and the product of inertia about these axes.
fuselage. The stringer areas are such as to
Table A20.7 (Columns 1 to 11) gives the calcu-
make the cross-sections unsymmetrical relative lations of the section properties for station
to bending material. Again for simplicity, we
FUSELAGE STRESS ANALYSIS
Fig A20 9
=--
d
~
(.1)
--.0::
e
-
a
10.5
Y'
<{,+----''------t-----.---tY'
Y
(.3) (.8)
......-:
-=
Sta. 150
Z
(.2) (.2) g - L- x
(.2)
-
foo
. ~---------150"
o.
l ----------_.I Section at Sta. 150
Z' Z Sta. 0
The skin stringers are located symmetrically with respect to the
Section at Station 0 centerline axes, however the stringer areas as given in ( ) on
the figure are not symmetrical with these axes. It is assumed in this problem that
the stringers taper uniformly between the values as given for station 0 and 150.
The cell would of course have interior transverse frames which are not shown on the
figure.
TABLE A20. 7
1 2 3 4 5 6 7 8 9 10 11 12 13 14
a .60 10.5 -12.00 6.30 66.20 -7.20 86.50 - 75.50 11.36 -14.46 -15080 -441 -9312
b .10 18.98 - 8.48 1. 90 36.00 -0.85 7.20 - 16.10 19.84 -10.94 -21960 -441 -2239
c .10 22.50 0 2.25 50.80 -0 0 0 23.36 - 2.46 -22600 -441 -2304
d .10 18.98 8.48 1. 90 36.00 0.85 7.20 16.10 19.84 6.02 -16742 -441 -1719
e .80 10.5 12.00 8.40 88.10 9.60 115.10 100.80 11. 36 9.54 - 7692 -441 -6506
f .80 -10.5 12.00 -8.40 88.10 9.60 115.10 -100.80 - 9.65 9.54 11958 -441 9216
II: .20 -18.98 8.48 -3.80 72.00 1. 70 14.40 - 32.30 -18.13 6.02 18798 -441 3673
h .20 -22.50 0 -4.50 101. 60 0 0 0 -21. 65 - 2.46 19485 -441 3809
i .20 -18.98 - 8.48 -3.80 72.00 -1. 70 14.40 32.30 -18.13 -10.94 13610 -441 2634
j .30 -10.50 -12.00 -3.15 33.10 -3.60 43.25 37.80 - 9.65 -14.46 4592 -441 1246
Reference Axes Z' and Y' are taken as the centerline axes. General Notes:
(see Fig. A20.9)
Col. 12 0b = 307.0y - 936.1z
Location of centroid and transfer of properties to
centroidal axes. Col. 14 Since the total tensile stresses equal to
total compressive stresses in bending,
Z = -2.90/3.40 = -.855" the sum of Col. 14 should equal the ex-
ternal applied normal load.
y = 8.40/3.40 = 2.46"
Iy = 643.9 - 3.40 x 855 2 = 641. 4
TABLE A20.8
1 2 3 4 5 6 7 8 9 10 11 12 13 14
z = y= 0c = Fa a Ps =
Stringer Area Arm Arm az' az,2 ay' ay,2 az'y' °b
No. a z' y' z' - Z y' - y = -1500/2.98 a(Ob + 0c)
a .50 9.80 -11. 2 4.90 48.1 -5.60 62.8 -54.9 10.90 -13.31 -14800 -503 -7651
b .10 17.72 - 7.92 1.77 31. 4 -0.79 6.3 -14.0 18.82 -10.03 -21090 -503 -2159
c .10 21. 00 0 2.10 44.1 0 0 0 22. 10 - 2.11 -21447 -503 -2195
d .10 17.72 7.92 1.77 31. 4 0.79 6.3 14.0 18.82 5.81 -15647 -503 -1615
e .66 9.8 11. 20 6.47 63.2 7.40 83.0 72.5 10.90 9.09 - 7088 -503 -5011
f .66 - 9.8 11. 20 -6.47 63.2 7.40 83.0 -72.5 - 8.7 9.09 -11282 -503 7100
g .20 -17.72 7.92 -3.54 62.8 1. 58 12.6 -28.0 -16.60 5.81 -17583 -503 3416
h .20 -21. 00 0 -4.20 88.2 0 0 0 -19.90 - 2.11 17943 -503 3489
i .20 -17.72 - 7.92 -3.54 62.8 -1. 58 12.6 28.0 -16.60 -10.02 12135 -503 2327
j .26 - 9.8 -11.20 -2.55 25.0 -2.92 32.6 28.6 - 8.70 -13.31 3570 -503 797
NOTES:
- -- N Z'
(j)
Reference Z' and Y' axes are taken as the centerline axes. r0-
b d
Z = -3.29/2.98 = -1. 10" .10 .10
7f"'Y.2"
Y= 6.28/2.98 = a
-Iz e Section at
It'·
2.11 .50 _.66 Station 30
I y = 520.2 - 2.98 x 1. 10 2 =516.6 lL.~
~Y
, -1.1
c-= b-- -~
I z = 299.2 - 2.98 x 2.11 2 = 286.0 1t Y=
9.8 2.11
I zy = -26.3 - 2.98 x 2.11 x -1. 10 = -19.4 j .26 .66 f
\ I 1 . "
FIg. A20.10
(0) and the similar columns of Table A20.8 gives My = 4000x120+1500x8.10 = 492150 in.lb.
the calculations for station (30). Mz = -1000 x 120 + 1500 x 2.11 = 116820 in .lb •
Before the bending and shear stresses can Pn = -1500 lb., Vz =4000 lb., Vy = -1000
be calculated, the external bending moments, lb.
shears and normal forces at stations (0) and (30)
must be known. Calculation of Bending Stresses.
Stat i on (0):
At station (0): -
The bending moment about y neutral axis at
station (0) equals, where
My = Pz (150) Px (7.85)
+
K1 = Iyz/(IyI z - I yz 2)
= 4000 x 150 + 1500 x 7.85 = 611800 in. lb. ~ = Iz/(IyI z I yz 2)
Mz = Py (150) - Px (2.46) K" = Iy/(IyI z I yz 2)
= -1000 x 150 + 1500 x 2.46 = -146310 in. lb. Substituting values from Tables A20.7 and
A20.8:
The shears at station (0) are Vz = Pz =
4000 lb. and Vy =
Py = -1000 lb. K1 = -30.55/(641.4 x 382.6 - 30.55 2 ) =
The normal load Pn at station (0) referred = -30.55/244670 = -.0001248
to centroid of section equals ZPx -1500 lb. =
K2 = 382.6/244670 = .00156
In a similar manner, the values at station
(30) are, (see Fig. A20.10) K" = 641.4/244670 = .00262
A20 14 FUSELAGE STRESS ANALYSIS
SUbstituting K values in equation for ab: moments in the plane, of all internal and ex-
ternal forces must be zero. Column (7) of Table
ab = - [00262 x -146310 - (-.0001248 x A20.9 gives the moment of the flexural shear
about this point. (See notes and Fig. below
611800)J y - [.00156 x 611800 - Table for explanation.)
(-.0001248 x -146310)J z
TABLE A20.9
whence SHEAR FLOW CALCULATIONS
Station (30): q~ ~ ~
= -19.4/147620 = -.0001315 a
b
-9312
-2239
-7651
-2159
55.37
2.67 55.37
150.04 8300 -52 3.37
-2195 58.04 202.46 11820 -52 6.04
c -2304 3.63
K 2 = 286/147620 = .001936 d -1719 -1615 3.47 61.67 202.46 12500 -52 9.67
-52
e -6506 -5011 49.83 65.14 150.04 9780 13.14
K 3
= 516.6/147620 = .0035 f 9216 +7100 -70.53 114.97 252.0
150.04
29000
6660
-52
-52
62.97
g 3673 3416 - 8.57 44.44 - 7.56
35.87 202.46 7270 -52 -16.13
ab = - [:0035 c -116800 - (-.0001315 x h 3809 3489 -10.67
-52
i 2634 2327 -10.23 25.20 202.46 5100 -26.8
492150] y - [001936 x 492150- 1246 797 -14.97 14.97 150.04 2240 -52 -37.03
j
-52 -52.0
(-.0001315 x 116820 II z
a 0 252.0
Sum
0
92670
whence NOTES:
Since an external load of 1500 lb. is act- Col. (6) m ~ double areas
ing normal to the sections and through the (see Fig. a).
Col. (7) mq ~ moment of
section centroids, an axial compressive stress shear flow q on
a c is produced on the sections. (See Columns each web element
13). The total load Ps in each stringer equals about 0' (Fig. a)
the area of the stringer times the combined
bending and axial stresses. (See column 14 of
each table).
Calculation of Flexural Shear Flow q. Moments Due to In Plane Components of Stringer Loads.
Table A20.9 gives the necessary calcula-
tions to determine the shear flow at station (0) Since the stringers are not normal to the
based on the change in stringer loads between section at station (0), the stringers have in-
stations (0) and (30). The correction of the plane 'components which may produce a moment
average shear due to the taper in the skin about the intersection of the symmetrical axes
panels as was done in example problem (1), which has been selected as a moment center.
Table A20.6, column (11), is omitted in this Table A20.10 gives the calculations for the in-
solution since it tends toward the conservative plane components and their moments about point
side. Since the effective cross section is un- 0' .
symmetrical, the value of the flexural shear Moment of External Load System About Point (0').
flow q at any point is unknown thus a value for
q at some point is assumed. In Table A20.9 The 1000 lb. load at station (150) acting
the shear flow q in the web aj is assumed zero. in the Y direction has a moment arm of 7" about
Column (5) gives the results at other points the point 0' of station (0).
under this assumption.
Hence external moment = 1000 x 7 = 7000
Moment of Shear Flow about Intersection of in. lb.
Centerline Axes
Therefore the total moment about the
For equilibrium in the plane of the cross assumed moment center 0' =
section at station (0), the summation of the
A20.15
;1;'
TABLE A20.10
1 2 3 4 5 6 7 8
Py = Mo = P z = Mo =
Stringer P x ~ P~ dz Px~
No. (lbs. ) dx xdx P y z' dx dx P z y'
NOTES: 115
94 72
Col. (2) from Table A20. 7 c
+r IT
Col. (3) equals the slope 217 2,3 Fig. A20.11
of stringers
in y and z ~48
Ft 1?l
18.98"
directions. Shear flow distribution.
(see Fig. A20. 9)
H2'-'-l10.5" ~
Col. (5) Values of z'
y' ~-~-- where large concentrated loads are applied can
and (8) and y' from o Fig.b W be determined by the procedure given in Articles
Table A20. 7. 33~ j : I M 18 to 20 of Chapter A19. A more rigorous
Fig. b shows ~-I f ~45
29 analysis can be made by the application of the
the P y and Pz .00 I g 214 basic theory as ~iven in Chapter A8.
components from 49 1 h 69
Cols. (4) and (7). 154
111190 The problem of shell stresses due to in-
Total moment about 0' = Looking Toward Sta.150 ternal pressures is presented in Chapter A16.
-1634 + 1612 = -22"# The strength design of the fUselage skin in-
volves a question of combined stresses. The
broad problem of the strength design of struc-
92670 due to shear flow q tural elements and their connections under all
-22 due to in plane components of types of stress conditions is covered in Volume
stringers. II.
7000 due to the external loads.
A20.9 Problems.
Total= 99648 in. lb.
(1)
Therefore for equilibrium a moment of
-99646 is required which can be provided by a
constant shear flow q ~ around the cell, hence
I
_ T _ -99648 = -52 Ib./in. (957 =
20"
q ~ - 2A - 2 x 957
-~"-----
Skin .035
enclosed area of cell)
ADDITIONAL DATA. Area stringer 8~ = .12 sq.in.; The stresses that are found in the
82 =.25 sq. in.; S" .08 sq.in. E 10,500,000 = = stringers or longerons of a typical fuselage by
psi •. use of the modified beam theory or by the more
rigorous theory of Chapter AS, are referred to
(3) Fig. A20.15 shows a tapered circular as primary stresses. Because of the necessity
fuselage with 8 stringers. The area of each of weight saving, most fuselage structures are
stringer is 0.1 sq.in. Assume stringers develop designed to permit skin buckling, which means
entire bending resistance. Find the axial load that shear loads in the skin are carried by
in stringers at station (110) due to Pz and Px diagonal semi-tension field action. This
loads at station (0). Also find shear flow diagonal tension in the skin panels produces
system at station 110 using ~P method. Use additional stresses in the stringers and also
properties at station (90) IN OBTAINING AVERAGE in the fuselage rings. These resulting stresses
SHEAR FLOWS. are referred to as secondary stresses and must
be properly added to the primary stresses in
P z = 2300# Pz
1
r============:if
I-----------i 7
9 fl
n"~"
Px=500#
the strength design of the individual stringer
or ring. Chapter Cll covers the SUbject of
these secondary stresses due to diagonal semi-
tension field action in skin panels. It is
suggested to the stUdent that after studying
l:==~======::::::::::~
L Sta.O 6 5 ~ Chapters A19 and A20, that Chapters C10 and Cll
5 ta.110 110,, .1 be referred to in order to obtain a complete
stress picture for skin covered structures.
Fig. A20.l5
CHAPTER A21
LOADS AND STRESSES ON RIBS AND FRAMES
A21.1 Introduction. For aerodynamic reasons the loads into the fuselage shell such as those
wing cantou, in the chord direction must be from landing gear reactions, wing reactions,
maintained without appreciable rJistortion. tail reactions, pONer Dlant reactions, etc.
Unless the wing skin is quite thick, spanwise The dead wei";ht of all the payload and fixed
stringers must be attached to the skin in order eqUipment inside the fuselan'e must be carried
to increase the bending efficiency of the wing. to frames by other structure such as the
Therefore to hold the skin-stringer wing surface fuselage floor system and then transmitted to
to contour shape and also to limit the length the fuselage shell structure. Since the dead
of strinc\ers to an efficient coltunn compressive weight must be multiplied by the design accel-
stren,a;th, internal support or brace units are eration factors, these internal loads become
reqUired. These structural units are referred qUite lanse in mo!:nitude.
to as wing ribs. The ribs also have another
major purpose, nam81y, to act as a transfer or Another important purpose or action of ribs
distribution unit. All the loads apnlied to and frames is to redistribute the shear at dis-
th8 wina; are react8d at the wing supporting continuities and Dractical Wings and fuselages
points, thus these applied loads must be trans- contain many cut-outs and openings and thus
ferred into the wing cellular c'tructure com- discontinuities in the basic structural layout.
posed of skin, stringers, snars, etc., anrJ then
reacted at the win': support Doints. The 3TJTJlied A21. 2 Types of Wing Rib Construction.
loads may be only the rJistributed surface air-
loads which requi re relativ!31y light int8rnal FiGS. A21.1 to 6 illustrate the COffinon
ribs to pr'ovids this carry through or transfer types of winG construction. FiG' 1 illustrates
reqUirement, to rather rugged or heavy ribs a sheet metal channel for a leading edge 3
which must absorb and transmit large concen-
trated applied loads such as those from landing
gear reactions, power GIant reactions and fuse-
lage reactions. In between these two extremes +
of applied load mao;nitudes are such loads as
O~O
reactions at supporting points for ailerons,
flaps, leading edge high lift units and the
many internal dead wei,:ht loads such as fuel
and military armament and other insta1l8tions.
Thus ribs can vary from a very light structure Web
which serves primarily as a former to a h3avy
structure which must receive and transfer loads Fig. A2L 1
involving thousands of pounds.
Wing Skin
Fig. A21. 6
stringer, single spar, single cell wing were used to transmit external loads into the
structure. The rib is riveted, or spot-welded, wing cellular beam structure. Concentrated
or glued to the skin along it boundary. Fig. external loads must be distributed to the rib
2 shows the same leading edge cell but with before the rib can transfer the load to the
spanwise corruga~ions on the top skin and wing beam structure. In other words, a con-
stringers on the bottom. On the top the rib centrated load applied directly to the edge of
flange rests below the corrugations, whereas a thin sheet would cause sheet to buckle or
the stringers on the bottom pass through cut- cripple under the localized stress. Thus a
outs in the rib. Fig. 3 illustrates the gen- structural element usually called a web stiff-
eral type of sheet metal rib that can be ener or a web flan~e is fastened to the web and
qUickly made by use of large presses and rubber the concentrated load goes into the stiffener
dies. Figs. 4 and 5 illustrate rib types for which in turn transfers the load to the web.
middle portion of wing section. The rib To ~et the load into the stiffener usually re-
flanges may rest below stringers or be notched quires an end fitting. In general the distri-
for allowing stringers to pass through. Ribs buted air loads on the wing surface are usually
that are sUbjected to considerable torsional of such magnitude that the loads can be distri-
forces in the plane of the rib should have some buted to rib web by direct bearing of flange
shear ties to the skin. For ribs that rest normal to edge of rib web without causing local
below~tringers this shear tie can be made by buckling, thus stiffeners are usually not
a few sheet metal angle clips as illustrated in needed to transfer air pressures to wing ribs.
Fig. 5. Fig. A21.7 shows an artist's drawing
of the wing structure of the Beechcraft Bonanza EXAMPLE PROBLEM ILLUSTRATING TRANSFER OF CON-
commercial airplane. It should be noticed that CENTRATED LOAD TO SHEET PANEL.
various types and shapes of ribs and formers
are required in airplane design. Photographs Fig. A21.8 shows a cantilever beam com-
A21.1 to 3 illustrate typical rib construction posed of 2 flanges and a web. A concentrated
in various type aircraft, both large and small. load of 1000 lb. is applied at point (A) in the
Since ribs compose an appreciable part of the direction shown. Another concentrated load of
wing structural weight, it is important that 1000 lb. is applied at point (E) as shown.
they be made as light as safety permits and
also be efficient relative to cost of fabrica- To distribute the load of 1000 lb. at (A),
tion and assembly. Rib development and design a horizontal stiffener (AB) and a vertical
involves considerable static testing to verify stiffener (CAD) are added as shown. A fitting
and assist the theoretical analysis and design. would be required at (A) which would be attached
to both stiffeners. The horizontal component
A21. 3 Distribution of Concentrated Loads to Thin of the 1000 lb. load which equals 800 lb. is
Sheet Panels. taken by the stiffener (AB) and the vertical
component which equals 600 lb. is taken by the
In Art. A21.1 it was brought out that ribs
A21. 3
vertical stiffener CD. The vertical load at E Figs. A21.9 and A21.10 show free bOdies of that
would be transferred to stiffener EF through portion inclUding web panels (1) and (2) and
fitting at E. The problem is to find the shear stiffeners CAD and AB and the external load at
flows in the web panels, the stiffener loads (A) . In Fig. A2l. 9 the shear flows q ~ and q 2
and the beam flange loads.
~ 10"--t-- 10"
G
-+--
flange
10" ----t- 10"-----1
F I
r q~
tr---"'=-~"'=-nD
I~- -D
q~
PHOTO. A21. 1 Type of Wing Ribs Used in Cessna 180 Model Airplane, a 4 Place Commercial Airplane .
• • • ,6" • t
\
1 '
PHOTO. A21. 2 Rib Type Used in Outer Panel-Fuel Tank Section- of Douglas DC-8 Commercial Jet Airliner.
PHOTO. A21. 3 Rib Construction and Arrangement in High Speed, Swept Wing, Fighter Type of Aircraft.
North American Aviation - Navy Fury - Jet Airplane.
A21. 5
L:F): = -50 x 10 + 10q2 = 0, whence q 2 = The shear flow q3 in web panel (3) is ob-
50 Ib./in. tained by considering stiffener EBF as a free
body, see Fig. A21.16.
Combining the two shear flows for the two loads,
ZFy = 12q3 + 10 x 3 - 9 x 70
q:l. = 20 + 50 = 70 Ib./in.
- 1000 = 0
50 = 10 Ib./in.
whence, q3 = 133.33 Ib./in.
Fig. A21.11 shows the results. Fig.
A21.12 shows stiffener AB as a free body, and The shear flow q3
Fig. A21.13 the axial load diav,ram on stiffener could also be found by
AB, which comes directly from Fig. A21.11 by treating entire beam to
starting at one end and adding the shear flows. right of section through
panel (3). For this free
body,
r--q-1.-=~
770~~D
800#(tension) Fig. A21.16 = -600 - 1000 + 12q 3 = 0
L:F y
,e) B~ Fig. A21. 13
whence, q3 = 133.33
q, =70
A 800 B' =-___.800
- -~A Fig. A21.17 shows diagram of axial load in
600 q2 = 10 stiffener EF as determined from Fig. A21.16 by
C starting at one end and adding up the forces to
Fig. A21.11 Fig. A21. 12 any section.
Fig. A21.14 shows a free body of the After the web shear
F flows have been determined
vertical stiffener CAD, and FiG' A21.15 the ~ Fig. A21. 17
axial load diagram for the stiffener. the axial loads in the
beam flanges follow as
B~lb. the algebraic sum of the
shear flows. Fig. A21.18
E 1000 lb. shows the shear flows
(tension)
along each beam flange
as previously found. The
upper and lower beam flange loads are indicated
by the diagrams adjacent to each flange.
Fig. A21. 15
4700 r-----
- -
-
700 lb.
.-
13i33- 133.33 133-:33- 70
consider free body in Fig. A21.15a. /
/
./ -800
f- 10"-1 / 133.33
F_..h.-_ D ..!31:~ ~ 1~3.:Jt. - -- -10- - 600
1000
I 100# tenslOn
I 9" ........----===
~;g.A21.18
qq
Fig. A21. 15a
~y
~~I
I~B===~IFA.:...-800#
600 lb.
-+
3" 3900# compression
E- - C -L
q2
In this example problem the applied extern-
ZF x = 800 - 10q, (1) al load at point (A) was acting in the plane of
the beam web, thus two stiffeners were suffi-
L:F y = -600 + 9q:l. 3q" =0 - (2) cient to take care of its two components. Often
A21. 6 LOADS AND STRESSES ON RIBS AND FRAMES
z
~y
x
ZM~ = -Ph + 2Aq =0 The rib in the leading edge portion of the
wing as illustrated in Fig. A21.22 will be
hence analyzed.
Solution:
~\~
~
Area
A = 139.3 sq. in.
I
,
l~r 10" Fig. A21. 25
! 160
~===_====.=::::::::::.==-(
1) P = 213. 2 lb.
Fig. A21. 23
Bending moment at section B-B = To find Gadc' take moments about point (b)
8x2.5x1.25+4.42x2x15.4 = 161 in.lb.
A21. 8 LOADS AND STRESSES ON RIBS AND FRAMES
= 157.3
74.611,
.I~d
9.3"
- 9.3 q~d =0
whence, Qadc
assumed.
lb./in. with sense as
2034-4 1_
a t-
ell
3000 Whence, q~d = 463 lb./
in.
To find qcb take ZF z =0 5000
Fig. A21. 27
ZF z =5000 + 200 - 157.3 x 11.5 - 11.5
To find flange
load C' take ZFx 0, =
qcb =0 considering joint (a) as a free body,
Whence, qcb = 295 lb./in. ZFx = 2034 + 3000 - C' = 0, whence, C' 5034 lb. =
At joint (d) T' obviously equals 2158 lb. The
To find qba take ZFx =0 stiffener ad carries a compressive load of 5000
lb. at its (a) end and decreases uniformly by
ZF x = -500 + 3000 + 500 - 157.3 x 15 the amount equal to the two shear flows or
-15 qba =0 463 + 74.6 =
537.6 lb./in.
~T'
(1-
t I 9.3"
Fig. A21. 26 t~d I
~::::::;:==~ ----l...- e '
a./<3000
To find flange load T take moments about 15000
point (a), Fig. A21. 28
Fig.A21. 29
SOLUTION:
To find flange load C take moments about The total shear load on the wing in the Z
point (b). direction equals Vz = -6000 - 5000 + 2000 =
-9000 lb. and Vx = -8500 + 7500 - 4000 + 4500 =
mb = -157.3x2 (160+60) + 5000 x 15 -500 lb.
-11.5 C = 0 The boundary forces on the rib will be
whence, C =500 lb. equal to the shear flow force system on the
cell walls due to the given external force
To find flange load T take ~Fx =0 system.
1 2 3 4 5 6 7 8 9 10 11
When there are more than three spanwise
stringers in a wing, there are four or more Flange Area
No. A Z' X' AZ' AZ,2 AX' AX,2 AX' z' Z=Z'- Z X=Xl_X
panels in the cell walls, thus the reactions of
the cell walls upon the rib boundary cannot be a 2.00 5.50 0 11.00 60.5 0 0 0 6.36 -11.8
found by statics as was possible in the 3 b 1. 25 3.55 30 4.43 15.72 37.50 1125 132.9 4.41 18.2
stringer case of the previous example problem.
c 1. 15 -8.40 30 - 9.66 82.20 34.50 1035 -289.8 -7.54 18.2
Fig. A21.30 illustrates a wing section con- d 1. 70 -6.50 o -11. 04 71. 90 0 0 0 -5.64 -11.8
sisting of four spanwise flange members. The ~ 6.10 - 5.27 230.3 72.00 2160 -156.9
concentrated loads acting at the four corners
of the box might be representative of reactions
from the engine mount or nacelle structure and z =~Z'/~ =-5.27/6.10 = -.865"
x- = ~'/~ = 72.0/6.10 = n.8"
the reactions from a rib which supports the
wing flap. These loads must be distributed
into the walls of the wing box beam which neces-
sitates a rib. Before the rib can be designed, Centroidal x and z moments of inertia:
the bending and shear forces on the rib must be
determined. The calculations which follow
Ix =230.3 - 6.10 x .865 = 225.8 2
With the wing section properties known, ZM c .g. = -llOOO x ll.8 - 8500 x 6.36 - 7500 x
the constants K can be calculated. 5.65 - 4000 x 4.41- 4500 x 7.54 - 2000 x 18.2 - 444 x
K1 = 1xz/(1x1 z - 1xz 2
) 2x90-748x2xl08.7-461x2x99 =-648400 in.lb.
= -94.7/(225.8x1310-94.7 = 2
)
F0r equilibrium ZMc • g • must equal zero,
-94.7/286700 = -.00033 therefore a constant flow shear ql acting around
the rib perimeter is necessary which will pro-
K2 = 1z/286700 = 1310/286700 = .00456 duce a moment of 648400 in. lb.
K3 = 1x/286700 = 225.8/286700 = .000786 M
ql = 2A = 2648400
x 368.5 = 880 lb./in.
SUbstituting in equation (1),
(Note: 368.5 = total area of cell)
qy = - G000786 (-500) - (-.00033) (-90008
ZxA - [00456 (-9000) - (-.00033)(-5001 Adding this shear flow to that of Fig.
A21.31, the resulting force system of Fig.
ZzA A21.32 is obtained. The reactions of the beam
cell walls on the rib have now been determined
whence, and the bending moments and shears on the rib
can now be calculated.
qy = 3.363 ZxA + 41.205 ZzA - (2)
B
rooo Fig. A21. 31 - ' - - Fu
850() a ~= 444
~ ~ , I -;-; , - ; - - - b 00
d6 = -'~~~rea=900" ~/I' ~ 4 41
~. ~rea -~.~~ t~
I - 70.8 1 / ~rea = 108. 7 lex> r
5.64 - /IL- 7,/// ~ 754
- ~Area = 99 / til'
7500 d-~_ I I 1\ ~
~ -t=-
--q :46lib7in.-
C
4liOO -t
5000 2000 Fig. A21. 33 Fig.A21. 34
l--- 11.8 18.2 -----l
I Total Cell
Area = 368. 5°" Moments at section B-B will be referred to
the point (0):
The moments of the forces in the plane of ZMO = -llOOO x 11. 8 - 8500 x 6.36 - 7500 x 5.64 + 436
the rib will now be calculated: x 2 x 36 + 880 x 2 x 70.8 + 419 x 2 x 38.3 =
Taking moments about the c.g. of the beam -38200 in. lb.
cross section (See Fig. A21.31):
A21. 11
The resultant external shear force along the boundary forces on a rib located adjacent
the section B-B equals the summation of the z to a cut-out is to find the applied shear flows
components of all the forces. in the wing on two sections, one on each side
of the rib. Then the algebraic sum of these
v = ZFz = -11000 + 12 x 880 - 436 x 0.36 + 419 two shear flows will give the rib boundary
forces. With the boundary forces known the rib
x 0.96 = -195 lb.
web and flange stresses can be found as pre-
The resultant load normal to the section viously illustrated. The procedure can best
B-B equals the summation of the force compon- be illustrated by example problems.
ents in the x direction. A21. 8 Example Problem. Wing with Cut-Out
Subjected to Torsion.
H = ZF x = -8500 + 7500 + (436 - 419) 11.8
Fig. A21.35 shows a rectangular single
= -800 lb. cell wing beam with four stringers or flanges
located at the four corners. The upper surface
Fig. A21.34 shows these resultant forces
skin is discontinued in the center bay (2).
referred to point (0) of the cross-section. If
The wing is SUbjected to a torsional moment of
we assume that the rib flanges develoD the
entire resistance to normal stresses·, we can
find flange loads by simple statics. STA.O
cS"
~ ~
T: jRlb I' 1 •
To find upper flange load Fu take moments ~'rt <Ii 'I
I
1 _
about lower flange point. 'I II
STA. 2~ 1~1j /,
= 12.6 =0
~~f:/1
ZM Fu -38200-800x6.6
Rib
whence, Fu = 3443 lb. tension /""
"'~ .~';"V 1-------:;;7';t--f-::!L Upper skin
<no ":) ~ surface re-
~ ~ moved in
To find FL use ZF x = 0
STA.5¥ 4,,,0. /, 4000 lb. Bay (2)
ZF x = 3443 - 800 - FL = 0, whence FL = B ~ ! I( Rib (A) I, I "
2643 lb. compression. ~
:. <Ii If II
1 ;-- - - - / , -I" 4000 lb.
STA~O I I; I, II
The shear flow on web equals V/12.6 = .
8 1/ Rib
~
\ 80000 in. lb. /
j
Fig. A21. 35
195/12.6 = 15.5 Ib./in. This result neglects
effect of flanges not being normal to section
B-B, which inclination is negligible in this
case.
80000 in. lb. at Station (70) and a couple force
If the entire cross-section of rib is at Station (50) as shown in Fig. A21.35. The
effective in bending, then the web thickness problem will be to determine the applied forces
and flange sizes of the rib would be needed on rib (A).
to obtain the section momGnt of inertIa which
is necessary in the beam equation for bending SOLUTION:
stresses. The forces at (0) would then be
referred to neutral axis of section before The applied shear flow on the cell walls
bending and shear stresses on the rib section will be found for two cross-sections of the
could be calculated. Wing, one on each side of rib (A).
To obtain a complete picture of the web In bay (1) the torsional moment M is 80000
and flange forces, several sections along the in.lb. The applied shear flow on a cross-
rib span should be analyzed as illustrated for section of the wing in bay (1) thus equals,
section B-B.
M
q -_ 2A = 80000 = 100 Ib./in.
A21. 7 Rib Loads Due to Discontinuities in Wing 2 x 10 x 40
Skin Covering.
This shear flow system is shown on Fig.
As referred to before, ribs in addition to A21.36 which is a free body of rib (A). In
transmitting external loads to wing cell bay (2), since the top skin is removed, the
structure are also a means of re-distributing torsional moment must be taken by the front and
the shear forces at a discontinuity, the most
rear vertical webs, since any shear flow in the
common discontinuity being a cut-out in one or
bottom skin could not be balanced.
more of the webs or walls of the wing beam
cross section. The usual procedure in finding The torsional moment in bay (2) is,
A21. 12 LOADS AND STRESSES ON RIBS AND FRAMES
Bay (2)
Jf..:..:.=-----""-----7'
Rib A
t__ 4000# Bay (1) --1--- - - - - L = 30"----+1+_ Bay (3)
t
= 100 t
,+- - --;
'/- - ..9. -
100
-
- -, :
-llJOll
t q'=300 lO" P~i 3000
h =T Fig. a
q t/-.~ ..-+.-+ - ---. 4000# "1 P-'?====-- r:::-P
Deflected Front Bea;;Cmm======d~
u. _V=3000
P
q = 100
Fig. A21. 37
--='!'-=="'-q----='="--l~~""O~-""__=_""_'-=_ _ _ 4000
flange loads at the beam ends are P = 45000/10
= 4500 lb o (See Figo a).
q = 200 11 _ _ _ _ _ _ _ Ii q = 200
-4000
The deflection of the rear beam would be
q = 100 the reverse of Fig. a, and thus all forces
would also be reversed.
up and on the other side a shear flow of 300 Fig. A21.39 shows bay (1) of the wing as a
is acting down, thus the rib web must take the free body acted upon by the flange loads due to
difference or 200 acting down. On the right end bending of the beams in bay (2). These internal
of the rib the load on the rib web is 200 Ib./ flange forces from bay (2) must be held in
in. up. The loads on the top and bottom equilibrium by the internal stresses in the ad-
flanges of the rib is obviously 100 Ib./in. jacent wing structure of bay (1).
Fig. A21.37 shows the loads applied to the rib
boundary when the torsion in bay (1) and the r§ <§'
external couple force is transferred to the ::J ::J
r§
cross-section of bay (2).
STA.50 - - , -
I 1/
I
I /~
,cRIb (A)
,~
I
/
::J
ADDITIONAL EFFECTS DUE TO DIFFERENTIAL BENDING y" II~ 1/
OF BEAMS IN BAY (2). $ J] / ;./~ /7
t:I:; I I ,I 11/ h
The torsion in bay (1) and the external ST A. 70 - L ,: /
I
.f! /, / Bay (1)
couple force are thrown off as couple force on II h
the front and rear beams of middle bay (2), ~ ~t
flange member. To find the slE~ar flow on the bay (1) as shown, The areas of corner stringers
cross-section the front web is first assumed a, b, c and d are shown in ( ) adjacent to each
cut, and thus the static shear flow qs = Z6P stringer.
from cut face where qs is zero. Fig. A2l.40
shows this static shear flow.
8800 lb.
No Skin on
Bottom of
Bay (2).
r--15"~
Fig. A21. 40 '-.:
~.'
~ ~:I Rib
j:2;'00
t<:;'" ~ I
-l __ /-- T
1"
1 1/
lower left hand corner of the qs force system, 4,~_ / - - - - - --
I 1------
=
M 225x40xlO = 90000 in.lb. For eqUili- I /
brium a moment of -90000 is necessary •. There- &., Rib
fore a constant shear flow system q must be ~ ~
t<:;'" }------
added to develop a moment of -90000. Thus
q = M/2A = (-90000/2xlOx40) = -112.5 lb./in. I (1) / (0.5)
Adding this shear flow to that for qs in Fig.
A21.40 gives the final values in Fig. A21.41.
fa / b t
12" / Rib 8" Fig. A21. 43
This shear flow system represents the stress
-h)~/d~-----c----:: -1
(0.5)
I~ 30" --_>1
112.5
312.5
tl
t
- - -If
212.5
-4000#
1312.5
is similar to the previous example involving
wing torsion only.
'c-.-=::-:;;=--=:-::;;:::-::;=-:;=- t __ 4000# We will first calculate the shear flow in
212.5 wing bay (1). Fig. A21,44 shows the cross-
Fig. A21. 42
section.
8800#
known, the rib flange and web stresses can be
found as previously explained. b-15"
1
-,- (0.5)
A21. 9 Example Problem. Wing with Cut-Out Subjected 6"
to Bending and Torsional Loads. x---t-- b
x
-f'
single cell cantilever beam composed of 3 bays T~) _ I
formed by the four ribs. The loads on the jX ---
structure consist of loads applied to end of rIO" Iz 30" Fig. A21. 44
A21. 14 LOADS AND STRESSES ON RIBS AND FRAMES
The section moments of inertia are needed Therefore we consider bay (2) in its true
in calculating shear flows. condition with bottom skin removed. Fig.
A21.47 shows the cross-section of bay (2).
2
in~
2
Ix = (lx6 x2) + (0.5X4 X2) = 88
I .8800
X = 'ZAx/'ZA = (1 x 30)/3 = 10 in. I- 15,,--J
qcb = 640 -100 (-4) (0.5) - 4 (20) 0.5 = 800 'ZFz = 8800 - 8 x 640 + 2 x 80 - 12 Clad = 0
whence Qad = 320
qba = 800-100 (4)(0.5) -4 (20) 0.5 = 560
Fig. A21.48 shows the results. This shear
Fig. A21.45 shows these static shear flows. flow system is the final or true shear on bay
(2) •
L .4- 8800
I[a-
,- 15'~
80 lb. /in.
a --- _.~ .!!::Ji~
cut
bl
r 320 I blfi, 640
2~ A=Cell Area = 300 c t 800 Id c
~ l:d~--,=-==--:;::=-=;:::::::---::;;=~
-~-640-- Fig. A21. 48
Fig. A21. 45
Since we have a channel or ODen wing cross-
section in bay (2), any torsional moment on
To this shear flow, a constant shear flow this bay must be transmitted by differential
must be added to make ZM = O. Take moments bendin~ of the front and rear beams. To obtain
about Doint (d). the torsional moment on bay (2), the shear
center location must be knoNll.
'ZMd = -8800 x 15 + 2400 x 3 + 560 x 30 x 12 + 800
x 8 x 30 = 268800 in. lb., or -266800 Horizontal location of shear center: -
Assume the section bends about centroidal X axis
is required for eqUilibrium, hence the required without twist under a Vz load of 8800 lb.
constant shear flow q = -M/2A -268800/2 x 300=
= -448. Adding this shear flow to that of Fig.
q =- ~~ 'ZzA, or q =- 100 'ZzA
A21.45, we obtain tha shear flow of Fir;. A21.46.
qcb
v
=- ~
Iz
l:xA =-
= -4x20xO.5
4 l:xA
=- 40 Ib./in. ~
71 1----_
L /
qba = -40 - 4 x 20 x 0.5 = - 80
Fig. A21. 52
Fig. A21.50 shows the shear flow results. ~I /
~
R=2400~ ~
~=--4"
~t~-;::=----:;8p::,0-=---- b
l40 40~ uniformly in bay (1) over a distance of 30
d inches, or the shear flow per inch produced by
Fig. A21. 50
c these flange loads equals ~P P/30, whence =
~Pa = ~Pd = 4000/30 =133.3 and ~Pb = ~Pc
The vertical distance Z from point (a) to
the line of action of the resultant which = 6000/30 = 200 lb.
locates the vertical location of shear center
is, Fig. A21.53 shows an element of bay (1)
one inch wide with these ~P loads. The shear
flow q assuming the front web cut equals l:~P.
Z = L:Ma /2400 = (40 x 8 x 30)/2400 = 4 in. The resulting static shear flows which equals
l:~P is shown in Fig. A21.53.
Fig. A21.51 shows the shear center location
and the external loads. The moment about the
shear center which equals the torsion on the
wing bay (2) equals, lOlL
" s.c.
1_ Ql~.554
8800
4
, t--
133.3
---
133.3 I
200
2400 I9 ~~
.n
d
Fig. A21. 51
The moment of this shear flow system about
point (d) =
133.33x30x12-66.7x8x30 = 31980.
For L:M = 0, we need a constant shear flow q
- 31980/2 x 300 = -53.3 Ib./in. Adding this
=
constant shear flow to that of Fig. A21.53
gives the shear flow system of Fig. A21.54.
This torsional moment must be resisted by These results represent the effect on bay (1)
front and rear beams. Hence shear load on
each beam =
115240/30 3841" lb. =
80
As in the previous example problem in- -~-
~ ~
volving torsion, the beams in bay (2) will be 53.3 t f 120 501.3 ~ '232
assumed to bend without rotation of their ends, Fig. A21.54 t +
f -So-"""- t
or in other words the bending moment at mid-
point of bay is zero. The flange loads at
points a, b, c and d on bay (1) from the differ-
A21. 16 LOADS AND STRESSES ON RIBS AND FRAMES
of removing the bottom skin in bay (2). Adding 192 with sense as shown. Since this skin is
the shear flows of Fi~. A21.54 to those of Firs. missing we reverse this shear flow and find the
A21.46, we obtain the final shear flows in bay resisting shear flows on the other three sides
(1) as sho,vn in Fig. A21.55. of the bay cross-section. Fig. A21.58 shows
the section, with the 3 unknown shear flows
BOUNDARY LOADS ON RIB (A) Qab, qbc and Qad'
The bouncary loads on rib (A) will equal
f~l
the aif ference bet\veen th3 shear flows in bays
(1) and (2). Fi~. A21.56 shows a free body of
rib (A) with the shears flows obtained from qad=128 t~~t %c=288
Figs. A21.55 and A21.48.
- 192 - Fig. A21. 58
'::la'-'?>
80
Solution No.2
_
flows upon the shear flows as found for bays
(1) and (2) when bottom skin in bay (2) was Fig. A21. 60
not removed.
.--- --- 192 --- ........ ..- -
The first step is to find the shear flows
in all bays ass~~ing bottom skin in bay (2) is
not removed. The calculations would be exactly Fig. A21.60 shows the corrective shear
like those in solution (1) and the shear flow flows of Fig. A21.58 applied to bay (2). On the
in all bays would be those in Fig. A21.46. The bottom skin the corrective shear flow is shown
bottom skin in Fig. A21.46 has a shear flow of on the boundary of the cut-out. These shear
A21. 17
~
(check)
Double angles at 11. 5 , 4 - 11. 5 ' ]
3,3',9,9'. 1 I' Fig. A21.62 shows the frame with its
2 , bulb angle balanced load system. The internal stresses
stringers can now be found by the methods of Chapters
31~~:f::::::=ff:::::::::::::::::::-. AS to All.
2000 2000
5' t f
6' Y
11 11'
Fig. A21. 61
Iy = .15 (17.6"+16.2'"+13.5'"+13.5'"+10'"+
5")4 = 637 in.4
Fig. A21. 62
Due to symmetry of effective section and
external loading, the shear flow in the fuselage
skin on the z axis or between stringers 1 and 1 Example Problem 2. Unsymmetrical Vertical Loading
and 11 or 11 will be zero. Thus starting with
stringer (1) the shear flow in the skin resist- In certain conditions in flying and land-
ing the external loads of 4000 lb. can be ing unsymmetrical concentrated loads are applied
written around the circumference of the section. to 'the fuselage or hull structure. For example,
Fi s . A21.63 shows the same section and frame as
wa~ used in Problem 1. Due to an unsymmetrical
q = _VI z ~zA = _ 4000
637
~zA = _ 6.275 ~zA load on the horizontal tail, the reactions from
y the tail on the fuselage are as illustrated in
the figure. The total load in the z direction
= -6.275x .15x17.6 = -16.57 Ib./in. is still 4000 lb. but the loads are not sym-
= -16.57-6.275x .15x16.2 = -31.82
metrical about the z axis. For analysis pur-
poses, consider the loads as transferred to the
A21. 19
,-.--
shear load Vz =
4000 produces the same shear / h f
(3)
c
3"
flow pattern as Fig. A21.62. To balance the /
moment of -11500, a constant shear flow q J. / (1) e l- f.- 500
b
/' (2)
around the frame is necessary. / ;.'Ieb
(4)
- M-
q - 2A - 2
11500
x 11 x 18" =. 5 65 Ib /i
. n.
; /"
tg }d
I
600 500
(A = area of fuselage cross-section)
Fig. A21. 66
Adding this constant force system to that
of Fig. A21.62, gives the final boundary (2)
supporting forces on the frame as illustrated
in Fig. A21.65. The elastic stress analysis
of the frame can now proceed.
1500 2500
l .....~l- ~......,+
Fig. A21. 68
r 600
I 7 1000.. of<.
10
A21. 13 Problems.
Fig. A21.68 shows a 3 stringer single cell
wing beam. A rib is inserted to distribute the
(1) Fig. A21.66 shows a cantilever beam loaded concentrated loads as shown.
as shown. Find the shear flow in each of the
4 web panels. Draw axial load diagram for (1) Find shear flows in rib web panel (1)
each of the vertical web stiffeners and also (2) and (3).
the horizontal stiffener be. Plot axial load
diagram for beam flange members as obtained (2) Find rib flange loads at sections dc
from web shear flows. and abo
A21. 20 LOADS AND STRESSES ON RIBS AND FRAMES
(4) Fig. A21.69 (8) Same as problem (7) but with top skin re-
shows a 2 stringer, moved instead of lower skin.
2 cell wing beam.
A rib is inserted (9) Same as (5) but with read spar web removed
.03 to transfer 1000 lb. instead of bottom skin.
load to beam struc-
ture. (10) Same as problem (7) but with rear spar web
Fig. A21. 69 b removed instead of bottom skin.
r-- 10
"--1 Find shear flow
in rib web in each
cell adjacent to
(ll) In Fig.
A21.71 the external
line abo Also rib bulkhead loads Pl
flange loads ad- and P" equal 4000
jacent to points lb. each and P3
(a) and (b). equals zero. The
fuselage stringer
material consists
of four omega
~71:I
sections with an
area of .25 sq.
'-= .,. in. each. Deter-
J:l:;~ ~ I mine the skin re-
sisting forces on
~ :-/-------:
~ _....,0 I Rib B
the bulkhead in
balanc ing the
above loads. Fig. A21. 71
~ ~ § ,/ Neglect any effec-
J:l:;~ 4," ~ _ tive skin in this
1--I-~~'~P4
problem.
(13)
=
6000.
T-!
~
I r\
a c
c:rner Stringer Areas:-
(a) = 1 sq. in.
(14) In a water
landing condition
---,o'-r-__ p"
(b) =.8 sq. in.
(c) = • 5 sq. in. the hull frame ~15"-1
-l. ;b of Fig. A21. 72
b~
d (d) = 0.4 sq. in.
is SUbjected to a
toj~ i
normal bottom
pressure of 200
Fig. A21. 70
lb. per in.
The area of the ~I 40"
~I
bulb angle
Art. A22.2 - stresses around a panel cutout To enhance the usefulness of these problems,
Art. A22.3 - Shear lag problem all the structures chosen for analysis were
Art. A22.4 - cutout in a box beam taken from referenced NACA (National Advisory
Art. A22.5 - swept wing box beam Committee for Aeronautics) publications wherein
the reader may find detailed discussions of the
Aside from presenting one analytic treatment problems, other methods of analysis and data
of these problems, a discussion is given of the obtained from tests upon the specimens. Where
physical nature of each phenomenon. An under- available, these data have been used herein for
standing of the nature of the problem is of , comparison.
prime importance, Since no one analytic
technique can be all-powerful in the solution A22. 2 Stresses Around a Panel Cutout
of stress problems. The analyst must exercise
judgment and ingenuity in approaching each new "Cutouts in Wings and fuselages constitute
situation. one of the most troublesome problems confront-
ing the aircraft designer. Because the stress
In this chapter all analyses are made concentrations caused by cutouts are localized,
using the matrix formulation of the Method of a number of valuable partial solutions of the
Dummy Unit Loads (Chapters A7, AS), a famili- problem can be obtained by analyzing the be-
arity with which is assumed. havior, under load, of simple skin-stringer
panelS" (1)**
Such problems as those listed above are
too unWieldy to be studied here in great detail; ThUS, in the case of a wing beam With a
hence no attempt at exhaustive analyses has panel cutout of the upper surface (Fig. A22.1),
been made. To bring into relief the main it would be feasible to analyze the section
features of each problem, the structure selected immediately around the cutout as a flat sheet-
for analySis is one which is simple in con- stringer panel under the action of axial
struction and so loaded as to exhibit clearly stringer loads and edge shears (coming from the
the phenomenon under study. Many practical spar webs). The axial stringer forces could be
detailS, such as the effects of sheet wrinkling, computed with sufficient accuracy by the en-
rivet and fitting "give", stress concentrations, gineering theory of bending (E.T.B.) since
etc., have been side-stepped so as not to be- these are removed sufficiently far from the
cloud the objective. Further, the problems of cutout proper. The edge shear flows are readily
idealization of the original structure, into computed by those elementary considerations
which give the spar-web shear flows.
* One other important special problem - the so-called "bend-
ing stresses due to torsion" - is not treated here specifically.
As indicated in Chapter A8, the general box beam analysis
presented there encompasses this problem (Example Prob. ** Numbers in parentheses refer to the bibliography at the
15, p. p. A8. 24 through A8. 27). end of the chapter.
A22.1
A22.2 ANALYSIS OF SPECIAL WING PROBLEMS
Fig. A22.1
Fig. A22. 2 Idealization of the half panel by
use of substitute stringers
The sheet-stringer panel may, in general,
contain a large number of longitudinal ele- Fig. A22.3 gives the geometry of the ideal-
ments (stringers). The labor involved in ized panel.
treating this multi-element structure in detail A 1 = .703 in 2
is prohibitive, and thus an appropriate ideal- I-Tr~~-n-----,-----,
ization must be made. First, it is likely that I A I
F==H:=;:!£ ~~-~
the panel may be considered to be symmetric 2
about a longitudinal axis, so that only the A",= 1.045 in
J
half-panel need be handled. Second, the com- , ) AR = 0.25 in 2
plex, multi-stringer structure is replaced by I=::H=~-~---"~
one having but three stringers. As indicated I
I
I 1
t 1 = 0.0331 in
in Fig. A22.2, these stringers are: L_ _ ~ ~
I I " t 2 = 0.0331 in
,..b1 -b2 "; iL
#1, a substitute stringer having for its
area all the effective area of the fully con- ate
A1 A
1--0
A",
b 1 = 5.96 in
2
tinuous members to one side of the "combing 1 t1 ' t2 b2 = 7.56 in
stringer" (the stringer bordering the cutout)
and placed at the centroid of the area of Fig. A22.3 L = 15.0 in
material for which it substitutes. The stress
which this stringer develops is then the SOLUTION:
average stress for the material it replaces.
Fig. A22.4 is an exploded view of the half-
#2, the combing stringer, being simply the panel showing the placement and numbering of the
main continuous stringer bordering the cutout. internal generalized forces (Art. A7.9, Chapter
A7) and the external loading. Note that the
#3, another substitute stringer, this applied axial stresses were assumed to be con-
one replacing all of the effective material stant chordwise, giving stringer loads pro-
made discontinuous by the cutout. It is located portional to the stringer areas; their sum is
at the centroid of the material it replaces, P 2 , one of external loads.
and its stress is the average stress for this
same material. * The applied edge shear flows, coming from
the spar web, were assumed constant spanwise,
The sheet thicknesses used are the same as from a constant shear load. Other load
as those of the actual structure.** distributions may be handled by allowing these
applied shears to vary from panel to panel.
For very extreme load variations additional
* An alternate idealization, in which stringers #1 and #3 are transverse members could be inserted to create
located along the lines AB and CD, respectively (Fig. A22.2), more spanwise panels allowing a better fit to
was used in Reference (2) for a box beam loaded in torsion.
the spar shear variation. The applied shear
** When the longitudinal members themselves contribute to flows were considered as the other external
the shear stiffness of the cover (as is the case for "hat" section load and designated P 1 •
stringers riveted to the skin so as to form small closed
cross sections), an effective thickness must be used. This Panels on the centerline have zero shear
point is discussed in Reference (3). In this source, however, due to symmetry (Fig. A22.3).
the increase in shear stiffness is accounted for, not by
increasing skin thickness, but by decreasing the panel width -
an equivalent procedure.
A22.3
~~ 1 2 6 8
1:\ 1 2 3 4 5 6 7 8 9 10 11 12 13 14
-3.56
15 16 17 18 19
1 14.22 3.56
2 3.56 14.22 -3.56
3 47.17 11. 79 -11.79
4 11.79 47.17 -11. 79
5
6
7
8
9 7 024
10 7,024
11 7 024
12 8 909
13 8 909
14 -3. 56 7.11
15 -11. 79 23.58
16 4.785
17 -3.56 7.11
18 -11. 79 23.58
19 4.785
The "off diagonal" values have negative sign because the sense of those internal generalized forces having subscripts (14), (15),
(17) and (18) was taken opposite to that used in the derivation in Art. A7.10. A change in sense requires a change in sign in
off -diagonal coefficients only.
A22.4 ANALYSIS OF SPECIAL WING PROBLEMS
r=
~rs
] = [392.1
293.5
293.5l
392.l.J .5
~ 1 2
1 -10.93 -.65 ~ 5 6 7 8
~i~ FLEXIBLE
17 16.16 .359 8 14.61 .372
18 4.86 .108 = 9 - .26 .017
RIBS
19 23.98 .359 10 - .62 0
The above analytical results are compared 11 - 1.05 -.017
with NACA test data (1) in Fig. A22.5 for the 12 0 .036
loading P 2 = 1, P l. = O. Agreement is seen to
be good. 13 - 1.60 -.036
14 0 .359
EFFECT OF RIB FLEXIBILITY:
15 0 .108
To investigate the influence of rib 16 0 .533
flexibility, the problem was reworked assuming
aluminum rib caps, of constant area A = .25 in.~ 17 16.11 .359
as the transverse members bordering the cutout. 18 4.86 .108
The appropriate member flexibility coefficients
19 23.98 .359
A22.5
Comparison of this result with the pre- shear. The action may be comprehended readily
vious one for the rigid ribs reveals that the by visualizing an extreme case: a large degree
most important effect of rib flexibility was of "lag" would occur if the load transferring
to increase the concentration of stresses in skin were made of a highly flexible material
the combing stringer bordering the cutout. It such as a plastic sheet or even rubber. In such
should be noted, however, that for this sym- a case the inside stringers would be out of
metric panel, the use of a very flexible rib action almost entirely! With the inside stringer
as compared with a rigid rib led to stress in- stresses lagging, the outside stringers and spar
creases of the order of only 10% in the combing caps must carry an over-stress to maintain
stringer. Thus, the "rule of thumb" that trans- equlli bri um ("actual" in the figure).
verse flexibilities may be neglected in sum-
metric panels is re-affirmed. Fig. A22.7 shows the beam analyzed herein.
SOLUTION:
intermediate sheet, which transfers the loads To permit the handling of the calculation
in from the edges, is not perfectly rigid in in a limited space, it was elected to analyze
A22.6 ANALYSIS OF SPECIAL WING PROBLEMS
the structure for a single transverse (vertical) to zero, the stress distribution due to a one-
tip load symmetrically placed. In that case, half pound load at the tip ( a unit load
because of symmetry, it was necessary to treat divided equally between beam halves) was readily
only one-half of the structure. In addition, computed.
no shear flows could appear in the middle .07692
panels. Further, it is known that the influence
of rib flexibility on shear lag is slight for 0
symmetric systems, so that the ribs were con- -.9230
sidered rigid in their own planes; hence no
generalized forces were needed on the ribs to .9230
describe their strain energies. 0
Fig. A22.8 shows the placement and number- .07692
ing of the generalized forces on the half-beam. 0
-1.8461
1.8461
0
~imJ = .07692
0
-2.769
2.769
0
.07692
0
Fig. A22.8 Choice of generalized forces for shear -3.692
lag problem.
3.692
0
Member flexibility coefficients were
computed with the formulas of Chapter A7 and
arranged in a matrix. Next, the unit redundant stress distribu-
tion was computed. Fig. A22.9 illustrates a
The shear flows q", q7' q1." and q1.7 were typical calculation, showing the stresses in the
selected as redundants. Setting these equal tip bay for q2 = 1, q7 = q1.2 = q1.7 = O.
I~ 1 2 3 4 5 6 7 8 9 10 11 12 13 14 15 16 17 18 19 20
1 3271
2 16 640
3 10.74 2.684
4 20.05 4.724
5 17.99 4.202
6 3271
7 16 640
8 2.684 8.939 1. 785
9 4.7M 17.84 4.255
10 4.202 15.78 3.708
11 3271
12 16 640
13 1. 785 7.142 1. 785
14 4.255 16.084 3.840
15 3.708 13.89 3.324
16 3271
17 16 640
18 1. 785 3.571
19 3.840 7.518
20 3.324 6.439
A22.7
q = -12
9 '" q =+12
~
\
+12
(Constant on in
__-12
to Root
Ecrj
= f2175 }
-1848
-1260
~ 1\ \ \; '\ - 460.7
tv~ l.~.l.b \~
3,041 3,041 3,041 18,650
~ 2 7 12 17 - .9230
.5086
.4144
1 0 0 0 0
.07692
2 1 0 0 0 .03136
-1.846
3 0 0 0 0
1.055
4 -12 0 0 0 .7907
.07692
5 12 0 0 0
.02401
6 0 0 0 0 -2.7691
1.690
7 0 1 0 0
1.079
8 0 0 0 0 .07692
.01005
9 -12 -12 0 0
-3.692
= 10 12 12 0 0 2.493
1.199
11 0 0 0 0
12 0 0 1 0 Fig. A22.10 shows the above computed
stresses and those reported by the NACA as ob-
13 0 0 0 0
tained by test. Agreement is seen to be qUite
14 -12 -12 -12 0 good. 4. 589 ~ ('1. 885
15 12 12 12 0 1. 888
16 0 0 0 0 CALCULATED
17 0 0 0 1 3.416
18 0 0 0 0 2.364
1. 278
19 -12 -12 -12 -12
20 12 12 12 12 5. 1
A22. 4 Stress Analysis of a Box Beam With a Cutout. The calculation for step (2) above, will
now be carried out by considering a unit shear
In Article A22.2, one technique was em- flow, q:l.2 = 1, applied while all other loads
ployed for computing the stresses around a cut- are zero. Under this loading condition the
out. In that analysis the effect of the cutout relative displacements at redundant cuts 2, 7
was presumed to have been localized about the and 17 are equal to OC 2 :1.2' e-e 7 :1.2 and =:1.7 :1.2
, 1
cutout region; consequently, the problem was respectively, where these coefflcients have '
treated by isolating the affected panel. Quite been computed previously in Article A22.3.
often, when the cutout is placed well inboard
on the wing, its influence on the root stresses To restore continuity (reduce the relative
is appreciable. Therefore, it is desirable to displacements to zero) three redundant forces
be able to consider the overall problem of the are applied, one each, at the cuts 2, 7 and 17.
box beam with a cutout for such cases. The appropriate equations specifying continuity
are
"The most convenient and the most rapid
method of analyzing structures with cutouts is
the indirect, or inverse, method. The analysis
by the indirect method is made in two steps.
First, the structure is analyzed for the basic
condition that exists before the cutout is
made. The results of this basic analysis are Note that these equations say simply that
used to calculate the internal forces that
exist along the boundary of the proposed cutout. the deflections at the redundant cuts due to
the (unknown) redundant forces must be equal
External forces equal and opposite to these
internal forces are then introduced; these and opposite to the deflections due to q:l.2 = 1.
external forces reduce the stresses to zero All coefficients in the above equation
along the boundary of the proposed cutout, and
consequently the cutout can now be made without were computed in Art. A22.3. Specifically,
disturbing the stresses." (3)*
3'04~
~
0'210 16,810
9,536}
It is desired to modify the calculation of 16,810 32,160 3,041 {q2}
q7 = - 9,536
the previous article (A22.3, "Shear Lag { 3,041
AnalYSiS") to allow for the presence of two 3,041 3,041 18,650 q :J.7
cutouts symmetrically placed. Panel "q :1.2" was
removed while the single tip load remained. The above matriX is the [ocrsJ of Art.
Since the unit remained symmetric, the data A22.3, with the "q:l.2 row" and "q:l.2 column" re-
from the previous analysis, in which the trans- moved.
verse rib stiffnesses were taken to be
infinitely great, shOUld still yield satis- The equations were solved** to give the
factory results. values of the redundants for a unit applied
load, q:l.2 = 1, as
SOLUTION:
q2 - .1399
The calculation was accomplished in three
steps: q7 - .2134
* The procedure described here is quite generally useful for ** See Appendix for a method of "extracting" the inverse of
studies of the effect of removing one or more members; this matrix from that preViously found for the complete
such might be required for an analysis of the effects of
structural damage. ~rJ matrix.
A22.9
.07692
.03789
- .9230
.4683
CALCULATED
.4547
.07692
.03648
-1.8461
.9536
l = fa l
{ a imjcUToUTl -.02401{a
imjBASIC i, u
J= .8925
.07692
o
-2.7691
1.8763
.8927
MEASURED
.07692
.01258
-3.692 Fig. A22. 11 Comparison between calculated and measured
2.649 stresses (psi) in a box beam with cutouts.
1.037
Note that q~2 is now zero and the cutout A22.5 Analysis of a Swept Box Beam.
panel may be "lifted out".
"Experimental investigations of swept box
In the case of a structure under a variety beams have shown that the stresses and distor-
of external loadings (m = 1, 2, 3 ---), the tions in a swept wing can be appreciably differ-
more general equation, corresponding to the ent in character from those that would exist if
above, is the root were normal to the wing axis. The
principle effect of sweepback on the stresses
l
l imJBASIC -fal i,~~1 L
= la l
la a a ,mJ occurs under bending loads and consists in a
Li~CUTOUT concentration of bending stress and vertical
A22.10 ANALYSIS OF SPECIAL WING PROBLEMS
shear in the rear spar near the fuselage. With found in reference (8). One method of analyz-
regard to distortions, the effect of sweep is ing such wings is given in Chapter A23.
to produce some twist under loads that would
produce only bending of an unswept wing and THE STRUCTURE:
some bending under loads that would produce
only twist of an unswept wing." (6) The structure shown is Fig. A22.l2 is an
idealization of the NACA test beam of refer-
In the following example a swept box beam ences (6) and (9), in which a single substitute
is analyzed by the matrix methods of Chapter A8 stringer has been employed along the cover
and, in particular, by the specific techniques sheet to allow for the antitipated shear lag
of reference (7). The method accounts for the effect. The figure shows only one-half of the
interaction between the swept cover panels and complete unit, which was built symmetrically
the longitudinal members. It is this action about the axis corresponding to the longitudinal
that is responsible for the distinctive axis of the airplane.
structural characteristics of the swept box
beam. Only tip loads were to be applied (at
points A and B). The outer section of the beam
Again, we emphasize that the method used was assumed to carry stresses which could be
here is strictly applicable to thin-skinned calculated reasonably well by the engineering
wings of beam-like proportions only. Consider- theory of bending (E.T.B.). For this purpose
ing the wide variety of structural layouts it was judged satisfactory to consider the
which may be employed in swept wing configura- outer 66" of the beam as a single bay (A-B-D-C).
tions, a comprehensive treatment cannot be given If loads were to have been applied inboard of
here. An excellent review of methods better the tip, it would have been necessary to con-
adapted to thick-skinned construction and to sider additional bay divisions between A-B and
"plate-like" (very thin, wide) wings, may be C-D (that is, insert additional ribs at stations
1-
I
52"
.050 .050
I
I
I
C H D~I
!
i
1
yA=.539
I
I
Section C-C '?
.050 .050 (Steel) ~=. 125
I
66"
.078 t
0--""0
Ac..l. 373
Sectio~ A-A
I-'r
I...,A=l. 121
A A
t J A=l. 966
o-----....;""~o-----,>
A=l. 430
of load application). Rib C-D was located at affects total deflections only and can be
one of the actual rib locations in the NACA omitted in a stress analyses where deflections
test specimen and was assumed rigid in its own are not sought.
plane.
Since only symmetric loadings were con-
The choice of bay C-D-F-E as a single bay sidered in this analysis no shear was trans-
was somewhat arbitrary. For improved accuracy, mitted by the carry-through bay and hence no
additional ribs inboard of C-D could have been shear flows were shown in that portion.
used. Note that any ribs placed inboard of
point F will produce triangular skin panels in Sets of additional axial forces (q~6
the cover sheets. Examples of treatments for through q~e) were applied to the ends of the
such panels may be found in references (9), flanges and stringers adjacent to the obliquely
(10) and (ll). cut ends of the cover sheet panels in bay
C-D-F-E. These forces are necessary to account
Rib E-F was considered flexible in its own for the interaction between the swept covers
plane, it being known that the flexibility of a and the longitudinals. As shown in Fig. A22.14,
rib is important at a location where a structure the pure shear flow on the oblique edge is ob-
changes direction.* Note that this rib was tained by superposing onto the panel a zero-
made of steel in the test specimen. resultant system consisting of a uniform tensile
stress of intensity 2q plus a pair of concen-
Effective areas of longitudinals as shown trated balancing loads. The balancing loads
in Fig. A22.12 were computed by considering all must be contributed by the bordering longitudi-
of the skin to be effective. The spar cap nals and hence react on these as tensile loads
areas are equal to the sum of the areas of the (Fig. A22.14c). The balancing loads applied to
angle member at the cap location, plus one- the stringers are shown dashed since they are
half of the effective area of material between internal forces within the bay and are not to
the cap and the sUbstitute stringer (this area be entered into the equilibrium equations for
inclUdes several stringers as well as skin) the structure.
plus one-sixth of the attached spar web area:*
The substitute stringer area was collected in
like manner from the half-panels to either side.
~611 -
U
P p .. ~-==
LJ
~
P=qL
- 1
(a)
P_""'--
(b)
p~--!
...
(c)
"=
used on the specimen (.050") plus a weighted Fig. A22.14 Showing how the uniform shear stress on
increase to allow for the presence of splice an oblique panel end (b) is created by
plates along the plane of symmetry (see refer- superposition of a uniform tensile stress
ence 9). plus two balancing forces (a). The
balancing forces react as tensile loads on
the bordering longitudinals (c).
INTERNAL GENERALIZED FORCES:
Fig. A22.13 shows the choice and numbering From an energy viewpoint, these dashed
of the generalized forces. forces account for the additional strain energy
stored by the axial components of shear flows
The beam was rigidly supported at pOints in the non-rectangular panels. This energy is
E, F and at the two corresponding points on the stored in the cover panels themselves (and is
other beam half. These might correspond to the accounted for in this manner since the longi-
fuselage ring attach points in an airplane. tudinals contain the effective area from the
The vertical end caps on rib E-F were considered cover sheet***) and in the longitudinals which
rigid axially, so that no flexibility co- react against these components.
efficients were associated with the reactions
q,,~ and q" e' Flexibility of these members "Dashed loads" are applied to the longi-
tudinals adjacent to any obliquely cut panel
end. Similar dashed loads would be applied to
* The effect of neglecting this rib's flexibility is demon-
strated later in this example.
*** This much of the energy could be accounted for in another
** The factor of 1/6 is used so that the effective area con- fashion by modifying the member flexibility coefficient for
tribution of the web results in a structure haVing the same the sheet panel. See Reference 12, where this was done.
moment of inertia about a horizontal axis as the original. However, that referenc e incorrectly neglects the additional
Some of the problems of idealization are discussed in energy stored in the longitudinals, as was demonstrated in
reference (2), p. 16. Reference 13.
A22 12 ANALYSIS OF SPECIAL WING PROBLEMS
the outboard ends of the panels in bay C-D-F-E Stresses in inner bays:
if they too were cut obliquely. Such panel
configurations arise often in swept wing con- According to the discussion of Art. A8.12,
struction having ribs parallel to the air- Chapter A8, the determinate stress distribution
stream. Formulae for more general quadrilateral
panels are given in Reference 7. ~i~' may be any stress distribution in equil-
ibrium with the applied loads, and preferably
THE STRESS DISTRIBUTIONS one close to the final true stress distribution
The magnitude of the redundant forces is re-
For the symmetric loadings considered duced by use of a satisfactory estimate of the
here the structure was indeterminate only two true stresses.
times since the outer bay was assumed to be
determinate by the E.T.B. The stresses in the two inner bays were
determined for both gim and gir simUltaneously.
Stresses in outer bay by E.T.B.: Since this inner portion of the structure is
two times indeterminate we can estimate two
Flange stresses at rib C-D (for both loads. For this purpose the two flange loads
P~ = 1 and P" = 1) q ~" and q1.4 were written as
M = 66"# qu = q~" + q~"
I = 88.57 in. 4 ; C = 3.5" q1.4 = q~4 + q~4
fb =IMC = 2.608 psi where the (single) primed values are approxi-
mate values determined by the E.T.B. and the
Therefore, double primed values are the unknown corrections
(the redundants). Using My/I at stations 66"
and 118" from the tip gave*
q6 = q7 = 2.608 x 1.121 = 2.924 Ibs.
q6 = 2.608 x 1.373 = 3.581 Ibs. q~" = 5.228 P~ + 5.228 P"
For a unit transverse load at the shear q~4 = 3.899 P~ + 3.899 p..
center (midpoint, because of symmetry)
The equilibrium equations for the elements
of the structure were written next by summing
1#
Qa
=3.581 = 02713
2 x 66·
Ibs
in forces and moments.
q~t --->-.qa
qb~~~qb
Joint F
2.924
Qb =.02713 +~
~ ~
q"3 + q"6 =0
~ ~ = .07143 Ibs
in q"3 -+ q"6 = 5.513
1.414 qu
= P~ + 5.513 P" + 1.414 q~ 4
The unit load was shifted 15" to either side
by application of a torque, T = 15 in.lbs. Joint E
The uniform shear flow superposed was
q..4 + q"o =0
q = JA = 2 ~5 210 = .03571 ~b; q"4 - q"o = 1.414 q~"
= 7.392 P1. + 7.392 P" -+ 1.414 q~
"
Finally, superposition gave the stresses m about E-F
for the outer bay as
q"4 + q"6 + q"6 = 11.918 P1. -+ 8.888 P"
!~ 1 2
1 .1071 .03572
2 .06284 - .00858
[gimJ
= 3 .00858 - .06284
4 .03572 .1071
* This is a rather crude way to estimate these loads and is
5 2.924 2.924 used here only for simplicity. The analyst is generally better
advised to exercise a little more ingenuity in making these
6 3.581 3.581 estimates, even to the extent of being guided by other swept
wing solutions.
7 2.924 2.924
A22.13
E 1 0 0 0 -1 0 0 qs
~
dlJJ
G 1 -.7115 0 0 0 0 0 qe
'" G
c'
'" 0 0 21.2 0 21.2 0 1 qJ,o
~J
c'
t~... .
0)
uo
uo tco.
N
c:c
...
0 0 1.682 1 0 0 0 q1.1. =
0 0 0 1 1 0 0 q 1.e
C H H ~
"'m r
1 2 12" 14" q:L6 = "PD" = 2 x 52.3 x 15 x (-) qe
125.8
= -12.5 qe
i\
= -.989 P:L + .260 P 2 + .2191 q~2 + .1188 q~ .
1 •1071 .03572 0 0
2 .06284 - .00858 0 0 q:L e = tIp c " =2 x 73.5 x 15 x (-) q"
125.8 = -17.5 qe
3 .00858 - .06284 0 0
= -1.384 P:L + .360 P2 + .3068 q"1.2 + .1662 q'1.I ..
4 •03572 .1071 0 0
Similarly, for the panel HGFD;
5 2.924 2.924 0 0
6 3.581 3.581 0 0 q:L7 = -.0146 P:L + .460 P2 -.0449 q~2 - .2576 q~ ..
7 2.924 2.924 0 0 q:L6 =
-.0244 P:L + .773 P2 -.0754 q~2 - .4324 q~ .
The following matrix products were formed:
8 .1006 .0295 .00676 - .00676
(cf. eqs. 17, 18, 23 of Chapter A8)
9 .0791 - .0208 - .01753 - .00950
10 .0013 - .0411 .00401 .0230 [e><rJ =[ 17.70
-173.7
250.2J
- 50.43
11 .0422 .1135 - .00675 .00675
12 5.228 5.228 1.0 0 [coerJ = [114.5
42.96
42'~IJ
83.54
m:gi~ 13 7.729 3.444 -1.0 -1.0
[:<rs -~ =[: 0.01082 - .00556~
14 3.899 3.899 0 1.0 - .005563 0.01483
15 - .989 .260 .2191 .1188
Finally, the true stress distribution was found
16 -1.384 .360 .3068 .1662 as,
17 - .0146 .460 - .0449 - .2576
1
~ 2
'< yl~
d 17 - .6514 .0431
Pc=C+d qLx 2
18 -1.093 .073
/ c
PD=c + d qL x 2 19 .2150 .2060
PC7,/ T
d Iq L Balancing Load 20 -2.877 -1.584
Po.-.- / ---L (= Dashed Load)
21 3.249 3.022
V Formulae from Ref. 7
22 -1.145 - .015
I 2 3 4 5 6 7 8 9 10 11 12 13 14 15 16 17 18 19 20 21 22 23 24 25 26
15400
1 .~~L
102960
15400
70.170 15.46 15.46
50.01 8.984 8.984 8.984
52.33 6.542 6.542
12 130
69400
46000
5 130
15.46 30.92 30.92
8.984 17.97 17.97 17.97
6.542 13.08 13.08
15.46 30.92 30.92
8.984 17.97 17.97 17.97
8.984 17.97 17.97 17.97
6.542 13.08 13.08
2130
9.046 4.523
4.523 9.046
9.046 4.523
4.523 9.046
19.58
14.24
19.58
COMPARISON WITH TEST DATA: requirement. More details concerning the test-
ing techniques and method of data presentation
4.6 would probably resolve this conflict. Both test
and calculated values clearly exhibit the
characteristic build-up of stresses in the rear
spar of a swept wing.
Considering that rib EGF was relatively (7) Wehle, L. B., and Lansing, W., A Method for
rigid to begin with - being made of heavy Reducing the Analysis of Complex Redundant
gage steel - it may be seen that neglect of Structures to a Routine Procedure, Journal
the flexibility of a corresponding all-aluminum of Aero. Sci., 19, Oct. 1952
rib could lead to serious errors.
(8) Williams, M. L., A Review of Certain
REFERENCES Analysis Methods for Swept Wing Structures,
Journ. of Aero. Sci., 19, p. 615, 1952
(1) Kuhn, P., Duberg, J. E., and Diskin, J. H.,
Stresses Around Rectangular Cut-Outs in (9) Heldenfels, R., Zender, G., and Libove, C.,
Skin-Stringer Panels Under Axial Load - II, Stress and Distortion Analysis of a Swept
NACA WR L368 (ARR 3J02), Oct. 1943 Box Beam Having Bulkheads Perpendicular to
the Spars, NACA TN 2232.
(2) Rosecrans, R., A Method for Calculating
Stresses in Torsion-Box Covers with Cut- (10) Bisplinghoff, R., and Lang, A., An In-
outs, NACA TN 2290, Feb. 1951 vestigation of Deformations and Stresses
in Sweptback and Tapered Wings with Dis-
(3) Kuhn, P., and Chiarito, P. T., Shear Lag continuities, Mass. Inst. of Tech. Rept.,
in Box Beams: Methods of Analysis and JUly, 1949
Experimental Investigations, NACA TR 739, (See also, Journ. of Aero. Sci. 18, p. 705,
1942 1951)
(4) Hoff, N. J., and Libby, P. A., Recommenda- (11) Denke, P., The Matric Solution of Certain
tions for Numerical Solutions of Reinforced Non-Linear Problems in Structural Analysis,
Panel and Fuselage-Ring Problems, NACA TR Journ. of Aero. Sci., 23, 1956
934, 1949
(12) Levy, S., Computqtion of Influence Co-
(5) Kuhn, P., Approximate Stress Analysis of efficients for Aircraft Structures with
Multi-Stringer Beams with Shear Deformation Discontinuities and Sweepback, Journ. of
of the Flanges, NACA TR 636, 1938 Aero. Sci., 14, Oct. 1947
(5a) Kuhn, P., Deformation Analysis of Wing (13) Islinger, J. S., Stress Analysis and
Structures, NACA TN 1361, July 1947 Stress Measurements for a Swept Back Wing
Having Ribs Parallel to the Airstream,
(6) Zender, G., and Libove, C., Stress and McDonnell Aircraft Corp. Report 1127, April
Distortion Measurements in a 45° Swept Box 1949.
Beam Subjected to Bending and Torsion,
NACA TN 1525
CHAPTER A23
A23. 1
A23.2 ANALYSIS BY THE METHOD OF DISPLACEMENTS
coordinates, and are of two kinds, discrete and Here again, an exact representation cannot be
distributedo Both kinds involve a simplified given because, for practical reasons, the series
modeling of reality. used is of finite extent 0
Discrete coordinates are used if the When a collection of elements are joined
principal modes of deformation can be identified into a system, there are certain connectivity or
in terms of displacements of a finite number of constraint conditions and boundary conditions
points. which characterize the system, and which distin
gUish it from a mere pile of elements.
For example, with a beam element, we as-
sume that the deformed shape can be adequately To fix these concepts, we consider the ex-
described in terms of the motion of the ends of ample of a simple portal frame whose deformation
the element. For each end there are six com- is assumed to be restricted to the plane of the
ponents of motion, three for linear displace- figure (Fig. A23.3).
ment and three for rotation (Fig. A23.1). The
deformation of the element therefore is des-
cribed only to the degree possible by defining c\
B
t) 03 Oat )Oe
-----+ -----+---'--------' -----+
On t')o.:" ) 0~6
C-
the deflection and rotation of ends This 0
O" 07 15,.0 0H
description is obviously completely adequate if 3 elements
no external loads are applied along the element.
18 coordinates
If loads are applied along the element and
greater detail is desired, nodal points can be
defined at interior locations, effectively
breaking each member into two or more elements.
Fig. A23.3
r==-=-------------------=~<I> ~
u'
u~
--+
ti U'3
u~ t u's u'
'\
~ rB~--=-----C=- - 4
u'
--- "-- _ . / ------------ 'Y
th 3
desired, may be applied all at once in a subse- Various methods have been used; we will illus-
quent step. A further simplification is common trate the procedure by employing the theorem of
in many structural problems. The stresses and virtual work in the case of an elastic rod, and
strains due to axial loads within an element the differential equation of flexure in the
are often much smaller than those produced by case of a beam. The forces and displacements
bending loads. For such cases one may assume of elements will be denoted by P and 0, re-
that the frame members are inextensible. Thus, spectively, as distinguished from forces and
in Fig. A23.4, we may take u~ = u~ = 0, and u~ = displacements at nodal points of a structural
ui, and are left with three generalized coordi- system, denoted by F and u, respectively.
nates for this level of modeling abstraction.
Fig. A23.5 shows the structural deformation pat- undeformed
terns corresponding to these generalized co- rod
ordinates.
(
I
I
I
-I
I
I
(
/
,
\
I
L
\
I
/" '\
I
I
,
\
l?
O~
P...----..I
_x _ ______..,
L~
lJ
[-
Pa
deformed
rod
= U'l.= u~ I I U,a = U'3 U3 = u'e
I " til. I
Fig. A23.6
-,,>- 71~ -,;\-- 77- -,1,-- For a uniform elastic rod SUbject to axial
disolacements O~ and O2 at the ends, the dis-
Fig. .A23. 5 placement at any section x is given by
Three Generalized System
Coordinates for Portal Frame - - - - - (3)
A23.3 Theoretical Basis of the Method of Displacements The total axial strain is uniform, and given by
1
To perform a static analysis of a given ex = L (0 2 - o~) - (4)
structure, we must derive relationships between
external, applied forces JFI and the nodal If the rod is SUbjected to an increase in tem-
points' generalized deflections luI. In par- perature of T degrees as well as an axial stress
ticUlar, since the deflections are to be treated (load), the total strain ex is equal to the
as independent quantities, we seek to derive an elastic strain EX, plus the thermal strain~T.
equation The elastic strain is the quantity needed to
calculate the strain energy, that is,
lF~ = [KJ lu~ - - - - - - - - - - - - - (2)
- - - (5)
wher~ the matriX [KJ is the system stiffness
matrix. (Throughout this chapter, matrix no- Rewriting Eq. 2, page A7.1, in terms of strain,
tation has been employed. The symbols used are using the substitutions GxA = Sand ExE = Gx
defined in Appendix A, except where noted). for the elastic rOd, we find
periment in which one of the u's, say us, is Using the theorem of virtual work (Eq. 14, page
given a unit deflection. The elements of the A7.5)
sth column of [KJ are equal to the forces neces-
sary to give the structure this deflected con- Pr - au - - - - - - - - - - - - - - - (7)
- aOr
figuration, or equally, the elements of the sth
column are the forces developed at each of the so
nodal points when coordinate s is deflected a
p _ AE
aU = - L
unit amount with all other coordinates held ~ -a6~ [(0 2 - O~) - O<TLJ - - -(Ba)
fixed. Within this sth column, the element Krs
is the force developed at coordinate r in that and
deformed configuration. The system stiffness
matrix is compiled by adding together the indi- p
2
= a0
at! = AE
L
[(0 -0 )
2 ~
-o(TL] - - -(Bb)
vidual element stiffness matrices, as will be 2
demonstrated later. where the forces have the same positive direc-
tional sense as O~ and O2 , This information
Stiffness properties of structural elements can be eXhibited in matrix form as
are defined by applying the theory of elasticity
to the particular configuration of interest.
A23.4 ANALYSIS BY THE METHOD OF DISPLACEMENTS
{ P~}
3
P ,L
~
P
_2_ = EI
:= A + AEo<T { -6- - 2 (14a)
P2 -1 1 02 -1
(14b)
and we recognize the square matrix as [kJ, the _ 12EI
element stiffness matrix for axial deflection which have the solution P~ - - ~ and
of an elastic rod. The elements of [kJ may 6EI
P2 =- 1)"' The values of P 3 and P4 , found
also be calculated directly by rewriting the
Theorem of Virtual Work. Recalling the defi- from the equations of static equilibrium, are
nition of k rs ' it is the force Pr per unit de- 12EI 6EI
P 3 = 13 and P 4 = - L2' These results estab-
flection in coordinate os' Thus
lish the third column of the stiffness matrix.
a2 V - - - - - -(10) The remaining terms are found by a similar pro-
cedure, with the result
= P 4 L/EI
2
= P 4 L /2EI
Fig. A23.7
P3L 2 + P4L _ 0
at x 0 - - - - - - - - (12a) °4 = °43 + °44 = 2EI EI - - (16b)
where
- L"
P" -12 -6L 12 -6L 0"
P4 6L 2L2 -6L 4L2 °4
t 1
other transverse direction.
04
D. Tapered Beam 1
t0 1
fO" ------r
02e Ij
L ] Ii ) 04 l :5~
a
J
I·
L
~
A23.6 ANALYSIS BY THE METHOD OF DISPLACEMENTS
P1. 1 1 -1 -1 01. o o 0-
1
1 o o
P2 Gt
1 -1 -1 °2
- ab o 1 o
P" -1 -1 1 1 0" o o o
P4 -1 -1 1 1 °4 o o o
o o o
In this case, also, the units are pounds psr
inch and square inches for the "forces" and 1 o o
"displacements," respectively. Note that there
o o o
FJ --
is only one independent coordinate here, which
~
is represented physically by the change in angle o 1
at one of the corners due to the shearing action
of the applied forces. 1 o (18)
o o
G. Other Elements
o o 1
Only the simplest elements have been shown
in this section. Fortunately, many practical o o o
problems can be worked using this information, 1 o o
since a comprehensive catalog of elements could
easily fill a separate chapter. Other elements o o 1
having widespread application are two-dimen- o o o
sional elements with membrane forces (including
triangular, square, and trapezoidal elements), o o o
two-dimensional plate bending elements (includ- o o o
ing triangular, square and trapezoidal elements),
thin shell elements (including conical frusta
and doubly curved elements), axisymmetric thick For convenience, the system [e] matrix may be
shell elements (including ring elements having partitioned into element [?~ matrices, as
various cross-section configurations), and shown. These smaller matrices may be used
general three-dimensional elements (including one at a time to transform element properties,
as will be shown subsequently.
tetrahedron and hexahedron elements). These
and other elements are being developed and per-
fected by structural analysts. It is worth Similarly, we will want to transform
noting that lack of interelement compatibility forces from system to system. Tne method for
is an imDortant s!1ortcoming with the more com- doing this is derived from the expression for
Virtual work.
plex finite elements, and provides the moti-
vation for much continuing work. For details,
the reader is referred to the abundant liter- Consider forces F, associated with the u
ature in this field. coordinate system. For a set of virtual dis-
placements (6 U} , the virtual work is*
A23.5 Transformation of Coordinates
6\-1 = (F} T 16U} - - - - - - - - - - - (19)
We will often need to transform displace-
ments between original coordinates (or coordi- For the ° system,
nates in which it was convenient to define
element properties) and generalized or inde-
pendent coordinates. For this purpose, we use
We also have the compatibility relationship
geometry to define a transformation matrix G?J
given by
- - (17)
derived from the coordinate transformation.
For the portal frame of Figs. A23.3 and A23.5, Now, since virtual work is a scalar, and since
by inspection, we have the pIS and F's are in equilibriwn, and the
o's and u's are compatible, the Virtual work is
the same in either system.
Thus
and since the ~u' 3 are independent so tF*l = [K u ] {U*} + [Ka ] tuo 1 - -(32a)
with to} =
[fJ lUI' so the first expression may
be rewritten and, once the lU*} are known in any given prob-
lem, the {u o} may be found immediately from
Eq. 33.
we can then substitute for }p} using Eq. 27 and Fig. A23.9
derive the final expression
Thus we have
[a] = [bJT [a] [b] - - - - - - - - - - (29)
where we have used the fact that the ~Fl forces
are independent.
and by analogy with the ca8e of zero forces
Often there will be displacements in ~u1
which are associated with zero applied forces - - - (37)
in those· coordinates. Let
where
- - - - - (30)
- (38)
where the ru*} are displacements on coordinates
with applied forces tF*], and the {u o} are dis- The flexibility method is useful also in
placements on coordinates with zero valued deriving a relationship between system forces
a;:Jpl1ed forces. and element forces for a statically indetermi-
nate structure.
Then U:: r!-.L~~ ! K
r } = LKe
0 I
~h!l.J
ee u
f.-u-~-1- - - - - - - - (31)
A23.8 ANALYSIS BY THE METHOD OF DISPLACEMENTS
so
(40)
1
Fig. A23.1O
which is the desired transformation. We have The coordinates chosen are shown in Fig. A23.11.
seen how to transform information about forces,
displacements, stiffnesses and/or flexibilities
from one coordinate system to another. Our next
task is to apply these methods to transform from
element properties to system properties. Rather
than repeat the several derivations, the results
are tabulated below. U2
Table A23.1 u~
System
Properties
6"'~ t6,
Transformation
03
[K]~["r[k][~] [a]~[br[Q]H
Transformation
of
Structural
Fig. A23. lIb
Properties or or Element Coordinates
111 T In
[K]~L [~e] [ke][~e] [a]~ L [be][Qe][be] Note that system coordinates at the boundary
e~l e~l points A, Band C could have been included, but
where m = number of elements
would have dropped out when the boundary con-
in structural system ditions were applied. For simplicity, these
coordinates have been deleted at the beginning
In the case of zero force coordinates of the analysis. To compile the system stiff-
ness matrix, the contribution from each element
~F*3 = ([KuJ - [KaJ [K 22] -~ [K2~]) (u*\ must be found. For example, for element OC,
the element stiffness is given by
= [K*J (u*}
In the case of zero displacement (redundant) [kocJ = AocE [ 1
L oc -1
-1] = 1~3[2
1 -2
- - (42)
coordinates
~u*} :: ([auJ - [aaJ [a22]-~[a2~]) IF*1 and the transformation from element to system
- - - - - - - - - - - - - - - - - (44)
to the system stiffness matrix. Similarly, the Fig. A23. 12b
following information may be compiled for ele- Element Coordinates
ments OA and OB.
Table A23. 2
[K] = [~]T [kJ [~J ' where
r---------,------------------
Element In general,
Quantity
OA OB
'"1 0 0
~[-: -: ]
Element Stiffness [ 1. 414 -1. 414]
E 0 1 0
in Local Coordinates
W -1.414 1. 414
[ keJ 0 0 0
[: ;]
0 0 0
Coordinate
Transformation [-:.707 :.707] 1--- --- ---
0 0 0
[ @e]
0 1 0
Element Stiffness
in System Coordinates
[K ]
e
E
W
[0.707 -0.707]
-0.707 0.707
[ 0
~ 0
[B] =
[::] =
0
r--
0 0
0
---
0
1
---
- - - - (47)
Exa~ple Problem 2
Assemble the system stiffness matrix for with each eke] equal to the result given in
the portal frame of Fig. A2:3.12a, aSS11,'1l11g uni- case C, section A23.4.
form element properties and neglecting axial
deformation. The element coordinates are shown The final result, after carrying out the
in Fig. A23.12b, and the element stiffness indicated operations (using either the complete
matrix is given by case C of section A23.4. matrices of Eqs. 47 and 48, or the partitioned
U3'---"_ _~ _
form of Eq. 45) is
u~
24
[KJ = EI
L3
6L - - - - - - (49)
L [ -6L
7~
A23.6 Calculation of Displacements and Stresses
I
There are several methods for solving for
Fig. A23.12a the deflections resulting from the application
System Coordinates of given forces. The most straightforward is
A23.10 ANALYSIS BY THE METHOD OF DISPLACEMENTS
to invert the stiffness matrix (for details, We wish to systematize the solution by first
see Appendix A) triangularizing the left-hand side of the
80 that equations, resulting in fewer unknowns in each
successive equation, and culminating in a final
equation in the single unknown x n . The solu-
{u I = [KJ -'- [FJ - - - - - - - (50)
tion is completed by solving first for the last
or for the three member frame of example prob- unknown, xn, then using this information to
lem 1, section A23.5, solve for x n _ 1, and so on. In the triangu-
larized form, each unknown, x r say, depends
only on the xm' With n ~ m> r.
UJl.} = l~a [ 4.547 -0.225]{F'-J - - - (51)
{ Ua -0.225 3.129 Fa The first step in the triangularization
consists of calculating new coefficients for
Given that F,- = 0 and Fa =-lOOOlb., we find the second equation and all subsequent equa-
that the horizontal deflection U1 22500/E = tions using
inches and the vertical deflection Ua == -312900/E
inches. The final step is to compute element i=2, , n
forces. To do this, the element deflections j =2, , n+l - - (55)
must be found using the transformation matrix
[B] or [ee]. For member DC, using Eq. 43, which brings the equations to the form
1 •.5303
Note that G is the angle between the global -0.7071 1. 7071 0 o 0.7071
(system) x axis and the st"uctural axis of a (64)
member. [K"J :: 10 8 -1 o 1.5303 0.5303 0
o o 0.b303 1.5303 0
tyz U4 -0.7071 0.7071 0 o 1.7071
U
~",-u-=-:L t
--,;_ U"
~5
75
0.4580 10000 -75 U2 ={3J
2500 ] {u:l.} 0 - - - - (67)
Each row of the matrix equation represents [ 2500
-75 10000 u" 0
an algebraic equation in the unknown u values.
These are solved in inverse order, with the For the iteration, a further reduction is
following results worthwhile, wherein only one unknown appears
on the left-hand side of each scalar equation.
ue = 0.3952 X 10- 6 F" FollOWing this manipulation, the equations to
be solved are
u4 = -0.604'7 X 10- 6 F"
U" = 1.7450 X 10- 6 F" -(68a)
u2 =
0.3953 X 10'-6 F"
u 2 = -0.0075u:l. - 0.25u" -(68b)
u:l. 1.349'7 X 10- 6 F"
=
u" = 0.0075u:l. - 0.25u 2 -(68c)
The static deflections are found by SUbstituting
the given value of F" = 1000. As a trial solution, we take U:l. = 1, u 2 = U" =
0, inasmuch as we expect sidesway to be the
Member forces are determined in the usual way, dominant deflection. The results at selected
by means' of the appropriate U3eJ and eke] steps of the procedure are given in Table
matrices as before. The results are shown in A23.4. The convergence does not appear to be
Fig. A23.15. rapid, but methods exist for accelerating the
O.395x 10-"" solution. For further details, the reader is
~",-,=-1.:..::3--=-5-=-0.:.:x--=-1-=-O~-"_'_'-"r- 1. 745 x 10 -"" referred to a text on numerical analysis.
395.3#
'11* 0.605x 10-""
fCO'r.>'r.>· ;::
'S:Q"", ""
g
'~.jf"'
Fig. A23.15
A23.13
Table A23. 4
Ibl~
0 0
~~
(a) structure to a properly formulated mathematical
L!2 problem. The nwnbers produced by the solution
of the mathematical problem are then used to
verify the design concept and to provide a
rational basis for redesign, where necessary.
I Finite element modeling is one method for pro-
(b)
~.----'-----~J
L!2 ducing a mathematical description of the struc-
ture. The next two sections comprise several
representative exmnple problems, intended to
(c)
provide the reader with an appreciation for the
nature of typical finite element models and the
kinds of results which can be obtained with them.
2PX
I~
Example Problem 4
(d)
Reconsider the truss of Fig. A7.16, repro-
I. L ~ duced in Fig. A23.18. A truss is defined as a
pinned structure wherein moments are not trans-
Fig. A23.17
~~----jJ
(b) Concentrated 3M M 3M M
Moment at -2L -4 2L -"4
Midspan
(c) Uniform
Load
P!2 PL!12 P!2 -PL!12
~30"+30"~30" H
Note that these nodal forces are related to the mHted between members at the joints. This
fixed end moments used in the moment distri- idealizati on is seldom met, and t he truss is,
bution method of Chapter All. The moments P2 in reality, a frame composed of relatively
and P4 are fixed end moments, while the re- slender members. The structure shown will be
actions P", and P" may be obtained by statics. reanalyzed as a frame, and the results com-
For other cases of interest, the reader is re- pared with the truss ~nalysis presented in
ferred to the information displayed in Chapter section A7.7. The previous problem statement
All, Table All-l, herein, from which tile ap- does not include data on section properties
propriate nodal forceR can be derived. Loadings for the members of the structure. Calculated
not appearing in the table can often be obtained values have been supplied, based on a plaus-
by superimposing the given cases. ible design philosophy. Assuming that the
material is alliminum, having a modulus of
A23.1O Structural Modeling Using Finite Elements elasticity E = 107 psi, member areas can be
found from Table A7.3. A typical aircraft I
The process of structural modeling occurs section is shown in Fig. A23.19, along with
at several levels of abstraction during the dimensions given in terms of the material
course of any design. Typically, one first has thickness. For this section, we have
a conceptual model, selecting the type of struc-
-.r
ture necessary to transmit the applied loads
from among many possibilities, including trusses,
ca~les, frames, arches, and shells. A more de-
tailed model is necessary when making the design
'------'1 20t
drawings. One must detail connections and
brackets, size members based on available sheet
metal gages, and rolled or extruded sections,
actu.al material properties, etc. An analytical Fig. A23. 19
'--1
4
--2-0t-~(Tt
J
A23.15
L A I 1.50
Struss Sframe Mframe
Member . 2 4
in. m. in. lb. lb. in. -lb.
AB
BC
CD
30
30
30
0.4785
0.3074
0.3074
0.323
O. 130
O. 130
10500
2250
2250
10500
2262
2244
208
692
95
-
1000# B
2.25 2.25
I
C
50"
-4801 In.-lb.
Fig. A23.23
12 @ 2" ~I
-27627 in. lb. ///// 10881 in. -lb. kk/
T
3 @ 2"
Case 1
36 elements
52 nodal circles
Fig. A23.21
~
Example Problem 6
f-- - - - - - - - - - - - - -
The thick walled pipe of Fig. A23.22 is to
be modeled using finite elements. The dimen-
sions of the pipe are) = 24 in., ri = 4 in., ~ Case 2
=
and r o 10 in. 6 @ 1" 144 elements
175 nodal circles
~
x
x
24 @ 1" .1
Fig. A23.24
150,-----,----,------,--------,-----,------,
system is sufficiently complex to preclude an
Solution Symbol Distance from Load analytical solution in closed form. Note that
Elasticity the coarser nodal grid gives results comparable
Theory in accuracy to those achieved with the refined
DiSPlacementE 6 Inches division using 144 elements. The decision re-
Method ~
36 Elements
1--- - -1- - -
1:'------)1,
-
12 Inches
18 Inches
garding how many elements to use must be based
on experience with comparative studies of this
sort.
100
A23. 11 A Delta Wing Example Problem*
10
Radial Distance - Inches
Fig. A23.25
Fig. A23.27
150 ,---~---~---~---~----r----...,
Idealized Delta Wing Structure
Solution Symbol Distance from Load
Elasticity
Theory IDEALIZATION
Displacement { 6 Inches
Method 1---- - -1-- - - , 12 Inches
The delta wing has been idealized into a
", , 144 Elements "t--~- ~ 18 Inches grid-like structure having grid points numbered
as in Fig. A23.28. Grid numbers increase to
the rear and outward.
100
4 7 10
~ -~
~
~,~--~~-
'"'" ',------
Q)
H
1i'i
2 r-------;Y
50
r-48"-----r- 48"-::
I ! f= . I i
20.79 3.59 47.23 40.82 12.3~
(b)
Rib 8-9 Spar 4-7-9-10
Fig. A23.29
Effective Widths of Cover Sheet for Rib and I-- 48"- "'---3@48"
Oblique Spar Caps (After Levy, Ref. 8) I I
£ 1 L is },
I = 6.23 2.87 5.09 4.78 3.82 0.26
I
[email protected]'1--'l-----1
* A rational, systematic means of treating cover sheet 1 I £ !
panels is given in Reference 10).
I = 16.02 12.29 8.30 0.26
A23.19
An (assumed) average rat;'J of tWist, fl, is com- 17 of this chapter. Then using equation 26 we
puted approximately by using the GJ at a repre- obtain the square stiffness matrix
sentative section half way along the box.*
mnrs
_ T _ Pmbrn - Pnbn
fl- GJ - GJ
Then the deflection on and Om are given by
= Pmbm GJ- Pnbn Lnbn
H+ GJ Ln/Lm 1
(Lnbm+Lmbn)(bn+bm) -Lm/Ln
A = 7. 26 x 48 = 348 in 2
Note:
f ds 48
t = 2 x . 051 = 1. 88 x 10
" G=~
2.6
_ 4(348)2. 1 _ . 2
where GJE=l- 1.88 x 10"x 2.6 - 99.3lb.lil.
_ Lnbm + Lmb n + bs (Lm - Ln ) (ds/t contribution of vertical webs neglected)
A
Ln(b r + bsl
Fig. A23.31
A
-(Lnbm + Lmb n ) + br (Lm - Ln ) Calculation of GJ for the Representative
- Ln (b r + bs)
Section of Box 2-3-5-6
For box 6-7-8-9 One now writes the relations which trans-
form the displacements. As an example, one
GJE=l =34.3 writes
Finally, the stiffness matrices for the u
1 2 3 4 5 6 ----- 11 12 13 14 15 16 ----- --29 30
three boxes of the example become: 6
0 1 0 0 0 o ----- 0 0 0 0 0 0
t3 1f- " ;;;:
l
-~ -~]
4 0 0 0 0 0 0 0 0 0 1 0 0 0
-1 -1
1 1
0.000898 Then
-1 1 1 -1
1 -1 -1 1
[ -~ -~~
-1 -1
2 .003850 -.003850 .n08 .07404
1 1
.5 -.003850 .003850 -.n08 -.07404
[KJ = 0.000482 -1 1 1 -1
12 .n08 -.n08 3.929 1.387
1 -1 -1 1
1.5 .07404 -.07404 1.387 2.167
For box 6-7-8-9 Here for economy we have written only the non-
zero terms of the transformed element stiffness
[ -~ -~]
-1 -1
matrix. These simple transformations, involving
1 1 no coordinate rotations may be written by in-
0.000310 spection since obviously, for element 2-5
-1 1 1 -1
1 -1 -1 1 6~ ----.. U 2
62 ----.. u~ 2
SYSTEM DISPLACEMENTS/TRANSFORMATION OF 63~U6
COORDINATES
64 ----.. u~ 6
1 1 0 0 0 0 0
2 0 0 .707 0 .707 0
3 0 1 0 0 0 0
4 0 0 0 .707 0 .707
Then
Fig. A23.32 i~ 1 5 11 15 21 25
Delta Wing System Displacements. 1 .0005295 -.0005295 .01326 .01214 .01326 .01214
All rotations positive by right-hand rule. = 5 -.0005295 .0005295 -.01326 -.01214 -.01326 -.01214
11 .01326 -.01326 .4361 .2004 .4361 .2004
15 .01214 -.01214 .2004 .3821 .2004 .3821
21 .01326 -.01326 .4361 .2004 .4361 .2004
25 .01214 -.01214 .2004 .3821 .2004 .3821
A23.22 ANALYSIS BY THE METHOD OF DISPLACEMENTS
Proceeding in the above manner, stiffness matri- found on span 9-10. This situation suggests
ces for all 15 bea~ elements are transformed. that a single beam, continuous across grid
The torque boxes are already in "system" co- point 9, be used instead of the separate ele-
ordinates since each was designated by the grid ments.
numbers of its four corners.
The sUbstructures selected for this prob-
ANALYSIS BY SUBSTRUCTURES lem, along With their element deflections, are
shown in Fig. A23.33. They are the rather
From this point on the analysis proceeds in natural choices of the continuous wing beams
a very direct manner as previously illustrated. (spars and ribs) with only the minimum neces-
However, as is also apparent, any large struc- sary deflections considered. While the fore-
ture having many nodal points is going to yield going reasoning had dealt largely With forces,
a very high order matric equation when analyzed it is the corresponding element displacements
in such a completely elemental fashion. Which are used as the variables and are so-
shown in the figure.
It is possible, however, to reduce the
order of the problem by some preliminary re-
combining of the elements into sUbstructures in t 0" tO 2 t O~ to.
the course of which one can Beam 1-2-3-4 I I I
(a) use certain a priori information to t0 6 t0 7
Rib 5-6-7 I
eliminate some loads (and corresponding dis- I
placements) t Os
Rib 8-9 I
(b) extract some portions of the problem
out of the main calculation, putting them into
sUbproblems solved on the substructures.
These devices will be employed in the following
section. Spar 2-5 (e-I_--II) 0".
0 ••
Certain of the external loads associated f O~ to 6 t0 6
hence three of the reactions expressed by the ficients are those for only the half-wing
structural equations in the matrix may be found acting alone and supported by constraints as
from the others by the equations of statics. asswned above. To account for the presence of
To remove the "singularity" from the stiffness the other half of the wing, 'it is necessary to
matrix it is only necessary to drop out three specify additional geometric conditions along
equations - achieved by removing three rows and the airplane centerline. This step is accom-
corresponding columns (so as to retain a sym- plished by asswning the following deflections
metric stiffness matrix). zero (eliminating their corresponding rows and
columns from the matrix):
The act of removing the three equations
selected is also equivalent to assuming the u~~ for symmetric loadings (zero
corresponding deflections to be zero. In this lateral slope or rolling
way a reference base for the deflections is also u~"
established. The choice of reference base is
somewhat arbitrary, but, following a suggestion
of Williams (13), a triangular base will be
U~4
U"
J rotation along the airplane
centerline)
U2 Uo U. U. U, U. U. U. O U.. U.. U •• U •• u ••
accuracy is assured only by completing an in- production runs can be performed. Use of on-
finite number of steps. By terminating the line terminals operating on a time-sharing
calculations after a finite number of iterations, system 1s an efficient way to debug, as it
we produce a truncation error. Further error eliminates most of the waiting time between
may be produced if the matrix coefficients are runs. This is especially useful during the
obtained empirically. Placing bounds on the initial runs to discover spelling and syntax
errors is a difficult but important part of the errors. Often a machine failure will terminate
calculation procedure. For details, the reader a run before completion. It is desirable to
is referred to texts on the SUbject of numeri- output intermediate results so that something
cal analysis (5,6). is learned from every run. The amount of
running time for each portion of the program
A23. 13 Management of Problem for the Computer can be checked easily, and can pinpoint those
parts of the calCUlations where reprogramming
A discussion of the management of a would be most productive.
structural problem for solution on an electronic
digital computer could easily occupy a separate Once a program is ready for production,
book. We will be content here to give the it should be compiled and stored on tape or
reader a checklist of topics Which can be con- disk in machine language rather than FORTRAN
sidered as a guide to the questions he should or PL/I. This will save considerable time per
consider and discuss with a computer consultant. run. Where possible, established, widely used
It is assumed that the desirability and feasi- programs should be used; with a new program
bility of performing a computer analy8is have debugging represents a significant proportion
already been determined, and the analytical of the work required to produce results. How-
approach has been selected. ever, one must beware of poorly documented
programs; the cost of the learning cycle could
At this point the key factor is the cost of easily exceed the cost of writing and debugging
the calculation for a given degree of accuracy. a new program.
The method employed, and thus the cost will be
intimately related to the rate structure at the REFERENCES
local computer facility. Often costs not cen-
tral to the computation are the controlling (1) Rubinstein, M. F., structural Systems -
ones. A typical charge rate includes central Statics, Dynamics and stability, Prentice-
processor unit time, number of input-output Hall, Englewood Cliffs, N. J., 1970.
requests, and required storage region size.
Other factors, such as cost of cards and printed (2) Rubinstein, M. F., MatriX Computer Analysis
output, tape and disk storage charges, graphic of Structures, Prentice-Hall, Englewood
plotter usage, etc., may be prorated or charged Cliffs, N. J., 1966.
directly. For a given computer, one of the
elements of the rate structure generally domi- (3) Przemieniecki, J. S., Theory of MatriX
nates the programming strategy. Earlier in the structural Analysis, McGraw-Hill, New York,
evolution of computers, processing time and 1968.
storage were the limiting factors. With im-
provements in these areas, current computer (4) Zienkiewicz, The Finite Element Method in
technology is often bound by input-output limi- Structural and Continuum Mechanics,
tations. In addition to cost considerations, McGraw-Hill, Ltd., London, 1967.
the possibility of committing a blunder in data
inputting has led to use of special data gen- (5) Crandall, S. H., Engineering Analysis,
eration routines and checking programs. MCGraw-Hill, New York, 1956.
Graphical output is especially useful in de-
tecting errors in nodal point location and in (6) Ralston, A., A First Course in Numerical
checking element definitions. Because o~ in- Analysis, MCGraw-Hill, New York, 1965.
put-output costs, it is desirable to keep as
much of the program in core storage as possible. (7) Sokolnikoff, I. S., Mathematical Theory
The cost of storing ann fetching information of Elasticity, McGraw-Hill, New York,
may exceed the cost of regenerating data by a 1956.
factor of ten, twenty, or more. When moving
data in and out of the central processing unit, (8) Levy, S., Structural Analysis and Influ-
the format used can have a critical influence ence Coefficients for Delta Wings, Journal
on cost. It is often advantageous to transfer of the Aeronautical Sciences, 20. 1953.
many record lines at a time using the simplest
format definition available. (9) Schuerch, H., Delta Wing Design Analysis,
Society of Automotive Engineers National
Every program contains errors or bugs which Aeronautic Meeting Preprint No. 141,
must be discovered and corrected before reliable September, 1953.
A23.27
(10) Turner, M. J., Clough, R. W., Martin, (12) Wooley, Ruth, Check of Method for Computing
H. C., and Topp, L. J., Stiffness and De- Influence Coefficients of Delta and Other
flection Analysis of Complex Structures, Wings, National Bureau of Standards Report
Journal of the Aeronautical Sciences, 3655, 1954. (Available as ASTIA No.
23, September, 1956. AD46866).
(11) Kroll, W., Effect of Rib Flexibility on (13) Williams, D., Recent Developments in the
the Vibration Modes of a Delta Wing Air- Structural Approach to Aeroelastic Prob-
craft, Institute of Aeronautical Sciences, lems, Journal of the Royal Aeronautical
Preprint No. 585, 1956. Society, 58, 1954 (see also, Aircraft
Engineering, 25, 1954).
INTRODUCTION TO
THE THEORY OF ELASTICITY AND THERMOELASTICITY
BY
R. ]. BOLLARD, Ph.D.
CHAPTER A24
THE 3-DIMENSIONAL EQUATIONS OF THERMOELASTICITY
The second requires that the elastic The violation of these last two assumptions,
properties are the smne in all directions at small deformations and linear stress-strain re-
any point in the elastic body. If a material lations, leading to the non-linear problem
is non-isotropic, such as in extruded graphite Where, of course, the rules of superposition no
or laminated plastics, the relationships be- longer apply, is mentioned not to cause con-
tween stress and strain are no longer the fusion but to serve as a warning of the limita-
simple equations containing two independent tions of the theory presented herein.
elastic constants. In the most general non-
A24.2 Formulation of the 3-Dimensional Equations of
isotropic material it requires 21 elastic Thermoelasticity.
constants to define the stress-strain re-
lationship. The existence of ~lanes of elastic The problem of thermoelasticity is to
symmetry reduces this number.(4j determine at any point within and on the bound-
ary of a heated elastic body subject to external
A third important assumption is that the loads and constraints the stresses, strains and
displacements are so small that the position displacements. The stress condition at any
of a point in the body does not require re- chosen point within or on the boundary of the
definition after deformation and that the body must be such that equilibrium of forces
higher order terms of displacement derivatives and moments exists at that point. Consequently,
are negligible. If these assumptions are not the expression of the equilibrium state of the
made the relationship between strains and dis- stress components will be independent of the
placements become non-linear and the much more material properties, the deformations, the
difficult theory of large deformations must be temperature and temperature distribution, the
employed. stress-strain law, etc. The relationship
between stress components must be only such as
The final assumption is that the material to satisfy the equations of equilibrium. It is
behaves elastically. 'I'his assumption requires well known that the stress at a pOint, being the
force intensity in a chosen direction, can be
(4) Numbers in parentheses refer to the bibliography at the represented by a single vector, haVing magnitude
end of the chapter. and direction, which is related to a chosen cut
A24.1
A24.2 AN INTRODUCTION TO
plane surface of infinitesimal area at that solution when all 18 equations are simUltan-
point. Thus the definitions of the stress at eously satisfied. The development of the
the point requires more than the statement of solution to a specific problem reqUires a set
the stress vector. It requires in addition of boundary condition equations for evaluation
the definition of the cutting plane through of the unknown coefficients existing in the
the point of interest to which the stress general solution.
vector is referred. The stress at a point can
be fully defined by choosing a Sllfficient nllill- As in the theory of elasticity, the solu-
ber of cuttl~g planes and determining the tion to a chosen physical problem in thermo-
stress vectors, usually defined by its compon- elasticity must satisfy the defined 18 equation
ents, associated with eaGh. In 3-diJlensional set and, as well, the boundary conditions.
bodies a minImum of 3 cutting planes which are The equations of thermoelasticity are derived
mutually at right angles to eaGh other (orthog- in the following sections in the following
onal) is reqUired. Thus, vlhen eacIl stress sequence.
vector associated with one of these cutting
pla~es is defined by its 3 comnonents, there (A) Equilibrium Equations
will exist 9 stress components to be deter-
mined. These 9 components of the stress at a (B) Strain - Displacement Relations
point mak c3-up the stress tensor(4) which is
the most convenient representation for the (c) Stress-Strain Relations
more difficult problems in theoretical analysis.
The three cutting planes are usually chosen (D) Boundary Conditions
parallel to a chosen space fixed cartesian co-
ordinate system, the choice usually clearly It should be noted at this time that the
indicated for the physical system under only change that is necessary in the equations
analysis, and thus, the stress components will of the linear theory of elasticity to the linear
all be parallel to this axis system. Other theory of thermoelasticity is in the stress-
axis systems such as cylindrical axes, can be strain relations.
chosen so long as there is provided the 3
mutually perpendicular planes at the point of A24.3 The Equilibrium Equations.
interest. These 9 stress com~onents are
related by the equilibrium equations as shown Consider the point 0 in an elastic body
in the next section. There will be 6 equations which is SUbject to external loading and intern-
of eqUilibrium and thus there eXists more un- al, or body, forces. It is desired to deter-
knowns (9) than there are equations which fact mine the state of stress at O. This could be
prevents any possible general solution. done completely by defining the resultant stress
tensor with respect to some spatial reference
Under the influence of this stress field but is most commonly done in the following man-
the elastic body will deform in a manner de- ner. Take the point of interest 0 as the origin
pendent upon the stress strain relations. The of three mutually perpendicular axes oriented
stress strain relations for the isothermal parallel to a chosen spatial cartesian axis
problem (the same temperature througllOut) are system. The choice of this reference axis
well known from the linear Hook's Law. (2,3,4) system is usually clearly indicated by the
The introduction of temperature modifies these geometry of the system under study.
relations by the addition of another strain
component due to the thermal expansion z
properties of the material being studied. 3-Dimensional Body
~,
These stress-strain relations provide 6 more
equations but at the same time 6 additional Point 0 within the
unknown strain components are introduced. The body
system of equations is still inc~nplete.
x
The strains Give rise to displacements.
)----------~y
The relationship between strains and displace-
ments which are independent of temperature and
temperature distribution requires 6 equations
for definition and at the same time introduces x
only 3 unlmowns which are the displacement
components. Assume that the body is cut by the planes
xy, xz, yz, having the cownon point of inter-
The number of unknowns is now 18 and there section O. Associated with each of these cut-
exists 18 equations so that the equation set is ting planes there will be a reSUltant stress
complete and a general solution is possible. vector at 0 reqUiring three components for
This general solution will be the unique definition. It is logical to choose the di-
THE 3-DIMENSIONAL EQUATIONS OF THERMOELASTICITY A24.3
rections of these components as those of the equation, surface areas over which the stresses
axis system. (being force intensities) are acting must be
provided. This is most conveniently done by
For example, consider the cutting plane considering the stress components acting over
yz. The resultant stress vector at 0 associ- the faces of a small parallelepiped constructed
ated with this cutting plane can be defined by with faces in or parallel to the cutting planes
the three components ax; ~xy; ~xz where contained in the cartesian coordinate system
with the point of interest 0 as the origin.
ax component normal to the zy plane This parallelepiped is considered arbitrarily
and therefore parallel to the small and in the limit as the edge dimensions
x - aXiS, hence the subscript x approach zero the equations resulting from
equilibrium of the element will describe the
~xy = oneof the stress components in the stress conditions at the point O. Consider
yz plane which is identified by the then such a parallelepiped which is already
normal direction x, hence the first considered infinitesimally small with edge
subscript, and in the y direction dimensions then correctly specified as dx, dy
as indicated by the second sub- and dz. The stress components on those faces
script. This component could be z
written as ~(yz)y but since the
axes are mutually perpendicular the
yz plane is more economically
identified by the subscript x as
shown.
When in doubt during the physical interpreta- 'Txy = 'Tyx ; 'Txz = 'Tzx ; 'Tyz = 'T zy - - - (5 )
tion of solutions to the equations of elasti-
city one should return to these two figures The equations 2, 3, 4 and the three equations
which define the positive sense of the stress of 5 make up the 6 eqUilibrium equations. In
components. the following discussions equations 5 will be
accepted and thus the preceding discussion on
The stress components have all been in- the equation set of 18 must be modified to the
creased an amount equal to the rate change of set of 15 with a similar reduction of 3 in the
magnitude in the direction of movement times number of unknowns.
the di~tance moved. This increase is, of
course, the change in magnitude between the It should be noted that these equations
components on the planes passing through 0 2, 3 and 4 define the equilibrium state of the
and those displaced an infinitesimal distance volume element and their derivation requires
away in the positive direction. In this no limiting assumptions as to the elastic
volume element, dxdydz, it will also be nature of the material. The equations are thus
possible to have body forces X, Y, Z defined qUite general and will be the same for non-
as the force per unit volume and which can linear systems and those in which displacements
arise as gravitational forces, magnetic are created by temperature differences. For
forces, etc., acting in the direction x, y convenience, these eqUilibrium equations are
and z respectively and through the centroid written in cylindrical coordinates r, Q and z.
of the volume element. It should be borne
in mind that these are internal forces within dar + ~ d'Trg + d'Trz + or - 0g
the element considered and are distinct from dr r dQ dZ r
the external forces which will appear in the z + R ;:: 0
equations defining the boundary conditions.
= i ~y -
In the case of the heated elastic body
Ey y(oz + oxTI these strain expressions must be modified by
Ez -If G
- 1 Oz - Y(Oy + oxTI
the addition of a component arising from the
thermal expansion property of all structural
materials. This component will be proportional
1 to the temperature level, T, above or below
txy =Cf'T;xy some reference temperature (usually room temper-
_ 1 ature). The constant of proportionality is the
t yz -G~YZ familiar linear coefficient of thermal expan-
sion, which, for isotropic materials is the
1 same in all directions of observation at the
lI'xz =Cf~xZ point O. It is obvious that the direct strain
components, Ex' Ey , EZ' being the change in
where E = Young's modulus = slope of the uni- length per unit length of a fictitious fiber
axial stress strain oriented in the x, y, z directions respectively,
curve are directly affected by the thermal expansion.
The shear stress components will not, however,
be affected for isotropic materials since the
G = shear modulus = 2(1 ~ Y) angular deformation of a small volume element
located at 0 will not be changed during the
v = Poisson's ratio = 0.3 for steel uniform expansion of this volume element. The
stress-strain relations for thermoelasticity
= 0.25 -. 0.50 for thus become:
all elastic mater-
ials at room EX =i ~x - y(Oy + oz] + aT - - - - (12)
temperature.
Ey =~
dy /YZ =dZ
dV + oW
oy
dw du oW
- - - - - - - - - - - - (15) EZ =dz- =-+--
oZ dx
1 6 strain-stress relations
7JXY = G 'txz - - - - - - - - - - - - (16)
EX = ~ ~x-V(OY+oz~ + aT
If yz =~ 'tyz - - - (17)
'txy = fJ,tXY
The equation set is now complete since the
'ryz = fJ,2lyZ number of independent equations is equal to the
number of unknowns.
'tzx = fJ,tzx A24.7 The Compatability Equations.
where the definitions of).., fJ, and e remain un- It will be noted in the strain displacement
changed from those previously given in the equations that 6 strain components are necessary
isothermal equations relating stress and to define the 3 displacement components. Thus,
strain. if one were to choose at random a set of 6
strain components this does not mean that there
A24.6 The General Equations of 3-Dimensional eXists a possible set of 3 displacement compon-
Thermoelasticity. ents which will allow satisfaction of the strain-
displacement equations.
The unknowns are: Consequently the choice of these 6 strain
components cannot be arbitrary and, in fact,
6 stress components ox,oy,oz,'rxy,'txz,'tyz must be such that the compatability equations,
6 strain components EX,Ey,Ez,txy,txz,tyZ which are equations relating the derivatives of
3 displacement components u, v, w the strain components, are satisfied. These
15 compatability equations thus ensure the exist-
ence of continuous single-valued displacement
The necessary 15 equations are given by functions. This, of course, ensures the unique-
ness of the solution when, as is possible, the
3 equations of equilibrium analysis is formulated in terms of stresses and
strains only. These equations are automatically
dox + d'rxy + d'rxz + X satisfied if the complete 15 equation set is
dX dy oZ =0 satisfied.
dOl + d'rxy + d'rlz + y These equations are derived in the follow-
dy dx dz =0 ing manner.
From equation 9
doz + d'txz + d'ryz + Z =0 J _ du dv
dZ dx dy °xy - -
dy +dx-
NORTH AMERICAN
LOCKHEED
Proposed Designs For Supersonic Airliners. Boeing won the competition but later changed design to
fixed wing type. Later Congress killed the SST project, a bad decision in this author's opinion.
CHAPTER A25
THE 2-DIMENSIONAL EQUATIONS OF ELASTICITY
AND THERMOELASTICITY
z
A25.1 mtroduction:In any elastic body wherein Assume: Oz = ~xy = ~yz = 0
the stress components at a point of interest w, oxz, ~yz small
all lie in the one plane and, due to symmetry
in the body itself or in the external loading, Unknowns: OX> Oy, ~xy, Ex' Ey , E z
the stresses throughout the body all lie in oxy, u, v
this one plane, the stress analysis becomes Plane of middle
2-dimensional. All stresses normal to this surface parallel
plane are zero. The analysis is then referred to xy plane
to as one of plane stress.
Z =°
25.5
where X, Yare the components of the external
surface forces per unit area in the directions
t XY = lG .,..'xy, 25.6
x and y respectively.
EX
au
= ax 25.7 u = f(s)
v = g(s) 25.15
av
Ey 25.8
ay w= °
t xy au + av The discussion of Chapter A24 applies to
= ay ax 25.9
these boundary conditions and should be studied
again if necessary. This discussion outlines
The equation set is complete for the the usual procedure of treating the thermal
general solution. There is still required an stress problem in stress formulation with all
equation for the strain components EZ and the surfaces traction free and then determining the
compatability equation required for uniqueness effect of external loads and removal of thermal
of solution from a stress formulation. The displacements where prescribed by means of an
expression for EZ is not usually included in isothermal analysis.
the completed set since it results directly
from the solution of this set and is given by, A25.3 Derivation of Equations.
25.13
The equations of equilibrium.
Consider the figure shown in Chapter A24
x =y= 0, on which the positive stress components are
shown for an element extracted from a three-
and E and a are constants (independent of dimensional body. If one were to view this
temperature and position in the bOdy). See figure from the z-axis in a positive z-direction
note in section A25.4 for case where X = 0, the following figure would be observed.
THE 2-DIMENSIONAL EQUATIONS OF ELASTICITY AND THERMOELASTICITY A25.3
+-
a'L
i y 'Lxy+~dx
OX---:-j dy L X Ox + a~XX dx
'L xy ~Jf:::::::==-~dx~.-=-=1 x
In matrix form, these equations can be written 1
'Lxy----r as
~oy
For the plane stress problem the stress com-
ponents shown are all that will be acting
since, by definition, the stress components and from Cramer's Rule it therefore follows that
out of the x-y plane are zero. The equations
of equilibrium are then readily derived by
summing the forces in the x and y direction,
remembering that the figure actually repre-
sents a plan view of a three-dimensional ele-
ment of depth dZ, and equating this algebraic and that
sum to zero.
+ __....:0..!!2_a. "-J.=-J._ _
Consider ZFy = O. aJ.:l. a 2" - a,,:l. aJ."
au 1 dr + au m dr
uJ. = u + ax ay
av
VJ. =v + - 1 dr + av m dr
ax ay
txy = ~ 't"xy
where 1 and m are the direction cosines of the
If the elastic body is heated it should be fiber. The coordinates of <\are u and v. The
obvious that the small element under consider- coordinates of OJ. are 1 dr and m dr. The co-
ation will expand uniformly and the resulting ordinates of O~ are
deformation due to temperature T will be equal
direct extensions aT, where a is the linear 1 dr + uJ.
coefficient of thermal expansion, in both the and m dr + vJ.
x and y directions. The shear strain component
will be unaffected. Thus the thermoelastic Thus the projection of the strained fiber
stress strain relations in the plane stress on the x-axis is given by
problem can be written as,
_ 1 ldr + uJ. - U =ldr + ~~ ldr + ~~ mdr
- E (ax - Yay) + aT
and along the y-axis by
av av
mdr + (vJ. - v) = mdr + ax ldr + ay mdr
J _1
6xy - G 't"xy From the right angle triangle it follows
that
Strain - Displacement Relations.
av aOx a'txy_
ey = ay ax + --a:y -
when the body
au av aoy a'txy_ =
forces X = Y 0
The term ay + ax is known as the shear --+---
ay ax 25.16
strain. The physical meaning can be seen by
consideration of a fiber oriented at an angle and
of 45 0 to the x-y axis system and therefore
subtending an angle of 90 0 at the origin.
It is left as an exercise to prove then that
au + av is the reduction in this subtended
ay ax =0 25.17
angle when the fiber is strained. This
physical picture is, perhaps, better under- when both E and a are independent of the
stood by reference to the figure below. temperature, T, and position in the body. The
derivation of this compatability equation in
terms of stresses is left as an exercise for
the stUdent.
av Y=
X = - -ax , av
- ay 25.18
ixy = reduction in original 90° angle.
the inverse solution technique usually proves
au av the most profitable. Since, in the linear
=-
ay+ax
-
problems of elasticity and thermoelasticity, the
stress formulation occurs most frequently this
It should be noted that the general ex- inverse approach is illustrated here in some
pression for strain, e, can now be rewritten detail. This approach is not restricted to the
in terms of ex, ey and txy as follows, plane stress problem now under discussion and
can be applied equally as well to plane strain
e = 12 ex 2
+ m ey + 1m txy or any two-dimensional stress formulations.
which is exactly analogous to the direct stress In general, one can choose a stress function
expression on a cutting plane oriented at an ¢ defined by the relations
angle to the x-y axis system, viz:
Ox =~
a2 ¢
a = 12 0x + m 2 0 y + 1m 'txy • ay
(a) stress Formulation - All boundary This function ¢ is commonly referred to as the
conditions given in terms of surface tractions. Airy stress function since Airy first applied
this well known mathematical technique to
The 2 eqUilibrium equations are al- problems in elasticity.
ready written in terms of the 3 stress com-
ponents OX> Oy, 'txy • The compatability equation It is obvious that the equilibrium
is given by 25.12. The equations which must equations are automatically satisfied by 0 When
therefore be satisfied by the three unknowns chosen according to 25.19. To obtain a general
ox, Oy, 'txy are: solution for the stresses it remains, therefore,
to solve only the compatability equation, 25.17,
which, when written in terms of 0, becomes
A25.6 AN INTRODUCTION TO
2.5.20 r
lil
=X lil
+ Y
lil
when X =Y =0 9
and
and the compatability equation from which the
general solution is obtained (equivalent to
equation 25.20 when the body forces are con-
when the body forces X & Yare defined by sidered zero).
25.18.
The stress function, 0, is defined by the
In both 25.20 and 25.21 equations
0=0
and a0
an
=0 25.22
This reduction of the 3-dimensional case
where n is ·the outward drawn surface normal at to the 2-dimensional plane stress problem can
all points on the boundary. ThUS, the specific be generalized to an eqUivalent problem which
solution can be obtained in terms of 0 directly includes the case where the temperature varies
and the specific solutions for ox, oy, and through the plate or slice thickness, t, by
~y by direct substitution into 25.19. employing the equations
ary conditions given in terms of boundary dis- grain, the rocket nozzle) atomic reactors
placements. By choosing the displacement (cylindrical fuel rods, heat exchanger tUbes)
formulation the compatability equation is auto- etc.
matically satisfied. The general solution for
the two displacements u and v are obtained ~he question of whether a system under
from the two equilibrium equation written in analysis can be considered one of plane stress
terms of displacements as follows: or one of plane strain is frequently raised and
should be answered at this point. One must
E a (au + av) E 'V,. consider the system carefully and check as to
2(I-Y) ax ax ay + 2(I+V) u whether all the requirements pertinent to one
or the other states has been satisfied. If any
aT
-aE
- (l-Y)
--+ ax
x=o have not then the problem is essentially a 3-
dimensional one and the 15-equation set must be
25.25 employed. However, in some cases, this decision
is not a clear cut one. Consider, for example,
the case of a cylindrical shell of finite
length h, of diameter b, and which is loaded
normal to its axis, body forces are zero and
temperature is independent of the axial co-
where, again, E, a and Yare constants. These ordinate. If one considers the ratio of length
equations are derived directly by the substi- to diameter, hlb, the problem can be classified
tution of equations 25.4, .5, .6, .7, .8 and .9 as follows
into equations 25.2 and .3.
h » 1 plane strain
The specific solution is obtained by b
sUbstituting the general solution into the
boundary condition equations, 25.15, and h
solving for the unknown constants. b «1 plane stress
8 equations of the plane strain problem result: resulting stress, strain and displacement com-
ponents would then be carried out and super-
2 equations of equilibriU~ imposed on the preceding solutions to obtain
the actual solution.
aox + a~XY + X =0
ax y 25.27 This problem arises from the fact that 0z
is prescribed by the solution of the complete
a~XY + aOy + y = 0 equation set. A similar problem would arise in
x cry 25.28 the plane stress problem if surface strains
were prescribed as boundary conditions which,
3 stress-strain relations (where EZ =0 of course, is never done.
Case 1. Traction boundary conditions. A25.6 Mathematical Equality of the Plane Stress and Plane
Strain Problems.
x = 0x l + 'Lxyill
It will be noted that if one makes the
Y = 0yill + 'Lxyl 25.35 following sUbstitutions,
In this plane strain problem there will result E
from the solution an apparent surface traction El. =1 _v 2
at the free ends given by
lJ
v~ =1 - )I
25.37
These equations are exactly of the same form (b) Displacement Formulation
as the same relations 25.4, 25.5, 25.6 in the
plane stress problem. In fact the two equation In this case again only the equilibrium
sets are now identical except for the change equations expressed in terms of displacements
in the elastic constants as given above. Thus must be satisfied for the general solution.
the mathematical problem is identically the These two equilibrium equations are:
same and consequently the discussion of section
A25.4 pertaining to the mathematical formulation
is directly applicable to this plane strain
problem so long as the reader changes the
elastic constants E, Y and a to E1 , Y1 and a 1 •
To avoid any confusion these discussions are
briefly repeated here and the pertinent 25.43
equations rewritten.
A25. 7 Formulation of Plane Strain Problem.
¢ =0 aOr + ~ a'trQ + or - oQ + av = 0
ar r aQ r ar
a¢ _
an - 0 ~ aOQ + a'trQ + 2 'trQ =0
r aQ ar r
where n is the outward drawn normal.
aOr + ~ a'trQ + or - oQ = 0
ar r aQ r
1. aOQ
r aQ
+ ~ + ~
ar r
=0
(5) Prove that the definition of the
stress function in cylindrical coordinates is
given by the equations
when the r - Q plane corresponds to the x-y (1) "Elasticity in Engineering" E. E. Sechler,
plane and body forces are absent. Galcit Aeronautical Series, John Wiley &
Sons, Inc., 1952.
(6) Establish the plane stress compat-
ability equation as given in the text and, by (2) "Applied Elasticity" Chi-Teh Wang, McGraw-
direct substitution for the elastic constants, Hill Book Co., Inc., 1953.
write the compatability equation for the plane
strain thermoelastic problem. Modify both (3) All references of Chapter A24.
these equations to include the eXistence of
CHAPTER A26
SELECTED PROBLEMS IN ELASTICITY AND THERMOELASTICITY
A26.1 Introduction.The preceding chapter pre- initial temperature, To, are completely pre-
sen~ed the general fonnulation of the two- scribed by the cross-sectional area A and the
dimensional equations of elasticity and thenno- length L. The only restraint is that imposed
elasticity. This chapter is devoted to the on the length L by the rigid walls. The
development of solutions of simple problems in problem is to determine the stress distribution
elasticity and th3nnoelasticity employing these throughout the bar and the change in shape when
equations rather than the Strength of Materials temperature is increased to T.
approach which will be discussed in the chapter
on bea~s. These problems h~ve been selected Assume the bar is unrestrained during the
to illustrate clearly the fonnulation of the heating. The change in length will be
governing equations for the chosen mathematical
model, the sO~ltion of these equations and the l\L a(T - To)L
interpretation of the results. A large store
of such problems exist in the literature, or, the new heated total length will be
especially in the case of the isothermll
analysis, which is referenced at the close of L~ = L [! + arT - To)]
this chaptero Consequently the problems chosen
for this cl1apter are to illustrate the technique T To '= Temperature change
only and, while they may appear trivial to
some readers, they are intended to provide a where a = Coefficient of linear thennal
reasonably sound knowledge of the tec~niques expansion
so that the literature can be read easily and
with confidence. In all of the problems of = Change in length per unit length
this chapter the basic assumntions of Chapter (at a chosen reference tempera-
A24 are observed and consequently the dis- ture level) per degree change in
cussion is restricted to elastic materials temperature.
whose properties are not affected by temper-
ature. In engineering it is common practice to
measure temperature chan~e in OF and to use the
The chapter opens with a selection of reference temperature as 32 0 F. So long as a is
simple one-dimensional problems which are inter- a constant the reference temperature could be
jected at this point merely to provide the as well chosen as the original temperature
student with an understanding of the behaviour since, by the definition above, the value of a
of metals when they are heated and the elastic would be the same. However, for most materials,
constants are not affected by the temperature a is a coefficient of the temperature and there-
rise. This section is then followed by a fore the values given in the following table are
selection of problems in which th3 fonnulations applicable only over the stated temperature
of the preceding chapter are employed.
TABLE A26.1
A26.2 One-Dimensional Problems. REPRESENTATIVE VALUES OF Q
(Original length measured at 32 0 F)
&"'{ample 1. Material Temp. OF Q
Consider a cylindrical bar of metal Aluminum (99.95%) 13.22 x 10- 8
68 - 212
mounted between two rigid walls as shown in Commercial 68 - 212 13.33 x 10- 6
Figure A26.1. Beryllium (98. 9%) 68 - 212 6.83 x 10- 6
Brass (66 Cu, 34 Zn) 32 - 212 10.42 x 10- 6
Bronze (93.5 Cu, 6. 5 Sn) 68 - 212 9.72 x 10- 6
Graphite 100 4.37 X 10- 6
Copper 70 - 212 9.33 x 10- 6
Glass 32 - 212 4.63 x 10- 6
Steel 32 - 212 5.88 x 10- 6
L Molybdenum 70 - 212 2.72 x 10- 6
Invar (Nickel Steel) 100 0.46 X 10- 6
Stainless Steel 70 - 212 5.33 x 10- 6
Fig. A26.1 Tungsten (99. 978%) - 212 2.33 x 10- 6
32 - 950 2. 56 x 10- 6
(powder metallurgy) 4000 5.65 X 10- 6
The geometrical properties of the bar at the Magnesium Oxide 75 - 212 5.6 x 10- 6
A26.1
A26.2 SELECTED PROBLEMS IN ELASTICITY AND THERMOELASTICITY
range. Furthermore, every alloy of the for each bar. The problem is to determine the
materials shown possess a characteristic Q. axial stress in each bar and the displacement
The table can, therefore, be used only as a of the interface under the assu~ption of no
guide to the appropriate values of Q and the restraints in the radial direction and no sur-
exact value obtained from a more extensive face tractions.
source.
Again we remove the restraint from one
Now, for the exa~ple under discussion, it end. The total expansion will be
can be seen that the bar will have to be com-
pressed an amou~t 6L to fit between the rigid
walls. This will give rise to a compressive
strain of 6L/L, or and the bars must be co~pressed by a common
force P so that this total displacement is re-
£ = - a(T - To) in the axial direction, moved.
and a compressive stress of
The stress in bar 1 will be a ~ -- A~
-P
a = - Ea(T - To)
Thermal expansion in the radial direction and the stress in bar 2 will be
as well as an elastic strain due to Poisson's
effect will also occur. However there will be The corresponding strains will be
no stresses in the radial direction since
P
there are no restraints or expansion in this
direction and no surface tractions are applied.
£~ =- A~E
Thus there will exist the compressive stress and
uniformly distributed over the cross-section £2 = - APE
2
throughout the length of the bar, if, and only
if, no restraints on the radial deformations The corresponding displace~ent, or change
exist at the ends against the walls. The in length, of each bar will be
total load reacting against the wall will be
P
u~ = - A E L~
~
Bar 1 Bar 2
The final position of the interface will
be located at a distance x from the left hand
A~ A2 wall given by the original length of bar I plus
Q ~ Q2
L~ L2 its thermal elongation plus u~, or
E~ E2 (Young's modulus the
same in both bars. ) L~ (a~L~+a2L2)(T-To)
Fig. A26.2
x = L~ + a~L~ (or - To) - E (h + A~L2)
E A2 E
The geometrical properties and the linear It is interesting to note that this problem is
coefficient of thermal expansion are different essentially not this simple. The analysis re-
A26.3
r
ul = 0
~ x
u
Al
Ql
n ,
x~
1
Az
Q z r
L~ Lz U z= 0
E~ Ez (Young's modulus
different)
Fig. A26.3 and
Example 4.
~,(T~-To)L-aa(Ta-To)L- (C~ +c a:2] - PL
E,A
A direct application of the preceding
examples is the determination of shear and and the motion of bar 2 will be EP~, Since
a a
tensile stresses developed in bolts of a bolted these motions must be the same it follows
connection. Consider the bolted connection directly that
between tVIQ dissimilar metals which are heated
to different temperatures T~ and T a above
ambient temperature To. Initial bolt clear-
ances are normally allowed in bolted connections
to relieve thermal stresses and such clear-
ances are shovm in Figure A26.4 as C~ and Ca'
In this chosen case, P will produce compression
in the bar 1 and tension in bar 2 since the ex-
pansion of bar 1 was initially greater than that
Bar 1
of bar 2 and it was assumed that this relative
expansion was greater than the total clearance,
T1 whiCh, of course, is necessary for the creation
of thermal stresses at all. The solution should
be generalized to
Bar 2
Fig. A26.4
Bar 1 expands an amount a~(T~ - To)L = There will be an elastic contraction however,
displacement of hole center line. due to the bolt increased tension force P which
will be
Bar 2 expands an amount ua(T a - To)L
z
~
~ V
~VL..-_--'
/
X
(u)
1 DT
M
compatability
Example 1.
As the first exa~ple let us consider the
problem of the simple beam subjected to an and the boundary conditions
externally applied moment and with the whole
system at room temperature. The physical
system is as shown beloN in Figure A26.5.
(summation of ~xy =°
moments = 0)
(This could have been assumed directly, of
course, since there is no externally applied
shear present, but the matter of zero internal
From assumption (3) shear has been belabored since many students
appear to have some difficulty in grasping this
0z = ~Xz = ~yz =° simple fact.)
an,1 thus Thus the only stress component present is
Ox and the equations of equilibrium are satis-
fZ =tf xz = -Iyz = ° fied. It is obvious also that since
Since the external loading is bending only
and there is no variation of same in the
x-direction, it is reasonable to assume the ~ Ox
strain distribution to be of the following f y - - E
form:
Ixy=o,
f X = Co + C],y (plane sections
remain plane) the equation of compatability is also satisfied.
Thus we have the unique general solution to the
fy =f (y)
mathematical model chosen.
and thus
Co = 0
M M
C], = = -EI
or E df(y)
dy
= 0, a constant or a function of x.
E
i+~
.sl t;' dy
Since u 0 at x = =
0, it follows that equations. From the first, viz;
f~(y) =
O. Then, from the third strain-
displacement relation it follows that
dg,(x) Mx
dx E1
peL - x)
E1
w = O. so that
Example 2. ax
=- P(L - x)
I Y
Consider now the same problem but with
the external load a transverse force, P,
applied at the end. From the first equation of equilibrium it
follows that
~ r-- L --------->l~
d'Txy = Py
~ l__ _
y
--+l X dY I
/
By integration
Fig. A26.6
'Txy =_ Py 2
+ f(x)
The same basic asswnptions are made as in 21
the previous example and the basic equations
are the same. The boundary condition Now, since there must be zero shear on the
equations are the same when the applied moment, top and bottom faces of the beam, i.e., at
11, is given by + d
x = - 2 it follows directly that f(x) is a
11 = peL - x) constant given by
We will again assume that the strain distri-
bution is of the following form
EX = (Co + C~y)g(x)
from Hooke's law and the assumptions to give It is interesting to note that this
solution differs from that developed from the
EZ = ~XZ = Oyz = 0 Strength of Materials approach, The term
PLd" .
BIG arlses from the shearing force and is
P
EX = - Ef (L - X)y effectively a rotation at the end x =0 given
by
Ey =+ ~ P(L - X)y
EI oV I
ax
x=O
y=0
j( _ P d2
iJ xy - 2IG (4 - y2)
Thus, the complete expressions for u and v
ani then, from the strain-displacement relations will be
u =- il
P (L - ~) + f (y) u = _ Pxy (L _~) + ~ (d 2y _ y3) + ).IPy3
EI 2 2IG 4 3 6EI
v =~
2EI
(L - X)y2 + g(x)
Pd 2x Px2L Px 3
- YP
where f(y) and g(x) are functions respectively V - - x) Y2 + 8IG + 2EI - 6EI
2EI ( L
of y and x alone. It is the aim in this
example to evaluate these functions in a more
general sense than was done in Example 2.
Substitution of the expressions for u and v Now to fix the beam at the end x = 0 it is
necessary to prevent a translation in the u and
in the third strain-displacement relation, viz: v directions and to prevent rotation at this
end. Thus the end conditions can be written as
u(o,o) = v(o,o) = 0
gives
and au I =0
Pd2
2IG (""4 - y2)
P
= - E~ (L -1) + f' (y) ayx--0 , y=o
_ I1P y
2'2.:1
2 + g' (x) on ~I =0
axx=o, y=O
, Pd 2 Px X
av I
ax
=0
g (x) = 8IG + EI (L -"2) - CJ. x=o, y=o
Applying these conditions and using the
other available equation in CJ. and C2 provides
A26.10 SELECTED PROBLEMS IN ELASTICITY AND THERMOELASTICITY
Oy = w, at y =- 2"d
Fig. A26.7
that the stress components are:
2
2 8 EI 4
2 2
2
6
4
v = _wt{Lx_ }d _wt{L X _LX" +x }
24
2
wt d 2 J:: wt d 2 +1:. wt d
EI 40 x - E I 16 x G I ""8 Lx
The solution follows directly the steps of the
preceding example. In the majority of problems General Note: Since we are dealing with linear
the variation of w along the beam is linear, problems, it is possible to break down any
lumped into a sequence of discrete loads, or w loading given into components matching those of
is a constant. In any of these cases this the problems given above and by direct super-
integral is readily evaluated in the initial position of the separate solutions corresponding
steps of the problem solution. For example, to these component loadings obtain the final
if w = constant, solution for the complex combined loading states.
This general technique will be employed to
obtain solutions to general thermoelastic
problems in beams.
= C
v at the root (x = 0) is
negligible. This assumption Solution:
is questionable when the root
fixing is considered. The The general solution is obtained by direct
effect of the inclusion of integration as
root restraint on v can be
evaluated by superposing a Ox = - aET + C~ Y + C 2
self-equilibrating isothermal
stress pattern to restrain v. Use of the boundary conditions allows the
The effect will only be evaluation of C~ and C 2 as
appreciable in an area local
to the root of a beam (L» d)
and therefore will be
neglected.
(iv) Material properties are
independent of temperature.
Since Ox and T are functions of y only, this ox' [-T+~L :dy ~; yL;Yd
oE + Y]
equation further reduces to the ordinary
differential equation 2 2
Eq. (A)
and it follows directly from the general stress-
strain relations with Oy = 0;
Boundary Conditions.
The cantilever beam is replaced by a beam
of length 2L to satisfy the condition imposed
by symmetry of zero deformation in the
x-direction at the root. On this beam all
surfaces must be zero. Hence the boundary
conditions on Ox can be written as;
and
(1) The reSUltant force over the end
face at x + L is zero or
A26.12 SELECTED PROBLEMS IN ELASTICITY AND THERMOELASTICITY
reduces to
+ aT (1 + )) )
t XY = O.
Integrating both sides, with respect to y, will
and the displacements obtained by direct inte- give
gration of the strain-displacement relations
with satisfaction of the boundary conditions
u = 0 at x = o.
+ C",y + C4
a2
v a2 u
and ax 2 = - axay and the strains and displacements obtained as
in the preceding example.
d
L
2 Note that if T is linear, or satisfies
=- 12 V2 T = 0,
then ¢ m 0 everywhere and consequently
a ~ _£ Tydy no stresses will result. Strains, however, will
exist and therefore deformations will result.
2
Example 7. Numerical example.
=constant.
Consider a cantilever beam with the follow-
Thus the curvature is constant along the ing dimensions and material properties.
length of the beam and v can be considered due
to an effective end moment of value L = 10 ins. a = 7 x 10-6
~
Example 6.
The preceding problem can, of course, be
solved employing the inverse formulation in
terms of the stress function ¢ as outlined in lOOOOF
Chapter A25.
Y~ d d
The thermal gradient has been chosen as a
2 0 -2
function of y only, and since cry = 0, it follows
A26.13
J.I ~dY
distribution is linear across the depth and
Then = 2133 ; (~YdY = 66. given by T(g) = A + By. The stress components
will then be given by
l )-1
Ox = ox~ (due to moment M) + Ox" (due to
Then Ox = 210 (-50y" + 16) From Eq. A temperature)
Oy =0
Thus Ox / d = -6,740 Ib./ins." compression
'JIxy = 0
y=::!:"2
and the displacements by
= + 0.57 ins.
Concluding Remarks.
trivial problems in the theory of elasticity the inclusion of the kinematical variables) and
and thermoelasticity. We have restricted our that the elasticity problem due to this
attention to simple beam problems where the temperature distribution can be considered
bea~ depth is small compared to the length and separately. This assumption is valid only if
where the temperature varies across the depth the rate processes involved are slow.
only. However, the basic equations and
solution techniques are basic to the solution REFERENCES
of more complex problems where the temperature
is a function of two spatial coordinates, 1. Boley, Bruno A. and Weiner, Jerome H.,
T = T(x,y), and the geometry is that of a "Theory of Thermal stresses". John Wiley
general two dimentional plate. In these & Sons, New York, 1960.
problems all the stress components will exist
and displacements in the z-direction may occur 2. Gatewood, B. E., "Thermal stresses" with
in the general two-dimensional plate problem. Application to Airplanes, Missiles,
Generalization of the teclLniques illustrated Turbines and Nuclear Reactors. McGraw
here in this class of problems is beyond the Hill, New York, 1957.
scope of this discussion and the r~ader who
wishes to pursue this sUbject is referred to 3. Timoshenko, S. H. and Goodier, J. N.,
references listed in next column which can now "Theory of Elasticity", 2nd Ed., Chapter 14,
be read with ease. McGraw Hill, New York, 1951.
applied. Relative to the length of time in thus repeated loads are important in design of
applying the load to a member, two broad structures.
classifications appear logical, namely,
(1) Static loading and (2) Dynamic loading. Dynamic or Impact Loading. A dynamic or impact
For purposes of explanation and general loading when applied to a member produces
discussion, these two broad classifications appreciable shock or vibration. To produce
will be further broken down as follows:- such action, the load must be applied far more
rapid than in a static loading. This rapid
Continuous Loading. application of the load causes the stresses
Gradually or slowly applied in the member to be momentarily greater than
Static Loading loading. if the same magnitUde of load was applied
{ Repeated gradually applied statically, that is slowly applied. For
loading. example, if a weight of magnitUde W is
gradually placed on the end of a cantilever
Impact or rapidly applied beam, the beam will bend and gradually reach
Dynamic Loading loading. a maximum end deflection. However if this
{ Repeated impact loading. same weight of magnitude W is dropped on the
end of the beam from even such a small height
Static Continuous Loading. A continuous load as one foot, the maximum end deflection will
is a load that remains on the member for a long be several times that under the same static
period of time. The most common example is the load W. The beam will Vibrate and finally
dead weight of the member or the structure it- come to rest with the same end deflection as
self. When an airplane becomes airborne, the under the static load W. In bringing the
weight of the wing and its contents is a con- dynamic load to rest, the beam must absorb
tinuous load on the wing. A tank subjected to energy equal to the change in potential energy
an internal pressure for a considerable period of the falling load W, and thus dynamic loads
of time is a continuous load. Since a contin- are often referred to as energy loads.
uous load is applied for a long time, it is a
type of loading that provides favorable con- From the basic laws of Physics, force
ditions for creep, a term to be explained later. equals mass times acceleration (F = Ma) and
For airplanes, continuous loadings are usually acceleration equals time rate of change of
associated with other loads acting simultan- velocity. Thus if the velocity of a body
eously. such as an airplane or missile is changed in
magnitude, or the direction of the velocity
Static Gradually or Slowly Applied Loads. A of the vehicle is changed, the vehicle is
static gradually applied load is one that accelerated which means forces are applied to
slowly builds up or increases to its maximum the vehicle. In severe flight airplane
value without causing appreciable shock or maneuvers like pulling out of a dive from
vibration. The time of loading may be a high speeds or in striking a severe trans-
matter of seconds or even hours. The stresses verse air gust when flying at high speed, or
in the member increases as the load is in- in landing the airplane on ground or water,
creased and remains constant when the load the forces acting externally on the airplane
becomes constant. As an example, an airplane are applied rather rapidly and are classed
which is climbing with a pressurized fuselage, as dynamic loads. Chapter A4 discusses the
the internal pressure loading on the fuselage subject of airplane loads relative to Whether
structure is gradually increasing as the they can be classed as static or dynamic and
difference in air pressure between the inside how they are treated relative to design of
and outside of the fuselage gradually increases aircraft structures.
as the airplane climbs to higher altitudes.
Bl. 4 The Static Tension Stress-Strain Diagram.
Static Repeated Gradually Applied Loads. If a
gradually applied load is applied a large num- The information for plotting a tension
ber of times to a member it is referred to as a stress-strain diagram of a material is ob-
repeated load. The load may be of such nature tained by loading a test specimen in axial
as to repeat a cycle causing the stress in the tension and measuring the load With corres-
member to go to a maximum value and then back ponding elongation over a given length, as
to zero stress, or from a maximum tensile the specimen is loaded statically (gradually
stress to a maximum compressive stress, etc. applied) from zero to the failing load. To
The situation envolving repeated loading is standardize results standard size test
important because it can cause failure under specimens are specified by the (ASTM) American
a stress in a member which would be perfectly Society For Testing Materials. The speed of
safe, if the load was applied only once or a the testing machine cross-head should not
small number of times. Repeated loads usually exceed 1/16 inch per inch of gage length per
cause failure by fracturing Without warning, minute up to the yield point of the material
Bl. 3
The clad aluminum alloys have two E values proportional limit stress.
as indicated in the lower diagram of Fig. Bl.l.
The initial modulus is the same as for other Tensile Yield Stress (Fty). In referring
aluminum alloys, but holds only up to the pro- to the upper diagram in Fig. Bl.l, we find
portional limit stress of the soft pure that some materials show a sharp break at a
aluminum coating material. Immediately above stress considerably below the ultimate stress
this point there is a short transition stage and that the material elongates considerably
and the material then exhibits a secondary with little or no increase in load. The
modulus of Elasticity up to the proportional stress at which this takes place is called
limit stress of the stronger core material. the yield point or yield stress. However many
This second modulus is the slope of the second materials and most flight vehicle materials do
straight line in the diagram. Both modulus not show this sharp break, but yield more
values are based on a stress using the gross gradually as illustrated in the middle diagram
area which includes both core and covering of Fig. Bl.l, and thus there is no definite
material. yield point as described above. Since
permanent deformations of any appreciable
Tensile Proportional Limit Stress. (Ftb). The amount are undesirable in most structures or
proportional limit stress is that stress which machines, it is normal practice to adopt an
exists when the stress strain curve departs arbitrary amount of permanent strain that is
from the initial straight line portion by a considered admissible for design purposes.
unit strain of 0.0001. In general the pro- Test authorities have established this value
portional limit stress gives a practical of permanent strain or set as 0.002 and the
diViding line between the elastic and inelastic stress which existed to cause this permanent
range of the material. The modulus of strain when released from the material 1s
elasticity is considered constant up to the called the yield stress. Fig. Bl.l shows how
B1.4 BEHAVIOR OF MATERIALS AND THEIR PROPE RTIES
it is determined graphically by drawing a line Figs. Bl.3 and Bl.4 compare the shapes of
from the 0.002 point parallel to the straight the tension stress-strain curves for some
portion of the stress-strain curve, and where common aircraft materials.
this line intersects the stress-strain curve
represents the yield strength or yield stress. B1. 5 The Static Compression Stress-Strain Diagram.
Ultimate Tensile Stress (Ftu). The ultimate Because safety and light structural weight
tensile stress is that stress under the maxi- are so important in flight, vehicle structural
mum load carried by the test specimen. It design, the engineer must consider the entire
should be realized that the stresses are based stress-strain picture through both the tension
on the original cross-sectional area of the and compressive stress range. This is due to
test specimen without regard to the lateral the fact that buckling, both primary and local,
contraction of the specimen during the test, is a common type of failure in flight vehicle
thus the actual or true stresses are greater structures and failure may occur under stresses
than those plotted in the conventional stress- in either the elastic or plastic range. In
strain curve. Fig. Bl.2 shows the general general the shape of the stress-strain curve
relationship between actual and the apparent as it departs away from the initial straight
stress as plotted in stress-strain curves. line portion, is different under compressive
The difference is not appreciable until the stresses than when under tensile stresses.
higher regions of the plastic range are Furthermore, the various flight vehicle
reached. materials have different shapes for the region
lIJ , . . - - - - - - - - - - ,
of the stress-strain curve adjacent to the
lIJ
Ql Actual Stres~ / straight portion. Since light structural
M
.....
00 '" weight is so important, considerable effort is
Ql made in design to develop high allowable
::::lIJ compressive stresses, and in many flight
5i
Eo<
Apparent j vehicle structural units, these allowable
..... Stress Ultimate design compressive stresses fall in
~ L..- -' the inelastic or plastic zone.
Fig. B1. 2
Unit Strain
Fig. 81.5 shows a comparison of the
stress-strain curves in tension and compres-
sion for four Widely used aluminum alloys.
240 Below the proportional limit stress the
modulus of elasticity is the same under both
0 200 tension and compressive stresses. The yield
...
0
0 stress in compression is determined in the
same manner as explained for tension.
...
~
00
160
P. Compressive Ultimate Stress (F cu )' Under a
lIJ~ 120 static tension stress, the ultimate tensile
lIJ
Ql
M stress of a member made from a given material
i15 80 is not influenced appreciably by the shape of
the cross-section or the length of the member,
40 however under a compressive stress the
ultimate compressive strength of a member is
.05 greatly influenced by both cross-sectional
shape and length of the member. Any member,
unless very short and compact, tends to
Fig. B1. 3
buckle laterally as a whole or to buckle
200
laterally or cripple locally when under
compressive stress. If a member is quite
0 short or restrained against lateral buckling,
160
...
0
0
~
then failure for some materials such as stone,
wood and a few metals will be by definite
Fip..l 120 fracture, thus giving a definite value for
the Ultimate compressive stress. Most air-
ai 80 craft materials are so· ductile that no fracture
lIJ
Ql
M
is encountered in compression, but the material
i15 yields and swells out so that the increasing
cross-sectional area tends to carry increasing
load. It is therefore practically impossible
Unit Strain to select a value of the ultimate compressive
stress of d uctlle materials without having
Fig. B1. 4 Stress-Strain Curves. Initial Portions.
Bl.5
70 70
24S-T3 SHEET 24S-T4 EXTRUSIONS
THICKNESSc O.2~O-IN THICKNESS < 0.2~0-IN. / E,
Ej'
-
60 60
~. X-COMPRES;:'O:'\
W-TENSION _ TENSI~N"""\ f ------,
I ;'
"'- ........... Ii I
40 " "- ~ "'-W'COMPRESSION
........ 40
u::;:N
7"/(11. !
:!
J~r-;- ~ / r--... 51
.,
"
<II
<II
30 W 30
20
....rr<II
V 20
/
Sfrain e
10
/ 10 /
Fig. Bl. 6
0
0
/ 2 4 6 8 10 00
V The slope of this tangent gives the local rate
of change of stress With strain. The secant
e X 10' IN liN E X 10-· PSI
e
2 4
X 10' IN liN
6 8 10
E X 10-· PSI
modulus Es is determined by drawing a secant
(straight line) from the origin to the point
80 .---,---,--~:-:-c::-~-:--:-~--,
W- COMPRESSION ..I
I COMPRE SSI0J.
TENSION I _
in question. This modulus measures the ratio
.. X - etlMPRESSION --.. IL 80 t--.......t - - - t - - j - - - + '1;;,_'-"""",,! between stress and actual strain. Curves
701--"'" W-TENSION~rt: r---r---L~ which show how the tangent modulus varies
601--
~~~ENSIO~ 70 / .............
"\
With stress are referred to as tangent modulus
curves. Fig. Bl.5 illustrates such curves
201--
/ f,\Lct.AD TIlS-n SHEET ~ 20/-----.f---t--t--+--t+---1
If a member is SUbjected to a certain
'0 L AND PLATE /
stress, the member undergoes a certain strain.
I If this strain vanishes upon the removal of
0
I' THICKNlESS 0·101I - 0 •4
I"-IN
•
10I-j-+---+----+----+---+--+I-----l
75S-T6 EXTRUSIONS the stress, the action is called elastic.
0 2 4 6 • 10 THICKNESS. 3-IN
Generally speaking, for practical purposes,
0
e X 10· INIIN E X 10" PSI 0 2 4 6 8 10
12 a material is considered elastic under stresses
X-Crossgrll.ln e X 10' IN liN E X 10-· PSI
up to the proportional limit stress as
Fig. Bl. 5 previously defined. Fig. Bl.7 illustrates
elastic action. However, if when the stress
is removed, a residual strain remains, the
some arbitrary measure or criteron. For action is generally referred to as inelastic
wrought materials it is normally assumed that or plastic. Fig. Bl.8 illustrates inelastic
Fcu equals Ftu' For brittle materials, that action.
are relatively weak in tension, an Fcu higher
!~:~~t
than Ftu can be obtained by compressive tests Elastic Action
of short compact specimens and this ultimate
: t~OP'Umit
compressive stress is generally referred to
as the block compressive stress.
dE
Percent elongation = (La L- 0L 0) 100 = measureof
ductility.
H I K
where Lo = original gage length and La = gage
length after fracture. In referring to
ductility in terms of percent alongation, it
is important that the gage length be stated, high and the strain at this stress is high,
since the percent elongation will vary with or from equation (2), when the proportional
gage length, because a large part of the total limit stress is high and modulUS of elasticity
strain occurs in the necked down portion of the is low.
gage length just before fracture.
In Fig. Bl.9 if the stress is released
B1. 9 Capacity to Absorb Energy. Resilience. Toughness. from point D in the plastic range, the recovery
diagram will be approximately a straight line
Resilience. The capacity of a material to DD parallel to AO, and the area CDE represents
absorb energy in the elastic range is referred the energy released, and often referred to as
to as its resilience. For measure of hyper-elastic resilience.
resilience we have the term modulus of
resilience, which is defined as the maximum Toughness. Toughness of a material can be
amount of energy per unit volume which can be defined as its ability to absorb energy when
stored in the material by stressing it and stressed in the plastic range. Since the
then completely recovered when the stress is term energy is involved, another definition
removed. The maximum stress for elastic would be the capacity of a material for
action for computing the modulus of resilience resisting fracture under a dynamic load.
is usually taken as the proportional limit Toughness is usually measured by the term
stress. Therefore for a unit volume of modulus of Toughness which is the amount of
material (1 cu. in.) the work done in stressing strain energy absorbed per unit volume when
a material up to its proportional limit stress stressed to the ultimate strength value.
would equal the avera~e stress f p /2 times the
elongation (E p ) in one inch. If we let U In Fig. Bl.9, let f equal the average
represent modulus of resilience, then stress over the unit strain distance dE from
F to G. Then work done per unit volume in
stressing F to G is fdE which is represented
U =(: p) Ep ----- --- -- ---- - (1) by the area FGHI. The total work done in
stressing to the ultimate stress f u would
Ek
But Ep = fplE, hence
then equal I fdE, which is the area under
the entire stress-strain curve up to the
=(f;) (f~)
ultimate stress point, or the area 0 A J K 0
U - fp2/2E (2) in Fig. Bl.9 and the units are in. lb. per
cu. inch. Strictly speaking it should not
include the elastic resilience or the energy
absorbed in the elastic range, but since this
Under a condition of axial loading, the modulus area is small compared to the area under the
of resilience can be found as the area under curve in the plastic range it is usually
the stress-strain curve up to the proportional included in toughness measurements.
limit stress. Thus in Fig. Bl.9, the area OAB
represents the energy absorbed in stressing It should be noted that the capacity of
the material from zero to the proportional a member for resisting an aXially applied
limi t stress. dynamic load is increased by increasing the
length of a member, because the volume is
High resilience is desired in members increased directly With length. However, the
subjected to shock, such as springs. From Ultimate strength remains the same since it
equation (1), a high value of resilience is is a function of cross-sectional area and not
obtained when the proportional limit stress is of volume of the material.
Bl. 7
When a material is stressed, it will (2) Locate any other point on the typical
deform in the direction of the stress and also curve such as point C, and draw line
at right angles to it. For axial loading and from 0 through C.
for stress below the proportional limit stress,
the ratio of the unit strains at right angles (3) Locate point D on line 0 C by the follow-
to the stress, to the unit strain in the ing ratio:-
direction of the stress is called Poisson's
ratio. It is determined by direct measurement OD = OB
OA x OC
in a tensile or compressive test of a specimen,
and is approximately equal to 0.3 for steel (4) Repeat step 3 to obtain a number of
and 0.33 for non-ferrous materials. In many points as shown by dots on Fig. Bl.ll,
structures there are members which are subjected and draw smooth curve through these points
to stresses in more than one direction, say to obtain desired stress-strain curve.
along all three coordinate axes. Poisson's
ratio is used to determine the reSUltant stress
and deformation in the various directions.
Poisson's Ratio in Plastic Range. Information
is somewhat limited as to Poisson's ratio in f
(PIll
the plastic range and particularly during the
transition range from elastic to plastic action.
For the assumption of a plastically
incompressible isotropic SOlid, Poisson's ratio
assumeS a value of 0.5. Gerard and Weldform
found from their research, that the transition ~ (1Dcbu/1IIcbl
of Poisson's ratio from the elastic value of Fig. B1.11
around 1/3 to 1/2 in the plastic range is
gradual and is most pronounced in the yield B1. 12 Non-Dimensional stress-strain Curves.
point region of the stress-strain curve.
The structural designer is constantly
confronted with the design of structural units
which fail by inelastic instability. The
B1.8 BEHAVIOR OF MATERIALS AND THEIR PROPERTIES
solution of such problems requires information on the stress-strain curve. Table Bl.l gives
given by the compressive stress-strain curve. the values of F o • 7 , F o • s &, n, etc., for many
Since flight vehicles make use of many differ- flight vehicle materials. Notice that the
ent materials, and each material usually has shape parameter varies Widely for materials,
many different states of manufacture which being as low as 4 and as high as 90.
give different mechanical properties, the
question of time required to obtain certain B1. 13 Influence of Temperature on Material Properties.
design information from stress-strain curves
becomes important. For example, in the Before the advent of the supersonic air-
aluminum alloys alone there are about 100 plane or the long range missile, the aero-
different alloys, and when elevated temperatures nautical structures engineer could design the
at various time exposures are added, the number airframe of aircraft using the normal static
of stress-strain curves required is further mechanical properties of materials, since the
greatly increased. temperature rise encountered by such aircraft
had practically no effect on the material
Fortunately, this time consuming work was strength properties. The development of the
greatly lessened when Ramsberg and Osgood turbine jet and rocket jet power plants pro-
(Ref. 1) proposed an equation to describe the vided the means of opening up the whole new
stress-strain curve in the yield range. Their field of supersonic and space flight. The
proposed equation specifies the stress-strain flight environmental conditions were now
curve by the use of three parameters, the greatly expanded, the major change being that
modulus of elasticity E, the secant yield aerodynamic heating caused by high speeds in
stress F o • 7 , which is taken as the line of the atmosphere caused surface temperatures
slope 0.7E drawn from origin (see Fig. Bl.12), on the airframe which would greatly effect
and a parameter n which describes the shape the normal static material strength propertIes
of the stress-strain curve in the yield region. and thus temperature and time became important
In order to evaluate the term n, another stress in the structural desIgn of certain types of
F o s& is needed, which is the intersection of flight vehicles.
the curve by a line of slope of O.85E through
the origin (see Fig. Bl.12). B1. 14 Creep of Materials.
E/l Stress-Strain
Curve
a material because if the temperature is high
enough the material will melt or flow and thus
have no load carrying capacity as a structural
member. When a stressed member is subjected
to temperature, it undergoes a change of shape
F 0.86 in addition to that of the well known thermal
expansion. The term creep is used to describe
this general influence of temperature and time
f on a stressed material. Creep Is defined in
general as the progressive, relatively slow
change in shape under stress when subjected
to an elevated temperature. A simple illustra-
tion of creep is a person standing on a
E
bituminous road surface on a very hot summer
day. The longer he stands on the same spot
Fig. B1.12 the deeper the shoe soles settle into the road
The Ramsberg and Osgood proposed three surface, whereas on a cold Winter day the same
parameter representation of stress-strain time of standing on one spot would produce no
relations in the inelastic range Is:- noticeable penetration of the road surface.
High temperature, when used in reference
- - - - - (3) to creep, has different temperature values for
different materials for the same amount of
The equation for n is, creep. For example, mercury, which melts at
-38 0 F, may creep a certain amount at -750F,
n =1 + loge (17/7)/loge(F o • 7 /F o • u ) (4) whereas tungsten, which melts at 6l70 0r, may
not creep as much at 2000 0F as the mercury
Fig. Bl.13 is a plot of equation (4). The under -75OF. All materials creep under
quantities EE/F 0.7 are non-dimensional and may conditions of temperature, stress and time of
be used in determining the non-dlmenslonal stress application. The simplest manner in
curves of Fig. B1.l4. E, n, and F o • 7 must be whIch to obtain the effects of creep is to
known to use these curves in obtaining values study Its effect on the static stress-strain
diagram for the material.
Bl.9
Fig. Bl.13
50 .....
40 I'..
30
25
"- I"
20 ~
15
~
log (17/7)
"~t'\ n=l+ e
loge (FO. /F0.85)
", "-
10
9
8
7
n
6 "'-.
5
f'...
4
r-... ~
2
"""'"
'" r-...
....
1.5
1.0
1.01 1. 02 1. 03 1.04 1.051.06 1.08 1. 1 1. 15 1.2 1.3 1.4 1.5 1.6 1.8 2.0
FG• 7 /F O. 85
Fig. Bl.14
1.4
1.2
~ 3
1.0
.. ~ ~
0.8 ~~ ~
~
~
0.6
L4
~
~
0.4
V
0.2 III
o
V 0.2 0.4 0.6 0.8 1.0 1.2 1.4 1.6 1.8 2.0
Bl.IO BEHAVIOR OF MATERIALS AND THEIR PROPERTIES
.
Table BI 1 Values of Ftu, F ev, E e, FO 7, FO 85, n , for Various Materials Unde r Room & Elevat e d Temperatures (From Ref 8)
Temp. F tu' Fey' Ee F O. 7 F O. 85
MATERIAL Temp. e,
Exp. OF n
Hr. % ksl ksl 106pSI ksl ksi
STAINLESS STEEL
AISI 301 174 Hard Sheet 1/2 RT 25 125 80 27.0 73 63 6.9
Transverse Compression
Longitudinal Compression 1/2 RT 25 125 43 26.0 28.2 23 5.2
AISI301 1/2 Hard Sheet 1/2 RT 15 150 118 27.0 116.5 105 9.2
Transverse Compression 1/2 400 118 108.5 23.2 108.5 97 8.6
1/2 600 110 107.5 20.9 108.5 96.5 8.2
1/2 1000 86 86 16.2 94.5 83.5 8.0
Longitudinal Compression 1/2 RT 15 150 58 26.0 48 37 4.4
1/2 400 118 53.3 22.4 45.5 36 4.7
1/2 600 110 52.8 20.1 44 31 3.5
1/2 1000 86 45.2 15.6 40 30.5 4.3
AISI 301 3/4 Hard Sheet 1/2 RT 12 175 160 27.0 163.5 151. 5 13.2
Transverse Compression 1/2 400 148 148 24.1 153 142.5 13.2
1/2 600 138 138 22.4 152 140 11. 2
1/2 1000 112 112 18.9 127 121 19.2
Longitudinal Compression 1/2 RT 12 175 76 26.0 70 61. 5 7.6
1/2 400 148 71 23.3 65 56 6.8
1/2 600 138 70.3 21. 6 65.5 56.5 6.8
1/2 1000 112 59.3 18.2 55 46 5.9
AISI 301 Full Hard Sheet 1/2 RT 8 185 179 27.0 183 172 16
Transverse Compression 1/2 400 168 168 25.1 174 164 16
1/2 600 159 159 23.8 172 162 16
1/2 1000 131 130 21. 6 141. 5 135.5 21. 5
Longitudinal Compression 1/2 RT 8 185 85 26.0 77.5 63 5.2
1/2 400 168 80.8 24.2 74 59.5 5
1/2 600 159 79.9 22.9 74 58 4.6
1/2 1000 131 66.3 20.8 58 42.5 3.9
17 -4 PH Bar & Forgings 1/2 RT 6 180 165 27.5 166 160 24
1/2 400 162 135 25.3 137 129 16
1/2 700 146 105.5 23.1 106 97 11
1/2 1000 88 62.6 21. 2 60 52 7.1
17-7 PH (THl050) Sheet, Strip & Plate, 1/2 RT 180 162 29.0 166 145 7.4
t = .010 to .125 In. 1/2 400 169 144 27.8 146 126 6.8
1/2 700 144 118 24.9 117 104 8.4
1/2 1000 88 61. 5 20.3 56 47 6
17-7 PH (RH950) Sheet, Strip & Plate, 1/2 RT 210 205 29.0 208 196 16.4
t = .010 to . 125 in.
19-9DL (AMS 5526) & 19-9DX 1/2 RT 30 95 45 29.0 36.5 32 7.6
(AMS 5538), Sheet, Strip & Plate
19-9DL (AMS 5527) & 19-9DX 1/2 RT 12 125 90 29.0 85 74 7.2
(AMS 5539) Sheet, Strip & Plate
PH15-7Mo (TH1050) Sheet & Strip, 1/2 RT 5 190 170 28.0 171 164 22.5
t = .020 to . 187 In.
PH15-7Mo (RH950) Sheet & Strip, 1/2 RT 4 225 200 28.0 218 189 7.3
t = .020 to • 187 in.
LOW CARBON & ALLOY STEELS
AISI 1023 & 1025 Tube, Sheet & Bar,
Cold Finished RT 22 55 36 29.0 32.7 31. 5 24
AISI 4130 Normalized, to>. 188 In. 1/2 RT 23 90 70 29.0 61. 5 53 6.8
1/2 500 81 61. 5 27.3 55 48 7.3
1/2 800 68 46.2 23.8 40 32.5 5.2
1/2 1000 46 30.8 20.6 28 22 4.7
AISI4130, 4140, 4340 Heat Treated 1/2 RT 23 125 113 29.0 111 102 10.9
1/2 500 113 98.3 27.3 96 88 10.9
1/2 850 88 68.9 23.2 66.5 61. 5 12
1/2 1000 64 49.7 20.6 45.5 41 9.2
AISI4130, 4140, 4340 Heat Treated 1/2 RT 18.5 150 145 29.0 145 140 25
1/2 500 135 126 27.3 126 122 29
1/2 850 105 88.5 23.2 88 83.5 18.5
1/2 1000 76 63.8 20.6 62 57 10.9
AISI4130, 4140, 4340 Heat Treated 1/2 RT 15 180 179 29.0 179 176 50
1/2 500 162 156 27.3 156 153 46
1/2 850 126 109.3 23.2 109.4 105 22
1/2 1000 92 77 20.6 75 68 9.8
AISI 4130, 4140, 4340 Heat Treated 1/2 RT 13.5 200 198 29.0 198 196 90
1/2 500 180 170 27.3 172.5 169 46
1/2 850 140 121 23.2 121. 5 117 25
1/2 1000 104 87.1 20.6 87 83 19
HEAT RESISTANT ALLOYS
A-286 (AMS 5725A) Sheet, Plate 1/2 RT 15 140 95 29.0 93 87 14
& Strip 1/2 600 129 88.4 24.4 87 81 13.5
1/2 1000 115 81. 7 19.8 81 75 12.5
1/2 1400 52 50.3 14.2 50 47 15.3
K-MONEL Sheet, Age Hardened 1/2 RT 15 125 90 26.0 88 82 13.5
MONEL Sheet, Cold Rolled & Annealed 1/2 RT 35 70 28 26.0 20 17 6.4
INCONEL-X 1/2 RT 20 155 105 31. 0 104 100 23.5
1/2 400 152 95.6 28.9 94 89 17
1/2 800 141 90.2 26.4 88.6 84 18.5
1/2 1200 104 83 23.2 82 78.6 21
Bl.ll
Table BI. I Values of Ftu, Fey, E e , FO 7, FO 85, n, for Various Materials Under Room & Elevated Temperatures (From Ref. 6) (Continued)
Temp. F tu' Fey' Ee F O• 7 F O. 85
MATERIAL Exp. Temp. e,
of n
Hr. % ksi ksi 106psi ksi ksi
ALUMINUM ALLOYS
2014-T6 Extrusions 2 RT 7 60 53 10.7 53 50.3 18.5
t,,; 0.499 in. 2 300 51 42.5 10.2 41. 5 40 24
2 450 28 21 9.2 20.5 19.5 25
2 600 10 8.0 7.4 5.5 4.5 5.4
1/2 300 51 43.5 10.2 44.0 42.5 25
1/2 450 31 26 9.2 26 25.2 29
2014-T6 Forgings 2 RT 7 62 52 10.7 52.3 50 20
t ~ 4 in. 2 300 53 41 10.2 40.5 38.5 19
2 450 29 22 9.2 21. 5 20 12.6
2 600 10 7.5 7.4 4.5 3.0 3.2
1/2 300 53 43 10.2 42.5 40 15.8
1/2 450 32 25.5 9.2 25.0 23.5 15.6
2024- T3 Sheet & Plate, 2 RT 12 65 40 10.7 39 36 11.5
Heat Treated, t,,;. 250 in. 2 300 37 10.3 35.7 33.5 15
2 500 26 8.4 24.8 22.8 10.9
2 700 7.5 6.4 6.2 5.5 8.2
2024-T4 Sheet & Plate, 2 RT 12 65 38 10.7 36.7 34.5 15.6
Heat Treated, t ,,; O. 50 in. 2 300 34 10.3 32.5 30.5 14.6
2 500 24 8.4 23 21 10.2
2 700 7 6.4 60 5.7 18.5
2024-T3 Clad Sheet & Plate, 2 RT 12 60 37 10.7 35.7 33 12
Heat Treated, t =. 020 to • 062 in. 2 300 34 10.3 33 30.3 11
2 500 24.5 8.4 22.7 20 7.9
2 700 6.5 6.4 5.8 5.5 18.5
2024-T6 Clad Sheet & Plate, 2 RT 8 62 49 10.7 49 45 11
Heat Treated, t '" O. 063 in. 2 300 45 10.3 44.3 40.7 11
2 500 22 8.4 31. 5 28 8.3
2 700 6 6.4 7.0 6.0 6.6
2024- T6 Clad Sheet & Plate, 2 RT 8 60 47 10.7 47 43 10.6
Heat Treated, t 0< O. 063 in. 2 300 43.2 10.3 42.3 38.7 10.8
2 500 21 8.4 29.5 26 7.8
2 700 6 6.4 5.00 4.0 4.9
2024-T81 Clad Sheet, Heat Treated, 2 RT 5 62 55 10.7 56 51. 6 11.2
to< 0.064 in. 2 300 50.5 10. :i 51. 2 46.5 10
2
2
6061-T6 Sheet, Heat Treated & Aged, 1/2 RT 10 42 35 10.1 35 34 31
to< 0.25 in. 1/2 300 29.5 9.5 29 28 26
1/2 450 20.5 8.5 19.3 17.7 10.9
1/2 600 7.5 7.0
.
6.6 6.2 15.2
7075-T6 Bare Sheet & Plate, 2 RT 7 76 67 10.5 70 63 9.2
t,,;; O. 50 in. 2 300 54 9.4 55.8 52.5 15.6
2 425 25.5 8.1 25.4 23.5 12.1
2 600 8 5.3 7.2 5.2 3.7
1/2 425 30 8.1 34.5 32.5 16
7075- T6 Extrusions, 2 RT 7 75 70 10.5 72 68 16.6
t :,,; O. 25 in. 2 300 54 9.4 58.5 54.5 13.4
2 450 22.5 7.8 21. 3 18.5 7.2
2 600 8 5.3 6.5 4.3 3.2
1/2 450 25 7.8 29 26 8.8
7075-T6 Die Forgings, 2 RT 7 71 58 10.5 58.5 55.1 15.2
t :,,; 2 in. 2 300 47.6 9.4 47.8 45 15.6
2 450 18.5 7.8 17.3 16 12
2 600 7.0 5.3 5.0 3.7 3.9
1/2 450 23 7.8 24 22 10.9
7075- T6 Hand Forgings, 2 RT 4 72 63 10.5 63.8 61. 5 25
Area:"; 16 sq. in. 2 300 51. 6 9.4 52.2 50 21. 5
2 450 20.2 7.8 20.3 19 13.7
2 600 7.6 5.3 6.0 5.0 5.8
1/2 450 24 7.8 26.5 25.3 19.5
7075- T6 Clad Sheet & Plate, 2 RT 8 70 64 10.5 64.5 61. 6 19.5
t:,,; 0.50 in. 2 300 50 9.4 54 51. 7 20
2 450 20.5 7.8 19.7 17.5 4.6
2 600 7.7 5.3 7.7 5.5 3.6
1/2 450 23 7.8 27.2 25.3 12.4
7079-T6 Hand Forgings, 1/2 RT 4 67 59 10.5 59.5 57.5 26
t :,,; 6. 0 in. 1/2 300 47 9.4 46.5 45 29
1/2 450 21 7.8 20 18.5 12
1/2 600 7.0 5.3 5.5 3.5 3.0
MAGNESIUM ALLOYS
AZ61A Extrusions, RT 8 38 14 6.3 12.9 12.3 19
t ,,; O. 249 in.
HK31A-0 Sheet 1/2 RT 12 30 12 6.5 10 8.4 6
t = 0.016 to 0.250 in. 1/2 300 20 11.1 6.16 8.9 6.9 4.5
1/2 500 15 9.3 4.94 7.5 5.6 4.2
1/2 600 10 4.9 3.77 3.3 1.6 2.2
HK31A- H24 Sheet, 1/2 RT 4 34 19 6.5 17.3 14.6 6.2
to< O. 250 in. 1/2 300 22 17.7 6.2 15.6 12.6 5.1
1/2 500 17 14.8 4.9 13.1 10.5 4.9
1/2 600 11 7.8 3.8 6.7 5.2 4.5
TITANIUM A LL YS
Ti-8Mn Annealed SMet, 1000 RT 10 120 110 15.5 119.5 102 13.7
Plate & Strip
Ti-6Al-4V Annealed Bar 1/2 RT 10 130 126 16.0 127 124.5 43
& Sheet, t::,,;. 187 in. 1/2 400 105 96 14.1 97 93 22
1/2 600 99 84.5 13.0 85.5 82 22
1/2 800 87 79.4 11.8 80.5 77 21. 5
1/2 1000 70 60.6 7.7 61 59.5 36
Bl.12 BEHAVIOR OF MATERIALS AND THEIR PROPERTIES
Fig. Bl.15 (curves A and B) show the Bl. 15. The General Creep Pattern.
stress-strain curve for a material. Curve (A)
is for a low elevated temperature condition A typical manner of plotting creep-rupture
and curve (B) that of a high elevated test data is illustrated in Fig. 81.16. For
temperature condition. The results were ob- metals tested at high value of stress or
tained by a normal testing machine procedure temperature, three stages in the creep-time
requiring a short time test period, hence the relation can be observed as shown in Fig. Bl.16.
results can be considered as independent of The initial stage, often called the stage of
time. primary creep, inclUdes the elastic deformation
and that region where the rate of creep de-
formation decreases rather rapidly with time,
which no doubt indicates an influence of strain
hardening. The second stage, often referred
to as the secondary creep stage, represents a
stage where the rate of strain has decreased
to a constant value (except for high stress)
for a considerable time period, and this stage
represents the period of minimum creep rate.
The third stage, often called the tertiary
creep stage, represents the period where the
reduction in cross-sectional area leads to a
higher stress, a greater creep rate and finally
rupture.
'00
e
,··J·+····I······· ~ 80
j
E 60
~
C 40
~
.r
'i: 20
u"
.r
TI"E~
100
60
subjected to rapid aerodynamic heating. The Fig. Bl. 21 - Tensile yield stress of 2024-T3 aluminum
results of tests indicate that in general alloy for temperature rates from O. 20 F to lOOoF per
metals can withstand substantially higher second and of stress-strain tests for 1/2-hour exposure.
Bl.14 BEHAVIOR OF MATERIALS AND THEIR PROPERTIES
80
0
70 ~,
r~\~
I i
I i , •
·
.: 60
30
I
\\ •
~
i
i I "
I \
\ ..
0
\\ ~ NCO
",,~ l2i;;;~
-]00
~emp.roture rote,
""-
Fig. Bl.22 - Tensile rupture stress of 2024-T3 aluminum Fig. 1. 23 (Ref. 3) - Effect of Temperature on Ultimate
alloy for temperature rates from O. 20 F to lOOoF per second Strength (Ftu) of 2014-T 6 Aluminum Alloy.
and ultimate tensile stress of stress-strain tests for 1/2-
hour exposure.
vehicles. Fatigue failure is failure due to
strength respectively of aluminum alloy as being stressed a number of times. For example,
compared to values when loaded after the a beam may be designed to safely and efficient-
material has been exposed 1/2 hour at constant ly carry a design static load and it will carry
temperature. this static load indefinitely without failure.
However, if this load is repeated a large
Bl. 19 General Effect of Low Temperatures Upon Material enough number of times, it will fail under this
Properties. static design load. The higher the beam stress
under the static design load, the less the
The development of the missile and the number of repeated loadings to cause failure.
space vehicle brought another factor into the
ever increasing number of environmental con- To date no adequate theory has been
ditions that effect structural design, namely, developed to clearly explain the fatigue
extremely low temperatures. For example, in failure of materials. Fatigue failure appears
space the shady side of the flight vehicle is to begin with a crack starting at a point of
subjected to very low temperatures. Missiles weakness in the material and progressing along
carry fuels and oxidizers such as liquid crystal boundaries. A microscopic examination
hydrogen and oxygen which boil at -423 and of metals indicates there are many small
-297 oF respectively. In general, low cracks scattered throughout a material. Under
temperatures increase the strength and stiff- the action of repeated stress these small
ness of materials. This effect tends to cracks open and close during the stress cycle.
decrease the ductility of the material or, in The cracks cause higher stress to exist at
other words, produce brittleness, a property the base of the crack as compared to the
that is not desirable in structures because stress if there were no crack. Under this
of the possibility of a catastrophiC failure. repeated concentration of stress, the cracks
In general, the hexagonal closely packed will gradually extend across the section of
crystalline structures are best suited for the member and finally causing complete
giving the best service under low temperatures. failure of the member.
The most important of such materials are
aluminum, titanium, and nickel-base alloys. Fatigue testing consists of 3 types:-
Fig. Bl.23 shows the effect of both elevated (1) the testing of material crystals, (2) the
and low temperatures on the Ultimate tensile testing of small structural test specimens,
strength of 20l4-T6 aluminum alloy under and (3) the testing of complete composite
various exposure times. structures. A tremendous amount of test
information is available for the second type
Bl.20 Fatigue of Materials. of testing. More and more attention is being
given to the third type of testing. For
Designing structures to provide safety example, a complete airplane Wing or fuselage
against what is called fatigue failure is one is often subjected to elaborate fatigue
of the most important and difficult problems testing in order to insure the safe design
facing the structural designer of flight life of the airplane.
Bl.15
The strength of ferrous metals under There are in general two types of tests to
repeated stresses is often referred to as the determine the behavior of materials under
endurance or fatigue limit. The endurance impact loads. The usual impact test which has
limit stress is the stress that can be repeated been conducted for many years is referred to as
an infinite number of times without causing the notched bar test and consists of subjecting
fracture of the material. Non-ferrous notched specimens to aXial, bending and torsional
materialsjsuch as the aluminum alloys do not loads by the well known Charpy or Izod impact
have an endurance limit as defined above but testing machines. In both of these machines
continue to weaken as the stress cycles are an impact load is applied to the specimen by
increased. Due to this fact and also since swinging a weight W from a certain vertical
the required service life of structures and height (h) to strike and rupture the notched
machines vary greatly, it is customary to specimen and then stopping at a vertical
refer to the strength under repeated stresses height (h'). The energy expended in rupturing
as endurance or fatigue strength instead of the specimen is then equal approximately to
endurance limit. Thus the fatigue strength (Wh-Wh'). This type of test is primarily used
is the maximum stress that can be repeated for studying the influence of metallurgical
for a specified number of cycles Without variables.
producing failure of the structural unit.
The other type of impact testing is made
The results of testing a specimen under on unnotched specimens and the general purpose
repeated stresses such as tension, compression, is to obtain the stress-strain diagram of
bending, etc., is often plotted in a form materials under impact load or the load-
which is referred to as the S-N (stress versus distortion diagram of a structural member or
cycles) diagram, as illustrated in Fig. Bl.24. composite structure as the unit is completely
fractured under an impact load.
~ ,, Q Q
n
of Materials.
10 6 10 6 /00
i I
I
Number of Cycles for Failure, logN- I v- 64 fljsec i
Fig. Bl. 24 (' v v "'-'Stalic
: !
The problem of fatigue design of aircraft V- I !
airframes is covered in Chapter C13 of this I I
book. I
-Q 30
1/
-- l- V ~ 60 ft/sec Table B1. 2
::::- -----...
Proc. A.S. T. M. 1938, Vol. 38.
0- 160
~ Stalic~
'\ I'---- REFERENCES
~
a
§120
I v = 15 II/sec 1""- r- Ref. 1. Ramberg & Osgood. Description of
«i ,
V) I Stress-Strain Curves of 3 Parameters.
~ 80
V)
NACA, Tech. Note 902.
Ref. 2. Time and Temperature Gremlins of
40 Destruction. By L. A. Yerkorvich,
Cornell Aero. Lab. Research Trends.
02 .04 .06 .08 10 ./2 .14
Sept. 1956.
0
Unit elongation, /n.Jin. Ref. 3. Military Handbook (MIL-HDBK-5 )
Aug. 1962.
Ref. 4. NACA Technical Note 3462.
Fig. B1.27 - Stress-strain curves, SAE 6140, drawn 1020o F. Ref. 5. NACA Technical Note 868.
Ref. 6. From Structures Manual, Convair
Astronautics .
.
CHAPTER B2
MECHANICAL AND PHYSICAL PROPERTIES OF METALLIC
MATERIALS FOR FLIGHT VEHICLE STRUCTURES
It would require several hundred pages to list the properties of the many
General Explanation.
materials used in flight vehicle structural design. The metallic materials presented in this
chapter are those most widely used and should be sUfficient for the use of the student in his
structural analysis and design problems. All Tables and Charts in this chapter are taken from
the government publication "Military Handbook, MIL-HDBK-5, August, 1962. Metallic Materials and
Elements for Flight Vehicle Structures". This publication is for sale by the Supt. of Public
Doc~~ents, Washington 25, D.C. The properties given in the various tables are for a static
loading condition under room temperature. The effect of temperature upon the mechanical
properties is given in the various graphs.
Alloy ..................... AISI 4130, 8630, AISI 4130, 4140, 4340, I 4140 AISI
and 8735 8630, 8735, and 8740 4340 4340
8740
I
Form ..................... Sheet, strip, All wrought forms
plate, tubing
Mechanical properties:
F,u, ksi ............... 95 90 125 150 180 200 260
F,y, ksi ............... 75 70 103 132 163 176 217
Fey, ksi ............... 75 70 113 145 179 198 242
F. u, ksi. .............. 55 55 82 95 109 119 149
Fb,u, ksi:
(eID=1.5) . ....... ....... . . ....... 194 219 250 272 347
(eID=2.0) ........ 140 140 251 287 326 355 440
F bry , ksi
(eID=1.5) . ....... ....... . . ....... 151 189 230 255 312
(eID=2.0) . ....... .. . .... . . ....... 180 218 256 280 346
e, percent ............. bL 10
See table See table 2.3.1.1(c)
2.3.1.1 (b) bT 3
I I
E, 10 6 psi. ............ 29.0
E c' 10 6 psi ............ 29.0
G, 10 6 psi. ............ 11. 0
Physical properties:
w, Ib/in. 3 . . . . . . . . . 0.283.
C, Btu/(1b) (F) ........ 0.114 (at 32° F).
K, Btu/[(hr) (ft2) (F) 1ft] . 22.0 (at 32° F).
a, 10- 6 in./in./F ....... 6.3 (0° to 200° F).
82.1
B2. 2 MECHANICAL AND PHYSICAL PROPERTIES OF METALLIC MATERIALS FOR FLIGHT VEHICLE STRUCTURES
AlSl ALLOY STEELS (Cont.)
100
e:>
g
e 80
Q.
...
Q)
Strength at temperature
Exposure up to 1/2 hr.
! 60
E
Q)
j
I- 120 ;; 40
E .P
o
o c 20
a: ~
~
C 100 200 400 600 800
u
W Temperature, F
"0 Fig. 82. 2. Effect of temperature on the compressive
c:
0 E yield strength (Fey) of heat-treated AISI alloy steels.
w
80
::>
u:-
,.,
e
i!
-
LL
c
Q)
60
Fft Ftu i
§
0
...
Q)
;;
Il. u.1-
~
40 .r
Temperature, F
o
-400 o 400 800 1200
Temperature, F
Fig. 82.1. Effect of temperature on Ftu> Fty, and E of
AISI alloy steel
i§
80
60
;; 40
.i
~
20
Temperature, F
E
00
~
1
:
TUle Ba.2 ~Ip .........tca1lU1d PbJa1cal Propert1ea of ICr-IIo-Y Aircraft Steel ..
i
"
..i i
Alloy..................... 5Cr-Mo-V aircraft steel. ..I
Form .................... All wrought forms.
Condition ................. Heat treated to obtain the Flu indicated.
Up to 12 in. diam. or equivalent. 1'Ig. B2. 8 Effect of temperature OIl
Section size ..... , .........
)
the tensile and compressive modulus
(E and Ed of 5 Cr-Mo-V aircraft
Basis ..................... (0) (0) (0) IIteela.
Mechanical properties:
FlU, ksi. .............. 240 260 280
FlU, ksi. .............. 200 220 240
Feu, ksi. .............. 220 240 260
F. u, ksi ............... 145 155 170
F bru , ksi:
(ejD=1.5) . ....... ............. . ............. . . ....... ..... _
Physical properties:
1&>, Ib/in. 3 • • • • • . • • • . • . • • 0.281. J'Ig. 81. 10 Efteet 01 te. . .rature OIl
C, Btu/(lb)(F) ...... , .. O.l1(e) (32 0 F). the ultimate sbea'r IItrea&th (Pau) of
K, Btu/[(hr)(ft2)(F)/ft]. 16.6 (400° to 1,100° F). 5 Cr-IIo-Y aircraft...... .
a, 10-6 in./in./F ........ 7.1 (80° to 800° F); 7.4 (80°-1,200° F).
i 40
i
~ 20
I °o
Fig. B2. 6 Effect of temperature OIl Fig. B2. 7 Effect of temperature OIl PIg. B2. 12 Effect of temperature OD
the ultimate tensile strength (Foo) of the tensile yield strength (Fty) of 5 the bearing yield strength (Fbry) of 5
Cr-Mo-V aircraft steels. Cr-Mo-V aircraft steels. Cr-Mo-V aircraft steels.
B2.4 MECHANICAL AND PHYSICAL PROPERTIES OF METALLIC MATERIALS FOR FLIGHT VEHICLE STRUCTURES
Mechanical properties:
F 'u, ksi. .............. 180 210 200 170 185
F tv ' ksi. .............. 150 190 180 140 150
F ev , ksi ............... 158 200 189 147 158
F. u , ksi ............... 117 136 130 111 120
F bru , ksi:
(eID=1.5) . ....... 2'97 346 330 280 305
(eID=2.0) ........ 360 420 400 340 370
F brv , ksi:
(eID=1.5) . ....... 225 285 270 210 225
(eID=2.0) . ..... " 247 313 297 231 247
e, percent ............. See table 2.7.2.1(b) 6 6
Physical properties:
w, Ib/in. 3 • • • • • . . • • . . • • • 0.276.
C, Btu/(lb)(F) ......... 0.11. (d)
K, Btu/[(hr)(ft2)(F)/ft] .. 9.75 (at 300 0 F).
a, 10- 6 in./in./F . 6.3 (70 0 to 600 0 F) for TH 1050.
6.8 (70 0 to 600 0 F) for RH 950.
• Test directiolIongitudinal for widths less than 9 in.; transverse for short transverse (thickness) direction.
widths 9 in. and over. c Vendors guaranteed minimums for Ftu., Fly, and e.
• Test direction longitudinal; these properties not applicable to the d Calculated value.
Fig. B2. 13. Effect of temperature on the ultimate Fig. B2. 14. Effect of temperature on the tensile yield
tensile strength (Fro) of 17-7 PH (THI050) stainless steel. strength (Fty) of 17-7 PH (THI050) stainless steel.
B2.5
IJ
-
..."
...
200 400 600 800 1000 1200 1400 1600 200 400 600 800 1000 1200 1400 1600
TemperolIn, F Tempe..." ... F
Fig. B2.15. Effect of temperature on the compressive Fig. B2. 18. Effect of temperature on the ultimate
yield strength (Fey) of 17-7 PH (TH1050) stainless steel. bearing strength (Fbru) of 17-7 PH (TH1050) stainless steel.
100
I
j
1;
.:
Fig. B2.16. Effect of temperature on the ultimate Fig. B2.19. Effect of temperature on the bearing yield
shear strength (Fsu) of 17-7 PH (TH1050) stainless steel. strength (Fbry) of 17-7 PH {TH1050) stainless steel.
Mechanical properties:
Flu. ksi ............... 190 190 ~ 20
F l.. ksi ............... 170 170 !
Feu. ksi. .............. 178 178
Temperature, F
FlU. ksi. .............. 123 123
F bru • ksi: Fig. B2. 24 Effect of temperature on
(e/D=1.5) ..... ... 313 313 the ultimate shear strength (F su ) of
(e/D=2.0) ...... .. 17-4 PH (H900) stainless steel.
380 380
F bru • ksi:
(e/D=1.5) ...... .. 255 255
(e/D=2.0) ..... ... 280 280
e, percent:
In 2 in............ 10 ...
In 4 D ............ ... 10 1; 40
...1.
~ 20
E, 10 6 pSl . . . . . . . . . . . . • 29.0 ! 0
o 200 400 600 BOO 1000 1200 1400 1600
i 80
! 60
J
1; 40
...2
~ ~
20
20
~ ~ 0
o 200 400 600 800
Temperature. F
Fig. B2.21 Effect of temperature on Fig. B2. 22 Effect of temperature on Fig. B2. 27 Effect of temperature on
the ultimate tensile strength (Ftu) of the tensile yield strength (Fty) of the tensile and compressive modulus
17-4 PH (H900) stainless steel. 17-4 PH (H900) stainless steel. (E and Ed of 17-4 PH (H900)
stainless steel.
B2.7
Table B2. 5 Design Mechanical and Physical Properties of AM-350 Stainless Steel
Physical properties:
w, Ibjin. 3 . . . . . . . . . . . . . . . 0.282.
C, Btuj(lb)(F) ......... 0.12 (32 0 to 212 0 F).
K, Btuj[ (hr)(ft2)(F) jft] . .. 8.4 (at 100 0 F); 11.7 (at 800 0 F).
a, 10-6 in.jin.jF ......... 6.3 (70 0 to 212 0 F); 7.2 (70 0 to 932 0 F).
• Test direction longitudinal for widths less than 9 in.; transverse for widths 9 in. and over.
100
~
e
!
j
80
I
j
0 40 0
"" ~-
~ 5
~ .r
Temperature, F Temperature, F Temperature, F
Fig. B2. 28 Effect of temperature on Fig. B2. 29 Effect of temperature on Fig. B2. 33 Effect of temperature on
the ultimate tensile strength (Ftu) of the tensile yield strength (Fty) of the tensile yield strength (Fty) of
AM-350 stainless steel (double-aged). AM-350 stainless steel (double-aged). AM-350 stainless steel (SCT).
B2.8 MECHANICAL AND PHYSICAL PROPERTIES OF METALLIC MATERIALS FOR FLIGHT VEHICLE STRUCTURES
100 100
~
!
.=e
e 80
~
80
"~
0!!- 0!!-
60
J
0
60
J
0
40 40
.;t
.P
'20
20
'" ~
~ cf
.f 200 400 600 800
Temperature. F
200 400 600 800
Temperature, F
1000 1400
Fig. B2. 34 Effect of temperature on the tensile and Fig. B2. 36 Effect of temperature on the compressive
compressive modulus (E and Ed of AM-350 stainless yield strength (Fey) of AM-350 stainless steel (SCT).
steel (SCT).
100 !OO
!~
!
e
I~ 80
"~
0!!-
80
g
a:
60
J
60
0 40 0 40
... 1;
.f
~ ~
20 20
~ cf
200 400 600 800 1000 1200 1400 1600
Temperature. F Temperature, F
Fig. B2.35 Effect of temperature on the bearing Fig. B2. 37 Effect of temperature on the ultimate
yield strength (Fbry) of AM-350 stainless steel bearing strength (Fbru) of AM-350 stainless steel
(SCT). (SeT).
100
i §
80
60
0 40
,}
~
u
20
cf
Temperature, F
Table B2. 6 Design Mechanical and Physical Properties of AISI 301 Stainless Steel
Basis .................... S S S S S
Mechanical properties:
Flu, ksi:
L ............... 75 125 150 175 185
T ............... 75 125 150 175 185
Flu, ksi:
L ................ 30 75 110 135 140
T ................ 30 75 110 135 140
F cu, ksi:
L ................ 35 43 58 76 85
T ................ 35 80 118 160 179
F,u, ksi. .............. 40 67.5 80 95 100
F bru , ksi:
(eID= 1.5) ........ ......... . ......... . . ......... ......... . . .........
(eID=2.0) . ....... 150 250 300 350 370
F bru , ksi:
(eID=1.5) . ...... . . . . . . . . . . ......... . ......... . . ......... . .........
(eID=2.0) . ...... 50 140 200 240 270
e, percent ............. (c) (c) (c) (c) (c)
E, 10 6 psi:
L ................ 29.0 27.0 26.0 26.0 26.0
T ................ 29.0 28.0 28.0 28.0 28.0
E c, 10 6 psi:
L ................ 28.0 26.0 26.0 26.0 26.0
T ................ 28.0 27.0 27.0 27.0 27.0
G, 10 6 psi. ............ 12.5 12.0 11.5 11.0 11. 0
Physical properties:
w, Ib/in. 3 • . • . . . . . . . . . . . 0.286.
C, Btu/(lb)(F) . ........ 0.108 (at 32 0 F).
K, Btu/[(hr)(ft2)(F) 1ft] . 7.74 (at 32 0 F).
a, 10-6 in./in./F . ....... 9.2 (70 0 to 200 0 F).
~
I
j
80
80
J
j
80
80
1; 40 1; 40
~u
..:-
~ ~
20 20
'00
.r .l:
Temperatur8,F
60
200 400 600 eoo
Temperature, F
H'O
.l: 01 temperature
up 10 thr 1
n +:j ~l
Temperature. F
!f
] 20
.r ! tL 1 '600
'00
Temperolur.,F
Temperature, F
1; 40
u
~ '"~
2 20
80
! 60
~
j .r
1; 40
~. Fig. B2.45 Effect of temperature on
the tensile and compressive modulus
".t.5 20
(E and Eel of AISI 301 (half-hard)
stainless steel.
Temperature, F Temperature,F
iJ
" ;E~posure up 10 t hr
80 •• 1" '
60
1; 40
~~
~
.l:
lJJlJ!\llilliM!\ill eoo 1000 1200 1400 1600
Fig. B2.46 Effect of temperature on Temperolure,F
Temperature, F
the ultimate tensile strength (Ftu) of
Fig. B2.42 Effect of temperature on AISI 301 (full-hard) stainless steel. Fig. B2. 51 Effect of temperature on
the ultimate shear strength (Fsu) of the bearing yield strength (Fbry) of
AISI 301 (half-hard) stainless steel. AISI 301 (full-hard) stainless steel.
'00 '00
j
2 ~ so
i 80
'j"
~
80
I J 80
80
j
80
j
1; 1; 40
40 1; 40
.:t
J; ..:-
~ 20
~ ]
20 20
'"
.t .l: ~
200 400 600 BOO 1000 .l: 200 400 600 eoo 1000 1200 1400 1600
Temperature, F
Temperature, F Temperoture,F
Fig. B2.43 Effect of temperature on Fig. B2.47 Effect of temperature on Fig. B2. 52 Effect of temperature on
the ultimate bearing. strength (Fbru) the tensile yield strength (Fty) of AISI the tensile and compressive modulus
of AISI 301 (half-hard) stainless steel. 301 (full-hard) stainless steel. (E and Ec) of AISI 301 (full-hard)
stainless steel.
B2.. 11
Table B2. 7 Design Mechanical and Physical Properties of 2014 Aluminum Alloy (Sheet and Plate)
Physical properties:
w,lb/in. 3 . 0.101
C, Btu/(lb)(F). ....... 0.23 (at 212°F)
K, Btu/[(hr)(ft')(F)/ft] no (at 77°F)
a, 10-' in./in./F .... 12.5 (68° to 212°F)
B2.12 MECHANICAL AND PHYSICAL PROPERTIES OF METALLIC MATERIALS FOR FLIGHT VEffiCLE STRUCTURES
Table B2. 8 Design Mechanical and Physical Properties of Clad 2014 Aluminum Alloy (Clad Sheet and Plate)
Basis.......................... A B A B A B A B
----
Mechanical properties:
FlU, ksi
L .. ................... 64 64 65 67 65 67 64 65
T ..................... 63 63 64 66 64 66 64 65
ST .... ...............
FIll, ksi
L .. ................... 56 56 58 60 58 60 57 60
T ..................... 55 55 57 59 57 59 57 60
ST .... ...............
F cy, ksi
L .. ................... 56 56 58 60 58 60 59 62
T ..................... 57 57 59 61 59 61 59 62
ST .... ...............
F ou, ksi . .................. 39 39 39 40 39 40 39 40
Fbru, ksi b
(e/D=1.5) ... .......... 96 96 98 101 98 101 96 98
(e/D=2.0) ............. 122 122 124 127 124 127 122 124
FbTII' ksi b
(e/D=1.5) ............. 78 78 81 84 81 84 80 84
(e/D=2.0) ............. 90 90 93 96 93 96 91 96
e, percent
L .... ................. 7 8 6 6
T ..................... 7 8 6 4
E, 10' psi. ................. 10.5
Eel 10' psi. ................ 10.7
G, W. psi. ................. 4.0
B2.13
Table B2.9 Design Mechanical and Physical Properties of 2014 Aluminum Alloy (Extrusions)
Heat treated
and aged
by user"
Thickness, in. b ... ....... .... . 0.125-{).499 0.500-0.749 0.75(}-1.499 1.500-2.999 3.000-4.499 :;;:0.750 0.125-4.499
Basis........................ A B A B A B A B A B A A
------ ------------
Mechanical properties:
Flu, ksi
L . ....... ........... 60 61 64 68 68 73 68 73 68 73 68 60
T .. .......... ....... 60 61 64 67 63 66 61 63 58 61 56 56
FIll' ksi
L ................... 53 57 58 62 60 65 60 65 60 65 58 53
7'........ ........... 53 57 55 59 54 58 52 55 49 53 47 47
Fell' ksi
L . ........ ......... . 55 59 60 64 62 67 62 67 62 67 ·. 53
Too ................. 53 56 58' 62 57 61 57 61 57 61 ·. 48
F,u, ksi .... .............. 35 35 37 39 39 42 39 42 39 42 39 35
Fbru, ksi
(eID=1.5) .. ......... 90 92 96 102 88 95 88 95 88 95 ·. ·.
(eID=2.0) .. " ....... 114 116 122 129 109 117 109 117 109 117 ·. ·.
Fbrll' ksi
(eiD = 1.5) ........... 74 80 81 87 78 85 78 85 78 85 ·. ·.
(eID=2.0) .. ......... 85 91 93 99 84 91 84 91 84 91 ·. ·.
e, percent
L ................. 7 .. 7 .. 7 .. 7 .. 7 .. 6 7
Td ................ 5 .. 5 .. 2 .. 2 .. 1 .. 1 ·.
E, 10' psi ... ............. 10.5
E e, 10' psi ............... 10.7
G, 10' psi . ............... 4.0
B2.14 MECHANICAL AND PHYSICAL PROPERTIES OF METALLIC MATERIALS FOR FLIGHT VEillCLE STRUCTURES
Table B2. 10 Design Mechanical and Physical Properties of 2014 Aluminum Alloy (Forgings)
-T6
Condition .. ...... . ......... -T4
I
Thickness, in ........... ... . <4 inches <6 inches
140
1+ +"-i
20
.,-' ...
.!L= ......
o o 100 200 300 00 500 600 700 lJoo
TEMPERATlJlE, of
Fig. B2. 53 Effect of temperature on the ultimate strength (Ftu) of 2014-T6 aluminum alloy (bare and clad sheet 0.020-0.039
in. thick; bare and clad plate 1. 501-4. 000 in. thick; rolled bar, rod and shapes; hand and die forgings; extruded bar, rod and
shapes 0.125-0.749 in. thick with cross-sectional area < 25 sq. in.).
+- ft
NI.HI-Li+H'·
i1
120 +
I j~ fH'
£11 + tt:j=+~.rOO··j.+·HSTO'H.TH
ltt:Hti' EXP05UO'
AT TEMPERATURE
UP TO 10 000 HO
-!- r ."' I+ J :~ i
L _ -/__)..
r+-,-
·tl-
.,~ '-
.L 1
J
+-
I 1/2 HR
1 10
H=j , t 100
HR
HR
1000 HR ~
10 000 HO .1
.]-
II'
1'j , jo
H,-l-
I t]fi.nil~t
j'Hitrm i .
I
o
1
,·,1
-400
1!!I hlH 1t
-300
iH
-200
t +
-100 o 100 200 300 00 500
t· tlP'
l-T·,-.. '·
+, f
I> 0 700 1O
TEMPERATURE, OF
Fig. 82.54 Effect of temperature on the tensile yield strength (Fty) of 2014-T6 aluminum alloy (bare and clad plate
3.001-4.000 in. thick; rolled bar, rod and shapes; hand and die forgings; extruded bar, rod and shapes O. 125-0.499
in. thick with cross-sectional area < 25 sq. in.).
B2.16 MECHANICAL AND PHYSICAL PROPERTIES OF METALLIC MATERIALS FOR FLIGHT VEHICLE STRUCTURES
100 100
e }
~
~ I eo
'" '~"
J
0
E
0
60
40
~,
~.
~ ~
~ l
200 300 400 500 600 700 800 100 200 300 400 500 600 700 800
Temperature, F 100 •
Temperature. F
e
~
!l
Fig. B2.55 Effect of temperature on E Fig. B2. 58 Effect of temperature on
the compressive yield strength (Fey)
J'" the bearing yield strength (Fbry) of
60
•i eo Temperature, F
j
'"
E
~
Fig. B2. 57 Effect of temperature on
the ultimate bearing strength (Fbru) w
0
of 2014-T6 aluminum alloy (all §
~
w
products except thick extrusions). C 40
~ tJ
l ,:; 20 .
200 400 500 600 700 800 -600 -400 -200
Temperalure,F TemperQI<Jre, F
2024 ALUMINUM ALLOY (BARE SHEET & PLATE, EXTRUSIONS, BAR, ROD & WIRE)
Table B2. 11 Design Mechanical and Physical Properties of 2024 Aluminum Alloy (Sheet and Plate)
2024 ALUMINUM ALLOY (BARE SHEET & PLATE, EXTRUSIONS, BAR, ROD & WIRE) (Cont.)
Table B2. 12 Design Mechanical and Physical Properties of 2024 Aluminum Alloy (Extrusions)
Heat Heat
treated treated
Heat treated by user b cold
worked
and aged
Thickness, in ............... 0.050- 0.250- 0.500- 0.750- 1.500- 3.000:;;: All thick- ::<0.250
0.249 0.499 0.749 1.499 2.999 ness
Mechanical properties:
F,u. ksi
L .... ............. 57 61 60 62 60 62 65 70 70 74 70 74 57 64
T ................. 57 61 60 62 60 62 58 61 54 57 50 53 50 . ........ .
F,u. ksi
L .. ............... 42 47 44 47 44 47 46 54 52 54 52 54 38 56
T ................. 42 46 43 46 42 45 41 44 38 41 36 39 36 . .........
Feu. ksi
L ................. 38 41 39 42 39 42 44 52 50 52 50 52 38 ..........
T ................. 38 41 39 42 39 42 42 48 42 44 42 44 38 . .........
F elD • ksi ... ............. 30 32 32 33 32 33 34 38 38 40 38 40 30 . .........
F bru . ksi
(ejD= 1.5) ......... 85 91 85 91 85 91 85 91 85 91 85 91 85 ..........
(ejD=2.0) . ........ 108 114 108 114 108 114 108 114 108 114 108 114 108 · . . . . . . . . .
F bru . ksi
(ejD= 1.5) ......... 59 66 60 66 60 66 61 66 62 66 62 66 53 ..........
(ejD=2.0) . ........ 67 75 69 75 69 75 71 75 73 75 73 75 61 · . . . . . . . . .
e, percent
L ................. 12 ... . 12 ... . 12 ... . 10 ... . 10 ... . 10 . ... 12 ..........
Ta ................ ... . ... . 6 ... . 6 ... . 5 . ... 2 ... . 2 ... . . ......... · . . . . . . . . .
E, 106 psi ... ........... 10.5
E e • 106 psi . ............ 10.7
G, 106 psi . ............. 4.0
-
B2.18 MECHANICAL AND PHYSICAL PROPERTIES OF METALLIC MATERIALS FOR FLIGHT VEHICLE STRUCTURES
2024 ALUMINUM ALLOY (BARE SHEET & PLATE, EXTRUSIONS, BAR, ROD & WIRE) (Cont.)
Table B2. 13 Design Mechanical and Physical Properties of 2024 Aluminum Alloy (Bar, Rod, and Wire);
Rolled, Drawn or Cold Finished; Rolled Tubing
Form ..................... Bar, rod and wire; rolled, drawn or cold-finished Tubing
Heat Heat
treated treated
Condition ................. -T4 or -T351 Heat treated by cold
user' worked
and aged
Thickness, in ......... .... Up to 1.001- 2.001- 3.001- 4.001- 5.001- 6.001- 0.018 to 0.500 0.018-
1.000 2.000 3.000 4.000 5.000 a 6.000 a 6.500 a 0.500
- - - --- - - - - - - - - - - - -
Basis ..................... A A A A A A A A B A A
---------------------
Mechanical properties:
F tu , ksi
L. ............... 62 62 62 62 62 62 62 64 70 62 68
T .. .............. 61 59 57 55 54 52 ..... . ...... ..... . ....... . . .......
Ft., ksi
L ......... ....... 40 40 40 40 40 40 40 42 46 40 60
T . ............... 40 39 38 37 37 36 ..... . ...... ..... . ....... . . .......
F,., ksi
L ................ 32 32 32 32 32 32 ..... . 42 46 40 ........
T ................ ..... . ..... . ..... . ..... . . . . . . . ..... . ............ ..... . ....... . . .......
F,u, ksi ............... 37 37 37 37 37 37 37 39 42 39 .... . .' ..
F bTu , ksi
(e/D=1.5) ..... .... 93 93 93 93 93 93 ..... . 93 105 96 ........
(e/D=2.0) ......... 118 118 118 118 118 118 ..... . 118 133 122 . .......
'
F bT ., ksi
(e/D=1.5) .. oo 56
••••• 56 56 56 56 56 ..... . 59 64 56 ........
(e/D=2.0) ..... .... 64 64 64 64 64 64 ..... . 67 74 64 ........
e, percent
L .. .............. lOb 10 10 10 10 10 10 10 d 0 . .. .. .. . . . ... . . .
•••••
Physical properties:
w, Ib/in. 3 . . . . . . . . . . . . . . 0.100
C, Btu/Clb) (F) .........
K, Btu/[(hr) (ft 2) (F)/ft] .
6
ai, 10- in./in./F ........
70 (at 77 0 F) .
0.23 (at 212 0 F)
0.010-1 0.063- 0.020-1 0.003-/ 0.250- 0.500- 1.001- 2.001- 0.008- 0.063- 0.250- O. 500- I. 001- 2.001- 0.010- 0.063- 0.010-10.063- <0.0030.020-10.063-10.250-1
Thickness, in. __ . 0.062 0.249 0.062 0.499 0.500' 0.499 1.000' 2.000' 3.000' 0.062 0.249 0.499 I. 000- 2.000' 3.000' 0.062 0.128 0.062 0.249 ;;'0.063 0.062 0.249 0.499 0.500' o
Width, in ..... --____________________ <48 <48 s;:
1-,--.--,---1--.--.--';----.----,-- - - - - - - - - - - - - - - - - - - - - - - -----,-1----1--- 1--.---.----,----,---,-,--,.-- t:::J
Basis . A nAB A BAil A B A BAn A BAn A B A B A A A A A B A B A A A BAn A B A B
-- --------- -------------------- - --- ----- [I)
o
Mechanical prol)('rtil's:
F, •• los; ~
L. m fi~ f>3 fi.' f>3 fifi fi, fi'l fi, fi! fi3 fi5 61 fi5 5~1 62 M 58 5~1 61 6~ 60 5X 5·1 5' 61 61 fi3 60 62 61 67 6, 6U 71 73 70 71 69 70 ~
~
7' 5" fil fi2 r,' fi2 fi.' fifi r" fill Ii' 62 61 60 61 5, 61 51 57 56 58 5U 61 62 60 5X 54 5' 61 61 fi3 60 62 62 6., 66 6, 70 72 70 71 6J 70
F,~. k.'Ji
~
I,. I., 4, 4r, 4; 55 5' 5' fiO 5' fir 46 4~ 42 4fi 40 44 31 35 36 ~ 38 36 36 3fi 3, ~ 3~ 40 47 49 5, 5U 63 65 Ii, 6U 65 66 61 65
7'. :1" 41 4/1 4~ 4.' 50 5/1 52 50 52 40 42 ~ 42 ~ 42 ~ 42 34 35 36 ~ 3S 36 36 36 3, ~ 3., 40 47 4U 51 56 62 61 66 5" 65 fm fil 65
F~", kid
L. 3, 3!1 3, W 4fi 4S 4s 48 4'1 4! 3' 40 36 40 36 40 34 35 36 3, 38 36 36 36 3, ~ ~ 40 47 4U 55 5, 63 65 6, 6U 65 66 61 65 ~
t-<
T. 42 H 13 4:, 51 .51 51 5f> .,1 56 43 45 41 45 10 41 34 35 36 38 as 36 36 36 3, 3, ~ 40 47 49 55 57 65 67 69 71 68 W 67 68 t-<
F•• , k&i.. 3' 3!11O 41 3!/ 41 41 42 41 42 40 41 37 31 34 35 35 3; 37 36 35 3, 3, 3S 36 37 38 39 39 40 41 42 41 42 41 41 Q
F.r •• k.,i d
IHI 1/3 !I.5 9' !15 9!1 101 1Il1 101 101 95 ~ 92 9S 89 93
~
(e/D)=1.5.... 84 87 89 92 93 90 8, 81 S7 92 92 95 90 93 96100 100 103 106 109 105 106 103 105 UJ
114 II~ 121112112.. 12.5127131127131120124116124112118
~
(e/D)=2.0. 100 110 112116 liS 114 110 103 1111 Ilfi 116120 114 liS 12212, 127131 13513!1133135131133
1-'6r". ksi"
(e/D)=1.5... . f>1 f>, f>1 6' ;; SI SI s4 SI S4 61 6,/ 5!1 61 56 62 4S 49 50 53 5.1 50 50 50 52 53 53 56 66 69 7s S3 &j 91 94 97 91 92 90 91 ~
(elD)-2.0... . 73 ,f> 71 ,; SS 93 93 96 93 96 74 7S 6, 74 61 70 51 56 58 61 61 5s 58 5X 59 61 61 64 75 7S IHI 91 101 1114 111,110104 1Of> 102104
e. flt'rC'l'nt "U
T. (et leI 8 9 lOllS (e) 4. (e) 15 . 11 8 (e) 4 (e) 15 8 S 5 5 3. 4 4 ... 4 ... !;
E,lII'psi >-3
1'1i.. 10.5 t"J
~
:r~~i ~I
o
10 o
R" !I.5 1 I 9.5 I 10.0 I 9.5 I 111.0 I 95 1 10 0 I 95
1 10 0 19 51 9.5 I 10 0 H
&;
Pri. 10.7 t:::J
Rl'r.... !I. 7 110.2 I 9.7 I 10.2 1 li.7 I 10.2 I 9., I 111.2 I 9.7 1 1 ~I
10. 2 9. 71 9.7 I 10.2
UJ
~
(;, 10' p.,i ..
~
Physic'ul pron'rlh's:
"', Ih/in'. ... . . . . . . . . . . . O.]()() 0.100
C, Btu/(I6)(F)... . . . . ..... , 0.23 (at 2120 (0') 0.23 (at 212 0 F)
K. Btu/[(hr)(ft ')(F)I. .• " 70 (at ; ; 0 Jo') 86 (at 77 0 F)
a 10-' in.jin.W.·. . ... 12.6 (f>,o to 212 0 F) 12.6 (68 0 to 2120 F)
• For the stress relieved temP<'r-T351 (platt' onl}'), all valtl('s fo1- th,· -T4 t"mp"r apply with tIl(' inasmuch as a round t('5t sI){'cimpn is rpfluin'd for testing. The valu('s given hefe fOf thicknl'ss 0.500
exception of ~' •• which may he som('what lo\\'l'r. inch and grl'atpr hav(' hepn adjustp(l to n'pn'spnt till' U\'l'ragc propl'rtil's across till' whole section,
... Heat treated hy user" ref"rs to all mall' rial solution hl'at tr"att'd hy th" USl'r n'garcll,'ss of the inrll1lling til(' dadding.
prior temper of the material. d S"l' Tahll' 3.1. I. I. I.
, Hpeclflcation minimums for elad mahorial 0.5IH) ind' t hi"k and Il('avil'r an- for th" rOr" mat"rial. • Tilt' ('longation valut's for t'W5(' rolumns an' Counll in TahIl' 3.2.3.0 (p).
...~
<0
B2.20 MECHANICAL AND PHYSICAL PROPERTIES OF METALLIC MATERIALS FOR FLIGHT VEHICLE STRUCTURES
100 100
!
~
~
E
~
~ '0
J 60
J 60
"
.,z "
,,-b
40
~ 20
~
or or
200 300 400 500 600 700 800
Temperature, F Temperature, F
Fig. B2. 60 Effect of temperature on Fig. B2. 64 Effect of exposure at Fig. B2. 68 Effect of temperature on
the ultimate tensile strength (Ftu) of elevated temperatures on the room- the ultimate bearing strength (Fbru)
2024-T3 and 2024-T4 aluminum alloy temperature ultimate tensile strength of clad 2024-T3 and clad 2024-T4
(all products except extrusions). (Ftu) of 2024-T3 and 2024-T4 aluminum aluminum alloy (sheet).
alloy (all products except thick
extrus ions).
t
p
~ eo eo eo
!l E
E E
~ ~ ~
E
J 60
J 60
~
60
"..: 40
"
~
40
"
~i;
40
~ 20
~ 20
~
20
or or .r
200 300 400 500 600 700 800 200 300 400 500 600 700 800 100 200 300 400 500 600 700 800
Temperature, F Temperature, F Temperoture,F
Fig. B2.61 Effect of temperature on Fig. B2. 65 Effect of temperature on Fig. B2. 69 Effect of temperature on
the ultimate tensile strength (Ftu) of the compressive yield strength (Fcy) the bearing yield strength (Fbry) of
2024-T3 and 2024-T4 aluminum alloy of clad 2024-T3 and clad 2024-T4 clad 2024-T3 and clas 2024-T4
(extrusions). aluminum alloy (sheet). aluminum alloy (sheet).
i eo
~
~
E 60
~
"
Fig. B2. 62 Effect of temperature on Fig. B2. 66 Effect of exposure at Fig. B2. 70 Effect of temperature on
the tensile yield strength (Fty) of elevated temperatures on the room- the tensile and compressive modulus
2024-T3 and 2024-T4 aluminum alloy temperature tensile yield strength (E and Eel of 2024 aluminum alloy.
(all products except extrusions). (Fty) of 2024-T3 and 2024-T4
aluminum alloy (all products except
thick extrusions).
100
eo
20
100 200 300 400 500 600 700 800 100 200 300 400 500 600 700 800 100 200 300 400 500 600 700 800
Temperature, F Temperature, F Temperature. F
Fig. B2. 63 Effect of temperature on Fig. B2. 67 Effect of temperature on Fig. B2. 71 Effect of temperature on
the tensile yield strength (Fty) of the ultimate shear strength (F su ) of the elongation of 2024-T3 and 2024-T4
2024-T3 and 2024-T4 aluminum alloy clad 2024-T3 and clad 2024-T4 aluminum alloy (all products except
(extrusions). aluminum alloy (sheet). thick extrusions).
Table B2. 15 Design Mechanical and Physical Properties of 7075 Aluminum Alloy (Sheet and Plate)
Fbry, ksi b ~
(eID=1.5) ........... 92 97 94 98 87 90 90 94 88 92 81 84 78 81 75 73 t::J
(eID=2.0) ........... 106 110 107 112 100 104 104 108 102 106 93 97 90 93 87 84 OJ
::r::
e, percent tel
L .... ............... . 7 · 8 ·. 8 ·. 6 ·. 5 ·. 5 ·. 5 ·. 5 .. ~
T ................... 7 ·. 8 ·. 8 ·. 6 ·. 4 ·. 3 ·. 3 ·. 3 2 Ro
ST .. ................ ·. ·. ·. ·. ·. ·. ·. ·. ·. ·. 1 ·. 1 ·. 1 ..
E, 10' psi . ..............
E e, 10' psi ........ .......
10.3
10.5
S
tel
0, 10' psi................ 3.9
Physical properties:
w, Ib/in. 3 . . • . • . • • • • . . . • • . 0.101
C, Btu/(lb)(F) . .......... 0.23 (at 212°F)
K, Btu/[(hr)(ftt)(F)/ftj ... 76 (at 77°F)
lX, 10-' in./in./F . ......... 12.9 (68° to 212°F) .,
G For the .tress relieved temper -T661. all values for the -T6 temper apply with 2.001-2.600 L 62
the ex~eption of F ••• Applicable F •• values are B8 folio..... : 2.601-3.000 L 60
Tkic1me.. (in.) Direeno.. 0/ teat Fell (A "alue.) b See Table 3.1.1.1.1.
0.260-2.000 L 66
...
...
k,
tl:J
~
N
N
Table B2. 16 Design Mechanical and Physical Properties of Clad 7075 Aluminum Alloy (Sheet and Plate)
s::ttl
(')
.~
(eID=1.5) . ............... 101 107 97 102 97 102 97 102 97 102 97 101 94 97 94 97
(eID=2.0) . ............... 125 131 130 136 130 136 130 136 130 136 122 126 117 125 117 12 5
F bru , ksi >xj
(eID=1.5) . ............... 91 96 80 85 79 84 79 84 79 84 78 81 77 80 75 78 @
(eID=2.0) . ............... 98 104 102 108 101 106 101 106 101 106 92 96 91 95 88 92 Q
H
~
e, percent
L ... ..................... 7 8 7 8 7 8 7 8 7 8 7 8 6 ..... . 6 ..... ~
~
....""
B2.24 MECHANICAL AND PHYSICAL PROPERTIES OF METALLIC MATERIALS FOR FLIGHT VEHICLE STRUCTURES
Table B2. 18 Design Mechanical and Physical Properties of 7075 Aluminum Alloy
(Hand Forgings and Die Forgings)
Cross-sectional area, in. t . . . <16 >16, <36 >36, <144 <16 >16, <36 >36, <144
Basis .................... A A A A A A A
Mechanical properties:
F,u, ksi
L .. ............. 75 73 71 75 73 71 75
T ............... 75 71 69 73 71 69 71
ST .............. 72 68 66 70 68 66 ·.
F'll, ksi
L . .............. 64 61 60 63 60 59 65
T ............... 63 60 58 61 59 57 62
ST . ............. 63 60 58 61 59 57 ·.
Fell' ksi
L .. ............. 64 61 60 63 60 59 65
T . .............. 63 60 58 61 59 57 58
ST . ............. .. .. .. .. .. .. · .
F..., ksi .............. 45 44 43 45 44 43 45
Fbru, ksi
(e/D=1.5) ....... 97 95 85 97 95 85 ·.
(e/D=2.0) ....... 135 124 114 135 124 114 ·.
Fbrt/,ksi
(e/D=1.5) ....... 90 79 78 88 78 77 ·.
(e/D=2.0) ....... 96 91 90 94 90 88 ·.
e, percent
L .. ............. 9 7 4 9 7 4 7
T ............... 4 3 2 4 3 2 3
ST . ............. 2 2 1 2 2 1 ·.
E, 10- psi ............ 10.3
E e, 10- psi . .......... 10.5
G, 10- psi . ........... 3.9
B2.25
Table B2. 19 Design Mechanical and Physical Properties of 7075 Aluminum Alloy
(Bar, Rod, Wire and Shapes; Rolled, Drawn or Cold-Finished)
Form ................. Bar, rod, wire and shapes, rolled, drawn or cold-finished
Basis ................. A A A A
Mechanical properties:
Flu, ksi:
L ... ......... 77 77 77 77
LT ........... 77 75 72 69
Ft., ksi:
L ............ 66 66 66 66
LT ........... 66 66 63 60
Fe., ksi:
L ... ......... 64 64 64 64
LT ........... ........... . ........... . ........... . . ...........
F ou , ksi. ......... 46 46 46 46
F bru , ksi:
(e{D=1.5) .... 100 100 100 100
(e{D=2.0) .... 123 123 123 123
F br ., ksi:
(e{D=1.5) .... 86 86 86 86
(e{D=2.0) .... 92 92 92 92
e, percent:
L ... ........ b7 7 7 7
LT .......... 4 3 2 1
'00
1
J
80
60
.
'6 40
~
20
,t
100 200 300 400 500
'00 Temperolure,F '00
iJ 80
60
Fig. 82. 75 Effect of exposure at
elevated temperatures on the room-
temperature tensile yield strength
'6 40 (Fty) of 7075-T6 aluminum alloy
.t (all products).
5 20
,t
100 200 300 400 500 600 100 800
Tempera'ure, F Tempwah..... F
'00
Fig. B2. 72 Effect of temperature on Fig. B2.79 Effect of temperature on
the ultimate tensile strength (Ftu) of the bearing yield strength (Fbry) of
7075-T6 aluminum alloy (all products). 7075-T6 aluminum alloy (all products).
11 40
..t
~ 20
~
s '00
of
"
120
,t 0
0 200 300 400 !lOO 700 800 -400 -200 0 200 400 600 800 1000
'00
Temperoture,F Temperature. F
'00
60
Ptg.- B2~ 80 Effect of temperature Oil
the tensile and compressive modulus
(E and Ec) of 7075-T6 aluminum alloy.
'6
..~
40
~
20
,t
100 200 300 400 500 600 700 800
Temperature, F
100 200 300 400 500 600 700 800 100 200 300 400 500 600 700 800
T.mperat...... F Temperature, F
,00
Fig. B2. 74 Effect of exposure at Fig. B2.81 Effect of exposure at
elevated temperatures on the room- elevated temperatures on the elongation
temperature ultimate tensile strength of 7075-T6 aluminum alloy (all products
(Ftu) of 7075-T6 aluminum alloy (all except thick extrusions).
products).
Thickness (in.) ................ 0.016- 0.061- 0.250- 0.501- 0.016- 0.250- 0.375- 0.501- 1.001- 0.250- 0.375- 0.439- 0.501- 0.751- 1.001- 1.501-
0.060 0.249 0.500 2.000 0.249 0.374 0.500 1.000 2.000 0.374 0.438 0.500 0.750 1.000 1.500 2.000
- - - - - -- -- -- - - - - - - - - - ~
Ba.sis ........................ A B A B B A N
-- - - - - -- - - - - - - - - - - -- -- -- - - -- - CN
f-'
t:.d
Mechanical properties:
F,., kBi.
L .................... 32
T ................... ... .
F,,, k8i.
36 32 36
... . ... . ... .
32 30
..... . ..... .
39
40
29
40
.., .
38
39
26
37
38
24
36
37
22
34
35
20
39
40
27
38
39
26
38
39
26
37
38
25
37
38
23
35
36
22
35
36
21
I
H
~
L .................... 18 19 15 19 15 15 30
~
T ................... ... . ... . ... . ... . ..... . ..... . 32 .... 29 27 25 23 30 29 29 28 26 25 24 L'
L'
Fell' kBi. Q
L .................... 12 13 12 14 10 10 24 25 20 16
13 10 22 21 18 17 16 15 14
Tb ................... ... . .. , . . , .. ... . ..... . ..... . ... . ... . •••• ..... . ..... .
0 • ..... . ..... . ..... . ..... . .. ... ..... . ...... ..... . ~
UJ
F••, ksi . ................. 17 23 17 23 17 ..... . 18 26 18 18 ..... . ..... . 18 18 18 ..... . ...... ..... . ...... :r::
F b,., k8i.
...... ..... . ...... ..... .
~
(e/D=1.5) ............ 50 52 50 52 50 ..... . 58 60 56 54 ..... . ..... . 58 56 56 Be
(e/D=2.0) ............ 60 61 60 61 60 ...... 68 70 65 63 ..... . ..... . 68 65 65 ..... . ..... . •••• ..... .
0 •
F b,.", k8i.
(e/D=1.5) ............ 29 30 29 31 27 ..... . 43 44 38 34 ..... . ..... . 40 39 36 ..... . ..... . ..... . ......
'"~
trl
(e/D=2.0) .......... .. 29 30 29 31 27 ..... . 43 44 38 34 ..... . ..... . 40 39 36 ..... . ..... . ..... . ......
e, percent
L .................... 12 18 12 19 12 10 6 8 8 8 8 8 6 6 6 6 6 6 6
T ................... ... . ... . ... . .... . ..... . ..... . 8 .... 10 10 10 10 8 8 8 8 8 8 8
E, 10° p8i . ............... 6.5
E., 10° p8i ................ 6.5
,
G, 10° psi ................ 2.4
Physical properties:
til, lb/in.' ................. 0.0639
C, BTU/(lb)(F) ........... 0.25 (at 780 F)o
K, BTU/(hr)(ftl)(F)/ft ..... 56 (212 to 5720 F)
4, lO*t in./in./F ........... 14 (65 to 212 0 F)
• Estimated. • Transve1'llll F. r allowables are equal to or greater than the 10ngltudlDai F. r allowables.
~
N
"""
B2.28 MECHANICAL AND PHYSICAL PROPERTIES OF METALLIC MATERIALS FOR FLIGHT VEffiCLE STRUCTURES
100
IE
~
80
0
il
IL
~
20
~
Leoend 100 200 300 400 500 600 700 800
A:Ttnsil~train
B=Compressiye stress-strain Tomperoture, F
C:TensUt tanl;Jent modulus
O=Cor'1)rtllive ton;ent
modulus Fig. 82. lie Effect of temperature on the ultimate shear
strength (Fsu) of AZ31B-H24 magnesium alloy.
4 • 10
Strain,QOOI in./1n. 100
Tanoent Modulus. 10' 1'1;
100
e
~ 80
~ 100 200 300 400 500 600 700 800
!!
Temperature, F
j 60
I
100
80
100 200 300 400 500 600 700 800
j 60 Temperature. F
10
4 I 10
S_,o.OOII~/1n.
Basis ........................... A
0.250
B
0.500 1.000 3.000
A
0.125
B A
0.250
B
0.500 1.000 2.000 3.000 ..........
(e)
I
r:n
H
- -- - §
~
Mechanical properties: t""
t""
F ,a, ksi ...... ............... 30 32 30 30 29 34 36 34 35 34 34 33 33 27 Q
F,~, ksi ...... ............... 18 19 16 15 14 26 28 23 24 21 18 15 15 13
F~, ksi ..................... 12 13 10 10 10 20 21 19 22 18 13 11 11 13 (j;"
F ,a , ksi ........ ............. 22 22 22 22 . ....... 23 23 23 23 23 23 . ....... ....... . ......... . ~
F b• a , ksi
(e/D=1.5) .............. 43 46 43 . ...... . ....... . 49 52 49 51 49 . ....... ....... . ....... . ......... . ..ga
....... . . ....... 57 . .......
Fbm ksi
(e/D=2.0) .............. 51 54 51 57 60 57 58 . ....... . ....... . .........
~
8
(e/D=1.5) .............. 24 25 21 ....... . ....... . 34 35 33 36 31 . ....... . ....... ....... . ......... . tr:l
(e/D=2.0) .............. 24 25 21 . ....... ....... . 34 35 33 36 31 . ....... ....... . ....... . . ......... Be
r:n
e, percent ................... 12 20 12 12 12 4 6 4 8 4 10 10 8 4 ~
E, lOS psi .... ............... 6. 5 o
Es, lOS psi ............... '" 6. 5 55
8
G, lOS psi ............ .... '" 2. 4 H
Physical properties: ~
101, lb./in.3 ••••••••••••••••••• 0.0647
C, BTU/(lb.)(F) ............. 0.25 (32 0 to 212 0 F)
K, BTU/(hr.)(ft.I)(F) ft ...... 60.0 (at 680 F)
a, lO-s in./in./F ............. 15 (68 0 to 3920 F)
• Properties for sheet and plate are taken parallel to the direction of rolllng. Transverse properties I Mechanical properties are based upon the guaranteed tensile properties from separately-east
are equal to or greater than the longitudinal properties. test bars. The mechanical propertIes of bars cut from castings may he as low as 75 percent of the
• Reference should he made to the specific requirements of the procuring or certificating agency tabulated values.
with regard to the use of the above values In the design of castjngs.
~
~
co
82.30 MECHANICAL AND PHYSICAL PROPERTIES OF METALLIC MATERIALS FOR FLIGHT VEffiCLE STRUCTURES
'00 '00
~
!
80
i. 80
~
0-
E
~ ~
J
15
60 E
8
0:
15
60
40 40
u=-
~ ,.
....
~ 20
u
.
1; 20
:f cf
200 300 400 500 600 700 800 '00 200 300 400 500 600 700 800
Temperature, F Temporalure, F
Fig. B2. 90 Effect of temperature on the ultimate Fig. B2.93 Effect of exposure at elevated
tensile strength (Ftu) of HK31A-H24 magnesium temperatures on the room-temperature tensile
alloy. yield strength (Fty) of HK31A-H24 magnesium
alloy.
'00 '00
!
I~
E
eo
60
i0-
E
{!!
§
0:
80
60
15 40
0 40
,,! ~
1;
u
. 20
u
Ii 20
cf ~
'00 200 300 400 500 600 700 800 '00 200 300 400 500 600 700 800
Temperature. F
Temperature, F
Fig. B2.91 Effect of temperature on the tensile Fig. B2.94 Effect of temperature on the ultimate
yield strength (Fty) of HK31A-H24 magnesium tensile strength (Ftu) of HK31A-T6 magnesium
alloy. alloy (sand casting).
100 '00
! !
2
t 80 .
2
0-
E
eo
E
{!! ~
E 60
§ 60
~
0:
15 15 40
40
,.
....2 'L
1; 20 ~ 20
~ U
~ cf
200 300 400 500 600 700 800 '00 200 300 400 500 600 700 800
Temoeroture, F Temperature, F
Fig. B2. 92 Effect of exposure at elevated Fig. B2.95 Effect of temperature on the tensile
temperatures on the room-temperature ultimate yield strength (Fty) of HK31A-T6 magnesium
tensile strength (F tu ) of HK31A-H24 magnesium alloy (sand casting).
alloy.
B2.31
Table B2. 22 Design Mechanical and Physical Properties of AZ61Aa Magnesium Alloy
(Extrusions and Forgings)
Alloy . AZ61A
Condition . -F
Basis .
Mechanical properties:
F,u, ksi
L . 38 39 36 36 38
T .
F'II' ksi
L . 21 24 16 16 22
T .
Fell' ksi
L . 14 14 11 11 14
T .
F , ksi . 19 19 19
F/wu,ksi
(e/D=1.5) . 45 45 50
(e/D=2.0) . 55 55 60
FIn-t/,ksi
(e/D=1.5) . 28 28 28
(e/D=2.0) . 32 32 32
e, percent . 8 9 7 7 6
E, 10< psi . 6.3
E e, 10< psi . 6.3
G, 10< psi . 2.4
Physical properties:
w, lb/in. ' . 0.0647
C, Btu/(lb)(F) . 0.25 (at 78°F) b
K, Btu/[(hr) (ft 2) (F)/ft) . 46 (212° to 572°F)
ac, 10-< in./in./F . 14 (65° to 212°F)
a ProPertif.S for extruded bars, rods, shaPes, tubes, and forgings are during fabrication.
taken parallel to the direction of extrusion or maximum metal 1I0w b Estimated.
B2.32 MECHANICAL AND PHYSICAL PROPERTIES OF METALLIC MATERIALS FOR FLIGIIT VEffiCLE STRUCTURES
Table B2. 23 Design Mechanical and Physical Properties of AZ80Aa Magnesium Alloy
(Extrusions and Forgings)
Basis..........................
Mechanical properties:
F,u, ksi
L .. ................... 43 43 43 47 48 48 42 42
7'.....................
F'I/' ksi
L ..................... 28 28 28 30 33 33 26 28
T ... ...................
p'el/' ksi
L ..................... 17 17 28 27 18 25
T .....................
J1'.u, ksi . .................. 19 19 19 20 20 20 20 20
Fb...., ksi
(e/D = 1.5) ............. 48 48 48 50
(e/D=2.0) ............. 56 56 56 70
Fbrl/' kb'i
(e/D=1.5) ............. 36 36 36 42
(e/D=2.0) ... .......... 40 40 40
e, percent.................. 9 8 6 4 4 4 5 2
E, 10' psi .... .............. 6.5
E e, 10' psi . ................ 6.5
G, 10' psi... ............... 2.4
B2.33
Alloy . AZ63A
o
Sand and permanent- o 2 4 6
Strain,Oool in./in.
• 10
Thickness (in.) .
L89.~d
A-T.n.ll••Ir...·.lraln
Basis b . B -Compres.lv••Ir••• ·.lraln
C- T.n.lI. laRV.nl modulu.
------------1--- --- --- --- 20 O-C~ve Ianvenl·modulu.
~echanical properties:
F,u, ksi . ............. 24 34 24 34 A
F'll, ksi ............... 10 10 11 16 B
F CI/, ksi . ............. 10 10 11 16
FlU, ksi . ............. 16 17 19
Fbru,ksi
(e/D=1.5) ...... ,. 36 36 50
(e/D=2.0) . .... " 50 50 65
Flny,ksi
(e/D=1.5) ....... 28 32 36
(e/D=2.0) ....... 30 36 45
e, percent ............ 4 7 2 3
E, 10' psi . ........... 6.5
E e, 10' psi ... ......... 2 4 6 6 10
6.5
Strain,Oool in./in.
G, 10' psi . ........... 2.4 TORVenl Modulus, 10' psi
i
~
of 8Mn Titanium Alloy
i c.o
lI>-
! 80
i!::
Alloy \ 8Mn
j J 60
t:I:J
(')
t;
~
o
,!
40 ~
(')
~
Form 1 Sheet, plate, and strip >-
~
~ 20 t"
~'tl
~ ,f
400 600 800 1000 '200 '400 1600 600 800 '000 '200 1400 1600
Condition. Annealed 200
Temoerature. F Temperature, F
iJ ~
~
Mechanical properties: 2 H ~
~
60 § l;;j
Flu, ksi ~ :J>
00
L . 120 t""' 0
7' . 120 60 j t""' 'oj
~
200 400 .. 600 800 1000 200 400 600 800 '000 '200 '400 '600
T .
~...,
110 Tempera'ure; f Temperature, F
>-3
F BU, ksi . 84 t:I:J
~t"
Fig. B2.99 Effect of temperature on the tensile Fig. 2. 102 Effect of temperature on the ultimate t<:l
Fbru, ksi yield strength (Fty) of 8 Mn annealed titanium bearing strength (Fbru) of 8 Mn annealed titanium
(e/D=1.5) . 170 Be
alloy. alloy. I:/)
00
(e/D=2.0). >-'3 'oj
Fbry, k.~i ::0
H @
(e/D=1.5) . 130 '"U 'oj
f
(e/D=2.0) .
If
100
t
100
...0
t"
e, percent . 10
E, 10' psi . 15.5 80 ~ 80
~
E
E e, 10' psi . 16.0
F
8 60 <
G, 10' psi .
j 60
It:
;;
t:I:J
...IJ::
'""
(')
t; 40 40 t"
,.. g t:I:J
IL"
Alloy . 6AI-4V
Condition . Annealed
Basil!
--_. __.--------- ----- ------
. A A Fig. B2. 104 Effect of temperature on Ftu, Fty and E of
6Al-4V annealed titanium alloy (sheet and bar)
Mechanical propertielS:
F,u, hi
L . 130 130
T .. 130
1"/1, ksi
L 00', 120 120
T . 120
[<'Cy, hi
L . ]211 126
T . 126
[< , ksi . 80 76
1'" ksi
(eID=1.5) . 196 191 Fig. B2. 105 Effect of temperature on the compressive
(eID=2.0) . yield strength (Fey) of 6Al-4V annealed titanium alloy
248 244 (sheet and bar).
1'",.", ksi
(eID=1.5) . 174 163 100
(eiD =2.0) . 205 198
e, percent . 10 10
E, 10- psi
L 00 .. ]6.0 15.4
T . 16.4
E c, 10- psi
L .. 16.4 16.0
T . 16.9
G, 10- psi . 6.2
Alloy. . . . . . . . . . . . . . . . . . . .. Inconel X
Form . Sheet
Basis .
:\Iechanical properties:
Flu, ksi
L . 155
Temperature,F
T . 155
Fly, ksi Fig. B2. 109 Effect of temperature on
the tensile yield strength (Fty) of
L . 100
precipitation heat treated Inconel X
T . 100 nickel alloy.
Fey, ksi
L .. 105
T .. '" 105
F. u , ksi. 108
Fbrl" ksi
(elD=1.5)
(e/D=2.0). 286 .! zo
Fbry, ksi ~ 0
o
(e/D=1.5)
(e/D=2.0) 186 Fig. B2. 110 Effect of temperature on
e, percent . 20 the compressive yield strength (Fey) of
E, 10' psi . 31.0 precipitation heat treated Inconel X
Eel 10' psi . 31.0 nickel alloy.
G, 10' psi .
Phyaical properties:
w, lb/in. 3 .. 0.304
C, Btu/(ib)(F) ... 0.109
K, Btu/[ (hr)(ft 2)(F)/ft] 8.7 (80 0 to 212°F)
(x, 10-' in./in./F . 6.4 (100 0 to 200°F)
1j
r,mperotUAl, F Temperotu.... F
Fig. B2. 112 Effect of temperature on Fig. B2. 113 Effect of temperature on Fig. B2. 114 Effect of temperature on
the tensile modulus (E) of Inconel X the ultimate bearing strength (Fbru) the bearing yield strength (Fbry) of
nickel alloy. of precipitation heat treated Inconel X precipitation heat treated Inconel X
nickel alloy. nickel alloy.
PART C
PRACTICAL STRENGTH ANALYSIS &
DESIGN OF STRUCTURAL COMPONENTS
CHAPTER Cl
COMBINED STRESSES. THEORY OF YIELD AND ULTIMATE FAILURE.
Cl. I Uniform Stress Condition ~l
T~:; ~1
;' /'
, , jected to pure shear stresses of intensity ~,
normal stresses of the same intensity as the o
~zjb:t~z A Po, to
shear stresses are produced on a plane at 45
dz ~LJ---- ~
with the shearing planes.
2 1
L '':xdxdy - -
Cl. 4 Principal Stresses
Fig. Cl.I Fig. Cl.2
For a body SUbjected to any combination of
stresses 3 mutually perpendicular planes can be
Fig. Cl.l shows a circular solid shaft found on which the shear stresses are zero. The
subjected to a torsional moment. The portion normal stresses on these planes of zero shear
(A) of the shaft exerts a shearing stress ~z on stress are referred to as principal stresses.
section (1-1) and portion (B) exerts a resist-
ing shearing stress ~z on section (2-2). Fig.
Cl. 5 Shearing Stresses Resulting From Principal
Cl.2 illustrates a differential cube cut from Stresses.
shaft between sections (1-1) and (2-2). For
equilibrium a resisting couple must exist on In Fig. Cl.5 the differential block is
top and bottom face of cube. Taking moments SUbjected to tensile principal stresses Ox and
about lower left edge of cube: Oz and zero principal stress 0Y' The block is
cut along a diagonal section giVing the free
~x dxdy (dz) - ~z dzdy (dx) =0 body of Fig. Cl.6. The stresses on the diagonal
section have been resolved into stress compon-
hence, ~x = ~z - - - - - - - - - - - (1) ents parallel and normal to the section as shown.
For equilibrium the summation of the stresses
Thus if a shearing unit stress occurs on along the axes (1-1) and (2-2) must equal zero.
one plane at a point in a body, a shearing unit
stress of same intensity eXists on planes at
right angles to the first plane.
Cl. 3 Simple Shear Produces Tensile and Compressive
On dudy - Ox dzdy cos Q - Oz dydx sin Q = 0,
Stresses. whence on = Ox dzdy cos Q + 0z dydx sin Q
dUdy dudy
Fig. Cl.3 shows an elementary block of
unit dimensions SUbjected to pure shearing
Cl.I
C1.2 COMBINED STRESSES. THEORY OF YIELD AND ULTIMATE FAILURE.
z dydx
Fig. Cl. 5
1
/
/
/
n dudy
Ox dzdy
/
Fig. Cl. 7
l'
o z dydx
Fig. Cl. 6
dzdy _ dydx _
But dudy - cos Q and dudy - sin Q, whence
(2a)
The normal stress on at a point is always
less than the maximum principal stress Ox or
Oz at the point.
ZF._. =0 Fig. Cl. 8
ZFx =0
=
hence, 't Oz sin Q cos Q - Ox cos Q sin Q
or, 't =
(Oz - ax) sin Q cos Q an dudy cos
=0 - -
Q + 't dudy sin Q - ax dzdy - 'txzdxdy
- - - - - - - - - - - - - - - -(4)
or, 't =
(l/2)(az - ax) sin 2 Q, where Oz is
maximum principal stress and ax is minimum ZF z = 0
principal stress.
an dudy sin Q - 't dudy cos Q - a z dydx - 'txzdzdy
Since sin 2 Q is maximum when Q 45 a , = =0 - - - - - - - - - - - - - - - - - -(5)
Stated in words, the maximum value of the shear- ~~: :: cos Q and ~~: =sin 9, we obtain:
ing unit stress at a point in a stressed body
is one-half the algebraic differences of the
maximum and minimum principal unit stresses. (an-(Jx) cos Q + ('t - 'txz) sin g =0 - - - -(6)
C1.3
(an - Oz) sin Q - (~ - ~xz) cos Q =0 - - -(7) Cl. 7 Mohr's Circle for Determination of Principal
Stresses.
It is sometimes convenient to solve
The maximum normal stress an will be maximum
when Q equals such angle Q' as to make ~ zero. = graphically for the principal stresses and the
Thus if ~ = =
0 and Q Q' in equations (6) and
maximum shear stress. MOhr's circle furnishes
a graphical solution. (Fig. Cl.9a). In the
(7), we obtain,
Mohr method, two rectangular axes x and z are
chosen to represent the normal and shearing
(an - ax) cos Q' - ~xz sin Q' =0 - - - - -(8) stresses respectively. Taking point 0 as the
origin layoff to scale the normal stresses ax
(an - Oz) sin Q' - ~xz cos Q' =0 - - - - -J9) and Oz equal to OB and OA respectively. If ten-
In equations (8) and (9) an represen~£he sion, they are laid off to right of point 0 and
principal stress. Dividing one equation by to the left if compression. From B the shear
another to eliminate Q' , stress ~xz is laid off parallel to Oz and with
the sense of the shear stress on the face DC of
Fig. Cl.9b, thus locating point C. With point
an - ax = ~xz
whence, E the midpoint of AB as the center and with
~xz an - Oz radius EC describe a circle cutting OB at F and
G. AD will equal BC and will represent the
an" - (ax - Oz) an + 0xOz = ~xz .. , or shear on face AB of Fib. b. It can be proven
that OF and OG are the principal stresses Omax
a
n
= ax + Oz + _ '(ax - Oz)
2 - ·V 2
il ..
+ ~xz -
_ (10) and Omin respectively and EC is the maximum •
shear stress ~max .• The principal stresses occur
on planes that are parallel to CF and eG. (See
In equation (10), tensile normal stress is
plus and compression minus. For maximum an use Figs. c and d). The maximum shear stress occurs
plus sign before radical and minus sign for on two sections parallel to CH and CI where HEI
minimum an. is perpendicular to OB. Ifox should equal zero
then 0 would coincide With A.
To find the plane of the principal stress- z
es, the value of Q' may be solved for from
equations (8) and (9), which gives: Oz °n(max)
~ 1 ~xz
tan 2 Q' =ax2 -~xzOz (ll )
Pl---
or-[]I~ox
~xz-~-
Q' is measured from the plane of the
largest normal stress ax or oz. The direction
(b) Oz ~m1
of rotation of Q' from this plane is best de-
termined by inspection. Thus if only the Oz
shearing stresses ~xz were acting, the maximum
principal stress would be one of the 45 0 planes,
L~xz 0
~~
E
the particular 450 plane being easily deter- (0) I
mined by inspection of the sense of the shear /
stresses. Furthermore if only the largest 61\1..t1-\'30i'l Ox I ;
~
normal stress were acting it would be the maxi- I;
mum principal stress and Q' would equal zero.
Thus if both a and ~ act, the plane of the
principal stress will be between the plane on
(d)
~xz
ZT
~
J;foX; Oz
0x+ Oz
--2-
~x-Oz
H
(a)
which a acts and the 45 0 plane. As stated 2
before a refers to either ax or Oz whichever Oz ax
is the largest. Z Fig. Cl. 9
Maximum Value of Shearing Stress. (~max.)
Cl. 8 Components of stress From Principal stresses by
Mohr's Circle.
The maximum value of ~ from equation (3)
equals, In certain problems the principal stresses
may be known as in Fig. Cl.9 and it is desired
~ax. = (On(max.) - On(min.))/2 - - - - - (12) to find the stress components on other planes
designated by angle Q. In Fig. Cl.ll the axes
Substituting the maximum and minimum values of x and z represent the normal and shear stresses
an from (10) in (12), we obta1n maximum shear- respectively. The principal stresses are laid
ing stress as follows: off to scale on ox giving points D and E respec-
tively. Construct a circle with A the midpoint
~max. =+ of DE and With diameter ED. Draw angle CAB equal
CL4 COMBINED STRESSES. THEORY OF YIELD AND ULTIMATE FAILURE.
Oz
""_+-_ _,,,c • to
Fig. CL 12 ~xz =5000
d
Z
-t--_Ox
z
On on plane (defc) ~
I _I
Fig. CL 13
~-r
~ on plane 0n(min)
= -2070
1 -+--{1,
Z
E
(defc)
x 0 X
SUbstituting values,
On
__ Ox + Oz + .
2 -
From eq. (10),
YI(ox 2- Oz\
-I
2+
~xz
2
Ox"~ ~jDI~'OX'lO~
~xz = 12000
t
Oz = 20000
Fig. CL 14
Cl.5
Fig. Cl.15 shows the graphical solution 0" - (ox + 0y + 0z)O· + (0xOy + 0yOz + 0xoz
using Mohr's circle. From point 0, Ox 10000 =
and 0z =
-20000 are laid off equal to OB and OA -r yz • _Yxy2)0 - (0xOyOz + 2YyzYxzYxy - 0xYyz·
respectively. ~xz equal to 12000 is laid off - 0yYxz • - 0zYxy • =0 - - - - - - - - - - -(14)
parallel to OZ at B locating C. With E the
midpoint of AB as the center of a circle of
radius EC a circle is drawn which cuts the ox
axis at F and D. The maximum normal and shear
stresses are indicated on the figure.
~
ITYX-
/ 'l;
Fig. Cl. 15 / Yxy
/0
Y
/
- -- t ~~
Y Fig. Cl.16
...I?,
.- ...~-1-
~~ Fig. Cl.17 shows the principal stress
X X system which replaces the system of Fig. Cl.16.
E 0 B
It can be shown that the maximum shear stress
max. is one of the following values.
I
I '(max. = +-"2
1
(01 0.)
+ 1
or Y max. = -2 (02 0,,) -- - - - -(15)
r
largest of the shear stresses in equations (15)
On = Ox 2+ °z ± -J (Ox 2 °z + ~xz
•
depends on the rr.agnHude and signs of the
principal stresses, remembering that tension
is plus and compression is minus when making
- 20000 1: ~(10000-~-20000))·+ 12000· the substitution in equations (15).
= 10000 2
°l.
- - 5000 :!: 19200 "0,,
.
Iffi
, I
,
hence, °n(max. ) = -5000 =
19200 -24200 psi
0,,'
-- ,/,- -
I
0.
Cl. 10 Triaxial or Three Dimensional Stresses The strains under combined stresses are
usually expressed as strains in the direction
For bodies which are stressed in three of the principal stresses. Consider a C8,se of
directions, the state of stress can be defined simple tension as illustrated in Fig. Cl.19.
completely by the six stress components as The stress 01 causes a lengthening unit strain
illustrated in Fig. Cl.16. Using the same e in the direction of the stress 01' and a
procedure as was carried out for a two-dimen- shortening unit strain e' in a direction at
sional stress system, it can be shown that there right angles to the stress 01,.
are three principal stresses 01,' 0. and 0e,
whose values are the three roots of ° in the The ratio of e' to e is called Poisson's
following cubic equation. ratio and is usually given the symbol~. ThUS,
~ =e'/e
C1.6 COMBINED STRESSES. THEORY OF YIELD AND ULTIMATE FAILURE.
Yield Factor of Safety. This term is defined incapable of performing its required function.
as the ratio of the yield strength of the Failure may be due to rupture or collapse or
structure to the limit load. due to excessive deflection or distortion.
Ultimate Factor of Safety. This term is Cl. 15 Determination of the Ultimate Strength of a Structural
defined as the ratio of the ultimate strength Member Under a Combined Load System. Stress
of the structure to the limit load. Ratio-Interaction Curve Method.
Yield Load. This term is defined as the limit Since the structural designer of flight
load multiplied by the yield factor of safety. vehicles must insure that the ultimate loads
can be carried by the structure without failure,
Ultimate Load. This term can be defined as the it is necessary that reliable methods be used
limit load multiplied by the ultimate factor to determine the ultimate strength of a
of safety. This resulting load is often structure. Structural theory as developed to
referred to by engineers as the design load, date is in general sufficiently developed to
which is misleading because the flight vehicle accurately determine the ultimate strength of
structure must be designed to satisfy both a structural member under a single type of
yield and ultimate failure and either one may loading, such as axial tension or compression,
be critical. pure bending or pure torsion. However, many
of the members which compose the structure of
Yield Margin of Safety. This term usually a flight vehicle are subjected simultaneously
expressed in percent represents the additional to various combinations of axial, bending and
yield strength of the structure over that torsional load systems and thus a method must
strength required to carry the limit loads. be available to determine the ultimate strength
of a structure under combined load systems.
Yield Margin of Safety - Yield Strength - 1 A strictly theoretical approach appears too
- Limit Load difficult for solution since failure may be
due to overall elastic or inelastic buckling,
Ultimate Margin of Safety. This term usually or the local elastic or inelastic instability.
expressed in percent represents the additional
ultimate strength of the structure over that The most satisfactory method developed to
strength required to carry the ultimate loads. date is the so-called stress ratio, inter-
action curve method, originally developed and
_ Ultimate Strength presented by Shanley. In this method the
Ultimate Margin of Safety - Ultimate Load - 1 stress conditions on the structure are repre-
sented by stress ratios, which can be con-
Cl. 14 Required Strength of Flight Structures. sidered as non-dimentional coefficients
denoting the fraction of the allowable stress
Under Limit Loads:- or strength for the member which can be
The flight vehicle structure shall be developed under the given conditions of com-
designed to have sufficient strength to carry bined loading.
simultaneously the limit loads and other
accompanying environmental phenomena for each For a single simple stress, the stress
design condition without undergoing excessive ratio can be expressed as,
elastic or plastic deformation. Since most
materials have no definite yield stress, it is
common practice to use the unit stress where a R = stress ratio = ~ -(22)
.002 inches per inch permanent set eXists as
the yield strength of the material, and in where f is the applied stress and F the
general this yield strength stress can be used allowable stress. The margin of safety in
as the maximum stress under the limit loads terms of the stress ratio R can be written,
unless definitely otherwise specified.
1
Under Ultimate Loads:- M.S. = R- 1.0 - - - - - - - - - -(23)
The flight vehicle structure shall be
designed to withstand simultaneously the Load ratios can be used instead of stress
ultimate loads and other accompanying environ- ratios and is often more convenient.
mental phenomena without failure. In general
no factor of safety is applied to the environ- For example for axial loading,
mental phenomena but only to the limit loads.
R - P/Pa , where P = applied axial load
Failure of a Structure:- and Pa the allowable load.
This term in general refers to a state or
condition of the structure which renders it
Cl. 8 COMBINED STRESSES. THEORY OF YIELD AND ULTIMATE FAILURE.
R = T/Ta , where T is applied torsional The reader may review the explanation and
moment and Ta the allowable torsional moment. derivation of these 6 theories by referring
to such books as listed at the end of this
For combined loadings the general chapter.
conditions for failure are expressed by
Shanley as follows:- Test results indicate that the yield
strength at a point in a stressed structure
R~ + R~ + R~ + ------ = 1.0 - -(24) is more accurately defined by theories 5 and
6 followed in turn by theory 2. Since
In this above expression, R~, R 2 and R3 theories 5 and 6 give the same result, they
could refer to compression, bending and shear might be considered as the same general theory.
and the exponents x, y, and z give the In this chapter we will only give the resulting
relationship for combined stresses. The equations as derived by theory 6, since
equation states that the failure of a theories 5 and 6 appear to be the theories
structural member under a combined loading will used in flight vehicle structural design.
result only when the sum of the stress ratios
is equal to or greater than 1.0. Cl. 17 The Octahedral Shear Stress Theory.
For some of the simpler combined load Since this theory gives the same results
systems, the exponents of the stress ratios in as the well known energy of distortion method
equation (24) can be determined by the various it is often referred to as the Equivalent
well known theories of yield and failure that Stress Theory. The octahedral shear stress
have been developed. However, in many cases theory may be stated as follows:- In elastic
of combined loading and for particular types action at any point in a body under combined
of structures the exponents in equation (24) stress action begins only when the octahedral
must be determined by making actual failure shearing stress becomes equal to 0.47 fe'
tests of combined load systems. where f e is the tensile elastic strength of
the material as determined from a standard
Since the stress ratio method was pre- tension test. Since the elastic tensile
sented by Shanley many years ago, much testing strength is somewhat indefinite, it is common
has been done and as a result reliable inter- practice to use the engineering yield strength
actiop equations with known exponents have Fty. In this theory it is assumed that the
been obtained for many types of structural tensile and compressive yield strengths are
members under the various combined load the same.
systems. In a number of the following chapters,
the interaction equations which apply will be Figs. Cl.21 and Cl.22 illustrate the
used in determining the Ultimate strength conditions of equilibrium involving the octa-
design of structural members. hedral shear stress. In Fig. Cl.21, the cube
is SUbjected to the 3 principal stresses as
Cl. 16 Determination of Yield Strength of a Structural shown. A tetrahedron is cut from the cube
Member Under a Combined Load System. and shown in Fig. Cl.22. Three of the sides
of this tetrahedron are parallel to the
As explained in Art. Cl.14, the flight
vehicle structure must carry the limit loads
without yielding, which in general means the
yield strength of the material cannot be
exceeded when the structure is sUbjected to
the limit loads. In some parts of a flight
vehicle structure involving compact unit or
pressure vessels, biaxial or triaxial stress
conditions are often produced and it is
necessary to determine Whether any yielding 2
will occur under such combined stress action
when carrying the limit loads. For cases
where no elastic instability occurs, the
following well known theories of failure have
been developed.
Fig. Cl. 21 Fig. Cl. 22
C1.9
principal axes, while the normal to the fourth For a triaxial stress system,
side makes equal angles with the principal
axes. The octahedral shear and normal stresses _1/ 2 2 2 2
are the resulting stresses on the fourth side. f= ~v (fx-fz) +(fz-f y ) +(f,~fx) +6(fs +f s +f s )
y2 J D ~ P
- - - - - - - - - - - - - -(31)
The equation for the value of the normal
octahedral stress is, For a biaxial stress system, f y , f syz '
f sp = 0
1
foct = ~ (f~ + f 2 + fa) - - - - - -(25) f = J f~ 2
+ f~ - fxf z + 3f sxz - - -(32)
The equation for the octahedral shear C1. 18 Example Problem 1.
stress is,
A cylindrical stiffened thin sheet fuse-
lage is fabricated from 2024 aluminum alloy
sheet which has a tensile yield stress
Fty = 40000. Find the yield margin of safety
Now the octahedral shear stress is 0.47 under the following limit load conditions.
of the normal stress.
(1) A limit bending moment produces a bending
Let t be the effective axial stress in stress of 37000 psi (tension) at top
uniaxial tension or compression which results point of fuselage section. The flexural
in the given octahedral shear stress. shear stress is zero at this point.
t = J f~ + f: - f~f2 - - - - - - -(30)
There are no flexural shear stresses at
the fuselage point being considered. Since
It is often more convenient to use the x, y no torsion is being applied
and z component of stresses instead of the to fuselage no torsional
principal stresses. Fig. Cl.23 illustrates shear stresses exist. The
the various component stresses. stress system at the point
being considered is thus a
fz biaxial stress system and
f~ and f 2 are principal
stresses.
f~ = 37000 + 4300 =41300 psi
f 2 = 8600 psi
C2.1 Methods of Column Failure. Column Equations. The range AB in Fig. C2.1 is for a range
of Lip values of below 20 to 25, and repre-
In Chapter AlB, the theory of the elastic sents a range where failure is due to plastic
and inelastic instability of the column was crushing of the column. In other words, the
presented. The equations from Chapter AlB column is too short to buckle or bow under
for a pin end support condition are:- end load but crushes under the high stresses.
This column range of stresses is usually
For elastic primary failure, referred to as the block compression strength.
The range BC represents the range of LiP If we let P = failing or critical load,
values Where failure is due to inelastic in- equation (5) can be written as equation (6)
stability of the column as a whole and equation
(2) applies. This range BC is often referred by realizing that p = FcA and p = vIlA.
to as the short column range. n 2 EI nBEt I
p = --
(L' )B
p =- - ------
(L' )2
(6)
C2.1
C2 2 STRENGTH OF COLUMNS WITH STABLE CROSS-SECTIONS
I'f;- 107,400
112,200
117,000
24.65 x 10 6
23.20 x 10 6
21. 75 x 10 6
47.57
45.15
42.81
C=2.05 20.30 x 10 6
121, 500 40.58
125,800 18.85 x 10 6 38.43
Fig. C2.2 130,000 17.40x10 6 36.32
134,000 15.95 x 10 6 34.25
C2.3 Design Column Curves for Various Materials. 137,700 14.50 x 10 6 32.22
147,000 11.60x10 6 27.89
For routine design purposes it is con- 158, 100 8.70 x 10 6 23.29
venient to have column curves of allowable 167, 600 5.80 x 10 6 18.47
failing column stress Fc versus the effective 173,800 2.90 x 10 6 12.82
slenderness ratio I' /p. In equation (5) we
will assume values of Fc , then find the This equation is plotted in Fig. C2.16.
tangent modulus Et corresponding to this For a given material, n, F O • 7 and E must be
stress and then solve for the term L' /p. known. Then assuming values of F, we can
Table C2.1 shows the calculations for find corresponding values of Et/E from Fig.
17.7 PH (THI050) stainless steel sheet at C2.16. For values of E, F O • 7 and n refer to
room temperature. The results are then plotted Table Bl.l in Chapter Bl.
in Fig. C2.7 to give the column strength curve.
Similar data was calculated for the material C2.5 Non-Dimensional Column Curves.
under certain exposure time to different
elevated temperatures and the results are also Quite useful non-dimentional column
plotted in Fig. C2.7. Figs. C2.3 to C2.15 curves have been derived by Cozzone and
give column curves for other materials under Melcon (See Ref. 3).
various temperature conditions. Use of these
curves will be made in example problems later The Euler column equation is
in thiS chapter. The horizontal dashed line F = n2Et/(L'/p)2, which can be written,
is the compressive yield stress. Values
above these cut-off lines should be sUb- Et (L'/p)2
stantiated by tests. F - n2
C2.4 Tangent Modulus Et from Ramberg-Osgood
Equation.
The problem therefore resolves itself
into obtaining and expression for Et/F from
The basic Ramberg-Osgood relationship the non-dimensional relationship. To do this
multiply both sides of equation (7) by F o • 7 /F
for Et is given as follows: (See Ref. 1) and equate to B 2.
Et 1
- - - - - - - (7)
E - 3
n
(_F_)n-l. - - (8)
1 +"7 FO• 7
120
. Fig. C2.3
Steel, AJSI 4130, 4140, 4340
70
I'" 0.188
Tell!l'. !lql(l6\Ire up to 1/2 SR.
Fill = $0000 (R. T.)
Fey ~ '10000 (R. T.)
60 i
50
50
r:: 40
40
30
30
20 ;.. " . . . . ..-~.' ... .
10
ViP
10 Figure C2. 4
20 30- 40 50· 90
Vip
Figure C2. 3
150 '.
30
20
10
o 20 30 4050· 60 70 80 90 100
L'If'
Figure C2. 5
F c is in ksi
C2.4 STRENGTH OF COLUMNS WITH STABLE CROSS-SECTIONS
180
170
160
150
140
130
120
100
r.."
90
80
70
60
50
40
30
VIP
~ W W ~ ~ 00 ro 00 00 ~
70
60
50
40
r.."
r.. "
20
10
L'lp
Figure C2. 10
L'/P
Figure C2. 9
Fe is in ksi
C2.5
50 90
L'/p
Figure C2.12
Figure C2.1l
Fe is in kai
L'/('
Figure C2.13
C2.6 STRENGTH OF COLUMNS WITH STABLE CROSS-SECTION
F c is in ksi
~1.00
....
If
g:
o
n
.... .90
I"" "'"
""'.
---.... ...........
-
~ --........ ~
-......... r--:: r-...... -.......::: ~~ ~
.........
~~
~
1'-.." ~~ .10
~
\ ·20
\. -n- O
.. n F-
1. 00
.90
.........
"," [\.~
~
N
..3 .80
~
--........
""'.
""
.......... ~ r-....
"-
i"--. r-....
"'" '"
'"
'"'
~ "~" "'- r\!\
~\
\\ 9 1\ 1\1\
,\\~ \
-\ 1\\\ \
.80
~.O. >-.....
""
~~ K 3 1'(\ ,\I\~~
2~ ~ ~. ",'\ \\ ~\\ 1\
.70
.60
~ ~l:\~ \ .60
~
~ H--
Et
E .50
~~
I--
--'---
-........
I---
- -"----
.50
\~ I'-. r----- I-- r-- ~' 2.0
~ .~
...........
r----- f--. -
.30 Et
E =1
1
3 ( F )n- ~
~~ ",," "'- "" --........
~
....... -- 3- r--
.30
.20
I
~ Fig. C2.16 Dimensionless tangent modulus stress curves.
I
+ifn F o • 7
I I I I I
~~
\\
~~ "'"'
""-I'--
~~ "'-- I'--
""-
""""
r-----
-
-........ ~
- ! r-
.20
---- --
'6 .10
.10
\""- R E=::: ~, r-- r:-r ---.... r--
o . 10 . 20 . 30 . 40 . 50 . 60 . 70 . 80 .90
-.1. 00
~ ""
1. 10
~
lit>
1. 20
11
1. 30 1. 40 1. 50
F/F o • 7 Fig. C2. 16 (Ref. NACA T. N.902) 0 = s/s~
0.3
O. 2 ~~...:._~. --.
0.1·~
...
()
-J
C2.8 STR_ENGTH QXCOI.lJ~tNS WITH §,!,_~BLE CR.OE!S-SECTIONS
particular properties of the material EO.7/F. Thus in Fig. C2.18, the deflection of
point (0) away from tangent at midpoint c
Inserting value of F - nBEt/(L'/p)B in equals unity in our assumed conditions and it
equation (8), also equals the first moment of the area of
the M/EI diagram between (0) and (C) about (0).
Et ~
( n B Et/(L'!p)B)
(F-E-j=
0.7'\ BB (Fig. C2.19).
The value of the ordinate for M/EI diagram
P 1l X
_ 1 at any point x from 0 is EI sin L
or B --
n
- - - - - - (9)
The total area under the curve is,
The use of the curves in Fig. C2.17 will area =-EIP 10L sin -x
1l dx =-P [-L-n cos -nx] L
be illustrated later in the example problem L EI L 0
solutions.
C2.6 strength of Columns with Variable Cross-Section
or Moment of Inertia.
= :1 - {[~ (- l)J [- (l)J}* = ~ (~ + *)
To save weight in a built up column or
hence area =nEI
2PL and half area =nEI
PL
forged column, the member is tapered or is
made with a non-uniform cross-sect1on. To The center of gravity of the half area is
find the ultimate strength of such columns,
it is usually necessary to use a trial and Ai =1 x da
error method. The general method of solution
involving a consideration of column deflection P L/2
will be illustrated for a case of a long ( PL) - =-1 x sin [- x dx,
nEI x EI
column with uniform cross-section. 0
~..~
material. The practical problem usually
involves a slenderness ratio where failure is
due to inelastic bending and thus E is not
constant. For this case, a trial and error
Pl. C CI '6 C;O"l6TANT
method of solution in necessary using the
.3Bl tangent modulus of elasticity which varies
Fig. C2.20 with stress in the inelastic stress range.
We can now apply the same procedure to C2.7 Design Column Curves for Columns with Non-
Uniform Cross-Section.
a column with non-uniform cross-section. The
steps in this procedure for a column Figs. C2.21 and C2.22 give curves for
symmetrical about the center point are as rapid solution of two types of stepped columns.
follows:- Figs. C2.23 and C2.24 gives curves for the
rapid solution of two forms of tapered columns.
(1) Assume a sine curve for the deflected Use of these curves will be illustrated later
column with unit deflection at center in this chapter.
point.
C2.8 Column Fixity Coefficients c for Use with Columns
(2) Plot a moment of inertia (I) curve for with Elastic Side Restraints and Known End
column cross-section. Bending Restraint.
(3) Find the bending moment curve due to end Figs. C2.25 and C2.26 give curves for
load P times the lateral deflection. finding fiXity coefficient c for columns with
one and two elastic lateral restraints and
(4) Divide these moment values by the EI Fig. C2.27 gives curves for finding c when
values to obtain M/EI curve. The modulus restraining moments at column ends are known.
of elasticity E is considered constant. Use of these various curves will be
illustrated later.
(5) Find the deflected column curve due to
this M/EI loading. C2.9 Selection of Materials for Elevated Temperature
Conditions.
(6) Compare the shape of the derived column
deflection curve with that originally Light weight is an important requirement
assumed as a sine curve. This can be in aerospace structural design. For columns
done by multiplying the computed de- that fail in the inelastic range of stresses,
flections by a factor that makes the a comparison of the Fcy/W ratio of materials
center deflection equal to unity. Since gives a fairly good picture of the efficiency
the assumed sine curve is not the true of compression members when subjected to
column deflection curve, the computed elevated temperature conditions. In this
deflection will differ somewhat from the ratio Fcy is the yield stress at the particular
sine curve. temperature and w is the weight per cu. inch
of the material. Fig. C2.28 shows a plot of
(7) With the computed deflection curve, Fc /w for temperature ranges up to 600 0 F.
modified to give unity at center point, Wirh 1/2 hour time exposure for several im-
repeat steps 3, 4, 5 and 6. The results portant aerospace materials.
this time will show derived deflection
curve still closer to the assumed C2.10 Example Problems.
deflection curve.
PROBLEM 1.
(8) To obtain the desired accuracy, the pro-
cedure in step (7) will usually have to Fig. C2.29 shows a forged (I) section
be repeated again. member 30 inches long, which is to be used as
CRITICAL LOADSJNON-UNIFORM COLUMNS
Fig. C2.21
CRITICAL LOADS-NON-UNIFORM COLUMNS
Fig. C2.22 ....
o
Sin~le Stepped - Pin Ended Double Stepped - Pin Ended
10 r;:;::-- _
10~ __
9
&/L
8
·9
8
7
-------~..:.-- ::
• 5 • 5
·7
.8
.6
2 2
o
6 7 12 13 1~ 15 1b 3 6 7 8 91OU12~
(U),/ (U)2
00
CRITICAL LOADS-NON-UNIFORM COLUMNS CRITICAL LOADS-NON-UNIFORM COLUMNS
>-:l
Constant Thickness - Taper in Plan-Form Solid Prisms Tapering in Wi4th and Thickness
Solid Cylinders Tapering to Cones Fig. C2.24 >
tll
t"'
t>.l
(j
::tl
o
00
00
I
00
t>.l
(j
>-:l
....
o
z
00
•
2 7 8 9 w U 12 ~ " ~ ~
(U),j(ul?,
Fig. C2.23
7 2.0
For j..L = 9Q, C = 2.05
6 1.8
C One End Restrained
5 1.6
4 --=-- 1.4
3' 1.2
2 1.0
1
o 0.1 0.2 0.3 0.4 0.5 0.6 0.7 0.8 0.9 1.0 o 10 20 30 40 50 60 70 80 90 100 110
Fig. C2.26 xh Ref. Convair structure Manual l..LL ....
"EI
Fig. C2. 27 ....
C2.12 STRENGTH OF COLUMNS WITH STABLE CROSS-SECTIONS
~~)
by the end restraint conditions. For failure
by bending about the x-x aXiS, the end restraint
against rotation is zero as the single fitting
~ bolt has an axis parallel to the x-x axis and
thus c the fiXity coefficient is 1. For
failure by bending about the y-y axis we have
end restraint which will depend on the rigidity
of the bolt and the adjacent fitting and
structure. For this example problem, this
restraint will be such as to make the end
fiXity coefficient c = 1.5.
se;,.A-A
C2.13
For failure about x-x axis, Thus we make use of the curves in Fig.
C2.17.
L' = L/v'C= 30/v'l= 30, L'/px = 30/ .83
Case 1. Material 7079-T6 Alum. Alloy forging.
= 36 Table Bl.l of Chapter Bl summarizes certain
material properties. The properties needed
For failure about y-y axis, to use Fig. C2.17 are the shape factor n, the
moduls Ec and the stress Fa.?' Referring to
L' = 30/.,[1:5= 24.6, L'/Py = 24.6/.60 Table Bl.l, we find that n = 26, Ec =
= 41 10,500,000 and Fa.? = 59,500.
Therefore failure is critical for bending The horizontal scale in Fig. C2.17 in-
about y-y axis, with L'/p = 41. volves the parameter,
1 - :_1
for all materials in their many manufactured and thus c 1. The
forms plus the various temperature conditions material is 7075-T6 I 8
would require several hurldred individual Extrusion. The problem r--~I
column charts. The use of such curves can be is to find the failing I y
avoided if we know several values or parameters stress Fc under room
regarding the material as presented by temperature conditions. Fig. C2.31
Ramsburg and Osgood and expanded by Cozzone
and Melcon (see Arts. C2.4 and C2.5) for use This (I) section corresponds to Section
in column design. 15 in Table A3.15 in Chapter A3. Reference
C2.14 STRENGTH OF COLUMNS WITH STABLE CROSS-SECTIONS
to this table gives, is due to the fact that the stress existing
under a L' /p value of 51.7 is near the pro-
A = .594 sq. in. Px = .618 portional limit stress or Et is not much
different than Ec , the elastic modulus.
L' =L, sin c = 1, L'/px = 32/.618 = 51.7
To illustrate a situation where the 7075
Fig. C2.11 gives the column curves for material becomes more efficient in comparison
this material. For L' /p =
51.7 and room to the 2014 alloy, let us assume that our
temperature we read Fc = 38,500 psi. member has a rigid connection at its end which
will develop an end restraint equivalent to a
Solution by using Fig. C2.17, fixity coefficient c 2. =
From Table Bl.l, n =8.8, Ec = 7,800,000 Then L' = 32/y'2:= 22.6 and L'/p =
and FO • 7 = 29,000. 22.6/.618 =
36.7.
B =~ yl7,~~O~ggo (51.7) =1.00 For the 7075 material from Fig. C2.11, Fc =
58,300
From Fig. C2.17 we find Fc /F o • 7 = .74 For the 2014 material, we use Fig. C2.17
Then Fc =
.74 x 29,000 =
21,450 psi
B =~n vi 10,700,000
53,000 (36.7) = .823.
Consider the member is SUbjected to a
temperature of 450 0 F for 1/2 hour.
From Fig. C2.17 for B .823 and n= =
From Fig. C2.11, Fc = 21400 psi 18.5, we read Fc /F o • 7 =
.87, when Fc .87 x =
53,000 =
46,100 as compared to 58,300 for the
Using Fig. C2.17:- 7075 material, thus 7075 material would permit
lighter weight of required structural material.
From Table Bl.l, n = 8.8, Ec = 7,800,000 The student should realize that if the
and FO • 7 = 29,000.
stress range is such as to make Et =
Ec ' then
the bending failure is elastic instead of
B- nj
_ 1 29,000
7,800,000 (51.7) = 1.00 inelastic and equation (5), using Young's
modulus of elasticity Ec ' can be solved
directly without resort to column curves or
From Fig. C2.17 we find Fc /F o • 7 = .74 a consideration of Et, since Et is equal to
Then Fc = .74 x 29,000 = 21,450 psi Ec '
A very common aluminum alloy in aircraft The student should realize that equation
construction is 2014-T6 extrusions. Let it (5) is for strength under primary column
be required to determine the allowable stress failure due to bending as a whole and not due
Fc for our member when made of this material. to local buckling or crippling of the member
or by tWisting failure. The subject of
Since we have not presented column curves column design when local failure is involved
for this material, we will use Fig. C2.17. is covered in a later chapter.
From Table Bl.l, for our material, we In example problem 2, we have assumed
that local crippling is not critical, which
find n = 18.5, Ec =
10,700,000 and F O • 7 = calculation will show is true as explained
53,000
and covered in a later chapter.
- 1 / 53,000 (51 7) 1.10
Then B - nv10,700,000 . = C2. 12 Strength of Stepped Column.
7075-T6 material. The problem is to find the This is a relatively short column so the
maximum compressive load this member will failing stress should fall in the inelastic
carry. range where E is not constant, therefore the
solution is a trial and error procedure. We
Portion 2 Portion 1 will base our first guess or trial on an
~3/4" Dia. j t1"Dia. j PJrtion 2 average L/p value.
, t
~b=15'~_a=30"_I~b=15"
: t ; =j' p for portion 1 is 0.25 inches
The stresses in each portion are, Referring to Fig. C2.16 and using 0.465
and n = 16.6, we read Et/E =
1.0, thus Et =
f1 = 1000/0.7854 = 1280 psi E and thus no plasticity correction for
Portion 1.
f2 = 1000/.4418 = 2270 psi
= 59,500/72,000
For Portion 2, f 2 /F o • 7
These compressive stresses are below the = .826
proportional limit stress of the material so
Ec is constant and our solution is correct. From Fig. C2.16, we obtain Et/E = .675
whence, Et = .675 x 10,500,000 = 7,090,000.
Case 2. Inelastic Failure.
EI 1 10,500,000 x .0491
The column has been shortened to the
dimensions as shown in Fig. C2.33. The EI 2
= 7,090,000 x .0155 =4.7
diameters and material remain the same as in From Fig. C2.22 for aiL = .5, we obtain
Case 1.
Portion 1 Portion 2
B =5.6.
.J j
: : I
Then Pcr =B EI 1 = 5.6 x 10,500,000 x .0491
r-b=3" --¥-- a=6" ------¥-b=3""} L 2
144
I~· L=12" ----~ = 20,000 lb.
Fig. C2.33
C2.16 STRENGTH OF COLUMNS WITH STABLE CROSS-SECTIONS
Our guessed strength was 26,300 lb. OUr C 1-1/4 -.058 A 1-1/4 -.058 B 1-1/4 -.058 D
guessed strength and calculated strength must
be the same so we must try again. T "
~'O
~
..,.
a>
0
I
Then L' = L/ve= 24/..) 2.92 = 14.08 (6 ) The pin ended single stepped column as
shown in Fig. C2.37 is made of AISI-4130
L'/p = 14.08/0.125 = 113 normalized steel, Ftu =
90,000, FCY =
70,000. Determine the maximum compressive
Fc =
2
= n x 29,000,000 = 22,500 load member will carry.
(113)2 psi I" Sq. Bar 3/4" Square Bar
r -----*"'----'----.tL----,
If the stress is above the proportional I I
limit stress fo~ the material, then the trial a = 21" ~_1'__9_"=4
and error approach must be used as illustrated L = 30" --+l
in the problem dealing With a tapered column. Fig. C2.37
C2.18 STRENGTH OF COLUMNS WITH STABLE CROSS-SECTIONS
(7) Same as Problem (5) but member is exposed (11) The cylindrical tapered member in Fig.
1/2 hour to a temperature of 500 oF. C2.39 is used as a compression member.
If member is made from AIBI Steel 4140,
(8) Same as Problem (5) but change dimension Ftu = 125,000, what is the failing load.
(a) to 10 inches, and L to 14.28 inches.
(9) Find the failing compressive load for til" Dia. Rod .1/2" Dia. Rod
the doubly stepped column in Fig. C2.38 c:::==: ~T
i f member is made from 7079-T6 hand
forging. t b
=9 ~a;~~~b=91
The value of f o is determined by making Fig. C3.7 k Factor for Some Typical Shapes
the requirement that the internal moment of Shape K Shape K
1
-e- 1.7
K • lOcI!
IIc' S.eU.. modlllvi
Let, Mb = total internal resisting moment. Qo Slotlc "',_IItobo\lt
1IlI1I1'lloIII.'.,.. 0110,..
" • •r n'lItr,l.sl,
mr = internal moment developed by
portion (r).
mb = internal moment developed by
portion (b).
rectangular, thus
Fig. C3.8 Section Factor K for 1,0, and C Sections (Ref. 2)
(5) Beam-column, curvature and shear lag to give the value of f o . The value of f o from
effects are considered negligible. this chart operation gives f o = 29000. Then
from equation (3)
C3.6 General Accuracy of Method.
Fb
yield
= 65000 + 29000 (1.5 - 1) = 79500 psi.
(1) It is exact for a rectangular section
under pure bending with moment vector parallel Then yield bending moment = Myp = FbI/c
to a principal axis.
Thus Myp = 79500 x .0938 = 7460 in. lb.
(2) For double s~metric sections under
pure bending and moment vector parallel to a For finding the ultimate resisting
principal axis, the accuracy should be within bending, we use Ftu which is 75000 as the
5 percent. value of f m in equation (3). Again going to
Fig. C3.17 to stress of 75000 on stress-strain
(3) Single s~metric sections will vary curve and the vertically d01m to f o curve, we
from practically exact to definitely uncon- obtain f o = 70500.
servative (moment vector normal to axis of
sYilunetry) . Then Fb(Ult)= 75000 + 70500(1.5-1)= 110250 psi
(4) For sections subject to combined Then MUlt = FbI/c = 110250x.0938 = 10370 in. lb.
bending and axial load, the results will vary
from practically exact to conservative. Let us asswne that is is desired to limit
the strain in the extreme fiber to .03 inches
(5) For unsymnetrical bending, with and per inch. What would be the bending moment
without axial load, the results will vary from developed under this limitation.
practically exact to conservative.
From Fig. C3.17 for a unit strain of .03
C3.7 Example Problems in Finding Bending Strength. the corresponding stress from the stress
strain curve is 74700 and the f o stress is
KXAMPLE PROBLEM 1. 61200.
A rectangular bean section is 0.25 inches Then Fb = 74700 + 61200 (1.5-1) = 105300
wide and 1.5 inches deep. What yield and
ultimate bending moment will the section Then M =FbI/c = 105300 x .0938 = 9900 in. lb.
develop when made from 7075-T6 extruded
alLLminum alloy. EXAMPLE PROBLEM 2.
Solution: The modulus of bending stress is The symmetrical I beam section in Fig.
given by equation (3), (a) is subjected to an ultimate design pure
=
bending moment M 14000 in. lb. What is
Fb = f m + f o (k - 1) - - - - - - (3) the margin of safety if
the beam is made of
k = 2Q =2 x 0.75 x .25 x .375 x = .1408 magnesium forging AZ6lA
=
I/c 1
(12 ) .25 x 1. 5 3( 1/ . 75 )
.0938 which has Ftu 38000
and Fty = 22000.
k = 1.50 Ix = J:....
12
x 1.375 x 2 3
-
From Fig. C3.19, for f m Ftu 38000 we = = bending strength was developed. From Fig.
find in projecting vertically downward to f o C3.19, the unit strain when stress is 38000
curve gives f o = 23700. Then sUbt. in is .035. Then since plane sections remain
equation (3) plane after bending, the unit strain at point
.50 inch from neutral axis is (.5/1) (.035) =
=38000 + 23700 (1.17 - 1) = 40770
Fb .0175. From Fig. C3.19, the stress existing
at this strain is f 31000 psi. A linear =
MUlt = FbI/c =40770 x .358 = 15000 in. lb. variation of stress as used in the flexural
equation would give half the maximum stress
Margin of Safety = M~lt _ 1 = 14000
15000 - 1 = _.-07 or 38000/2 =
19000 psi as against the true
11 stress of 31000.
50
60
~ 40
t" '" '
~ 30
~ 40 J'
10
20
60
60
.~ 40
A
~
~024"'I'4 Alum. Alloy Clad ~024-T81 Alum. Alloy Clad Sheet
Sheet &. Plate, Heat Treated s t < 0.Q64
t=0.25tQO.50 - 20
20 F = 62000
F tu =62000 tu
Fty" 5iOOO
Fty-= 40,000 Fig. C~.ll Fig. C3.l~
60
- Stiress ~ strain. ,Curve t k:q 1
40
'"
~
_0
0.02 0.04 0.06 0.08 0.10 0.12 0.02 0.04 0.06 0.08 0.10 0.12
E In/In E In/In
70 b
'iii
~
S 40'
~Ei J'
20
0.01 0.02 0.03 0.04 0.05 0.055 0.01 0.02 0.03 0.04 0.05
E: in/in E in/in
'iii
~
~ 40
S ~
J s
J
E in/in
40
30
~ 20
""ji
10 AZ61A lIiIas:ne&ium AlluY Forgings (Lol>g)
Ftu = 38000
Fty .22000
Fig. C3.19
160
~ 120
~ Fig. C3.22
17-4 PH Stainless Steel
Flu = 180,000 psi
_" 80 FlY = 165,000 psi
E = 27,5 x 10 6 psi
Elongation = 6%
40 Ileal Treated Bars & Forgings
o 0.01 0.02 0.03 0.04 0.05 0.008 0.012 0.016 0.020 0.024 0.028 0.032 0.035
e in/in E (inches/inch)
_
i
I--{--N.A.
1. 39
-=l
-:.r-u
found for this entire
Fig. (d)
section in Fig. (d). One
o 0.01 0.02 0.03 0.04 0.05
half of this amount will
E: in/in
then be the true moment developed by portion (1).
EXAMPLE PROBLEM 3. Unsymmetrical Section.
I =~
12 bh = ~
12 x
3
0.1 X 2.78
3
= 0.178
Fig. (b) shows a tee beam section,
symmetrical about the vertical axis. If the I 1 /c 1 = 0.178/1.39 = .128
material is 17-4 PH stainless steel, what
ultimate bending moment will be developed if Q1 = 1.39 x 0.1 x .695 = .0965
bottom portion is the tension flange.
1
Lo. 1"
...No" - - ----i------
to. 610
From Fig. C3.22, Ftu = 180000 which equals
Solution 1
= .0154.
The stress for a unit strain of .0154 As explained in solution (1), for e =
from the stress strain curve in Fig. C3.22 is .0154, f m = 172000 and f o = 129400.
172000, and f o = 129400.
Fb 2 = 172000 + 129400 (1.15-1) = 191400
Then Fb 2 = 172000 + 129400 (1.14-1) = 190100
M2 = 191400 x .0842 = 16100
M2 = 21 (Fb2 I 2/c 2) = 190100 x .1704 = 16200 Mtotal = M~ + M2 = 16620 + 16100 =
32720 in. lb.
Total resisting moment = M~ + M2 = 16550 +
16200 = 32750 in. Ibs. EXAMPLE PROBLSM 4. Fig. C3.24 shows an un-
s~~etrical I beam section. The material is
Solution 2. 7079-T6 aluminum alloy die forging. The
upper portion is in bending compression. It
Instead of making each portion a will be assumed that the compressive crippling
s~rrnetricalsection as was done in solution (1) stress for the outstanding upper legs of the
and dividing the results by two, we will find section is 65000 psi. (The theory and method
the internal bending resistance of each portion of calculating crippling compressive strength
as is when bending about the neutral axis of is given in another chapter.) The Ultimate
the entire section. Equation (3) now becomes design bending moment is 16500 in. lb. Find
for each portion of beam section, M.S.
~
'10
I
(g) shows lower portion (1). _
I~
1
= "3bc~ = "31
3
x .1 x 1.39
3 . l·lO
TN.A.
e~=. 0129--
= .0895
= .0895/1.39 = .0645 b
lJ
---+1
\ 39=c ~
I+- O. 1
1--- 3/4--1
Fig. C3.24 Fig. C3.25
I~/c~
Q~
..= 1.39 x .1 x .695
Fig. (g) Solution:
1 2 = ~ x 1.5 x .61
1
3
3
_
~1~1.5~
1~ Y.61=C2
N.A~
Upper portion:
Fig. 1)
(See
T II--l.
0.822
---+l0~1
50
= .0514
Fig. (h)
~N.A.-- ---N.A.
x 1.4 X 0.722 3 = .102 Fig. (i)
C3.9
f max = 65000. From Fig. C3.23, fo = 30000 Then the moment ratio Rb for combined
bending is,
Then fb = 65000 + 30000 (1.142-1) = 69260
m 1 = fbIlc = 69260 x .124 = 8580 in. lb.
_ 1
Lower Portion. Then margin of safety M.S. - Rb - 1.
'I~N'A'
Vector not Parallel to Either Axis.
1
- 3" x 0.65 X 1.078 3 1. 178 Since the symmetrical axis is a principal
INA = .137, I/c .137/ = 1 1 aXiS, the procedure in this case is the same
as for the double sYillTIetric case.
1.178 = .1164 1+-0. 75--+1 to. 1
Fig. (j) Rb = Rbx + Rb y
k = .1425/.1164 = 1.222
f max = 67500 M.S. =Rb1 - 1
From Fig. C3.23, f o = 44500 C3.10 Unsymmetrical Section with No Axis of Symmetry.
C3.11 Alternate More Exact Method for Complex a shear V-=600 Ibs. The problem is to find
Bending. the margin of safety under this combined
loading.
A beam section when resisting a pure
external bending moment bends about an axis For pure bending only the stress ratio is
that is called the neutral axis. No matter Rb =lL = 14000 = .933 (the value of 15000 is
what the shape of the beam cross-section for Ma 15000
any given external moment, there is an axis obtained from example problem 2).
about which bending takes place. The general
case involves an unsymmetrical beam cross- The stress ratio in shear is Rs =
fs/F su ,
section and material which has different where f s is the flexural shear stress and Fsu
stress-strain curves in compression and the ultimate shear stress of the material.
tension in the inelastic range. The neutral The problem therefore is to find the value of
axis therefore does not pass through the
fs'
centroid of the cross-section and thus the
method of solution is a trial and error The eqUivalent trapezoidal and triangular
approach. The solution procedure is outlined bending stress distribution will be determined
in Chapter A19, Article A19.17, and therefore for the design bending moment of 14000 in. lbs.
will not be repeated here. Also the chapter
dealing with the design of beams with non- For a triangular stress variation,
buckling webs explains and illustrates how
the ultimate bending resistance of an entire Fb =Mc/I =
= 14000/0.358 39150 psi.
beam section is determined.
From example problem (2) the shape factor
k was 1.17. On Fig. C3.19, the curve for
C3.12 Strength Under Combined Bending and k = 1.17 has been plotted. Starting with the
Flexural Shear. Fb stress of 39150 at the left scale, run
horizontal to an intersection with the k =
The previous part of this chapter has 1.17 curve, the projecting vertically down-
dealt with the determination of the strength ward to intersections with the stress- strain
of a beam section in pure bending. The usual curve and the f o curve to give 35800 for f m and
beam design problem involves flexural shear 19700 for f o ' The stress results are shown
with bending. In finding the true internal graphically in Fig. C3.28a and Fig. C3.28b.
resisting moment, the Cozzone simplified
method derives a trapezoidal bending stress 1 3 39150
distribution which will produce the same
internal resisting moment as the true internal 8;!;118---'-=-~~~rr~~-_--:~
bending stress system. A triangular stress
J__ T Lil~l~ . _
system is then derived which will also give
the true bending moment.
N.A
Now the equation for flexural shear stress -+1 1- -+1970/)
for a triangular bending stress distribution 1/2 Beam Section Fig. C3. 28a Fig. C3. 28b
is Case 1 Case 2
f
s
=VQ
It - (A)
The flexural shear stress is a function
of the rate of change of the bending stress.
Thus we can obtain a shear correction factor
Thus to use equation (A) for a C by comparing the bending stresses in the
trapizoidal bending stress, a correction factor two stress distribution diagram.
(C) must be applied or equation (A) becomes
The shear stress is maximum at the
f - CVQ neutral axis in this particular problem.
s - It - - - - - (B)
The total normal force on the cross-section
of beam above the neutral axis equals the
To illustrate how the correction factor stress times the area.
(C) can be determined, the I beam section used
in example problem (2) will be used. For simplification, the beam section will
be divided into the two portions labeled (a)
We will assume the ultimate design moment and (b).
of 14000 in. lbs. is produced by a load of
600 lbs. acting on a cantilever beam at a
point 23.30 inches from the fixed end of the
beam. Thus the beam section at the support
is subjected to bending moment of 14000 and
C3.11
19700 x 0.875 x 0.125 = 2155 C3. 13 Strength Under Combined Bending Flexural Shear
and Axial Compression.
14100 x 0.5 x 0.875 x 0.125 = 771
The subject of the ultimate strength
Total Force 8906 lbs. design under combined loads is treated in
detail in a later chapter.
For Case 2:-
A conservative interaction equation for
Portion (a) combined bending, shear and axial load is,
(2)
coincide with center line axis, thus use
trial and error method.
t~:14~1
1
rD~81Il:
.081 inch wall thickness.
8 1
Use the Cozzons method for solving the ~ +-16
following problems.
~I .!. I~
(3) Find the ultimate bending moment that 2
Fig. 4 Fig. 5 Fig. 6
each of the following beam sections will
develop when bending about the principal
axis and made from each of the following (4) A simply supported beam has a span of 24
materials, inches. Depth of beam limited to 2 inches.
It must carry an ultimate load of 4000
(a) 7075-T6 Alu~inum Alloy Extrusion. Ibs. located at midpoint of beam. Material
Ftu = 75000, Fty = 65000, is 7075-T6 aluminum alloy extrusion.
Design an I shaped section to carry this
(b) Ti-6Al-4V Titaniwn Alloy, load, Neglect areas of corner fillets
Ftu = 130,000, Fty = 120,000. that would be used in extruded shapes.
REFERENCES.
C4. 3 Design for Compression. any tendency of the test curve to rise rapidly
for very short lengths where failure is by
The strength of members With stable cross- block compression. Table C4.l shows the
sections when acting as columns can be cal- reSUlting short and long column equations
culated by Euler's equation if the bending after values of Fco and E have been SUbstituted
failure is elastic, or E is constant (eq. C4.1) in equations C4.3 and C4.4 and E in the Euler
and for inelastic bending failure, Euler's equation. The column headed transitional
equation With the tangent modulus Et replacing L' /p, represents the value of L' /p where failure
E (eq. C4.2) checks test reSUlts. (The stUdent change from inelastic to elastic failure or, in
should refer to Chapters A18 and C2 for theory other words, it is the dividing point between
on column strength.) the so-called long and short column range.
Thus if the equations are used, the L'/p value
- - - - - - - - - - - (C4.1) must be known in order to select the proper
equation.
(C4.2) C4. 5 Column Formulas for Aluminum Alloy Tubes.
L' is the effective length and equals From (Ref. 1), the basic short column
L/~ where c is the column end fixity equations for aluminum alloys are:-
coefficient.
FC - Fco [1 - 0.385 (L' /p)/n) E/FcoJ (C4.5)
Long and Short Columns
Fc = Fco [1 0.333 (L'/p)/nv E/FcoJ (C4.6 )
For many years the problem or subject of
inelastic column strength or failure was Fc - Fco [1 - 0.272 (L'/p)/n"; E/F co ] (C4.7)
treated almost entirely from a consideration
of test results. That is, sufficient tests For long columns:-
were made to establish the shape of the failing
stress curve in the region where the failure Fc = n lil E/(L'/p)2 ------- - (C4.8)
was at stresses above the proportional limit
stress of the material. Mathematical curves The equations for determining Fco are
were then derived to fit the test results. given in Table C4.2 (from Ref. 1). The table
Engineers referred to the columns which failed also indicates which of the three short column
by inelastic bending as short columns, and equations to use for the various aluminum alloy
thus referred to the equations that fit the materials.
test data as short column equations. The
columns that failed by elastic bending were To illustrate the use of Table C4.2, the
then referred to as long columns. The test column formula for 2024-T3 aluminum alloy
curve for long columns would follow the Euler tubing will be derived:-
column equation (C4.l) and thus tests were
not necessary to establish allowable failing From Chapter B2, we find the following
stresses in the so-called long column range. strength properties for 2024-T3 tubing,
Thus over the years short column equations
based on test results have been presented by Ftu = 64000, Fcy = 42000
official government agencies for use in
structural design. The official publication From Table C4.2, the equation for Fco is,
for the aerospace field is the Military
Handbook MIL-HDBK (Ref. 1). Fco = Fcy (1 + ..; Fcy/lOOO SUbstituting,
C4. 4 Column Formulas for Round Steel Tubes. Fco = 42000 (1 + ..; 42/1000 = 42000 (1 + .205)
Where Fco is the column yield stress or Fc = 50600 - 431 L'/p - - - - - - - - (C4.9)
(upper limit of column stress for primary
failure). It can be determined from test C4. 6 Column Formulas for Magnesium Alloys.
reSUlts by extending the short column curve to From (Ref. 1) the following short column
a point corresponding to zero length, ignoring equations for various magnesium alloy materials
C4.3
1025 ...•,••.•..•.... 55 36 36 36, 000-1. 172(L'/p)'" .•• C4.3 122 286 x 1Q6/(V/p) 2 .... (d)
4130 ••.....•••.•••• 95 75(e) 79.5 79,500-51. 9(V /p) 1.6•• C4.4 91 286 x 10 6/(V/p)'" .... (d)
Heat-treated e alloy
steel ••....•..... 125 103 113 113,000-11. 15(L'/p) '" .• C4.3 73 286 x 10 6/(V/p)2 .... (d)
Heat-treated alloy
steel ............ 150 132 145 145,000-18. 36(V/p)'" .. C4.3 63 286 x 10"/(V/p)'" .... (d)
Heat-treated alloy 2
steel ..•••..•.•.. 180 163 179 179, 000-27. 95(L'/p) .. C4.3 56 286 X 10 6/(V/p)'" .... (d)
a Equation C4.1 may be used in the short column range if E d Not necessary to investigate for local instability when
is replaced by Et obtained from the combined stress- D/t<: 50.
strain curves for the material. e ThiS value is applicable when the material is furnished
b L'/p = L!p.,;C: L'/p shall not exceed 150 without specific in condition N (MIL-T-6736) but the yield strength is
authority from the procuring or certificating agency. reduced when normalized subsequent to welding to 60
c Transitional L/p is that above which columns are "long" ksi.
and below which they are "short." These are approximate
values.
are given in Tables C4.4 and C4.4a. or inelastic instability of the column as a
whole. As the slenderness ratio L'/p gets
Table C4. 4 Column Formulas for Magnesium- smaller, the Fc stress increases. Now if the
Alloy Extruded Open Shapesa diameter of the tube is relatively large and
GENERAL FORMULA the wall thickness relatively small or, in
n other words, if the diameter/thickness (D/t)
K(Fcy) ratio is large, failure will result by local
F c = (V/p)m crippling or crushing of the tube wall and
this local failing stress is usually repre-
(Stress values are in ksi) sented by the symbol Fcc. The values of Fcc
in general have been determined by tests (see
Alloy K n m Max. Fc design charts for Fcc versus D/t ratio).
MIA .• ...... ... 180 1/2 1.0 0.90 F cy C4. 9 Design Column Charts.
AZ3lB, AZ61A, AZ80A. 2,900 1/4 1.5 Fc~
AZ80A-T6, ZK60A-T5 • 3,300 1/4 1.5 O. 6 F cy In design, column strength charts are a
a Formulas given above are for members that do not fail by great time-saver as compared to SUbstituting
local buckling. in the various column equations, thus a number
of column charts are presented in this chapter
to facilitate the strength check of columns
Table C4. 4a Column Formula for AZ31B-H24 and the strength design of columns. Fig.
Magnesium-Alloy Sheet C4.2 is a chart of L'/p versus Fc for heat
treated round alloy steel tubing. Fig. C4.3
is a similar type of chart for aluminum alloy
round tubing. Fig. C4.4 gives column charts
Max. F c = F cy for magnesium alloy materials. All three
charts are taken from (Ref. 1). Figs. C4.5
and C4.6 represent a further simplication for
the design of steel and aluminum round tubing.
C4. 7 Short Column Equations for Other Materials.
C4. 10 Section Properties of Round Tubing.
For other metals for which short column
equations are not available, the use of Euler's Table C4.3 gives the section properties
equation, using the tangent modulus Et can be of round tUbing. A tube is designated by
used (eq. C4.2). Refer to Chapter C2 for giving its outside wall diameter (D) and its
information on how to construct column strength wall thickness (t). Thus a 2-1/4 - .058 means
curves using this equation. a tube with 2-1/4 inch outside diameter and a
wall thickness in inches of .058. Since a tUbe
C4. 8 Column Failure Due to Local Failure. is symmetrical about any axis, the polar moment
of inertia, which is needed in torsion problems,
The equations as presented give the equals twice the rectangular moment of inertia
allowable stress due to failure by bending of as given in Table C4.3. For weight comparison,
the column as a whole and the action is elastic the weight of steel and aluminum tubing is
STRENGTH & DESIGN OF ROUND, STREAMLINE, OVAL AND SQUARE TUBING
C4.4 IN TENSION, COMPRESSION, BENDING, TORSION AND COMBINED LOADINGS.
5052-All Tempers
5083-All Tempers
5086-All Tempers
5454-All Tempers All Products Fcu(l +.../ F.u/lOOO) Equation 1.732r.../E/F•• Equation
5456-All Tempers C4.5 C4.8
6061-T4, T451, T451O,
T4511
2014-T6, T651
2024-T6, T81, T86, T851 Sheet and Plate·;
7075-T6, T651 Rolled Rod, Bar and F eu(l + .../ F eu/1333) Equation 1.346r.../E/F•• Equation
7178-T6, T651 Shapes; Drawn C4.6 C4.8
Tube
Rolled Rod, Bar and F.u(l +..; F .u/2OOO) Equation 1.224r.../ElFe. Equation
Shapes; C4.7 C4.8
6061-T6, T651, T651O, Drawn Tube;
T6511 Extrusions
·Includes clad as well as bare sheet and plate. Equation C4. 8 may be used In the short column range If E' Is
Transitional L'lp Is that above which the columns are "long" and replaced by E, obtained (rom the compressive stress-strain curve (or the
below which they are "short". material.
C4.5
C4. 11 Some General Facts in Tubing Design. Tension Allowables Near Welds in Steel TUbing (X-4130)
5. The student should realize that practical Solution: The compressive (M.S.) will be
limitations such as clearance requirements determined first. As the simplest solution,
may determine the diameter of the tube we can use the column curves in Fig. C4.5.
instead of strength-weight considerations. =
For a length of 30 and C 1, from the upper
Thus design can consist of checking the right chart we project upward to the inter-
tubes available under the given section with the 1-1/2 diameter tube and then
restrictions. horizontally to the left hand scale to read
the column strength of 14800 lbs. which we
C4. 12 Effect of Welding of steel Tubes Upon the Tension will call the allowable failing Pa.
and Column Strength.
S = Pa _ 1 = 14800 _ 1 = 02
Since welding affects the grain structure
11..
M
P 14500 •
of the tube material adjacent to the weld, The tube strength could also be found by
tests show the strength of the material using Fig. C4.2 as follows:
adjacent to the weld is decreased as compared
to the unwelded material. If a tapered weld L' = L/Y-C- = 30/y-r- = 30
is used, the effect of the weld is decreased.
Table C4.5 shows the allowable stresses in L'/p =
30/.5102 =
58.7. P is found from
tension to use when tension loads are carried. Table C4.3 as well as the tube area 0.2628
sq. in. Using 58.7 for L'/p on lower scale
In short columns, the primary column and projecting upward to the Ftu = 95000
failing stress may be greater than the local curve, which is the lower curve, and then
crippling strength of the tube adjacent to horizontally to left hand scale we read
the weld at the end of the tube. This local Fc =56500 psi.
failing stress due to welding is referred to
as the weld cut-off stress and the column Whence, Pa = FcA = 56500 x .2628 =
compressive stress Fc should not exceed this 14850 lb.
value. This cut-off weld stress is shown by
the horizontal lines in Fig. C4.2 and C4.5. The solution obViously could be made by
SUbstituting in the short column equation for
steel having Ftu =
95000, or
STRENGTH & DESIGN OF ROUND, STREAMLINE, OVAL AND SQUARE TUBING
C4.6 IN TENSION, COMPRESSION, BENDING, TORSION AND COMBINED LOADINGS.
180
.... , I I
Tobie of Weld Cutoffs
I
\
170 Heat Treated After Weldina I--
~.Ftu= 180 ksi
Heat Treated Cutoff,ksi Reference
160
\ to Ftu. ksi
\\ 180
150
144
120
REF. 1
150 125 95 I--
~ \ Normalized
140 ' ... '-\ Cutoff heat treated 100 76.6
f--
95 67.5
after weld ing
130
\ \ Ftu= 150 ksi
120 \' 60 ~ 50
on
on
~
, r\\~/
- - - _ Unwelded column
~ ii> 40
110
r- ...... aliowobles .A 0'
C
on
~
:c.on
Ftu=125 ksi
\ ii>
c
S 30
100 E u
~ \ ::0 IE
-0 30
1\ u
90
'"
\'
:0
0
'J 20
E
80
, <i
,,-0
10
"'-",,\
70
-"weld cutoff after heat
treat 4130 0
60 0 20 4Q 60 80 100 120 140
I L'lp
f t Y=75 ksi
50
Ftu=95 ksi Fig. C4.3 2024-T3 and 6061-T6
~~ round aluminum alloy tubing.
40
30
20
0
I0 20 40 60
""
80 100 120
"-I'-
140 160 180 200 220
L'/p
50
VV
AZ80A-T5 and ZK60A-T5
III extruded shapes (except hollow)
III
~ 30 _ 0 96Fcy I I I -
~
in AZ80A-F extruded shapes
c
E
.2 20
o
:/ (except hollow)
I I I
/' Round tubes, Fcy : 10 ksi
"'t2(
U'
0
IJ...
10
Fcy
:'--- ~
o I
o 20 40 60 80 100 120 140 160
LIP
~ 20
Po<
r-.. o 20 40 60 80
0
COLUMN LENGTH - INCHES
~ 16
~
g 12 36
Eo<
20 40 60 80
COLUMN LENGTH - INCHES
rn
~
Po<
50
S
~ 40
8 4
~ 30
::t:
Eo< C=2
~ 20 C=l 20 40 60 80 100
\i::l
~
rn
COLUMN LENGTH - INCHES
~ 10
Fig. C4.5
6
t.l STRENGTH OF CHROME MOLY ROUND STEEL TUBES
C=2 FTY = 75, 000 PSI FTU = 95, 000 PSI
C=l 40 80 120 160
COLUMN LENGTH - INCHES
STRENGTH & DESIGN OF ROUND, STREAMLINE, OVAL AND SQUARE TUBING
C4.8 IN TENSION, COMPRESSION, BENDING, TORSION AND COMBINED LOADINGS.
7000 "'\"
,
(J)
24000 ".'- ...... ", ,'''" ' ,A.., 1/4
~ 6000 (J) ll- 3/B
'C ~1/2;
5
Po< ~ 0- 5/B
E - S/4
I 5000 ~ 20000 F -7/S
0-1
~o-l A.:1/4 H - 1-1/s
, I _ 1-1/4
1'<1 4000
a- 3/8
C - l/Z'
J) • 5/8'*
~o-l 16000 J - I-S/S
K - 1-1/2;
o-l L - 1.5/B
, E - S/•• 1'<1
~ F - 7/8' o-l
N- 1-3/4
N -1-7/B
~ 3000
0-'1·
H-l-1/B
~ 12000 0-2
P - 2;-1/4
o-l
o-l
1 -1-1/4'
J - l-S/B
~ Q - 2-I/Z"
< K - 1-1/2
o-l R - 2;-3/.'
:;i 8-30.0"
[:~.
8000
1'<1
Eo<
'0 g 4000
.'
~ C ' Eo<
=3
rmARD SIZES
o 10 20 30 40 50 o 20 40 60 80 100 120
COLUMN LENGTH - INCHES (C = 1) COLUMN LENGTH - INCHES (C = 1)
35000
14000 A - S/8
(J)
B - 1/2;
(J)
~ 30000 C - 5/S
~ 12000
A- 1/4 Oc 3/4
B - 3/B 0 E -7/S
0 C - 1/2 Po< Y-I
Po< 0- 5/B I G· 1-1/S
E - 3/4 ~ 25000 Ii - 1-1/4
.049 F - 7/B I - l-:VS
~ 10000 G -'1 0 J - 1-1/2
H-l-1/B o-l , K- 1-5/s
0
, L - 1-3/4
o-l I - 1-1/."
J - 1-3/8 ~ 20000 M- 1-7/B
1'<1 iII
o-l 8000 K - 1-1/2' N-Z
iII
<
L - 1-5/S
M- 1-S/4* ~ 0- 2-1/4
P - 2;-1/2
~
o-l 6000
N - 1-7/S
0-2
315000
o-l
'Q.2-3/4 '
R- s* 0.0.
o-l P -2-1/4 <
Q - 2-1/2 0. D.
< 1'<1
1'<1
Eo<
~ 10000
g 4000
~
o-l
Eo< ~ 5000
o-l
~
*AN STArfuARDSIZES
*AN STANDARD SIZES o 20 40'" 60 8() 10() 120
o 20 . 4()6() 80 100 120 COLUMN LENGTH - INCHES (C = 1)
COLUMN LENGTH - INCHES (C = 1) 35000
Pa = Ftu (weld factor) (area of tUbe) Pa = FcA = 63000 x .1849 = 11650 lb.
= 95000 x .841 x .2628 = 21000 Ibs. In this case heat treating produced
additional strength, whereas in Case 2 it did
M.S. = {Pa/p)-l = (21000/18500) - 1 = not. The reason for this is that failure
0.13, thus compression is critical. occurs in the inelastic stress range and heat
treating raises the material propertiBs in the
PROBLEM 2 inelastic range. The end fixity changed the
column from a so-called long column to a short
Case 1. Tube size 1-1/4 - .049, L = 40 in. column.
c =1
Material: Alloy steel, Ftu = 95000 The strength could be found also by sub-
Find ultimate compressive load it will stituting in the short column equal for
carry. 150,000 steel as given in Table C4.1,
L'/p = 40/.425 = 94. From Fig. C4.2, Solution: L' = L/ve : 24/~ = 19.7
using the 150,000 curve, we find Fc 32500. = = =
Then Pa = FcA =
32500 x .1849 = 6000 lb. Thus From Fig. C4.5 for L 19.7 on c 1
scale, we project upward to the 2 inch tube
heat treating the tUbe from 95000 to 150,000
for Ftu did not increase the column strength. and note that it intersects the horizontal
For a L'/p = 94, it is a long column and weld cut-off line which gives an allowable
failure is elastic and E is constant. column load at left scale of Pa = 26700 lb.
Failure in this case is local crippling
The strength could also be calculated by adjacent to welds at the tube ends.
Euler's equation from Table C4.I.
M.S. = Pa / p = 26700/25000 - 1 = .07
Fc =286,000,000/{L'/p)2 Case 2. Assume tUbe is heat treated to Ftu
= 286,000,000/(94)2 = 32500 psi, the = 125000 after welding. What is tube
same as previously calculated.------ strength.
giving Fc = 95000, whence Pa = 95000 x .3951 The results show that 1-1/4 - .035 is the
= 37500 Ibs. lightest. Since there is danger in welding
.035 thickness to the other heavier tUbe gauges
If the tube had not been welded at ends particularly the engine mount ring which is
the dashed part of the column curve could have usually relatively heavy for this size engine,
been used, thus giving additional strength. a minimum tUbe thickness of .049 will be used,
hence the 1-1/8 - .049 tube will be selected.
PROBLEM 4
Consider Member (2)
Fig. C4.7 shows a steel tubular ,engine
mount structure for a 1050 H.P. radial engine. Design loads 11650 tension and 4250
The ultimate design tension and compressive compression. Since the tension load appears
load in each member as determined from a stress critical, the tube will be designed for the
analysis for the various flying and landing tension load and then checked for the
conditions are shown in ( ) adjacent to each compressive load. The Ftu of the material
member. The true length L of each member is equals 95000 psi. Since the engine mount in a
also shown. Using chrome-moly steel tUbes, welded structure, the strength of the tube ad-
Ftu = 95000, select tube sizes for the given jacent to the end welds must be reduced to .841
loads. It is common practice to assume the x 95000 = 80000 psi (see Table C4.5).
column end fixity c = 1 for engine mount
members, since the mount is subjected to Hence tube area required = 11650/80000 =
considerable vibration and the true rigidity 0.146 sq, in. From Table C4.3, which gives
given by the engine mount ring is difficult the section properties of round tUbes, we
to accurately determine. select the following sizes:
Case 1. Tube size 1 - .049 round. If a round tube is exposed to the air-
L = 24, c =
1, Material 2024-T3 stream, the air drag is about 15 times greater
Find failing compressive load. than if it were given a streamlined shape, thus
streamline tubes are used when the member is
Solution: The col1nnn curves in Fig. C4.6 are exposed to the airstream.
slightly conservative because the equation used
was slightly different from the equCLtion now Streamline tubes are drawn from round
specified in (Ref. 1). tubes. In designating a streamline tUbe, the
round tube from which it was made is used and
Use L = 24, we read for 1-049 tube a then the fineness ratio is also given. The
failing load of 2800 lb. fineness ratio is the ratio L/D, which dimen-
sions are shown in Fig. C4.8. The most common
As a second solution, we will use Fig. fineness ratio used is 2.5 to 1. Table C4.6
C4.3. L' = L/y'C: = 24/v-r- = 24. L'/P = shows the section properties of streamline
24/.3367 =
71.3. From Fig. C4.3, we read tubing haVing a fineness ratio of 2.5 to 1.
Fe = 20000. Then Pa FcA = =
20000 x .1464 = Figs. C4.9 and C4.10 give curves for finding
2930 lb. the column failing stress Fe and the local
crushing stress Fcc.
The answer could be obtained by substi-
tilting in equation C4.9, Fig. C4.8
For L'/p =
58.5 and using lower curve on Tables C4.8 and C4.7 give the section
Fig. C4.4, we read Fc = 7600. Then Pa FcA = = properties for square and oval shaped tubes
7600 x .1464 = 1110 lb. respectively. For the design of these shaped
C4.13
Streamline ·TABLE C4. 6 SECTION PROPERTIES OF STREAMLINE TUBING (Fineness Ratio 2. 5to 1)
Equiv. Wall Axes Wt. I Equiv. Wall Axes
Round Z p I Z p
Area per fr. Major Major Major Round Wt. I Z p I Z P
O.D. Oec'l Gage Major Minor Minor Minor Minor Minor Area per ft. Major Major Major Minor Minor
O.D. Dec'l Gage Major Minor
'l4 .035 20 1.0114 .4286 .0786 .2673 .0017 .0079 .1486 .0073 .0140 .3046 2% .049 18 3.3713 1.4285 .3773 1.283 .1018 .1425 .5194 .4063 .2343 1.0377
.049 18 .1079 .3668 .0022 .0103 .1435 .0097 .0187 .2998 .058 17 .. ..
..
.4450 1.513 .1186 .1660 .5163 .4764 .2747 1.0346
)08 .035 20 1.1800
.. .5000 .0924 .3140 .0028 .0112 .1753 .0118 .0195 .3572
.065 16
.. .4972 1.690 .1312 .1837 .5137 .5300 .3057 1.0325
.049 18 .. .1272 .4323 .0037 .0148 .1703 .0158 .0261 .3525
.083 14 .. .. .6302 2.143 .1621 .2269 .5071 .6640 .3832 1.0265
.058 17
.. .1489 .5061 .0042 .0168 .1671 .0182 .QJOI .3495
.095
.120
13
11 .. .. .7178
.8972
2.440
3.050
.1815
.2188
.2541 .5028
.3063 .4938
.7504
.9226
.4333
.5329
1.0224
1.0141
1 .035 20 1.3485 .5714 .1061 .3607 .0043 .0151 .2020 .0178 .0257 .4100 2}i .049 18 3.7085 1.5714 .4158 1.414 .1366 .1739 .5732 .5434 .2848 1.1432
.049
.058
18
17
.. .. .1464
.1716
.4977
.5835
.0057
.0065
.0200
.0228
.1972
.1939
.0239
.0278
.0345
.0402
.4044
.4023
.058 17 .. .. .4905 1.668 .159.3 .2028 .5699 .6372 .3340 1.1398
.065 16
.. .. .5483 1.864 .1765 .2247 .5674 .7099 .3723 1.1379
17S .035 20 1.5170 .6428 .1199 .4074 .0063 .0196 .2292 .0257 .0329 .4626
.083 14
.. .. .6954 2.364 .2188 .2785 .5609 .8907 .4673 1.1318
.049 18
.. .. .1656 .5631 .0083 .0258 .2240 .0347 .0445 .4573
.095
.120
13
11
.. .. .7924
.9915
2.694
3.371
.2455
.2973
.3125
.3785
.5566
.5476
1.0081
1.2427
.5289
.6523
1.1279
1.1196
.058
.065
17
16
.. .. .1944
.2165
.6609
.7359
.0094
.0103
.0292
.0320
.2205
.2182
.0402
.0443
.0516
.0569
.4549
.4525 3 .058 17 4.0455 1.7142 .5361 1.822 .2085 .2433 .6236 .8318 .3997 1.2456
1}4 .035 20 1.6857
.. .7143
.. .1336 .4542 .00118 .0246 .2559 .0355 .0409 .5154
.065
.083
16
14
.. ..
..
.5993
.7606
2037
2586
.2312
.2872
.2698
.3352
.6211
.6145
.9265
1.1645
.4452
.5599
1.2434
1.2373
..
.049 18
.. .. .1849 .6285 .0116 .0325 .2509 .0482 .0557 .5107 .095 13 .. .. .8670 2.947 .3229 .3768 .6103 1.3184 .6338 1.2332
.058
.065
17
16
.. .. .2172
.2420
.7384
.8226
.0133
.0145
.0372
.0406
.2477
.2451
.0560
.0618
.0647
.0714
.5077
.5053
.120
.156
11
%~
.. .. 1.0857
1.)959
3.691
4.746
.3925
.4824
.4580
.5630
.6013
.5879
1.6289
2.0497
.7839
.9869
1.2249
1.2118
1% .035 20 1.8543 .7857 .1473 .5009 .0118 .QJOO .2828 .0476 .0549 .5683 3)4 .058 17 4.3828 1.8571 .5816 1.977 .2668 .2873 .6773 1.0617 .4708 1.)511
.049 18 .. .. .2041 .6939 .0157 .0400 .2777 .0648 .0680 .5635 .065 16
.. .. .6504 2.211 .2961 .3189 .6747 1.1830 .5248 1.)487
.058 17 .. .. .2400 .8158 .0181
.0198
.0461 .2745
,2.719
.0754 .0791 .5604 .083 14
.. .. .8258 2807 .3689 .3973 .6684 1.4886 .6604 1.)426
.065 16 .2675 .9094 .0504 .0833 .0875 .5581 .095 13 .. .. .9416 3201 .4150 .4470 .6639 1.6873 .7489 1.)386
1% .035 20 2.1913
.. .9285 .1748 .5943 .0198 .0426 .3365 .0793 .0704 .6736
.120
.l56
11
"
/32
.. .. 1.2742
1.6414
4.332
5.580
.6400
.7934
.6400
.7934
.7087
.6952
2.6269 1.0828
3.3221 1.3705
1.4358
1.4227
.049 18 .. .. .2426 .8248 .0266 .0573 .3314 .1085 .0964 .6689
16 5.0570 2.1428 .7525 2.558 .4604 .4297 .7822 1.8299 .7033 1.5594
.058 17 .. .. .2855 .9707 .QJ08 .0448 .3282 .1266 .1124 .6659 3)4 .065 ..
..
.065 16 ..
..
.. .3186 1.083 .OJJ8 .0728 .3256 .1402 .1245 .6634 .083 14
..
.. ..
.9562
1.0908
3.251
3.708
.5754
.6490
.5371
.5777
.7757
.7713
2.3073
2.6195
.8871
1.0075
1.5534
1.5497
.083
.095
14
13
.. .4021
.4566
1.367
1.552
.0410
.0453
.0883
.0976
.3192
.3149
.1738
.1949
.1545
.1732
.6574
.6541
.095
.120
13
11
.. .. 1.)685 4.652 .7955 .7425 .7624 3.2505 1.2502 1.5412
.156 %2 1.7641 5.997 .9894 .9235 .7489 4.1192 1.5855 1.5281
1~ .035 20 2.3600 1.0000 .1886 .6411 .0249 .0498 .3633 .0995 .0820 .7263
2.2856 .8035 2.732 .5614 .4912 .8359 2.2270 .8025 1.6648
.049 18
.. .. .2618 .8902 .0336 .0672 .3583 .1363 .1123 .7216 4 .065 16 5.3940
..
..
.. 1.0214 3.472 .7024 .6146 .8293 2.8161 1.0148 1.6604
.058 17 .. .. .3083 1.048 .0389 .0778 .3551 .1592 .1312 .7186 .083
.095
14
13 .. .. 1.1655 3962 .7933 .6941 .8250 3.1917 1.1506 1.6549
.065 16
.. .. .3441 1.170 .0428 .0856 .3525 .1765 .1455 .7163
.120 11
.. .. 1.4627 4.973 .9739 .8522 .8160 3.9661 1.4303 1.6467
.083 14 .. .. .4347 1.478 .0521 .1042 .3461 .2192 .1809 .7101
.. .. 6.414 1.2153 1.0634 .8026 5.0332 1.8164 1.6333
".
.095 13 .4939 1.679 .0577 .1154 .3418 .2463 .2034 .7061 .156 %2 1.8867
.188 2.2457 7.635 1.4109 1.2345 .7926 5.9364 2.1431 1.6259
1)08 .035 20 2.5285 1.0714 .2023 .6878 .0308 .0575 .3902 .1227 .0943 .7789
.065 16 5.7313 2.4285 .8546 2.905 .6761 .5568 .8895 2.6784 .9082 1.7704
.049 18 .. .. .2811 .9556 .0417 .0779 .3851 .1685 .1296 .7743 4)4
.083 14
.. .. 1.0866 3.694 .8473 .6978 .8830 3.3817 1.1471 1.7641
.058 17 .. .. .3311 1.126 .0482 .Q900 .3815 .1969 .1515 .7712
.095 13 .. .. 1.2400 4.216 .9574 .7885 .8787 3.8426 1.3035 1.7604
.065 16 .. .. .3696 1.257 .0532 .0993 .3794 .2186 .1683 .7690
.120 11
.. .. 1.5570 5.293 1.1779 .9701 .8698 4.7789 1.6216 1.7519
.083 14 .. .. .4673 1.589 .0650 .1214 .3729 .2719 .2093 .7624
..
".
6.0748 2.0628 1.7387
.095 13 .5312 1.806 .0722 .1348 .3687 .3598 .2770 .7523 .156
.188
%2 .. 2.0095
2.3930
6.832
8.135
1.4729 1.2131
1.7144 1.4120
.8573
.8464 7.1732 2.4365 1.7314
2 .035 20 2.6970 1.1428 .2161 .7345 .0376 .0658 .4170 .1494 .1076 .8313 .7860 4.0256 1.2894 1.8696
411 .083 14 6.0683 2.5713 1.1517 3.915 1.0105 .9367
.049 18
.. .. .3003 1.021 .0510 .0893 .4120 .2053 .1480 .8269
.095 13 .. .. 1.)147 4.469 1.1429 .8889 .9324 4.5759 1.4662 1.8656
.. ..
.058
.065
.083
.095
17
16
14
13
..
..
..
..
.3539
.3951
.4999
.5685
1.203
1.)43
1.699
1.933
.0591
.0652
.0799
.0889
.1034
.1141
.1398
.1556
.4088
.4062
.3998
.3954
.2402
.2668
.3325
.3744
.1732
.1925
.2399
.2703
.8238
.8217
.8156
.8115
.120
.156
.188
11
%,
%.
..
..
..
..
..
..
1.6512
2.1322
2.5403
5.613
7.249
8.636
1.4082 1.0953
1.7650 1.3728
2.0580 1.6007
.9235
.9098
.9001
5.6952
7.2508
8.5740
1.8254
2.3247
2.7498
1.8572
1.8441
1.8372
.219 %2 2.9421 10.Q! 2.3221 1.8061 .8884 9.8001 3.1451 1.8251
2X .049 18 3.0343 1.2857 .3388 1.152 .0735 .1144 .4657 .2946 .1887 .9325
.058 17 .. .. .399~ 1.358 .0854 .1329 .4625 .3450 .2212 .9294
.065 16
.. .. .4462 1.517 .0944 .1469 .4600 .3835 .2458 .9271
80
LLU-LW.~'-
o .: 80,000
.083 14
.. .. .5650 1.921 .1161 .1807 .4533 .4794 .3075 .9223 ...;
.095
.120
13
11
.. .. .6432
.8030
2.186
2.730
.1298
.1556
.2020
.2421
.4492
.4402
.5409
.6629
.3470
.4255
.9170
.9086
rn
P<
j-
+
~ ~~-
c :I::
rz. D/t 20 rn
~
0 70
rn ~ rn 70,000
&:l u rn
riI
Z 30 I
OJ p:;
4; OJ
rn Eo-<
60 ~ 50
:::> 60 rz. rn 60,000
li
\
40
0
~ 1
:I:: "-n1J
til 4 If
50~
Eo-< 50
~ '"
t:
"0-, ~
10 ~ 50,000
V> ::> rn 50
40f----203O~ rn 0 25 50
V>
U, 30 f
~ Iitt', I I IIIII
""'-
riI "
,
U
t:
D/t 4 u. U
t
~I Eo-< Tube
E
.2
30
"'" 20
0 2040 60 80 100
rn 40
z
..
0
"'" I
u D/t t
~
:0 20 :::>
0
........ ..:l
3 0
51
<i 10 ~ U
riI
30
NOTE: Higher values of allowable
u.U """'"- t--- ..:l stress can be used in short column
D' BaSIC round tube diameter
I:l:I range if substantiated by tests
0 I I II 4;
20
0 20 40 60 80 100 120 140 :::
0
"-
L'lp ..:l Fig. C4.9 tHttltt
..:l ~
<>::
I 10 ALLOWABLE COLUMN AND CRUSHING STRESSES
Fig. C4.10 Streamline 2024-T3 Tubing OJ CHROME MOLYBDENUM STREAMLINE TUBING.
rz. F ty = 75000 PSI
0
o 20 40 60 100 120 140
STRENGTH & DESIGN OF ROUND, STREAMLINE, OVAL AND SQUARE TUBING
C4.14 IN TENSION, COMPRESSION, BENDING, TORSION AND COMBINED LOADINGS.
1" .035
.049
.058
20
18
17
.1599
.2217
.2608
.5435
.7536
.8865
.0378
.0513
.0595
.0604
.0821
.0952
.4891
.4831
.4778
.083
.095
.120
14
13
11
.8326
.9494
1.1898
2.830
3.227
4.045
.9481
1.0724
1.3213
.6896
.7799
.8653
1.0671
1.0628
1.0541
.065 16 .2908 .9886 .0657 .1051 .4753 .156 li. 1.5291 5.198 1.6569 1.1121 1.0410
.083 14 .3666 1.246 .0806 .1290 .4960 .188 H• 1.8238 6.200 1.9334 1.3264 1.0296
.095 13 .4161 1.414 .0899 .1438 .4647
.120 II .5161 1.755 .1074 .1718 .4S61 .9860
.065 16 .7166 2.436 .6573 l.l730
1% .035 20 .1763 .5992 .0506 .0736 .5360 .083 14 .9102 3.094 1.2394 .8263 l.l669
.049 18 .2446 .8316 .0689 .1003 .5309 .095 13 1.0383 3.530 1.4032 .9355 l.l625
.058 17 .2879 .9787 .0801 .1I66 .5276 .120 11 1.3021 4.426 1.7325 1.1550 l.l544
.065 16 .3212 1.092 .0886 .1288 .5251
.083 14 .4055 1.378 .1091 .1587 .5187 .156 li. 1.6750 5.694 2.1793 1.4529 l.l406
.095 13 .4605 1.S66 .1219 .1773 .5145 .188 H• 1.9997 6.798 2.5S03 1.7002 l.l381
.120 II .5723 1.946 .1464 .2130 .5058 .219 H. 2.3081 7.847 2.8873 1.9249 l.l184
00
r:il
....
Eo<
~
r:il
Po.
o
~(
o.} ~ ~~ ~~ ~~ ~~~ ~~~ ~~~~ ~~~ ~~~ ~~~~ ~~~~~ ~~~~ ~~~~~ ~~~~~ ~~~~~~ ~~~%~~ ~~~~~~
Po. z ,:ll--t-------------------------------------i
Z ....
01Il
.... ::>
Eo< Eo<
U,.J
r:il..:
00>
<-0
"':r..
uO
~: ....
r:il
,.J
oi
III
..: ~: ~:
Eo<
.
~ ~?a~ ~~ ~~ ~~~ ~~~ ~~~~ ~~~ ;!::!::;;e ~::;:!::~ ~::;:!::~~::;:s~:::;'!:;:!::;!;";::;';::: :!::;!;"~:::ooI:~;!;.::~-{:oo!=:!::;!;~:::_="!::!::;!;~;:::ooI:~
~I-t-----------------------------------------;
~ ~ ~~ ~~ ~~ ~~~ ~~~ ~~~~ ~~~ ~~~ ~~~~ ~~~~~ ~~~~ ~~~~~ ~~~~~ ~~~~~~ ~~~~~~ ~~~q~~
C4.15
tubes the primary column strength can be found C4. 18 Problems InvolVing Bending Strength of Tubes.
by using the curves in Figs. C4.9 and C4.10.
The crushing stress Fcc for oval shaped tubes PROBLEM 1
can conservatively be taken as that for
streamline tubes as given in Figs. C4.9 and A 1-1/4 - .058 round tube is used as a
C4.10. For square tubes the local crushing simply supported beam with the supports at the
stress Fcc can be taken as the crippling stress ends. The span or length of the beam is 24
of a flat plate. For this stress refer to the inches. It carries a uniform distributed load
chapter which covers the buckling and crippling w in pounds per inch. Find the value of w to
stress of flat plates with various widths, cause the tube to fail in bending if the tube
thickness and boundary edge conditions. is made from the following materials:- alloy
steel Ftu =
95000, 6061-T6 aluminum alloy, and
ULTIMATE BENDING STRENGTH 6AL-4V titanium.
Then w =
(44500 x .06187)/72 =38.1.
Tube material 6AL-4V titanium. ----
PROBLEM 2
A beam simply supported at its ends has
a span of 30 inches. The ultimate design load
consists of two equal loads of 2000 Ibs. each.
The beam is symmetrically loaded with each
load located 12 inches from the ends.
Fig. C4. 14 Bending modulus of rupture for round M = 2000 x 12 =24000 in. lb.
6A1-4V tubing. (Titanium)
1.7
1.6 ~
15 ~\
1.4
\~0..
\\ ~
_- -...
3
',.~
12 ,,
'" ..........
-. .. 450
.... .... --
u..&>':
I I
I ...
.... """ ............... ........ 6061-T6
[1
1.0
t--... . . . ---i.. . . . . . . _ 400
IT
0.9
~-T6ond7075-T6
0.8
0.7
2024-T3,T4 -- ..~ \
"\
0.6
0 10 20 30 40 50 60 70 80 90
350
\ \. "' I'. r...
Oil
I'..
:\ ~
Fig. C4. 12 Bending modulus of rupture for aluminum-alloy round tubing. .....
] 300
I'.
..... ..... ......
I':. ....
" .....
....
r-..
I'
r-..
......~
...
'0 250
-.... t'--
-
- - -
.... 10.. I"'- Flu, 260 ksi
on
:>
:;
........ -10.. Ftu. 240 ksi
"0
Flu. 220 ksi
o I"-
:::E ....... Flu, 200 ksi
-r-.
\ '"
c 200
.
"\ " ........
... -r-. Flu. 180 ksi
- - --
"0 I'"
"iii 60 C
....
\\
.>r. CD
01' .... ...
Flu. 150 ksi
~ r-..
Q
:l
:l
a::
50
\'\
150
"'.....I".:
... ........ - -- Flu ,125 ksi
" ::::-----.---
'0 40
....
<Jl
:l
\\l~ Fey
30 ksJ
100 Flu ,95 ksi
Flu. 90 ksi
--------
. /
-~
"3 ..........
"8 30
25 ksi_
........ I I I
~............... ~
20 ksi
~ .............. I I
--
15 ksi
g' .......... 50
'i5 20 o 10 20 30 40 50 60
..........-. 10ksi -
~ ........ ~
c
ell
CD
10
- 5 ksi
Fig. C4. 11 Bending modulus of rupture for round alloy-steel tubing.
Oil
lL..o i
10 20 30 60 70 80 90
Since the allowable or failing bending Therefore we will select a tube from Table
stress is a function of D/t, and since we do C4.3 that has a I/y value in the region of .30
not have a tube size, the design or solution to .37.
procedure is by trial and error.
Try 2-3/4 - 058. I/y =
.3233, D/t = 47.4.
Observation of the modulus of rupture From Fig. C4.12, Fb/Ftu =
0.90. Then Fb = .90
curves show that as D/t increases Fb decreases. x 77000 =
69300 psi. Then Ma Fb I/y =69300 =
This is due to the fact that failure in bending x .3233 =
22400. This is less than the design
is a local failure and the thinner the wall and bending moment of 24000 so this tube is weak.
larger the diameter, the lower the buckling or
crushing stress. However, the larger the D/t Try 3-058. I/y =
.3868, D/t 51.7 =
value the greater the section modulus I/y of Fb/Ftu =
.885 , Fb =
.885 x 77000 68000 =
the tUbe, which means increasing bending =
Ma Fb I/y = 68000 x .3868 26300 =
resistance. Thus we have two influences which M.S. =
(26300/24000)-1 = .09.
act oppositely relative to effecting the bending
strength. A study of other tubes in Table C4.3
shows that no other tube would be lighter in
There are many ways of guessing a tube weight.
size for checking purposes. In this example
problem we will assume two values for D/t and Tube weight =
30 x .5361 x .101 1.70 =
see what I/y would calculate to be. The two lbs., as against 2.22 lbs. for the alloy steel
values of D/t will be 45 and 25. heat treated to 220,000. Thus aluminum alloy
tubes from a weight standpoint usually yield
Consider the material alloy steel Ftu=220000:- results better than most materials. This
conclusion applies to only low temperatures,
For D/t = 45 from Fig.C4.11, Fb=232000 below 2500 F, as aluminum alloys lose strength
Then I/y = M/Fb = 24000/232000 = .103 rapidly for temperatures above 250 0 to 3000 F.
For D/t = 25, Fb = 266000 The student should calculate the lightest
Then I/y = 24000/266000 = .089 titanium tube and the lightest magnesium tube
using Figs. C4.14 and C4.13 respectively and
Therefore we will refer to Table C4.3 and compare the weight results with the steel and
select tubes that have an I/y value near the aluminum as found above •
•089 to .103 range and then find their true
bending strength. Table (A) shows the ULTIMATE TORSIONAL
selection and the necessary calculations,
using Fig. C4.11. STRENGTH OF ROUND TUBES
C4. 19 Torsional Modulus of Rupture.
Table A
In Article A6.2 of Chapter A6, the torsion
Tube Size IIY Area D/t Fb 1n..I/y (Ma/M)-l
M=~u M. S. = formula for circular sections, f s = Tr/J, was
1-7/8 -.035 .09136 .2023 53.6 227000 20800 -0.13
derived. This equation assumes the maximum
shear stress on the cross-section of a round
1-3/4 -.049 .1083 .261835.7 248000 26400 +0.10 bar or tube does not exceed the proportional
1-5/8 -.049 .0928 .2426 33.15 250000 23200 -0.04 limit of the material, or the stress variation
is linear as shown in Fig. C4.15 and this
The lightest available tube with a situation exists under the flight vehicle limit
positive margin of safety is 1-3/4 - .049 and loads. Before a round bar made of ductile
its weight for a 30 inch length is 30 x .2618
x 0.283 = 2.22 lbs.
Consider the tube made from 7075-T6 aluminum
alloy material which has a Ftu = 77000.
For D/t =
60, Fb/Ftu = .84, thus Fb = .84 x
77000 = 64700
I/y =M/Fb = 24000/64700 = .37
Fig. C4.15 Fig. C4.16
For D/t = 30, Fb/Ftu = 1.045, thus Fb = 1.045 x
77000 = 80500 material fails in torsion, the shear stresses
I/y = 24000/80500 = .30 fall in the inelastic or plastic range and
STRENGTH & DESIGN OF ROUND, STREAMLINE, OVAL AND SQUARE TUBING
C4.18 IN TENSION. COMPRESSION BENDING, TORSION AND COMBINED LOADINGS.
-
I--- r-....
) r-.... I- " I'
t"':xl
otzJ
I'
90
"
2
90 " t;0-3
5251
I, r.....
......
" " r-..
.......
I
·0
~52
70 5 70 5
I' 1'0
I' r-.... 1'0
50
r-.... 50 ....... 10
10 .....
30
o 10 20 30 40
Oil
50 60
I-....
70
~
80
20 30
o 10 20 30 40
0/,
50 60
j--.
70
- 80
20
Fig. C4.23 Torsional modulus of rupture - Fig. C4.24 Torsional modulus of rupture -
alloy steels heat treated to Ftu = 240 ksi. alloy steels heat treated to Ftu = 260 ksi.
C4.21
lro .0
0
II.
112
'14
~
112
30 L2' 30
20 20
10
10
• t~f'£lOlN~GETH~'~'''~T13N~GE'!l'3N~'E'E~fGf§f§iEiElE~f§3
• ,. 2. 30
0/.
. so . 7• 10
0
0 10 20 ,0 '0
0"
'0 60 70 .0
Fig. C4.25 Torsional modulus of rupture - Fig. C4.26 Torsional modulus of rupture -
2014-T6 aluminum alloy forging. 2024-T3 aluminum alloy tubing.
101A-,. ruetNG
Ftu '62 kSI
L'D
~
'14
.: '0
'14
112
,,,
1/2
2D
20
,.
10 ID
'0
,0
10
o
o
o '0 20 30 ..
010
so 60 70 BO
o 10 20
'0 .0 so 60 70 '0
D/I Fig. C4.28 Torsional modulus of rupture -
6061-T6 aluminum alloy tubing.
Fig. C4.27 Torsional modulus of rupture -
2024-T4 aluminum alloy tubing.
lro 50 lro
• ,I.•
. '14
,/2 .. ,/2
20
,. I.
20
2.
10
o tmtEtmtE±t8EttmlttmlE::E±tttE
o 10 20. JO 40 30 eo o
o I. . 7D
0/,
70 20
'" 50
Fig. C4. 29 Torsional modulus of rupture - Fig. C4.30 Torsional modulus of rupture -
7075-T6 aluminum alloy rolled rod. 7075-T6 aluminum alloy forging.
STRENGTH & DESIGN OF ROUND, STREAMLINE, OVAL AND SQUARE TUBING
C4.22 IN TENSION, COMPRESSION, BENDING, TORSION AND COMBINED LOADINGS.
The lightest aluminum tUbe with a positive equation C4.14 is not used.
margin of safety is 2 - .049. The weight of a
10 in. length = .3003 x .101 x 10 =
.303 lbs.
The lightest steel tube is 1-1/4 - .035 and its 1.1
weight is 0.1336 x .283 x 10 = .378 lbs.
Although the steel is heat treated to Ftu = 1.0
200,000, it still is heavier than the aluminum
alloy tube. .9
.8 " . , , , , ,
STRENGTH OF ROUND TUBES r» .• " , •.• " '1
.7
UNDER COMBINED LOADINGS
Reference should be made to Artiale Cl.15 .6
Rc
of Chapter Cl for general explanation of
.5
stress-ratios and interaction equations as
used in determining the ultimate strength of .4
structural members under combined loadings.
.3
C4.22 Combined Bending & Compression.
.2
In tUbes subjected to combined bending .1
and compression, the stress due to compression
is uniform over the cross-section whereas the
bending stress is not uniform over the cross- .1 .2 .3
section. The following stress ratio equation
is possibly somewhat conservative but is C4.23 nIustrative Problem InvolVing Combined Bending
recommended by (Ref. 1). and Compression.
Rb + Rtl.· 6 =1 (C4.15)(Ref.2)
1.1
Bt
1.0
wj2 = 10 x 860 = 8600 .9
L _ 40 L 1
2j - 58.6 = .683 sec 2T = L .8
cos 2j
.7
.9'--~
The student should notice that the maximQ~
bending moment of 2490 in. lb. is 24 percent
greater than the primary moment which equals
wL 2/8 = 10 x 40 2/8 = 2000. The lateral de- .7
flection at the midpoint of the tube thus equals
490/400 =
1.22 inch. of .6
.5
The secondary moments due to lateral de-
flection do not vary linearly, so if design
loads were increased the calculation of the .4
maximum bending should be repeated instead of
.3
assu~ing that the moment would increase directly
as the applied load to the beam. .2
for various values of Rb and Rst are also shown Te = MIl + (TIM)" - -- - - - - - - - (C4.21)
on the figure.
Having the value of Te , select tube sizes
The expression for Margin of Safety is, that will develop this torsional moment Te as
was done in Problem 2 of Art. C4.21. These
sizes are then checked for combined bending and
1 - - - (C4.17)' torsion as illustrated above in the example
problem.
ILLUSTRATIVE PROBLEM 1 C4.24a Ultimate Strength in Combined Compression,
Bending and Torsion.
A 1-1/2 - .• 058 round steel tube (Ftu =
125000) is 30 inches long. It is subjected to The interaction equation for combined
an ultimate design bending moment of 10,000 compression, bending and torsion from (Ref. 1)
in. Ibs. and a torsional moment of 6000 in. is,
Ibs. Find the Margin of Safety.
procedure.
.6
Let f s (max ) =Ter/2I (C3.18) ~ .5
+-L
,.-t
.1 +
- - (C4.20) t
1.0 .9 ,8 .7 .6 .5 .4 .3 .2 .1 0
Equating C4.20 and C4.18 and solving for Te , Rc
C4.25
tfoe-- 25" 1-1/8 -.049 2024-T3 Rd. TUbe~ o.,,-Q"* The interaction curve from this type of
- .'t~"'J
combined loading from (Ref. 2) is,
Fig. C4.38
Rb + R~ =1 (C4.24)
Solution:
1
M.S. =--;:::.=:=== - 1 (C4.25)
The maximum bending moment due to symmetry J Rb + R;
will occur at midpoint of tube. For a beam
column carrying a uniform side load with no end The allowable flexural or transverse shear
moments, the maximum moment is given by the stress is taken as 1.2 times the allowable
following expression. (See Chapter A5, Table torsional stress of the tube (Fst).
A5.1) .
ILLUSTRATIVE PROBLEM
M
max. = Wj" (1
- s e c 2j
L)
A 1-1/4 - .058 round aluminum alloy
j = J¥ = 10,300,000 x .0240 =
1600
2024-T3 tube is used as a simple beam and
..; 154.7 = 12.43 carries an Ultimate design load of 600 lb.
as shown in Fig. C4.39.
STRENGTH & DESIGN OF ROUND, STREAMLINE, OVAL AND SQUARE TUBING
C4.26 IN TENSION COMPRESSION BENDING TORSION AND COMBINED LOADINGS.
fs = 2 .2172
x 300
= 2760 psi.
_ f s _ 2760 _
C4. 27 Ultimate Strength in Combined Tension and Torsion.
Rs - Fs - 34000 - .081
1 1
The interaction equation for this type of
M. S. =--;::::::::;::::=::;: - 1 = ---;::-==;;===~ 1 loading as presented in Ref. 3 is,
VRb + R~ V.915 2
+ .081
2
= .09 - - - - - - - - - - - (C4.27)
Fig. 1
T t
30"
T
20
1 -l
(6) The loads shown in Fig. 4 are to be
30" 30"
transmitted to the support at the left, or
1000# 2000#
in other words, a cantilever structure.
Fig. 2
The top and bottom longerons should be The problem is to design the lightest
truss configuration using round tUbes of
continuous members. Minimum size 3/4 - alloy steel Ftu = 95000 and welded to-
=
.035. Use C 2 for web members and C = gether at the truss joints. Use C 1.5 =
1.5 for longerons. Material (chrome-moly for end fixity of all members. There are
steel Ftu = 95000 psi).
STRENGTH & DESIGN OF ROUND, STREAMLINE, OVAL AND SQUARE TUBING
C4.28 IN TENSION COMPRESSION BENDING TORSION AND COMBINED LOADINGS.
Fig. 6
From Chapter A18, the equation for the Fig. C5. 1 (Ref. 1) Transition from column to plate as
elastic instauility of flat sheet in compression supports are added along unloaded edges. Note changes in
is, buckle configurations.
2
n kC E t Fig. C5.2 gives curves for finding the
Ocr =----- (_)2 - - - - - - (C5.l) buckling coefficient k c for various boundary
12 (l-ZJ~) b or edge conditions and alb ratio of the sheet.
Where k c = buckling coefficient which depends The letter C on edge means clamped or
on edge boundary conditions and fixed against rotation. The letter F means a
sheet aspect ratio (alb) free edge and SS means simply supported or
hinged. Fig. C5.3 shows curves for k c for
E =modulus of elasticity various degrees of restraint (E) along the
Ve = elastic Poisson's ratio sides of the sheet panel, where E is the ratio
of rotational rigidity of the plate edge
b = short dimension of plate or loaded stiffener to the rotational rigidity of the
edge plate.
t = sheet thickness
Fig. C5.4 shows curves for k c for a
C5. 3 Buckling Coefficient k c flange that has one edge free and the other
With various degrees of edge restraint. Fig.
Fig. C5.l shows the change in buckled C5.5 illustrates where the compressive stress
shape as the boundary conditions are changed varies linearly over the length of the Sheet,
on the unloaded edges from free to restrained. a typical case being the sheet panels on the
upper side of a cantilever wing under normal
In Fig. (a) the sides are free, thus sheet flight condition. .
acts as a column. In Fig. (b) one side is
restrained and the other side free, and such a Fig. C5.6 gives the k c factor for a long
restrained sheet is referred to as a flange. sheet panel With two extremes of edge stiff-
In Fig. (c) both sides are restrained and this ener, namely a zee stiffener which is a
restrained element is referred to as a plate. torsionally weak stiffener and a hat section
C5.1
BUCKLING STRENGTH OF FLAT SHEET IN COMPRESSION, SHEAR,
C5.2 BENDING AND UNDER COMBINED STRESS SYSTEMS
16 /5
I
I
I C
A C I I ss
8
C
I
14 f - - -
I
/4 1-----1 13
I I'
l-
,, I
I
I
Sf
c SS I IF 0
C
I
12
(:~::3
/2 \
\I 1--------. - /I
1\
....
l- "'''''''R')':¥\ \
~ FEE
I T
6 /0 r---- \ ~La4OED E06ES CLAMPED
\1 \r".
II
, \
J
1\ 9
V
,I \ \ ,
---- - LOADED ED6ES CLAMPED \
I- I'\ \
LOAOED £06E$ SIMPlY 1\ "- ......... ,
,
TYPE OF
""'" SlIPPORrED
S.fr~~T
8 :--- f'-_ I
~ 11\~
, I, '--
. . . --r--__ ----- II.WLOADED
E06ES
7 f-- fA /r-.... /'
-I aJ
I-
\
' ....,
A
~ I"---"
l'-- r:::- ~
~
50
20
W ~ ~-,
6
~
.~ --~
8
/0
I- ..., 5 5
~~
~'
\
I
I- \
,, \
\
\
I
2 \, "
I-
~ " t--'------
~'
- --_
-- ----- ----
D
2
,I I I
o /
" % 8 12 16 20 M 2.8 .J2 36 4.0
7,---.,.---------------.,
I
, \ I \
I?
---++ \ ~ ·1
16
,
14
--1-- I ,
5. 1-----111--+-
13
--1- 1
/2 1--+~+m\*-----,7-J'-A-~"k----'-~.-IC-~..,-----l
;
I--If----+-\\\-\--"'-+----+I~d_''"'"'__+-'''f=--+~ :
k.
II
5
10
.9
2
B
" ~ "-
.7
.5
D 2
:--
:--
.I
.5
0 1....,..._--T---3'i---4:T---5r--~<X>
o }t'-'......'........
"0 .5 1.0 2.0 2.5 30 :J5 4.0 4.5 50 o/b
a~
Fig. C5. 5 (Ref. 1) Average compressive-buckling-stress
coefficient for rectangular flat plate of constant thickness
Fig. C5.4 (Ref. 1) Compressive-buckling-stress coefficient with linearly varying axial load. k c n 2 E 11.-) 2
of flanges as a function of alb for various amounts of edge a av - av 'b'
rotational restraint. 12 (1 - V e 2 )
C5.3
31----+------+
Ocr = n'( 11. kc E
(~)2 __ - - __ (C5.2)
l2(1-Ve 2 ) b
1.4 ..---,---,----r---,...--"""T"----,----r-----r-----,-----, n
121----t---+---t---+---+---+-----l---+---:;;~.,.jL'-------I
8 1------1----+----+----7f~~~~---+---+---+---+-----l
°crP O • 7
or
Fcc/F o. '7 6 I---t---t-------,~n,£-+_--+---+-----l
ff.1---+--:Q;"L--t----+---+----t-------I
.21--~L--+---t-----t----+----t----t------t---+_---I
OIlW.LJ.ll.uL-l..---l_.l.-....I---L--l_.l.-....I---l..--l_.l...-....I----L..--'_.l--....I---"--'
o .2 .4 .6 .8 LO L2 /4 2.0
2
kc n E t 2
12 (1 - Ve 2 )O (j)
0.7
1.4 r---,.---,.---,.---,.---,...---,---,---,---,----, n
2
1.2 1---+---+----+---t-----j---j---+---+---*<!C---:::7'l 3
.8 1---+---+----+---.r,~,;£-..i_;."r;£--+----+----+----+----1
Ocr /o O.7
or
F ccrlF 0 • 7 .6 1---+---+-----¥-~hLf----j---+--___1
.4 1---+---""~--+---+-----1---+----1
.21-----,.p.---+----+---I------1r----+---+----t----+---i
Ve = 0.3. Find the buckling stress ocr. Thus a further correction must be made for
alclad sheets because of the lower strength
Solution: We use Fig. C5.8 since it covers clad covering material. Thus the buckling
the boundary conditions of our problem. The stress for alclad sheets can be written:
parameter for bottom scale is,
k n 2E (C5.3)
c
- - - - - - - - (A)
ft/2
~
t
For alb = 9/3 =
3, we find k c from curve ,~
(c) of Fig. C5.2 equals 4.0.
The use of Fig. C5.8 involves the use of J -*--
00.7 and n the Ramberg-Osgood parameters. tft/2 Cladding
Referring to Table Bl.l of Chapter Bl, we find Fig. C5.9
for 2024-T3 aluminum alloy that 00.7 = 39000
and the shape factor n = 11.5. Reference 1 gives simplified cladding
reduction factors as summarized in Table C5.l.
Substituting in (A):- Thus the buckling stress for alclad sheets is
determined for the primary strength properties
2
4.0 n x 10,700,000 (.0~4)2 = .98 as normally listed for such materials as
illustrated in the two previous example
12 (1 - .3 2) 39,000 problems. The resulting ocr is then reduced
by use of equation C5.3, using values of
From Fig. C5.8 using .98 on bottom scale
and n =
11.5 curve, we read on left hand scale ~ from Table C5.1.
that Ocr/crce? = .84.
C5.7 Buckling of Flat Rectangular Plates C5. 8 Buckling of Flat Plates Under Bending
Under Shear Loads. Loads.
The critical elastic shear buckling stress The equation for bending instability of
for flat plates with various boundary flat plates in bending is the same as for
conditions is given by the following equation: compression and shear except the buckling co-
n" k E efficient kb is different from k c or k s ' When
7 = __'__S (.!.-) "
- - - - - - - (C5.4)
a plate in bending buckles, it involves
cr relatively short wave length buckles equal to
12 (1 _ Ve ") b
2/3 b for long plates with simply supported
Where (b) is always the shorter dimension of edges (see Fig. C5.14). Thus the smaller
the plate as all edges carry shear. k s is the buckle patterns cause the buckling coefficient
shear buckling coefficient and is plotted as a kb to be larger than k c or ks '
function of the plate aspect ratio alb in Fig.
C5.11 for simply supported edges and clamped
edges.
...----,,.....-...,.---r---,----r----r----r----r---,---,n
.6 t---+--+------i---+--+---i---+--+---:7'~~--i
.4t----+--_+_--+--~f7"""""''.A;r-_+--_+_--4---t-----1
'T:crl 0 0 • 7
or
Fsc/Fo. 73 1----1---+--~~~_+--_+--+-
2t---+----:#7>'-----if--_+---+---t--
./
.I .2 .4 .5 .6 .8 .9 1.0
k s n" E t"
2 (-)
12 (1- Ve )00 • 7 b
Fig. C5. 13 (Ref. 1) Chart of Nondimensional Shear Buckling Stress for Panels With
Edge- Rotational Restraint. 71 = (Es/E) (1 .. Ve ")1(1 - V').
60
15..------r---~----r---...,....----.
56 -
131------+-\-----+----+----+-----1
52 f------
48
~..
/
//////////////
////////
,
.
or
./ /
\\ J::.
~r;/
44
.......
~ ~ P::
--
'
/It-----t--f---'~+_----+---+_--______l 40
v
~\'- I "'-.
"- ..........
r--- ---
v-
CD
100
50
k. 36
t'--" ~ -
L./ "-
.~ ----
-
I
- 20
\, --- - -
9t-----fV----+--f---+----+--I----1 10
V) t--
./ 5
~~~
'--- 3
I
24 . o
7t-----+-~;:___+_----jL-+_--___j~--+_I i i
E.
._~--
I
16 I
Link to C5.8....
9 II 13 15 17 19 21 23
5 O~u..L..u..L..~/---..l:2~---:!:---~-----J5 3 5 7
14
o/b
Fig. C5. 11 (Ref. 1) Shear-Buckling-Stress Coefficient of Fig. C5.15 Bending-Buckling Coefficient of Plates as a Function
Plates as a Function of alb for Clamped and Hinged Edges. of alb for Various Amounts of Edge Rotational Restraint.
BUCKLING STRENGTH OF FLAT SHEET IN COMPRESSION, SHEAR,
C5.8 BENDING AND UNDER COMBINED STRESS SYSTEMS
Fig. C5.15 .3
1.0 Combined Bending
& Long. Compression .2
.9 Rb :1..76+ R
c .1
.8
o .1 .2 .3 .4 .5 .6 .7 .8 .9 1.0 1.1
.7
R s , (Rs + Rst), Rst
Rc .6
(C5. 9) .9
1.8
~C~~~~~~JI
1.6
.._ 4 .. ·4 ...... .., ... _ ...
«(~~~~~. -=~(
1.4
1.2
.... 1.0
'"
p:;
1-0
0
.8
'"
p:;
.6
.4
.2
o
-2.0 -1. 8 -1. 6 -1. 4 -1. 2 -1. 0 -.8 -.6 -.4 -.2 o .2 .4 .6 .8 1.0 1.2
RL Longitudinal Tension RL Longitudinal Compression
and the rib flanges will be con- The Rc + R~ = .431 + .309 2 .526. Since =
servatively assumed as simply the result is less than 1.0, no buckling
supported. (F c is same as ocr) occurs.
cr
2
The M.S. = -----;.===== -
2
n kc E t 2 1
FCcr = 12 (1-V 2 ) ("b) Rc + J R~ + 4R~
e
(See Eq. C5.1) = .431 + J .43122 + - 1 = .69
4 X .309 2
alb of skin panel = 15/5 = 3
Fig. (a)
Load Case 3.
From Fig. C5.2 for Case C,
we read k c 4.0 = P~ = 700, P2 = 500, P3 = 100 lb.
2
F - n
ccr-
X 4.0 X 10,700,000 (.0535 )2
12 (1 - 0.3 2 )
= 1900 psi The two loads P3 produce bending and
flexural shear on the beam. The bending moment
produces a different end compressive stress on
Since Fccr the buckling stress is greater the three sheet panels since the bending moment
than the applied stress f c , the panels will is not constant over the panel moment. To
not buckle. simplify we will take average bending moment on
the panel.
M.S. = (Fccr/f c ) - 1 = (1900/819) -1 = 1.32
Mx(av) = 200 x 52.5 = 10500 in. lb.
Load case 2.
f c due to this bending =MxZ/l x = 10500 x
p~ = 700 lb., P2 = 500, P3 =0 . 4.233/49.3 = 903 psi.
The two loads P2 acting in opposite Total f c = 903 + 819 = 1722 psi.
directions produce a couple or a torsional
moment of 500 X 16.5 = 8250 in. lb. on the Rc = fc/F c cr = 1722/1900 = .906
beam structure, which means we have added a
pure shear stress system to the compressive The two loads P3 produce a traverse shear
stress system of Case 1 loading. =
load V 200 lb. The flexural shear stress
must be added to the torsional shear stress as
The shear stress in the top panels A, B found in Case 2 loading.
and Cis,
Due to symmetry of beam section and P3
fs = T/2At = 8250/2 x 138 x .035 = 854 psi. loading the shear flow q at midpoint of sheet
panel (B) is zero. We will thus start at this
(Where A is the cell inclosed area) point and go clockwise around cell. The shear
flow equation (see Chapter A15) is,
The shear buckling stress is
20 7.20
n 2
k E 41!50~~ 10.94
Fs - s (.!/ - - - - (See Eq. C5.4) (1 --+ 2 i 3)l / / !
~~.5"'tt::jI;J
cr 12 (1 _ V e 2 ) b
The shear flow q on panel (A) varies from (4) A sheet panel 5" x 12.5" x .051 has all
4.20 to 7.20 or the average q = (4.2 + 7.2)/2 edges simply supported. The panel is
= 5.7. Thus the average shear stress is subjected to combined compression and
5.7/.035 =
163 psi. It is in the same shear loads which produce the following
direction as the torsional shear flow and thus stresses:-
is additive.
fc = 2400 psi, applied normal to 5" side.
Total f s =163 + 854 =917 psi
fs = 2800
psi. Will the sheet buckle
Rs = fs/F scr = 917/2760 = .332 under the given load system if made of
aluminum alloy 2024-T3 material. What
R; =1, Subt.:- .906 + .332 = 1.016,
2
Rc + is the margin of safety.
since the result is greater than 1.0, initial
buckling has started. The margin of safety is (5) If the material in problem (4) is changed
slightly negative and equals, to alloy steel Ftu = 95000 psi, what
would be the margin of safety. If sheet
M.S.
2
=------:;::===:0=====::::0 1 = - .01 was heat treated to Ftu =
180,000, what
.905 + J .905 + 4 x .332
2 2 would be the M.S.
In this example problem, the panels were (6) A 3" x 12" x .040" sheet panel is sub-
assumed simply supported, which is conservative. jected to the following combined stresses.
Reference to Fig. C5.6 shows that k c could be f c = 3000, fb = 10000, f s = 8000. The f c
assumed higher as the panel is riveted to a and fb stresses are normal to the 3" side.
zee shaped stringer which has some torsional If sides are simply supported, will panel
resistance and thus panel is not free to buckle if made of 7075-T6 aluminum alloy.
rotate at its boundaries. What is M.S.
Panel (C) is less critical because the What will be the M.S. if material is
flexural shear is acting opposite to the changed to Titanium Ti-8I1n.
torsional shear stress, thus f s total = 845 -
163 =
682 psi. The Rs = 682/2760 = .246. References:-
Rc + R; = .905 + .246 2 = .966. Since the (1) NACA Tech. Note 3781, 1957.
result is less than 1.0, panel will not buckle.
(2) ANC-5 Amendment 2. Aug. 1946.
Panel (B) carries a small shear flow,
being zero at center of panel and increasing (3) NACA ARR.No. L6A05
uniformly to 1.5 lb. per inch at the edges,
and flowing in opposite directions from the (4) NACA ARR.No. 3K13
centerline. Thus transverse shear will have
negligible effect. Thus Rs = 854/2760 = .309. (5) ANC-5 Revision of 1942
Rc + R; = .905 + .309 2 =
1.00, or panel General References on Theory
(B) is on the verge of buckling under the
assumptions made in the solution. (6) NACA Tech. Note 3781.
The simplest equal flanged member that can For formed angles, the flange width b
be formed is the angle shape. Other shapes extends to centerline of adjacent leg, but for
with equal flanges are the T section and the extruded angles, the width b extends to inside
cruciform section as shown in Fig. C6.1. edge of the adjacent flange or leg.
~rlange ~~lange ~
C6.3 Compressive Buckling Stress for Simple Flange-Web
Elements.
~
Angle
~
T-Section
~ciform
Section
The most common flange-web structural
shapes are channels, zees, and hat sections. A
flange has one unloaded edge free, whereas a
Fig. C6. 1 web has no free unloaded edge and thus has an
These sections can be considered as a group of unknown restraint on the boundary between the
long flanges, as illustrated, for the angle web and the flange. Fig. C6.3 shows the break
section in Fig. C6.2. Since the flanges which down of a Z section into two flange and one web
make up the section are equal in size, each plate elements.
flange will buckle at the same stress. There-
fore each flange cannot restrain the other and
thus it can be assumed that each flange is
simply supported along the flange junction as
illustrated in Fig. C6.2.
Q)
Q)
~ Flange
cr cr cr cr
Fig. C6.3
buckling stress for each element, the critical For the plasticity correction of shapes
buckling stress will be the smaller of the two. covered in Fig. C6.4 (Ref. 4), the plasticity
The buckling load based on the buckling stress correction for a flange free on one edge can
is not the failing load as more load can be be used with accuracy. Thus we can use chart
taken by the material in the corner regions in Fig. C5.7 of Chapter C5.to correct for
before local failure or crippling takes place. plasticity effects.
The subject of local crippling of formed and
extruded shapes is covered in Chapter C7. The parameter for bottom scale of Fig.
C5.7 is equation (A) divided by 0 0 . 7 , or
Using the moment distribution method or a 56100/39000 =
1.44. Using this value and the
step by step analysis procedure, several n = 11.5 curve, we read from Fig. C5.7 that
research studies have determined the restraint ocr/a 0.7 = 1.02.
factors between web and flange elements for
simple shapes like channels, Z, H, square tUbes Therefore the local buckling stress is
and formulated design charts for such shapes.
(Refs. 1 to 5 inclusive.) ocr =39000 x 1.02 = 39800 psi.
= .064/.064 = 1.0 2
tw/tf
ocr
=2.9 n12(1-.3
x 15,500,000 ( .064 )2 = 81200
2
1.436------ ps •
)
i
From Fig. C6.4, we read kw = 2.9
This stress is near the proportional limit
2
ocr
= 12k~ I-V
n E
2)
(~)2
bw - - - - (A)
stress so plasticity correction should be small
i f any.
e
2 ocr/a 0.7= 81200/n9500 = .68
Ocr = 2.9 n x 10,700,000 ( .064)2 - 56100 psi
12{1-.3 2 ) 1.436-
From Fig. C5.7, using n = 13.7 curve, we
This stress is above the proportional read ocr/00'7 = .68. Then ocr = 119500 x 0.68 =
limit stress of the material, thus a plasticity 81300, thus no plasticity correction.
correction must be made. The buckling occurs
on the flange. PROBLEM 4.
From Table Bl.l of Chapter Bl, we obtain The rectangular tube has the dimensions as
for 2024-T3 aluminum alloy:- 0 0 . 7 = 39000 and shown in Fig. (b). It is extruded from aluminum
n = 11.5. alloy 2014-T6. Determine the local compressive
buckling stress.
CG.3
7r------~-~____::___r--r_-_,
WEB
7
Kw
31---+-\\\'Irt--~rt+\-~rl~--+-~----i
2
"J;
~
WEB,b"
FLANGE,bf
L2
.2 /2
°cr = 12 (1 - V
2 ) 2
2
e 2) bw
Yl
~:::::t:==::t:===i====
I- is
2.0
i'
i4
/('"
4
3 L----l.----+---+----'\--+-'\-""',+---j
kr 31---+---+---\-l-~-l-~--+~----"*...----+------t
~-+-~-k:---""'Ik;::-....-t~
./1
--.l.-.~
.8 10 i2 14
t = tf = t w =tT; Ocr = 2 2
(Data of Ref. 12.)
12 (1 - V e ) bT
C6.4 LOCAL BUCKLING STRESS FOR COMPOSITE SHAPES
~fT
Solution:- In the design of rectangular tUbes, the
designer should select the tube thicknesses for
b =1 .08 = .92 ~~~O both long and short sides so that buckling
occurs on both sides, thus giving the lightest
h =2 .08 = 1.92
1:- 2.0 '~I section for buckling strength.
b/h = .92/1.92 = .479
As pointed out previously in this chapter,
tb/th = 1.0 Fig. (b)
the load on the member which causes local
From Fig. Cl.6, we read kh = 5.2. buckling is not the failing or maximum load for
a short length of the member. This local failing
2 or crippling stress is treated in the next
Ocr - 5.2
-
TI x 10,700,000 ( .04)2
12(1-.3 2 ) 1.92
= 21900 psi
• chapter. Since the buckling stress may fall in
the inelastic stress range, the buckles will not
entirely disappear when load is removed. Since
As shown in Fig. C6.6, buckling occurs on the h limit or applied loads must be carried without
side of the tube. The computed buckling stress permanent distortion, it is thus important to
is below the proportional limit stress, thus know when local buckling starts. For those
no plasticity correction. missile and space vehicles that carry no
humans, the factor of safety on limit loads is
PROBLEM 5. considerably less than for aircraft, thus the
spread between local buckling and local failing
Same as Problem 4 but change the thickness
strength becomes important in design.
of the h side to .072, but leave the b side
.04 in thickness.
C6.6 Buckling of Stiffened Flat Sheets Under Longitudinal
Compression.
Solution:-
In supersonic aircraft, it is important
b =1 = .856
.144- that the surface skin, particularly that on the
h =2 .08 = 1.92 wing, not buckle under flight conditions since
a buckled surface could effect the aerodynamic
b/h = .856/1.92 = .44-6 characteristics of the airflow around the wing,
tb/th = .04/.072 = .555 thus it is important to know when the skin or
its stiffening units initially buckle in order
From Fig. C6.6, kh = 4.3 to design so that such buckling will not occur
under flight conditions.
2
_ 4.3 x 10,700,000 (.072)2
ocr -
TI
12 (1-.3
2) 1.92 = 5860 0 psi.
Gallaher and Boughan (Ref. 6) and Boughan
and Baab (Ref. 7) determined the local buckling
This stress is above the proportional limit coefficients for idealized web, Z and T stiffened
stress thus a plasticity correction is necessary. plates. The results of their studies are shown
(Ref. 4) gives no value for a plasticity in Figures C6.8 to C6.12 and were taken from
correction but recommends the correction for a (Ref. 4). The initial local buckling stress
clamped long flange which is a slightly con- for plate or stiffener is given by the equation:-
servative correction. This plasticity
correction should also be used for the hat shape ks TI 2 E (.!a) 2
as shown in Fig. C6.7. Ocr 12 (l-))e ") bs - - - - - - - - (B)
Thus Fig. C5.8 of Chapter C5 can be used If the buckling stress is above the pro-
to correct for effect of plasticity. portional limit stress of the material, correct
for plasticity effect by using Fig. C5.8 of
From Table Bl.l, for our material 00.7 = Chapter C5.
53000 and n 18.5. =
Problem Illustrating Use of Charts.
The value of the parameter for bottom
scale of Fig. C5.8 is Ocr/OO.7 58600/53000 = = Fig. C shows a plate with idealized Z
1.10. section stiffeners. The material is 2024-T3
/f=.5
From Fig. C5.8, using n = 18.5 curve, we tf =.0625
read Ocr/00'7 =
.91. Therefore ocr 53000 x =
.91 =
48100. t w =·0625
t s =·125
Thus by changing the long side of tube
from .04 to .072, the buckling stress was in-
creased from 21900 to 48100 psi.
~ b s =4.0 .1
Fig. (c)
C6.5
6
7,...-----r---...,---....,...--..,.---.------,
Bf./CKLING OF SKIN
RESTRAINED BY STIFFEN~~"
5
t•
6 BllCKLlNG OF $rtFFENER
RESTRAINED BY SKIN
.79
.50
t,
4 r---1---+--'\'~rr---"...-'~:__-1 b,;
k.
3
.~,
.5
2.
.5
b. .4
r
Is
'
's .5
2.0 b.
/8 0
0 .2 II 6 .8 1.0 1.2
/6
br
/~
/2
Ii;
/0
.9
.8
Fig. C6. 9 (Ref. 6) Z-section stiffeners.
.7.6 twits = 0.50 and 0.79 .
.5
.8 I1J /2
6 , - - - - , - - ,-------,----.----,---,
!6.
Is
5 /0 --=F'-=+---+----j
Web stiffeners. O. 5 < twits < 2.0
.65
Fig. C6. 8 (Ref. 7) Compressive-local-buckling coefficients for 4
infinitely wide idealized stiffened flat plates.
ocr = 12 (:
k n"E
x V ")
(tb:)\" 5 r----J---'----+-\----'-P~----I
.5
e
2. _--f',;:_""".5
4
.5
7,...----,----"T---,----,..--....,...----,
o0~=w..l,-----l,---.6L...-----l.8---I..L:o,------JI2.
b,
6 f--=-+-"""'=--t----+----j br ~
bs
0.3,.4, .5
Fig. C6. 10 (Ref. 6) Z-section stiffeners.
5 twits = O. 63 and 1. O•
.-' 7
~
t
15
k. bf
6
i-i-=r-='::t====t:==1 b.
10 0.4,.5,.6
3 .5
.-' 5
2 !I
tr k. .6
.5
3 f---+--r-----,------'l:+--'<--'!4
2.
.6 .8 lO l2
Fig. a
Basic Angle
t
Basic Angle
A n g l e - e 4such
unit I
I
units
ilf·o-
I
Basic Angle
Unit. Two Unit. One Unit. No
edges free. edge free. edge free.
where,
As the load on the section is increased, Fcs = crippling stress (psi)
the buckles on the flat portions get larger but Fcy = compression yield stress (psi)
most of the increasing load is transferred to E = young's modulus of elasticity in
the much stiffer corner regions until the stress compression (psi)
intensity reaches a high enough value to cause b'lt = equivalent bit of section =
sufficient deformation to cause failure. (a + b)/2t
Ce = coefficient that depends on the
A theoretical solution for the local degree of edge support along the
crippling stress for all types of shapes has edges of contiguous angle units.
not been developed as the boundary restraint Specifically they are:-
between flange and plate elements is unknown Ce = 0.316 (two edges free)
and also the manner in which the stress bUilds Ce =
0.342 (one edge free)
up in the corner regions is not well understood. Ce = 0.366 (no edge free)
Consequently, the methods of solution are semi-
empirical in Character, and the results of such The crippling stress for angles, channels,
methods have been sufficiently proven by tests. zees and rectangular tubes can be determined
Two methods of calculating crippling stresses directly from use of equation C7.1. The
will be presented in this chapter. crippling load on an angle unit is then,
C7.2 METHOD 1. THE ANGLE METHOD, or the (C7.2)
Needham Method.
where A is the area of the angle.
This method which will be referred to as
the angle method or the Needham method was The crippling stress of other formed
C7.1
C7.2 CRIPPLING STRENGTH OF COMPOSITE SHAPES AND SHEET-STIFFENER PANELS IN COMPRESSION
'-6
-
/" <'" TWO EDGES FREE _
0.08
- -
V V
1.6 /' "- ONE EDGE FREE _ I \
/
/' V
NO EDGE FREE ,7 60
a i
/ / /" 0.07 lI
.-;-T ._ _ -
/ /' /
1.4
/
/
1/
/
/
I 40
b-J
/ / 0.06 -
1.2 I / / I 20 \\
Pcc /
/
0.05 1\\\
2
t. .JFc71.0 /
I00 Fcc \'\
f
/
A ~ \l\\
"/ 0.04
0.8
f 80
t2
I\\C\ NO EDGE FREE
~~
TWO EDGES FREE
/
~-t
0.6 60
t::::: :::::--
'< I'-.::
~
/ 0.02
---- ::::::::: f2:: ~
0.4
/
/
~b---J
40
0.01
-
/
0.2 20
1/ o 0 10 20 30 40 50 60 70 80
1/ b a+b
o -:t="2t
o 10 20 30 40 50 60 70 80
5l = .9±.b.. (Note: Fee same as F es )
t 2t
Fig. C7.3 Dimensionless Crippling Stress vs. b'/t (Ref. 1)
(Note: Pee same as Pes)
Fig. C7.4 Dimensionless Crippling Load vs. by'lt (Ref. 1)
C7 4 CRIPPLING STRENGTH OF COMPOSITE SHAPES AND SHEET-STIFFENER PANELS IN COMPRESSION
~-+i-i-f++-
alclad Sheets, the clad covering acts to reduce
L
Bas ic Section 1 Cut
Cut
I
4 Cuts
the value of buckling stress and thus a
n
correction factor must be used to take care
of this reduction in strength. This correction
from (Ref. 3) is,
2 Flanges 8 Flanges
g=2
3=g 12 =g
where,
+
Straight Unloaded Edges
T-Section Cruciform H-Section °cl = cladding yield stress
ocr = buckling stress
T Icut f = ratio of total cladding thickness to
total thickness. f = 0.10 for alclad
2024-T3 and .08 for alclad 7075-T3.
Basic Section o Cuts 1 Cut
4 Flanges 6 Flanges C7.8 Maximum Values for Crippling Stresses.
g=3
4=g 7=g The cut-off or maximum crippling stress for
a composite section should be limited to the
values in Table C7.1 unless test results are
Fig. C7.6 Method of cutting simple elements to determine g. obtained to substantiate the use of higher
crippling stresses.
Table C7.1
values in Table C7.1 unless higher values can Type of Section Max. F cs
be EUbstantiated by test results. The cut-off Angles .7 Fey
values given in Table C7.1 are no doubt slightly
conservative. Design curves for equation C7.4, V Groove Plates Fcy
5 and 6 are given in Figs. C7.7, C7.8 and C7.9. Multi-Cqrner Sections, Including Tubes .8 F CY
Stiffened Panels F cy
Tee, Cruciform and H Sections .8 F cy
2 Corner Sections. Zee, J, Channels .9 F cy
.20
.15
.1
.1 .15 7 8 9 10
C7.5
1.5
: Fig. C7.8
, .1 'tiCurve for Plates, Tees, Cruci Form'
}- and H Sections.
1.0
.9 F cs = 0.67[(gt'"!A)(E/FCy ):I./2]o.4o
.8 Fcy
.7
See Table C7. 1
.6 for Cut-off Values
.5
F cs .4
F cy
.3 -
.25
.1_
.2
.15
-{"
Fig. C7.9
1.5 Crippling Stress Fcs for 2 Corner Sections,
Z, J and Channel Sections.
~ .4
F cy
.3
.25
.2
.15
.j
. 1 .t±±±tlttr:t:tdJ±tl±±!
.1 1.5 2 2.5 3 4 5 6 7 8 9 10 15 20 25 30 40 50 6070 8090100
(~) (~):I./3
2 t Ec
C7.6 CffiPPLING STRENGTH OF COMPOSITE SHAPES AND SHEET-STIFFENER PANELS IN COMPRESSION
C7.9 Restraint Produced by Lips and Bulbs. From Chapter C5, the compressive buckling
coefficient using Ve = .3 is 0.388 for a flange
Quite often in formed sections, the flange element and 3.617 for a plate element. There-
element which has a free edge is rather small fore,
in width as illustrated in Fig. a. Also for
extruded sections, a bUlb is often used as tL 2 > E(t f )2
illustrated in Figs. b. The question then 0.388 E (bL) = 3.617 bf (C7.10)
arises, is the lip or bUlb sufficiently large
enough to provide a simple support to the From equations C7.9 and C7.10, the follow-
adjacent plate element. Since the compressive ing relationship is obtained,
buckling coefficient for a plate element is
bL < bf
tL = 0.328 tf - - - (C7.11)
R
Lip Lip Bulb
J~_b Fig. b
Fig. C7.11 shows the results as a curve.
8
LIP PROVIDES AT LEAST
SIMPLE SUPPORT FOR FLANGE
w L~ ,~ ~
4.0 and 0.43 for a flange element, the use of a BUCKLIN/l
OF LIP v,uv, ~:;>Y
small lip or bUlb can increase the value of the b
f~V
6
L
coefficient considerably above 0.43 and thus t 5
produce a more efficient load carrying element. 4
If'
The problem of determining the dimensions of a
lip or bulb to give at least a simply supported 30 10 20 30 40 50 60 70 BO
edge condition to the adjacent plate element has bt
been investigated theoretically by Windenburg T
(Ref. 5). The results of his studies gives the
following design criterion. Fig. C7. 11 Minimum lip dimensions required for flange to
buckle as simply supported plate (Ref. 5).
In extruded sections, a circular bulb is
- - - - - (C7.8) often used to stiffen a free edge as illustrated.
in Fig. C7.10b. The moment of inertia of the
Where IL and AL are the moment of inertia and bulb area about the centerline of the plate
area of the lip or bulb respectively. (See element is,
Fig. C7 .10).
Fig. C7.10
Fig. C7.12 shows design curve representing
From Fig. C7.10a for the lip, AL = bLt, the above equation.
and IL =
tbL 8 /3.
--
In substituting these values in equation
(C7.8), the dimensions of the lip are expressed
as V
~
V
0.910 (b L )8 _ btL = 5 b: (C7.9) V
t
V
10 20 b 30 40 50
To determine bL and t, an additional re- '!f
quirement is specified, namely, that the
buckling stress of the lip must be greater or
equal to the buckling stress of the adjacent Fig. C7. 12 Minimum bulb dimensions required for buckle as
plate element. simply supported plate (Ref. 5).
C7.7
A plot of this equation is given in Fig. C7.7. a+b .725 + .725 = 14.5 Fig. c
The parameter for lower scale is, 2t 0.1
A Fcy 1./2
(gt2)(~) , where A = section area and g From Fig. C7.3 for b'/t = 14.5, we read
equals the number of flanges plus cuts, or Fcc/vFcyE = .045 (for one ed~e free)
g = 2 for an angle section. Substituting,
Then Fcs = Fcc = .045 x
( .093 2) ( 40,000 )1. /2 = 1.138
2 x .05 10,700,000 )40,000 x 10,700,000 = 29400 psi
From Fig. C7.7, Fcs/F cy = 0.40, whence Fcs = .045 x V67,000 x 10,500,000 =38200 psi
Fcs = .40 x 67,000 = 26800 psi.
Solution by Method 2.
Problem 3. Same as Problem 1 but change
material to Titanium Ti-8Mn. Fcy 110,000, = (.137)( 67,000 )1/3 = 10.17
Ec =
15,500,000. .05 2 10,500,000
C7.8 CRIPPLING STRENGTH OF COMPOSITE SHAPES AND SHEET-STIFFENER PANELS IN COMPRESSION
Solution by Method 2.
25,600 psi
l1.5
Area A = .373. g = number of cuts plus T
flanges or 4 + 8 = 12. 1 ~~Ic=l.o==:j:=:
.375
Area of angle (1) = .0309 =A as Section No.2. The area from that table is
0.113 sq. in.
Pcc = .0309 x 39,300 = 1215 lb.
Solution by Method 2 (Gerard)
Angle unit (2) Area = .0459 (no edge free)
For this material Fcy = 53,000, Ec
b'/t = (a + b)/2t = (.48+ .73)/.08 = 15.1 10,700,000.
From Fig. C7.3 Fcc/~ = .0475 The first question that arises is the bulb
size sUfficient to give an end stiffness to the
Fcc = )40,000 x 10,700,000 x .0475 = 31,100 psi (a) leg so that the bulb may be equivalent to
the normal corner.
Pcc = 31,100 x .0459 = 1428 lb.
In Fig. f, bf =0.78, hence bf/t = 15
Angle unit (3) Area = .0509 (no edge free)
Referring to Fig. C7.l2, we observe that
b' /t = (.73 + .605)/.08 = 16.7 for a bf/t value of 15 we need a D/t ratio of
at least 3.8. The D/t value for our bulb
From Fig. C7.3, Fcc/~ = .046 angle is (7/32)/.05 = 4.4, thus bulb has
sufficient stiffness to develop a corner. The
Fcc = .046 x )40,000 x 10,700,000 = 30,200 psi next question that arises should the bulb
angle still be classed as an angle section
Pcc = .0509 x 30,200 = 1540 lb. for which equation C7.4 applies or be classed
as a channel or 2 corner section With the bUlb
Fcs = ~Pcc/area acting as a short thick leg of the channel.
For this case, equation C7.6 would apply.
= (2 x 1215 +2 x 1428 + 2 x 1540)/0.255 =
32,800 psi The crippling stress will be calculated
by both equations.
Solution by Method 2 (Gerard)
By equation C7.4 or Fig. C7.7:-
...!..- (~)J./"
gt" Ec If bulb is considered as a full corner
then g = 4 flanges plus 1 cut 5.
g = number of flanges plus number of
cuts = 12 + 5 = 17.
SUbstituting in the above term,
From Fig. C7.7,
(17 .255 ) ( 40, 000 ) J. /" _
x .04" 10,700,000 - .573 FCS/Fcy = .82, hence Fcs = .82 x 53000 = 43500
From Fig. C7.7, Fcs/F CY = .895, whence By equation C7.6 or Fig. C7.9,
Fcs = 40,000 x .895 = 35,600 psi.
FCsmax . = .8 x 40,000 = 32,000 psi. Since Possibly the best estimate of the
crippling stress would be the average of the
this is less than the above calculated values, two above results or 40,300 psi.
it should be used.
In Table C7.l, the so-called cut-off
Problem 10. Find the stress for angles is .7 Fcy and channels .9
crippling stress for the Fcy • If we use the average value or .8 Fcy ,
extruded bulb angle shown
in Fig. f if material is it gives Fcs = .8 x 53,000 = 42,400 as
20l4-T6 extruded aluminum maximum permissible because of limited test
alloy. results on bulb angles.
This particular bulb For the case where the bulb or lip does
not develop the stiffness necessary to
section is taken from assume a full corner, then the bulb is only
Table A3.l6 of Chapter A3 Fig. f
C7.10 CRIPPLING STRENGTH OF COMPOSITE SHAPES AND SHEET-STIFFENER PANELS IN COMPRESSION
considered as an additional flange and the g Fig. b Sheet stress distribution before buckling
count would be four instead of 5, thus reducing
the crippling stress.
Sheet stress distribution after buckling
EFFECTIVE SHEET WIDTHS
C7. 11 Introduction.
2 = Rivet Lines
I-- w
Fig. C7.16
One
Rivet Line
Fig. C7.14
L
I-w ~ f+ w--l
For Case 1, find the crippling stress for
the tee section alone, assuming the vertical
stem of the tee has both ends simply supported.
For value of t in equation C7.15, use
(t s + tf)/2. The effective stiffener area
equals the area of the tee plUS the area of the
The crippling stress is determined for sheet of width w.
the stiffener alone. This stress is then used
in equation C7.15 to determine the effective For Case 2, determine the crippling stress
Widths w. The total area then equals the for the I section acting alone. Calculate w/2
stiffener area plus the area of the effective from equation C7.15 to include as effective
sheet width w. The radius of gyration should sheet area. The column properties should
include the effect of the effective skin area. include I section plus effective sheet.
Fig. C7.15 illustrates the case where C7.12 Effective Width WJ. for Sheet with One
stiffeners are fastened to sheet by two rows Edge Free.
of rivets on each stiffener flange. In this
case, the rivet lines are so close together In normal sheet-stiffener construction,
that the effective width w for each rivet line the sheet usually ends on a stiffener and thus
would overlap considerably. A common practice we have a free edge condition for the sheet as
in industry for such cases is to use the illustrated in Fig. C7.16a. The sheet ends at
effective width for one rivet line attachment
as per equation C7.16 to represent sheet width
to go with each stiffener flange. However, Fig. C7.16a
in calculating the crippling stress of the
stiffener alone, the stiffener flange which is
attached to sheet is considered as haVing a
thickness equal to 3/4 the sum of the flange
thickness plus the sheet thickness.
a distance b' from the rivet line. For a sheet
free on one edge, the buckling coefficient in
equation C7.13 is 0.43, thus equation C7.13
Staggered reduces to,
Rivet Rows Fig. C7.15
Fer = .387E(t/b' )2, and replacing b' by w , we
obtain,
- - - - - - - - - - (C7.17)
C7.12 CRIPPLING STRENGTH OF COMPOSITE SHAPES AND SHEET-STIFFENER PANELS IN COMPRESSION
Then the total effective sheet width for before the crippling stress of the stiffener
this end stiffener would thus equal w~ + w/2. is reached, which means the sheet is less
effective in helping the stiffener carry a
C7. 13 Effective Width When Sheet and Stiffener Have subjected compressive load. Thus, in general,
Different Material Properties. to save structural weight, structural designers
select rivet spacings that will prevent inter-
In practical sheet-stiffener construction rivet buckling of the sheet. In general, the
it is co~~on to use extruded stiffeners which rivet spacing along the stiffeners in the
have different material strength properties in upper surface of the wing will be closer to-
the inelastic stress range as compared to the gether than on the bottom surface of the
sheet to which the stiffener is attached. For wing since the design compressive loads on the
example, in Fig. C7.17 the stiffener material top surface are considerably larger than those
could have the stress-strain curve represented on the bottom surface.
by curve (1) and the sheet to which it is
attached by the curve (2). Now when the The following method is widely used by
stiffener is stressed to point (B), the sheet engineers concerned with aerospace structures
directly adjacent to the stiffener attachment relative to calculating inter-rivet buckling
line must undergo the same strain as the stresses. It is assumed that the sheet
stiffener and thus the stress in the sheet between adjacent rivets acts as a column with
will be that given at point (A) in Fig. C7.17. fixed ends.
This difference in stress will influence the
effective width w. Correction for this The general col~~ equation from Chapter
condition can be made in equation C7.17 by C2 for stable cross-sections is,
mUltiplying it by FSH/FST, which gives
- (C7.19)
w = 1.90t(FSH/FST) ( / / ) - - - (C7.18) Where C is the end fixity coefficient and
ST
varies from a value of 1 for a pin end support
Where fST is the stiffener stress and fSH is the to 4 for a fixed end support.
sheet stress existing at the same strain as
existing for the stiffener and obtained from a The effective column length L' = Live,
stress-strain curve of the sheet material. thus equation C7.19 can be written
70
I I
- - (C7.20)
I
Curve 2 f-
~
60 A
oW-- I I Let p the rivet spacing be considered the
c..
V> 50 IJ~ I Curve 1 column length L. Assume a unit of sheet 1 inch
o B..,. wide and t its thickness. Then moment of
Z
ji 40 I inertia of cross section = 1 x t 3 /12, and area
:::> f A = 1 x t = t. Then radius of gyration p =
o::c 30 0.29t. Then substituting in equation C7.20 to
.... / I
obtain the inter-rivet buckling stress Fir,
z
V>
20
I
V>
UJ I
~ 10 - - - - (C7.21)
V>
/ I
I
o1/
o .002 .004 .006 .008 .010 .012
For clamped ends C = 4, thus
STRAIN - IN./IN.
The effective sheet area is considered to The fixity coefficient C = 4 can be used
act monolithically with the stiffener. How- for flat head rivets. For spot welds it should
ever, if the rivets or spot welds that fasten be decreased to 3.5. For the Brazier rivet type
the sheet to the stiffener are spaced too far use C = 3 and for counter-sunk or dimpled rivets
apart, the sheet will buckle between the rivets use C = 1.
C7.13
gag
~ .
()
~I:I:II;
~" ~
'"C
~o
P="-:
'"C
r-<
~ :: .. Z
Fig. C7.19 (;)
~:,,:i;; 70
..,
u.l
Material Alum. Alloy Clad
~ :::: ..
Curve Designation Thickness Basis i;J
~4R ...• (CLAD) Z
..,
... 2024-T4 'C. 064
(;)
r"l'i ::z:
I
I 60 2024-T3
~
u> 2024-T36
u>
~~
r"l 2024-T6
~ ()
u> 2024-T81
e
~
2024-T86 ~
i
:
~),,~
;"!"1
...u 50
..:I 7075-T6
7075-T6 ~
.==~::~::~ P
IQ ::3
"==->"'"
~ ... t%J
u.l
;::::=~ ::;
-..--;::;s
,
E
ll:l ~
I '"C
40 t%J
:"ii
; ;; ~ : ...f:l u.l
==i(j ~
.....
I
ra. 30
20
_ _ _ _ _:..i
i,= .:I•••
:;;;;~;'
-==., -, ,_._--,
----~,
_.::
10
30 40 50 60
pit
I:=::
§~
C7.15
of the dimensions in Fig. C7.21 shows the usually of the flush surface type, either
effective width with each skin rivet line would countersunk or dimpled. If we make the skin
overlap slightly thQS we use only the width rivets of the countersunk type, the end fixity
between rivet line (see Fig. b). coefficient must be reduced to 1 to be safe.
Then the corrected p/t ratio to use with
For the web t = .064. Since the web has Fig. C7.18 would be (14 / Vl)p/t = 2 x .875/.05
a free edge, the effective width calculation = 35. From Fig. C7.18 and curve 8, Fir = 29,000
will be in two steps. whi~h is far below the cal~ulated crippling
stress, thus the rivet spacing WOQld have to be
reduced. Use 9/16 inch spacing corrected
~ = \~9 x .064 v!Io,.500,000/b7,400 :: 0.82
p/t = 2 x .5625/.05 = 22. From Fig. C7.18,
Fir = 57,400 psi, which happens to be the
Wl from equation C7.18 = .62t vE/F cs = .62 x crippling stress and therefore satisf~ctory•
•064 )10,.500,000/57,400 = 0.55 inch.
C7. 16 Failing Strength of Short Sheet-stiffener Panels
in Compression.
Fig. (b) shows w/2 \
the effective sheet \-.64~ .875 Gerard (Refs. 2, 3) from a ~omprehensive
width as calculated.
Total effective
,
\
I
study of test results on short sheet-stiffener
sheet area = 1.28 T panels in compression, has sho\lVYl that his
x .05 + .875 x .05 wl = .55 equation C7.4, or Fig. C7.7, can be used to
+ ( 0 • 55 + O. 82 )
.064 = .195. Total w
t- Fig. b give the local monolithic crippling stress for
sheet panels stiffened by Z, Y and H at shaped
2=·82 stiffeners. The method of calcul~ting the
area = .195 + 0.24 value of the g f~ctor is illustrated in Fig •
•435. 1 C7.22. Fig. C7.23 is a photograph showing the
crippling type of failure for a short panel
The total
compressive load that entire unit can carry involving the Z shape stiffener.
before failure is then equal to AF cs = .435 x
57,400 = 25,000 Ibs. C7. 17 Failure by Inter-Rivet Buckling.
This result assumes that no inter-rivet Howland (Ref. 8) assumed that the sheet
buckling occurs under the stress of 57,400 psi acts as a wide column which is clamped at its
in the sheet between rivets. ends and whose length is equal to the rivet
spacing. The inter-rivet buckli.ng stress
Th8 skin rivets are Brazier head type equation is then,
spaced 7/8 inch apart or p = 7/8. cn 2 n n ~ 2
Fir = 12 (1 _ lie l!) ( p) - - - - - (C7.24)
As discussed under inter-rivet buckling,
the end coefficient c for this type of rivet The end fixity coefficient C is taken as
should be less than 4 or assumed as 3. 4 for flat head rivets and reduced for other
types as previously explained for equation
Fig. C7.18 gives the inter-rivet buckling C7.21.
stress versus the p/t ratio. This chart is
based on a clamped end condition or C = 4. T/ is the plasticity correction factor
Since C = 3 will be used for the Brazier type -
rivet, we correct the p/t ratio by the ratio 71 is the clad correction factor
14 / /3:: 1.16. lie is Poisson's ratio (use 0.30)
The corrected p/t = 1.16 x .87 5/.05 = 20.3 t s = sheet thickness, inches.
p = rivet spacing or pitch in inches.
From Fig. C7.18 using curve (8) which is
our materi~l, we read Fir = 60,000 psi, thus For non-clad materials the curves of Fig.
skin will not buckle between rivets as C7.24 can be used. This figure is the same as
crippling stress is 57,400 psi. Fig. C5.8 of Chapter C5. For the clad correction
see Table C5.1 of Chapter C5.
The web rivets are of the flat head type
and C = 4 can be used. Spacing is 1 inch. C7.18 Failure of Short Panels by Sheet Wrinkling.
Hence p/t = 1/.064 = 15.6 and from Fig. C7.18,
curve 8, we read Fir - 64,600, which is In a riveted sheet-stiffener panel, if the
considerably more than the unit crippling rivet spacing is relatively large, the sheet will
stress. buckle between rivets, such as illustrated in the
photograph of Fig. C7.25. This inter-rivet
In wing construction, the skin rivets are buckling stress was discussed in the previous
C7.16 CRIPPLING STRENGTH OF COMPOSITE SHAPES AND SHEET-STIFFENER PANELS IN COMPRESSION
C
fiilDililD
c- - - -c
F I F I
FFF?ii g=I7 : g;:I7
C C
I I
J
I
g=i7 ; g=I7
Average g = 16.83
I
Ig=u;
5 Cuts
12 Flanges (c) Hat-stiffened panel.
17 = g
F F :
C
g=8 g=8 I g=8 g=8 I g=7
Average g = 18.83
5 Cuts
14 Flanges (a) Y-stiffened panel.
19 - g
Fig. C7.22 Method of cutting stiffened panels to determine g.
Fig. C7.23 A 24S-T aluminum-alloy Y-stiffened panel (on the left) and its 75S-T counterpart after failure.
C7.17
article. This sheet buckling does not deform approximate criterion for rivet strength from
the flange of the stiffener to which the sheet Ref. 2 is,
is attached. However, if the rivet or spot
weld spacing is such as to prevent inter-rivet S :> 0.7 bs..E (F )"
buckling of the sheet, then failure often occurs rEst d d w - - - - - - - (C7.27)
by a larger wrinkling of the sheet as
illustrated in Fig. C7.26. The larger wrinkle The tensile strength of the rivet Sr is
shape sUbjects the flange of the stiffner to defined in terms of the shank area and it may
which the sheet is attached to lateral forces be associated with either shank failure or
and thus the stiffener flange often deforms pUlling of the countersunk head of the rivet
with the sheet wrinkle shape. This deforming through the sheet.
of the stiffener flange produc~s stresses on
the stiffener web, thus wrinkling failure is a For aluminum alloy 2117-T4 rivets whose
combination of sheet and stiffener failure. tensile strength is s = 57 ksi, the criteria
The action of the wrinkling sheet to deform are:-
the stiffener flange places tension loads on
the rivets, thus rivet design enters into the s = 57 kSi., de/t av ~ 1.67
failing strength of sheet-stiffener panels
under compression. (C7.28)
190 160
s= de I t -( deIt av )",de/tav >l.67
av
where t av . is the average of sheet and stiffener
thickness in inches. The effective diameter de
is the diameter for a rivet made from 2117-T4
material.
1.2
/
~
V-- 5
l-
0
n
~ --::::::: -~~ .../0
LO aJ
I-
ld~ ~
s
I-
.2
I--
,
~
.r.
~
tr
C7.24 and
F w as per
eq. C7.25.
o V
o .2
I
s
I
e
I
1.0
1
1.2
I I
1.$
I
I.a
I \
2.0
c n" E ts "
12 (1 - V e ")F,0 . 7 (p-) or
10
9
1\ \ 1\ 1\
11
\\ \ \
7
.\ \\ \ \
6
1\ \ 1\ 1\
\
f \ \ 1\\
.H- b;' O'f .5
\
l\ ~ \ r\
4
1'.25
2
r-- r---
~
...........
~ t - r-=::: ~ t-- r-- I-- t -
I
"'"
o .2 If .6 .6 1.0 1.2 /,4 1.6 ll1 2.0 2.2 2.4 2.6
~
b, /1,
--- -- -
10
llz..L
tor -
r-- 10
I--
9 -II 9-
-:::::---
r- I-- ~ r-- 11
7-
11 - ' 0
!............
,,-.-- ~
I-- ~ ~ ~
_6
!1
.L 7 -9
t.
-,,/ V V
-;::: t::=-I--
" 7 /V" V V
~
s
V
1-5
6
/
6 7
I' >-; 1=
I
" 9 10 11 12 1$ 14 Its
p
7'
Fig. C7. 28 (Ref. 2) Experimentally determined values of effective
rivet offset.
C7.19
greater than the calculated failing stress of (5) plD ::= 8 - - - - - - - - - - Face wrinkling
27,800 psi.
(6) Tensile strength of rivet or spot weld
Check of Rivet strength. attachment per inch should be ~ 0.05 Fcyt s
in order to prevent failure in wrinkling.
From expression C7.26 (7) As a rough guide do not use bent up
stringers if bslt s < 30 in order to prevent
plb s < 1.27/kw1/2 face wrinkling weakness.
0.75 _ I C7. 23 Y Stiffened Sheet Panels•
2:0- •375 < 1.27/(4.4)1 2 = .603(satisfactory)
A Y shape cannot be formed from sheet,
The criterion for required rivet strength thus it must be extruded. To make the Y shape
to make the stiffener flange follow the efficient, the various parts usually have a
wrinkled sheet is from C7.27 different thickness. Furthermore, the extruded
material has different mechanical properties
in the inelastic stress range as compared to
rolled sheet that is used for the panel, thus
these effects must be considered in calculating
s ( 0.7 )( 2 )(0.75)( )2 the crippling stress of the stiffener and the
r::> 10,700,000 3/32 3/32 35,100 complete panel unit.
Sheet stiffener units can be designed as Fcy = {FCY(S~) + FCY(ST) [(twits) -l]}/tw/t s
columns if the secondary forms of failure such
as inter-rivet buckling and face wrinkling are - - - - - - - - - - - (C7.30)
avoided. The following design rules referring
to Fig. (A) will usually avoid these secondary The monolithic crippling stress for the
weaknesses. sheet stiffener panel can be calculated from
equation C7.4 or by the curve in Fig. C7.7.
Equation C7.4 is
1.2
I I I
o HA r STIFFENED PANELS
Fig. A • r STIFFENED PANELS
C7. 24 Example Problem Y Stiffened Panel. unit. Ec will be taken as 10,600,000 which is
the average E for stiffener and sheet. Substi-
The compressive monolithic failing stress tuting in the above parameter:-
of a Y stiffened panel, as illustrated in Fig.
C7.22a, will be calculated by the Gerard method. ( .808 ) ( 64,700 ):1./2 = 6
Fig. C7.31 shows details of the panel unit. 18.83 x .0763 x .064 10,600,000 .68
The stiffener is extruded from 2014-T6 aluminum
alloy for which Fcy = 53000 and Ec = 10,700,000. From Fig. C7.7 we read
The skin or panel sheet is aluminum alloy
7075-T6 for which FCY = 67,000 and Ec =
10,500,000.
Thus the monolithic crippling a failing
~ 4. 21" ------l
stress FcsM = FCY x .76 = 64,700 x .76 =
j .136t
.068
- -r
~ 1+-1.111 -.\
~
I 48,200 psi.
COLUMN STRENGTH
Fig. C7.31
C7.25 Column Curve for Members With
Unstable Cross-Sections.
Stiffener area = 0.538 in. 2
Sheet area = 4.21 x .064 = .270 in. 2 Chapter C2 dealt with the column strength
Total area (A) per stiffener unit = 0.808 in. 2 of members with stable cross-sections. For
Radius of gyration of stiffener alone = 1.123 in. example, if we took a round tube with relatively
heavy wall thickness and tested various lengths
Solution:- in compression to obtain the failing stress and
then plotted these stress values Fc against the
Since the elements of the Y stiffener have slenderness ratio L'/p of the member, the test
different thicknesses, the effective t w by results would closely follow the curves ABFC in
equation C7.29 is needed. Fig. C7.32. The type of failure would be
elastic overall bending instability at stresses
Z bit i between points B-C and inelastic bending
tw = =
instability at stresses for most of the range
Zbi
BA. EUler's equation as shown in Fig. C7.32
1.111 x .136 + 1.432 x .068+3.2x .064+ 1.188 x .064 can be used to determine the failing stress for
1.111 + 1.432 + 3.2 + 1.188 both elastic and inelastic bending instability
with the tangent modulus Et being used in the
or tw = .0763 inches inelastic stress range.
Since stiffener and sheet have different Now suppose we test various lengths of a
material properties, an effective Fcy from member composed of the same material as used for
equation C7.30 will be calculated. obtaining curve ABC, but use a member with an
open cross-section, such as a channel, hat
Fc= {67' 000 + 53000 [( .0763)_1] 0763
}/· .064 = 64700 section, etc., with relatively small material
y .064 thickness. The test results for such members
would often follow a curve similar to DEFC in
The curve in Fig. C7.7 will be used to Fig. C7.32. Thus it is obVious that EUler's
solve the equation for FcS(M)' The !ower s~ale equation cannot be used in the range DEF, as the
parameter for Fig. C7.7 modified by twand Fcy true failing stresses are far less than that
is, given by the Euler column equation. A short
length of the member with L'/p less than 20 will
fail at practically the same stress, thus the
failing stress for lengths up to L'/p = 20 will
be practically the same and this stress has been
The value of g from Fig. C7.22a is 18.83 given the name of crippling stress (F cs ) and the
as the average value for a 6 stiffener panel previous portion of this chapter has been con-
C7.22 CRIPPLING STRENGTH OF COMPOSITE SHAPES AND SHEET-STIFFENER PANELS IN COMPRESSION
D
0EW",
~ Curve
n 2
E
This method is qUite simple to use as the
only additional calculation required is the
crippling stress of the column section which is
obtained by methods previously explained in
Crippling
Stress
I Fe = (V/p) 2 this chapter. The other methods usually involve
the buckling stress as well as the crippling
Region stress. Since the crippling stress is normally
I constant below L'/p :: 20, the assumption that
Fcs is zero at L'/p = 0 is slightly conservative.
I METHOD 2.
1-- Transition Region
The following method or slight variations
of it appears in the structural design manuals
C
of a number of aerospace com)anies. The method
or procedure for determining the column failing
L'/p stresses in the so-called transition range
involves the use of the basic column curve for
Fig. C7.32 stable cross-sections. The procedure can best
be explained by reference to Fig. C7.34.
cerned with methods of calculating this local
crippling or failing stress. At point (F) the A
elastic buckling of some part of the cross-
section begins. Between stress points F to E Fey 0
-+- -
the action for the member involves both overall
elastic bending instability plus local buckling I
which becomes more extensive as the stress I
increases. The portion EF of the column I
strength curve is often referred to as the I
transition range. At present no reliable I
theoretical theory has been developed for D
F es -. - - .-.J~
determining the failing stresses in this
transition range, thus resort is made to semi-
empirical methods which have been checked
against test results and found to give reason-
ably close results.
C7.26 Methods Used for Determining the Column
Failing Stress in the Transition Region.
C7.27 Example Problems Involving the Finding C7.33 for a value of L'/p = 50 and a value of
of the Column Strength of Columns With Fc = Fcs = 38,200 at L'/p = O.
Unstable Cross-Sections.
Column Strength by Method 2
PROBLEM 1.
This method requires a graphical
A rectangular tube construction which involves the column curve
21 inches long has the for the given material and for a stable cross-
cross-section as shown section.
in Fig. (a). The
material is aluminum Fig. C7.35 shows the column curve for our
alloy sheet 7075-T6, Fig. a material for a column with stable cross-section.
with Fcy = 67,000 and Ec = 10,500,000. If the It is identical to Fig. C2.10 of Chapter C2
member has a pinned end condition, what 1s the for the room temperature condition, except the
column failing stress. curve in Fig. C7.35 has been drawn to a smaller
vertical scale.
Solution. The area of cross-section = .234 in. 2
The least radius of gyration is 0.42 inches.
bw = 1.5 - .04 = 1.46, bf = 0.75 - .02 = 0.73 If the attached sheet is relatively thin,
that is, less than the stiffener thickness, the
bF/bw = .73/1.46 = 0.5, tw/tF = 1.0 method of using the effective sheet width as a
part of the column area is widely used by
From Fig. C6.4 of Chapter C6, we read structural designers and will be used in this
Kw = 2.9. example solution.
F
cr
= 2.9 12n 2
x 10,500,000 (~) 2
(1 _ .3 2 ) 1.46
= 20 ' 900 ps
i Example Problems.
For an example
The crippling stress will be calculated by problem, we will assume
the Gerard Method using Fig. C7.9, which applies that the Z stiffener in
for Z shapes. The lower scale parameter is, Problem 2 is one of
several stiffeners riveted
~{cY)~h = (.117)( 67,000 )~/" = 13.6 to a sheet of .025 thick-
t 2 Ec .04 2 10,500,000 ness and of the same
material as the stiffener.
From Fig. C7.9, we read, Fig. c
Fcs/F cy = .455, hence, Fcs = 67,000 x .455 = Solution.
30,600 psi.
The rivet or spot weld spacing is made
Column Strength by Method 1 such as to prevent inter-rivet buckling. Thus
the column area will be as shown in Fig. c,
L' = L/ve-= 30 ~= 24.5 namely, the stiffener area plus the area of
the sheet for the effective width w. Since
L'/p = 24.5/.535 = 45.8 the effective sheet width w is a function of
the stiffener stress and since the stiffener
C7.26 CRIPPLING STRENGTH OF COMPOSITE SHAPES AND SHEET-STIFFENER PANELS IN COMPRESSION
--
Determination of
V
- e~-r-~
Po - FST
/ !""'-
.......
--
/
/
./
.... r..... ..... "'-
I J I'
I 1// r---. I'--- .... ~
2.0
V .... .....1' .... .... n-
I--,
....
01 ""
rz..(!;
II
U/
,,-
-- -
r-
....... r--..
r- ....
. . . . r--.
I'---
....... '-.
-- - - -- - - 01""
r-. ......
io-..
I""-
r- r--..
-.....;
io-..
r-
') I--
rz.. rz..r:Il
II
,.---. ....
- -- - -- -
til til
V 4.
~
Qlo l,.;"
-I""-
........
"'- l""- t- I""-
I""-
2 I""-
~
0
0-1
l""-
-- I-- t- l""- I""- I- ~
~1.0 ....1- '-.
-I-
r- r- r-
--- - -- r- I-
..... t-- I""- "",
l"""- I--
I-- .....:.
t-:.~Ft
STIFFENER ALONE
I- -I--
-
I""-
....1- t- I--
r- t-- I-
wt
r- I -
-l-
~
,-,
Ao - Area of Stiff-
ener Alone
p 0 = Radius of Gy-
(/J2 _ Fe _ 1+[I+{pSOr]
- FST- (1+~r
Ao
ration of 0 Q
Stiffener Alone 0 1.0
wt 2.0 3.0 4.0
Ao
C7.27
The revised effective width based on this (4) Same as (2) but subjected to 300 0 F for
stiffener stress is, 1/2 hour duration.
w = 1.9 x .025 VIO,500,000/26,050 = .99 in. t:= 1-1/2 --l I-- --I
f
7/8"
1-
LLig.a
3
16 1..
7/8"
tli::
1
~
16
ig b
.
.072
1.
of the flat head type. Material is 2024-T3
aluminum alloy. Also use 7075-T6 aluminum
material.
Find crippling stress for stiffener.
I- 7/8 -J T064 I- 1-1/2 --i T Will inter-rivet buckling occur. Find
effective sheet widths and total failing
load unit will carry.
(2) Find the crippling stress for the two
channel shapes of Figs. c and d when .035
formed of following materials. (1) AISI
4130 steel, heat-treated to Ftu =
f::.r:~~~2~nrf\-Skin
180,000. (2) Same as (1) but SUbjected to
an elevated temperature of 500 0 F for 1/2
duration. (3) Ti-8Mn Titanium.
Tf
(4) 7075-T6 aluminum alloy.
7/8
~
16
1-3/.
Fig. c
Trcl
3/4
..l
I--l-l/4~
16
Fig. d
.04
Fig.
--l ~.064
(7 ) Determine the compressive failing strength
of a short Z stiffened sheet panel unit such
(3 ) Find the crippling stress of the as that illustrated in Fig. C7.29. The
rectangular tubes in Figs. e and f when various dimensions are as follows. Refer
formed from following materials. to Fig. C7.29 for meaning of symbols.
(1) 2024-T3 aluminum alloy. (2) HK3lA-0
magnesium. (3) Ti-8Mn Titansium. tw = .072 bF = 1.0 bs = 2.50
t s = .072 bA = 0.625
bw =2.50 bo = 0.375
C7.28 CRIPPLING STRENGTH OF COMPOSITE SHAPES AND SHEET-STIFFENER PANELS IN COMPRESSION
Stiffener and sheet material is 2024-T3 (5) Windenberg, D. F.:- Proc. 5th Inter-
aluminum alloy. Rivets are 1/8 diameter national Congress for Applied Mechanics.
Brazier Head type 2117-T3 material and pp. 54-61, 1939.
spaced 7/8 inch.
(6) Experimental StUdy of Deformation and
(8) S&TIe as (7) but change material to 7075-T6 Effective Widths in Axially Loaded Sheet
aluminum alloy. Stringer Panels. NACA TN 684, 1939.
(9) Same as (7) but change material to Ti-8Mn (7) Gerard, F.:- Effective Width of
Titanium. Elastically Supported Flat Plates. Jour.
Aero. Sci., Vol. 13, Oct. 1946.
(10) In the Example Problem on a Y stiffened
panel as given in Art. C7.25, change sheet (8) Howland, W.L.:- Effect of Rivet Spacing
thickness from .064 to .081 and calculate on Stiffened Thin Sheet in Compression.
the resulting panel strength. Jour. Aero. Sci., Vol. 3, Oct. 1936.
(11) The hat stiffener in Fig. g of Problem (4) (9) Argyris, J. H. and Dunne, P.C.:-
is one of several stiffeners riveted to Structural Principles and Data. Handbook
skin of thickness .032 and of the same of Aero., 4th Edition, 1952.
material as the stiffener. If the length
of the panel is 20 inches, what will be (10) Bizlaard, P. D. and Johnston, G. S.:-
the column failing stress if end fiXity Compression Buckling of Plates Due to
c = 1.5. Also for c =
2.0. Use method Forced Crippling of Stiffeners. Preprint
involVing effective sheet Widths. No. 408. S.M.F. Fund Paper. Inst. of
Aero Sci. Jan. 1953.
(12) Same as Problem 11 but use Z stiffener of
Fig. h of Problem 4. (11) Semonian, J. W. and Peterson, J. P.:-
An Analysis of the Stability and Ultimate
References Compressive strength of Short Sheet-
Stringer Panels With Special Reference to
(1) Needham, R. A.:- The Ultimate strength of the Influence of Riveted Connection
Aluminum Alloy Formed Structural Shapes Between Sheet and Stringer. NACA TN 3431,
in Compression. Jour. Aero. Sci. Vol. 21 March, 1955.
April, 1954.
(12) Gerard, G.:- Handbook of Structural
(2) Gerard, G.:- Handbook of Structural Stability, Part V. Compressive Strength
Stability. Part IV. Failure of Plates of Flat Stiffened Panels. NACA TN 3785.
and Composite Elements. NACA TN 3784. Aug. 1957.
August, 1957.
(13) Thesis by R. J. White:- The Ultimate
(3) Gerard, G.:- The Crippling Strength of Compressive strength of Thin Sheet Metal
Compression Elements. Jour. Aero. Sci., Panels. Appendix A. C.I.T. 1935.
Vol. 25, Jan. 1958.
Fig. ca. I Type of Failure Axial Compression and No Fig. ca. la Type of Failure Under Axial Load With Internal
Internal Pressure. (Ref. 1.) Pressure. (Ref. 1.)
Fig. ca. lc Type of Buckling Failure Under Pure Bending Fig. ca. ld Type of Buckling F .iilure Under Pure Bending
Load. No Internal Pressure. (Ref. 9.) With High Internal Pressure. (Ref. 9.).
ca.3
±i
A similar solution for cylinders with fixed
i2
edges deviates from the solution for simply 2
i-j-
'ft)
.8
Figs. C8.2 to C8.5 inclusive taken from F
Ccr
Reference 1, show the theoretical curve which ~ .8
shows the buckling coefficient as a function
of the geometrical parameter Z. Theoretically,
the short cylinder range would occur at Z = O.
The values of Kc of 1.12 and 4.12 for Z = 1
correspond quite closely to the buckling
coefficients of simply supported and fixed 8 10 IA /8 i8 If)
ended plate columns. The long cylinder behavior Kc n
2
E t 2
Figs. C8.2 to C8.5 show the plot of ca. 5 Buckling of Monocoque Circular Cylinders Under Axial
extensive test data and a 90 per cent proba- Load and Internal Pressure.
bility curve derived by the author of (Ref. 1)
by a statistical approach. Fig. C8.6 from Experiments conducted many years ago
(Ref. 1) shows a plot of much test data on a definitely showed that the compressive buckling
logarithmic chart of r/t versus Kc for Z = strength of monocoque cylinders was increased
10,000. A best fit curve and 90 and 99 per- if internal pressure was added to the closed
cent probability derived curves are also seen cylinder. Since weight saving is very important
as well as the theoretical curve for C = .605. in missile design, the use of pressurized
ca.4 BUCKLING STRENGTH OF MONaCO UE CYLINDERS
o BALLERSTEDT 6 WAGNER tDONNELL .1.O, CRATE 81 SCHWARTZ o BALLERSTmT a .cJNEIt • DONNELL .1.O, CRATE 81 SCHWARTZ
D FUNG a SEQtLER "LUNDQUIST _BRIDGET CI FUNG .. SECHLER eLlN)QUIST
- BRIDGET
• BUOIY (NAA) oNAA 4 Rll8ERTSON • SUCHY (NAA) .NAA _ROBERTSON
.. a..ARK a HOLT • KANEMITSU a NOJtMA o WILSON 81 NEWMARK III CLARK a HOLT • KANEMrrsu .. NOJIMA • WILSON a NEWMARK
~
6
4
""e,
'I' Ke 12[1-,u2) L
'JT2 E [-IT I ,,"cr ,,-2 E
TJ • Ke 12[1';#2] L
['r
~r
2 z. ~v'1-,u2 Z· ~1IT-,li2 V
KJ38 6
rl
/"
KJ3
8
6
rl
4 4
2
1/ V */
THEORETICAL aJRVE
If7.
THEORETICAL CURVE
/" Ke 102
8 8
6
4 .
/
90% PROBABILITY
DESIGN CURVE (r/l.3OOl
--SIMPLY SUPPORTED EDGES
~~
6
4
V- /
90% PROBABILITY
DESIGN CURVE (,/l.r50)
2 --SIMPLY SUPPORTED EDGES
---CLAMPED EDGES V ---Cl.AMPED EDGES
10 10
8 8
6
4
1/
- 6
4 -
2
1.0
./V
1.0
.~....,v
4 68 4 68 4 •• 4 68 468
468KJ2
1.0 10 102 10 3 1.0 10 105
Z
Z
Fig. ca.2 r/t = 100 to 500. Fig. ca.3 r/t = 500 to 1, 000.
o BALLERSTEOT a WAGNER ODONNELL • Lo, CRIlrE 81 SCHWARTZ
_BRIDGET o FUNG 81 SECHLER "LUNDQUIST o BALLERSTEDT a WAGNER • DONNELL • Lo, CRATE 81 sc.-.rrz
.1lUCHY \NAAI _NAA aROBERT'SOfli _IlRIOGET D FUNG a SEOtLER
• LUMlQUIST
.. CLARK a HOLT • KANEMITSU a NOJIMA o WILSON 81 NEWMARK • BIJCHY (NAA) oNAA - ROllERTSON
III CLARK a HOLT • KANEMITSU a NO.JIMA • WILSON a NEWMARK
10~
6 ""cr
4
'JT2 E
TJ .Ke 12[ljl.L2j L
[I r I~
4
6 ""cr 'JT2 E
11'Kc 12[1",2]
[IL r -
~
Z. "VI-,u2
y ~
z' rt v'i7
• I
10 3 /
8
6
4
1\ I IIIII
THEORETICAL CURVE
~ THEORETICAL CURVE ---: ~
10~
90% PROBABILITY
DESIGN CURVE 6 . 90% lBAB LI Y
'.
DESIGN CURVE
(r/l.15OO) • (r/l.3OOO)
V
• --SIMPLY SUPPORTED EDGES V- SIMPLY SUPPORTED EDGES
10
1 : / ---CLAMPED EDGES
10
/ ---CLAMPED EDGES
8 8
5
4 - 6
4
~~ V 10
./" .-/
1.0 4 68 4 68